CME4PAC

Online CME for Physician Assistants

Menu

Skip to content
  • Home
  • Membership Benefits
  • My Account
  • About Us
  • Log In
  • Contact Us
  • Ortho on the Go

Board Review Essentials Quiz

Time limit: 0

Quiz-summary

0 of 220 questions completed

Questions:

  1. 1
  2. 2
  3. 3
  4. 4
  5. 5
  6. 6
  7. 7
  8. 8
  9. 9
  10. 10
  11. 11
  12. 12
  13. 13
  14. 14
  15. 15
  16. 16
  17. 17
  18. 18
  19. 19
  20. 20
  21. 21
  22. 22
  23. 23
  24. 24
  25. 25
  26. 26
  27. 27
  28. 28
  29. 29
  30. 30
  31. 31
  32. 32
  33. 33
  34. 34
  35. 35
  36. 36
  37. 37
  38. 38
  39. 39
  40. 40
  41. 41
  42. 42
  43. 43
  44. 44
  45. 45
  46. 46
  47. 47
  48. 48
  49. 49
  50. 50
  51. 51
  52. 52
  53. 53
  54. 54
  55. 55
  56. 56
  57. 57
  58. 58
  59. 59
  60. 60
  61. 61
  62. 62
  63. 63
  64. 64
  65. 65
  66. 66
  67. 67
  68. 68
  69. 69
  70. 70
  71. 71
  72. 72
  73. 73
  74. 74
  75. 75
  76. 76
  77. 77
  78. 78
  79. 79
  80. 80
  81. 81
  82. 82
  83. 83
  84. 84
  85. 85
  86. 86
  87. 87
  88. 88
  89. 89
  90. 90
  91. 91
  92. 92
  93. 93
  94. 94
  95. 95
  96. 96
  97. 97
  98. 98
  99. 99
  100. 100
  101. 101
  102. 102
  103. 103
  104. 104
  105. 105
  106. 106
  107. 107
  108. 108
  109. 109
  110. 110
  111. 111
  112. 112
  113. 113
  114. 114
  115. 115
  116. 116
  117. 117
  118. 118
  119. 119
  120. 120
  121. 121
  122. 122
  123. 123
  124. 124
  125. 125
  126. 126
  127. 127
  128. 128
  129. 129
  130. 130
  131. 131
  132. 132
  133. 133
  134. 134
  135. 135
  136. 136
  137. 137
  138. 138
  139. 139
  140. 140
  141. 141
  142. 142
  143. 143
  144. 144
  145. 145
  146. 146
  147. 147
  148. 148
  149. 149
  150. 150
  151. 151
  152. 152
  153. 153
  154. 154
  155. 155
  156. 156
  157. 157
  158. 158
  159. 159
  160. 160
  161. 161
  162. 162
  163. 163
  164. 164
  165. 165
  166. 166
  167. 167
  168. 168
  169. 169
  170. 170
  171. 171
  172. 172
  173. 173
  174. 174
  175. 175
  176. 176
  177. 177
  178. 178
  179. 179
  180. 180
  181. 181
  182. 182
  183. 183
  184. 184
  185. 185
  186. 186
  187. 187
  188. 188
  189. 189
  190. 190
  191. 191
  192. 192
  193. 193
  194. 194
  195. 195
  196. 196
  197. 197
  198. 198
  199. 199
  200. 200
  201. 201
  202. 202
  203. 203
  204. 204
  205. 205
  206. 206
  207. 207
  208. 208
  209. 209
  210. 210
  211. 211
  212. 212
  213. 213
  214. 214
  215. 215
  216. 216
  217. 217
  218. 218
  219. 219
  220. 220

Information

Board Review Essentials

You have already completed the quiz before. Hence you can not start it again.

Quiz is loading...

You must sign in or sign up to start the quiz.

You have to finish following quiz, to start this quiz:

Results

0 of 220 questions answered correctly

Your time:

Time has elapsed

You have reached 0 of 0 points, (0)

Categories

  1. Not categorized 0%
  1. 1
  2. 2
  3. 3
  4. 4
  5. 5
  6. 6
  7. 7
  8. 8
  9. 9
  10. 10
  11. 11
  12. 12
  13. 13
  14. 14
  15. 15
  16. 16
  17. 17
  18. 18
  19. 19
  20. 20
  21. 21
  22. 22
  23. 23
  24. 24
  25. 25
  26. 26
  27. 27
  28. 28
  29. 29
  30. 30
  31. 31
  32. 32
  33. 33
  34. 34
  35. 35
  36. 36
  37. 37
  38. 38
  39. 39
  40. 40
  41. 41
  42. 42
  43. 43
  44. 44
  45. 45
  46. 46
  47. 47
  48. 48
  49. 49
  50. 50
  51. 51
  52. 52
  53. 53
  54. 54
  55. 55
  56. 56
  57. 57
  58. 58
  59. 59
  60. 60
  61. 61
  62. 62
  63. 63
  64. 64
  65. 65
  66. 66
  67. 67
  68. 68
  69. 69
  70. 70
  71. 71
  72. 72
  73. 73
  74. 74
  75. 75
  76. 76
  77. 77
  78. 78
  79. 79
  80. 80
  81. 81
  82. 82
  83. 83
  84. 84
  85. 85
  86. 86
  87. 87
  88. 88
  89. 89
  90. 90
  91. 91
  92. 92
  93. 93
  94. 94
  95. 95
  96. 96
  97. 97
  98. 98
  99. 99
  100. 100
  101. 101
  102. 102
  103. 103
  104. 104
  105. 105
  106. 106
  107. 107
  108. 108
  109. 109
  110. 110
  111. 111
  112. 112
  113. 113
  114. 114
  115. 115
  116. 116
  117. 117
  118. 118
  119. 119
  120. 120
  121. 121
  122. 122
  123. 123
  124. 124
  125. 125
  126. 126
  127. 127
  128. 128
  129. 129
  130. 130
  131. 131
  132. 132
  133. 133
  134. 134
  135. 135
  136. 136
  137. 137
  138. 138
  139. 139
  140. 140
  141. 141
  142. 142
  143. 143
  144. 144
  145. 145
  146. 146
  147. 147
  148. 148
  149. 149
  150. 150
  151. 151
  152. 152
  153. 153
  154. 154
  155. 155
  156. 156
  157. 157
  158. 158
  159. 159
  160. 160
  161. 161
  162. 162
  163. 163
  164. 164
  165. 165
  166. 166
  167. 167
  168. 168
  169. 169
  170. 170
  171. 171
  172. 172
  173. 173
  174. 174
  175. 175
  176. 176
  177. 177
  178. 178
  179. 179
  180. 180
  181. 181
  182. 182
  183. 183
  184. 184
  185. 185
  186. 186
  187. 187
  188. 188
  189. 189
  190. 190
  191. 191
  192. 192
  193. 193
  194. 194
  195. 195
  196. 196
  197. 197
  198. 198
  199. 199
  200. 200
  201. 201
  202. 202
  203. 203
  204. 204
  205. 205
  206. 206
  207. 207
  208. 208
  209. 209
  210. 210
  211. 211
  212. 212
  213. 213
  214. 214
  215. 215
  216. 216
  217. 217
  218. 218
  219. 219
  220. 220
  1. Answered
  2. Review
  1. Question 1 of 220
    1. Question

    A 16-year-old male presents to the office for evaluation of a left knee injury while playing football three days ago. He reports while jumping to make a catch, he collided with another player, then landed “weird” on his left leg, causing his knee to twist. He felt a “weird sensation” deep in his knee upon landing. He is a very active and healthy teenager, with no previous injuries or significant past medical history. On physical exam of the lower extremities, his left knee appears much larger than the right, with a moderate effusion appreciated. ROM is limited due to pain. Lachman test is positive. Valgus stress induces pain but not laxity. McMurray’s test is negative.

    Correct

    Answer: C. Anterior Cruciate Ligament Injury. Based on this patient’s history, presentation of injury mechanism, examination findings, and MRI results, the diagnosis is likely Anterior Cruciate Ligament Tear. ACL tears are one of the most common knee injuries among athletes. They are most common in late adolescence, affecting females 2-8 times more than males2. Athletes who are at greatest risk for tearing their ACL include skiers, soccer players, snow boarders, basketball players, and football players2,3. There are reported average of 90,000 ACL repairs performed in the United States annually, with 1/3,000 reported ACL ruptures2. These numbers vary depending on the gender, sport, and level of competition. There are many risk factors for injuring an ACL. Some intrinsic factors include decreased flexibility or strength, abnormal foot morphology, genetic predisposition, and ligament/leg dominance2. Extrinsic factors to consider include shoe-surface interaction, wet/muddy/uneven playing surface, higher/more aggressive competition, and extreme temperature changes2. Females are reported to have a greater risk of injury due to their anatomy; larger pelvic width, smaller ACL size, increased q angle, valgus deformity, narrow intercondylar notch, generalized joint laxity, and quad dominance2.
    The anterior cruciate ligament runs through the knee joint, connecting the posterior lateral femoral condyle to the anterior tibia3. The ACL is the primary source of stabilization for the knee. It helps with rotational stability of the tibia on the femur as well as, more importantly, controlling anterior translation of the tibia on the femur3. ACL injuries can occur due to both contact and noncontact forces. Noncontact injuries make up about 70% of ACL injuries2. Typically, the mechanism is either caused by forceful quadriceps contraction during deceleration of lower extremity, along with a change in direction such as internal rotation with hyperextension (noncontact), or is due to a planted foot with a torque (valgus blow) applied to the lower extremity causing the tissue to tear (contact)2.
    Patient’s usually present with a chief complaint of knee pain following an injury where they most often report hearing a “pop” or feeling a “pop” at the time. Often they will mention their pain developed immediately following, with swelling within the next few hours. When asking questions, depending on time since injury, patients will often report decreased range of motion of the knee, stiffness, and feeling of instability3. Upon physical exam, a Lachman test is the most sensitive and useful for testing anterior laxity of the knee4. The anterior drawer test is often positive as well, although not as sensitive. It may show excessive anterior translation of the tibia in comparison to the other knee. Also on exam, decreased ROM and effusion are commonly noted.
    As with most orthopedic injuries, x-rays are often the first imaging study to be done. In a case of ligament injury, an x-ray is more important to rule out fracture or other underlying problem but will not help in diagnosing a ligament injury. More detailed than an x-ray, an MRI can diagnose an ACL tear with 95% or better accuracy4. Along with visualizing the tear, 80% of the time bone bruises are seen along both the lateral tibial plateau and lateral femoral condyle4.
    Unlike most other ligaments in the knee, the ends of a torn ACL do not reconnect. Therefore, if the patient is young or hoping to return to physical activity, ACL tears require reconstruction via surgery. Prior to surgery, if the injury is strictly ACL related, physical therapy is recommended to help regain range of motion of the knee as well as to strengthen the muscles surrounding the knee. If the patient chooses a nonsurgical approach, they may attempt to return to physical activity slowly following PT clearance. If the patient hopes to return to activity and regain stability, surgical reconstruction is needed. The most commonly used and most successful include autograft bone-patellar tendon-bone, semitendinosus/gracilis hamstring autograft, and bone-patellar tendon-bone allograft reconstructions4. A more recent option, currently still being studied at Boston Children’s Hospital, is the Bridge-Enhanced ACL Repair Clinical Trial (BEAR Trial). This study, currently on stage 3, uses stitches and a scaffold sponge injected with the patient’s blood to act as a bridge to help stimulate healing of the torn ACL1. The benefit is no graft is needed and the patient’s own ACL tissue will be preserved. The goal is to help improve long term health benefits and decrease risk of arthritis in the future.
    Overall, studies have shown at least 90% of patients are able to return to play following an ACL tear with physical therapy rehab protocols4. They are typically out of sports for 6 months, then slowly able to return as their quad and hamstrings strengthen. Most patients return to sports following a successful surgery, but there is always the possibility of re-tearing their ACL. The rate can be up to 20% risk of re-tear in teenagers1. Another complication is the increased risk of arthritis 15-20 years post-op, roughly 80% risk1. With the surgical advances today, more and more ACL’s are being reconstructed allowing athletes to return back to what they love to do.

    There are many other differential diagnoses to consider when someone presents with a knee injury. Often if it’s an acute injury the patients pain will be diffuse and hard to pinpoint. They also may present with swelling that makes the knee hard to evaluate on physical exam. Another difficulty is the patient guarding on exam or not relaxing the knee due to the pain. All these factors can affect your diagnosis. In this patient’s case, it was less likely a meniscus injury because we would have expected a positive McMurry’s test with more significant joint line tenderness. A tear of the posterior collateral ligament was also less likely based on the mechanism of injury, negative posterior drawer test, and low occurrence of PCL tears in teenage female athletes. Medial/Lateral collateral ligament injuries are less likely based on the patient’s physical exam, with no laxity/significant pain with valgus or Varus stress on the knee. Patellar instability is also a common problem in this population. The patient denied feeling her patella dislocate. Also, if this is what happened, the patient would likely have less pain with minimal effusion still present. Overall, the only way to ensure diagnosis is an MRI and x-ray. In this case, although our history and exam pointed in the direction of an ACL tear, the MRI was able to confirm that therefore ruling out the other possible diagnoses.

    References:

    1. Bridge-Enhanced ACL Repair Clinical Trial | Boston Children’s Hospital. Boston Children’s Hospital. http://www.childrenshospital.org/aclbeartrial. Accessed December 2, 2018.

    2. DynaMed Plus. Anterior Cruciate Ligament (ACL) Injury. https://www.dynamed.com/ topics/dmp~AN~T114675/Anterior-cruciate-ligament-ACL-injury. Published 1995. Updated June 26, 2017. Accessed December 1, 2018.

    3. McPhee Sj, Papadakis MA, Rabow MW. Current medical diagnosis & treatment 2017. New York: McGraw Hill Education; 2017.

    4. Skinner HB, McMahon PJ. Current Diagnosis & Treatment Orthopedics . 5th ed. McGraw-Hill Education; 2014.

    Incorrect

    Answer: C. Anterior Cruciate Ligament Injury. Based on this patient’s history, presentation of injury mechanism, examination findings, and MRI results, the diagnosis is likely Anterior Cruciate Ligament Tear. ACL tears are one of the most common knee injuries among athletes. They are most common in late adolescence, affecting females 2-8 times more than males2. Athletes who are at greatest risk for tearing their ACL include skiers, soccer players, snow boarders, basketball players, and football players2,3. There are reported average of 90,000 ACL repairs performed in the United States annually, with 1/3,000 reported ACL ruptures2. These numbers vary depending on the gender, sport, and level of competition. There are many risk factors for injuring an ACL. Some intrinsic factors include decreased flexibility or strength, abnormal foot morphology, genetic predisposition, and ligament/leg dominance2. Extrinsic factors to consider include shoe-surface interaction, wet/muddy/uneven playing surface, higher/more aggressive competition, and extreme temperature changes2. Females are reported to have a greater risk of injury due to their anatomy; larger pelvic width, smaller ACL size, increased q angle, valgus deformity, narrow intercondylar notch, generalized joint laxity, and quad dominance2.
    The anterior cruciate ligament runs through the knee joint, connecting the posterior lateral femoral condyle to the anterior tibia3. The ACL is the primary source of stabilization for the knee. It helps with rotational stability of the tibia on the femur as well as, more importantly, controlling anterior translation of the tibia on the femur3. ACL injuries can occur due to both contact and noncontact forces. Noncontact injuries make up about 70% of ACL injuries2. Typically, the mechanism is either caused by forceful quadriceps contraction during deceleration of lower extremity, along with a change in direction such as internal rotation with hyperextension (noncontact), or is due to a planted foot with a torque (valgus blow) applied to the lower extremity causing the tissue to tear (contact)2.
    Patient’s usually present with a chief complaint of knee pain following an injury where they most often report hearing a “pop” or feeling a “pop” at the time. Often they will mention their pain developed immediately following, with swelling within the next few hours. When asking questions, depending on time since injury, patients will often report decreased range of motion of the knee, stiffness, and feeling of instability3. Upon physical exam, a Lachman test is the most sensitive and useful for testing anterior laxity of the knee4. The anterior drawer test is often positive as well, although not as sensitive. It may show excessive anterior translation of the tibia in comparison to the other knee. Also on exam, decreased ROM and effusion are commonly noted.
    As with most orthopedic injuries, x-rays are often the first imaging study to be done. In a case of ligament injury, an x-ray is more important to rule out fracture or other underlying problem but will not help in diagnosing a ligament injury. More detailed than an x-ray, an MRI can diagnose an ACL tear with 95% or better accuracy4. Along with visualizing the tear, 80% of the time bone bruises are seen along both the lateral tibial plateau and lateral femoral condyle4.
    Unlike most other ligaments in the knee, the ends of a torn ACL do not reconnect. Therefore, if the patient is young or hoping to return to physical activity, ACL tears require reconstruction via surgery. Prior to surgery, if the injury is strictly ACL related, physical therapy is recommended to help regain range of motion of the knee as well as to strengthen the muscles surrounding the knee. If the patient chooses a nonsurgical approach, they may attempt to return to physical activity slowly following PT clearance. If the patient hopes to return to activity and regain stability, surgical reconstruction is needed. The most commonly used and most successful include autograft bone-patellar tendon-bone, semitendinosus/gracilis hamstring autograft, and bone-patellar tendon-bone allograft reconstructions4. A more recent option, currently still being studied at Boston Children’s Hospital, is the Bridge-Enhanced ACL Repair Clinical Trial (BEAR Trial). This study, currently on stage 3, uses stitches and a scaffold sponge injected with the patient’s blood to act as a bridge to help stimulate healing of the torn ACL1. The benefit is no graft is needed and the patient’s own ACL tissue will be preserved. The goal is to help improve long term health benefits and decrease risk of arthritis in the future.
    Overall, studies have shown at least 90% of patients are able to return to play following an ACL tear with physical therapy rehab protocols4. They are typically out of sports for 6 months, then slowly able to return as their quad and hamstrings strengthen. Most patients return to sports following a successful surgery, but there is always the possibility of re-tearing their ACL. The rate can be up to 20% risk of re-tear in teenagers1. Another complication is the increased risk of arthritis 15-20 years post-op, roughly 80% risk1. With the surgical advances today, more and more ACL’s are being reconstructed allowing athletes to return back to what they love to do.

    There are many other differential diagnoses to consider when someone presents with a knee injury. Often if it’s an acute injury the patients pain will be diffuse and hard to pinpoint. They also may present with swelling that makes the knee hard to evaluate on physical exam. Another difficulty is the patient guarding on exam or not relaxing the knee due to the pain. All these factors can affect your diagnosis. In this patient’s case, it was less likely a meniscus injury because we would have expected a positive McMurry’s test with more significant joint line tenderness. A tear of the posterior collateral ligament was also less likely based on the mechanism of injury, negative posterior drawer test, and low occurrence of PCL tears in teenage female athletes. Medial/Lateral collateral ligament injuries are less likely based on the patient’s physical exam, with no laxity/significant pain with valgus or Varus stress on the knee. Patellar instability is also a common problem in this population. The patient denied feeling her patella dislocate. Also, if this is what happened, the patient would likely have less pain with minimal effusion still present. Overall, the only way to ensure diagnosis is an MRI and x-ray. In this case, although our history and exam pointed in the direction of an ACL tear, the MRI was able to confirm that therefore ruling out the other possible diagnoses.

    References:

    1. Bridge-Enhanced ACL Repair Clinical Trial | Boston Children’s Hospital. Boston Children’s Hospital. http://www.childrenshospital.org/aclbeartrial. Accessed December 2, 2018.

    2. DynaMed Plus. Anterior Cruciate Ligament (ACL) Injury. https://www.dynamed.com/ topics/dmp~AN~T114675/Anterior-cruciate-ligament-ACL-injury. Published 1995. Updated June 26, 2017. Accessed December 1, 2018.

    3. McPhee Sj, Papadakis MA, Rabow MW. Current medical diagnosis & treatment 2017. New York: McGraw Hill Education; 2017.

    4. Skinner HB, McMahon PJ. Current Diagnosis & Treatment Orthopedics . 5th ed. McGraw-Hill Education; 2014.

  2. Question 2 of 220
    2. Question

    A 76-year-old male presents with an erythematous, crusty lesion on the left side of his nose. The lesion has not gone away for the past 5 months and bleeds sometimes. You greet the patient and he tells you that he has spent many years in the sun growing up without wearing sunscreen. You view the lesion and it looks pearly with rolled borders and a central ulceration. What is the patient’s most likely diagnosis?

    Correct

    Explanation:
    The answer is not seborrheic keratosis. Seborrheic keratoses do not typically bleed and are not erythematous by nature. They do not have rolled borders with central ulcerations, but instead a greasy, stuck-on appearance. The answer is not squamous cell carcinoma. Although skin cancers do bleed sometimes, squamous cell carcinoma has a different appearance. Squamous cell carcinoma appears as a red, elevated thickened nodule with white scaly, or crusted margins. This description is different from basal cell carcinoma. Benign nevus is incorrect because they are usually flesh colored, not pearly or crusty, and do not bleed. Basal cell carcinoma is the correct answer. This diagnosis fits the description in the case vignette.

    Bibliography

    1. Samarasinghe V, Madan V, Lear JT. Focus on Basal Cell Carcinoma. Journal of Skin Cancer. 2011;2011:328615. doi:10.1155/2011/328615.

    2. Long-term recurrence rates in previously untreated (primary) basal cell carcinoma: implications for patient follow-up. Rowe DE, Carroll RJ, Day CL Jr J Dermatology Surgery Oncology. 1989 Mar; 15(3):315-28.

    3. Tallon, B. (2016, December). Basal cell carcinoma pathology. Retrieved from https://www.dermnetnz.org/topics/basal-cell-carcinoma-pathology/

    4. Wong CSM, Strange RC, Lear JT. Basal cell carcinoma. BMJ: British Medical Journal. 2003;327(7418):794-798.

    5. Baheti, A. D. (2015). Basal Cell Carcinoma: A Comprehensive Review for the Radiologist. American Journal of Roentgenology. Retrieved from https://www.ajronline.org/doi/pdf/10.2214/AJR.14.13160

    Incorrect

    Explanation:
    The answer is not seborrheic keratosis. Seborrheic keratoses do not typically bleed and are not erythematous by nature. They do not have rolled borders with central ulcerations, but instead a greasy, stuck-on appearance. The answer is not squamous cell carcinoma. Although skin cancers do bleed sometimes, squamous cell carcinoma has a different appearance. Squamous cell carcinoma appears as a red, elevated thickened nodule with white scaly, or crusted margins. This description is different from basal cell carcinoma. Benign nevus is incorrect because they are usually flesh colored, not pearly or crusty, and do not bleed. Basal cell carcinoma is the correct answer. This diagnosis fits the description in the case vignette.

    Bibliography

    1. Samarasinghe V, Madan V, Lear JT. Focus on Basal Cell Carcinoma. Journal of Skin Cancer. 2011;2011:328615. doi:10.1155/2011/328615.

    2. Long-term recurrence rates in previously untreated (primary) basal cell carcinoma: implications for patient follow-up. Rowe DE, Carroll RJ, Day CL Jr J Dermatology Surgery Oncology. 1989 Mar; 15(3):315-28.

    3. Tallon, B. (2016, December). Basal cell carcinoma pathology. Retrieved from https://www.dermnetnz.org/topics/basal-cell-carcinoma-pathology/

    4. Wong CSM, Strange RC, Lear JT. Basal cell carcinoma. BMJ: British Medical Journal. 2003;327(7418):794-798.

    5. Baheti, A. D. (2015). Basal Cell Carcinoma: A Comprehensive Review for the Radiologist. American Journal of Roentgenology. Retrieved from https://www.ajronline.org/doi/pdf/10.2214/AJR.14.13160

  3. Question 3 of 220
    3. Question

    A 52-year-old Caucasian female presents to her primary care with pain and tingling in the first and second digit of her right hand. The patient states she has been working as a waitress for 12 years and often has increased pain after a long shift. The patient also notes that she occasionally is awoken at night from the tingling sensation. When the median nerve is tapped at the wrist, the patient notes “shocks” in her hand. What modality would be the best initial test to confirm the diagnosis?

    Correct

    Correct answer is B. The patient is presenting with common symptoms of carpal tunnel syndrome therefore, EMG Nerve Conduction studies are the most appropriate modality. This modality has an over 95% specificity and is able to determine the severity and location of the nerve compression. MRI is incorrect because it is only recommended when the clinical picture is not straightforward or if nerve conduction studies are inconclusive.
    Carpal tunnel syndrome is a collection of symptoms caused by compression of the median nerve as it travels through the carpal tunnel in the wrist. Symptoms include pain, paresthesia of the fingers, and less commonly weakness, in the distribution of the median nerve which includes the thumb, index, middle finger and radial half of the ring finger. The classic examination includes performing the Phalen and Tinel test. The Phalen maneuver involves fully flexing the palm at the wrist with the elbow in full extension. Alternatively, the backs of the hands are placed against each other to provide hyperflexion of the wrist with the elbows remaining flexed. Pain and/or paresthesia in the median nerve distribution with one minute of wrist flexion is a positive test. Sensitivity is 68%, specificity is 73%. The Tinel test involves percussion over the proximal or directly on top of the carpal tunnel region. Again, pain and/or paresthesia is a positive test with a 50% sensitivity anda 77% specificity. Treatment options involve splinting, clucocorticoid injection, or surgical decompression.

    Sources:
    1) Middleton WD, e. (2018). MR imaging of the carpal tunnel: normal anatomy and preliminary findings in the carpal tunnel syndrome. – PubMed – NCBI. [online] Ncbi.nlm.nih.gov. Available at: https://www.ncbi.nlm.nih.gov/pubmed/3492109 [Accessed 8 Jul. 2018].

    2) W,B.(2018). Radiologic imaging of the caroal tunnel. –PubMed- NCBI. [online] Ncbi.nlm.nih.gov available at: http://www.ncbi.nlm.nih.gov/pubmed/9283839 [accessed8 Jul. 2018].

    3) UpToDate

    Incorrect

    Correct answer is B. The patient is presenting with common symptoms of carpal tunnel syndrome therefore, EMG Nerve Conduction studies are the most appropriate modality. This modality has an over 95% specificity and is able to determine the severity and location of the nerve compression. MRI is incorrect because it is only recommended when the clinical picture is not straightforward or if nerve conduction studies are inconclusive.
    Carpal tunnel syndrome is a collection of symptoms caused by compression of the median nerve as it travels through the carpal tunnel in the wrist. Symptoms include pain, paresthesia of the fingers, and less commonly weakness, in the distribution of the median nerve which includes the thumb, index, middle finger and radial half of the ring finger. The classic examination includes performing the Phalen and Tinel test. The Phalen maneuver involves fully flexing the palm at the wrist with the elbow in full extension. Alternatively, the backs of the hands are placed against each other to provide hyperflexion of the wrist with the elbows remaining flexed. Pain and/or paresthesia in the median nerve distribution with one minute of wrist flexion is a positive test. Sensitivity is 68%, specificity is 73%. The Tinel test involves percussion over the proximal or directly on top of the carpal tunnel region. Again, pain and/or paresthesia is a positive test with a 50% sensitivity anda 77% specificity. Treatment options involve splinting, clucocorticoid injection, or surgical decompression.

    Sources:
    1) Middleton WD, e. (2018). MR imaging of the carpal tunnel: normal anatomy and preliminary findings in the carpal tunnel syndrome. – PubMed – NCBI. [online] Ncbi.nlm.nih.gov. Available at: https://www.ncbi.nlm.nih.gov/pubmed/3492109 [Accessed 8 Jul. 2018].

    2) W,B.(2018). Radiologic imaging of the caroal tunnel. –PubMed- NCBI. [online] Ncbi.nlm.nih.gov available at: http://www.ncbi.nlm.nih.gov/pubmed/9283839 [accessed8 Jul. 2018].

    3) UpToDate

  4. Question 4 of 220
    4. Question

    A 54-year-old male presents to the office complaining of a non-productive cough for the past 5 months, intermittent dyspnea and fatigue. The patient denies current medications. The patient’s vital signs are unremarkable except for a SPO2 of 94%. Upon examination you notice a barrel chested patient with diminished air movement bilaterally. PFTs show an FEV1 of 84% and FEV1/FVC <70%. What is the recommended medical treatment for this patient according to the GOLD criteria?

    Correct

    The answer is B. The patient presents with the signs and symptoms of emphysema. The patient has an FEV1/FVC ration of <70% which indicates an obstructive condition as well as diminished lung sounds which are consistent with emphysema.Chronic bronchitis patients will typically be overweight and cyanotic while those with emphysema will present with a barrel chest, accessory muscle use, and will appear underweight utilizing a puff-like inhalation technique.
    Pulmonary function tests (PFTs) are used to confirm the diagnosis and help to determine the type of COPD that a patient has as well as the effectiveness of treatment regimens. The most widely accepted criteria in determining the appropriate treatment according to the disease stage is the GOLD criteria which is composed of 4 stages. Stage 1 = FEV1  80% and the patient should receive a short acting bronchodilator as well as vaccines for pneumonia and influenza. Stage 2 = FEV1 of 50-79% and the patient should add a long acting bronchodilator. Stage 3 = FEV1 of 30-50% and the patient should undergo pulmonary rehabilitation as well as receive an inhaled corticosteroid for acute flare-ups. Stage 4 = cor pulmonale, heart or respiratory failure or an FEV1  30%. These patients should add oxygen therapy to their regimen. Additionally, if the SPO2% drops below 88% or the PaO2 drops below 55mmHg then the patient should also begin oxygen therapy.

    References:
    1. COPD. dynamed.com. http://www.dynamed.com/topics/dmp~AN~T115557#Epidemiology. Published January 19, 2018. Accessed February 7, 2018.
    2. Ferguson G, Make B. Management of Stable Chronic Obstructive Pulmonary Disease. UpToDate.
    3. Papadakis MA, McPhee SJ, Rabow MW. 2017 Current medical diagnosis and treatment. New York.,
    NY: McGraw Hill Education; 2017.
    4. Williams DA. PANCE prep pearls: a medical study and review guide for the PANCE, PANRE & medical
    examinations. North Charleston, SC: CreateSpace Independent Publishing Platform; 2017.

    Incorrect

    The answer is B. The patient presents with the signs and symptoms of emphysema. The patient has an FEV1/FVC ration of <70% which indicates an obstructive condition as well as diminished lung sounds which are consistent with emphysema.Chronic bronchitis patients will typically be overweight and cyanotic while those with emphysema will present with a barrel chest, accessory muscle use, and will appear underweight utilizing a puff-like inhalation technique.
    Pulmonary function tests (PFTs) are used to confirm the diagnosis and help to determine the type of COPD that a patient has as well as the effectiveness of treatment regimens. The most widely accepted criteria in determining the appropriate treatment according to the disease stage is the GOLD criteria which is composed of 4 stages. Stage 1 = FEV1  80% and the patient should receive a short acting bronchodilator as well as vaccines for pneumonia and influenza. Stage 2 = FEV1 of 50-79% and the patient should add a long acting bronchodilator. Stage 3 = FEV1 of 30-50% and the patient should undergo pulmonary rehabilitation as well as receive an inhaled corticosteroid for acute flare-ups. Stage 4 = cor pulmonale, heart or respiratory failure or an FEV1  30%. These patients should add oxygen therapy to their regimen. Additionally, if the SPO2% drops below 88% or the PaO2 drops below 55mmHg then the patient should also begin oxygen therapy.

    References:
    1. COPD. dynamed.com. http://www.dynamed.com/topics/dmp~AN~T115557#Epidemiology. Published January 19, 2018. Accessed February 7, 2018.
    2. Ferguson G, Make B. Management of Stable Chronic Obstructive Pulmonary Disease. UpToDate.
    3. Papadakis MA, McPhee SJ, Rabow MW. 2017 Current medical diagnosis and treatment. New York.,
    NY: McGraw Hill Education; 2017.
    4. Williams DA. PANCE prep pearls: a medical study and review guide for the PANCE, PANRE & medical
    examinations. North Charleston, SC: CreateSpace Independent Publishing Platform; 2017.

  5. Question 5 of 220
    5. Question

    A 53 year old woman comes into your office because she noticed one spot of blood on her underwear last week. Her last known menstrual period was 12 months ago. She has no other symptoms. She has a history of heavy vaginal bleeding and thinks this could just be her period coming back again. Her medical history includes uterine fibroids, hypertension, and obesity. A transvaginal ultrasound shows an endometrial lining of 4.5mm. What is the next step?

    Correct

    Answer: B, Endometrial biopsy
    Endometrial cancer is the most common gynecologic malignancy and is most common in women who are perimenopausal or postmenopausal 3. It is a malignancy of the lining of the uterus. Ninety six percent of all uterine cancers occur in women who are older than 40 1. Endometrial cancer most commonly presents with abnormal uterine bleeding, in this case, postmenopausal bleeding. However, patient’s may also present with pelvic pain, abdominal pain, or changes in urination or bowel function 3. Risk factors include unopposed estrogen, nulliparity, history of infertility, irregular menses, obesity, high blood pressure, and a first degree relative with endometrial cancer 2. The patient may have a history of obesity, hypertension, or diabetes mellitus as these are common comorbid conditions 2. Transvaginal ultrasound with or without saline infused sonohysterography may be performed to measure the thickness of the endometrial lining. An endometrial lining greater than 4mm is an indication for biopsy of the endometrium 1. Diagnosis is confirmed via biopsy with histology. In patient’s whose biopsy comes back and is non-diagnostic, dilation and curettage may be performed 1. Dilation and curettage is more invasive than an endometrial biopsy and is therefore not a first line diagnostic tool for endometrial cancer unless there are complicating factors such as cervical stenosis or significant discomfort with biopsy 1. Pap smears are not a diagnostic tool for endometrial cancer. Pap smears that come back normal do not rule out endometrial cancer 1. If a pap smear comes back with abnormal cells in a patient that has endometrial cancer, it may be evidence of advanced disease 1. Pap smears should be performed on all women without history of cervical cancer or HPV every 3-5 years between the ages of 21 and 65 to screen for cervical dysplasia. Laparoscopy is the diagnostic tool of choice for patients with endometriosis where endometrial tissue may be found outside of the uterus for example on the ovaries or in the pelvis 2. Laparoscopy will typically not show any abnormalities with endometrial cancer unless there is metastasis into the abdominal cavity or the primary tumor involves the bladder and bowel 1. Staging is based on the primary tumor, the regional lymph nodes, and whether or not there is distant metastasis 1. The primary treatment for endometrial cancer is to perform a total abdominal hysterectomy with bilateral salpingo-oopgorectomy. Lymphadenectomy should be done based on the stage of cancer 3. Chemotherapy and radiation may be used as an added treatment modality 1. Radiation therapy alone may be used in patients who are not candidates for surgery such as those who are morbidly obese, have unresectable disease, or have other medical contraindications 3. Patients who have recurrence may be managed with high dose progestins or antiestrogens 2. Regular follow ups for pelvic and rectal exams should be done for patients with endometrial cancer every 3-6 months for 2 years and then every 6 months for 3 years 1.

    References:
    1. Endometrial Cancer. In DynaMed Plus [database online]. EBSCO Information Services. http://www.dynamed.com/topics/dmp~AN~T113952/Endometrial-cancer. Updated May 24, 2017. Accessed February 1, 2018.
    2. O’Connell C. A Comprehensive Review For The Certification And Recertification Examinations For Physician Assistants. 5th ed. Lippincott Raven; 2015:151-153.
    3. Endometrial Carcinoma. In UpToDate [database online]. Wolters Kluwer. https://www.uptodate.com/contents/overview-of-endometrial-carcinoma. Updated August 22, 2017. Accessed February 2, 2018.

    Incorrect

    Answer: B, Endometrial biopsy
    Endometrial cancer is the most common gynecologic malignancy and is most common in women who are perimenopausal or postmenopausal 3. It is a malignancy of the lining of the uterus. Ninety six percent of all uterine cancers occur in women who are older than 40 1. Endometrial cancer most commonly presents with abnormal uterine bleeding, in this case, postmenopausal bleeding. However, patient’s may also present with pelvic pain, abdominal pain, or changes in urination or bowel function 3. Risk factors include unopposed estrogen, nulliparity, history of infertility, irregular menses, obesity, high blood pressure, and a first degree relative with endometrial cancer 2. The patient may have a history of obesity, hypertension, or diabetes mellitus as these are common comorbid conditions 2. Transvaginal ultrasound with or without saline infused sonohysterography may be performed to measure the thickness of the endometrial lining. An endometrial lining greater than 4mm is an indication for biopsy of the endometrium 1. Diagnosis is confirmed via biopsy with histology. In patient’s whose biopsy comes back and is non-diagnostic, dilation and curettage may be performed 1. Dilation and curettage is more invasive than an endometrial biopsy and is therefore not a first line diagnostic tool for endometrial cancer unless there are complicating factors such as cervical stenosis or significant discomfort with biopsy 1. Pap smears are not a diagnostic tool for endometrial cancer. Pap smears that come back normal do not rule out endometrial cancer 1. If a pap smear comes back with abnormal cells in a patient that has endometrial cancer, it may be evidence of advanced disease 1. Pap smears should be performed on all women without history of cervical cancer or HPV every 3-5 years between the ages of 21 and 65 to screen for cervical dysplasia. Laparoscopy is the diagnostic tool of choice for patients with endometriosis where endometrial tissue may be found outside of the uterus for example on the ovaries or in the pelvis 2. Laparoscopy will typically not show any abnormalities with endometrial cancer unless there is metastasis into the abdominal cavity or the primary tumor involves the bladder and bowel 1. Staging is based on the primary tumor, the regional lymph nodes, and whether or not there is distant metastasis 1. The primary treatment for endometrial cancer is to perform a total abdominal hysterectomy with bilateral salpingo-oopgorectomy. Lymphadenectomy should be done based on the stage of cancer 3. Chemotherapy and radiation may be used as an added treatment modality 1. Radiation therapy alone may be used in patients who are not candidates for surgery such as those who are morbidly obese, have unresectable disease, or have other medical contraindications 3. Patients who have recurrence may be managed with high dose progestins or antiestrogens 2. Regular follow ups for pelvic and rectal exams should be done for patients with endometrial cancer every 3-6 months for 2 years and then every 6 months for 3 years 1.

    References:
    1. Endometrial Cancer. In DynaMed Plus [database online]. EBSCO Information Services. http://www.dynamed.com/topics/dmp~AN~T113952/Endometrial-cancer. Updated May 24, 2017. Accessed February 1, 2018.
    2. O’Connell C. A Comprehensive Review For The Certification And Recertification Examinations For Physician Assistants. 5th ed. Lippincott Raven; 2015:151-153.
    3. Endometrial Carcinoma. In UpToDate [database online]. Wolters Kluwer. https://www.uptodate.com/contents/overview-of-endometrial-carcinoma. Updated August 22, 2017. Accessed February 2, 2018.

  6. Question 6 of 220
    6. Question

    A 23-year-old female presents complaining of an acute onset of watery diarrhea that started 1 week ago. She states that she has had diarrhea up to 5 times per day with lower abdominal cramping and occasionally a small amount of blood in her stool. She had 2 episodes of vomiting and a low-grade fever prior to the diarrhea starting which has since resolved. She denies black stools, mucous in stools, or recent travel outside the US. The last meal she had prior to the onset was chicken salad at a new restaurant the day before she had the diarrhea. Vitals are HR 78, BP 110/78, RR 16, Temp 98.4 deg F. Based on her presentation, which of the following is the most likely cause of her symptoms?

    Correct

    Correct answer is A. Campylobacter jejuni, a gram-negative bacterium, is the most common cause of bacterial enteritis in the US and the second most common cause of foodborne disease behind Salmonella. It typically comes from the intestinal tracts of animals, the most common being poultry, but can also come from contaminated water and unpasteurized milk. Typical incubation period is no more than 3 days. Shigella sonnei is usually associated with more severe diarrhea described as explosive with mucous or blood. Vibrio Cholera usually causes high volume diarrhea that is odorless and described as “rice water.” Giardia lamblia is associated with swimming or drinking from a fresh water source commonly associated with camping or outdoor activities. The diarrhea from Giardia is more described as “frothy, greasy or foul.” E. coli is usually associated with more significant bloody diarrhea and is most commonly found in beef.

    Other symptoms associated with Campylobacter include fever, body aches, headache, dizziness and sometimes vomiting that may occur before the onset of diarrhea. Neurological symptoms such as progressive muscle weakness may indicate Guillain-Barre syndrome as Campylobacter is the most commonly identified causing agent of this disorder. The diagnosis is most commonly made with a stool culture. Campylobacter induced enteritis is usually self-limited. Most cases do not require antibiotics and education on supportive care with hydration and correcting electrolyte imbalances is sufficient. Antibiotics may shorten the duration if infection. Azithromycin 1 gram single dose or 500mg twice a day for 3 days can be used. Fluoroquinolones are also effective but have increased incidence of resistance and should not be first line.

    References:
    1. Williams DA. PANCE Prep Pearls: A Medical Study and Review Guide for the PANCE, PANRE & Medical Examinations. 2nd ed. North Charleston, SC: CreateSpace Independent Publishing Platform; 2017.
    2. Papadakis MA, McPhee SJ. Current Medical Diagnosis and Treatment 2017. (Rabow MW, ed.). McGraw Hill Education and Lange Medical Publications; 2017.
    3. Russell SM. Salmonella and Campylobacter: A Methods Update. Food Safety Magazine. https://www.foodsafetymagazine.com/magazine-archive1/december-2004january-2005/salmonella-and-campylobacter-a-methods-update/. Published 2005.
    4. Allos BM. Uptodate: Clinical manifestations, diagnosis, and treatment of Campylobacter infection. UpToDate, Inc. https://www.uptodate.com/contents/clinical-manifestations-diagnosis-and-treatment-of-campylobacter-infection?search=infectious colitis&source=search_result&selectedTitle=5~150&usage_type=default&display_rank=5. Published November 3, 2017.

    Incorrect

    Correct answer is A. Campylobacter jejuni, a gram-negative bacterium, is the most common cause of bacterial enteritis in the US and the second most common cause of foodborne disease behind Salmonella. It typically comes from the intestinal tracts of animals, the most common being poultry, but can also come from contaminated water and unpasteurized milk. Typical incubation period is no more than 3 days. Shigella sonnei is usually associated with more severe diarrhea described as explosive with mucous or blood. Vibrio Cholera usually causes high volume diarrhea that is odorless and described as “rice water.” Giardia lamblia is associated with swimming or drinking from a fresh water source commonly associated with camping or outdoor activities. The diarrhea from Giardia is more described as “frothy, greasy or foul.” E. coli is usually associated with more significant bloody diarrhea and is most commonly found in beef.

    Other symptoms associated with Campylobacter include fever, body aches, headache, dizziness and sometimes vomiting that may occur before the onset of diarrhea. Neurological symptoms such as progressive muscle weakness may indicate Guillain-Barre syndrome as Campylobacter is the most commonly identified causing agent of this disorder. The diagnosis is most commonly made with a stool culture. Campylobacter induced enteritis is usually self-limited. Most cases do not require antibiotics and education on supportive care with hydration and correcting electrolyte imbalances is sufficient. Antibiotics may shorten the duration if infection. Azithromycin 1 gram single dose or 500mg twice a day for 3 days can be used. Fluoroquinolones are also effective but have increased incidence of resistance and should not be first line.

    References:
    1. Williams DA. PANCE Prep Pearls: A Medical Study and Review Guide for the PANCE, PANRE & Medical Examinations. 2nd ed. North Charleston, SC: CreateSpace Independent Publishing Platform; 2017.
    2. Papadakis MA, McPhee SJ. Current Medical Diagnosis and Treatment 2017. (Rabow MW, ed.). McGraw Hill Education and Lange Medical Publications; 2017.
    3. Russell SM. Salmonella and Campylobacter: A Methods Update. Food Safety Magazine. https://www.foodsafetymagazine.com/magazine-archive1/december-2004january-2005/salmonella-and-campylobacter-a-methods-update/. Published 2005.
    4. Allos BM. Uptodate: Clinical manifestations, diagnosis, and treatment of Campylobacter infection. UpToDate, Inc. https://www.uptodate.com/contents/clinical-manifestations-diagnosis-and-treatment-of-campylobacter-infection?search=infectious colitis&source=search_result&selectedTitle=5~150&usage_type=default&display_rank=5. Published November 3, 2017.

  7. Question 7 of 220
    7. Question

    A 61-year-old female presents to the office complaining of gnawing epigastric pain that she’s had periodically over the last year but has progressed over the last month. Last week she reports coughing up a small amount of blood. She states that the pain is relieved with eating and with Tums. She states that it often wakes her up at night and is often worse when she drinks Fireball whiskey. Pt states she drinks about a pint of Fireball most nights of the week. Denies fatigue, shortness of breath, dizziness, nausea, or vomiting. On exam, vitals are normal, there is epigastric tenderness to palpation, and guaiac negative stool. What is the most likely organism responsible for her condition?

    Correct

    Correct answer is C. The risk factors associated with Peptic Ulcer Disease (PUD) include: Helicobacter Pylori, NSAID use, smoking, cytomegalovirus, Crohn’s disease, genetic factors, and associated conditions (hiatal hernia, Meckel diverticulum). H. pylori has a high urease activity that protects it from gastric acid secretions and stimulates increased acid secretion which can result in ulcers in the stomach or duodenum. Patients typically described a “gnawing” pain in the epigastric region which usually starts 2-5 hours after eating a meal and is relieved with eating and with antacids.
    Testing options include breath, stool and serology samples. Treatment typically involves one of two options: triple therapy (PPI, clarithromycin, and amoxicillin or metronidazole) or quadruple therapy (bismuth subsalicylate, metronidazole, tetracycline and a PPI), either is given X 14 days.
    References:
    1. Peptic Ulcer Disease. Dynamed. http://www.dynamed.com/topics/dmp~AN~T116374/Peptic-ulcer-disease#General-Information. Accessed February 7, 2018.
    2. National Institute for Health and Clinical Excellence (NICE). Dyspepsia and gastro-esophageal reflux disease: Investigation and management of dyspepsia, symptoms suggestive of gastro-esophageal reflux disease, or both. NICE 2014 Sep:CG184 PDF
    3. Malfertheiner P, Chan FK, McColl KE. Peptic ulcer disease. Lancet. 2009 Oct 24;374(9699):1449-61, commentary can be found in Lancet 2010 Feb 13;375(9714):553

    Incorrect

    Correct answer is C. The risk factors associated with Peptic Ulcer Disease (PUD) include: Helicobacter Pylori, NSAID use, smoking, cytomegalovirus, Crohn’s disease, genetic factors, and associated conditions (hiatal hernia, Meckel diverticulum). H. pylori has a high urease activity that protects it from gastric acid secretions and stimulates increased acid secretion which can result in ulcers in the stomach or duodenum. Patients typically described a “gnawing” pain in the epigastric region which usually starts 2-5 hours after eating a meal and is relieved with eating and with antacids.
    Testing options include breath, stool and serology samples. Treatment typically involves one of two options: triple therapy (PPI, clarithromycin, and amoxicillin or metronidazole) or quadruple therapy (bismuth subsalicylate, metronidazole, tetracycline and a PPI), either is given X 14 days.
    References:
    1. Peptic Ulcer Disease. Dynamed. http://www.dynamed.com/topics/dmp~AN~T116374/Peptic-ulcer-disease#General-Information. Accessed February 7, 2018.
    2. National Institute for Health and Clinical Excellence (NICE). Dyspepsia and gastro-esophageal reflux disease: Investigation and management of dyspepsia, symptoms suggestive of gastro-esophageal reflux disease, or both. NICE 2014 Sep:CG184 PDF
    3. Malfertheiner P, Chan FK, McColl KE. Peptic ulcer disease. Lancet. 2009 Oct 24;374(9699):1449-61, commentary can be found in Lancet 2010 Feb 13;375(9714):553

  8. Question 8 of 220
    8. Question

    75-year-old obese female with HTN, CKD and OA presents after falling in her driveway due to weakness in her legs. The patient reports a 2-day history of progressive weakness and paresthesias that started in her hands and feet and has progressed up to the forearms and knees bilaterally. On exam, the patient has diminished reflexes distally in the upper and lower extremities. Based on the most likely diagnosis, which of the following is this patient most at risk for?

    Correct

    Correct answer is D. The ascending weakness/paralysis described is characteristic of Guillain-Barre syndrome (GBS) and can affect not only the muscles of the extremities but also the respiratory muscles and less commonly, the bulbar muscles. Because of this many patients with GBS develop respiratory failure as the syndrome progresses. Fasciculations are commonly found in patients with ALS who will likely also have hyperreflexia. Optic Neuritis is a common symptom of Multiple Sclerosis and there is usually hyperreflexia and a positive Babinski. Ptosis is a common symptom of patients with Myasthenia Gravis and can be fatigable. Reflexes and sensation are usually normal throughout.

    GBS is defined as an immune-mediated acute polyneuropathy that is most often preceded by an infection and usually presents as ascending weakness and absent or decreased deep tendon reflexes with mild sensory deficits. Symptoms progresses over two weeks’ time reaching the end at four weeks. Diagnostic criteria include: 1) progressive weakness of legs and arms, which can range from mild to paralyzing and 2) decreased reflexes or areflexia in the effected extremities. Diagnosis is typically supported by CSF which characteristically demonstrates elevated protein with normal white cell count.

    Supportive care is the key to treatment of all patients with GBS. It is common for patients to be monitored in the ICU due to cardiac and respiratory complications of the disease, as upwards of 30% of patients develop respiratory failure during their course of the disease requiring mechanical ventilation. Plasma exchange or Immune Globulin (IVIG) is the standard of care. Deciding between the two treatments options is based on availability and patient/provider preference. Patients and their families must be aware that it can take upwards two to three months before some patients are able to walk without assistive devices.

    References:
    1. Dimachkie MM, Barohn RJ. Guillain-Barré Syndrome and Variants. Neurologic clinics. 2013;31(2):491-510. doi:10.1016/j.ncl.2013.01.005.
    2. Hahn AF. Guillain-Barre syndrome. Lancet. 1998 Aug 22;352(9128):635-41.
    3. Seneviratne U. Guillain-Barre syndrome. Postgrad Med J. 2000 Dec;76(902):774-82.
    4. Vriesendorp, F., MD. Guillain-Barré syndrome in adults: Clinical features and diagnosis. 2017, August 21.

    Incorrect

    Correct answer is D. The ascending weakness/paralysis described is characteristic of Guillain-Barre syndrome (GBS) and can affect not only the muscles of the extremities but also the respiratory muscles and less commonly, the bulbar muscles. Because of this many patients with GBS develop respiratory failure as the syndrome progresses. Fasciculations are commonly found in patients with ALS who will likely also have hyperreflexia. Optic Neuritis is a common symptom of Multiple Sclerosis and there is usually hyperreflexia and a positive Babinski. Ptosis is a common symptom of patients with Myasthenia Gravis and can be fatigable. Reflexes and sensation are usually normal throughout.

    GBS is defined as an immune-mediated acute polyneuropathy that is most often preceded by an infection and usually presents as ascending weakness and absent or decreased deep tendon reflexes with mild sensory deficits. Symptoms progresses over two weeks’ time reaching the end at four weeks. Diagnostic criteria include: 1) progressive weakness of legs and arms, which can range from mild to paralyzing and 2) decreased reflexes or areflexia in the effected extremities. Diagnosis is typically supported by CSF which characteristically demonstrates elevated protein with normal white cell count.

    Supportive care is the key to treatment of all patients with GBS. It is common for patients to be monitored in the ICU due to cardiac and respiratory complications of the disease, as upwards of 30% of patients develop respiratory failure during their course of the disease requiring mechanical ventilation. Plasma exchange or Immune Globulin (IVIG) is the standard of care. Deciding between the two treatments options is based on availability and patient/provider preference. Patients and their families must be aware that it can take upwards two to three months before some patients are able to walk without assistive devices.

    References:
    1. Dimachkie MM, Barohn RJ. Guillain-Barré Syndrome and Variants. Neurologic clinics. 2013;31(2):491-510. doi:10.1016/j.ncl.2013.01.005.
    2. Hahn AF. Guillain-Barre syndrome. Lancet. 1998 Aug 22;352(9128):635-41.
    3. Seneviratne U. Guillain-Barre syndrome. Postgrad Med J. 2000 Dec;76(902):774-82.
    4. Vriesendorp, F., MD. Guillain-Barré syndrome in adults: Clinical features and diagnosis. 2017, August 21.

  9. Question 9 of 220
    9. Question

    A 15-year-old male presents to the ED complaining of left foot pain. He states that he twisted his foot when he was tackled in a football game. His foot is swollen, tender to palpation, and there is significant ecchymosis on the plantar aspect. He cannot put weight on the foot. The patient admits to increased pain in the forefoot when passively abducted and pronated the. X-rays are obtained. What is the diagnosis?

    Correct

    The answer is C. A Lisfranc injury is a disruption of the tarsometatarsal joint (TMT) causing a displacement between the 1st and 2nd metatarsals either with or without fracture. A Jones fracture is a fracture through the diaphysis of the fifth metatarsal. A Maisonneuve fracture is a spiral fracture of the proximal fibula. It is caused by the rupture of the distal talofibular syndesmosis and the interosseous membrane due to a distal medial malleolar fracture and/or a deltoid ligament rupture. A Charcot’s joint occurs when the joint of the midfoot is damaged and destroyed secondary to peripheral neuropathy from diabetes mellitus or peripheral vascular disease. Radiographs of a Charcot foot will show obliteration of the joint space.

    The diagnosis of a Lisfranc injury requires a high degree of clinical suspicion. Injuries to the tarsometatarsal joint can be very subtle and are not always seen on radiographs. Approximately, 20% of the Lisfranc injuries initially go undiagnosed and nearly 40% do not receive treatment within the first week. AP, lateral and obliques radiographs are recommended to help diagnose a Lisfranc injury. Comparing views in weight bearing x-rays are sometimes helpful if non-weightbearing films are inconclusive.

    Treatment of Lisfranc injuries can be both operative and nonoperative depending on the severity of the joint damage. Nonoperative injuries include sprains and nondisplaced ligamentous injuries. These patients are placed in a non-weight bearing cast for six weeks followed by a weight bearing cast for an additional four to six weeks. The most common surgical approach to a Lisfranc injury is open reduction internal fixation (ORIF). Approximately 25% of patients who undergo ORIF for a Lisfranc injury will experience complications including broken hardware and arthritis. Posttraumatic arthritis, compartment syndrome, infection, complex regional pain syndrome, neurovascular injury, and nonunion are potential complications.

    References
    1. Physiopedia contributors. Foot and Ankle Structure and Function. Physiopedia. https://www.physio-pedia.com/Foot_and_Ankle_Structure_and_Function. Published February 8, 2018.

    2. Taylor T. Gliding Joint – Pivot Joints. InnerBody. http://www.innerbody.com/image_skel07/skel32.html#continued.

    3. Egol KA, Koval KJ, Zuckerman JD. Handbook of Fractures. Philadelphia, PA: Lippincott Williams & Wilkins; 2010.

    4. Jain M. Lisfranc’s amputation. LinkedIn Corporation. https://www.slideshare.net/mahaksayz/lisfranc-injury-72776736. Published March 3, 2017.

    5. Blomberg J. Lisfranc Injury (Tarsometatarsal fracture-dislocation). Orthobullets. https://www.orthobullets.com/foot-and-ankle/7030/lisfranc-injury-tarsometatarsal-fracture-dislocation.

    6. Fischer SJ. Lisfranc (Midfoot) Injury – OrthoInfo – AAOS. Lisfranc (Midfoot) Injury. https://orthoinfo.aaos.org/en/diseases–conditions/lisfranc-midfoot-injury/.

    7. Wheeless CR. Wheeless’ Textbook of Orthopaedics. Wheeless Online. http://www.wheelessonline.com/ortho/orif_of_lisfrancs_fracture. Published September 4, 2012.

    8. Williams DA. PANCE PREP PEARLS. 2nd ed. North Charleston, SC: CreateSpace Independent Publishing Platform; 2017.

    Incorrect

    The answer is C. A Lisfranc injury is a disruption of the tarsometatarsal joint (TMT) causing a displacement between the 1st and 2nd metatarsals either with or without fracture. A Jones fracture is a fracture through the diaphysis of the fifth metatarsal. A Maisonneuve fracture is a spiral fracture of the proximal fibula. It is caused by the rupture of the distal talofibular syndesmosis and the interosseous membrane due to a distal medial malleolar fracture and/or a deltoid ligament rupture. A Charcot’s joint occurs when the joint of the midfoot is damaged and destroyed secondary to peripheral neuropathy from diabetes mellitus or peripheral vascular disease. Radiographs of a Charcot foot will show obliteration of the joint space.

    The diagnosis of a Lisfranc injury requires a high degree of clinical suspicion. Injuries to the tarsometatarsal joint can be very subtle and are not always seen on radiographs. Approximately, 20% of the Lisfranc injuries initially go undiagnosed and nearly 40% do not receive treatment within the first week. AP, lateral and obliques radiographs are recommended to help diagnose a Lisfranc injury. Comparing views in weight bearing x-rays are sometimes helpful if non-weightbearing films are inconclusive.

    Treatment of Lisfranc injuries can be both operative and nonoperative depending on the severity of the joint damage. Nonoperative injuries include sprains and nondisplaced ligamentous injuries. These patients are placed in a non-weight bearing cast for six weeks followed by a weight bearing cast for an additional four to six weeks. The most common surgical approach to a Lisfranc injury is open reduction internal fixation (ORIF). Approximately 25% of patients who undergo ORIF for a Lisfranc injury will experience complications including broken hardware and arthritis. Posttraumatic arthritis, compartment syndrome, infection, complex regional pain syndrome, neurovascular injury, and nonunion are potential complications.

    References
    1. Physiopedia contributors. Foot and Ankle Structure and Function. Physiopedia. https://www.physio-pedia.com/Foot_and_Ankle_Structure_and_Function. Published February 8, 2018.

    2. Taylor T. Gliding Joint – Pivot Joints. InnerBody. http://www.innerbody.com/image_skel07/skel32.html#continued.

    3. Egol KA, Koval KJ, Zuckerman JD. Handbook of Fractures. Philadelphia, PA: Lippincott Williams & Wilkins; 2010.

    4. Jain M. Lisfranc’s amputation. LinkedIn Corporation. https://www.slideshare.net/mahaksayz/lisfranc-injury-72776736. Published March 3, 2017.

    5. Blomberg J. Lisfranc Injury (Tarsometatarsal fracture-dislocation). Orthobullets. https://www.orthobullets.com/foot-and-ankle/7030/lisfranc-injury-tarsometatarsal-fracture-dislocation.

    6. Fischer SJ. Lisfranc (Midfoot) Injury – OrthoInfo – AAOS. Lisfranc (Midfoot) Injury. https://orthoinfo.aaos.org/en/diseases–conditions/lisfranc-midfoot-injury/.

    7. Wheeless CR. Wheeless’ Textbook of Orthopaedics. Wheeless Online. http://www.wheelessonline.com/ortho/orif_of_lisfrancs_fracture. Published September 4, 2012.

    8. Williams DA. PANCE PREP PEARLS. 2nd ed. North Charleston, SC: CreateSpace Independent Publishing Platform; 2017.

  10. Question 10 of 220
    10. Question

    A 61-year-old male with a history of DM and significant tobacco use present to the ED complaining of left leg pain for several weeks. The pain is worse when he is walking and is relieve with rest. On examination, vitals are unremarkable and there are diminished pulses of both posterior tibialis and dorsalis pedis, with cool extremities and smooth and shiny skin. Which of the following is the most likely diagnosis?

    Correct

    Correct answer is C. The patient’s symptoms are most consistent with intermittent claudication secondary to peripheral artery disease (PAD). Deep Vein thrombosis (DVT) occurs when a blood clot develops in the deep venous system. On presentation, there will be unilateral leg pain, pitting edema, increase calf circumference and possible tenderness at the site of the clot. Chronic venous insufficiency is a condition in which the valves in the leg veins are not working effectively. This causes blood to collect in these veins resulting in progressive pitting edema of the leg. For superficial venous thrombosis, the pain is localized to the site of the superficial thrombus. The skin is generally indurated, warm, and tender with a possible palpable cord extending along the superficial vein and is most common along the saphenous vein.

    The most common cause of PAD in adults older than 40 years old is atherosclerosis. The other causes of PAD are thrombosis, embolism, trauma, vasculitis (Giant cell and Takayasu’s arteritis), fibromuscular dysplasia, and entrapment (thoracic outlet syndrome). Risk factors believed to contribute to the plaque formation are cigarette smoking, diabetes mellitus, hypercholesterolemia, and hypertension. Symptoms include pain, aching, cramping, numbness or muscle fatigue or atrophy. An ankle brachial index (ABI) of  0.9 is diagnostic of PAD. All pt’s with PAD should be counseled on smoking cessation and aggressive management of risk factors (BP, cholesterol, and glucose control). The primary medication with evidence for improved claudiacation is cilostazol, a phosphodiesterase inhibitor that suppresses platelet aggregation and is a direct arterial vasodilator. Antiplatelet agents may provide some improvement. Surgical options include angioplasty/stenting and bypass graft.

    Reference
    1. Peripheral Artery Disease. National Heart Lung and Blood Institute. https://www.nhlbi.nih.gov/health-topics/peripheral-artery-disease.

    2. Deep vein thrombosis (DVT). Mayo Clinic. https://www.mayoclinic.org/diseases-conditions/deep-vein-thrombosis/symptoms-causes/syc-20352557. Published July 12, 2017.

    3. Superficial Thrombophlebitis. Background, Pathophysiology, Etiology. https://emedicine.medscape.com/article/463256-overview. Published August 17, 2017.

    4. Venous Insufficiency. Background, Anatomy, Pathophysiology. https://emedicine.medscape.com/article/1085412-overview. Published September 29, 2017.

    5. Clinical features and diagnosis of lower extremity peripheral artery disease. Uptodate.
    https://www.uptodate.com/index.html#!/contents/clinical-features-and-diagnosis-of-lower-extremity-peripheralarterydisease?search=Peripheral%20artery%20disease&source=search_result&selectedTitle=1~150&usage_type=default&display_rank=1 .

    Incorrect

    Correct answer is C. The patient’s symptoms are most consistent with intermittent claudication secondary to peripheral artery disease (PAD). Deep Vein thrombosis (DVT) occurs when a blood clot develops in the deep venous system. On presentation, there will be unilateral leg pain, pitting edema, increase calf circumference and possible tenderness at the site of the clot. Chronic venous insufficiency is a condition in which the valves in the leg veins are not working effectively. This causes blood to collect in these veins resulting in progressive pitting edema of the leg. For superficial venous thrombosis, the pain is localized to the site of the superficial thrombus. The skin is generally indurated, warm, and tender with a possible palpable cord extending along the superficial vein and is most common along the saphenous vein.

    The most common cause of PAD in adults older than 40 years old is atherosclerosis. The other causes of PAD are thrombosis, embolism, trauma, vasculitis (Giant cell and Takayasu’s arteritis), fibromuscular dysplasia, and entrapment (thoracic outlet syndrome). Risk factors believed to contribute to the plaque formation are cigarette smoking, diabetes mellitus, hypercholesterolemia, and hypertension. Symptoms include pain, aching, cramping, numbness or muscle fatigue or atrophy. An ankle brachial index (ABI) of  0.9 is diagnostic of PAD. All pt’s with PAD should be counseled on smoking cessation and aggressive management of risk factors (BP, cholesterol, and glucose control). The primary medication with evidence for improved claudiacation is cilostazol, a phosphodiesterase inhibitor that suppresses platelet aggregation and is a direct arterial vasodilator. Antiplatelet agents may provide some improvement. Surgical options include angioplasty/stenting and bypass graft.

    Reference
    1. Peripheral Artery Disease. National Heart Lung and Blood Institute. https://www.nhlbi.nih.gov/health-topics/peripheral-artery-disease.

    2. Deep vein thrombosis (DVT). Mayo Clinic. https://www.mayoclinic.org/diseases-conditions/deep-vein-thrombosis/symptoms-causes/syc-20352557. Published July 12, 2017.

    3. Superficial Thrombophlebitis. Background, Pathophysiology, Etiology. https://emedicine.medscape.com/article/463256-overview. Published August 17, 2017.

    4. Venous Insufficiency. Background, Anatomy, Pathophysiology. https://emedicine.medscape.com/article/1085412-overview. Published September 29, 2017.

    5. Clinical features and diagnosis of lower extremity peripheral artery disease. Uptodate.
    https://www.uptodate.com/index.html#!/contents/clinical-features-and-diagnosis-of-lower-extremity-peripheralarterydisease?search=Peripheral%20artery%20disease&source=search_result&selectedTitle=1~150&usage_type=default&display_rank=1 .

  11. Question 11 of 220
    11. Question

    A 74-year-old female patient with osteoporosis who had been taking alendronate for over about 8.5 years sustained right atypical femoral fracture when her grandson ran into her with his bicycle, causing a minor trauma. Subsequently, she was found to have a left lower parathyroid adenoma. Medical history is notable for hypertension and type 2 diabetes mellitus. She has no kidney stones. Her family history is positive for osteoporosis in her sister. She experienced a natural menopause at age 50. Laboratory results reveal a Calcium 11.0 mg/dl (normal 8.5 – 10.8 mg/dl), Parathyroid hormone 72.3 pg/ml (normal 15-65 pg/ml)and Thyroid stimulating hormone 1.7 mU/L (normal 0.4-5.0 mU/L). DXA bone densitometry scan shows a T-score of -3.0 at the left distal forearm, which is consistent with osteoporosis.Which of the following is the most likely cause of this patient’s osteoporosis?

    Correct

    Correct answer: C (Primary hyperparathyroidism).

    Rationale:
    Normally, elevated plasma calcium suppresses PTH secretion. However, in primary hyperparathyroidism (PHPT), PTH secretion from autonomously functioning parathyroid adenomas or parathyroid hyperplasia is not suppressed. This patient has hypercalcemia with an elevated PTH level, suggesting PHPT. PHPT leads to increased bone turnover, low bone mineral density, and increased fracture risk. These effects are, however, preferentially seen in the distal forearm, which is rich in cortical bone. Bone density is lowest at the forearm in patients with PHPT, and the worst T-score is commonly found in the distal forearm in patients with PHPT9.

    Choice A: Incorrect. Milk-alkali syndrome is characterized by hypercalcemia caused by ingestion of large amounts of calcium and absorbable alkali (e.g., calcium-containing antacids). Calcium would be elevated, but PTH would be suppressed. Dietary supplements taken to prevent osteoporosis are common sources of calcium and alkali.10

    Choice B: Incorrect. Hyperthyroidism, if left untreated, can cause rapid bone loss, leading to osteoporosis and increased risk of fracture. All patients with primary hyperthyroidism have a low TSH. However, the TSH in this patient is normal, suggesting that she does not have hyperthyroidism.11

    Choice D: Incorrect. Hypoparathyroidism is characterized by low serum calcium, increased serum phosphate, and low serum parathyroid hormone levels. This patient has hypercalcemia with an elevated PTH level.12

    References:

    1. Riggs BL, Melton LJ 3rd. The worldwide problem of osteoporosis: insights afforded by epidemiology. Bone. 1995;17(5 Suppl):505S.

    2. National Osteoporosis Foundation, http://www.nof.org 2011.

    3. Hudec SM, Camacho PM. Secondary causes of osteoporosis. Endocr Pract. 2013;19(1):120-128.

    4. Griebeler ML, Kearns AE, Ryu E, Hathcock MA, Melton LJ, Wermers RA. Secular Trends in the Incidence of Primary Hyperparathyroidism Over Five Decades (1965-2010). Bone. 2015;73:1-7.

    5. Bilezikian JP, Silverberg SJ. Clinical practice. Asymptomatic primary hyperparathyroidism. N Engl J Med. 2004;350(17):1746.

    6. Cosman F, de Beur SJ, LeBoff MS, et al. Clinician’s guide to prevention and treatment of osteoporosis. Osteoporos Int. 2014;25(10):2359-81.

    7. Silverberg SJ, Shane E, Jacobs TP, et al. A 10-year prospective study of primary hyperparathyroidism with or without parathyroid surgery. N Engl J Med. 1999; 341(17):1249-1255.

    8. Hofbauer LC, Hamann C, Ebeling PR. Approach to the patient with secondary osteoporosis. Eur J Endocrinol. 2010;62(6):1009-1020.

    9. Wood K, Dhital S, Chen H, Sippel RS. What is the utility of distal forearm DXA in primary hyperparathyroidism? Oncologist. 2012; 17(3):322-325.

    10. Patel AM, Adeseun GA, Goldfarb S. Calcium-Alkali Syndrome in the Modern Era. Nutrients. 2013;5(12):4880-4893.

    11. De Leo S, Lee SY, Braverman LE. Hyperthyroidism. Lancet (London, England). 2016;388(10047):906-918.

    12. Abate EG, Clarke BL. Review of Hypoparathyroidism. Frontiers in Endocrinology. 2016;7:172.

    Incorrect

    Correct answer: C (Primary hyperparathyroidism).

    Rationale:
    Normally, elevated plasma calcium suppresses PTH secretion. However, in primary hyperparathyroidism (PHPT), PTH secretion from autonomously functioning parathyroid adenomas or parathyroid hyperplasia is not suppressed. This patient has hypercalcemia with an elevated PTH level, suggesting PHPT. PHPT leads to increased bone turnover, low bone mineral density, and increased fracture risk. These effects are, however, preferentially seen in the distal forearm, which is rich in cortical bone. Bone density is lowest at the forearm in patients with PHPT, and the worst T-score is commonly found in the distal forearm in patients with PHPT9.

    Choice A: Incorrect. Milk-alkali syndrome is characterized by hypercalcemia caused by ingestion of large amounts of calcium and absorbable alkali (e.g., calcium-containing antacids). Calcium would be elevated, but PTH would be suppressed. Dietary supplements taken to prevent osteoporosis are common sources of calcium and alkali.10

    Choice B: Incorrect. Hyperthyroidism, if left untreated, can cause rapid bone loss, leading to osteoporosis and increased risk of fracture. All patients with primary hyperthyroidism have a low TSH. However, the TSH in this patient is normal, suggesting that she does not have hyperthyroidism.11

    Choice D: Incorrect. Hypoparathyroidism is characterized by low serum calcium, increased serum phosphate, and low serum parathyroid hormone levels. This patient has hypercalcemia with an elevated PTH level.12

    References:

    1. Riggs BL, Melton LJ 3rd. The worldwide problem of osteoporosis: insights afforded by epidemiology. Bone. 1995;17(5 Suppl):505S.

    2. National Osteoporosis Foundation, http://www.nof.org 2011.

    3. Hudec SM, Camacho PM. Secondary causes of osteoporosis. Endocr Pract. 2013;19(1):120-128.

    4. Griebeler ML, Kearns AE, Ryu E, Hathcock MA, Melton LJ, Wermers RA. Secular Trends in the Incidence of Primary Hyperparathyroidism Over Five Decades (1965-2010). Bone. 2015;73:1-7.

    5. Bilezikian JP, Silverberg SJ. Clinical practice. Asymptomatic primary hyperparathyroidism. N Engl J Med. 2004;350(17):1746.

    6. Cosman F, de Beur SJ, LeBoff MS, et al. Clinician’s guide to prevention and treatment of osteoporosis. Osteoporos Int. 2014;25(10):2359-81.

    7. Silverberg SJ, Shane E, Jacobs TP, et al. A 10-year prospective study of primary hyperparathyroidism with or without parathyroid surgery. N Engl J Med. 1999; 341(17):1249-1255.

    8. Hofbauer LC, Hamann C, Ebeling PR. Approach to the patient with secondary osteoporosis. Eur J Endocrinol. 2010;62(6):1009-1020.

    9. Wood K, Dhital S, Chen H, Sippel RS. What is the utility of distal forearm DXA in primary hyperparathyroidism? Oncologist. 2012; 17(3):322-325.

    10. Patel AM, Adeseun GA, Goldfarb S. Calcium-Alkali Syndrome in the Modern Era. Nutrients. 2013;5(12):4880-4893.

    11. De Leo S, Lee SY, Braverman LE. Hyperthyroidism. Lancet (London, England). 2016;388(10047):906-918.

    12. Abate EG, Clarke BL. Review of Hypoparathyroidism. Frontiers in Endocrinology. 2016;7:172.

  12. Question 12 of 220
    12. Question

    A 45-year-old woman presents to the ER via ambulance. She was found wandering the streets very disheveled, muttering to herself, and does not respond to paramedics’ questions. Her affect is flat and she expresses no emotion. She states that she believes that people are following her and hears people talking to her that no one else can see. Physical exam reveals her pupils are reactive and normal. When staff is able to find her family members, they tell doctors that she has been acting strange for five years now, and they are concerned that she could be on drugs. What is the most likely cause of this patient’s current state?

    Correct

    Correct answer is E. Schizophrenia is a mental disorder in which the mind of the patient and reality do not agree. Drug addiction could fit the scenario, but her pupils are reactive and her affect is flattened. Bipolar disorder is characterized by episodes of mania, hypomania, and major depression with delusions and hallucinations possible but usually without frank paranoia. Brief psychotic disorder would only have symptoms lasting less than six months. B-12 deficiency can cause some psychological symptoms, very closely resembling schizophrenia but the psychotic symptoms occur along with anxious symptoms. This patient did not have the hyper-anxious symptoms and exhibited more flattened symptoms.
    Schizophrenia commonly occurs in late adolescence or early adulthood, between ages of 16-45. Males will have a slightly younger onset age than females. Symptoms should be present for 6 months to confirm the diagnosis. Risk factors include genetics, environmental and socioeconomic factors. The period of onset is referred to as the prodromal phase which is characterized by the negative symptoms of the disease. Negative symptoms of schizophrenia include lack of motivation, lack of attention and social isolation, decline in self-care, and flattened affect. Positive symptoms which are more difficult to notice in the patient are hallucinations, delusions, debilitating thoughts. Patients with paranoid schizophrenia can be so overwhelmed by the positive symptoms that they are unable to lead a normal life.
    Antipsychotics are the treatment of choice to ease the positive symptoms. First generation medications for schizophrenia include Chlorpromazine, Fluphenazine, Haloperidol, Perphenazine, and Thiothixene. Second generation medications include Aripiprazole, Asenapine, Olanzapine, Quetiapine, and Risperidone. These medications can be used alone or in a cocktail to provide the greatest relief to the patients for symptom relief. These medications have a lot of side effects that can be a deterrent to the patient including weight gain, sexual problems, drowsiness, dizziness, restlessness, dry mouth, constipation, blurred vision, hypotension, and seizures.

    References
    1. Childhood trauma linked to schizophrenia. ScienceDaily. https://www.sciencedaily.com/releases/2012/04/120419102440.htm. Published April 19, 2012. Accessed July 8, 2018.
    2. Misdiagnosis of Schizophrenia, Paranoid Schizophrenia. The History of Schizophrenia. http://schizophrenia.com/family/misdiag.html. Accessed July 8, 2018.
    3. Paranoid Schizophrenia: Overview of Causes, Symptoms, & Treatments. PsyCom.net – Mental Health Treatment Resource Since 1986. https://www.psycom.net/paranoid-schizophrenia. Accessed July 8, 2018.
    4. schizophrenia P. What Is Paranoid Schizophrenia? WebMD. https://www.webmd.com/schizophrenia/guide/schizophrenia-paranoia#1. Accessed July 8, 2018.
    5. Tufan AE, Bilici R, Usta G, Erdoğan A. Advances in pediatrics. https://www.ncbi.nlm.nih.gov/pmc/articles/PMC3404901/. Published 2012. Accessed July 8, 2018.

    Incorrect

    Correct answer is E. Schizophrenia is a mental disorder in which the mind of the patient and reality do not agree. Drug addiction could fit the scenario, but her pupils are reactive and her affect is flattened. Bipolar disorder is characterized by episodes of mania, hypomania, and major depression with delusions and hallucinations possible but usually without frank paranoia. Brief psychotic disorder would only have symptoms lasting less than six months. B-12 deficiency can cause some psychological symptoms, very closely resembling schizophrenia but the psychotic symptoms occur along with anxious symptoms. This patient did not have the hyper-anxious symptoms and exhibited more flattened symptoms.
    Schizophrenia commonly occurs in late adolescence or early adulthood, between ages of 16-45. Males will have a slightly younger onset age than females. Symptoms should be present for 6 months to confirm the diagnosis. Risk factors include genetics, environmental and socioeconomic factors. The period of onset is referred to as the prodromal phase which is characterized by the negative symptoms of the disease. Negative symptoms of schizophrenia include lack of motivation, lack of attention and social isolation, decline in self-care, and flattened affect. Positive symptoms which are more difficult to notice in the patient are hallucinations, delusions, debilitating thoughts. Patients with paranoid schizophrenia can be so overwhelmed by the positive symptoms that they are unable to lead a normal life.
    Antipsychotics are the treatment of choice to ease the positive symptoms. First generation medications for schizophrenia include Chlorpromazine, Fluphenazine, Haloperidol, Perphenazine, and Thiothixene. Second generation medications include Aripiprazole, Asenapine, Olanzapine, Quetiapine, and Risperidone. These medications can be used alone or in a cocktail to provide the greatest relief to the patients for symptom relief. These medications have a lot of side effects that can be a deterrent to the patient including weight gain, sexual problems, drowsiness, dizziness, restlessness, dry mouth, constipation, blurred vision, hypotension, and seizures.

    References
    1. Childhood trauma linked to schizophrenia. ScienceDaily. https://www.sciencedaily.com/releases/2012/04/120419102440.htm. Published April 19, 2012. Accessed July 8, 2018.
    2. Misdiagnosis of Schizophrenia, Paranoid Schizophrenia. The History of Schizophrenia. http://schizophrenia.com/family/misdiag.html. Accessed July 8, 2018.
    3. Paranoid Schizophrenia: Overview of Causes, Symptoms, & Treatments. PsyCom.net – Mental Health Treatment Resource Since 1986. https://www.psycom.net/paranoid-schizophrenia. Accessed July 8, 2018.
    4. schizophrenia P. What Is Paranoid Schizophrenia? WebMD. https://www.webmd.com/schizophrenia/guide/schizophrenia-paranoia#1. Accessed July 8, 2018.
    5. Tufan AE, Bilici R, Usta G, Erdoğan A. Advances in pediatrics. https://www.ncbi.nlm.nih.gov/pmc/articles/PMC3404901/. Published 2012. Accessed July 8, 2018.

  13. Question 13 of 220
    13. Question

    A 45 year-old female is brought in to the Emergency Room by ambulance with the following presentation: reduced consciousness, muscular rigidity, tachycardia, diaphoresis, and a fever of 103.7. The client takes medications, including the following: Zyrtec (cetirizine), Prozac (fluoxetine), and Zyprexa (olanzapine). Based on her symptoms, you suspect which of the following?

    Correct

    A. Agranulocytosis is a blood dyscrasia. It is characterized by symptoms such as sore throat, fever, malaise, and mouth sores, as well as possibly other flu like symptoms. Agranulocytosis is diagnosed using a CBC w/Differential to determine the presence of leukopenia, or agranulocytosis. Patients at risk for developing agranulocytosis are either conventional or atypical antipsychotics. If not properly diagnosed and treated, agranulocytosis can result in death. Treatment generally includes removing the offending agent and monitoring for any signs/symptoms of infection and treating those accordingly.

    B. Serotonin Syndrome is associated with the use of SNRIs/SNRIS/MAOIs and is characterized by abdominal pain, diarrhea, diaphoresis, fever, tachycardia, elevated blood pressure, delirium, muscle spasms, increased motor activity, irritability/mood swings, hostility, irrational thought process, and seizures. In severe cases, fever can become dangerously high and there is risk of cardiovascular shock and even death. Serotonin Syndrome is rare but life-threatening and is the possible result of too high a dose of an antidepressant medication or interactions with other medications. Risk for Serotonin Syndrome is highest when an SSRI/SNRI is combined with an MAOI or when an MAOI is given to someone who was taking an SSRI/SNRI without at least a 14 day (but up to 5 week) washout period of the SSRI/SNRI.

    C. Neuroleptic Malignant Syndrome (NMS) is associated with the use of antipsychotic medications and is characterized by reduced consciousness, muscular rigidity, tachycardia, diaphoresis, hyperpyrexia, drooling, tachypnea, and labile hypertension. NMS is a life threatening medical emergency and thought to be connected to a change in brain dopamine activity. While it usually occurs early in therapy, it can occur at any point during medication therapy. Approximately 10% of cases result in death.

    D. Tardive Dyskinesia (TD) is associated with the use of antipsychotic medications and is characterized by involuntary tonic muscular contractions of the face, limbs, or trunk. While TD is serious, it is not life threatening. Women and older adults are more at risk for developing TD. It affects up to 50% of those receiving long term antipsychotic medications. Symptoms involving the face include protruding/rolling tongue, smacking or licking of the lips, blowing type movement of the lips. Symptoms involving the limbs include pill rolling (fingers), choreic movements (rapid, purposeless, irregular) and/or athetoid movements (slow, complex, serpentine-like). Symptoms involving the trunk include neck and shoulder movements, hip movements (jerks and rocking motions).

    References:
    Herzog, E. A. & Varcarolis, E. M. (2010). Schizophrenia. In Varcarolis, E. M. & Halter,
    M. J. (Eds.), Foundations of Psychiatric Mental Health Nursing: A Clinical Approach (pp. 306-343). St. Louis, MO: W.B. Saunders.

    Kozy, M. &Varcarolis, E. M. (2010). Depressive Disorders. In Varcarolis, E. M. &
    Halter, M. J. (Eds.), Foundations of Psychiatric Mental Health Nursing: A Clinical Approach (pp. 246-279). St. Louis, MO: W.B. Saunders.

    Incorrect

    A. Agranulocytosis is a blood dyscrasia. It is characterized by symptoms such as sore throat, fever, malaise, and mouth sores, as well as possibly other flu like symptoms. Agranulocytosis is diagnosed using a CBC w/Differential to determine the presence of leukopenia, or agranulocytosis. Patients at risk for developing agranulocytosis are either conventional or atypical antipsychotics. If not properly diagnosed and treated, agranulocytosis can result in death. Treatment generally includes removing the offending agent and monitoring for any signs/symptoms of infection and treating those accordingly.

    B. Serotonin Syndrome is associated with the use of SNRIs/SNRIS/MAOIs and is characterized by abdominal pain, diarrhea, diaphoresis, fever, tachycardia, elevated blood pressure, delirium, muscle spasms, increased motor activity, irritability/mood swings, hostility, irrational thought process, and seizures. In severe cases, fever can become dangerously high and there is risk of cardiovascular shock and even death. Serotonin Syndrome is rare but life-threatening and is the possible result of too high a dose of an antidepressant medication or interactions with other medications. Risk for Serotonin Syndrome is highest when an SSRI/SNRI is combined with an MAOI or when an MAOI is given to someone who was taking an SSRI/SNRI without at least a 14 day (but up to 5 week) washout period of the SSRI/SNRI.

    C. Neuroleptic Malignant Syndrome (NMS) is associated with the use of antipsychotic medications and is characterized by reduced consciousness, muscular rigidity, tachycardia, diaphoresis, hyperpyrexia, drooling, tachypnea, and labile hypertension. NMS is a life threatening medical emergency and thought to be connected to a change in brain dopamine activity. While it usually occurs early in therapy, it can occur at any point during medication therapy. Approximately 10% of cases result in death.

    D. Tardive Dyskinesia (TD) is associated with the use of antipsychotic medications and is characterized by involuntary tonic muscular contractions of the face, limbs, or trunk. While TD is serious, it is not life threatening. Women and older adults are more at risk for developing TD. It affects up to 50% of those receiving long term antipsychotic medications. Symptoms involving the face include protruding/rolling tongue, smacking or licking of the lips, blowing type movement of the lips. Symptoms involving the limbs include pill rolling (fingers), choreic movements (rapid, purposeless, irregular) and/or athetoid movements (slow, complex, serpentine-like). Symptoms involving the trunk include neck and shoulder movements, hip movements (jerks and rocking motions).

    References:
    Herzog, E. A. & Varcarolis, E. M. (2010). Schizophrenia. In Varcarolis, E. M. & Halter,
    M. J. (Eds.), Foundations of Psychiatric Mental Health Nursing: A Clinical Approach (pp. 306-343). St. Louis, MO: W.B. Saunders.

    Kozy, M. &Varcarolis, E. M. (2010). Depressive Disorders. In Varcarolis, E. M. &
    Halter, M. J. (Eds.), Foundations of Psychiatric Mental Health Nursing: A Clinical Approach (pp. 246-279). St. Louis, MO: W.B. Saunders.

  14. Question 14 of 220
    14. Question

    A 34 year-old female presents to your office for a routine physical. During the examination, she shares with you that approximately 2 months ago, she was in a bank during an armed robbery. She is reporting flashbacks of the robbery, avoids going in to banks, is having difficulty sleeping due to nightmares, and is constantly checking her surroundings when out in public. You suspect she is suffering from which one of the following?

    Correct

    Generalized Anxiety Disorder (GAD), which lasts 6 months or more, is characterized by excessive anxiety or worry about numerous things, usually in excess of the event or situation about which the person is worried. Worries are usually related to daily life stressors, such as finances, relationships, job related concerns, etc. Symptoms include restlessness, fatigue, an inability to concentrate/focus, irritability, tension, and poor sleep.

    Acute Stress Disorder shares symptoms similar to Post-Traumatic Stress Disorder (PTSD), such as poor sleep, irritability, flashbacks, avoidant behaviors, etc. However, there are 3 hallmark differences between the two disorders: length of time to onset of symptoms (within one month following the traumatic event for Acute Stress Disorder), length of time to resolution of symptoms (no more than 4 weeks for Acute Stress Disorder) and presence of at least 3 dissociative symptoms either during or after the traumatic event (Acute Stress Disorder). Dissociative symptoms include emotional numbing/detachment, reduced awareness of surroundings, depersonalization (sense of unreality related to self), derealization (sense of unreality related to the environment), and loss of memory of an important aspect of the trauma.

    Obsessive Compulsive Disorder (OCD) is characterized by obsessions and/or compulsions. Obsessions, (thoughts, impulses, or images) are pervasive and unable to be dismissed. Typically, individuals experiencing obsessive thoughts don’t want to have the thoughts and being unable to stop them causes severe anxiety. Compulsions (the behaviors) are ritualistic in nature and individuals feel driven to perform them in an attempt to temporarily reduce or relieve the anxiety. Because the relief is only temporary, the compulsive act must be repeated again and again. Obsessions and compulsions are considered problematic when they begin to interfere with daily functions and lead to an inability to focus on anything but the obsession or compulsion.

    Post-Traumatic Stress Disorder (PTSD) refers to a set of symptoms experienced after witnessing or being a part of a traumatic event, such as a car accident, fire, natural disaster, etc. Symptoms can occur any time after the traumatic event-as early as a few months after, or as long as several years after the event. Symptoms include: re-experiencing the event, including flashbacks, nightmares, and intrusive thoughts, and can be triggered by internal or external factors. Diagnostic criteria also includes avoidant type behaviors, such as staying away from people, places, or things that trigger thoughts of the event, feeling numb and disinterested, including anhedonia, feelings of guilt or depression, and an inability to recall the traumatic event. People with PTSD tend to be hypervigilant, have an easy startle response, difficulty sleeping, and are described as irritable or edgy. PTSD also includes symptoms such as inability to focus/concentrate, poor memory regarding the traumatic event, and a negative outlook. To make the diagnosis, symptoms have to be present for at least one month duration and a person must experience at least one re-experiencing and one avoidance symptom, and two each of the arousal/reactivity and mood/cognition criteria.

    References

    National Institute of Mental Health. (2016). Post-Traumatic Stress Disorder. Retrieved
    March 20, 2016, from http://www.nimh.nih.gov/health/topics/post-traumatic-stress-disorder-ptsd/index.shtml.

    Halter, M. J., Varcarolis, E. M. & Shoemaker, N. C. (2010). Anxiety and Anxiety Disorders. In Varcarolis, E. M. & Halter, M. J. (Eds.), Foundations of Psychiatric Mental Health Nursing: A Clinical Approach (pp. 212-245). St. Louis, MO: W.B. Saunders.

    Incorrect

    Generalized Anxiety Disorder (GAD), which lasts 6 months or more, is characterized by excessive anxiety or worry about numerous things, usually in excess of the event or situation about which the person is worried. Worries are usually related to daily life stressors, such as finances, relationships, job related concerns, etc. Symptoms include restlessness, fatigue, an inability to concentrate/focus, irritability, tension, and poor sleep.

    Acute Stress Disorder shares symptoms similar to Post-Traumatic Stress Disorder (PTSD), such as poor sleep, irritability, flashbacks, avoidant behaviors, etc. However, there are 3 hallmark differences between the two disorders: length of time to onset of symptoms (within one month following the traumatic event for Acute Stress Disorder), length of time to resolution of symptoms (no more than 4 weeks for Acute Stress Disorder) and presence of at least 3 dissociative symptoms either during or after the traumatic event (Acute Stress Disorder). Dissociative symptoms include emotional numbing/detachment, reduced awareness of surroundings, depersonalization (sense of unreality related to self), derealization (sense of unreality related to the environment), and loss of memory of an important aspect of the trauma.

    Obsessive Compulsive Disorder (OCD) is characterized by obsessions and/or compulsions. Obsessions, (thoughts, impulses, or images) are pervasive and unable to be dismissed. Typically, individuals experiencing obsessive thoughts don’t want to have the thoughts and being unable to stop them causes severe anxiety. Compulsions (the behaviors) are ritualistic in nature and individuals feel driven to perform them in an attempt to temporarily reduce or relieve the anxiety. Because the relief is only temporary, the compulsive act must be repeated again and again. Obsessions and compulsions are considered problematic when they begin to interfere with daily functions and lead to an inability to focus on anything but the obsession or compulsion.

    Post-Traumatic Stress Disorder (PTSD) refers to a set of symptoms experienced after witnessing or being a part of a traumatic event, such as a car accident, fire, natural disaster, etc. Symptoms can occur any time after the traumatic event-as early as a few months after, or as long as several years after the event. Symptoms include: re-experiencing the event, including flashbacks, nightmares, and intrusive thoughts, and can be triggered by internal or external factors. Diagnostic criteria also includes avoidant type behaviors, such as staying away from people, places, or things that trigger thoughts of the event, feeling numb and disinterested, including anhedonia, feelings of guilt or depression, and an inability to recall the traumatic event. People with PTSD tend to be hypervigilant, have an easy startle response, difficulty sleeping, and are described as irritable or edgy. PTSD also includes symptoms such as inability to focus/concentrate, poor memory regarding the traumatic event, and a negative outlook. To make the diagnosis, symptoms have to be present for at least one month duration and a person must experience at least one re-experiencing and one avoidance symptom, and two each of the arousal/reactivity and mood/cognition criteria.

    References

    National Institute of Mental Health. (2016). Post-Traumatic Stress Disorder. Retrieved
    March 20, 2016, from http://www.nimh.nih.gov/health/topics/post-traumatic-stress-disorder-ptsd/index.shtml.

    Halter, M. J., Varcarolis, E. M. & Shoemaker, N. C. (2010). Anxiety and Anxiety Disorders. In Varcarolis, E. M. & Halter, M. J. (Eds.), Foundations of Psychiatric Mental Health Nursing: A Clinical Approach (pp. 212-245). St. Louis, MO: W.B. Saunders.

  15. Question 15 of 220
    15. Question

    You are working with a man who was medically admitted 3 days ago for acute alcohol intoxication and subsequent withdrawals. Today during your rounds you notice he presents with an onset of visual and tactile hallucinations, disorientation, increased agitation, and mood lability. You suspect your patient’s symptoms are most likely related to which one of the following?

    Correct

    Pancreatitis is a medical condition, specifically an inflammatory disorder of the pancreas. While it can be associated with chronic alcohol use, this is not always the cause of pancreatitis. Symptoms are typically physical in nature, and do not include psychiatric symptoms, as mentioned in the above patient. Symptoms of pancreatitis include, but are not limited to, nausea and vomiting, abdominal pain and distention.

    Delirium tremens (DTs), also referred to as alcohol withdrawal delirium, generally occurs 2 to 3 days (though onset can occur later than that) after alcohol cessation or reduction in alcohol intake, and can last for 2 to 3 days. Delirium tremens/alcohol withdrawal delirium is characterized by agitation, insomnia, anxiety, change in cognition (confusion, disorientation, etc.), hallucinations (visual, tactile, etc.), and mood disturbance. Physical symptoms include elevated blood pressure, tachycardia, and diaphoresis. Delirium tremens/alcohol withdrawal delirium is considered a medical emergency and can result in death even if treated.

    Post-acute withdrawal syndrome (also known as protracted withdrawal) can occur any time after the acute withdrawal phase has passed, usually at least 7 days after last use of alcohol. It can last anywhere from a few days or weeks up to two or three years. Symptoms can include, but are not limited to, sleep disturbance, anxiety, mood lability, including irritability and depression, short-term memory difficulties, and fatigue.

    Sleep deprivation occurs when one doesn’t get enough sleep, sleeps at the wrong time of the day, or does not get good quality sleep. Signs and symptoms can include feeling tired and unrested upon awakening, poor focus and concentration, difficulty remembering things, difficulty learning things, low frustration tolerance, difficulty making decisions, slowed reaction time (i.e. such as when driving a car), slower to finish tasks and likely to make more mistakes, irritability, and worried thinking.

    References

    Councill III, J., Halter, M.J. & Smith-Dijulio, K. (2010). Addictive Disorders. In Varcarolis, E. M. & Halter, M. J. (Eds.), Foundations of Psychiatric Mental Health Nursing: A Clinical Approach (pp. 402-432). St. Louis, MO: W.B. Saunders.

    Huether, S.E. (1998). Alterations of Digestive Function. In McCance, K.L. & Huether, S.E. (Eds), Pathophysiology: The Biologic Basis for Disease in Adults and Children (pp. 1322-1379). St. Louis, MO: Mosby-Year Book, Inc.

    U.S. Department of Health & Human Services, National Institutes of Health, National Heart, Lung and Blood Institute. (2012). What are Sleep Deprivation and Deficiency? Retrieved March 20, 2016 from http://www.nhlbi.nih.gov/health/health-topics/topics/sdd. U.S. Department of Health & Human Services, Substance Abuse and Mental Health Services Administration, Center for Substance Abuse Treatment. (2010).

    Incorrect

    Pancreatitis is a medical condition, specifically an inflammatory disorder of the pancreas. While it can be associated with chronic alcohol use, this is not always the cause of pancreatitis. Symptoms are typically physical in nature, and do not include psychiatric symptoms, as mentioned in the above patient. Symptoms of pancreatitis include, but are not limited to, nausea and vomiting, abdominal pain and distention.

    Delirium tremens (DTs), also referred to as alcohol withdrawal delirium, generally occurs 2 to 3 days (though onset can occur later than that) after alcohol cessation or reduction in alcohol intake, and can last for 2 to 3 days. Delirium tremens/alcohol withdrawal delirium is characterized by agitation, insomnia, anxiety, change in cognition (confusion, disorientation, etc.), hallucinations (visual, tactile, etc.), and mood disturbance. Physical symptoms include elevated blood pressure, tachycardia, and diaphoresis. Delirium tremens/alcohol withdrawal delirium is considered a medical emergency and can result in death even if treated.

    Post-acute withdrawal syndrome (also known as protracted withdrawal) can occur any time after the acute withdrawal phase has passed, usually at least 7 days after last use of alcohol. It can last anywhere from a few days or weeks up to two or three years. Symptoms can include, but are not limited to, sleep disturbance, anxiety, mood lability, including irritability and depression, short-term memory difficulties, and fatigue.

    Sleep deprivation occurs when one doesn’t get enough sleep, sleeps at the wrong time of the day, or does not get good quality sleep. Signs and symptoms can include feeling tired and unrested upon awakening, poor focus and concentration, difficulty remembering things, difficulty learning things, low frustration tolerance, difficulty making decisions, slowed reaction time (i.e. such as when driving a car), slower to finish tasks and likely to make more mistakes, irritability, and worried thinking.

    References

    Councill III, J., Halter, M.J. & Smith-Dijulio, K. (2010). Addictive Disorders. In Varcarolis, E. M. & Halter, M. J. (Eds.), Foundations of Psychiatric Mental Health Nursing: A Clinical Approach (pp. 402-432). St. Louis, MO: W.B. Saunders.

    Huether, S.E. (1998). Alterations of Digestive Function. In McCance, K.L. & Huether, S.E. (Eds), Pathophysiology: The Biologic Basis for Disease in Adults and Children (pp. 1322-1379). St. Louis, MO: Mosby-Year Book, Inc.

    U.S. Department of Health & Human Services, National Institutes of Health, National Heart, Lung and Blood Institute. (2012). What are Sleep Deprivation and Deficiency? Retrieved March 20, 2016 from http://www.nhlbi.nih.gov/health/health-topics/topics/sdd. U.S. Department of Health & Human Services, Substance Abuse and Mental Health Services Administration, Center for Substance Abuse Treatment. (2010).

  16. Question 16 of 220
    16. Question

    A 24 year old female with history of depression, alcohol use disorder and bulimia nervosa presents to Inpatient Psychiatry from the ICU after her second suicide attempt in two years. She reports that she took Prozac (fluoxetine) in high school but cannot remember the dose or its effects. She also reports that she was prescribed Lexapro (escitalopram) and Abilify (aripiprazole) when she was 21. However, she reports she was inconsistent and not compliant with her medications at that time. Of the following, which medication might be a good option for this patient at this time?

    Correct

    Venlafaxine is not a recommended choice due to the patient’s history of non-compliance with medications. Abrupt discontinuation of venlafaxine without a taper over the course of several weeks, is known to cause an intolerable discontinuation syndrome. Symptoms of discontinuation syndrome include anxiety, dysphoria, various flu like symptoms (i.e. sweats, chills, body aches, nausea), dizziness, excessive sweating, and insomnia.

    Sertraline is the best choice at this time. While all SSRIs have the potential to cause a discontinuation syndrome when abruptly stopped, sertraline is among the safer SSRIs due to its longer half-life. Fluoxetine is the safest. However, the patient could not remember the past benefits, if any, and therefore a retrial is not necessarily indicated. The results of abrupt discontinuation are an important consideration in patients with a history of non-compliance. Sertraline is also effective for binge eating behaviors.

    Bupropion is contraindicated when there is a history of anorexia/bulimia, or when there has been an abrupt discontinuation of alcohol use. Bupropion is associated with an increased risk for seizures and is contraindicated for use in patients who purge. Bupropion is known to lower the seizure threshold. Purging also lowers the seizure threshold due to causing imbalance in electrolytes.

    Although it is possible that fluoxetine helped this patient in high school, her condition ultimately worsened, suggesting that the medication failed and that a different one is warranted.

    References

    Hirsch, M. & Birnbaum, Robert J. Selective serotonin reuptake inhibitors:pharmacology, administration, and side effects. In: UpToDate, Post TW (Ed.), UpToDate, Waltham, MA. (Accessed on February 7, 2016.)

    Hirsch, M. & Birnbaum, Robert J. Atypical antidepressants: Pharmacology,administration, and side effects. In: UpToDate, Post TW (Ed.), UpToDate, Waltham, MA. (Accessed on February 7, 2016.)

    Harvey, Richard A. & Champe, Pamela C. (Eds.). (2009). Pharmacology. Baltimore:Lipincott Williams & Wilkins.

    Incorrect

    Venlafaxine is not a recommended choice due to the patient’s history of non-compliance with medications. Abrupt discontinuation of venlafaxine without a taper over the course of several weeks, is known to cause an intolerable discontinuation syndrome. Symptoms of discontinuation syndrome include anxiety, dysphoria, various flu like symptoms (i.e. sweats, chills, body aches, nausea), dizziness, excessive sweating, and insomnia.

    Sertraline is the best choice at this time. While all SSRIs have the potential to cause a discontinuation syndrome when abruptly stopped, sertraline is among the safer SSRIs due to its longer half-life. Fluoxetine is the safest. However, the patient could not remember the past benefits, if any, and therefore a retrial is not necessarily indicated. The results of abrupt discontinuation are an important consideration in patients with a history of non-compliance. Sertraline is also effective for binge eating behaviors.

    Bupropion is contraindicated when there is a history of anorexia/bulimia, or when there has been an abrupt discontinuation of alcohol use. Bupropion is associated with an increased risk for seizures and is contraindicated for use in patients who purge. Bupropion is known to lower the seizure threshold. Purging also lowers the seizure threshold due to causing imbalance in electrolytes.

    Although it is possible that fluoxetine helped this patient in high school, her condition ultimately worsened, suggesting that the medication failed and that a different one is warranted.

    References

    Hirsch, M. & Birnbaum, Robert J. Selective serotonin reuptake inhibitors:pharmacology, administration, and side effects. In: UpToDate, Post TW (Ed.), UpToDate, Waltham, MA. (Accessed on February 7, 2016.)

    Hirsch, M. & Birnbaum, Robert J. Atypical antidepressants: Pharmacology,administration, and side effects. In: UpToDate, Post TW (Ed.), UpToDate, Waltham, MA. (Accessed on February 7, 2016.)

    Harvey, Richard A. & Champe, Pamela C. (Eds.). (2009). Pharmacology. Baltimore:Lipincott Williams & Wilkins.

  17. Question 17 of 220
    17. Question

    A 27 year-old woman presents to your emergency department at 2:00 AM with complaints of flushing, palpitations, and blurred vision, all of which began one hour ago. The patient admits to consuming five alcoholic drinks while at a holiday party earlier this evening, but denies other drug use. She took her normal medications before bed, which include an oral contraceptive pill (OCP) and amitriptyline 50mg for insomnia. However, the boyfriend states that in order to avoid a hangover the next morning, she took a couple of her sleeping pills as well as 800mg of ibuprofen. On physical exam she is a well-nourished young woman in moderate distress. Vital signs include: T 99.6 F, BP 98/72, HR 138, RR 24, O2 98% (room air). Her face and arms are flushed and pupils are slightly dilated. Auscultation of the heart reveals a regular, tachycardic rhythm and an electrocardiogram (ECG) reveals a regular sinus tachycardia, as well as a QRS interval of 140 msec. She is becoming progressively more confused and asks several times what your credentials are. Which treatment should first be initiated at this time?

    Correct

    Although indicated for other anticholinergic syndromes, the use of physostigmine is contraindicated in the setting of TCA toxicity due to association with cardiac arrest.

    Procainamide is also contraindicated, as it is an inhibitor of rapid sodium channels, which will only heighten the TCAs effect.
    The use of magnesium sulfate is known to help control cardiac arrhythmias, but is reserved only for cases of TCA toxicity which are refractory to initial sodium bicarbonate treatment.

    Sodium bicarbonate is the correct answer. The patient is experiencing symptoms of a toxic exposure to tricyclic antidepressants (TCAs), as revealed in her history of taking more than the prescribed dose of amitriptyline. TCA overdose commonly presents as cardiac toxicity, usually in the form of tachycardic arrhythmias. QRS prolongation and other ECG findings, such as exaggerated S and R waves, are usually found in the setting of cardiac toxicity. Other symptoms include anticholinergic effects such as flushing, dry mouth, blurred vision, urinary incontinence, sedation, confusion, etc. The anticholinergic effects of TCAs are further precipitated by alcohol consumption. Rapid intravenous (IV) sodium bicarbonate is the initial management for TCA toxicity with cardiac manifestations. This will work to decrease the QRS interval and prevent the potential evolution of fatal tachyarrhythmia, such as ventricular tachycardia (VT) or ventricular fibrillation (VF).

    References

    Kerr, G. W., McGruffie, A. C., & Wilkie, S. (n.d.). Tricyclic antidepressant overdose: A review. In Emerg Med J 2001;18:236–241. Retrieved June 11, 2016, from http://www.ncbi.nlm.nih.gov/pmc/articles/PMC1725608/pdf/v018p00236.pdf

    Schneider, G. (May 2003). Never Use Physostigmine in a TCA Overdose. In Emergency Medicine News, 25(5); 44. Retrieved June 11, 2016, from http://journals.lww.com/em-news/Fulltext/2003/05000/Never_Use_Physostigmineiin_a_TCA_Overdose.30.aspx

    Incorrect

    Although indicated for other anticholinergic syndromes, the use of physostigmine is contraindicated in the setting of TCA toxicity due to association with cardiac arrest.

    Procainamide is also contraindicated, as it is an inhibitor of rapid sodium channels, which will only heighten the TCAs effect.
    The use of magnesium sulfate is known to help control cardiac arrhythmias, but is reserved only for cases of TCA toxicity which are refractory to initial sodium bicarbonate treatment.

    Sodium bicarbonate is the correct answer. The patient is experiencing symptoms of a toxic exposure to tricyclic antidepressants (TCAs), as revealed in her history of taking more than the prescribed dose of amitriptyline. TCA overdose commonly presents as cardiac toxicity, usually in the form of tachycardic arrhythmias. QRS prolongation and other ECG findings, such as exaggerated S and R waves, are usually found in the setting of cardiac toxicity. Other symptoms include anticholinergic effects such as flushing, dry mouth, blurred vision, urinary incontinence, sedation, confusion, etc. The anticholinergic effects of TCAs are further precipitated by alcohol consumption. Rapid intravenous (IV) sodium bicarbonate is the initial management for TCA toxicity with cardiac manifestations. This will work to decrease the QRS interval and prevent the potential evolution of fatal tachyarrhythmia, such as ventricular tachycardia (VT) or ventricular fibrillation (VF).

    References

    Kerr, G. W., McGruffie, A. C., & Wilkie, S. (n.d.). Tricyclic antidepressant overdose: A review. In Emerg Med J 2001;18:236–241. Retrieved June 11, 2016, from http://www.ncbi.nlm.nih.gov/pmc/articles/PMC1725608/pdf/v018p00236.pdf

    Schneider, G. (May 2003). Never Use Physostigmine in a TCA Overdose. In Emergency Medicine News, 25(5); 44. Retrieved June 11, 2016, from http://journals.lww.com/em-news/Fulltext/2003/05000/Never_Use_Physostigmineiin_a_TCA_Overdose.30.aspx

  18. Question 18 of 220
    18. Question

    You are working in the emergency room and are called in to see a 32 year old female. She tells you that a few days ago she was experiencing flu-like symptoms, including fatigue, fever, and cough. These symptoms have resolved but she has now developed a rash. The patient shows you her arms and the trunk of her body, which are covered in a reddish/purplish rash that has blistered in some areas. The patient tells you she takes medications which include lisinopril and Prozac. She also tells you she started lamictal 2 weeks ago and is supposed to be on 50mg but “because it wasn’t working,” the patient increased her dose to 150mg. You suspect which of the following?

    Correct

    Stevens-Johnson Syndrome is characterized by symptoms such as tongue/facial swelling, hives, skin pain, red or purple rash that spreads within a few hours up to several days, and blistering and shedding of the skin. These symptoms can be precipitated by more generalize flu-like symptoms such as fever, cough, fatigue, and sore throat. Stevens-Johnson Syndrome is often associated with medications, such as lamictal. It is a medical emergency and requires medical intervention, including hospitalization. Treatment includes removing the offending agent (if it’s medication induced), fluid and nutrition management, medications to treat pain, itching, or infection, wound care, and eye care if the symptoms involve the eyes.

    Cellulitis is a skin infection caused by staphylococcus and affects the dermis and subcutaneous layers of the skin. Cellulitis is generally secondary to a skin wound, ulcer, infected hair follicles, or boils. Presentation of the affected area includes swelling, pain, and erythema. Treatment includes antibiotics.

    Folliculitis is a bacterial infection of the hair follicle, typically caused by staphylococcus aureus that has spread in to the opening of the hair follicle. Folliculitis presents as pustules surrounded by erythema, generally without systemic symptoms. Contributing factors to folliculitis include poor hygiene, prolonged moisture on the skin, and/or skin trauma. Treatment may include topical antibiotics as well as cleaning the area with warm water and soap.

    Eczema is a general term that refers to various types of skin swelling. Eczema is also referred to as dermatitis. Eczema generally refers to an inflammatory skin disorder that causes dry, itchy skin, as well as rashes on the face, hands, and feet as well as inside the elbow and behind the knees. There is not a known cause, though it can be related to environmental factors, such as skin irritants or various allergies. Treatment depends upon the cause of the eczema but may include use of moisturizers, antihistamines or topical steroids to help reduce the itching, and avoiding anything that is known to worsen the condition.

    References

    Huether, S.E. (1998). Structure, Function, and Disorders of the Integument. In McCance, K.L. & Huether,S.E. (Eds), Pathophysiology: The Biologic Basis for Disease in Adults and Children (pp. 1517-1554). St. Louis, MO: Mosby-Year Book, Inc.

    Stevens-Johnson syndrome. (n.d.). Retrieved May 14, 2016, from http://www.mayoclinic.org/diseases-conditions/stevens-johnson-syndrome/basics/treatment/con-20029623

    Incorrect

    Stevens-Johnson Syndrome is characterized by symptoms such as tongue/facial swelling, hives, skin pain, red or purple rash that spreads within a few hours up to several days, and blistering and shedding of the skin. These symptoms can be precipitated by more generalize flu-like symptoms such as fever, cough, fatigue, and sore throat. Stevens-Johnson Syndrome is often associated with medications, such as lamictal. It is a medical emergency and requires medical intervention, including hospitalization. Treatment includes removing the offending agent (if it’s medication induced), fluid and nutrition management, medications to treat pain, itching, or infection, wound care, and eye care if the symptoms involve the eyes.

    Cellulitis is a skin infection caused by staphylococcus and affects the dermis and subcutaneous layers of the skin. Cellulitis is generally secondary to a skin wound, ulcer, infected hair follicles, or boils. Presentation of the affected area includes swelling, pain, and erythema. Treatment includes antibiotics.

    Folliculitis is a bacterial infection of the hair follicle, typically caused by staphylococcus aureus that has spread in to the opening of the hair follicle. Folliculitis presents as pustules surrounded by erythema, generally without systemic symptoms. Contributing factors to folliculitis include poor hygiene, prolonged moisture on the skin, and/or skin trauma. Treatment may include topical antibiotics as well as cleaning the area with warm water and soap.

    Eczema is a general term that refers to various types of skin swelling. Eczema is also referred to as dermatitis. Eczema generally refers to an inflammatory skin disorder that causes dry, itchy skin, as well as rashes on the face, hands, and feet as well as inside the elbow and behind the knees. There is not a known cause, though it can be related to environmental factors, such as skin irritants or various allergies. Treatment depends upon the cause of the eczema but may include use of moisturizers, antihistamines or topical steroids to help reduce the itching, and avoiding anything that is known to worsen the condition.

    References

    Huether, S.E. (1998). Structure, Function, and Disorders of the Integument. In McCance, K.L. & Huether,S.E. (Eds), Pathophysiology: The Biologic Basis for Disease in Adults and Children (pp. 1517-1554). St. Louis, MO: Mosby-Year Book, Inc.

    Stevens-Johnson syndrome. (n.d.). Retrieved May 14, 2016, from http://www.mayoclinic.org/diseases-conditions/stevens-johnson-syndrome/basics/treatment/con-20029623

  19. Question 19 of 220
    19. Question

    Your patient presents to you with complaints of low energy, depressed mood, some irritability, lethargy and fatigue, weight gain, and memory issues. You suspect your patient is likely suffering from a depressive disorder. However, you decide to also rule out which medical condition as a contributing factor to the above symptoms?

    Correct

    Pancreatitis is a medical condition, specifically an inflammatory disorder of the pancreas. While it can be associated with chronic alcohol use, this is not always the cause of pancreatitis. Symptoms are typically physical in nature, and do not include psychiatric symptoms, as mentioned in the above patient. Symptoms of pancreatitis include, but are not limited to, nausea and vomiting, abdominal pain and distention.

    Cirrhosis is an inflammatory disease of the liver and is irreversible. Cirrhosis typically develops slowly over a number of years, though severity and rate of progression are dependent upon the cause. Contributing factors can include alcoholism and hepatitis. Signs and symptoms of cirrhosis can include, but are not limited to, jaundice, nausea, weight loss, fatigue, swelling, accumulation of fluid in the abdomen.

    Hypothyroidism refers to an underactive thyroid, meaning it does not produce enough thyroid stimulating hormone (TSH). While hypothyroidism is physiological in nature, it can often times present like a depressive disorder. Signs and symptoms of hypothyroidism can include, but are not limited to, weight loss, fatigue, depression, poor memory, body aches, and muscle weakness. Hypothyroidism can be the result of things such as autoimmune disorders, treatment of hyperthyroidism, and medications. Hypothyroidism is usually diagnosed by obtaining a TSH level, and at times, other thyroid tests such as Free T3 and T4. Treatment is usually with a thyroid hormone replacement medication, such as levothyroxine.

    Serotonin Syndrome is associated with the use of SNRIs/SNRIS/MAOIs and is characterized by abdominal pain, diarrhea, diaphoresis, fever, tachycardia, elevated blood pressure, delirium, muscle spasms, increased motor activity, irritability/mood swings, hostility, irrational thought process, and seizures. In severe cases, fever can become dangerously high and there is risk of cardiovascular shock and even death. Serotonin Syndrome is rare but life-threatening and is the possible result of too high a dose of an antidepressant medication or interactions with other medications. Risk for Serotonin Syndrome is highest when an SSRI/SNRI is combined with an MAOI or when an MAOI is given to someone who was taking an SSRI/SNRI without at least a 14 day (but up to 5 week) washout period of the SSRI/SNRI.

    References

    Cirrhosis. (2016). Retrieved May 14, 2016, from http://www.mayoclinic.org/diseases-conditions/cirrhosis/home/ovc-20187218

    Huether, S.E. (1998). Alterations of Digestive Function. In McCance, K.L. & Huether, S.E. (Eds), Pathophysiology: The Biologic Basis for Disease in Adults and Children (pp. 1322-1379). St. Louis, MO: Mosby-Year Book, Inc.

    Kozy, M. &Varcarolis, E. M. (2010). Depressive Disorders. In Varcarolis, E. M. & Halter, M. J. (Eds.), Foundations of Psychiatric Mental Health Nursing: A Clinical Approach (pp. 246-279). St. Louis, MO: W.B. Saunders.

    Incorrect

    Pancreatitis is a medical condition, specifically an inflammatory disorder of the pancreas. While it can be associated with chronic alcohol use, this is not always the cause of pancreatitis. Symptoms are typically physical in nature, and do not include psychiatric symptoms, as mentioned in the above patient. Symptoms of pancreatitis include, but are not limited to, nausea and vomiting, abdominal pain and distention.

    Cirrhosis is an inflammatory disease of the liver and is irreversible. Cirrhosis typically develops slowly over a number of years, though severity and rate of progression are dependent upon the cause. Contributing factors can include alcoholism and hepatitis. Signs and symptoms of cirrhosis can include, but are not limited to, jaundice, nausea, weight loss, fatigue, swelling, accumulation of fluid in the abdomen.

    Hypothyroidism refers to an underactive thyroid, meaning it does not produce enough thyroid stimulating hormone (TSH). While hypothyroidism is physiological in nature, it can often times present like a depressive disorder. Signs and symptoms of hypothyroidism can include, but are not limited to, weight loss, fatigue, depression, poor memory, body aches, and muscle weakness. Hypothyroidism can be the result of things such as autoimmune disorders, treatment of hyperthyroidism, and medications. Hypothyroidism is usually diagnosed by obtaining a TSH level, and at times, other thyroid tests such as Free T3 and T4. Treatment is usually with a thyroid hormone replacement medication, such as levothyroxine.

    Serotonin Syndrome is associated with the use of SNRIs/SNRIS/MAOIs and is characterized by abdominal pain, diarrhea, diaphoresis, fever, tachycardia, elevated blood pressure, delirium, muscle spasms, increased motor activity, irritability/mood swings, hostility, irrational thought process, and seizures. In severe cases, fever can become dangerously high and there is risk of cardiovascular shock and even death. Serotonin Syndrome is rare but life-threatening and is the possible result of too high a dose of an antidepressant medication or interactions with other medications. Risk for Serotonin Syndrome is highest when an SSRI/SNRI is combined with an MAOI or when an MAOI is given to someone who was taking an SSRI/SNRI without at least a 14 day (but up to 5 week) washout period of the SSRI/SNRI.

    References

    Cirrhosis. (2016). Retrieved May 14, 2016, from http://www.mayoclinic.org/diseases-conditions/cirrhosis/home/ovc-20187218

    Huether, S.E. (1998). Alterations of Digestive Function. In McCance, K.L. & Huether, S.E. (Eds), Pathophysiology: The Biologic Basis for Disease in Adults and Children (pp. 1322-1379). St. Louis, MO: Mosby-Year Book, Inc.

    Kozy, M. &Varcarolis, E. M. (2010). Depressive Disorders. In Varcarolis, E. M. & Halter, M. J. (Eds.), Foundations of Psychiatric Mental Health Nursing: A Clinical Approach (pp. 246-279). St. Louis, MO: W.B. Saunders.

  20. Question 20 of 220
    20. Question

    A 21 year old female presents to you with C/O the following symptoms: intermittent mood lability, including irritability; impulsivity, including promiscuity and substance use; difficulty with controlling her anger, which is usually out of context to the situation; poor self-image; difficulty maintaining meaningful relationships due to extreme idealization and devaluation of others; and parasuicidal behavior, including cutting. You believe the symptoms are indicative of which disorder?

    Correct

    Bipolar Disorder, also known as manic-depression, is a chronic and recurrent illness. Bipolar disorder is characterized by extreme mood swings. Highs (mania or hypomania) are a period of persistently elevated or irritable mood, with symptoms such a grandiosity, inflated self-esteem, decreased need for sleep, hyperverbal and/or pressured speech, perception that thoughts are racing, flight of ideas, distractibility, impulsivity (i.e. excessive spending or sexual indiscretions), and increase in goal directed activity with trouble completing tasks. Elevated moods often alternate with periods of severe depression and both typically require hospitalization for safety and stabilization. However, there can be periods of normal functioning. To make a diagnosis of bipolar disorder, symptoms of mania must be present for at least one week OR symptoms of hypomania must be present for at least 4 days.
    Schizophrenia is a thought disorder that can affect one’s ability to think logically, perceive reality accurately, socialize and communicate with others, and control emotions. Schizophrenia is typically characterized by the presence of delusions, hallucinations, disorganized speech, thoughts, or behavior, as well as flat affect, avolition, and poverty of speech.

    Post-Traumatic Stress Disorder (PTSD) refers to a set of symptoms experienced after witnessing or being a part of a traumatic event, such as a car accident, fire, natural disaster, etc. Symptoms include: re-experiencing the event, including flashbacks, nightmares, and intrusive thoughts, and can be triggered by internal or external factors. Other symptoms include avoidant type behaviors, such as staying away from people, places, or things that trigger thoughts of the event, feeling numb and disinterested, including anhedonia, feelings of guilt or depression, and an inability to recall the traumatic event. People with PTSD tend to be hypervigilant, have an easy startle response, difficulty sleeping, and are described as irritable or edgy. PTSD also includes symptoms such as inability to focus/concentrate, poor memory regarding the traumatic event, and a negative outlook.

    Borderline Personality Disorder is probably the most well-known of the personality disorders. It can be easily confused with bipolar disorder, due to the mood instability and impulsivity. While there is typically marked emotional dysregulation associated with borderline personality disorder, these symptoms are intermittent and typically only last from a few hours to no more than a few days. Other symptoms associated with borderline personality disorder include intense and/or unstable interpersonal relationships (tend to see either the positive (idealization) or negative (devaluation) in others, but not both), ineffective coping and self-soothing skills such as cutting or engaging in reckless behaviors (i.e. promiscuity, substance use, etc), negative self-image, feelings of emptiness and abandonment (real or perceived), paranoid ideation that is transient in nature and stress/anxiety induced, and recurrent suicidal/parasuicidal thoughts, behaviors, and gestures. While there are no medications indicated for the treatment of borderline personality disorder per se, medication therapy is typically aimed at targeting symptoms, such as anxiety, depression, anger, etc. Effective therapy modalities include individual therapy, cognitive behavioral therapy, dialectical behavior therapy, and group therapy.

    References

    Cihlar, C.A. & Shoemaker, N.C. (2010). Personality Disorders. In Varcarolis, E. M. & Halter, M. J. (Eds.), Foundations of Psychiatric Mental Health Nursing: A Clinical Approach (pp. 433-460). St. Louis, MO: W.B. Saunders.

    Halter, M. J., Varcarolis, E. M. & Shoemaker, N. C. (2010). Anxiety and Anxiety Disorders. In Varcarolis, E. M. & Halter, M. J. (Eds.), Foundations of Psychiatric Mental Health Nursing: A Clinical Approach (pp. 212-245). St. Louis, MO: W.B. Saunders.

    Halter, M. J. & Varcarcolis, E.M. (2010). Bipolar Disorders. In Varcarolis, E. M. & Halter, M. J. (Eds.), Foundations of Psychiatric Mental Health Nursing: A Clinical Approach (pp. 280-305). St. Louis, MO: W.B. Saunders.

    Herzog, E. A. & Varcarolis, E. M. (2010). Schizophrenia. In Varcarolis, E. M. & Halter,
    M. J. (Eds.), Foundations of Psychiatric Mental Health Nursing: A Clinical Approach (pp. 306-343). St. Louis, MO: W.B. Saunders.

    Incorrect

    Bipolar Disorder, also known as manic-depression, is a chronic and recurrent illness. Bipolar disorder is characterized by extreme mood swings. Highs (mania or hypomania) are a period of persistently elevated or irritable mood, with symptoms such a grandiosity, inflated self-esteem, decreased need for sleep, hyperverbal and/or pressured speech, perception that thoughts are racing, flight of ideas, distractibility, impulsivity (i.e. excessive spending or sexual indiscretions), and increase in goal directed activity with trouble completing tasks. Elevated moods often alternate with periods of severe depression and both typically require hospitalization for safety and stabilization. However, there can be periods of normal functioning. To make a diagnosis of bipolar disorder, symptoms of mania must be present for at least one week OR symptoms of hypomania must be present for at least 4 days.
    Schizophrenia is a thought disorder that can affect one’s ability to think logically, perceive reality accurately, socialize and communicate with others, and control emotions. Schizophrenia is typically characterized by the presence of delusions, hallucinations, disorganized speech, thoughts, or behavior, as well as flat affect, avolition, and poverty of speech.

    Post-Traumatic Stress Disorder (PTSD) refers to a set of symptoms experienced after witnessing or being a part of a traumatic event, such as a car accident, fire, natural disaster, etc. Symptoms include: re-experiencing the event, including flashbacks, nightmares, and intrusive thoughts, and can be triggered by internal or external factors. Other symptoms include avoidant type behaviors, such as staying away from people, places, or things that trigger thoughts of the event, feeling numb and disinterested, including anhedonia, feelings of guilt or depression, and an inability to recall the traumatic event. People with PTSD tend to be hypervigilant, have an easy startle response, difficulty sleeping, and are described as irritable or edgy. PTSD also includes symptoms such as inability to focus/concentrate, poor memory regarding the traumatic event, and a negative outlook.

    Borderline Personality Disorder is probably the most well-known of the personality disorders. It can be easily confused with bipolar disorder, due to the mood instability and impulsivity. While there is typically marked emotional dysregulation associated with borderline personality disorder, these symptoms are intermittent and typically only last from a few hours to no more than a few days. Other symptoms associated with borderline personality disorder include intense and/or unstable interpersonal relationships (tend to see either the positive (idealization) or negative (devaluation) in others, but not both), ineffective coping and self-soothing skills such as cutting or engaging in reckless behaviors (i.e. promiscuity, substance use, etc), negative self-image, feelings of emptiness and abandonment (real or perceived), paranoid ideation that is transient in nature and stress/anxiety induced, and recurrent suicidal/parasuicidal thoughts, behaviors, and gestures. While there are no medications indicated for the treatment of borderline personality disorder per se, medication therapy is typically aimed at targeting symptoms, such as anxiety, depression, anger, etc. Effective therapy modalities include individual therapy, cognitive behavioral therapy, dialectical behavior therapy, and group therapy.

    References

    Cihlar, C.A. & Shoemaker, N.C. (2010). Personality Disorders. In Varcarolis, E. M. & Halter, M. J. (Eds.), Foundations of Psychiatric Mental Health Nursing: A Clinical Approach (pp. 433-460). St. Louis, MO: W.B. Saunders.

    Halter, M. J., Varcarolis, E. M. & Shoemaker, N. C. (2010). Anxiety and Anxiety Disorders. In Varcarolis, E. M. & Halter, M. J. (Eds.), Foundations of Psychiatric Mental Health Nursing: A Clinical Approach (pp. 212-245). St. Louis, MO: W.B. Saunders.

    Halter, M. J. & Varcarcolis, E.M. (2010). Bipolar Disorders. In Varcarolis, E. M. & Halter, M. J. (Eds.), Foundations of Psychiatric Mental Health Nursing: A Clinical Approach (pp. 280-305). St. Louis, MO: W.B. Saunders.

    Herzog, E. A. & Varcarolis, E. M. (2010). Schizophrenia. In Varcarolis, E. M. & Halter,
    M. J. (Eds.), Foundations of Psychiatric Mental Health Nursing: A Clinical Approach (pp. 306-343). St. Louis, MO: W.B. Saunders.

  21. Question 21 of 220
    21. Question

    You are treating one of your male patients, diagnosed with bipolar disorder, with lithium. He has been on the lithium for 8 months without any reported concerns. He comes to a regular medication appointment and tells you that for the past 4 days, he has been experiencing “side effects” of the lithium. You ask him to describe what he’s been experiencing. Your patient tells you that he has been having nausea and diarrhea, accompanied by stomach pains. He reports excessive thirst and frequent urination. He also reports new onset of bilateral hand tremors and muscle weakness. Your patient’s symptoms are most likely related to?

    Correct

    Renal failure refers to the gradual loss of kidney function, and is typically a chronic process. Early stages of renal failure may be asymptomatic, with symptoms not presenting until late stage failure. Usual signs and symptoms of renal failure are generalized and non-specific and can look like numerous other illness processes. The signs and symptoms of renal failure include nausea, vomiting, poor appetite, fatigue, poor sleep, weakness, muscle cramping/twitching, swelling, changes in urine output, cognitive changes, and possibly chest pain, shortness of breath, and HTN if fluids build up in/around the heart and lungs. Causes of renal failure include, but are not limited to, diabetes, HTN, and recurrent infections.

    Lithium toxicity is associated with lithium levels greater than 1.2 mEq/L. Therapeutic levels are considered to be between 0.6-1.2 mEq/L. Toxicity symptoms range from mild to severe and/or life threatening. Common side effects of lithium can include fine tremors, mild thirst, frequent urination, and mild GI discomfort. While these symptoms can be present for the duration of lithium therapy, they are often self-limited and subside during treatment. Early symptoms of toxicity (levels < 1.5 mEq/L) include nausea, vomiting, diarrhea, excessive thirst, frequent urination, fine hand tremors, muscle weakness, lethargy, and slurred speech. As toxicity progresses (1.5-2.0 mEq/L), tremors become more exaggerated, mental status becomes altered with changes in EEG noted, gait becomes uncoordinated, and there is increased sedation. Severe toxicity symptoms (2.0 to > 2.5 mEq/L) include, but are not limited to, significant changes in EEG, ataxia, confusion, blurred vision, seizures, stupor, coma, and in extreme cases, death. Treatment includes holding the lithium and checking levels, re-evaluating dosage based on blood level. In moderate to severe cases, hospitalization may be required and in extreme cases, hemodialysis may be required to expedite the removal of lithium from the blood.

    Tardive Dyskinesia (TD) is associated with the use of antipsychotic medications and is characterized by involuntary tonic muscular contractions of the face, limbs, or trunk. While TD is serious, it is not life threatening. Women and older adults are more at risk for developing TD. It affects up to 50% of those receiving long term antipsychotic medications. Symptoms involving the face include protruding/rolling tongue, smacking or licking of the lips, blowing type movement of the lips. Symptoms involving the limbs include pill rolling (fingers), choreic movements (rapid, purposeless, irregular) and/or athetoid movements (slow, complex, serpentine-like). Symptoms involving the trunk include neck and shoulder movements, hip movements (jerks and rocking motions).

    Agranulocytosis is a blood dyscrasia. It is characterized by symptoms such as sore throat, fever, malaise, and mouth sores, as well as possibly other flu like symptoms. Agranulocytosis is diagnosed using a CBC w/Differential to determine the presence of leukopenia, or agranulocytosis. Patients at risk for developing agranulocytosis are either conventional or atypical antipsychotics. If not properly diagnosed and treated, agranulocytosis can result in death. Treatment generally includes removing the offending agent and monitoring for any signs/symptoms of infection and treating those accordingly.

    References

    Chronic kidney disease. (n.d.). Retrieved May 15, 2016, from http://www.mayoclinic.org/diseases-conditions/kidney-disease/basics/definition/con-20026778

    Halter, M. J. & Varcarcolis, E.M. (2010). Bipolar Disorders. In Varcarolis, E. M. & Halter, M. J. (Eds.), Foundations of Psychiatric Mental Health Nursing: A Clinical Approach (pp. 280-305). St. Louis, MO: W.B. Saunders.

    Herzog, E. A. & Varcarolis, E. M. (2010). Schizophrenia. In Varcarolis, E. M. & Halter, M. J. (Eds.), Foundations of Psychiatric Mental Health Nursing: A Clinical Approach (pp. 306-343). St. Louis, MO: W.B. Saunders.

    Incorrect

    Renal failure refers to the gradual loss of kidney function, and is typically a chronic process. Early stages of renal failure may be asymptomatic, with symptoms not presenting until late stage failure. Usual signs and symptoms of renal failure are generalized and non-specific and can look like numerous other illness processes. The signs and symptoms of renal failure include nausea, vomiting, poor appetite, fatigue, poor sleep, weakness, muscle cramping/twitching, swelling, changes in urine output, cognitive changes, and possibly chest pain, shortness of breath, and HTN if fluids build up in/around the heart and lungs. Causes of renal failure include, but are not limited to, diabetes, HTN, and recurrent infections.

    Lithium toxicity is associated with lithium levels greater than 1.2 mEq/L. Therapeutic levels are considered to be between 0.6-1.2 mEq/L. Toxicity symptoms range from mild to severe and/or life threatening. Common side effects of lithium can include fine tremors, mild thirst, frequent urination, and mild GI discomfort. While these symptoms can be present for the duration of lithium therapy, they are often self-limited and subside during treatment. Early symptoms of toxicity (levels < 1.5 mEq/L) include nausea, vomiting, diarrhea, excessive thirst, frequent urination, fine hand tremors, muscle weakness, lethargy, and slurred speech. As toxicity progresses (1.5-2.0 mEq/L), tremors become more exaggerated, mental status becomes altered with changes in EEG noted, gait becomes uncoordinated, and there is increased sedation. Severe toxicity symptoms (2.0 to > 2.5 mEq/L) include, but are not limited to, significant changes in EEG, ataxia, confusion, blurred vision, seizures, stupor, coma, and in extreme cases, death. Treatment includes holding the lithium and checking levels, re-evaluating dosage based on blood level. In moderate to severe cases, hospitalization may be required and in extreme cases, hemodialysis may be required to expedite the removal of lithium from the blood.

    Tardive Dyskinesia (TD) is associated with the use of antipsychotic medications and is characterized by involuntary tonic muscular contractions of the face, limbs, or trunk. While TD is serious, it is not life threatening. Women and older adults are more at risk for developing TD. It affects up to 50% of those receiving long term antipsychotic medications. Symptoms involving the face include protruding/rolling tongue, smacking or licking of the lips, blowing type movement of the lips. Symptoms involving the limbs include pill rolling (fingers), choreic movements (rapid, purposeless, irregular) and/or athetoid movements (slow, complex, serpentine-like). Symptoms involving the trunk include neck and shoulder movements, hip movements (jerks and rocking motions).

    Agranulocytosis is a blood dyscrasia. It is characterized by symptoms such as sore throat, fever, malaise, and mouth sores, as well as possibly other flu like symptoms. Agranulocytosis is diagnosed using a CBC w/Differential to determine the presence of leukopenia, or agranulocytosis. Patients at risk for developing agranulocytosis are either conventional or atypical antipsychotics. If not properly diagnosed and treated, agranulocytosis can result in death. Treatment generally includes removing the offending agent and monitoring for any signs/symptoms of infection and treating those accordingly.

    References

    Chronic kidney disease. (n.d.). Retrieved May 15, 2016, from http://www.mayoclinic.org/diseases-conditions/kidney-disease/basics/definition/con-20026778

    Halter, M. J. & Varcarcolis, E.M. (2010). Bipolar Disorders. In Varcarolis, E. M. & Halter, M. J. (Eds.), Foundations of Psychiatric Mental Health Nursing: A Clinical Approach (pp. 280-305). St. Louis, MO: W.B. Saunders.

    Herzog, E. A. & Varcarolis, E. M. (2010). Schizophrenia. In Varcarolis, E. M. & Halter, M. J. (Eds.), Foundations of Psychiatric Mental Health Nursing: A Clinical Approach (pp. 306-343). St. Louis, MO: W.B. Saunders.

  22. Question 22 of 220
    22. Question

    You are treating a 35 year old male patient who has been diagnosed with schizophrenia. He often presents as stuporous and is unable to engage with you. However, there are other times when he exhibits sudden an unexpected increases in motor activity, including jumping up out of his chair. You attribute this presentation to?

    Correct

    Catatonia is typically associated with schizophrenia, though it can also be part of a severe and/or treatment resistive depression. Catatonia is often characterized by either a complete lack of movement or grossly delayed movements (psychomotor retardation) fast, strange, or unpredictable movements (psychomotor agitation), and mutism. While a stuporous presentation (no movement at all) is the most common, catatonia can also present with intermittent increases in movement, often times referred to as stereotyped behaviors/movements, as they are repetitive and serve no logical purpose. A person experiencing catatonia may be able to follow simple commands in a robot-like fashion, may do the opposite of what is asked, or they may not be able to follow commands at all. A patient with catatonia may also demonstrate statue-like behavior, referred to as waxy flexibility (i.e. the provider raises the patient’s arm and the patient holds it there even once the provider has let go).

    Delusions refer to an altered thought process whereby one experiences false fixed beliefs that do not respond to reality testing. The person with delusions believes his/her thoughts to be reality. Types of delusions include persecutory or paranoid, grandiose, religious, or somatic. Delusional thinking can also refer to one’s belief that he/she can thought broadcast or insert/withdraw thoughts from other’s minds. Delusions are considered to be a positive symptom (presence of something that is not normally present) of a thought disorder, such as schizophrenia.

    Hallucinations, another positive symptom of an illness such as schizophrenia, is an alteration in sensory perception. A patient with hallucinations has a sensory experience despite lack of stimulus to create such an experience (i.e. feels a bug crawling on his/her body even though there is nothing there). Hallucinations can be experienced across all five senses. They can be tactile (feeling bodily sensations), auditory (hearing voices, noises, sounds, music), olfactory (smelling odors), visual (seeing objects or people), and gustatory (experiencing tastes).

    Tardive Dyskinesia (TD) is associated with the use of antipsychotic medications and is characterized by involuntary tonic muscular contractions of the face, limbs, or trunk. While TD is serious, it is not life threatening. Women and older adults are more at risk for developing TD. It affects up to 50% of those receiving long term antipsychotic medications. Symptoms involving the face include protruding/rolling tongue, smacking or licking of the lips, blowing type movement of the lips. Symptoms involving the limbs include pill rolling (fingers), choreic movements (rapid, purposeless, irregular) and/or athetoid movements (slow, complex, serpentine-like). Symptoms involving the trunk include neck and shoulder movements, hip movements (jerks and rocking motions).

    References

    Electroconvulsive therapy (ECT). (n.d.). Retrieved May 23, 2016, from http://www.mayoclinic.org/tests-procedures/electroconvulsive-therapy/basics/why-its-done/prc-20014161

    Herzog, E. A. & Varcarolis, E. M. (2010). Schizophrenia. In Varcarolis, E. M. & Halter, M. J. (Eds.), Foundations of Psychiatric Mental Health Nursing: A Clinical Approach (pp. 306-343). St. Louis, MO: W.B. Saunders.

    Incorrect

    Catatonia is typically associated with schizophrenia, though it can also be part of a severe and/or treatment resistive depression. Catatonia is often characterized by either a complete lack of movement or grossly delayed movements (psychomotor retardation) fast, strange, or unpredictable movements (psychomotor agitation), and mutism. While a stuporous presentation (no movement at all) is the most common, catatonia can also present with intermittent increases in movement, often times referred to as stereotyped behaviors/movements, as they are repetitive and serve no logical purpose. A person experiencing catatonia may be able to follow simple commands in a robot-like fashion, may do the opposite of what is asked, or they may not be able to follow commands at all. A patient with catatonia may also demonstrate statue-like behavior, referred to as waxy flexibility (i.e. the provider raises the patient’s arm and the patient holds it there even once the provider has let go).

    Delusions refer to an altered thought process whereby one experiences false fixed beliefs that do not respond to reality testing. The person with delusions believes his/her thoughts to be reality. Types of delusions include persecutory or paranoid, grandiose, religious, or somatic. Delusional thinking can also refer to one’s belief that he/she can thought broadcast or insert/withdraw thoughts from other’s minds. Delusions are considered to be a positive symptom (presence of something that is not normally present) of a thought disorder, such as schizophrenia.

    Hallucinations, another positive symptom of an illness such as schizophrenia, is an alteration in sensory perception. A patient with hallucinations has a sensory experience despite lack of stimulus to create such an experience (i.e. feels a bug crawling on his/her body even though there is nothing there). Hallucinations can be experienced across all five senses. They can be tactile (feeling bodily sensations), auditory (hearing voices, noises, sounds, music), olfactory (smelling odors), visual (seeing objects or people), and gustatory (experiencing tastes).

    Tardive Dyskinesia (TD) is associated with the use of antipsychotic medications and is characterized by involuntary tonic muscular contractions of the face, limbs, or trunk. While TD is serious, it is not life threatening. Women and older adults are more at risk for developing TD. It affects up to 50% of those receiving long term antipsychotic medications. Symptoms involving the face include protruding/rolling tongue, smacking or licking of the lips, blowing type movement of the lips. Symptoms involving the limbs include pill rolling (fingers), choreic movements (rapid, purposeless, irregular) and/or athetoid movements (slow, complex, serpentine-like). Symptoms involving the trunk include neck and shoulder movements, hip movements (jerks and rocking motions).

    References

    Electroconvulsive therapy (ECT). (n.d.). Retrieved May 23, 2016, from http://www.mayoclinic.org/tests-procedures/electroconvulsive-therapy/basics/why-its-done/prc-20014161

    Herzog, E. A. & Varcarolis, E. M. (2010). Schizophrenia. In Varcarolis, E. M. & Halter, M. J. (Eds.), Foundations of Psychiatric Mental Health Nursing: A Clinical Approach (pp. 306-343). St. Louis, MO: W.B. Saunders.

  23. Question 23 of 220
    23. Question

    A 40 year old woman comes to your office for help in managing panic and anxiety symptoms. She tells you her PCP has ordered her Ativan (lorazepam) 0.5mg TID until she could see you for a psychiatric medication evaluation. Benzodiazepines target which neurotransmitter and its receptors, known to be associated with anxiety and panic symptoms?

    Correct

    Dopamine is a monoamine neurotransmitter that is naturally occurring in the brain. It is involved in fine muscle movement, including walking and talking, the integration of emotions and thoughts, decision making ability, and it stimulates the hypothalamus to release various hormones. A decrease in dopamine levels can result in conditions such as Parkinson’s disease or psychiatric conditions such as depression. An increase in dopamine levels can result in psychiatric conditions such as schizophrenia and mania.

    Gamma-aminobutyric acid (GABA), an amino acid, is another naturally occurring neurotransmitter in the brain. Its effects are known to calm nervous activity through inhibiting nerve transmission. GABA also has a possible role in pain perception, possesses anticonvulsant and muscle-relaxing properties, and may, in some instances, impair cognition as well as motor functioning. A decrease in GABA is associated with anxiety disorders, schizophrenia, mania, and Huntington’s disease. An increase in GABA is known to reduce anxiety. Benzodiazepines, including medications such as Ativan (lorazepam) and Valium (diazepam), bind to GABA receptors and enhance the effect of GABA, leading to a reduction in anxiety.

    Serotonin is yet another naturally occurring monoamine neurotransmitter in the brain. Serotonin’s effects are known to effect sleep regulation, mood regulation, hunger, pain perception, sexual behavior, aggression, and hormonal activity. A decrease in serotonin levels is associated with depression and an increase in serotonin levels is associated with anxiety. The SSRI and SNRI classes of medication are typically used to target mood and anxiety symptoms believed to be related to changes in serotonin levels.

    Glutamate, like GABA, is classified as an amino acid, and is another neurotransmitter in the brain. It is broken down in to two receptors-AMPA and NMDA. Glutamate is an excitatory neurotransmitter. The glutamate receptor AMPA is known to have an effect on learning and memory. A decrease in NMDA levels is associated with psychosis. Increased levels of NMDA can be neurotoxic if the elevated levels persist for a long period of time. Increases in NMDA are also associated with the neurodegeneration seen in Alzheimer’s. Increases in AMPA are known to improve cognitive performance.

    References

    Gutierrez, M.A. & Raynor, R. (2010). Biological Basis for Understanding Psychotropic Drugs. In Varcarolis, E. M. & Halter, M. J. (Eds.), Foundations of Psychiatric Mental Health Nursing: A Clinical Approach (pp. 45-75). St. Louis, MO: W.B. Saunders.

    THE BRAIN FROM TOP TO BOTTOM. (n.d.). Retrieved May 23, 2016, from http://thebrain.mcgill.ca/flash/d/d_04/d_04_m/d_04_m_peu/d_04_m_peu.html

    Incorrect

    Dopamine is a monoamine neurotransmitter that is naturally occurring in the brain. It is involved in fine muscle movement, including walking and talking, the integration of emotions and thoughts, decision making ability, and it stimulates the hypothalamus to release various hormones. A decrease in dopamine levels can result in conditions such as Parkinson’s disease or psychiatric conditions such as depression. An increase in dopamine levels can result in psychiatric conditions such as schizophrenia and mania.

    Gamma-aminobutyric acid (GABA), an amino acid, is another naturally occurring neurotransmitter in the brain. Its effects are known to calm nervous activity through inhibiting nerve transmission. GABA also has a possible role in pain perception, possesses anticonvulsant and muscle-relaxing properties, and may, in some instances, impair cognition as well as motor functioning. A decrease in GABA is associated with anxiety disorders, schizophrenia, mania, and Huntington’s disease. An increase in GABA is known to reduce anxiety. Benzodiazepines, including medications such as Ativan (lorazepam) and Valium (diazepam), bind to GABA receptors and enhance the effect of GABA, leading to a reduction in anxiety.

    Serotonin is yet another naturally occurring monoamine neurotransmitter in the brain. Serotonin’s effects are known to effect sleep regulation, mood regulation, hunger, pain perception, sexual behavior, aggression, and hormonal activity. A decrease in serotonin levels is associated with depression and an increase in serotonin levels is associated with anxiety. The SSRI and SNRI classes of medication are typically used to target mood and anxiety symptoms believed to be related to changes in serotonin levels.

    Glutamate, like GABA, is classified as an amino acid, and is another neurotransmitter in the brain. It is broken down in to two receptors-AMPA and NMDA. Glutamate is an excitatory neurotransmitter. The glutamate receptor AMPA is known to have an effect on learning and memory. A decrease in NMDA levels is associated with psychosis. Increased levels of NMDA can be neurotoxic if the elevated levels persist for a long period of time. Increases in NMDA are also associated with the neurodegeneration seen in Alzheimer’s. Increases in AMPA are known to improve cognitive performance.

    References

    Gutierrez, M.A. & Raynor, R. (2010). Biological Basis for Understanding Psychotropic Drugs. In Varcarolis, E. M. & Halter, M. J. (Eds.), Foundations of Psychiatric Mental Health Nursing: A Clinical Approach (pp. 45-75). St. Louis, MO: W.B. Saunders.

    THE BRAIN FROM TOP TO BOTTOM. (n.d.). Retrieved May 23, 2016, from http://thebrain.mcgill.ca/flash/d/d_04/d_04_m/d_04_m_peu/d_04_m_peu.html

  24. Question 24 of 220
    24. Question

    A 23 year old female is brought in to your emergency room after being found unconscious by a friend. Vital signs indicate that she is hypotensive and upon exam you determine she is dehydrated and emaciated and that her body weight is less than 85% of what would be expected for her height. When the patient regains consciousness, she tells you she hasn’t been eating well lately and cannot recall the last time she ate, but thinks it was 2 days ago. She goes on to say that she believes she is “too fat,” despite the fact that she is 5’5” and weighs 80 lbs, and therefore restricts her food intake to prevent weight gain. Your patient also discloses that she has not had her menses in at least 8 months. You suspect your patient has which of the following?

    Correct

    Generalized Anxiety Disorder (GAD), which lasts 6 months or more, is characterized by excessive anxiety or worry about numerous things, usually in excess of the event or situation about which the person is worried. Worries are usually related to daily life stressors, such as finances, relationships, job related concerns, etc. Symptoms include restlessness, fatigue, an inability to concentrate/focus, irritability, tension, and poor sleep.

    Anorexia Nervosa is an eating disorder that is characterized by abnormally low body weight (i.e. less than 85% of that expected for height), failure to thrive (i.e. unable to make expected weight gain during a period of growth that leads to a body weight less than 85% of that expected for height), an intense fear of gaining weight despite being grossly underweight, the denial of the seriousness of low body weight, and disturbance in perceived body weight or shape. Anorexia nervosa typically involves severely restricting the amount of food eaten to prevent weight gain or induce further weight loss. However, periods of restriction can also be interspersed with recurrent episodes of binging and purging behaviors. A person with anorexia nervosa might also exercise excessively and/or misuse diet aids and laxatives. The weight loss associated with anorexia nervosa can lead to metabolic changes as well as problems with muscle atrophy, nutritional deficiencies, and onset of amenorrhea (for at least 3 consecutive menstrual cycles) in girls/women.

    Obsessive Compulsive Disorder (OCD) is characterized by obsessions and/or compulsions. Obsessions, (thoughts, impulses, or images) are pervasive and unable to be dismissed. Typically, individuals experiencing obsessive thoughts don’t want to have the thoughts and being unable to stop them causes severe anxiety. Compulsions (the behaviors) are ritualistic in nature and individuals feel driven to perform them in an attempt to temporarily reduce or relieve the anxiety. Because the relief is only temporary, the compulsive act must be repeated again and again. Obsessions and compulsions are considered problematic when they begin to interfere with daily functions and lead to an inability to focus on anything but the obsession or compulsion.

    Bulimia Nervosa is a potentially life threatening eating disorder that is characterized by recurrent episodes of binge eating followed by recurrent inappropriate compensatory behaviors to prevent weight gain. Compensatory behaviors include things such as purging; abusing/misusing laxatives, diuretics, diet aids, or other medications; excessive exercise; or fasting. Binge eating is characterized by consuming a larger amount of food, in a particular period of time, than most people would consume in the same time period and under similar circumstances. People with bulimia nervosa often binge eat in discrete places. There is a sense of lack of control over eating and feeling that one cannot stop eating or control what they are eating.

    References

    Anorexia nervosa. (2016). Retrieved June 03, 2016, from http://www.mayoclinic.org/diseases-conditions/anorexia/home/ovc-20179508

    Bulimia nervosa. (2016). Retrieved June 03, 2016, from http://www.mayoclinic.org/diseases-conditions/bulimia/home/ovc-20179821

    Enright, C. R. & Ibrahim, K. (2010). Eating Disorders. In Varcarolis, E. M. & Halter, M.
    J. (Eds.), Foundations of Psychiatric Mental Health Nursing: A Clinical Approach (pp. 344-368). St. Louis, MO: W.B. Saunders.

    Halter, M. J., Varcarolis, E. M. & Shoemaker, N. C. (2010). Anxiety and Anxiety Disorders. In Varcarolis, E. M. & Halter, M. J. (Eds.), Foundations of Psychiatric Mental Health Nursing: A Clinical Approach (pp. 212-245). St. Louis, MO: W.B. Saunders.

    Incorrect

    Generalized Anxiety Disorder (GAD), which lasts 6 months or more, is characterized by excessive anxiety or worry about numerous things, usually in excess of the event or situation about which the person is worried. Worries are usually related to daily life stressors, such as finances, relationships, job related concerns, etc. Symptoms include restlessness, fatigue, an inability to concentrate/focus, irritability, tension, and poor sleep.

    Anorexia Nervosa is an eating disorder that is characterized by abnormally low body weight (i.e. less than 85% of that expected for height), failure to thrive (i.e. unable to make expected weight gain during a period of growth that leads to a body weight less than 85% of that expected for height), an intense fear of gaining weight despite being grossly underweight, the denial of the seriousness of low body weight, and disturbance in perceived body weight or shape. Anorexia nervosa typically involves severely restricting the amount of food eaten to prevent weight gain or induce further weight loss. However, periods of restriction can also be interspersed with recurrent episodes of binging and purging behaviors. A person with anorexia nervosa might also exercise excessively and/or misuse diet aids and laxatives. The weight loss associated with anorexia nervosa can lead to metabolic changes as well as problems with muscle atrophy, nutritional deficiencies, and onset of amenorrhea (for at least 3 consecutive menstrual cycles) in girls/women.

    Obsessive Compulsive Disorder (OCD) is characterized by obsessions and/or compulsions. Obsessions, (thoughts, impulses, or images) are pervasive and unable to be dismissed. Typically, individuals experiencing obsessive thoughts don’t want to have the thoughts and being unable to stop them causes severe anxiety. Compulsions (the behaviors) are ritualistic in nature and individuals feel driven to perform them in an attempt to temporarily reduce or relieve the anxiety. Because the relief is only temporary, the compulsive act must be repeated again and again. Obsessions and compulsions are considered problematic when they begin to interfere with daily functions and lead to an inability to focus on anything but the obsession or compulsion.

    Bulimia Nervosa is a potentially life threatening eating disorder that is characterized by recurrent episodes of binge eating followed by recurrent inappropriate compensatory behaviors to prevent weight gain. Compensatory behaviors include things such as purging; abusing/misusing laxatives, diuretics, diet aids, or other medications; excessive exercise; or fasting. Binge eating is characterized by consuming a larger amount of food, in a particular period of time, than most people would consume in the same time period and under similar circumstances. People with bulimia nervosa often binge eat in discrete places. There is a sense of lack of control over eating and feeling that one cannot stop eating or control what they are eating.

    References

    Anorexia nervosa. (2016). Retrieved June 03, 2016, from http://www.mayoclinic.org/diseases-conditions/anorexia/home/ovc-20179508

    Bulimia nervosa. (2016). Retrieved June 03, 2016, from http://www.mayoclinic.org/diseases-conditions/bulimia/home/ovc-20179821

    Enright, C. R. & Ibrahim, K. (2010). Eating Disorders. In Varcarolis, E. M. & Halter, M.
    J. (Eds.), Foundations of Psychiatric Mental Health Nursing: A Clinical Approach (pp. 344-368). St. Louis, MO: W.B. Saunders.

    Halter, M. J., Varcarolis, E. M. & Shoemaker, N. C. (2010). Anxiety and Anxiety Disorders. In Varcarolis, E. M. & Halter, M. J. (Eds.), Foundations of Psychiatric Mental Health Nursing: A Clinical Approach (pp. 212-245). St. Louis, MO: W.B. Saunders.

  25. Question 25 of 220
    25. Question

    You admit a patient, who is known to misuse alcohol, to your inpatient psychiatric unit. This patient has a history of daily alcohol consumption. He reports that for the past 2 months, he has been drinking at least 12 beers a day; his last drink was this morning prior to admission. You anticipate this patient will likely being to experience alcohol withdrawal symptoms within the next few hours. Which of the following would be an appropriate medication to use in the management of alcohol withdrawal?

    Correct

    Benadryl is an antihistamine medication that is also known to have anticholinergic properties. It is typically used to treat or prevent symptoms associated with allergies, such as sneezing, runny nose, and itchy, watery eyes. Benadryl can also be used to treat insomnias through its effect on the histamine receptors. When histamine levels are decreased, sedation can result. Benadryl, because of its anticholinergic properties, can also be used to treat stiffness and tremors associated with Parkinson’s disease as well as some of the extrapyramidal side effects (EPS) associated with the use of antipsychotics. Benadryl is not indicated for the management of alcohol withdrawal.

    Librium is a long acting benzodiazepine often times used to manage symptoms of alcohol withdrawal. It is one of the most widely used medications for the management of alcohol withdrawal, with a well-documented efficacy profile. Librium doses can be scheduled, to try and minimize withdrawal symptoms, or given as needed if symptoms are identified through the use of a detox protocol, such as the CIWA. The dosage of Librium given depends upon the severity of the withdrawal symptoms. Librium can help decrease withdrawal symptoms, stabilize vital signs (i.e. BP and pulse) and help to prevent complications such as seizures or delirium tremens (DTs). Because of its long half-life, Librium is a self-tapering medication, which helps to minimize the risk of withdrawal associated with benzodiazepines.

    Prozac is an antidepressant, and is classified as a selective serotonin reuptake inhibitor (SSRI). SSRIs are considered the first line of treatment for major depression. Prozac is indicated for the treatment of symptoms associated with depressive disorders as well as anxiety disorders. Prozac increases the activity of serotonin in the brain by blocking the reuptake of serotonin. Prozac is not indicated for the management of alcohol withdrawal.
    Seroquel is a second generation atypical antipsychotic. It is used in the treatment of both schizophrenia and bipolar disorder. The newer Seroquel XR has an additional indication as an adjunct for the treatment of depressive disorders. Seroquel is effective in treating both the positive and negative symptoms of schizophrenia. When used to treat bipolar disorder, Seroquel targets both the depressive and manic episodes, and is an effective maintenance medication as well. Seroquel is not indicated for the management of alcohol withdrawal.

    References

    Antihistamine (Oral Route, Parenteral Route, Rectal Route). (n.d.). Retrieved June 03, 2016, from http://www.mayoclinic.org/drugs-supplements/antihistamine-oral-route-parenteral-route-rectal-route/description/drg-20070373

    Councill III, J., Halter, M.J., & Smith-Dijulio, K. (2010). Addictive Disorders. In Varcarolis, E. M. & Halter, M. J. (Eds.), Foundations of Psychiatric Mental Health Nursing: A Clinical Approach (pp. 402-432). St. Louis, MO: W.B. Saunders.

    Halter, M. J. & Varcarcolis, E.M. (2010). Bipolar Disorders. In Varcarolis, E. M. & Halter, M. J. (Eds.), Foundations of Psychiatric Mental Health Nursing: A Clinical Approach (pp. 280-305). St. Louis, MO: W.B. Saunders.

    Herzog, E. A. & Varcarolis, E. M. (2010). Schizophrenia. In Varcarolis, E. M. & Halter, M. J. (Eds.), Foundations of Psychiatric Mental Health Nursing: A Clinical Approach (pp. 306-343). St. Louis, MO: W.B. Saunders.

    Kozy, M. &Varcarolis, E. M. (2010). Depressive Disorders. In Varcarolis, E. M. & Halter, M. J. (Eds.), Foundations of Psychiatric Mental Health Nursing: A Clinical Approach (pp. 246-279). St. Louis, MO: W.B. Saunders.

    Prater, C.D., Miller, K.E., & Zylstra, R. G. (1999). Outpatient Detoxification of the addicted or Alcoholic Patient. American Family Physician, 60(4):1175-1182. http://www.aafp.org/afp/1999/0915/p1175.html

    Incorrect

    Benadryl is an antihistamine medication that is also known to have anticholinergic properties. It is typically used to treat or prevent symptoms associated with allergies, such as sneezing, runny nose, and itchy, watery eyes. Benadryl can also be used to treat insomnias through its effect on the histamine receptors. When histamine levels are decreased, sedation can result. Benadryl, because of its anticholinergic properties, can also be used to treat stiffness and tremors associated with Parkinson’s disease as well as some of the extrapyramidal side effects (EPS) associated with the use of antipsychotics. Benadryl is not indicated for the management of alcohol withdrawal.

    Librium is a long acting benzodiazepine often times used to manage symptoms of alcohol withdrawal. It is one of the most widely used medications for the management of alcohol withdrawal, with a well-documented efficacy profile. Librium doses can be scheduled, to try and minimize withdrawal symptoms, or given as needed if symptoms are identified through the use of a detox protocol, such as the CIWA. The dosage of Librium given depends upon the severity of the withdrawal symptoms. Librium can help decrease withdrawal symptoms, stabilize vital signs (i.e. BP and pulse) and help to prevent complications such as seizures or delirium tremens (DTs). Because of its long half-life, Librium is a self-tapering medication, which helps to minimize the risk of withdrawal associated with benzodiazepines.

    Prozac is an antidepressant, and is classified as a selective serotonin reuptake inhibitor (SSRI). SSRIs are considered the first line of treatment for major depression. Prozac is indicated for the treatment of symptoms associated with depressive disorders as well as anxiety disorders. Prozac increases the activity of serotonin in the brain by blocking the reuptake of serotonin. Prozac is not indicated for the management of alcohol withdrawal.
    Seroquel is a second generation atypical antipsychotic. It is used in the treatment of both schizophrenia and bipolar disorder. The newer Seroquel XR has an additional indication as an adjunct for the treatment of depressive disorders. Seroquel is effective in treating both the positive and negative symptoms of schizophrenia. When used to treat bipolar disorder, Seroquel targets both the depressive and manic episodes, and is an effective maintenance medication as well. Seroquel is not indicated for the management of alcohol withdrawal.

    References

    Antihistamine (Oral Route, Parenteral Route, Rectal Route). (n.d.). Retrieved June 03, 2016, from http://www.mayoclinic.org/drugs-supplements/antihistamine-oral-route-parenteral-route-rectal-route/description/drg-20070373

    Councill III, J., Halter, M.J., & Smith-Dijulio, K. (2010). Addictive Disorders. In Varcarolis, E. M. & Halter, M. J. (Eds.), Foundations of Psychiatric Mental Health Nursing: A Clinical Approach (pp. 402-432). St. Louis, MO: W.B. Saunders.

    Halter, M. J. & Varcarcolis, E.M. (2010). Bipolar Disorders. In Varcarolis, E. M. & Halter, M. J. (Eds.), Foundations of Psychiatric Mental Health Nursing: A Clinical Approach (pp. 280-305). St. Louis, MO: W.B. Saunders.

    Herzog, E. A. & Varcarolis, E. M. (2010). Schizophrenia. In Varcarolis, E. M. & Halter, M. J. (Eds.), Foundations of Psychiatric Mental Health Nursing: A Clinical Approach (pp. 306-343). St. Louis, MO: W.B. Saunders.

    Kozy, M. &Varcarolis, E. M. (2010). Depressive Disorders. In Varcarolis, E. M. & Halter, M. J. (Eds.), Foundations of Psychiatric Mental Health Nursing: A Clinical Approach (pp. 246-279). St. Louis, MO: W.B. Saunders.

    Prater, C.D., Miller, K.E., & Zylstra, R. G. (1999). Outpatient Detoxification of the addicted or Alcoholic Patient. American Family Physician, 60(4):1175-1182. http://www.aafp.org/afp/1999/0915/p1175.html

  26. Question 26 of 220
    26. Question

    A 28 year-old male is taking clozapine for the treatment of schizophrenia. He presents to a medication follow-up appointment with you today and complains of excessive salivation since being on the clozapine. He tells you this occurs both at night while sleeping as well as during the day. Which class of medication could be given to him to help alleviate this side effect?

    Correct

    Selective Serotonin Reuptake Inhibitors (SSRIs) are a class of antidepressant medications, and include medications such as Prozac, Zoloft, and Lexapro. These medications act by blocking the reuptake of serotonin, which allows for serotonin to act at its receptor sites in the brain for extended periods of time. This in turn can lead to improvement in both mood and anxiety symptoms. SSRIs are considered the first line of treatment for depressive disorders.

    Dopamine antagonists generally refer to the atypical antipsychotics used to treat schizophrenia and bipolar disorder. These include medications such as Seroquel, Clozaril, Risperdal, and more. These medications work by blocking dopamine which in turn leads to a decrease in psychotic symptoms. However, a decrease in dopamine can lead to some untoward effects, including extrapyramidal side effects (EPS), cognitive changes, and possibly drug-induced parkinsonism, which can include symptoms such as hypersalivation and drooling. Therefore, a dopamine antagonist would not be a treatment choice to alleviate drooling as in the above scenario.

    Anticholinergic medications act to block or inhibit the action of the neurotransmitter acetylcholine, which helps to send messages from nerves to muscles. Examples of anticholinergic medications are Cogentin (benztropine mesylate) and Glycopyrrolate. Both of these medications are known to be effective in the treatment of drooling due to their effects on the parasympathetic innervation of the salivary gland. Anticholinergic agents tend to be drying, thereby improving hypersalivation and drooling. In addition to dry mouth, other side effects of anticholinergic medications include, but are not limited to, blurry vision, drowsiness, urinary retention, and memory impairment.

    Antihypertensive medications are comprised of several classes of medications, and are used to treat high blood pressure as well as other heart related conditions. Classes of antihypertensive medications include adrenergic blockers, diuretics, calcium channel blocker, and angiotensin-converting enzyme (ACE) inhibitors. All of the medications within these classes reduce blood pressure through their effect(s) on various mechanisms of action within the body. Antihypertensive medications are used in an effort to prevent the complications of high blood pressure, such as stroke and myocardial infarction.

    References

    Bavikatte, G., Sit, P.L., & Hassoon, A. (2012). Management of Drooling of saliva. British Journal of Medical Practitioners, 5(1);a507. http://www.bjmp.org/content/management-drooling-saliva

    Brashers, V.L., Haak, S.W., & Richardson, S. J. (2010). Alterations of Cardiovascular Function. In McCance, K.L. & Huether, S.E. (Eds), Pathopyhsiology: The Biologic Basis for Disease in Adults and Chidren (pp. 1024-1092). St. Louis, MO: Mosby-Year Book, Inc.

    Gutierrez, M.A. & Raynor, R. (2010). Biological Basis for Understanding Psychotropic Drugs. In Varcarolis, E. M. & Halter, M. J. (Eds.), Foundations of Psychiatric Mental Health Nursing: A Clinical Approach (pp. 45-75). St. Louis, MO: W.B. Saunders.

    Saltz, B.L., Robinson, D.G., & Woerner, M.G. (2004). Recognizing and Managing Antipsychotic Drug Treatment Side Effects in the Elderly. Prim Care Companion J Clin Psychiatry, 6(suppl 2): 14–19. http://www.ncbi.nlm.nih.gov/pmc/articles/PMC487007/

    Incorrect

    Selective Serotonin Reuptake Inhibitors (SSRIs) are a class of antidepressant medications, and include medications such as Prozac, Zoloft, and Lexapro. These medications act by blocking the reuptake of serotonin, which allows for serotonin to act at its receptor sites in the brain for extended periods of time. This in turn can lead to improvement in both mood and anxiety symptoms. SSRIs are considered the first line of treatment for depressive disorders.

    Dopamine antagonists generally refer to the atypical antipsychotics used to treat schizophrenia and bipolar disorder. These include medications such as Seroquel, Clozaril, Risperdal, and more. These medications work by blocking dopamine which in turn leads to a decrease in psychotic symptoms. However, a decrease in dopamine can lead to some untoward effects, including extrapyramidal side effects (EPS), cognitive changes, and possibly drug-induced parkinsonism, which can include symptoms such as hypersalivation and drooling. Therefore, a dopamine antagonist would not be a treatment choice to alleviate drooling as in the above scenario.

    Anticholinergic medications act to block or inhibit the action of the neurotransmitter acetylcholine, which helps to send messages from nerves to muscles. Examples of anticholinergic medications are Cogentin (benztropine mesylate) and Glycopyrrolate. Both of these medications are known to be effective in the treatment of drooling due to their effects on the parasympathetic innervation of the salivary gland. Anticholinergic agents tend to be drying, thereby improving hypersalivation and drooling. In addition to dry mouth, other side effects of anticholinergic medications include, but are not limited to, blurry vision, drowsiness, urinary retention, and memory impairment.

    Antihypertensive medications are comprised of several classes of medications, and are used to treat high blood pressure as well as other heart related conditions. Classes of antihypertensive medications include adrenergic blockers, diuretics, calcium channel blocker, and angiotensin-converting enzyme (ACE) inhibitors. All of the medications within these classes reduce blood pressure through their effect(s) on various mechanisms of action within the body. Antihypertensive medications are used in an effort to prevent the complications of high blood pressure, such as stroke and myocardial infarction.

    References

    Bavikatte, G., Sit, P.L., & Hassoon, A. (2012). Management of Drooling of saliva. British Journal of Medical Practitioners, 5(1);a507. http://www.bjmp.org/content/management-drooling-saliva

    Brashers, V.L., Haak, S.W., & Richardson, S. J. (2010). Alterations of Cardiovascular Function. In McCance, K.L. & Huether, S.E. (Eds), Pathopyhsiology: The Biologic Basis for Disease in Adults and Chidren (pp. 1024-1092). St. Louis, MO: Mosby-Year Book, Inc.

    Gutierrez, M.A. & Raynor, R. (2010). Biological Basis for Understanding Psychotropic Drugs. In Varcarolis, E. M. & Halter, M. J. (Eds.), Foundations of Psychiatric Mental Health Nursing: A Clinical Approach (pp. 45-75). St. Louis, MO: W.B. Saunders.

    Saltz, B.L., Robinson, D.G., & Woerner, M.G. (2004). Recognizing and Managing Antipsychotic Drug Treatment Side Effects in the Elderly. Prim Care Companion J Clin Psychiatry, 6(suppl 2): 14–19. http://www.ncbi.nlm.nih.gov/pmc/articles/PMC487007/

  27. Question 27 of 220
    27. Question

    Antisocial personality disorder is considered one of the cluster B personality disorders. One must be at least 18 years of age before this diagnosis can be made. It is primarily characterized by consistent and reckless disregard for safety of self and others, including lack of remorse for hurting others, failure to conform to societal norms (i.e. failure to adhere to lawful behaviors), deceitfulness, anger and aggression that lead to physical altercations, impulsivity, and irritability. Antisocial personality disorder has historically been synonymous with terms such as psychopath or sociopath. Which of the following childhood disorders is the best predictor of adult antisocial personality disorder?

    Correct

    Conduct Disorder is a disorder of childhood and adolescence, and one of the most frequently diagnosed disorders amongst these age groups. Characteristics of conduct disorder include persistent disregard for age-appropriate societal norm and rules as well as a range of antisocial types of behavior where the rights of others are violated. Conduct disorder involves ongoing emotional and behavioral problems that may lead to complications such as impulsivity, substance misuse, academic failure, school suspensions, and legal involvement. Childhood onset occurs prior to 10 years of age and is more common in males. Conduct disorder can be characterized as aggression towards people or animals, property destruction, theft/deceit, and serious violations of rules.

    Evidence of conduct disorder with onset prior to 15 years of age is one of the criteria for diagnosis of antisocial personality disorder. Conduct disorder can be characterized as aggression towards people or animals, property destruction, theft/deceit, and serious violations of rules.
    Oppositional Defiant Disorder (ODD) is another disorder of childhood and adolescence. There are some similarities to conduct disorder in that there is a recurrent pattern of negative behaviors, including angry/irritable moods, defiance, and hostility aimed at authority figures. Unlike conduct disorder, ODD does not involve aggression towards people or animals, there is no property destruction, no pattern of deceit, and no serious violations of the basic rights of others. Children with ODD tend to be stubborn, argumentative, unwilling to compromise, unable to accept responsibility/blame for poor behaviors/choices, and will repeatedly test limits that are set. Evidence of ODD is usually present before 8 years of age, and more common in males until puberty, after which the rate is equal.

    Attention Deficit Hyperactivity Disorder (ADHD) is a childhood disorder characterized by an inability to maintain an age-appropriate degree of attention, an inability to control impulses, and hyperactivity. Symptoms must be present and cause dysfunction and/or impairment in at least 2 separate settings. Additionally, onset of symptoms must occur prior to 12 years of age to make a diagnosis. Symptoms of inattention include difficulty with focus and concentration, difficulty paying attention, difficulty listening and following directions, and forgetfulness. Symptoms of impulsivity and hyperactivity include difficulty sitting still, fidgeting, unable to play quietly, difficulty with being patient and tends to be impulsive as a result (i.e. blurts out answers, trouble waiting for own turn, interrupts others, etc). While ADHD can often look similar to conduct disorder and ODD, it’s important to note that children with the latter two disorders tend to display consistent patterns of negative behavior and look for arguments regardless of the situation. Children with ADHD tend to display negative behaviors consistent with thei inability to tolerate situational and environmental stressors.

    Reactive Attachment Disorder is considered to be rare but serious in nature. A child is at risk for developing a reactive attachment disorder when as an infant or younger child, they are unable to form healthy bonds and relationships with their primary caregivers. When basic needs for comfort, stability, affection, and nurturing are not met, children are at risk for an inability to establish stable attachments to others. Reactive attachment symptoms can start as early as infancy and it is still unclear as to whether the disorder occurs in children 5 years of age and older. Symptoms include an inability to respond when comfort is given, an inability to seek comfort when it is offered, isolation and withdrawal, lack of social interaction with others, appearance of being sad, fearful, or irritable, and failure to ask for help/support when needed. Children with reactive attachment disorder tend to withdraw inside themselves and are not known to display any negative behaviors towards others.

    References

    Cihlar, C.A. & Shoemaker, N.C. (2010). Personality Disorders. In Varcarolis, E. M. & Halter, M. J. (Eds.), Foundations of Psychiatric Mental Health Nursing: A Clinical Approach (pp. 433-460). St. Louis, MO: W.B. Saunders.

    Conduct disorder: MedlinePlus Medical Encyclopedia. (n.d.). Retrieved June 12, 2016,from https://www.nlm.nih.gov/medlineplus/ency/article/000919.htm

    Erwin, E.H. & Colson, C. W. (2010). Disorders of Children and Adolescents. In Varcarolis, E. M. & Halter, M. J. (Eds.), Foundations of Psychiatric Mental Health Nursing: A Clinical Approach (pp. 626-652). St. Louis, MO: W.B. Saunders. Reactive attachment disorder. (n.d.). Retrieved June 11, 2016, from http://www.mayoclinic.org/diseases-conditions/reactive-attachment-disorder/basics/definition/con-20032126

    Incorrect

    Conduct Disorder is a disorder of childhood and adolescence, and one of the most frequently diagnosed disorders amongst these age groups. Characteristics of conduct disorder include persistent disregard for age-appropriate societal norm and rules as well as a range of antisocial types of behavior where the rights of others are violated. Conduct disorder involves ongoing emotional and behavioral problems that may lead to complications such as impulsivity, substance misuse, academic failure, school suspensions, and legal involvement. Childhood onset occurs prior to 10 years of age and is more common in males. Conduct disorder can be characterized as aggression towards people or animals, property destruction, theft/deceit, and serious violations of rules.

    Evidence of conduct disorder with onset prior to 15 years of age is one of the criteria for diagnosis of antisocial personality disorder. Conduct disorder can be characterized as aggression towards people or animals, property destruction, theft/deceit, and serious violations of rules.
    Oppositional Defiant Disorder (ODD) is another disorder of childhood and adolescence. There are some similarities to conduct disorder in that there is a recurrent pattern of negative behaviors, including angry/irritable moods, defiance, and hostility aimed at authority figures. Unlike conduct disorder, ODD does not involve aggression towards people or animals, there is no property destruction, no pattern of deceit, and no serious violations of the basic rights of others. Children with ODD tend to be stubborn, argumentative, unwilling to compromise, unable to accept responsibility/blame for poor behaviors/choices, and will repeatedly test limits that are set. Evidence of ODD is usually present before 8 years of age, and more common in males until puberty, after which the rate is equal.

    Attention Deficit Hyperactivity Disorder (ADHD) is a childhood disorder characterized by an inability to maintain an age-appropriate degree of attention, an inability to control impulses, and hyperactivity. Symptoms must be present and cause dysfunction and/or impairment in at least 2 separate settings. Additionally, onset of symptoms must occur prior to 12 years of age to make a diagnosis. Symptoms of inattention include difficulty with focus and concentration, difficulty paying attention, difficulty listening and following directions, and forgetfulness. Symptoms of impulsivity and hyperactivity include difficulty sitting still, fidgeting, unable to play quietly, difficulty with being patient and tends to be impulsive as a result (i.e. blurts out answers, trouble waiting for own turn, interrupts others, etc). While ADHD can often look similar to conduct disorder and ODD, it’s important to note that children with the latter two disorders tend to display consistent patterns of negative behavior and look for arguments regardless of the situation. Children with ADHD tend to display negative behaviors consistent with thei inability to tolerate situational and environmental stressors.

    Reactive Attachment Disorder is considered to be rare but serious in nature. A child is at risk for developing a reactive attachment disorder when as an infant or younger child, they are unable to form healthy bonds and relationships with their primary caregivers. When basic needs for comfort, stability, affection, and nurturing are not met, children are at risk for an inability to establish stable attachments to others. Reactive attachment symptoms can start as early as infancy and it is still unclear as to whether the disorder occurs in children 5 years of age and older. Symptoms include an inability to respond when comfort is given, an inability to seek comfort when it is offered, isolation and withdrawal, lack of social interaction with others, appearance of being sad, fearful, or irritable, and failure to ask for help/support when needed. Children with reactive attachment disorder tend to withdraw inside themselves and are not known to display any negative behaviors towards others.

    References

    Cihlar, C.A. & Shoemaker, N.C. (2010). Personality Disorders. In Varcarolis, E. M. & Halter, M. J. (Eds.), Foundations of Psychiatric Mental Health Nursing: A Clinical Approach (pp. 433-460). St. Louis, MO: W.B. Saunders.

    Conduct disorder: MedlinePlus Medical Encyclopedia. (n.d.). Retrieved June 12, 2016,from https://www.nlm.nih.gov/medlineplus/ency/article/000919.htm

    Erwin, E.H. & Colson, C. W. (2010). Disorders of Children and Adolescents. In Varcarolis, E. M. & Halter, M. J. (Eds.), Foundations of Psychiatric Mental Health Nursing: A Clinical Approach (pp. 626-652). St. Louis, MO: W.B. Saunders. Reactive attachment disorder. (n.d.). Retrieved June 11, 2016, from http://www.mayoclinic.org/diseases-conditions/reactive-attachment-disorder/basics/definition/con-20032126

  28. Question 28 of 220
    28. Question

    A 52 year old female is referred to you by her PCP for a psychiatric evaluation. She has presented to her PCP numerous times with complaints of headaches and is convinced that her headaches are indicative of a brain tumor. Numerous tests, including MRIs and CT scans have been done, and all are negative for any abnormal findings. She has no other symptoms to support her fears. Despite this, she continues to insist that she has a brain tumor and is dying. You suspect she is suffering from which of the following?

    Correct

    Acute Stress Disorder shares symptoms similar to Post-Traumatic Stress Disorder (PTSD), such as poor sleep, irritability, flashbacks, avoidant behaviors, etc. However, there are 3 hallmark differences between the two disorders: length of time to onset of symptoms (within one month following the traumatic event for Acute Stress Disorder), length of time to resolution of symptoms (no more than 4 weeks for Acute Stress Disorder) and presence of at least 3 dissociative symptoms either during or after the traumatic event (Acute Stress Disorder). Dissociative symptoms include emotional numbing/detachment, reduced awareness of surroundings, depersonalization (sense of unreality related to self), derealization (sense of unreality related to the environment), and loss of memory of an important aspect of the trauma.

    Malingering is a condition considered to be related to the factitious disorders. It is characterized by a conscious motivation to act for the purpose of deceiving, in order to achieve material gain. There is a conscious act to feign or exaggerate an illness or symptoms for the purpose of trying to become eligible for disability benefits, obtain prescription medications (likely narcotics), and/or avoid legal issues. Symptoms are consciously fabricated and not supported by any medical findings, and are generally vague and hard for providers to prove or disprove (i.e. stomach pain, back pain, head pain, dental pain). Malingering is thought to be more common in men than women, and is often associated with antisocial personality disorder.

    Generalized Anxiety Disorder (GAD), which lasts 6 months or more, is characterized by excessive anxiety or worry about numerous things, usually in excess of the event or situation about which the person is worried. Worries are usually related to daily life stressors, such as finances, relationships, job related concerns, etc. Symptoms include restlessness, fatigue, an inability to concentrate/focus, irritability, tension, and poor sleep.

    Hypochondriasis is known as an illness anxiety disorder, and is characterized by fears of having a serious illness or disease, or the idea that one is going to become seriously ill. These fears and/or ideas persist despite appropriate medical testing that does not support these fears or ideas. Reassurance that there are no obvious medical issues also does nothing to alleviate the fears or ideas of having a serious illness. People with hypochondriasis typically have no, or very mild, symptoms to support their belief of having a major illness. These patients seek frequent medical attention, frequently check their bodies for signs of illness, avoid doing things or being with people due to fear it will impact their health, constantly talking about their health and possible illnesses, and scour the internet for information on their perceived symptoms and possible illness/causes for such symptoms. Preoccupation with these fears or ideas typically causes significant impairment in social and/or occupational functioning and/or causes significant distress in general. In order to diagnose hypochondriasis, other delusional disorders and types of somatoform disorders must first be ruled out.

    References

    Grund, F.J. & Shoemaker, N.C. (2010). Somatoform, Factitious, and Dissociative Disorders. In Varcarolis, E. M. & Halter, M. J. (Eds.), Foundations of Psychiatric Mental Health Nursing: A Clinical Approach (pp. 502-526). St. Louis, MO: W.B. Saunders.

    Illness anxiety disorder. (n.d.). Retrieved June 12, 2016, from
    http://www.mayoclinic.org/diseases-conditions/illness-anxiety-disorder/basics/symptoms/con-20124064

    Incorrect

    Acute Stress Disorder shares symptoms similar to Post-Traumatic Stress Disorder (PTSD), such as poor sleep, irritability, flashbacks, avoidant behaviors, etc. However, there are 3 hallmark differences between the two disorders: length of time to onset of symptoms (within one month following the traumatic event for Acute Stress Disorder), length of time to resolution of symptoms (no more than 4 weeks for Acute Stress Disorder) and presence of at least 3 dissociative symptoms either during or after the traumatic event (Acute Stress Disorder). Dissociative symptoms include emotional numbing/detachment, reduced awareness of surroundings, depersonalization (sense of unreality related to self), derealization (sense of unreality related to the environment), and loss of memory of an important aspect of the trauma.

    Malingering is a condition considered to be related to the factitious disorders. It is characterized by a conscious motivation to act for the purpose of deceiving, in order to achieve material gain. There is a conscious act to feign or exaggerate an illness or symptoms for the purpose of trying to become eligible for disability benefits, obtain prescription medications (likely narcotics), and/or avoid legal issues. Symptoms are consciously fabricated and not supported by any medical findings, and are generally vague and hard for providers to prove or disprove (i.e. stomach pain, back pain, head pain, dental pain). Malingering is thought to be more common in men than women, and is often associated with antisocial personality disorder.

    Generalized Anxiety Disorder (GAD), which lasts 6 months or more, is characterized by excessive anxiety or worry about numerous things, usually in excess of the event or situation about which the person is worried. Worries are usually related to daily life stressors, such as finances, relationships, job related concerns, etc. Symptoms include restlessness, fatigue, an inability to concentrate/focus, irritability, tension, and poor sleep.

    Hypochondriasis is known as an illness anxiety disorder, and is characterized by fears of having a serious illness or disease, or the idea that one is going to become seriously ill. These fears and/or ideas persist despite appropriate medical testing that does not support these fears or ideas. Reassurance that there are no obvious medical issues also does nothing to alleviate the fears or ideas of having a serious illness. People with hypochondriasis typically have no, or very mild, symptoms to support their belief of having a major illness. These patients seek frequent medical attention, frequently check their bodies for signs of illness, avoid doing things or being with people due to fear it will impact their health, constantly talking about their health and possible illnesses, and scour the internet for information on their perceived symptoms and possible illness/causes for such symptoms. Preoccupation with these fears or ideas typically causes significant impairment in social and/or occupational functioning and/or causes significant distress in general. In order to diagnose hypochondriasis, other delusional disorders and types of somatoform disorders must first be ruled out.

    References

    Grund, F.J. & Shoemaker, N.C. (2010). Somatoform, Factitious, and Dissociative Disorders. In Varcarolis, E. M. & Halter, M. J. (Eds.), Foundations of Psychiatric Mental Health Nursing: A Clinical Approach (pp. 502-526). St. Louis, MO: W.B. Saunders.

    Illness anxiety disorder. (n.d.). Retrieved June 12, 2016, from
    http://www.mayoclinic.org/diseases-conditions/illness-anxiety-disorder/basics/symptoms/con-20124064

  29. Question 29 of 220
    29. Question

    A 21 year old unconscious male is brought in to the emergency room by his friends. Upon assessment of the patient, you notice that his pupils are pinpoint. The patient’s respirations are significantly decreased and his breathing is shallow and he is also bradycardic. The patient’s friends tell you they were “experimenting with pills to see what would happen,” and that the patient took an unknown quantity of oxycodone. The patient’s symptoms are consistent with an opiate overdose. You administer which of the following medications in an attempt to revive the patient?

    Correct

    Narcan, or naloxone, is an opiate antagonist used in the treatment of opiate (i.e. heroin, narcotic pain medication) overdose. Narcan blocks the effects of opiates and reverses the respiratory and central nervous system depression caused by the overdose. The mechanism of action is to push the opiate out of the opiate receptors in the brain, thus allowing the Narcan to occupy the opiate receptors. Length of time to onset of efficacy is generally 5 minutes with most of the Narcan being metabolized within 90 minutes. In some cases, duration of action of the Narcan may be less than that of the opiate ingested and additional doses of Narcan may be required. Narcan does not promote physical or psychological dependence and cannot be used to get high due to its mechanism of action. It is not known to have any effect, positive or negative, when given to someone who has not ingested opiates.

    Librium is a long acting benzodiazepine often times used to manage symptoms of alcohol withdrawal. It is one of the most widely used medications for the management of alcohol withdrawal, with a well-documented efficacy profile. Librium doses can be scheduled, to try and minimize withdrawal symptoms, or given as needed if symptoms are identified through the use of a detox protocol, such as the CIWA. The dosage of Librium given depends upon the severity of the withdrawal symptoms. Librium can help decrease withdrawal symptoms, stabilize vital signs (i.e. BP and pulse) and help to prevent complications such as seizures or delirium tremens (DTs). Because of its long half-life, Librium is a self-tapering medication, which helps to minimize the risk of withdrawal associated with benzodiazepines.

    Benadryl is an antihistamine medication that is also known to have anticholinergic properties. It is typically used to treat or prevent symptoms associated with allergies, such as sneezing, runny nose, and itchy, watery eyes. Benadryl can also be used to treat insomnias through its effect on the histamine receptors. When histamine levels are decreased, sedation can result. Benadryl, because of its anticholinergic properties, can also be used to treat stiffness and tremors associated with Parkinson’s disease as well as some of the extrapyramidal side effects (EPS) associated with the use of antipsychotics.

    Seroquel is a second generation atypical antipsychotic. It is used in the treatment of btoh schizophrenia and bipolar disorder. The newer Seroquel XR has an additional indication as an adjunct for the treatment of depressive disorders. Seroquel is effective in treating both the positive and negative symptoms of schizophrenia. When used to treat bipolar disorder, Seroquel targets both the depressive and manic episodes, and is an effective maintenance medication as well. Seroquel is not indicated for the management of alcohol withdrawal.

    References:
    Antihistamine (Oral Route, Parenteral Route, Rectal Route). (n.d.). Retrieved June 03,2016, from http://www.mayoclinic.org/drugs supplements/antihistamine-oral-route-parenteral-route-rectal-route/description/drg-20070373

    Councill III, J., Halter, M.J., & Smith-Dijulio, K. (2010). Addictive Disorders. In Varcarolis, E. M. & Halter, M. J. (Eds.), Foundations of Psychiatric Mental Health Nursing: A Clinical Approach (pp. 402-432). St. Louis, MO: W.B. Saunders.

    Halter, M. J. & Varcarcolis, E.M. (2010). Bipolar Disorders. In Varcarolis, E. M. & Halter, M. J. (Eds.), Foundations of Psychiatric Mental Health Nursing: A Clinical Approach (pp. 280-305). St. Louis, MO: W.B. Saunders.

    Naloxone – Medical Countermeasures Database. (n.d.). Retrieved June 10, 2016, from https://chemm.nlm.nih.gov/countermeasure_naloxone.htm

    Prater, C.D., Miller, K.E., & Zylstra, R. G. (1999). Outpatient Detoxification of the addicted or Alcoholic Patient. American Family Physician, 60(4):1175-1182. http://www.aafp.org/afp/1999/0915/p1175.html

    Incorrect

    Narcan, or naloxone, is an opiate antagonist used in the treatment of opiate (i.e. heroin, narcotic pain medication) overdose. Narcan blocks the effects of opiates and reverses the respiratory and central nervous system depression caused by the overdose. The mechanism of action is to push the opiate out of the opiate receptors in the brain, thus allowing the Narcan to occupy the opiate receptors. Length of time to onset of efficacy is generally 5 minutes with most of the Narcan being metabolized within 90 minutes. In some cases, duration of action of the Narcan may be less than that of the opiate ingested and additional doses of Narcan may be required. Narcan does not promote physical or psychological dependence and cannot be used to get high due to its mechanism of action. It is not known to have any effect, positive or negative, when given to someone who has not ingested opiates.

    Librium is a long acting benzodiazepine often times used to manage symptoms of alcohol withdrawal. It is one of the most widely used medications for the management of alcohol withdrawal, with a well-documented efficacy profile. Librium doses can be scheduled, to try and minimize withdrawal symptoms, or given as needed if symptoms are identified through the use of a detox protocol, such as the CIWA. The dosage of Librium given depends upon the severity of the withdrawal symptoms. Librium can help decrease withdrawal symptoms, stabilize vital signs (i.e. BP and pulse) and help to prevent complications such as seizures or delirium tremens (DTs). Because of its long half-life, Librium is a self-tapering medication, which helps to minimize the risk of withdrawal associated with benzodiazepines.

    Benadryl is an antihistamine medication that is also known to have anticholinergic properties. It is typically used to treat or prevent symptoms associated with allergies, such as sneezing, runny nose, and itchy, watery eyes. Benadryl can also be used to treat insomnias through its effect on the histamine receptors. When histamine levels are decreased, sedation can result. Benadryl, because of its anticholinergic properties, can also be used to treat stiffness and tremors associated with Parkinson’s disease as well as some of the extrapyramidal side effects (EPS) associated with the use of antipsychotics.

    Seroquel is a second generation atypical antipsychotic. It is used in the treatment of btoh schizophrenia and bipolar disorder. The newer Seroquel XR has an additional indication as an adjunct for the treatment of depressive disorders. Seroquel is effective in treating both the positive and negative symptoms of schizophrenia. When used to treat bipolar disorder, Seroquel targets both the depressive and manic episodes, and is an effective maintenance medication as well. Seroquel is not indicated for the management of alcohol withdrawal.

    References:
    Antihistamine (Oral Route, Parenteral Route, Rectal Route). (n.d.). Retrieved June 03,2016, from http://www.mayoclinic.org/drugs supplements/antihistamine-oral-route-parenteral-route-rectal-route/description/drg-20070373

    Councill III, J., Halter, M.J., & Smith-Dijulio, K. (2010). Addictive Disorders. In Varcarolis, E. M. & Halter, M. J. (Eds.), Foundations of Psychiatric Mental Health Nursing: A Clinical Approach (pp. 402-432). St. Louis, MO: W.B. Saunders.

    Halter, M. J. & Varcarcolis, E.M. (2010). Bipolar Disorders. In Varcarolis, E. M. & Halter, M. J. (Eds.), Foundations of Psychiatric Mental Health Nursing: A Clinical Approach (pp. 280-305). St. Louis, MO: W.B. Saunders.

    Naloxone – Medical Countermeasures Database. (n.d.). Retrieved June 10, 2016, from https://chemm.nlm.nih.gov/countermeasure_naloxone.htm

    Prater, C.D., Miller, K.E., & Zylstra, R. G. (1999). Outpatient Detoxification of the addicted or Alcoholic Patient. American Family Physician, 60(4):1175-1182. http://www.aafp.org/afp/1999/0915/p1175.html

  30. Question 30 of 220
    30. Question

    A 19 year old female patient is admitted to your inpatient psychiatric unit. She presents as well groomed, neatly dressed, appears to be healthy, and is articulate in conversation. The patient shares with you a long standing history of trouble with food. When you ask her to elaborate, she tells you that she does not eat at all during the day, and when she gets home in the evening, she binges on large portions of various types of food over the first hour at home. The patient then feels full to the point of feeling ill, experiences guilt over what she has done, and induces vomiting. However, she then binge eats again. This pattern continues for several hours over the evening, with induced vomiting up to 10-15 times. The patient tells you she does this at least one day every week, and some weeks these behaviors occur 2 to 3 days out of the week. You suspect your patient has which of the following?

    Correct

    Anorexia Nervosa is an eating disorder that is characterized by abnormally low body weight (i.e. less than 85% of that expected for height), failure to thrive (i.e. unable to make expected weight gain during a period of growth that leads to a body weight less than 85% of that expected for height), an intense fear of gaining weight despite being grossly underweight, the denial of the seriousness of low body weight, and disturbance in perceived body weight or shape. Anorexia nervosa typically involves severely restricting the amount of food eaten to prevent weight gain or induce further weight loss. However, periods of restriction can also be interspersed with recurrent episodes of binging and purging behaviors. A person with anorexia nervosa might also exercise excessively and/or misuse diet aids and laxatives. The weight loss associated with anorexia nervosa can lead to metabolic changes as well as problems with muscle atrophy, nutritional deficiencies, and onset of amenorrhea (for at least 3 consecutive menstrual cycles) in girls/women.

    Bulimia Nervosa is a potentially life threatening eating disorder that is characterized by recurrent episodes of binge eating followed by recurrent inappropriate compensatory behaviors to prevent weight gain. Compensatory behaviors include things such as purging; abusing/misusing laxatives, diuretics, diet aids, or other medications; excessive exercise; or fasting. Binge eating is characterized by consuming a larger amount of food, in a particular period of time, than most people would consume in the same time period and under similar circumstances. People with bulimia nervosa often binge eat in discrete places. There is a sense of lack of control over eating and feeling that one cannot stop eating or control what they are eating.
    Borderline Personality Disorder is probably the most well-known of the personality disorders. It can be easily confused with bipolar disorder, due to the mood instability and impulsivity. While there is typically marked emotional dysregulation associated with borderline personality disorder, these symptoms are intermittent and typically only last from a few hours to no more than a few days. Other symptoms associated with borderline personality disorder include intense and/or unstable interpersonal relationships (tend to see either the positive (idealization) or negative (devaluation) in others, but not both), ineffective coping and self-soothing skills such as cutting or engaging in reckless behaviors (i.e. promiscuity, substance use, etc), negative self-image, feelings of emptiness and abandonment (real or perceived), paranoid ideation that is transient in nature and stress/anxiety induced, and recurrent suicidal/parasuicidal thoughts, behaviors, and gestures. While there are no medications indicated for the treatment of borderline personality disorder per se, medication therapy is typically aimed at targeting symptoms, such as anxiety, depression, anger, etc. Effective therapy modalities include individual therapy, cognitive behavioral therapy, dialectical behavior therapy, and group therapy.

    Dysthymic Disorder is a depressive disorder in which a depressed mood occurs for most of the day, more days than not, over a 2 year period. Dysthymia is chronic in nature and therefore difficult to distinguish the depressed mood from usual pattern of mood and functioning. Symptoms include changes in appetite, changes in sleep patterns, decreased energy and/or chronic fatigue, trouble with focus and concentration, difficulty with making decisions, feelings of hopelessness, and a decrease in self-esteem. People with dysthymia typically demonstrate significant distress in various aspects of functioning, including social and occupational. However, symptoms are not usually severe enough that hospitalization is warranted, such as in a major depressive disorder.

    References

    Cihlar, C.A. & Shoemaker, N.C. (2010). Personality Disorders. In Varcarolis, E. M. & Halter, M. J. (Eds.), Foundations of Psychiatric Mental Health Nursing: A Clinical Approach (pp. 433-460). St. Louis, MO: W.B. Saunders.

    Enright, C. R. & Ibrahim, K. (2010). Eating Disorders. In Varcarolis, E. M. & Halter, M.J. (Eds.), Foundations of Psychiatric Mental Health Nursing: A Clinical Approach (pp. 344-368). St. Louis, MO: W.B. Saunders.

    Kozy, M. &Varcarolis, E. M. (2010). Depressive Disorders. In Varcarolis, E. M. & Halter, M. J. (Eds.), Foundations of Psychiatric Mental Health Nursing: A Clinical Approach (pp. 246-279). St. Louis, MO: W.B. Saunders.

    Incorrect

    Anorexia Nervosa is an eating disorder that is characterized by abnormally low body weight (i.e. less than 85% of that expected for height), failure to thrive (i.e. unable to make expected weight gain during a period of growth that leads to a body weight less than 85% of that expected for height), an intense fear of gaining weight despite being grossly underweight, the denial of the seriousness of low body weight, and disturbance in perceived body weight or shape. Anorexia nervosa typically involves severely restricting the amount of food eaten to prevent weight gain or induce further weight loss. However, periods of restriction can also be interspersed with recurrent episodes of binging and purging behaviors. A person with anorexia nervosa might also exercise excessively and/or misuse diet aids and laxatives. The weight loss associated with anorexia nervosa can lead to metabolic changes as well as problems with muscle atrophy, nutritional deficiencies, and onset of amenorrhea (for at least 3 consecutive menstrual cycles) in girls/women.

    Bulimia Nervosa is a potentially life threatening eating disorder that is characterized by recurrent episodes of binge eating followed by recurrent inappropriate compensatory behaviors to prevent weight gain. Compensatory behaviors include things such as purging; abusing/misusing laxatives, diuretics, diet aids, or other medications; excessive exercise; or fasting. Binge eating is characterized by consuming a larger amount of food, in a particular period of time, than most people would consume in the same time period and under similar circumstances. People with bulimia nervosa often binge eat in discrete places. There is a sense of lack of control over eating and feeling that one cannot stop eating or control what they are eating.
    Borderline Personality Disorder is probably the most well-known of the personality disorders. It can be easily confused with bipolar disorder, due to the mood instability and impulsivity. While there is typically marked emotional dysregulation associated with borderline personality disorder, these symptoms are intermittent and typically only last from a few hours to no more than a few days. Other symptoms associated with borderline personality disorder include intense and/or unstable interpersonal relationships (tend to see either the positive (idealization) or negative (devaluation) in others, but not both), ineffective coping and self-soothing skills such as cutting or engaging in reckless behaviors (i.e. promiscuity, substance use, etc), negative self-image, feelings of emptiness and abandonment (real or perceived), paranoid ideation that is transient in nature and stress/anxiety induced, and recurrent suicidal/parasuicidal thoughts, behaviors, and gestures. While there are no medications indicated for the treatment of borderline personality disorder per se, medication therapy is typically aimed at targeting symptoms, such as anxiety, depression, anger, etc. Effective therapy modalities include individual therapy, cognitive behavioral therapy, dialectical behavior therapy, and group therapy.

    Dysthymic Disorder is a depressive disorder in which a depressed mood occurs for most of the day, more days than not, over a 2 year period. Dysthymia is chronic in nature and therefore difficult to distinguish the depressed mood from usual pattern of mood and functioning. Symptoms include changes in appetite, changes in sleep patterns, decreased energy and/or chronic fatigue, trouble with focus and concentration, difficulty with making decisions, feelings of hopelessness, and a decrease in self-esteem. People with dysthymia typically demonstrate significant distress in various aspects of functioning, including social and occupational. However, symptoms are not usually severe enough that hospitalization is warranted, such as in a major depressive disorder.

    References

    Cihlar, C.A. & Shoemaker, N.C. (2010). Personality Disorders. In Varcarolis, E. M. & Halter, M. J. (Eds.), Foundations of Psychiatric Mental Health Nursing: A Clinical Approach (pp. 433-460). St. Louis, MO: W.B. Saunders.

    Enright, C. R. & Ibrahim, K. (2010). Eating Disorders. In Varcarolis, E. M. & Halter, M.J. (Eds.), Foundations of Psychiatric Mental Health Nursing: A Clinical Approach (pp. 344-368). St. Louis, MO: W.B. Saunders.

    Kozy, M. &Varcarolis, E. M. (2010). Depressive Disorders. In Varcarolis, E. M. & Halter, M. J. (Eds.), Foundations of Psychiatric Mental Health Nursing: A Clinical Approach (pp. 246-279). St. Louis, MO: W.B. Saunders.

  31. Question 31 of 220
    31. Question

    An 88-year-old nursing home patient with a past medical history of dementia presents to the emergency room with a three day history of cough, fever, and shortness of breath. His initial vital signs reveal a fever of 38.7o C and a blood pressure of 85/60. He is tachypneic with a respiratory rate of 26. His chest x-ray shows a right middle lobe infiltrate. What is the most appropriate treatment?

    Correct

    The correct answer is B. The chest x-ray shows a pneumonic process in the right middle lobe. Because the patient has dementia, one should suspect aspiration to be the cause. Dementia patients will often have a progressively worsening dysphagia, and are unable to adequately protect their airways. Aspiration often occurs on the right side. The right mainstem bronchus is large and oriented in a vertical position, allowing gravity to aid in the passage of foreign materials (food, secretions) into this region. Clindamycin is the best answer because it is in IV form and covers anaerobic bacteria. It is the least expensive drug and has a low rate of post-treatment infection with MRSA. Although Streptococcus Pneumoniae and Staphylococcus Aureus remain the most common causes of bacterial pneumonia, potential anaerobes implicated here include Fusobacterium and Peptostreptococcus. Flagyl is an inappropriate answer when used as a single agent, because it has a high failure rate against aerobic streptococci and microaerophilic isolates. Augmentin is incorrect because the patient is septic and requires hospitalization. In this instance an IV medication, as opposed to something by mouth, is far more appropriate. Zithromax is an outpatient regimen that covers atypical pneumonia and is incorrect. Thoracentesis would not be a first line treatment for aspiration pneumonia and is also incorrect.

    References

    1. Sanders CV, Hanna BJ, Lewis AC. Metronidazole in the treatment of anaerobic infections. Am Rev Respir Dis 1979; 120:337.

    2. Kadowaki M, Demura Y, Mizuno S, et al. Reappraisal of Clindamycin IV monotherapy for treatment of mild-to-moderate aspiration pneumonia in elderly patients. Chest 2005; 127:1276.

    Incorrect

    The correct answer is B. The chest x-ray shows a pneumonic process in the right middle lobe. Because the patient has dementia, one should suspect aspiration to be the cause. Dementia patients will often have a progressively worsening dysphagia, and are unable to adequately protect their airways. Aspiration often occurs on the right side. The right mainstem bronchus is large and oriented in a vertical position, allowing gravity to aid in the passage of foreign materials (food, secretions) into this region. Clindamycin is the best answer because it is in IV form and covers anaerobic bacteria. It is the least expensive drug and has a low rate of post-treatment infection with MRSA. Although Streptococcus Pneumoniae and Staphylococcus Aureus remain the most common causes of bacterial pneumonia, potential anaerobes implicated here include Fusobacterium and Peptostreptococcus. Flagyl is an inappropriate answer when used as a single agent, because it has a high failure rate against aerobic streptococci and microaerophilic isolates. Augmentin is incorrect because the patient is septic and requires hospitalization. In this instance an IV medication, as opposed to something by mouth, is far more appropriate. Zithromax is an outpatient regimen that covers atypical pneumonia and is incorrect. Thoracentesis would not be a first line treatment for aspiration pneumonia and is also incorrect.

    References

    1. Sanders CV, Hanna BJ, Lewis AC. Metronidazole in the treatment of anaerobic infections. Am Rev Respir Dis 1979; 120:337.

    2. Kadowaki M, Demura Y, Mizuno S, et al. Reappraisal of Clindamycin IV monotherapy for treatment of mild-to-moderate aspiration pneumonia in elderly patients. Chest 2005; 127:1276.

  32. Question 32 of 220
    32. Question

    A 56-year-old male presents to your office for a routine visit. His past medical history consists of obesity and type II diabetes mellitus. He was started on Metformin the last time you saw him, and is tolerating this well. That visit was well over six months ago, despite the fact that he has been urged to follow up more frequently. He has previously been counseled on diet and exercise. Today the nurse has obtained a blood pressure reading of 152/82. Of the following, which medication would be the most appropriate to initiate first?

    Correct

    The correct answer is D. The American Diabetes Association recommends initial treatment of systolic blood pressure readings in the 120-139mmHg range with diet and exercise. This is also true of diastolic readings in the 80-89mmHg range. In the case described above, the patient has already been counseled on diet and exercise, yet his blood pressure remains high. Although it is unlikely good blood pressure control will be achieved with a singular agent, an angiotensin converting enzyme inhibitor (ACE) is an excellent medication to start first. They are generally well tolerated. A dry cough is a common side effect, for which an angiotensin receptor blocker (ARB) can be substituted. ACE inhibitors protect against diabetic nephropathy. There is also some evidence to suggest that these drugs lower plasma glucose by increasing responsiveness to insulin. Norvasc is incorrect. Although used in the treatment of hypertension and diabetes, it is not a first line agent. It is often used in combination with an ACE inhibitor though. Lasix is incorrect. It is a diuretic and would be considered in a patient without diabetes as first line management. It can dehydrate patients and worsen renal failure if already present. Metoprolol is often used in diabetics, but it also is not a first line agent. This medication has been known to mask symptoms of hypoglycemia and may worsen peripheral arterial disease. Imdur is a drug often used to medically manage angina and coronary artery disease. It is typically initiated in diabetics but after patients have already developed complications of their disease.

    References

    1. Messerli FH, Bangalore S, Julius S. Risk/benefit assessment of beta-blockers and diuretics precludes their use for first-line therapy in hypertension. Circulation 2008; 117:2706.

    2. Alkharouf J, Nalinikumari K, Corry D, Tuck M. Long-term effects of the angiotensin converting enzyme inhibitor captopril on metabolic control in non-insulin-dependent diabetes mellitus. Am J Hypertens 1993; 6:337.

    Incorrect

    The correct answer is D. The American Diabetes Association recommends initial treatment of systolic blood pressure readings in the 120-139mmHg range with diet and exercise. This is also true of diastolic readings in the 80-89mmHg range. In the case described above, the patient has already been counseled on diet and exercise, yet his blood pressure remains high. Although it is unlikely good blood pressure control will be achieved with a singular agent, an angiotensin converting enzyme inhibitor (ACE) is an excellent medication to start first. They are generally well tolerated. A dry cough is a common side effect, for which an angiotensin receptor blocker (ARB) can be substituted. ACE inhibitors protect against diabetic nephropathy. There is also some evidence to suggest that these drugs lower plasma glucose by increasing responsiveness to insulin. Norvasc is incorrect. Although used in the treatment of hypertension and diabetes, it is not a first line agent. It is often used in combination with an ACE inhibitor though. Lasix is incorrect. It is a diuretic and would be considered in a patient without diabetes as first line management. It can dehydrate patients and worsen renal failure if already present. Metoprolol is often used in diabetics, but it also is not a first line agent. This medication has been known to mask symptoms of hypoglycemia and may worsen peripheral arterial disease. Imdur is a drug often used to medically manage angina and coronary artery disease. It is typically initiated in diabetics but after patients have already developed complications of their disease.

    References

    1. Messerli FH, Bangalore S, Julius S. Risk/benefit assessment of beta-blockers and diuretics precludes their use for first-line therapy in hypertension. Circulation 2008; 117:2706.

    2. Alkharouf J, Nalinikumari K, Corry D, Tuck M. Long-term effects of the angiotensin converting enzyme inhibitor captopril on metabolic control in non-insulin-dependent diabetes mellitus. Am J Hypertens 1993; 6:337.

  33. Question 33 of 220
    33. Question

    A 76-year-old female with a past medical history of hypertension and diabetes presents to your clinic with complaints of feeling weak and fatigued. She comes in today because she is unable to perform some of the more physically demanding ADLs that she usually is independent with. She states that for the past 3-4 days her heart will randomly start to “beat very quickly and abnormally” with no identifiable trigger. She otherwise denies chest pain, shortness of breath, dizziness, or any other associated symptoms. You order an EKG and notice there are no discernible P waves and the QRS complexes are present in an irregularly, irregular pattern. Her rate is 115 and her blood pressure is 152/78. You start a Cardizem drip to control her heart rate. What is the most appropriate next step in the management of this patient?

    Correct

    The correct answer is D. Atrial fibrillation is a supraventricular tachyarrhythmia. It is characterized by uncoordinated atrial electrical activity resulting in an irregularly irregular heart rate. Atrial fibrillation is a relatively common cardiac condition and its incidence increases with age. Men are more often affected than women. An EKG with atrial fibrillation is without P waves as described above. The three cornerstones of treating atrial fibrillation are rate control, rhythm control, and anticoagulation to prevent thromboembolism. Patients with atrial fibrillation should be placed on long-term anticoagulation therapy to maintain an INR of 2 to 3, according to their CHADS2 VASc score. Patients with a CHADS2 score of greater than or equal to 2 should receive anticoagulation unless contraindicated. Those with a score of 1 should be treated with anticoagulation or aspirin. Those patients with a score of 0 can receive aspirin or no treatment at all. Other agents such as Pradaxa (Direct Thrombin Inhibitor), Eliquis and Xarelto (Factor Xa Inhibitors) can also be used for anticoagulation in place of Warfarin.

    All of the above are viable treatment options for atrial fibrillation, but the only appropriate answer in this vignette is answer D. The question implies that the patient has most likely been in atrial fibrillation for 3-4 days. For those patients who are hemodynamically stable and in atrial fibrillation for greater than 48 hours, the treatment protocol is to anticoagulate for a period of approximately 4 weeks before proceeding with electrical cardioversion. Blood can sometimes pool in the chambers of the heart as it fibrillates. This can cause clot formation. The clots can become dislodged and cause an embolic stroke. Answer A puts the patient at significant risk for an embolic stroke because anticoagulation has not been initiated. Answer B is not the best choice either. Although it is an option for rhythm control, it is not as safe and efficacious as option D. Rate control with a calcium channel blocker, followed by beta blocker therapy, is widely accepted. Initiation of an anti-arrhythmic such as Amiodarone is done if the rate is refractory to those initial measures. Amiodarone requires hospital monitoring for loading doses to be administered. It is also known for its pulmonary and hepatic toxicity, and is not the best choice. Answer C is a good option for refractory atrial fibrillation but is not a first line treatment. Answer E is incorrect because the patient requires inpatient monitoring of her heart rate and hemodynamic stability.

    References

    1. Domino FJ, Baldor RA, Golding J, Stephens MB, Maloney ME, Ceci L. The 5-Minute Clinical Consult Premium 2016. 24th edition. Philadelphia, PA: Wolters Kluwer; 2015 Papadakis MA, McPhee SJ, Rabow MW. Current Medical Diagnosis & Treatment 2015. New York, NY: McGraw Hill Education; 2014

    2. Kasper D, Fauci A, Hauser S, Longo D, Jameson JL. Harrison’s Principles of Internal Medicine. 19th Edition. New York: McGraw-Hill Education; 2015.

    3. What Is Atrial Fibrillation? – NHLBI, NIH 2014. Available at: http://www.nhlbi.nih.gov/health/health-topics/topics/af. Accessed February 9, 2016.

    Incorrect

    The correct answer is D. Atrial fibrillation is a supraventricular tachyarrhythmia. It is characterized by uncoordinated atrial electrical activity resulting in an irregularly irregular heart rate. Atrial fibrillation is a relatively common cardiac condition and its incidence increases with age. Men are more often affected than women. An EKG with atrial fibrillation is without P waves as described above. The three cornerstones of treating atrial fibrillation are rate control, rhythm control, and anticoagulation to prevent thromboembolism. Patients with atrial fibrillation should be placed on long-term anticoagulation therapy to maintain an INR of 2 to 3, according to their CHADS2 VASc score. Patients with a CHADS2 score of greater than or equal to 2 should receive anticoagulation unless contraindicated. Those with a score of 1 should be treated with anticoagulation or aspirin. Those patients with a score of 0 can receive aspirin or no treatment at all. Other agents such as Pradaxa (Direct Thrombin Inhibitor), Eliquis and Xarelto (Factor Xa Inhibitors) can also be used for anticoagulation in place of Warfarin.

    All of the above are viable treatment options for atrial fibrillation, but the only appropriate answer in this vignette is answer D. The question implies that the patient has most likely been in atrial fibrillation for 3-4 days. For those patients who are hemodynamically stable and in atrial fibrillation for greater than 48 hours, the treatment protocol is to anticoagulate for a period of approximately 4 weeks before proceeding with electrical cardioversion. Blood can sometimes pool in the chambers of the heart as it fibrillates. This can cause clot formation. The clots can become dislodged and cause an embolic stroke. Answer A puts the patient at significant risk for an embolic stroke because anticoagulation has not been initiated. Answer B is not the best choice either. Although it is an option for rhythm control, it is not as safe and efficacious as option D. Rate control with a calcium channel blocker, followed by beta blocker therapy, is widely accepted. Initiation of an anti-arrhythmic such as Amiodarone is done if the rate is refractory to those initial measures. Amiodarone requires hospital monitoring for loading doses to be administered. It is also known for its pulmonary and hepatic toxicity, and is not the best choice. Answer C is a good option for refractory atrial fibrillation but is not a first line treatment. Answer E is incorrect because the patient requires inpatient monitoring of her heart rate and hemodynamic stability.

    References

    1. Domino FJ, Baldor RA, Golding J, Stephens MB, Maloney ME, Ceci L. The 5-Minute Clinical Consult Premium 2016. 24th edition. Philadelphia, PA: Wolters Kluwer; 2015 Papadakis MA, McPhee SJ, Rabow MW. Current Medical Diagnosis & Treatment 2015. New York, NY: McGraw Hill Education; 2014

  34. Question 34 of 220
    34. Question

    Mr. Jones is a 68-year-old male who presents to the Emergency Room after a syncopal episode. Earlier today, he describes walking uphill and experiencing exertional dyspnea. The next thing he remembers is being attended to by EMTs in the back of an ambulance. He thinks he may have passed out. He did not strike his head or sustain any injuries. He admits that he has not followed with a primary care provider in “years” and is not one to rush to the doctor. When pressed, he tells you he has had some chest discomfort from time to time but doesn’t believe there is anything that can be done about it. He is not having chest pain currently. He adds that he has intermittent swelling of his ankles and a wet sounding cough that comes and goes. On exam you hear a systolic ejection murmur at the left sternal border. His lungs are clear at this time. Neurologically he is intact. You order an echocardiogram as part of his work up and it reveals severe aortic stenosis. What is the best treatment option for this patient’s current condition?

    Correct

    The correct answer is E. The patient has severe aortic stenosis and is symptomatic. He describes having intermittent chest pain and exertional dyspnea. Additionally, he describes symptoms consistent with congestive heart failure (i.e. swollen ankles and wet cough). Worse yet, he syncopized with exertion. In the context of aortic stenosis, syncope is thought to occur because there is systemic vasodilation and a fixed cardiac output that ultimately drops the patient’s systolic blood pressure. Sublingual nitroglycerin is contraindicated because the patient is not having active chest pain and nitrates would worsen the systemic vasodilation already present. Lasix is a good treatment option for the management of congestive heart failure, but the patient’s lungs are clear, and he is not having an active exacerbation at this time. A cardiac catheterization is more diagnostic than therapeutic in this instance. Although the patient could have concomitant coronary disease, the aortic valve is the problem here. Patients with severe aortic stenosis have a high risk of sudden death. The mortality rate is as high as 50% in a 3-year period if the valve is not replaced. It is imperative that these patients receive prompt referral to cardiothoracic surgery to determine if they are good surgical candidates. For sicker patients and in instances where surgery might be prohibited, TAVR (transcutaneous aortic valve replacement) can be pursued instead.

    References

    Nishimura RA, Otto CM, Bonow RO, et al. 2014 AHA/ACC guideline for the management of patients with valvular heart disease: a report of the American College of Cardiology/American Heart Association Task Force on Practice Guidelines. J Am Coll Cardiol 2014; 63:e57.

    Tintinalli JE, Kelen GD, Stapczynski JS, eds. Valvular emergencies. 6th ed. Emergency Medicine: A Comprehensive Study Guide. New York: McGraw-Hill; 2004. 54.

    Vahanian A, Alfieri O, Andreotti F, Antunes MJ, Barón-Esquivias G, Baumgartner H, et al. Guidelines on the management of valvular heart disease (version 2012): The Joint Task Force on the Management of Valvular Heart Disease of the European Society of Cardiology (ESC) and the European Association for Cardio-Thoracic Surgery (EACTS). Eur Heart J. 2012 Oct. 33(19):2451-96. [Medline].

    Blase A. Carabello. (2002). “Evaluation and management of patients with aortic stenosis.” Circulation.

    Incorrect

    The correct answer is E. The patient has severe aortic stenosis and is symptomatic. He describes having intermittent chest pain and exertional dyspnea. Additionally, he describes symptoms consistent with congestive heart failure (i.e. swollen ankles and wet cough). Worse yet, he syncopized with exertion. In the context of aortic stenosis, syncope is thought to occur because there is systemic vasodilation and a fixed cardiac output that ultimately drops the patient’s systolic blood pressure. Sublingual nitroglycerin is contraindicated because the patient is not having active chest pain and nitrates would worsen the systemic vasodilation already present. Lasix is a good treatment option for the management of congestive heart failure, but the patient’s lungs are clear, and he is not having an active exacerbation at this time. A cardiac catheterization is more diagnostic than therapeutic in this instance. Although the patient could have concomitant coronary disease, the aortic valve is the problem here. Patients with severe aortic stenosis have a high risk of sudden death. The mortality rate is as high as 50% in a 3-year period if the valve is not replaced. It is imperative that these patients receive prompt referral to cardiothoracic surgery to determine if they are good surgical candidates. For sicker patients and in instances where surgery might be prohibited, TAVR (transcutaneous aortic valve replacement) can be pursued instead.

    References

    Nishimura RA, Otto CM, Bonow RO, et al. 2014 AHA/ACC guideline for the management of patients with valvular heart disease: a report of the American College of Cardiology/American Heart Association Task Force on Practice Guidelines. J Am Coll Cardiol 2014; 63:e57.

    Tintinalli JE, Kelen GD, Stapczynski JS, eds. Valvular emergencies. 6th ed. Emergency Medicine: A Comprehensive Study Guide. New York: McGraw-Hill; 2004. 54.

    Vahanian A, Alfieri O, Andreotti F, Antunes MJ, Barón-Esquivias G, Baumgartner H, et al. Guidelines on the management of valvular heart disease (version 2012): The Joint Task Force on the Management of Valvular Heart Disease of the European Society of Cardiology (ESC) and the European Association for Cardio-Thoracic Surgery (EACTS). Eur Heart J. 2012 Oct. 33(19):2451-96. [Medline].

    Blase A. Carabello. (2002). “Evaluation and management of patients with aortic stenosis.” Circulation.

  35. Question 35 of 220
    35. Question

    A 40-year-old male presents to your neurology office complaining of severe headaches that have been plaguing him for a little over a month. He has had no URI symptoms, specifically no fever, cough, or congestion. He states that the headaches are one-sided and are associated with tearing, and what he feels is swelling of the eye. The headaches seem to occur at the same time each day and he is growing depressed having to cope with their symptoms. An MRI of the brain was performed and noted to be negative. Which of the following medications should be initiated as a first line agent?

    Correct

    The correct answer is A. Sumatriptan is widely accepted as a first line agent for cluster headaches. Cluster headaches commonly affect males and their onset occurs in the late third and early fourth decade of life. They tend to occur at the same time each day and last anywhere from 15-20 minutes on average. Longer headaches can go on for an hour and a half or more. Patients will describe the headache as a severe stabbing pain behind the eye, sometimes accompanied by tearing, nasal congestion, and/or visible swelling in the region. Neurologists will often obtain MRI with/without contrast and/or a head CT to evaluate the pituitary gland, rule out tumor, and to assess for possible AVM. Octreotide is used in the treatment of cluster headaches but is felt to be inferior to Sumatriptan in terms of its overall response time. Lidocaine is a topical treatment that is inexpensive and somewhat effective, but not as helpful as Sumatriptan. The patient is depressed because of his headaches. It should be noted that many patients with cluster headaches voice wanting to pursue suicide due to the severity of their symptoms. They should be screened for suicidal ideation and treated aggressively. Prozac is indicated for the treatment of depression, but will not abort the headaches. Augmentin is incorrect because it treats acute sinusitis. Although the tearing, swelling, and nasal congestion can occur with cluster headaches, the pain from acute sinusitis is likely to be more constant and associated with fever or general malaise.

    References

    PubMed. Triptans for acute cluster headache. AU. Law S, Derry S, Moore RA, Cochrane Database Syst Rev. 2010.

    Treatment of acute cluster headache with sumatriptan. The Sumatriptan Cluster Headache Study Group. N Engl J Med. 1991;325(5):322.

    Cluster headache: pathogenesis, diagnosis, and management. May A. Lancet. 2005;366(9488):843.

    Incorrect

    The correct answer is A. Sumatriptan is widely accepted as a first line agent for cluster headaches. Cluster headaches commonly affect males and their onset occurs in the late third and early fourth decade of life. They tend to occur at the same time each day and last anywhere from 15-20 minutes on average. Longer headaches can go on for an hour and a half or more. Patients will describe the headache as a severe stabbing pain behind the eye, sometimes accompanied by tearing, nasal congestion, and/or visible swelling in the region. Neurologists will often obtain MRI with/without contrast and/or a head CT to evaluate the pituitary gland, rule out tumor, and to assess for possible AVM. Octreotide is used in the treatment of cluster headaches but is felt to be inferior to Sumatriptan in terms of its overall response time. Lidocaine is a topical treatment that is inexpensive and somewhat effective, but not as helpful as Sumatriptan. The patient is depressed because of his headaches. It should be noted that many patients with cluster headaches voice wanting to pursue suicide due to the severity of their symptoms. They should be screened for suicidal ideation and treated aggressively. Prozac is indicated for the treatment of depression, but will not abort the headaches. Augmentin is incorrect because it treats acute sinusitis. Although the tearing, swelling, and nasal congestion can occur with cluster headaches, the pain from acute sinusitis is likely to be more constant and associated with fever or general malaise.

    References

    PubMed. Triptans for acute cluster headache. AU. Law S, Derry S, Moore RA, Cochrane Database Syst Rev. 2010.

    Treatment of acute cluster headache with sumatriptan. The Sumatriptan Cluster Headache Study Group. N Engl J Med. 1991;325(5):322.

    Cluster headache: pathogenesis, diagnosis, and management. May A. Lancet. 2005;366(9488):843.

  36. Question 36 of 220
    36. Question

    The patient is an 18-year-old male who recently left home for college. He has felt distracted in his coursework. He feels very stressed out about new social situations and has been showering for several hours daily. He has had repetitive thoughts about maintaining his personal hygiene and avoids using public restrooms at all costs. He has come to the campus clinic to seek help. He denies any suicidal thoughts. Which of the following would be the best treatment for his condition?

    Correct

    The correct answer is B. This patient has Obsessive Compulsive Disorder or OCD. This condition is characterized by recurrent intrusive thoughts that compel one to repeat or ritualize unrelated behaviors. For instance, in the vignette described above the patient deals with his social anxiety by showering for “hours” each day. He is focused on a sense of cleanliness and is afraid of contamination associated with public restrooms. Zoloft is a selective serotonin re-uptake inhibitor (SSRI). These drugs are generally well tolerated and are the first line treatment for OCD. Side effects include impotency and occasional gastrointestinal distress. Clonipramine is an effective drug but has more adverse side effects than its counterpart, Zoloft. These include somnolence, dry mouth, constipation, and even cardiac conduction delay. Xanax is incorrect because it is short acting and can become habit forming. Lithium is used in the treatment of bipolar disorder and Geodon is used primarily for schizophrenia.

    References

    Selective serotonin re-uptake inhibitors (SSRIs) versus placebo for obsessive compulsive disorder (OCD). Soomro GM, Altman D, Rajagopal S, Oakley-Browne, Cochrane Database Syst Rev. 2008;

    Practice guideline for the treatment of patients with obsessive-compulsive disorder. Koran LM, Hanna GL, Hollander E, Nestadt G, Simpson HB, American Psychiatric Association. Am J Psychiatry. 2007;164(7 Suppl):5.

    Incorrect

    The correct answer is B. This patient has Obsessive Compulsive Disorder or OCD. This condition is characterized by recurrent intrusive thoughts that compel one to repeat or ritualize unrelated behaviors. For instance, in the vignette described above the patient deals with his social anxiety by showering for “hours” each day. He is focused on a sense of cleanliness and is afraid of contamination associated with public restrooms. Zoloft is a selective serotonin re-uptake inhibitor (SSRI). These drugs are generally well tolerated and are the first line treatment for OCD. Side effects include impotency and occasional gastrointestinal distress. Clonipramine is an effective drug but has more adverse side effects than its counterpart, Zoloft. These include somnolence, dry mouth, constipation, and even cardiac conduction delay. Xanax is incorrect because it is short acting and can become habit forming. Lithium is used in the treatment of bipolar disorder and Geodon is used primarily for schizophrenia.

    References

    Selective serotonin re-uptake inhibitors (SSRIs) versus placebo for obsessive compulsive disorder (OCD). Soomro GM, Altman D, Rajagopal S, Oakley-Browne, Cochrane Database Syst Rev. 2008;

    Practice guideline for the treatment of patients with obsessive-compulsive disorder. Koran LM, Hanna GL, Hollander E, Nestadt G, Simpson HB, American Psychiatric Association. Am J Psychiatry. 2007;164(7 Suppl):5.

  37. Question 37 of 220
    37. Question

    A 52-year-old female with a MRSA infection is requiring IV vancomycin as part of her inpatient treatment regimen. She develops a flushing of her neck which rapidly progresses to her face and trunk. The nurse pages you for further instructions. She tells you the patient’s vital signs are stable. What is the next best step in treatment?

    Correct

    The correct answer is E. The patient has developed Red Man Syndrome or a hypersensitivity reaction to vancomycin. Interestingly, this reaction is traditionally only seen with intravenous administration of the medication. Neck and facial flushing can occur, and as the reaction worsens, it can spread to the face and torso. The redness can be accompanied by erythema of the affected areas along with pruritis. The reaction occurs more frequently at higher infusion rates. It is therefore prudent to stop the medication and give Benadryl. An H2 blocker such as Zantac also offers additional antihistamine coverage. Epinephrine is incorrect because it is typically reserved for more severe reactions. It would be appropriate when hypotension is observed or the patient is experiencing anaphylactic shock. Decreasing the rate of vancomycin administration is incorrect because it doesn’t mitigate the patient’s ongoing symptoms and prevent progression to a more severe reaction. Normal Saline, although used to flush and dilute the medication in the patient’s system, is not treating the patient’s symptoms. Doing absolutely nothing is negligent. It is advised that a period of at least 4 hours pass before attempting to restart this medication and providers should cut the initial rate of administration in half. Most patients can be expected to make a full recovery.

    References

    PubMed. Mechanisms of vancomycin-induced histamine release from rat peritoneal mast cells.
    Horinouchi Y, Abe K, Kubo K, Oka M. Agents Actions. 1993;40(1-2):28.

    PubMed. Vancomycin-induced histamine release and “red man syndrome”: comparison of 1- and 2-hour infusions. Healy DP, Sahai JV, Fuller SH, Polk. Antimicrob Agents Chemother. 1990;34(4):550.

    Incorrect

    The correct answer is E. The patient has developed Red Man Syndrome or a hypersensitivity reaction to vancomycin. Interestingly, this reaction is traditionally only seen with intravenous administration of the medication. Neck and facial flushing can occur, and as the reaction worsens, it can spread to the face and torso. The redness can be accompanied by erythema of the affected areas along with pruritis. The reaction occurs more frequently at higher infusion rates. It is therefore prudent to stop the medication and give Benadryl. An H2 blocker such as Zantac also offers additional antihistamine coverage. Epinephrine is incorrect because it is typically reserved for more severe reactions. It would be appropriate when hypotension is observed or the patient is experiencing anaphylactic shock. Decreasing the rate of vancomycin administration is incorrect because it doesn’t mitigate the patient’s ongoing symptoms and prevent progression to a more severe reaction. Normal Saline, although used to flush and dilute the medication in the patient’s system, is not treating the patient’s symptoms. Doing absolutely nothing is negligent. It is advised that a period of at least 4 hours pass before attempting to restart this medication and providers should cut the initial rate of administration in half. Most patients can be expected to make a full recovery.

    References

    PubMed. Mechanisms of vancomycin-induced histamine release from rat peritoneal mast cells.
    Horinouchi Y, Abe K, Kubo K, Oka M. Agents Actions. 1993;40(1-2):28.

    PubMed. Vancomycin-induced histamine release and “red man syndrome”: comparison of 1- and 2-hour infusions. Healy DP, Sahai JV, Fuller SH, Polk. Antimicrob Agents Chemother. 1990;34(4):550.

  38. Question 38 of 220
    38. Question

    A 42-year-old male is currently being evaluated for severe epigastric pain that radiates to the back. His pain started yesterday and worsens when he eats. It is 10/10 and constant. He has had no fever or chills, but is markedly nauseous. He has vomited up bile several times. The patient initially denied alcohol use, but now admits to having a six pack of beer nightly. He has had a CT scan that shows pancreatic inflammation. What is the best lab value to order and confirm your diagnosis?

    Correct

    The correct answer is C. Acute pancreatitis is a condition marked by severe epigastric discomfort. The pain is described as being constant and is almost always accompanied by nausea and vomiting. Serum lipase has a high sensitivity and specificity for acute pancreatitis. Amylase is also a marker of this condition but is not as helpful as lipase in making the diagnosis. Lipase rises within 4-8 hours and peaks at 24 hours. Amylase rises in 6-12 hours after the initial onset of pancreatic inflammation, but has a short half life. Thus lipase elevations occur earlier and last longer, making it the better test, particularly if the patient presents greater than 24 hours after the onset of initial symptoms. A blood alcohol level will confirm whether or not the patient has alcohol in his system, but does not aid in making a diagnosis of acute pancreatitis. H. pylori is implicated in reflux disease and is an inappropriate marker to order. Total bilirubin is utilized to screen for the presence of liver disease and/or to follow chronic liver conditions.

    References

    PubMed. Comparative evaluation of the diagnosis of acute pancreatitis based on serum and urine enzyme assays. Gwozdz GP, Steinberg WM, Werner M, Henry JP, Pauley C, Clin Chim Acta. 1990;187(3):243.

    Amylase normal, lipase elevated: is it pancreatitis? A case series and review of the literature.
    Frank B, Gottlieb K, Am J Gastroenterol. 1999 Feb;94(2):463-9.

    Incorrect

    The correct answer is C. Acute pancreatitis is a condition marked by severe epigastric discomfort. The pain is described as being constant and is almost always accompanied by nausea and vomiting. Serum lipase has a high sensitivity and specificity for acute pancreatitis. Amylase is also a marker of this condition but is not as helpful as lipase in making the diagnosis. Lipase rises within 4-8 hours and peaks at 24 hours. Amylase rises in 6-12 hours after the initial onset of pancreatic inflammation, but has a short half life. Thus lipase elevations occur earlier and last longer, making it the better test, particularly if the patient presents greater than 24 hours after the onset of initial symptoms. A blood alcohol level will confirm whether or not the patient has alcohol in his system, but does not aid in making a diagnosis of acute pancreatitis. H. pylori is implicated in reflux disease and is an inappropriate marker to order. Total bilirubin is utilized to screen for the presence of liver disease and/or to follow chronic liver conditions.

    References

    PubMed. Comparative evaluation of the diagnosis of acute pancreatitis based on serum and urine enzyme assays. Gwozdz GP, Steinberg WM, Werner M, Henry JP, Pauley C, Clin Chim Acta. 1990;187(3):243.

    Amylase normal, lipase elevated: is it pancreatitis? A case series and review of the literature.
    Frank B, Gottlieb K, Am J Gastroenterol. 1999 Feb;94(2):463-9.

  39. Question 39 of 220
    39. Question

    Your patient is a 78-year-old female who is being discharged from the hospital after being treated for an acute embolic stroke. Which medication would be the best choice to start in addition to an appropriate anti-platelet agent such as aspirin or Plavix?

    Correct

    The correct answer is C. Statins, like Lipitor, have been found to not only prevent initial strokes but recurrent ones too. This is an interesting point, because in and of itself, hyperlipidemia is not a significant risk factor for stroke. It is hypothesized that perhaps the statins have a plaque stabilizing effect by reducing inflammation. Metoprolol is not the best choice. Many patients with an acute stroke have elevated blood pressures that are compensatory and intended to increase cerebral perfusion. This elevation in blood pressure should not be treated unless it is excessively high. Although stroke patients may often suffer from dizziness, Meclizine is intended for symptomatic treatment. It is not the best answer because it does not alter the disease course, as a statin does. The same is true of Fioricet. This medication is used for headache relief, commonly for migraines. Prilosec may be utilized to prevent gastrointestinal upset associated with the use of aspirin, but does not prevent a stroke from reoccurring.

    References

    PubMed. High-dose atorvastatin after stroke or transient ischemic attack. Amarenco P, Bogousslavsky J, Callahan A 3rd, Goldstein LB, Hennerici M, Rudolph AE, Sillesen H, Simunovic L, Szarek M, Welch KM, Zivin JA, Stroke Prevention by Aggressive Reduction in Cholesterol Levels (SPARCL) Investigators.

    PubMed. Antiatherothrombotic properties of statins: implications for cardiovascular event reduction. Rosenson RS, Tangney CC.

    Incorrect

    The correct answer is C. Statins, like Lipitor, have been found to not only prevent initial strokes but recurrent ones too. This is an interesting point, because in and of itself, hyperlipidemia is not a significant risk factor for stroke. It is hypothesized that perhaps the statins have a plaque stabilizing effect by reducing inflammation. Metoprolol is not the best choice. Many patients with an acute stroke have elevated blood pressures that are compensatory and intended to increase cerebral perfusion. This elevation in blood pressure should not be treated unless it is excessively high. Although stroke patients may often suffer from dizziness, Meclizine is intended for symptomatic treatment. It is not the best answer because it does not alter the disease course, as a statin does. The same is true of Fioricet. This medication is used for headache relief, commonly for migraines. Prilosec may be utilized to prevent gastrointestinal upset associated with the use of aspirin, but does not prevent a stroke from reoccurring.

    References

    PubMed. High-dose atorvastatin after stroke or transient ischemic attack. Amarenco P, Bogousslavsky J, Callahan A 3rd, Goldstein LB, Hennerici M, Rudolph AE, Sillesen H, Simunovic L, Szarek M, Welch KM, Zivin JA, Stroke Prevention by Aggressive Reduction in Cholesterol Levels (SPARCL) Investigators.

    PubMed. Antiatherothrombotic properties of statins: implications for cardiovascular event reduction. Rosenson RS, Tangney CC.

  40. Question 40 of 220
    40. Question

    A 24-year-old female comes into the clinic complaining of a tick bite that occurred 3 days ago. She now has a red rash at the site and is concerned she may have contracted Lyme disease. She has had flu like symptoms accompanying this rash. Which of the following is the best antibiotic therapy? Please note that the patient is allergic to doxycycline.

    Correct

    The correct answer is B. Early Lyme disease is characterized by a bull’s eye rash (erythema migrans), myalgias, lymphadenopathy, and fever. Interestingly, the rash is not always present. Unfortunately this patient is allergic to doxycycline, the preferred treatment for Lyme. It is preferred because it is a once daily regimen and can also treat a coexisting infection transmitted by ticks, Anaplasma phagocytophilum. If doxycycline cannot be used, however, Amoxicillin is the correct choice. It has equal efficacy to doxycycline and is well tolerated. Zithromax has been shown to be ineffective in the treatment of Lyme disease. Resistance to macrolides has been demonstrated amongst some strains of Some strains of B. burgdorferi. Longer courses of these antibiotics have not been studied and so their true efficacy is not known. Additionally first generation cephalosporins such as Keflex also have a high failure rate. Penicillin G is incorrect because it is used in later disease stages when neurological symptoms have developed. The patient described is presumed to have early disease, given that the tick bite was 3 days prior to her presentation. Levaquin is not appropriate. An erythema migrans rash may sometimes give the appearance of a cellulitis for which levaquin would be a reasonable choice, however, it is not utilized in the treatment of Lyme disease.

    References

    PubMed. Azithromycin compared with amoxicillin in the treatment of erythema migrans. A double-blind, randomized, controlled trial. Luft BJ, Dattwyler RJ, Johnson RC, Luger SW, Bosler EM, Rahn DW, Masters EJ, Grunwaldt E, Gadgil SD. Ann Intern Med. 1996;124(9):785.

    PubMed. Failure of treatment with cephalexin for Lyme disease. Nowakowski J, McKenna D, Nadelman RB, Cooper D, Bittker S, Holmgren D, Pavia C, Johnson RC, Wormser GP. Arch Fam Med. 2000;9(6):563.

    Incorrect

    The correct answer is B. Early Lyme disease is characterized by a bull’s eye rash (erythema migrans), myalgias, lymphadenopathy, and fever. Interestingly, the rash is not always present. Unfortunately this patient is allergic to doxycycline, the preferred treatment for Lyme. It is preferred because it is a once daily regimen and can also treat a coexisting infection transmitted by ticks, Anaplasma phagocytophilum. If doxycycline cannot be used, however, Amoxicillin is the correct choice. It has equal efficacy to doxycycline and is well tolerated. Zithromax has been shown to be ineffective in the treatment of Lyme disease. Resistance to macrolides has been demonstrated amongst some strains of Some strains of B. burgdorferi. Longer courses of these antibiotics have not been studied and so their true efficacy is not known. Additionally first generation cephalosporins such as Keflex also have a high failure rate. Penicillin G is incorrect because it is used in later disease stages when neurological symptoms have developed. The patient described is presumed to have early disease, given that the tick bite was 3 days prior to her presentation. Levaquin is not appropriate. An erythema migrans rash may sometimes give the appearance of a cellulitis for which levaquin would be a reasonable choice, however, it is not utilized in the treatment of Lyme disease.

    References

    PubMed. Azithromycin compared with amoxicillin in the treatment of erythema migrans. A double-blind, randomized, controlled trial. Luft BJ, Dattwyler RJ, Johnson RC, Luger SW, Bosler EM, Rahn DW, Masters EJ, Grunwaldt E, Gadgil SD. Ann Intern Med. 1996;124(9):785.

    PubMed. Failure of treatment with cephalexin for Lyme disease. Nowakowski J, McKenna D, Nadelman RB, Cooper D, Bittker S, Holmgren D, Pavia C, Johnson RC, Wormser GP. Arch Fam Med. 2000;9(6):563.

  41. Question 41 of 220
    41. Question

    A 60-year-old female with a 50-pack year history continues to smoke. Today she awoke short of breath and stated that her inhalers at home were not working for her. She presented to the Emergency Room with a respiratory rate of 24 and initial oxygen saturations of 89%. Her blood pressure and heart rate are stable. The Emergency Room physician has already given her supplemental oxygen and additional nebulizer treatments. She is somewhat better, but if she continues to have difficulty breathing, which of the following medications would be appropriate to try next?

    Correct

    The correct answer is E. Systemic steroids such as IV solumedrol have been shown to improve lung function, diminish symptoms of shortness of breath, and decrease the length of hospital stay for patients experiencing an acute exacerbation of COPD. Giving this medication is a means to rapidly address the patient’s compromised respiratory status. Zithromax is often used in conjunction with steroids for COPD exacerbations, mainly to protect the patient from an ongoing bacterial infection that may actually caused the flare. It is not going to help this patient in a more immediate sense, however. Advair is a maintenance medication for COPD and is not indicated acutely. Mucinex helps to clear bronchial secretions and can be used in these patients, but it is not the best answer. Some mucoactive agents may actually make patients’ symptoms worse. Finally, magnesium is sometimes utilized in the treatment of respiratory emergencies, specifically asthma exacerbations. With asthma it has been shown to have a bronchodilating effect. It is important to note however, this is not true of COPD.

    References

    PubMed. Controlled clinical trial of methylprednisolone in patients with chronic bronchitis and acute respiratory insufficiency. Albert RK, Martin TR, Lewis SW. Ann Intern Med. 1980;92(6):753.

    PubMed. Effect of systemic glucocorticoids on exacerbations of chronic obstructive pulmonary disease. Department of Veterans Affairs Cooperative Study Group. Niewoehner DE, Erbland ML, Deupree RH, Collins D, Gross NJ, Light RW, Anderson P, Morgan NA. N Engl J Med. 1999;340(25):1941.

    PubMed. Effect of intravenous magnesium sulfate on chronic obstructive pulmonary disease exacerbations requiring hospitalization: a randomized placebo-controlled trial. Abreu González J1, Hernández García C, Abreu González P, Martín García C, Jiménez A.
    Arch Bronconeumol. 2006 Oct;42(10):491.

    Incorrect

    The correct answer is E. Systemic steroids such as IV solumedrol have been shown to improve lung function, diminish symptoms of shortness of breath, and decrease the length of hospital stay for patients experiencing an acute exacerbation of COPD. Giving this medication is a means to rapidly address the patient’s compromised respiratory status. Zithromax is often used in conjunction with steroids for COPD exacerbations, mainly to protect the patient from an ongoing bacterial infection that may actually caused the flare. It is not going to help this patient in a more immediate sense, however. Advair is a maintenance medication for COPD and is not indicated acutely. Mucinex helps to clear bronchial secretions and can be used in these patients, but it is not the best answer. Some mucoactive agents may actually make patients’ symptoms worse. Finally, magnesium is sometimes utilized in the treatment of respiratory emergencies, specifically asthma exacerbations. With asthma it has been shown to have a bronchodilating effect. It is important to note however, this is not true of COPD.

    References

    PubMed. Controlled clinical trial of methylprednisolone in patients with chronic bronchitis and acute respiratory insufficiency. Albert RK, Martin TR, Lewis SW. Ann Intern Med. 1980;92(6):753.

    PubMed. Effect of systemic glucocorticoids on exacerbations of chronic obstructive pulmonary disease. Department of Veterans Affairs Cooperative Study Group. Niewoehner DE, Erbland ML, Deupree RH, Collins D, Gross NJ, Light RW, Anderson P, Morgan NA. N Engl J Med. 1999;340(25):1941.

    PubMed. Effect of intravenous magnesium sulfate on chronic obstructive pulmonary disease exacerbations requiring hospitalization: a randomized placebo-controlled trial. Abreu González J1, Hernández García C, Abreu González P, Martín García C, Jiménez A.
    Arch Bronconeumol. 2006 Oct;42(10):491.

  42. Question 42 of 220
    42. Question

    A 54-year-old man is brought to the emergency room by ambulance for chest pain. The paramedics were informed that he recently did multiple lines of cocaine four hours ago and then suddenly developed severe pain in his chest, left shoulder, and jaw. On arrival he is diaphoretic, tachycardic, and tachypneic. Serial troponins are performed and EKG shows ST segment elevations > 1 mm in leads V1-V4. Due to his history, which drug would be contraindicated for this patient?

    Correct

    The correct answer is B. Beta blockers are contraindicated for cocaine-induced MI. Cocaine causes vasoconstriction of the coronary arteries. It does this by stimulating alpha-adrenergic receptors and increasing the levels of endothelin-1. It also limits the chances of vasodilation by decreasing the body’s production of nitric oxide, a natural vasodilator. It also increases heart rate, contractility, and blood pressure, which all result in increased cardiac demand. Beta-blockers can cause unopposed alpha-constriction, increased blood pressure, and coronary vasospasm, leading to further ischemia. Nitroglycerin, aspirin, and benzodiazepines are all acceptable treatment modalities in the setting of a cocaine-induced MI. Nitroglycerin is a vasodilator that will help decrease cardiac demand by decreasing the preload and increase myocardial blood supply. Aspirin helps prevent platelet aggregation and activation. It will not break up an existing clot, but will help prevent new ones from forming. “Benzodiazepines decrease the central stimulatory effects of cocaine, which indirectly reduce cardiovascular toxicity. It also decreases neuropsychiatric symptoms associated with cocaine, which are closely related to cardiovascular complications.”2 Ace inhibitors are utilized with myocarditis or cardiomyopathy which are often seen secondary to the use of cocaine.

    References

    Williams, D. PANCE Prep Pearls. CreateSpace; 2014.

    McCord J, Hani J, Hollander J, el al. Management of Cocaine-Associated Chest Pain and Myocardial Infarction. American Heart Association. 2008; 117: 1897-1907.

    Incorrect

    The correct answer is B. Beta blockers are contraindicated for cocaine-induced MI. Cocaine causes vasoconstriction of the coronary arteries. It does this by stimulating alpha-adrenergic receptors and increasing the levels of endothelin-1. It also limits the chances of vasodilation by decreasing the body’s production of nitric oxide, a natural vasodilator. It also increases heart rate, contractility, and blood pressure, which all result in increased cardiac demand. Beta-blockers can cause unopposed alpha-constriction, increased blood pressure, and coronary vasospasm, leading to further ischemia. Nitroglycerin, aspirin, and benzodiazepines are all acceptable treatment modalities in the setting of a cocaine-induced MI. Nitroglycerin is a vasodilator that will help decrease cardiac demand by decreasing the preload and increase myocardial blood supply. Aspirin helps prevent platelet aggregation and activation. It will not break up an existing clot, but will help prevent new ones from forming. “Benzodiazepines decrease the central stimulatory effects of cocaine, which indirectly reduce cardiovascular toxicity. It also decreases neuropsychiatric symptoms associated with cocaine, which are closely related to cardiovascular complications.”2 Ace inhibitors are utilized with myocarditis or cardiomyopathy which are often seen secondary to the use of cocaine.

    References

    Williams, D. PANCE Prep Pearls. CreateSpace; 2014.

    McCord J, Hani J, Hollander J, el al. Management of Cocaine-Associated Chest Pain and Myocardial Infarction. American Heart Association. 2008; 117: 1897-1907.

  43. Question 43 of 220
    43. Question

    A 65-year-old man with known long-standing alcohol abuse comes into the emergency department with severe abdominal pain that has been getting progressively worse over the past week. He states that the pain is constant and mainly located in the epigastric area with some radiation to his back. Other than the alcohol use, his PMH is unremarkable. Serial troponins and EKG’s are performed with normal results. On physical exam he has decreased bowel sounds with tenderness to palpation in the epigastric area. Murphy’s sign is performed with negative results. Neither the spleen nor liver are able to be palpated as well. Lab values show increased WBC of 18,000, increased AST and ALT, hypocalcemia, and elevated lipase. What would be the best test to diagnose this patient?

    Correct

    The correct answer is A. The two most common causes of pancreatitis are alcohol abuse and gallstones. In this case, this man has acute pancreatitis due to his alcohol abuse. Key things that would lead you to the possible diagnosis of acute pancreatitis would be epigastric pain, history of alcohol abuse as well as elevated liver enzymes, lipase, and WBC. The diagnostic test of choice to make the official diagnosis of acute pancreatitis is an abdominal CT scan.

    Abdominal X-ray can also be used to diagnose pancreatitis, but it is not considered the test of choice. On x-ray you may see dilated small bowel in the LUQ, which is known as a sentinel loop. You also may see abrupt collapse of the colon near the pancreas. Sometimes pleural effusions can be seen at the lung bases and often an elevated hemidiaphragm will be present. HIDA scan is the gold standard for diagnosing cholecystitis. Cholecystitis is inflammation/infection of the gallbladder. Common signs of acute cholecystitis are episodic RUQ/epigastric pain usually precipitated by a fatty meal. Liver enzymes would be elevated with cholecystitis, but you would expect the patient to have a positive murphy’s sign on exam. Liver biopsy is used to diagnose cirrhosis. Although this man has a history of alcohol abuse, you would expect to find more signs on physical exam such as jaundice, ascites, and hepatosplenomegaly. Yes, his liver enzymes would be elevated, but you would also expect his bilirubin to be to elevated as well. Also, cirrhosis is due to chronic liver disease which this man has no history of. This is not a disease process that would occur within the mentioned time frame. An echocardiogram could be performed to ensure this patient does not have alcohol induced cardiomyopathy or acute pericarditis. An echo would also tell you if there is any valvular disease or ischemia that has resulted in wall motion abnormalities of the heart. This test is not really indicated in cases of acute pancreatitis though and would not help you to make the diagnosis.

    References

    Williams, D. PANCE Prep Pearls. CreateSpace; 2014.

    Respiratory complications in acute pancreatitis. Ranson JH, Turner JW, Roses DF, Rifkind KM, Spencer FC. Ann Surg. 1974;179(5):557.

    Incorrect

    The correct answer is A. The two most common causes of pancreatitis are alcohol abuse and gallstones. In this case, this man has acute pancreatitis due to his alcohol abuse. Key things that would lead you to the possible diagnosis of acute pancreatitis would be epigastric pain, history of alcohol abuse as well as elevated liver enzymes, lipase, and WBC. The diagnostic test of choice to make the official diagnosis of acute pancreatitis is an abdominal CT scan.

    Abdominal X-ray can also be used to diagnose pancreatitis, but it is not considered the test of choice. On x-ray you may see dilated small bowel in the LUQ, which is known as a sentinel loop. You also may see abrupt collapse of the colon near the pancreas. Sometimes pleural effusions can be seen at the lung bases and often an elevated hemidiaphragm will be present. HIDA scan is the gold standard for diagnosing cholecystitis. Cholecystitis is inflammation/infection of the gallbladder. Common signs of acute cholecystitis are episodic RUQ/epigastric pain usually precipitated by a fatty meal. Liver enzymes would be elevated with cholecystitis, but you would expect the patient to have a positive murphy’s sign on exam. Liver biopsy is used to diagnose cirrhosis. Although this man has a history of alcohol abuse, you would expect to find more signs on physical exam such as jaundice, ascites, and hepatosplenomegaly. Yes, his liver enzymes would be elevated, but you would also expect his bilirubin to be to elevated as well. Also, cirrhosis is due to chronic liver disease which this man has no history of. This is not a disease process that would occur within the mentioned time frame. An echocardiogram could be performed to ensure this patient does not have alcohol induced cardiomyopathy or acute pericarditis. An echo would also tell you if there is any valvular disease or ischemia that has resulted in wall motion abnormalities of the heart. This test is not really indicated in cases of acute pancreatitis though and would not help you to make the diagnosis.

    References

    Williams, D. PANCE Prep Pearls. CreateSpace; 2014.

    Respiratory complications in acute pancreatitis. Ranson JH, Turner JW, Roses DF, Rifkind KM, Spencer FC. Ann Surg. 1974;179(5):557.

  44. Question 44 of 220
    44. Question

    Your patient is a 24-year-old female who presents to the clinic complaining of reddened areas on the palms and soles of her feet. She is also experiencing fever, chills, general malaise, and headache. She admits to using IV heroin, and the last time she injected was 5 days ago. On physical exam you hear a heart murmur. You suspect she has infective endocarditis. Which of the following tests would help to best confirm your diagnosis?

    Correct

    The correct answer is E. Infective endocarditis is a very serious condition in which bacteremia can result in damage to the heart valves and/or colonization of an intracardiac device. The Modified Duke Criteria are used to determine the pretest probability of IE. Per the Duke Criteria, infective endocarditis is definitively present if a vegetation can be confirmed with echocardiogram testing or if blood cultures are positive. These represent the major criteria. Minor criteria include: recreational IV drug abuse, fever, Janeway lesions, and/or evidence of emboli. The patient in the above vignette has Janeway lesions on her palms and soles. She admits to injecting IV heroin, which puts her at significant risk for IE. The other associated symptoms are consistent with her bacteremia. Although a transthoracic echocardiogram can be obtained in this circumstance, and is often the first test performed, transesophageal echocardiogram testing is more sensitive. It is better at detecting aneurysm, abscess, and other potential cardiac complications. A chest CT would be helpful in determining distant sites of metastatic infection but does not aid in making the initial diagnosis.

    References

    Infective Endocarditis in Adults: Diagnosis, Antimicrobial Therapy, and Management of Complications: A Scientific Statement for Healthcare Professionals From the American Heart Association. Baddour LM, Wilson WR, Bayer AS, Fowler VG Jr, Tleyjeh IM, Rybak MJ, Barsic B, Lockhart PB, Gewitz MH, Levison ME, Bolger AF, Steckelberg JM, Baltimore RS, Fink AM, O’Gara P, Taubert KA, American Heart Association Committee on Rheumatic Fever, Endocarditis, and Kawasaki Disease of the Council on Cardiovascular Disease in the Young, Council on Clinical Cardiology, Council on Cardiovascular Surgery and Anesthesia, and Stroke Council. Circulation. 2015;132(15):1435.

    Diagnostic value of echocardiography in suspected endocarditis. An evaluation based on the pretest probability of disease. Lindner JR, Case RA, Dent JM, Abbott RD, Scheld WM, Kaul S
    Circulation. 1996;93(4):730.

    014 AHA/ACC guideline for the management of patients with valvular heart disease: a report of the American College of Cardiology/American Heart Association Task Force on Practice Guidelines. Nishimura RA, Otto CM, Bonow RO, Carabello BA, Erwin JP 3rd, Guyton RA, O’Gara PT, Ruiz CE, Skubas NJ, Sorajja P, Sundt TM 3rd, Thomas JD. J Am Coll Cardiol. 2014;63(22):e57.

    Incorrect

    The correct answer is E. Infective endocarditis is a very serious condition in which bacteremia can result in damage to the heart valves and/or colonization of an intracardiac device. The Modified Duke Criteria are used to determine the pretest probability of IE. Per the Duke Criteria, infective endocarditis is definitively present if a vegetation can be confirmed with echocardiogram testing or if blood cultures are positive. These represent the major criteria. Minor criteria include: recreational IV drug abuse, fever, Janeway lesions, and/or evidence of emboli. The patient in the above vignette has Janeway lesions on her palms and soles. She admits to injecting IV heroin, which puts her at significant risk for IE. The other associated symptoms are consistent with her bacteremia. Although a transthoracic echocardiogram can be obtained in this circumstance, and is often the first test performed, transesophageal echocardiogram testing is more sensitive. It is better at detecting aneurysm, abscess, and other potential cardiac complications. A chest CT would be helpful in determining distant sites of metastatic infection but does not aid in making the initial diagnosis.

    References

    Infective Endocarditis in Adults: Diagnosis, Antimicrobial Therapy, and Management of Complications: A Scientific Statement for Healthcare Professionals From the American Heart Association. Baddour LM, Wilson WR, Bayer AS, Fowler VG Jr, Tleyjeh IM, Rybak MJ, Barsic B, Lockhart PB, Gewitz MH, Levison ME, Bolger AF, Steckelberg JM, Baltimore RS, Fink AM, O’Gara P, Taubert KA, American Heart Association Committee on Rheumatic Fever, Endocarditis, and Kawasaki Disease of the Council on Cardiovascular Disease in the Young, Council on Clinical Cardiology, Council on Cardiovascular Surgery and Anesthesia, and Stroke Council. Circulation. 2015;132(15):1435.

    Diagnostic value of echocardiography in suspected endocarditis. An evaluation based on the pretest probability of disease. Lindner JR, Case RA, Dent JM, Abbott RD, Scheld WM, Kaul S
    Circulation. 1996;93(4):730.

    014 AHA/ACC guideline for the management of patients with valvular heart disease: a report of the American College of Cardiology/American Heart Association Task Force on Practice Guidelines. Nishimura RA, Otto CM, Bonow RO, Carabello BA, Erwin JP 3rd, Guyton RA, O’Gara PT, Ruiz CE, Skubas NJ, Sorajja P, Sundt TM 3rd, Thomas JD. J Am Coll Cardiol. 2014;63(22):e57.

  45. Question 45 of 220
    45. Question

    An otherwise healthy 62-year-old male was treated in the hospital for a small bowel obstruction. During his stay he did require an NG tube, but was able to be managed conservatively. The NG tube was eventually removed once bowel sounds were audible. Under the direction of a consulting surgeon, the patient’s diet was advanced and he was allowed to return home. Just prior to leaving the hospital though he developed a cough. A repeat chest x ray was negative on the day of discharge. He was treated symptomatically with antitussive agents and was not hypoxic. The patient returned to the emergency room three days later, however, complaining of a sore throat. The ER physician noticed that he was drooling and his voice was hoarse. His recorded temperature was 38.1. All of his other vital signs were stable. On physical exam the patient did not have any cervical lymphadenopathy. The most likely cause for this patient’s condition is?

    Correct

    The correct answer is D. Epiglottitis is an inflammation of the epiglottis. It can be caused by bacteria or trauma. In this case, the NG tube placement may have been responsible for causing trauma to the affected area. Adults can experience fever, hoarseness, sore throat, drooling, and even respiratory compromise. Airway compromise is less common in adults than children though, because of the larger anatomic structures they possess. Radiographs will often show a “thumb sign” or an enlarged epiglottis protruding out from the hypopharynx. Adult patients with epiglottitis frequently require ICU monitoring and empiric antibiotic therapy. A third generation cephalosporin with staph coverage is generally recommended. The course of treatment may vary but is routinely prescribed for 7-10 days. Pneumonia is not the correct answer. It is in the differential diagnosis, but is generally not responsible for causing a sore throat or drooling. Pharyngitis is typically accompanied by fever and sore throat, but symptoms of headache, malaise, anorexia, and swollen glands are also present. Infectious mononucleosis has some overlapping symptoms with epiglottitis, but is also known for causing profound and prolonged fatigue. White or even green colored exudates can be seen on the tonsils in this condition. A foreign body is the incorrect answer. The patient would likely have noticed something obstructing his airway upon removal of the NG tube, instead of experiencing a more gradual onset of his symptoms.

    References

    Radiographic parameters in adult epiglottitis. Schumaker HM, Doris PE, Birnbaum G
    Ann Emerg Med. 1984;13(8):588.

    Diagnosis and management of supraglottitis (epiglottitis). Glynn F, Fenton JE
    Curr Infect Dis Rep. 2008;10(3):200.

    Incorrect

    The correct answer is D. Epiglottitis is an inflammation of the epiglottis. It can be caused by bacteria or trauma. In this case, the NG tube placement may have been responsible for causing trauma to the affected area. Adults can experience fever, hoarseness, sore throat, drooling, and even respiratory compromise. Airway compromise is less common in adults than children though, because of the larger anatomic structures they possess. Radiographs will often show a “thumb sign” or an enlarged epiglottis protruding out from the hypopharynx. Adult patients with epiglottitis frequently require ICU monitoring and empiric antibiotic therapy. A third generation cephalosporin with staph coverage is generally recommended. The course of treatment may vary but is routinely prescribed for 7-10 days. Pneumonia is not the correct answer. It is in the differential diagnosis, but is generally not responsible for causing a sore throat or drooling. Pharyngitis is typically accompanied by fever and sore throat, but symptoms of headache, malaise, anorexia, and swollen glands are also present. Infectious mononucleosis has some overlapping symptoms with epiglottitis, but is also known for causing profound and prolonged fatigue. White or even green colored exudates can be seen on the tonsils in this condition. A foreign body is the incorrect answer. The patient would likely have noticed something obstructing his airway upon removal of the NG tube, instead of experiencing a more gradual onset of his symptoms.

    References

    Radiographic parameters in adult epiglottitis. Schumaker HM, Doris PE, Birnbaum G
    Ann Emerg Med. 1984;13(8):588.

    Diagnosis and management of supraglottitis (epiglottitis). Glynn F, Fenton JE
    Curr Infect Dis Rep. 2008;10(3):200.

  46. Question 46 of 220
    46. Question

    A 34-year-old woman presents to urgent care for a sore throat that she has had for four days. She is also experiencing back pain, chills, and fevers reaching 104 degrees Fahrenheit. She denies any sick contacts. She also denies any headaches, fatigue, abdominal pain, nausea, vomiting, diarrhea, coughing, or sinus congestion. She presents wrapped up in a heavy winter coat and has rigors when she removes it for the physical exam. She has noticeable anterior cervical lymphadenopathy and moderate erythema to the pharynx. The remainder of the exam is unremarkable. Based on the information provided above, what would be your first task in diagnosing this patient?

    Correct

    The correct answer is E. This patient is presenting with the typical symptoms of streptococcal pharyngitis. Although the most common cause of strep is viral, this patient has a bacterial infection. This is most commonly caused by Group A Beta Hemolytic Streptococcus. The Centor Criteria for strep throat consists of: Fever (>100.4 degrees fahrenheit), tender anterior cervical lymphadenopathy, absence of a cough, and pharyngotonsillar exudate. Each symptom is worth 1 point. Any patient with 2-3 points should get a throat culture. This patient presents with 3 of the 4 symptoms (fever, anterior cervical lymphadenopathy, and no cough), giving her a score of 3 points and therefore needing a throat culture. Choice A, a rapid influenza test via nasopharyngeal swab is incorrect. The common symptoms for influenza do include fever, myalgias, and chills but also a cough, nasal congestion, coryza, and headaches. This patient is not experiencing a majority of these symptoms, so therefore a rapid influenza test would not be the best choice. Choice B, MonoSpot test followed by Epstein Barr titers is incorrect. Mononucleosis is a good option to keep in the differential, but a key symptom that would be a negative indication of mononucleosis is “anterior” cervical lymphadenopathy. Most patients with mononucleosis will have posterior cervical lymphadenopathy. Although patients with mononucleosis will have a sore throat and fever, they will also complain of severe fatigue as well as abdominal discomfort. This patient denied these symptoms. The abdominal pain is due to the inflammation of the spleen and liver that occurs during the disease. Patients who have an enlarged spleen or liver should avoid contact sports and trauma for at least 1 month in order to avoid possible rupture. On physical exam, bilateral lid edema known as Hoagland sign and conjunctival hemorrhaging may be present as well. Answer C is done if epiglottitis is suspected. A thumb print sign is a positive finding. The patient’s airway should be protected and intubation should be considered. Choice D, do nothing because it’s viral, is not the correct answer. This patient has been experiencing high fevers for multiple days which would indicate more of a bacterial infection rather than viral. Most viral infections have lower grade fevers as opposed to bacterial infections where fevers tend to be higher. Also, in bacterial infections the fever will get worse a few days into the illness rather than getting better.

    References

    Williams D.A. PANCE Prep Pearls. CreateSpace. 2014.
    Papadakis M, McPhee S, Rabow M. Current Medical Diagnosis & Treatment 2015. 2015

    Duke University Health System. Is it a Bacterial Infection or Virus?. DukeMedicine. https://www.dukemedicine.org/blog/it-bacterial-infection-or-virus. Published Oct. 1, 2013. Accessed Jan. 25, 2016.

    Incorrect

    The correct answer is E. This patient is presenting with the typical symptoms of streptococcal pharyngitis. Although the most common cause of strep is viral, this patient has a bacterial infection. This is most commonly caused by Group A Beta Hemolytic Streptococcus. The Centor Criteria for strep throat consists of: Fever (>100.4 degrees fahrenheit), tender anterior cervical lymphadenopathy, absence of a cough, and pharyngotonsillar exudate. Each symptom is worth 1 point. Any patient with 2-3 points should get a throat culture. This patient presents with 3 of the 4 symptoms (fever, anterior cervical lymphadenopathy, and no cough), giving her a score of 3 points and therefore needing a throat culture. Choice A, a rapid influenza test via nasopharyngeal swab is incorrect. The common symptoms for influenza do include fever, myalgias, and chills but also a cough, nasal congestion, coryza, and headaches. This patient is not experiencing a majority of these symptoms, so therefore a rapid influenza test would not be the best choice. Choice B, MonoSpot test followed by Epstein Barr titers is incorrect. Mononucleosis is a good option to keep in the differential, but a key symptom that would be a negative indication of mononucleosis is “anterior” cervical lymphadenopathy. Most patients with mononucleosis will have posterior cervical lymphadenopathy. Although patients with mononucleosis will have a sore throat and fever, they will also complain of severe fatigue as well as abdominal discomfort. This patient denied these symptoms. The abdominal pain is due to the inflammation of the spleen and liver that occurs during the disease. Patients who have an enlarged spleen or liver should avoid contact sports and trauma for at least 1 month in order to avoid possible rupture. On physical exam, bilateral lid edema known as Hoagland sign and conjunctival hemorrhaging may be present as well. Answer C is done if epiglottitis is suspected. A thumb print sign is a positive finding. The patient’s airway should be protected and intubation should be considered. Choice D, do nothing because it’s viral, is not the correct answer. This patient has been experiencing high fevers for multiple days which would indicate more of a bacterial infection rather than viral. Most viral infections have lower grade fevers as opposed to bacterial infections where fevers tend to be higher. Also, in bacterial infections the fever will get worse a few days into the illness rather than getting better.

    References

    Williams D.A. PANCE Prep Pearls. CreateSpace. 2014.
    Papadakis M, McPhee S, Rabow M. Current Medical Diagnosis & Treatment 2015. 2015

    Duke University Health System. Is it a Bacterial Infection or Virus?. DukeMedicine. https://www.dukemedicine.org/blog/it-bacterial-infection-or-virus. Published Oct. 1, 2013. Accessed Jan. 25, 2016.

  47. Question 47 of 220
    47. Question

    A 38 year-old female presents with bilateral lower limb numbness and pain for 2 days. Her pain is 4/10. It is getting harder to walk, and she has never experienced this before. She had a sinus infection 1 week ago. She is not taking any medication and is generally healthy. She denies fever, chills, abdominal pain, however she had an “accident” with her bladder yesterday. On examination, Motor is 5/5, she is spastic, hyperreflexia, decreased sensation to pin prick and vibration in the lower limbs. What is the most likely diagnosis?

    Correct

    The answer is B. Transverse myelitis is a rare inflammation of the spinal cord which makes it an upper neuron lesion. About 1400 people per year develop transverse myelitis effecting males and females equally. However, in female there is an association with Multiple Sclerosis. This could happen at any age (3) . Transverse myelitis is caused by a number of conditions such as infection, secondary to Multiple Sclerosis, Neuromyelitis Optica, connective tissues, sarcoidosis and other autoimmune disorders (4) . A postinfectious patient could presents with a combination of bilateral pain, numbness, weakness, ataxia, spasticity, hyperreflexia, bladder and bowel dysfunction. There is no one diagnostic test for Transverse Myelitis. MRI, Lumbar puncture, chest CT, infectious and other immunological tests should be order. Enhanced MRI should show inflammation at the suspected region. CSF should show inflammation such as pleocytosis, elevated IgG and protein, and normal glucose (3,5) . For the treatment of TM , IV glucocorticoid is the first line for acute idiopathic cause. Plasma exchange may also be consider. The underlying cause should be thoroughly investigated and treated (3). Parkinson and TIA are incorrect because they are unlikely in a 38 year old. Parkinson is a chronic progressive disease, it is unlikely to start with a 2 day history of an acute neurological episode (6) . TIA by definition does not last longer than 24 hr, therefore among other reasons, this is not the case (7) . Guillain Barre syndrome is an immune mediated lower neuron neuropathy which starts at the distal portion of the nerve and move centrally. GBS presents with weakness and hyporeflexia or areflexia, while Transverse Myelitis is an upper neuron lesion which is hyperreflexia and spastic (8).

    References

    1. Bennetto. L, N Scolding. Inflammatory/postinfectious encephalomyelitis. Journal of Neurology, Neurosurgery & Psychiatry. 2004;75:i22i28 doi:10.1136/jnnp.2003.034256. Accessed date 5/31/2016

    2. Brenton. J, nicholas. Acute disseminated encephalomyelitis clinical presentation. Medscape.
    http://emedicine.medscape.com/article/1147044clinical#b4 . Last update dec 30, 2015. Accessed date
    5/31/2016

    3. Krishnan, Chitra. Transverse myelitis. Uptodate. http://www.uptodate.com/contents/transversemyelitis?source=search_result&search=transverse+myelitis&selectedTitle=1%7E101 . Last update Mar 07 2016. accessdate:5/25/2016

    4. Greenberg BM, Frohman EM, Immunemediated Myelopathies. Continuum. 2015; 21(1) 121131.

    5. Eisen, andrew, disorders affecting the spinal cord. Uptodate. http://www.uptodate.com/contents/disordersaffectingthespinalcord?source=search_result&search=transverse+myelitis&selectedTitle=4%7E101 . Last update May 5 2015. Accessed date 5/26/2016

    6. Chou. Kelvin. Diagnosis and differential diagnosis of parkinson disease. Uptodate.
    . http://www.uptodate.com/contents/diagnosisanddifferentialdiagnosisofparkinsondisease?
    source=search_result&search=parkinson&selectedTitle=3%7E150 . Last update Feb 29 2016. Accessed date 5/26/2016

    7. Furie, Karen. Etiology and clinical manifestation of transient ischemic attack. Uptodate.http://www.uptodate.com/contents/etiologyandclinicalmanifestationsoftransientischemicattack?
    source=machineLearning&search=TIA&selectedTitle=2%7E150&sectionRank=1&anchor=H7#H7 . Last update. Jan 10, 2014. Access date 5/26/2016

    8. Vriesendorp, Francine. Clinical features and diagnosis of Guillainbarre syndrome in adults. Uptodate. http://www.uptodate.com/contents/clinicalfeaturesanddiagnosisofguillainbarresyndromeinadults?source=search_result&search=guillain+barre&selectedTitle=1%7E150 . Last update Mar 16 2016. Accessed date 5/26/2016.

    Incorrect

    The answer is B. Transverse myelitis is a rare inflammation of the spinal cord which makes it an upper neuron lesion. About 1400 people per year develop transverse myelitis effecting males and females equally. However, in female there is an association with Multiple Sclerosis. This could happen at any age (3) . Transverse myelitis is caused by a number of conditions such as infection, secondary to Multiple Sclerosis, Neuromyelitis Optica, connective tissues, sarcoidosis and other autoimmune disorders (4) . A postinfectious patient could presents with a combination of bilateral pain, numbness, weakness, ataxia, spasticity, hyperreflexia, bladder and bowel dysfunction. There is no one diagnostic test for Transverse Myelitis. MRI, Lumbar puncture, chest CT, infectious and other immunological tests should be order. Enhanced MRI should show inflammation at the suspected region. CSF should show inflammation such as pleocytosis, elevated IgG and protein, and normal glucose (3,5) . For the treatment of TM , IV glucocorticoid is the first line for acute idiopathic cause. Plasma exchange may also be consider. The underlying cause should be thoroughly investigated and treated (3). Parkinson and TIA are incorrect because they are unlikely in a 38 year old. Parkinson is a chronic progressive disease, it is unlikely to start with a 2 day history of an acute neurological episode (6) . TIA by definition does not last longer than 24 hr, therefore among other reasons, this is not the case (7) . Guillain Barre syndrome is an immune mediated lower neuron neuropathy which starts at the distal portion of the nerve and move centrally. GBS presents with weakness and hyporeflexia or areflexia, while Transverse Myelitis is an upper neuron lesion which is hyperreflexia and spastic (8).

    References

    1. Bennetto. L, N Scolding. Inflammatory/postinfectious encephalomyelitis. Journal of Neurology, Neurosurgery & Psychiatry. 2004;75:i22i28 doi:10.1136/jnnp.2003.034256. Accessed date 5/31/2016

    2. Brenton. J, nicholas. Acute disseminated encephalomyelitis clinical presentation. Medscape.
    http://emedicine.medscape.com/article/1147044clinical#b4 . Last update dec 30, 2015. Accessed date
    5/31/2016

    3. Krishnan, Chitra. Transverse myelitis. Uptodate. http://www.uptodate.com/contents/transversemyelitis?source=search_result&search=transverse+myelitis&selectedTitle=1%7E101 . Last update Mar 07 2016. accessdate:5/25/2016

    4. Greenberg BM, Frohman EM, Immunemediated Myelopathies. Continuum. 2015; 21(1) 121131.

    5. Eisen, andrew, disorders affecting the spinal cord. Uptodate. http://www.uptodate.com/contents/disordersaffectingthespinalcord?source=search_result&search=transverse+myelitis&selectedTitle=4%7E101 . Last update May 5 2015. Accessed date 5/26/2016

    6. Chou. Kelvin. Diagnosis and differential diagnosis of parkinson disease. Uptodate.
    . http://www.uptodate.com/contents/diagnosisanddifferentialdiagnosisofparkinsondisease?
    source=search_result&search=parkinson&selectedTitle=3%7E150 . Last update Feb 29 2016. Accessed date 5/26/2016

    7. Furie, Karen. Etiology and clinical manifestation of transient ischemic attack. Uptodate.http://www.uptodate.com/contents/etiologyandclinicalmanifestationsoftransientischemicattack?
    source=machineLearning&search=TIA&selectedTitle=2%7E150&sectionRank=1&anchor=H7#H7 . Last update. Jan 10, 2014. Access date 5/26/2016

    8. Vriesendorp, Francine. Clinical features and diagnosis of Guillainbarre syndrome in adults. Uptodate. http://www.uptodate.com/contents/clinicalfeaturesanddiagnosisofguillainbarresyndromeinadults?source=search_result&search=guillain+barre&selectedTitle=1%7E150 . Last update Mar 16 2016. Accessed date 5/26/2016.

  48. Question 48 of 220
    48. Question

    A 78 year-old patient with DM Type II, HTN, and hyperlipidemia presents to the ER for increasing shortness of breath with large fluctuations in weight over the past couple of months. His vitals are Temp: 98.8, RR: 24, HR: 100, BP: 155/86, SpO2: 92% on RA. He is currently on Carvedilol 6.25mg QD, Digoxin 0.125 mg QD, Furosemide 40 mg PO BID, Lisinopril 2.5 mg QD. His serum creatinine is 1.6 and ejection fraction is 15%. Which option will give the patient the best clinical outcome?

    Correct

    Answer D, refer to a cardiac surgeon for placement of an LVAD. This man is clearly showing signs and symptoms of heart failure, which is refractory to medical management, stage D heart failure. Increasing Lisinopril would be inappropriate in this patient as his serum creatinine is elevated. Though adding an inotrope would increase his ejection fraction, it would further increase his blood pressure and/ or heart rate and put this man at risk for an arrhythmia. Increasing his Lasix dose may improve the patients’ shortness of breath; it has no effect on mortality. Due to this patients advancing age, he most likely would not be a candidate for a heart transplant. Also, if this man was candidate, the likelihood that he would receive a donor heart in time is slim.

    “Heart failure (HF) is a major public health issue, with a prevalence of over 5.8 million in the USA. The lifetime risk of developing HF is one in five… [and it] carries substantial morbidity and mortality, with 5-year mortality that rival those of many cancers.”1 In simple terms, heart failure can be defined as a “condition in which the heart can’t pump enough blood to meet the body’s needs.”1 The increase in life expectancy coupled with the steady rise in obesity rates has lead to an increased prevalence in heart failure patients. Heart failure is a clinical syndrome that can be broken down into systolic and/or diastolic failure and can affect the left and/or right side of the heart. The most prevalent type is systolic heart failure of the left ventricle. The key risk factors that have been isolated in the development of heart failure include hypertension, coronary artery disease, diabetes, valvular heart diseases, cardiomyopathy and being greater than 65 years old. 2

    The pathophysiology of chronic systolic heart failure is complex but fundamentally results from long periods of increased stress. As the heart begins to become an inefficient pump and the body senses that it’s not getting enough blood supply, a physiological response will occur. This response is an increase in blood pressure and heart contractility by neurohormones, such as epinephrine and norepinephrine, and activation of the RAAS. The Frank-Starling law of the heart states that an increase in end diastolic volume will create a proportional increase in stroke volume. In order to keep up with these demands, the myocardium hypertrophies. In prolonged periods of stress, myocyte apoptosis occurs which subsequently leads to a slow remodeling of the heart. Ultimately, the end result of this process is a weakened myocardium with dilated ventricles.3

    With diastolic heart failure, the ventricles become thickened and stiff. The total volume of the ventricle is reduced which concomitantly decreases the end diastolic volume.4 The end point of systolic and diastolic dysfunction is comparable. In both instances, there is a decreased stroke volume, cardiac output and cardiac index.

    Patients in heart failure may present with non-specific and constitutional symptoms. A majority of the symptoms are the result of fluid overload or inadequate perfusion. Cough and shortness of breath are common and typically due to excess fluid in the lungs as the left side of the heart is not able to efficiently eject the blood. Fluid retention is evident by swelling in the legs, feet abdomen and is the result of blood getting backed up on the venous side from the right side of the heart. Other common symptoms include fatigue, weakness, and chest pain. On physical exam, the findings depend on the type and severity of heart failure. Pitting edema, rales, murmurs, and increased JVD may be observed.

    A full workup should be started in a patient who presents with possible heart failure as it is a serious condition and there is not one gold standard test or lab to rule the disease in or out. Basic chemistries, a CBC, LFT’s, TSH and urinalysis should be drawn to assess possible other diagnosis’ or complications of the heart failure. A BNP should be drawn and is a valuable lab to assess the presence and severity of an individuals’ heart failure. A chest x-ray should be obtained to assess for pulmonary edema, pleural effusions and cardiomegaly, all signs of heart failure. An echocardiogram can also be a valuable tool to evaluate the filling, contractility, valve function and ejection fraction of the heart. Individuals in systolic heart function will have a decreased ejection fraction while diastolic heart failure will have either a normal or increased ejection fraction. CT scan, MRI and EKG can be used as well to rule out other diseases with similar presentations.

    As indicated above, a diagnosis of heart failure is not a benign issue and is often fatal. Lifestyle modifications such as maintaining a balanced diet low in sodium and regular exercise should be implemented early. There are numerous medications on the market indicated for heart failure, however there is no magic pill that can reverse heart failure. Beta-blockers such as, Metoprolol Tartrate, Bisoprolol, and Carvedilol, as well as ACE-I and ARBs have been proven to reduce mortality and slow the disease process down. Diuretics, Digoxin, and various other medicines have shown to decrease symptoms but have no effect on survival rates.

    Unfortunately, the only true treatment of end-stage heart failure is a heart transplant. In the United States, there is a large discrepancy between those with end stage hear failure that is refractory to medication and the number of available hearts that are available for transplants. Fortunately, a device called a Ventricular Assist Device (VAD) has become a favorable option. These can be implanted in patients who are ineligible for a heart transplant or who need support in the interim of a new heart. As left systolic heart failure is the most prevalent type of health failure, an LVAD device is the most common inserted VAD. Essentially, a VAD works in a similar fashion a healthy ventricle should. In the instance of an LVAD, a cannula is inserted into the left ventricle, which connects to a small pump that is located outside of the heart but within the chest cavity. On the opposite end of the LVAD is a graft that is stitched in to the ascending aorta. The VAD is continuously creating a negative pressure to suction the blood from the left ventricle and eject it into the systemic arterial system.

    Heart failure is and has been an epidemic due to obesity and technological advances that are leading to increased longevity. The mismatch in individuals with this disease and the number of hearts that become available for transplants is tragic. VAD’s have proven to decrease the number of lives that would have been taken prematurely due to heart failure. Though the outlook of many diseases is grim, technology has once again paved the way to the future of medicine.

    References

    1. Bui AL, Horwich TB, Fonarow GC. Epidemiology and risk profile of heart failure. Nature reviews Cardiology. 2011;8(1):30-41. doi:10.1038/nrcardio.2010.165.

    2. What Is Heart Failure? National Heart, Lung, and Blood Institute . https://www.nhlbi.nih.gov/health/health-topics/topics/hf. Accessed June 8, 2016.

    3. Dumitri I. Heart Failure. Medscape. http://emedicine.medscape.com/article/163062-overview#a4. Published January 11, 2016.

    4. Zile MR, Gaasch WH. Pathophysiology of diastolic heart failure. UpToDate. http://www.uptodate.com/contents/pathophysiology-of-diastolic-heart-failure. Published September 15, 2014.

    Incorrect

    Answer D, refer to a cardiac surgeon for placement of an LVAD. This man is clearly showing signs and symptoms of heart failure, which is refractory to medical management, stage D heart failure. Increasing Lisinopril would be inappropriate in this patient as his serum creatinine is elevated. Though adding an inotrope would increase his ejection fraction, it would further increase his blood pressure and/ or heart rate and put this man at risk for an arrhythmia. Increasing his Lasix dose may improve the patients’ shortness of breath; it has no effect on mortality. Due to this patients advancing age, he most likely would not be a candidate for a heart transplant. Also, if this man was candidate, the likelihood that he would receive a donor heart in time is slim.

    “Heart failure (HF) is a major public health issue, with a prevalence of over 5.8 million in the USA. The lifetime risk of developing HF is one in five… [and it] carries substantial morbidity and mortality, with 5-year mortality that rival those of many cancers.”1 In simple terms, heart failure can be defined as a “condition in which the heart can’t pump enough blood to meet the body’s needs.”1 The increase in life expectancy coupled with the steady rise in obesity rates has lead to an increased prevalence in heart failure patients. Heart failure is a clinical syndrome that can be broken down into systolic and/or diastolic failure and can affect the left and/or right side of the heart. The most prevalent type is systolic heart failure of the left ventricle. The key risk factors that have been isolated in the development of heart failure include hypertension, coronary artery disease, diabetes, valvular heart diseases, cardiomyopathy and being greater than 65 years old. 2

    The pathophysiology of chronic systolic heart failure is complex but fundamentally results from long periods of increased stress. As the heart begins to become an inefficient pump and the body senses that it’s not getting enough blood supply, a physiological response will occur. This response is an increase in blood pressure and heart contractility by neurohormones, such as epinephrine and norepinephrine, and activation of the RAAS. The Frank-Starling law of the heart states that an increase in end diastolic volume will create a proportional increase in stroke volume. In order to keep up with these demands, the myocardium hypertrophies. In prolonged periods of stress, myocyte apoptosis occurs which subsequently leads to a slow remodeling of the heart. Ultimately, the end result of this process is a weakened myocardium with dilated ventricles.3

    With diastolic heart failure, the ventricles become thickened and stiff. The total volume of the ventricle is reduced which concomitantly decreases the end diastolic volume.4 The end point of systolic and diastolic dysfunction is comparable. In both instances, there is a decreased stroke volume, cardiac output and cardiac index.

    Patients in heart failure may present with non-specific and constitutional symptoms. A majority of the symptoms are the result of fluid overload or inadequate perfusion. Cough and shortness of breath are common and typically due to excess fluid in the lungs as the left side of the heart is not able to efficiently eject the blood. Fluid retention is evident by swelling in the legs, feet abdomen and is the result of blood getting backed up on the venous side from the right side of the heart. Other common symptoms include fatigue, weakness, and chest pain. On physical exam, the findings depend on the type and severity of heart failure. Pitting edema, rales, murmurs, and increased JVD may be observed.

    A full workup should be started in a patient who presents with possible heart failure as it is a serious condition and there is not one gold standard test or lab to rule the disease in or out. Basic chemistries, a CBC, LFT’s, TSH and urinalysis should be drawn to assess possible other diagnosis’ or complications of the heart failure. A BNP should be drawn and is a valuable lab to assess the presence and severity of an individuals’ heart failure. A chest x-ray should be obtained to assess for pulmonary edema, pleural effusions and cardiomegaly, all signs of heart failure. An echocardiogram can also be a valuable tool to evaluate the filling, contractility, valve function and ejection fraction of the heart. Individuals in systolic heart function will have a decreased ejection fraction while diastolic heart failure will have either a normal or increased ejection fraction. CT scan, MRI and EKG can be used as well to rule out other diseases with similar presentations.

    As indicated above, a diagnosis of heart failure is not a benign issue and is often fatal. Lifestyle modifications such as maintaining a balanced diet low in sodium and regular exercise should be implemented early. There are numerous medications on the market indicated for heart failure, however there is no magic pill that can reverse heart failure. Beta-blockers such as, Metoprolol Tartrate, Bisoprolol, and Carvedilol, as well as ACE-I and ARBs have been proven to reduce mortality and slow the disease process down. Diuretics, Digoxin, and various other medicines have shown to decrease symptoms but have no effect on survival rates.

    Unfortunately, the only true treatment of end-stage heart failure is a heart transplant. In the United States, there is a large discrepancy between those with end stage hear failure that is refractory to medication and the number of available hearts that are available for transplants. Fortunately, a device called a Ventricular Assist Device (VAD) has become a favorable option. These can be implanted in patients who are ineligible for a heart transplant or who need support in the interim of a new heart. As left systolic heart failure is the most prevalent type of health failure, an LVAD device is the most common inserted VAD. Essentially, a VAD works in a similar fashion a healthy ventricle should. In the instance of an LVAD, a cannula is inserted into the left ventricle, which connects to a small pump that is located outside of the heart but within the chest cavity. On the opposite end of the LVAD is a graft that is stitched in to the ascending aorta. The VAD is continuously creating a negative pressure to suction the blood from the left ventricle and eject it into the systemic arterial system.

    Heart failure is and has been an epidemic due to obesity and technological advances that are leading to increased longevity. The mismatch in individuals with this disease and the number of hearts that become available for transplants is tragic. VAD’s have proven to decrease the number of lives that would have been taken prematurely due to heart failure. Though the outlook of many diseases is grim, technology has once again paved the way to the future of medicine.

    References

    1. Bui AL, Horwich TB, Fonarow GC. Epidemiology and risk profile of heart failure. Nature reviews Cardiology. 2011;8(1):30-41. doi:10.1038/nrcardio.2010.165.

    2. What Is Heart Failure? National Heart, Lung, and Blood Institute . https://www.nhlbi.nih.gov/health/health-topics/topics/hf. Accessed June 8, 2016.

    3. Dumitri I. Heart Failure. Medscape. http://emedicine.medscape.com/article/163062-overview#a4. Published January 11, 2016.

    4. Zile MR, Gaasch WH. Pathophysiology of diastolic heart failure. UpToDate. http://www.uptodate.com/contents/pathophysiology-of-diastolic-heart-failure. Published September 15, 2014.

  49. Question 49 of 220
    49. Question

    A 65 year-old male comes to the office with complaints of difficulty breathing for the past day. His symptoms developed after his friend’s football party, where he admits to having a few beers and pizza. He has a regular exercise program where he is able to walk a mile, but abruptly experienced difficulty breathing 3 minutes into his walk today. Patient admits to having slight chest discomfort but denies nausea, vomiting, diaphoresis, or lightheadedness. On physical exam, you notice a 15lbs weight gain since his last visit three weeks ago, 3+ bilateral pitting edema to his knees, bibasilar crackles, and JVD. His preliminary labs drawn showed a normal troponin, but elevated proBNP of 8,376 pg/mL. Normal sinus rhythm on EKG. Based on the patient’s presentation, which initial management is the most appropriate to relieve his shortness of breath?

    Correct

    Correct A. This patient presents with an acute exacerbation of congestive heart failure with a decrease in exertional capacity, dyspnea, significant weight gain, JVD, and signs of fluid overload. The most appropriate treatment would be a loop diuretic like Furosemide, which is the most effective treatment for symptom relief of mod-severe CHF with fluid overload. It works by inhibiting water transportation across the loop of Henle resulting in an increased excretion of water, chloride, sodium and potassium.5,7,8 Since this patient is having an acute onset, intravenous treatment is necessary for rapid relief of symptoms.

    Incorrect B This patient has negative troponins and a normal EKG and is not currently having angina or AMI. Isosorbide mononitrate does decrease the preload via venous relaxation but it is not the most effective treatment for immediate symptom relief.

    Incorrect C ACE inhibitors along with diuretics are first line treatment for CHF patients, but the Lisinopril takes 7-10 days for its pharmacological effect. ACE inhibitors are initiated as first line if the patient is not in fluid overload. Therefore, a diuretic would be the most appropriate rather than lisinopril.5,7

    Incorrect D Metoprolol Succinate takes about 7-10 days for its pharmacological effects to take place. It works by decreasing the patients HR and BP, improving the left ventricular function and dilation, improving symptoms within 7-10 days. It does not help with fluid excretion or the immediate relief of symptoms after the first dose, like Furosemide.5,7

    Reference: Mann D, Chakinala M. Heart Failure and Cor Pulmonale. In: Kasper DL, Fauci AS, Longo DL, Braunwald E, Hauser SL, Jameson JL, eds. Harrison’s Principles of Internal Medicine. 18th ed. New York, NY: McGraw-Hill; 2012: 1901-1913

    Incorrect

    Correct A. This patient presents with an acute exacerbation of congestive heart failure with a decrease in exertional capacity, dyspnea, significant weight gain, JVD, and signs of fluid overload. The most appropriate treatment would be a loop diuretic like Furosemide, which is the most effective treatment for symptom relief of mod-severe CHF with fluid overload. It works by inhibiting water transportation across the loop of Henle resulting in an increased excretion of water, chloride, sodium and potassium.5,7,8 Since this patient is having an acute onset, intravenous treatment is necessary for rapid relief of symptoms.

    Incorrect B This patient has negative troponins and a normal EKG and is not currently having angina or AMI. Isosorbide mononitrate does decrease the preload via venous relaxation but it is not the most effective treatment for immediate symptom relief.

    Incorrect C ACE inhibitors along with diuretics are first line treatment for CHF patients, but the Lisinopril takes 7-10 days for its pharmacological effect. ACE inhibitors are initiated as first line if the patient is not in fluid overload. Therefore, a diuretic would be the most appropriate rather than lisinopril.5,7

    Incorrect D Metoprolol Succinate takes about 7-10 days for its pharmacological effects to take place. It works by decreasing the patients HR and BP, improving the left ventricular function and dilation, improving symptoms within 7-10 days. It does not help with fluid excretion or the immediate relief of symptoms after the first dose, like Furosemide.5,7

    Reference

    Mann D, Chakinala M. Heart Failure and Cor Pulmonale. In: Kasper DL, Fauci AS, Longo DL, Braunwald E, Hauser SL, Jameson JL, eds. Harrison’s Principles of Internal Medicine. 18th ed. New York, NY: McGraw-Hill; 2012: 1901-1913

  50. Question 50 of 220
    50. Question

    A 42 year old white male presents complaining of fevers, chills, headache and fatigue which started gradually approximately past 6 days. Acetaminophen seems to help somewhat but only for a short time. Review of systems is also positive for mild to moderate myalgia and arthralgia as well as mild dyspnea on exertion. Denies any past medical history. His work for a non-profit forest registry organization has recently taken him to Thailand and the Congo and his symptoms began approximately 2 weeks after returning to the U.S. He reports taking Doxycycline for a short period of time while away and is uncertain of his travel vaccination status. Exam reveals a well-developed male in mild discomfort but non-toxic appearing, no rash or other significant findings. Pertinent results from the complete blood count (CBC) are as follows: WBC 5.5 with 31% bands, Hgb 12.9, Hct 37.1, Platelet count 40. Chest x-ray is negative for infiltrate. What is the patient’s most likely diagnosis base on the history and laboratory findings?

    Correct

    Correct answer: Malaria. Fever, malaise, anemia and thrombocytopenia are common findings in a patient infected with malaria. In this case, ring forms were identified on the smear for parasites. Dengue Fever will typically present with fever and usually severe myalgia (termed “break-bone”) and a maculopapular rash starting on the trunk and progressing to the extremities. Anemia in Dengue Fever usually only occurs if the condition becomes hemorrhagic. Ebola presents with fever and myalgia as well but is almost always accompanied by chest pain with cough, vomiting, diarrhea, and altered mental status and possibly progress to bleeding and hemorrhage. Leukopenia, in addition to thrombocytopenia, is usually present. Influenza is generally an abrupt onset of fever, chills, myalgia and is usually associated with a cough. Influenza is generally not associated with a bandemia or thrombocytopenia. Typhoid usually results in fever and abdominal pain. A rash, described as “rose spots”, occurs in up to 30% of cases. Thrombocytopenia is less commonly found with Typhoid.

    Malaria is a protozoan disease found mostly in Central and South America, Africa and Asia. There are five species of the protozoa Plasmodium which can be transmitted to humans via mosquito bites with the species P. falciparum being responsible for the vast majority of deaths related to malaria. Incubation times between the bite and clinical symptoms varies based on the specific species that the host is infected with and can be between 9-40 days. During this time the protozoa are replicating with the host’s liver. Symptoms typically include headache, fatigue, irregular fevers and sweats leading to myalgias, arthralgias, abdominal pain, vomiting and diarrhea. Rash and lymphadenopathy are very uncommon. Giemsa stain with thick and thin smears are used to assess for parasitemia. Thrombocytopenia is a more common finding than anemia. Treatment options include quinine plus doxycycline or tetracycline, amodiaquine, or a combination drug of atovaquone-proguanil. Chloroquine is still used however resistance has been occurring. There is no currently approved vaccine so travelers to these parts of the world are generally advised to use prophylactic medications. With proper treatment, outcome is very good and fevers usually resolve within 1-2 days.

    Dengue Fever is the transmission of the flavivirus to a human host also by a mosquito bite and is prevalent in similar geographic as malaria. Incubation time from bite is 3-14 days and classic symptoms include abrupt onset of fevers, headache, retroorbital pain, and severe myalgias. Adults are more often symptomatic than children. Macular rash, lymphadenopathy and vesicles on the palate are also possible as well as hemorrhagic symptoms. Leukopenia, thrombocytopenia, and elevated transaminases are seen in lab testing and the diagnosis is confirmed with IgM and IgG ELISA’s. Treatment is supportive care until the virus runs it’s course.
    Influenza A and B are prevalent in temperate countries throughout the world and are spread via respiratory droplets with a 1-4 day incubation time. Symptoms onset is generally abrupt and includes fevers and chills, fatigue, myalgias and cough as well as occasional vomiting and diarrhea. Physical exam is usually unremarkable except for the appearance of fatigue and discomfort. The diagnosis is made either clinically or via influenza testing using a nasal swab. CBC is generally normal or may reveal leukopenia. Treatment is supportive or an anti-influenza medication may be prescribed for more severe symptoms or for those with other underlying health conditions if the patient presents within 48 hours of symptom onset.

    Typhoid (enteric fever) is an infection with Salmonella typhi and usually occurs in areas with poor sanitary conditions within India, China, Africa and Southeast Asia. Incubation time is typically 10-14 days and classic symptoms are fever and abdominal pain. Headache as well as a rash are also relatively prevalent. The rash is usually maculopapular on the trunk and chest consisting of faint, pink, blanchable areas which have been described as “rose spots”. Definitive diagnosis on positive cultures of blood, bone marrow or stool. The Widal test for “febrile agglutinins” may be helpful but is not definitive. Treatment is usually with ciprofloxacin but ceftriaxone, azithromycin, amoxicillin or trimethoprim-sulfamethoxazole and others are also used with good success rates.

    References:

    Longo DL, Kasper D, Jameson J, Fauci A, Hauser S, Loscalzo J. Harrison’s Principles of Internal Medicine. New York: McGraw-Hill; 2012.

    Papadakis, Maxine A., Stephen J. McPhee, and Michael W. Rabow. 2015 Current Medical Diagnosis & Treatment. New York: McGraw-Hill Education/Medical, 2015. Print.

    Influenza. In DynaMed Plus® – the next-generation clinical information resource designed to decrease time to answer. DynaMed Plus. Available at: http://www.dynamed.com/home/. Accessed March 10, 2016.

    Incorrect

    Correct answer: Malaria. Fever, malaise, anemia and thrombocytopenia are common findings in a patient infected with malaria. In this case, ring forms were identified on the smear for parasites. Dengue Fever will typically present with fever and usually severe myalgia (termed “break-bone”) and a maculopapular rash starting on the trunk and progressing to the extremities. Anemia in Dengue Fever usually only occurs if the condition becomes hemorrhagic. Ebola presents with fever and myalgia as well but is almost always accompanied by chest pain with cough, vomiting, diarrhea, and altered mental status and possibly progress to bleeding and hemorrhage. Leukopenia, in addition to thrombocytopenia, is usually present. Influenza is generally an abrupt onset of fever, chills, myalgia and is usually associated with a cough. Influenza is generally not associated with a bandemia or thrombocytopenia. Typhoid usually results in fever and abdominal pain. A rash, described as “rose spots”, occurs in up to 30% of cases. Thrombocytopenia is less commonly found with Typhoid.

    Malaria is a protozoan disease found mostly in Central and South America, Africa and Asia. There are five species of the protozoa Plasmodium which can be transmitted to humans via mosquito bites with the species P. falciparum being responsible for the vast majority of deaths related to malaria. Incubation times between the bite and clinical symptoms varies based on the specific species that the host is infected with and can be between 9-40 days. During this time the protozoa are replicating with the host’s liver. Symptoms typically include headache, fatigue, irregular fevers and sweats leading to myalgias, arthralgias, abdominal pain, vomiting and diarrhea. Rash and lymphadenopathy are very uncommon. Giemsa stain with thick and thin smears are used to assess for parasitemia. Thrombocytopenia is a more common finding than anemia. Treatment options include quinine plus doxycycline or tetracycline, amodiaquine, or a combination drug of atovaquone-proguanil. Chloroquine is still used however resistance has been occurring. There is no currently approved vaccine so travelers to these parts of the world are generally advised to use prophylactic medications. With proper treatment, outcome is very good and fevers usually resolve within 1-2 days.

    Dengue Fever is the transmission of the flavivirus to a human host also by a mosquito bite and is prevalent in similar geographic as malaria. Incubation time from bite is 3-14 days and classic symptoms include abrupt onset of fevers, headache, retroorbital pain, and severe myalgias. Adults are more often symptomatic than children. Macular rash, lymphadenopathy and vesicles on the palate are also possible as well as hemorrhagic symptoms. Leukopenia, thrombocytopenia, and elevated transaminases are seen in lab testing and the diagnosis is confirmed with IgM and IgG ELISA’s. Treatment is supportive care until the virus runs it’s course.
    Influenza A and B are prevalent in temperate countries throughout the world and are spread via respiratory droplets with a 1-4 day incubation time. Symptoms onset is generally abrupt and includes fevers and chills, fatigue, myalgias and cough as well as occasional vomiting and diarrhea. Physical exam is usually unremarkable except for the appearance of fatigue and discomfort. The diagnosis is made either clinically or via influenza testing using a nasal swab. CBC is generally normal or may reveal leukopenia. Treatment is supportive or an anti-influenza medication may be prescribed for more severe symptoms or for those with other underlying health conditions if the patient presents within 48 hours of symptom onset.

    Typhoid (enteric fever) is an infection with Salmonella typhi and usually occurs in areas with poor sanitary conditions within India, China, Africa and Southeast Asia. Incubation time is typically 10-14 days and classic symptoms are fever and abdominal pain. Headache as well as a rash are also relatively prevalent. The rash is usually maculopapular on the trunk and chest consisting of faint, pink, blanchable areas which have been described as “rose spots”. Definitive diagnosis on positive cultures of blood, bone marrow or stool. The Widal test for “febrile agglutinins” may be helpful but is not definitive. Treatment is usually with ciprofloxacin but ceftriaxone, azithromycin, amoxicillin or trimethoprim-sulfamethoxazole and others are also used with good success rates.

    References:

    Longo DL, Kasper D, Jameson J, Fauci A, Hauser S, Loscalzo J. Harrison’s Principles of Internal Medicine. New York: McGraw-Hill; 2012.

    Papadakis, Maxine A., Stephen J. McPhee, and Michael W. Rabow. 2015 Current Medical Diagnosis & Treatment. New York: McGraw-Hill Education/Medical, 2015. Print.

    Influenza. In DynaMed Plus® – the next-generation clinical information resource designed to decrease time to answer. DynaMed Plus. Available at: http://www.dynamed.com/home/. Accessed March 10, 2016.

  51. Question 51 of 220
    51. Question

    Which condition causes a hypercoaguable state predisposing patients to pulmonary embolism?

    Correct

    Correct answer is A. Activated protein C resistance (Factor V Leiden) is the most common cause of pulmonary embolus in white patients. Other hypercoagulabe states include malignancy, deficiencies of protein C, protein S, and antithrombin III, extensive burns, recent long distance travel, and oral contraceptives. Von Willebrands, Factor 7 deficiency, and idiopathic thrombocytopenia purpura are bleeding disorders that reduce the body’s ability to clot.

    The most common cause of sudden death is cardiac related while the second most common cause is pulmonary emboli (PE). 1 “PE & deep vein thrombosis (DVT) are immediate & intermediate causes of death, respectively.”2 A PE is defined as the blockage of one or more pulmonary arteries. A blockage at the bifurcation of the pulmonary trunk is defined as a saddle embolus. PE is not a disease, but rather a complication of a venous thromboembolism; the most common is DVT.1

    A PE is considered acute if its location is central within the lumen or completely occludes the vessel. A PE is considered chronic if it is “eccentric & contiguous with the vessel wall, it reduces arterial diameter by more than 50%, evidence of recanalization within thrombus is present, & an arterial web is present.” 1 A central PE is located in the “main pulmonary artery, left and right main pulmonary artery, anterior trunk, right and left interlobar arteries, left upper lobe trunk, right middle lobe artery and right and left lower lobe arteries” whereas a peripheral PE is located in the ”segmental and subsegmental arteries of the right upper lobe, right middle lobe, right lower lobe, left upper lobe, lingual, and left lower lobe”. 1 A massive PE includes bilateral pulmonary arteries or results in hemodynamic compromise. 1

    The etiology of a PE can be attributed to Virchow’s Triad: stasis, hypercoagulability, and intimal damage.3 The most common etiology is a DVT but others include air, amniotic fluid, fat, foreign body, parasite eggs, septic emboli, and tumor cells. 4 Factor V Leiden, resistance to activated protein C, is the most common inherited etiology in the white population.1

    Of the percentage of patients that die in hospitals, around 60% have a PE and were misdiagnosed around 70% of times. For every 1,000 people, there is one case of PE per year. Between 60-80% of patients with DVTs end up with PEs, but, over 50% present asymptomatically. 1 The 3rd most common cause of deaths in hospitalized patients is PE. 1, 4

    A PE caused by right ventricle or pulmonary artery obstruction leads to the venous blood being damned which causes distention of venous vessels and flow through the pulmonary circuit to the left side of the heart to slow. This slowing of blood to the left heart and aorta cause the mechanical collapse of the entire cardiovascular system.5 The thrombus that causes the PE begins as a “platelet nidus”1, usually on a lower extremity venous valve. It continues to grow by accumulation of fibrin and platelets then progresses to a red fibrin thrombus. This can embolize or cause complete occlusion of the vessel.1

    The ‘classic’ presentation of PE, “abrupt onset pleuritc chest pain, shortness of breath, and hypoxia”1, is actually less common than the other signs and symptoms. Symptoms can include: dyspnea, chest pain, cough, leg pain, palpitations, wheezing, and aginal pain. Signs may include: tachypnea, rales, accentuated second heart sound, tachycardia, fever, diaphoresis, S3 and S4 gallop, hemoptysis, pleural friction rub, and cyanosis.1 The most common symptom is dyspnea while the most common sign is tachypnea.3 A PE may present with a normal pulmonary exam and (+) Homan’s sign.3
    A PE can be worked up with many diagnostic imaging studies. A plain chest radiograph (CXR) is most often normal but if the patient has hypoxia in the setting of a normal CXR, there should be a high suspicion for PE. There may or may not be a pleural effusion or atelectasis. There are classic findings on CXR indicating a PE, but these are not common. 3 The classic findings include Westermark’s Sign, Hampton’s Hump, and Fleischner Sign 3,6. On electrocardiograph, there may be changes in Lead I showing a deep, wide S and in Lead III Q waves and inverted T waves can be seen as well. This is written in medical abbreviation as S1Q3T3.3,6 Respiratory alkalosis on initial ABG will, with time, convert to respiratory acidosis. A D dimer has low specificity but high sensitivity. This can be helpful if it is negative to rule a PE out of the differential diagnosis. Other studies include CT and V/Q scans. A pulmonary angiography is the gold standard but is only performed if the following criteria are met: either CT or V/Q are negative and the clinician has a high suspicion. On ultrasound, the most common finding is DVT.3

    Treatment for a PE can be medical or mechanical. Medical treatment consists of anticoagulation, via heparin or low molecular weight heparin in combination with warfarin, or thrombolytic, via t-PA (Alteplase). “Heparin binds to and accelerates the ability of antithrombin to inactivate thrombin, factor Xa and factor IXa. It thus retards additional thrombus formation, allowing endogenous fibrinolytic mechanisms to lyse existing clot.”4 “Streptokinase, urokinase, and recombinant tissue plasminogen activator (rt-PA; Alteplase) increase plasmin levels and thereby directly lyse intravascular thrombi.”4 Mechanical treatment is rare but includes: inferior vena cava filter, pulmonary embolectomy, and pulmonary thromboendarterectomy.4

    References

    1. Ouellette D. Pulmonary Embolism: Practice Essentials, Background, Anatomy. Emedicinemedscape.com. 2016. Available at: http://emedicine.medscape.com/article/300901-overview#showarll. Accessed February 1, 2016.

    2. Dolinak D, Matshes E, Lew E. Forensic Pathology. Boston: Elesevier Academic Press; 2005:104.

    3. Williams D. PANCE Prep Pearls. Lexington, KY: CreateSpace; 2014:98-100.

    4. Papadakis M, McPhee S, Rabow M. Current Medical Diagnosis & Treatment 2015. 54th ed. New York, NY: McGrawHill; 2015:293-299.

    5. Spitz W, Spitz D. Fischer R. Spitz And Fischer’s Medicolegal Investigation of Death. 4th ed. Springfield, Ill.: Charles C. Thomas; 2006:331-332.

    6. Hrciz C. Pulmonary (Venous Thrombo)embolism (PE). [PowerPoint] Manchester, NH: MCPHS Physician Assistant Program; 2015

    Incorrect

    Correct answer is A. Activated protein C resistance (Factor V Leiden) is the most common cause of pulmonary embolus in white patients. Other hypercoagulabe states include malignancy, deficiencies of protein C, protein S, and antithrombin III, extensive burns, recent long distance travel, and oral contraceptives.  Von Willebrands, Factor 7 deficiency, and idiopathic thrombocytopenia purpura are bleeding disorders that reduce the body’s ability to clot.

    The most common cause of sudden death is cardiac related while the second most common cause is pulmonary emboli (PE). 1 “PE & deep vein thrombosis (DVT) are immediate & intermediate causes of death, respectively.”2 A PE is defined as the blockage of one or more pulmonary arteries. A blockage at the bifurcation of the pulmonary trunk is defined as a saddle embolus. PE is not a disease, but rather a complication of a venous thromboembolism; the most common is DVT.1

    A PE is considered acute if its location is central within the lumen or completely occludes the vessel. A PE is considered chronic if it is “eccentric & contiguous with the vessel wall, it reduces arterial diameter by more than 50%, evidence of recanalization within thrombus is present, & an arterial web is present.” 1 A central PE is located in the “main pulmonary artery, left and right main pulmonary artery, anterior trunk, right and left interlobar arteries, left upper lobe trunk, right middle lobe artery and right and left lower lobe arteries” whereas a peripheral PE is located in the ”segmental and subsegmental arteries of the right upper lobe, right middle lobe, right lower lobe, left upper lobe, lingual, and left lower lobe”. 1 A massive PE includes bilateral pulmonary arteries or results in hemodynamic compromise. 1

    The etiology of a PE can be attributed to Virchow’s Triad: stasis, hypercoagulability, and intimal damage.3 The most common etiology is a DVT but others include air, amniotic fluid, fat, foreign body, parasite eggs, septic emboli, and tumor cells. 4 Factor V Leiden, resistance to activated protein C, is the most common inherited etiology in the white population.1

    Of the percentage of patients that die in hospitals, around 60% have a PE and were misdiagnosed around 70% of times. For every 1,000 people, there is one case of PE per year. Between 60-80% of patients with DVTs end up with PEs, but, over 50% present asymptomatically. 1 The 3rd most common cause of deaths in hospitalized patients is PE. 1, 4

    A PE caused by right ventricle or pulmonary artery obstruction leads to the venous blood being damned which causes distention of venous vessels and flow through the pulmonary circuit to the left side of the heart to slow. This slowing of blood to the left heart and aorta cause the mechanical collapse of the entire cardiovascular system.5 The thrombus that causes the PE begins as a “platelet nidus”1, usually on a lower extremity venous valve. It continues to grow by accumulation of fibrin and platelets then progresses to a red fibrin thrombus. This can embolize or cause complete occlusion of the vessel.1

    The ‘classic’ presentation of PE, “abrupt onset pleuritc chest pain, shortness of breath, and hypoxia”1, is actually less common than the other signs and symptoms. Symptoms can include: dyspnea, chest pain, cough, leg pain, palpitations, wheezing, and aginal pain. Signs may include: tachypnea, rales, accentuated second heart sound, tachycardia, fever, diaphoresis, S3 and S4 gallop, hemoptysis, pleural friction rub, and cyanosis.1 The most common symptom is dyspnea while the most common sign is tachypnea.3 A PE may present with a normal pulmonary exam and (+) Homan’s sign.3
    A PE can be worked up with many diagnostic imaging studies. A plain chest radiograph (CXR) is most often normal but if the patient has hypoxia in the setting of a normal CXR, there should be a high suspicion for PE. There may or may not be a pleural effusion or atelectasis. There are classic findings on CXR indicating a PE, but these are not common. 3 The classic findings include Westermark’s Sign, Hampton’s Hump, and Fleischner Sign 3,6. On electrocardiograph, there may be changes in Lead I showing a deep, wide S and in Lead III Q waves and inverted T waves can be seen as well. This is written in medical abbreviation as S1Q3T3.3,6 Respiratory alkalosis on initial ABG will, with time, convert to respiratory acidosis.  A D dimer has low specificity but high sensitivity. This can be helpful if it is negative to rule a PE out of the differential diagnosis. Other studies include CT and V/Q scans. A pulmonary angiography is the gold standard but is only performed if the following criteria are met: either CT or V/Q are negative and the clinician has a high suspicion. On ultrasound, the most common finding is DVT.3

    Treatment for a PE can be medical or mechanical. Medical treatment consists of anticoagulation, via heparin or low molecular weight heparin in combination with warfarin, or thrombolytic, via t-PA (Alteplase). “Heparin binds to and accelerates the ability of antithrombin to inactivate thrombin, factor Xa and factor IXa. It thus retards additional thrombus formation, allowing endogenous fibrinolytic mechanisms to lyse existing clot.”4 “Streptokinase, urokinase, and recombinant tissue plasminogen activator (rt-PA; Alteplase) increase plasmin levels and thereby directly lyse intravascular thrombi.”4 Mechanical treatment is rare but includes: inferior vena cava filter, pulmonary embolectomy, and pulmonary thromboendarterectomy.4

    References

    1. Ouellette D. Pulmonary Embolism: Practice Essentials, Background, Anatomy. Emedicinemedscape.com. 2016. Available at: http://emedicine.medscape.com/article/300901-overview#showarll. Accessed February 1, 2016.

    2. Dolinak D, Matshes E, Lew E.  Forensic Pathology. Boston: Elesevier Academic Press; 2005:104.

    3. Williams D. PANCE Prep Pearls. Lexington, KY: CreateSpace; 2014:98-100.

    4. Papadakis M, McPhee S, Rabow M. Current Medical Diagnosis & Treatment 2015. 54th ed. New York, NY: McGrawHill; 2015:293-299.

    5. Spitz W, Spitz D. Fischer R. Spitz And Fischer’s Medicolegal Investigation of Death. 4th ed. Springfield, Ill.: Charles C. Thomas; 2006:331-332.

    6. Hrciz C. Pulmonary (Venous Thrombo)embolism (PE). [PowerPoint] Manchester, NH: MCPHS Physician Assistant Program; 2015

  52. Question 52 of 220
    52. Question

    question 2
    A 57 year old female complains of shortness of breath with no prior pulmonary or cardiac history.  What is the appropriate interpretation of her EKG above?

    Correct

    Answer: Sinus rhythm. This EKG demonstrates a moderate amount of baseline artifact however the QRS complexes are at regular intervals and leads V1 and V2 demonstrate p waves followed by QRS complexes. These 2 findings exclude atrial fibrillation and flutter. A Mobitz II block consists of a fixed, prolonged PR interval with an occasional non-conducted P wave resulting in a dropped beat. There are no dropped beats on this EKG. The rate is approximately 80bpm which eliminates supraventricular tachycardia (>100bpm).

    Artifacts on an EKG tracing can be due to one or more of several reasons including an expired electrode, poor skin contact, breakdown of the conduction gel in the center of the electrode, tautness of the skin underlying an electrode, and finally, patient movements including breathing, tremors or any other muscle use. These artifacts can mimic a wide variety of arrhythmias and the importance of a thorough assessment of all EKG leads as well as a complete assessment of the patient including any obvious signs of instability and a manual palpation of the pulse should be conducted when attempting to make a determination between artifact and an actual abnormal finding. If the patient is stable, any possible arrhythmia on a rhythm strip should be confirmed with a 12-lead EKG.

    When evaluating a 12-lead EKG, if sinus rhythm is not completely apparent, assess all leads for P followed by QRS. Many times P waves can be more easily appreciated in lead V1. Also, each column of leads (for example: leads I, II, III on a standard format) represents the same period of time, therefore, any irregularity noted in one lead should also be present in the other two that correspond to the same point in time. In the case of the attached EKG, P waves are most visible in V1 and V2 and there is no significant artifact which would correspond to the same timeframe as the lead II rhythm strip immediately below those leads.

    References

    Ortega R, Mazzini M, Xue K, Espaillat D. Electrocardiographic Monitoring in Adults. New England Journal of Medicine N Engl J Med 2015;372(8).

    El-Sherif N, Turitto G. Ambulatory Electrocardiographic Monitoring between Artifacts and Misinterpretation, Management Errors of Commission and Errors of Omission. Ann Noninvasive Electrocardiol Annals of Noninvasive Electrocardiology 2014;20(3):282–289.

    Dubin D. Rapid Interpretation of EKG’s: an Interactive Course. Tampa, FL: Cover Pub. Co.; 2000.

    Incorrect

    Answer: Sinus rhythm.  This EKG demonstrates a moderate amount of baseline artifact however the QRS complexes are at regular intervals and leads V1 and V2 demonstrate p waves followed by QRS complexes.  These 2 findings exclude atrial fibrillation and flutter.  A Mobitz II block consists of a fixed, prolonged PR interval with an occasional non-conducted P wave resulting in a dropped beat.  There are no dropped beats on this EKG.  The rate is approximately 80bpm which eliminates supraventricular tachycardia (>100bpm).

    Artifacts on an EKG tracing can be due to one or more of several reasons including an expired electrode, poor skin contact, breakdown of the conduction gel in the center of the electrode, tautness of the skin underlying an electrode, and finally, patient movements including breathing, tremors or any other muscle use.  These artifacts can mimic a wide variety of arrhythmias and the importance of a thorough assessment of all EKG leads as well as a complete assessment of the patient including any obvious signs of instability and a manual palpation of the pulse should be conducted when attempting to make a determination between artifact and an actual abnormal finding.  If the patient is stable, any possible arrhythmia on a rhythm strip should be confirmed with a 12-lead EKG.

    When evaluating a 12-lead EKG, if sinus rhythm is not completely apparent, assess all leads for P followed by QRS.  Many times P waves can be more easily appreciated in lead V1.  Also, each column of leads (for example: leads I, II, III on a standard format) represents the same period of time, therefore, any irregularity noted in one lead should also be present in the other two that correspond to the same point in time.  In the case of the attached EKG, P waves are most visible in V1 and V2 and there is no significant artifact which would correspond to the same timeframe as the lead II rhythm strip immediately below those leads.

    References

    Ortega R, Mazzini M, Xue K, Espaillat D. Electrocardiographic Monitoring in Adults. New England Journal of Medicine N Engl J Med 2015;372(8).

    El-Sherif N, Turitto G. Ambulatory Electrocardiographic Monitoring between Artifacts and Misinterpretation, Management Errors of Commission and Errors of Omission. Ann Noninvasive Electrocardiol Annals of Noninvasive Electrocardiology 2014;20(3):282–289.

    Dubin D. Rapid Interpretation of EKG’s: an Interactive Course. Tampa, FL: Cover Pub. Co.; 2000.

  53. Question 53 of 220
    53. Question

    question 3

    A 59 year old female presents to the emergency department with complaints of nausea, vomiting, diarrhea, weakness and dyspnea which started several days prior and has progressively worsened.  Medical history is significant for a CABG 1 month prior, thoracic aneurysm repair 2 years prior and end-stage renal disease requiring hemodialysis however the patient refused dialysis 2 days prior to arrival.  On exam, the patient appears very weak and fatigued and is unable to hold her head up on her own.  She was noted to be mildly tachypneic with bibasilar rales and bradycardia.  The EKG above is obtained.  What is the most likely cause of this patient’s symptoms and abnormal EKG?

    Correct

    Answer:  Hyperkalemia.  The bradycardia, flattened P waves, widened QRS complex, biphasic QRS-T complexes and conduction delay are all findings consistent with hyperkalemia.  Hypercalcemia can cause bradycardia and widened T waves however, it usually causes a shortened QT interval.  Hypermagnesemia EKG changes are usually a result of simultaneous hyperkalemia and not from the elevated magnesium itself.  Hypernatremia is not generally associated with specific EKG changes.

    Hyperkalemia is commonly associated with the phrase “peaked T waves”, however, the condition has been known to cause a multitude of other changes as well.  In addition to the findings mentioned above, hyperkalemia can also cause a first-degree block, bundle branch or atrioventricular blocks, deep S waves, ST elevations, and a merging of the S and T waves which can eventually develop into a since-wave and asystole.

    The “peaked” or “tented” T waves are generally the earliest changes (5.5-6.5 mEq/L) related to hyperkalemia and are a result of an acceleration to the terminal repolarization.  P wave flattening (6.5-7.5 mEq/L) as well as PR and QRS prolongation occurs are next to occur (7-8 mEq/L) as potassium continues to rise and are caused by a depression in conduction between adjacent myocytes.  As potassium continues to rise, the SA and AV node conduction is suppressed which can lead to escape beats, heart block, QRS widening leading to a sine-wave (>8 mEq/L) and eventually ventricular fibrillation or asystole.  These changes vary from individual to individual and are not necessarily present in all cases or at similar potassium levels from one person to another.

    Hypokalemia can also cause EKG changes, including:  low amplitude T wave, ST depression, broad-based U wave formation.  As hypokalemia becomes more profound, ventricular tachycardia, PVC’s, and ventricular fibrillation are possible.

    References:

    2005 American Heart Association Guidelines for Cardiopulmonary Resuscitation and Emergency Cardiovascular Care.  Part 10.1: Life-Threatening Electrolyte Abnormalities. Circulation 2005;112(24_suppl).

    Tintinalli JE, Kelen GD, Stapczynski JS. Emergency Medicine: a Comprehensive Study Guide. New York: McGraw-Hill, Medical Pub. Division; 2004.

    Papadakis, Maxine A., Stephen J. McPhee, and Michael W. Rabow. 2015 Current Medical Diagnosis & Treatment. New York: McGraw-Hill Education/Medical, 2015. Print.

    Webster A. Recognizing signs of danger: ECG changes resulting from an abnormal serum potassium concentration. Emergency Medicine Journal 2002;19(1):74–77.

    Longo DL, Kasper D, Jameson J, Fauci A, Hauser S, Loscalzo J. Harrison’s Principles of Internal Medicine. New York: McGraw-Hill; 2012.

    Incorrect

    Answer:  Hyperkalemia.  The bradycardia, flattened P waves, widened QRS complex, biphasic QRS-T complexes and conduction delay are all findings consistent with hyperkalemia.  Hypercalcemia can cause bradycardia and widened T waves however, it usually causes a shortened QT interval.  Hypermagnesemia EKG changes are usually a result of simultaneous hyperkalemia and not from the elevated magnesium itself.  Hypernatremia is not generally associated with specific EKG changes.

    Hyperkalemia is commonly associated with the phrase “peaked T waves”, however, the condition has been known to cause a multitude of other changes as well.  In addition to the findings mentioned above, hyperkalemia can also cause a first-degree block, bundle branch or atrioventricular blocks, deep S waves, ST elevations, and a merging of the S and T waves which can eventually develop into a since-wave and asystole.

    The “peaked” or “tented” T waves are generally the earliest changes (5.5-6.5 mEq/L) related to hyperkalemia and are a result of an acceleration to the terminal repolarization.  P wave flattening (6.5-7.5 mEq/L) as well as PR and QRS prolongation occurs are next to occur (7-8 mEq/L) as potassium continues to rise and are caused by a depression in conduction between adjacent myocytes.  As potassium continues to rise, the SA and AV node conduction is suppressed which can lead to escape beats, heart block, QRS widening leading to a sine-wave (>8 mEq/L) and eventually ventricular fibrillation or asystole.  These changes vary from individual to individual and are not necessarily present in all cases or at similar potassium levels from one person to another.

    Hypokalemia can also cause EKG changes, including:  low amplitude T wave, ST depression, broad-based U wave formation.  As hypokalemia becomes more profound, ventricular tachycardia, PVC’s, and ventricular fibrillation are possible.

    References:

    2005 American Heart Association Guidelines for Cardiopulmonary Resuscitation and Emergency Cardiovascular Care.  Part 10.1: Life-Threatening Electrolyte Abnormalities. Circulation 2005;112(24_suppl).

    Tintinalli JE, Kelen GD, Stapczynski JS. Emergency Medicine: a Comprehensive Study Guide. New York: McGraw-Hill, Medical Pub. Division; 2004.

    Papadakis, Maxine A., Stephen J. McPhee, and Michael W. Rabow. 2015 Current Medical Diagnosis & Treatment. New York: McGraw-Hill Education/Medical, 2015. Print.

    Webster A. Recognizing signs of danger: ECG changes resulting from an abnormal serum potassium concentration. Emergency Medicine Journal 2002;19(1):74–77.

    Longo DL, Kasper D, Jameson J, Fauci A, Hauser S, Loscalzo J. Harrison’s Principles of Internal Medicine. New York: McGraw-Hill; 2012.

  54. Question 54 of 220
    54. Question

    A 59 year old female presents to the emergency department with complaints of nausea, vomiting, diarrhea, weakness and dyspnea which started several days prior.  She has a history of end-stage renal disease requiring hemodialysis however the patient refused dialysis 2 days prior to arrival.  On exam, the patient appears very weak and fatigued and is unable to hold her head up on her own.  She was noted to be mildly tachypneic with bibasilar rales and bradycardia.  Potassium is 7, CO2 is 17 with an anion gap of 18 and blood sugar is 86.  Which medication is responsible for stabilizing cardiac myocytes during the treatment of hyperkalemia?

    Correct

    Answer:  Calcium gluconate.  All of the answers are appropriate treatments for hyperkalemia, however, calcium gluconate is the only one which is directly responsible for stabilizing the myocytes while initiating treatment to lower the extracellular potassium levels.

    Hyperkalemia is most commonly caused by renal failure or insufficient dialysis once the patient has been  started on chronic dialysis.  Other somewhat common causes include use of potassium-sparing diuretics, aldosterone deficiency, and rhabdomyolysis.  Pseudohyperkalemia is seen with hemolysis during the venipuncture process and can occur during a diabetic ketoacidosis (DKA) event.  During DKA, potassium is shifted into the extracellular space in exchange of hydrogen causing serum levels to be elevated.  However, in patients with good renal function, much of the potassium is excreted in the urine due to the hyperglycemic state causing osmotic diuresis.  This diuresis results in a depleted total body potassium state.

    A patient with hyperkalemia can present with palpitations, nausea/vomiting, myalgias or paresthesias, however, some patients may be asymptomatic if only mild elevations of potassium are present or if levels have gradually increased over a period of time.  Hyperkalemia is classified according to the serum potassium level and not necessarily the degree of symptoms.  Mild hyperkalemia is between 5.5-6.5 mEq/L, moderate is 6.5-7.5 mEq/L, and severe is a potassium greater than 7.5 mEq/L.  As levels increase, various EKG changes can occur.  “Peaked” or “tented” T waves are generally the earliest changes followed by P wave flattening, PR and QRS prolongation occurs are next to occur which can eventually lead to heart block, QRS widening, sine-wave formation and eventually ventricular fibrillation or asystole.

    Treatment of hyperkalemia in the non-DKA patient involves several medications which work on a variety of physiologic levels.  Mild hyperkalemia which is asymptomatic and without accompanying EKG changes may resolve spontaneously if the underlying cause is reversible and addressed.  Moderate to severe hyperkalemia requires both shifting the potassium back into the cells as well as removal of potassium from the body.  This is accomplished with insulin, glucose, albuterol (nebulized or intravenous), furosemide (Lasix), and sodium polystyrene (Kayexalate).  Insulin and glucose should both be provided and the typical doses are 10 units of regular insulin with 50 grams of glucose IV.  Albuterol 2.5mg nebulized can also be provided.  These three treatments will assist in shifting potassium back to the intracellular space.  Sodium bicarbonate 50-100 mEq IV will also promote some intracellular shift but is usually reserved for severe cases with acidosis and it’s efficacy is less predictable.  Furosemide (Lasix) 40mg IV will promote renal excretion of the potassium unless the patient has end-stage renal disease, in which case, dialysis would be needed.  Sodium polystyrene (Kayexalate) 25-50 grams oral or rectal will bind to enteral potassium and prevent it from being resorbed into the system, allowing for it to be excreted from the GI tract.

    Calcium chloride (10%) or calcium gluconate (10%) 10ml IV is necessary to initially stabilize the myocyte membrane potential while other agents are being given time to work.  Calcium gluconate is preferred due to the lack of potential tissue necrosis if the medication infiltrates at the injection site.  The onset of action is 1-3 minutes and the duration of effect can last up to 50 minutes.  Some sources cite using calcium in any patient with EKG changes, others recommend a more conservative use and reserve it for the following conditions:  widened QRS or sine wave, cardiac arrest due to hyperkalemia, or rapid changes in the EKG due to an ongoing process.  Calcium will narrow the QRS and improve bradycardia.  Calcium should be given with extreme caution to a patient taking digitalis (Digoxin) due to the possibility that the hypercalcemia could lead to cardiac tetany (“stone heart”) or ventricular arrhythmias.  Some studies have disproven this association.

    References:

    Longo DL, Kasper D, Jameson J, Fauci A, Hauser S, Loscalzo J. Harrison’s Principles of Internal Medicine. New York: McGraw-Hill; 2012.

    Lehnhardt A, Kemper MJ. Pathogenesis, diagnosis and management of hyperkalemia. Pediatr Nephrol Pediatric Nephrology 2010;26(3):377–384.

    Pfennig C, Whitmore S, Slovis C. ACEP. Focus On… Critical Decisions: Hyperkalemia //. Available at: http://www.acep.org/education/continuing-medical-education-(cme)/focus-on/focus-on—-critical-decisions–hyperkalemia/. Accessed March 16, 2016.

    Levine M, Nikkanen H, Pallin DJ. The Effects of Intravenous Calcium in Patients with Digoxin Toxicity. The Journal of Emergency Medicine 2011;40(1):41–46.

    Incorrect

    Answer:  Calcium gluconate.  All of the answers are appropriate treatments for hyperkalemia, however, calcium gluconate is the only one which is directly responsible for stabilizing the myocytes while initiating treatment to lower the extracellular potassium levels.

    Hyperkalemia is most commonly caused by renal failure or insufficient dialysis once the patient has been  started on chronic dialysis.  Other somewhat common causes include use of potassium-sparing diuretics, aldosterone deficiency, and rhabdomyolysis.  Pseudohyperkalemia is seen with hemolysis during the venipuncture process and can occur during a diabetic ketoacidosis (DKA) event.  During DKA, potassium is shifted into the extracellular space in exchange of hydrogen causing serum levels to be elevated.  However, in patients with good renal function, much of the potassium is excreted in the urine due to the hyperglycemic state causing osmotic diuresis.  This diuresis results in a depleted total body potassium state.

    A patient with hyperkalemia can present with palpitations, nausea/vomiting, myalgias or paresthesias, however, some patients may be asymptomatic if only mild elevations of potassium are present or if levels have gradually increased over a period of time.  Hyperkalemia is classified according to the serum potassium level and not necessarily the degree of symptoms.  Mild hyperkalemia is between 5.5-6.5 mEq/L, moderate is 6.5-7.5 mEq/L, and severe is a potassium greater than 7.5 mEq/L.  As levels increase, various EKG changes can occur.  “Peaked” or “tented” T waves are generally the earliest changes followed by P wave flattening, PR and QRS prolongation occurs are next to occur which can eventually lead to heart block, QRS widening, sine-wave formation and eventually ventricular fibrillation or asystole.

    Treatment of hyperkalemia in the non-DKA patient involves several medications which work on a variety of physiologic levels.  Mild hyperkalemia which is asymptomatic and without accompanying EKG changes may resolve spontaneously if the underlying cause is reversible and addressed.  Moderate to severe hyperkalemia requires both shifting the potassium back into the cells as well as removal of potassium from the body.  This is accomplished with insulin, glucose, albuterol (nebulized or intravenous), furosemide (Lasix), and sodium polystyrene (Kayexalate).  Insulin and glucose should both be provided and the typical doses are 10 units of regular insulin with 50 grams of glucose IV.  Albuterol 2.5mg nebulized can also be provided.  These three treatments will assist in shifting potassium back to the intracellular space.  Sodium bicarbonate 50-100 mEq IV will also promote some intracellular shift but is usually reserved for severe cases with acidosis and it’s efficacy is less predictable.  Furosemide (Lasix) 40mg IV will promote renal excretion of the potassium unless the patient has end-stage renal disease, in which case, dialysis would be needed.  Sodium polystyrene (Kayexalate) 25-50 grams oral or rectal will bind to enteral potassium and prevent it from being resorbed into the system, allowing for it to be excreted from the GI tract.

    Calcium chloride (10%) or calcium gluconate (10%) 10ml IV is necessary to initially stabilize the myocyte membrane potential while other agents are being given time to work.  Calcium gluconate is preferred due to the lack of potential tissue necrosis if the medication infiltrates at the injection site.  The onset of action is 1-3 minutes and the duration of effect can last up to 50 minutes.  Some sources cite using calcium in any patient with EKG changes, others recommend a more conservative use and reserve it for the following conditions:  widened QRS or sine wave, cardiac arrest due to hyperkalemia, or rapid changes in the EKG due to an ongoing process.  Calcium will narrow the QRS and improve bradycardia.  Calcium should be given with extreme caution to a patient taking digitalis (Digoxin) due to the possibility that the hypercalcemia could lead to cardiac tetany (“stone heart”) or ventricular arrhythmias.  Some studies have disproven this association.

    References:

    Longo DL, Kasper D, Jameson J, Fauci A, Hauser S, Loscalzo J. Harrison’s Principles of Internal Medicine. New York: McGraw-Hill; 2012.

    Lehnhardt A, Kemper MJ. Pathogenesis, diagnosis and management of hyperkalemia. Pediatr Nephrol Pediatric Nephrology 2010;26(3):377–384.

    Pfennig C, Whitmore S, Slovis C. ACEP. Focus On… Critical Decisions: Hyperkalemia //. Available at: http://www.acep.org/education/continuing-medical-education-(cme)/focus-on/focus-on—-critical-decisions–hyperkalemia/. Accessed March 16, 2016.

    Levine M, Nikkanen H, Pallin DJ. The Effects of Intravenous Calcium in Patients with Digoxin Toxicity. The Journal of Emergency Medicine 2011;40(1):41–46.

  55. Question 55 of 220
    55. Question

    image 5
    A 23 year old male presents to the ED complaining of chest pain which woke him up at 5am.  He rates it as 10/10 substernal sharp pain which is exacerbated with lying supine.  The patient denies any recent trauma or similar prior pains but does report having a mild, nonproductive cough and nasal congestion over the past 3-4 days.  Denies fevers.  Past medical history is significant for a pulmonary embolism 5 years ago.  It is uncertain what the underlying risk factor for developing the embolism was.  Physical exam reveals a patient in moderate discomfort but no respiratory distress with a regular rate and rhythm and no murmurs.  Lungs are clear to auscultation.  There is some tenderness to palpation of the epigastric area and along the lower L sternal boarder.  Extremities are nontender, w/o edema.  The EKG above is obtained.  Based on this information, what is the most likely cause of the patient’s chest pain?

    Correct

    Answer:  Pericarditis.  The pt. has a history consistent with a recent viral illness and the report of increasing pains with lying supine, thus improved with sitting up, leaning forward are classic findings in a patient with pericarditis.  While subtle, the EKG does show diffuse ST elevations consistent with pericarditis.  Costochondritis is usually much more reproducible with palpation of the chest wall and is less likely to improve with leaning forward.  Pleuritis would be diagnosed with the presence of pleural rubs on auscultation.  Neither costochondritis nor pleuritis should be associated with EKG changes.  Pulmonary embolus is less likely to be positional, is generally associated with tachycardia and if present, would have different EKG changes.  ST elevation MI by definition is 1mm or more of ST segment elevation in 2 contiguous leads.  This EKG does not meet that criteria.

    The pericardium is a 3-layered fibrous and elastic structure surrounding the heart.  Between the innermost (visceral) layer and the middle (parietal) pericardium is the pericardial cavity, or sack, which usually contains 5-15ml of fluid.  The purpose of the pericardium is to contain the myocardium, restrict chamber dilation, and to minimize friction and transference of infection between the heart, lungs, and vessels.  Inflammation of this structure, referred to as pericarditis, is most often due to viral etiologies.  Other causes include:  bacterial, fungal, tuberculous, recent myocardial infarction (Dressler’s syndrome), uremia, systemic inflammatory conditions, cancer, radiation, cardiac procedures and trauma.  The majority of cases are found in males less than 50 years of age and cases can be both acute and chronic.

    The classic history of pericarditis is a sharp, stabbing, substernal pain which is usually pleuritic and made significantly worse by lying supine, relieved with sitting forward.  Some patients will report shortness of breath as well.  Pertinent history questioning should inquire about recent illness, trauma, MI, or insect bites (including ticks).  In addition, a past medical history of cancer or renal failure, as well as, autoimmune and inflammatory disorders should be sought out.

    The most common physical exam findings is a pericardial friction rub, occurring in approximately 85% of these patients.  Pericardial effusions develop in approximately 75% of these patients which can cause diminished heart sounds and tachycardia.  Tenderness along the sternal boarder or with epigastric palpation can also occur.

    A complaint of “substernal chest pain” dictates obtaining an EKG.  With pericarditis, the EKG can go through 4 phases over the course of several weeks.  Phase 1 is diffuse ST elevations and PR depressions.  Phase 2 is a return to normal, followed by T wave inversions (phase 3) and finally another return to normal however it may take several weeks to months for the T inversions to resolve.  In patients with an associated pericardial effusion, low voltage will be present.  Laboratory tests may reveal elevations of CK-MB and troponin which is then diagnosed as myopericarditis.  In the patient above, the initial troponin was 2.98 with a peak of 16.7.  Additional labs that can be helpful in the workup if there is not a clear-cut viral or traumatic etiology are: CBC, chemistry, sed rate, C reactive protein, anti-nuclear antibody, rheumatoid factor and PPD.  A chest x-ray may reveal cardiomegaly if a moderate effusion is present.

    Treatment for viral etiologies includes rest and NSAIDS, including indomethacin.  Prednisone can also be beneficial for refractory cases or those secondary to connective tissue disorders.  Other potential etiologies should be addressed appropriately if the history is inconsistent with a viral cause.  Complications can include a chronic pericardial effusion, constrictive pericarditis or pericardial tamponade.

    References:

    Longo DL, Kasper D, Jameson J, Fauci A, Hauser S, Loscalzo J. Harrison’s Principles of Internal Medicine. New York: McGraw-Hill; 2012.

    Tintinalli JE, Kelen GD, Stapczynski JS. Emergency Medicine: a Comprehensive Study Guide. New York: McGraw-Hill, Medical Pub. Division; 2004.

    Papadakis, Maxine A., Stephen J. McPhee, and Michael W. Rabow. 2015 Current Medical Diagnosis & Treatment. New York: McGraw-Hill Education/Medical, 2015. Print.

    Andreeff, Renee A. MS, MPAS, PA-C. Quick Recertification Series:  Acute Pericarditis. Journal of the American Academy of Physician Assistants 2014;27(3):49-50.

    Khandaker MH, Espinosa RE, Nishimura RA, et al. Pericardial Disease: Diagnosis and Management. Mayo Clinic Proceedings 2010;85(6):572–593.

    Incorrect

    Answer:  Pericarditis.  The pt. has a history consistent with a recent viral illness and the report of increasing pains with lying supine, thus improved with sitting up, leaning forward are classic findings in a patient with pericarditis.  While subtle, the EKG does show diffuse ST elevations consistent with pericarditis.  Costochondritis is usually much more reproducible with palpation of the chest wall and is less likely to improve with leaning forward.  Pleuritis would be diagnosed with the presence of pleural rubs on auscultation.  Neither costochondritis nor pleuritis should be associated with EKG changes.  Pulmonary embolus is less likely to be positional, is generally associated with tachycardia and if present, would have different EKG changes.  ST elevation MI by definition is 1mm or more of ST segment elevation in 2 contiguous leads.  This EKG does not meet that criteria.

    The pericardium is a 3-layered fibrous and elastic structure surrounding the heart.  Between the innermost (visceral) layer and the middle (parietal) pericardium is the pericardial cavity, or sack, which usually contains 5-15ml of fluid.  The purpose of the pericardium is to contain the myocardium, restrict chamber dilation, and to minimize friction and transference of infection between the heart, lungs, and vessels.  Inflammation of this structure, referred to as pericarditis, is most often due to viral etiologies.  Other causes include:  bacterial, fungal, tuberculous, recent myocardial infarction (Dressler’s syndrome), uremia, systemic inflammatory conditions, cancer, radiation, cardiac procedures and trauma.  The majority of cases are found in males less than 50 years of age and cases can be both acute and chronic.

    The classic history of pericarditis is a sharp, stabbing, substernal pain which is usually pleuritic and made significantly worse by lying supine, relieved with sitting forward.  Some patients will report shortness of breath as well.  Pertinent history questioning should inquire about recent illness, trauma, MI, or insect bites (including ticks).  In addition, a past medical history of cancer or renal failure, as well as, autoimmune and inflammatory disorders should be sought out.

    The most common physical exam findings is a pericardial friction rub, occurring in approximately 85% of these patients.  Pericardial effusions develop in approximately 75% of these patients which can cause diminished heart sounds and tachycardia.  Tenderness along the sternal boarder or with epigastric palpation can also occur.

    A complaint of “substernal chest pain” dictates obtaining an EKG.  With pericarditis, the EKG can go through 4 phases over the course of several weeks.  Phase 1 is diffuse ST elevations and PR depressions.  Phase 2 is a return to normal, followed by T wave inversions (phase 3) and finally another return to normal however it may take several weeks to months for the T inversions to resolve.  In patients with an associated pericardial effusion, low voltage will be present.  Laboratory tests may reveal elevations of CK-MB and troponin which is then diagnosed as myopericarditis.  In the patient above, the initial troponin was 2.98 with a peak of 16.7.  Additional labs that can be helpful in the workup if there is not a clear-cut viral or traumatic etiology are: CBC, chemistry, sed rate, C reactive protein, anti-nuclear antibody, rheumatoid factor and PPD.  A chest x-ray may reveal cardiomegaly if a moderate effusion is present.

    Treatment for viral etiologies includes rest and NSAIDS, including indomethacin.  Prednisone can also be beneficial for refractory cases or those secondary to connective tissue disorders.  Other potential etiologies should be addressed appropriately if the history is inconsistent with a viral cause.  Complications can include a chronic pericardial effusion, constrictive pericarditis or pericardial tamponade.

    References:

    Longo DL, Kasper D, Jameson J, Fauci A, Hauser S, Loscalzo J. Harrison’s Principles of Internal Medicine. New York: McGraw-Hill; 2012.

    Tintinalli JE, Kelen GD, Stapczynski JS. Emergency Medicine: a Comprehensive Study Guide. New York: McGraw-Hill, Medical Pub. Division; 2004.

    Papadakis, Maxine A., Stephen J. McPhee, and Michael W. Rabow. 2015 Current Medical Diagnosis & Treatment. New York: McGraw-Hill Education/Medical, 2015. Print.

    Andreeff, Renee A. MS, MPAS, PA-C. Quick Recertification Series:  Acute Pericarditis. Journal of the American Academy of Physician Assistants 2014;27(3):49-50.

    Khandaker MH, Espinosa RE, Nishimura RA, et al. Pericardial Disease: Diagnosis and Management. Mayo Clinic Proceedings 2010;85(6):572–593.

  56. Question 56 of 220
    56. Question

    image 6
    A 58 year old female complains of dyspnea over the past several days, increasing with lying supine.  She also reports episodic nausea and swelling of both legs but denies chest pain, fever/chills, or cough.  Past medical history is significant for:  cirrhosis, type 2 DM, sleep apnea, gastroparesis and chronic back pain.  Physical exam reveals a middle aged woman with noticeable dyspnea at rest, able to speak only a few words before requiring a breath.  There is no audible wheezing or stridor.  Breath sounds in the right mid to lower lung field are significantly diminished with normal breath sounds on the left.  Abdomen is soft, nontender and without ascites.  What does the above lateral x-ray reveal to be the cause of the patient’s dyspnea?

    Correct

    Answer:  Pleural effusion.  There is opacification from the base to the mid-lung fields well delineated with a horizontal cutoff and darker (normal) lung above.

    Pleural effusions are accumulation of fluid within the pleural space secondary to either an abnormality of absorption by the pleural lymphatics or by excess production via the pleural capillaries.  Cirrhosis leads to an increase in fluid production either by causing a decrease in oncotic pressure or by migration of fluid from ascites through the lymphatic system.  Other etiologies of pleural effusions are:  congestive heart failure, pneumonia, uremia, pancreatitis, ARDS, TB, malignancy and trauma to the thoracic duct (Chylothorax) or a blood vessel (hemothorax).

    The patient’s symptoms are generally related to the length of time that the effusion takes to develop.  A slow developing effusion allows for compensation whereas a rapid accumulation results in acute shortness of breath.  Effusions can also be associated with pleuritic pain, dry cough and even fever depending on the underlying etiology.  Physical exam will reveal diminished breath sounds over the affected lung base, depending on the size of the effusion.  Dullness to percussion and decreased tactile fremitus are also present depending on the size of the effusion.  Tachypnea and hypoxia will be present with large or rapidly accumulating effusions.

    A chest x-ray is the most commonly used modality to diagnose a pleural effusion.  As little as 50ml of fluid can be visualized on the lateral x-ray compared to a minimum of 175-200ml being needed to be visible on a frontal view.  The first indication of an effusion is blunting of the normally sharp costophrenic angle(s).  Poor demarcation of the hemi-diaphragm and atelectasis may also be present.  A lateral decubitus film may be useful to distinguish between loculated and free flowing effusions.  Ultrasound can detect as little as 3-5ml of pleural fluid but is usually reserved for the treatment phase.

    Treatment may involve observation if small and asymptomatic however for symptom relief or to assist in diagnosis a thoracentesis is generally performed and the fluid obtained can be sent for analysis.  Lights criteria can aid in the determination between exudative and transudative fluids.  Exudative fluids must meet at least one of the following characteristics:  pleural fluid protein to serum protein ration > 0.5, or pleural fluid LDH to serum LDH ration of > 0.6, or pleural fluid LDH > 2/3rds of the upper limit of normal serum LDH.  Other treatment options include thoracostomy or pleurodesis for frequent reoccurrences.

    The patient in this case has required bi-weekly thoracentesis for some time and frequently has approximately 2L removed.  Despite this rather large amount of fluid, her PO2 was 100% RA.

    References:

    Longo DL, Kasper D, Jameson J, Fauci A, Hauser S, Loscalzo J. Harrison’s Principles of Internal Medicine. New York: McGraw-Hill; 2012.

    Papadakis, Maxine A., Stephen J. McPhee, and Michael W. Rabow. 2015 Current Medical Diagnosis & Treatment. New York: McGraw-Hill Education/Medical, 2015. Print.

    Pleural Effusion. In DynaMed Plus, EBSCO Information Services (Accessed April 1, 2016)

    Incorrect

    Answer:  Pleural effusion.  There is opacification from the base to the mid-lung fields well delineated with a horizontal cutoff and darker (normal) lung above.

    Pleural effusions are accumulation of fluid within the pleural space secondary to either an abnormality of absorption by the pleural lymphatics or by excess production via the pleural capillaries.  Cirrhosis leads to an increase in fluid production either by causing a decrease in oncotic pressure or by migration of fluid from ascites through the lymphatic system.  Other etiologies of pleural effusions are:  congestive heart failure, pneumonia, uremia, pancreatitis, ARDS, TB, malignancy and trauma to the thoracic duct (Chylothorax) or a blood vessel (hemothorax).

    The patient’s symptoms are generally related to the length of time that the effusion takes to develop.  A slow developing effusion allows for compensation whereas a rapid accumulation results in acute shortness of breath.  Effusions can also be associated with pleuritic pain, dry cough and even fever depending on the underlying etiology.  Physical exam will reveal diminished breath sounds over the affected lung base, depending on the size of the effusion.  Dullness to percussion and decreased tactile fremitus are also present depending on the size of the effusion.  Tachypnea and hypoxia will be present with large or rapidly accumulating effusions.

    A chest x-ray is the most commonly used modality to diagnose a pleural effusion.  As little as 50ml of fluid can be visualized on the lateral x-ray compared to a minimum of 175-200ml being needed to be visible on a frontal view.  The first indication of an effusion is blunting of the normally sharp costophrenic angle(s).  Poor demarcation of the hemi-diaphragm and atelectasis may also be present.  A lateral decubitus film may be useful to distinguish between loculated and free flowing effusions.  Ultrasound can detect as little as 3-5ml of pleural fluid but is usually reserved for the treatment phase.

    Treatment may involve observation if small and asymptomatic however for symptom relief or to assist in diagnosis a thoracentesis is generally performed and the fluid obtained can be sent for analysis.  Lights criteria can aid in the determination between exudative and transudative fluids.  Exudative fluids must meet at least one of the following characteristics:  pleural fluid protein to serum protein ration > 0.5, or pleural fluid LDH to serum LDH ration of > 0.6, or pleural fluid LDH > 2/3rds of the upper limit of normal serum LDH.  Other treatment options include thoracostomy or pleurodesis for frequent reoccurrences.

    The patient in this case has required bi-weekly thoracentesis for some time and frequently has approximately 2L removed.  Despite this rather large amount of fluid, her PO2 was 100% RA.

    References:

    Longo DL, Kasper D, Jameson J, Fauci A, Hauser S, Loscalzo J. Harrison’s Principles of Internal Medicine. New York: McGraw-Hill; 2012.

    Papadakis, Maxine A., Stephen J. McPhee, and Michael W. Rabow. 2015 Current Medical Diagnosis & Treatment. New York: McGraw-Hill Education/Medical, 2015. Print.

    Pleural Effusion. In DynaMed Plus, EBSCO Information Services (Accessed April 1, 2016)

  57. Question 57 of 220
    57. Question

    A 27 year old male presents to the emergency department complaining of generalized weakness, fever, cough, shortness of breath and myalgias.  He states his symptoms started 5-6 days ago and have gradually worsened to the point where he is so weak that he can barely ambulate.  He reports that his girlfriend has been admitted to a local hospital for over a week with similar symptoms.  Review of systems is positive for non-bloody vomiting and diarrhea but none over the past 24 hours.  Medical history is significant for Hepatitis B and he admits to using IV heroin daily.  Vitals include a blood pressure of 133/81, pulse 120, respiratory rate of 22, pulse oximetry 96% and an oral temp of 102°F.  Other than appearing fatigued and having a regular tachycardia, his exam is unremarkable including no nuchal rigidity or skin abscesses.  Pertinent labs are as follows:  WBC 16.1 with 9 bands, sodium 121, lactate 1.8, flu negative, CXR negative, and urinalysis negative.  What is the most likely microbiologic etiology to grow out in this patient’s blood cultures?

    Correct

    Answer:  Staphylococci aureus.  Given the patient’s history of IV drug use and lack of an apparent site of infection, endocarditis should be at the top of the differential diagnosis.  While all of these pathogens can be responsible for endocarditis, Staph aureus is by far the most common cause in an IV drug abuser, accounting for approximately 68% of cases.  Listeria risk factors include pregnancy, elderly or immunocompromised, and those patients with a history of an underlying valvular or structural abnormality.  Enterococci endocarditis has been associated with patients recently undergoing colonoscopies or dental procedures, especially in the presence of an underlying prosthetic valve.  Candida generally presents with a slow progression of symptoms and risk factors include:  indwelling catheters, artificial valves, pacemakers, IV drug abusers and immunosuppressive states.

    Endocarditis is refers to an infectious lesion which develops most commonly on a heart valve but can also occur on ventricular septal defect or on intracardiac devices.  These lesions consist of an infectious component, platelets, fibrin and inflammatory cells.  The elderly and those with prosthetic valves are at the highest risk due to some degree of damaged endothelium.  The risk of prosthetic valve infection is greatest during the first 6-12 months after replacement and then gradually declines to a relatively low rate.  Whether mechanical or bioprosthetic, the rate of infection is roughly the same.

    Symptoms of endocarditis can be vague and progress over several days to weeks.  Fever occurs in 80-90% of patients and a new murmur is found in nearly as many.  Other common symptoms include myalgias, arthralgias, fatigue, and malaise.  Less common findings include Osler’s nodes, Janeway lesions, Roth’s spots and splinter hemorrhages.  Roth’s spots are retinal hemorrhages and splinter hemorrhages are found under the nails.  Osler’s nodes are painful, erythematous nodules of the pads of fingers or toes whereas Janeway lesions are non-painful, erythematous nodules of the palms or soles.

    The definitive diagnosis of endocarditis is made when vegetative endocardial lesions are located and examined.  In the absence of vegetative lesion examination, Duke criteria is commonly used when endocarditis is high on the differential.  It consists of two major criteria and five minor.  Major criteria include positive blood cultures in a patient with no other source and either a positive echocardiogram or a new valvular regurgitation.  Minor criteria include the following:  underlying risk factors, fever > 100.4°F, vascular findings consistent with endocarditis, immunologic findings consistent with endocarditis, and positive blood cultures which fall outside of the more specific major criteria blood culture specifics which are not mentioned here.  A patient is considered to have endocarditis if both major criteria have been met, one major and three minor, or all five minor criteria are positive.

    Treatment consists of appropriate intravenous antimicrobial therapy either started empirically or based on culture sensitivities.  Penicillin G, ceftriaxone, vancomycin, and nafcillin or oxacillin are the most commonly used antibiotics.  Surgical removal of lesions may be required for more complicated patients.  The patient mentioned above was started on vancomycin and cefepime at the advice of the infectious disease physician and the following day an echocardiogram revealed a 2cm X 1cm tricuspid valve lesion, the most common site of endocarditis in an IV drug abuser.

    References:

    Summa C, Walker SAN. Endocarditis Due to Listeria monocytogenes in an Academic Teaching Hospital: Case Report. CJHP The Canadian Journal of Hospital Pharmacy 2010;63(4). doi:10.4212/cjhp.v63i4.935.

    Sexton, D, Chu, V. Infective endocarditis in injection drug users. In: UpToDate, Post TW (Ed), UpToDate, Waltham, MA. (Accessed on April 11, 2016.)

    Prasad, P.  Infective endocarditis.  In:  DynaMed Plus, EBSCO Information Services (Accessed April 13, 2016)

    Marle MGD. Infective endocarditis from Enterococcus faecalis complicating colonoscopy in Heyde’s syndrome. Postgraduate Medical Journal 2004;80(948):619–620. doi:10.1136/pgmj.2004.019711.

    Urbano, F.  Peripheral Signs of Endocarditis.  Hospital Physician 2000:41-46

    Longo DL, Kasper D, Jameson J, Fauci A, Hauser S, Loscalzo J. Harrison’s Principles of Internal Medicine. New York: McGraw-Hill; 2012.

    Incorrect

    Answer:  Staphylococci aureus.  Given the patient’s history of IV drug use and lack of an apparent site of infection, endocarditis should be at the top of the differential diagnosis.  While all of these pathogens can be responsible for endocarditis, Staph aureus is by far the most common cause in an IV drug abuser, accounting for approximately 68% of cases.  Listeria risk factors include pregnancy, elderly or immunocompromised, and those patients with a history of an underlying valvular or structural abnormality.  Enterococci endocarditis has been associated with patients recently undergoing colonoscopies or dental procedures, especially in the presence of an underlying prosthetic valve.  Candida generally presents with a slow progression of symptoms and risk factors include:  indwelling catheters, artificial valves, pacemakers, IV drug abusers and immunosuppressive states.

    Endocarditis is refers to an infectious lesion which develops most commonly on a heart valve but can also occur on ventricular septal defect or on intracardiac devices.  These lesions consist of an infectious component, platelets, fibrin and inflammatory cells.  The elderly and those with prosthetic valves are at the highest risk due to some degree of damaged endothelium.  The risk of prosthetic valve infection is greatest during the first 6-12 months after replacement and then gradually declines to a relatively low rate.  Whether mechanical or bioprosthetic, the rate of infection is roughly the same.

    Symptoms of endocarditis can be vague and progress over several days to weeks.  Fever occurs in 80-90% of patients and a new murmur is found in nearly as many.  Other common symptoms include myalgias, arthralgias, fatigue, and malaise.  Less common findings include Osler’s nodes, Janeway lesions, Roth’s spots and splinter hemorrhages.  Roth’s spots are retinal hemorrhages and splinter hemorrhages are found under the nails.  Osler’s nodes are painful, erythematous nodules of the pads of fingers or toes whereas Janeway lesions are non-painful, erythematous nodules of the palms or soles.

    The definitive diagnosis of endocarditis is made when vegetative endocardial lesions are located and examined.  In the absence of vegetative lesion examination, Duke criteria is commonly used when endocarditis is high on the differential.  It consists of two major criteria and five minor.  Major criteria include positive blood cultures in a patient with no other source and either a positive echocardiogram or a new valvular regurgitation.  Minor criteria include the following:  underlying risk factors, fever > 100.4°F, vascular findings consistent with endocarditis, immunologic findings consistent with endocarditis, and positive blood cultures which fall outside of the more specific major criteria blood culture specifics which are not mentioned here.  A patient is considered to have endocarditis if both major criteria have been met, one major and three minor, or all five minor criteria are positive.

    Treatment consists of appropriate intravenous antimicrobial therapy either started empirically or based on culture sensitivities.  Penicillin G, ceftriaxone, vancomycin, and nafcillin or oxacillin are the most commonly used antibiotics.  Surgical removal of lesions may be required for more complicated patients.  The patient mentioned above was started on vancomycin and cefepime at the advice of the infectious disease physician and the following day an echocardiogram revealed a 2cm X 1cm tricuspid valve lesion, the most common site of endocarditis in an IV drug abuser.

    References:

    Summa C, Walker SAN. Endocarditis Due to Listeria monocytogenes in an Academic Teaching Hospital: Case Report. CJHP The Canadian Journal of Hospital Pharmacy 2010;63(4). doi:10.4212/cjhp.v63i4.935.

    Sexton, D, Chu, V. Infective endocarditis in injection drug users. In: UpToDate, Post TW (Ed), UpToDate, Waltham, MA. (Accessed on April 11, 2016.)

    Prasad, P.  Infective endocarditis.  In:  DynaMed Plus, EBSCO Information Services (Accessed April 13, 2016)

    Marle MGD. Infective endocarditis from Enterococcus faecalis complicating colonoscopy in Heyde’s syndrome. Postgraduate Medical Journal 2004;80(948):619–620. doi:10.1136/pgmj.2004.019711.

    Urbano, F.  Peripheral Signs of Endocarditis.  Hospital Physician 2000:41-46

    Longo DL, Kasper D, Jameson J, Fauci A, Hauser S, Loscalzo J. Harrison’s Principles of Internal Medicine. New York: McGraw-Hill; 2012.

  58. Question 58 of 220
    58. Question

    An 86 year old male presents to the ER with a 5 hour history of having intermittent episodes “head pressure” and a “warming sensation” down both arms.  He denies chest pain, nausea, vomiting, LOC, or rapid palpitations.  His baseline rhythm is atrial fibrillation but while monitored in the emergency department for several hours, it is noted that he has several episodes of 8-9 second pauses and it is during these episodes that he experiences his symptoms.  He is advised that pacemaker placement is necessary.  Which type of device should this patient have?

    Correct

    Correct answer is B; V-paced pacer should be installed. Answer A is incorrect because AV pacing would be indicated if the patient’s underlying rhythm was sinus, not AF. 2Answer B is correct because of the patient’s underlying rhythm of AF. The AV conduction is not intact which would indicate V-pacing. 2 Answer C is incorrect because ICD’s are used for tachydysrhythmias such as ventricular tachycardia or ventricular fibrillation which this patient does not have. A pace-maker is indicated which differs from an ICD. 2 Answer D is incorrect because AV conduction is not intact during AF with pause, which is indicated for A-pacing. 2

    Discussion:

    Atrial fibrillation (AF) is defined as disorganized, irregular and rapid atrial activation resulting in loss of effective atrial contractility and an irregularly irregular ventricular response.2 AF is the most common sustained arrhythmia, see Figure 14. When looking at an EKG you will see a lack of discrete P waves and an irregularly irregular ventricular response (QRS complex). Etiology includes heart failure, hypertension, coronary artery disease, valvular disease, acute hyperthyroidism, acute alcohol intoxication, atrial enlargement, post-operative after a major surgery. AF is seen more in the age group over 65, men verse women, and Whites more than Blacks, Hispanics, or Asians. There are different types of AF, paroxysmal (returning to normal baseline within 7 days), persistent (lasting longer than 7 days and will require treatment), permanent (condition lasting indefinitely, and no further attempts to restore normal rhythm have been discussed). 1

    Symptoms vary in AF and include, fatigue, dizziness, SOB, weakness, rapid and irregular heartbeat, thumping in chest, and chest pain/pressure.2,3 Taking a full cardiac history of any patient suspected with AF is very important. Knowing all medications, heart disease, valvular disease, alcohol use are all important questions to ask. Upon examination, watching the vital signs, especially heart rate is extremely important. Patients will have an irregularly irregular pulse and commonly be tachycardiac. There may be a displaced PMI or S3 which would suggest ventricular enlargement and elevated LV pressure. Head and neck exam may show elevated JVP, cyanosis, carotid bruits, thyromegaly, or exophthalmos. Heart failure may be presents with rales and pleural effusions.2

    To diagnosis AF, an EKG is the golden standard. An EKG will reveal an irregularly irregular rhythm, no discrete P waves and narrow QRS complexes. An echocardiogram may be ordered to show any structural problems. CBC, BMP, TSH are also usually ordered to see if there is an underlying cause. Heart monitors can also be worn by the patient if they go in and out of AF to try and see is there is a cause correlated. 1,2,3

    Treatment and management for AF has three goals: control the rate, restore sinus rhythm (if possible), and decrease risk of clot formation which could lead to CVA.2 Pharmacological treatments include calcium channel blockers, beta blockers, and amiodarone for rate control. Amiodarone is used in refractory cases and better for those in heart failure. Chemical cardioversion using IV Ibutilide has seen to be about 50% successful.2,3 Direct current cardioversion is used in patient with an onset of >48 hours to restore a normal sinus rhythm.1,3 Cardiac ablation is also an option. During this nonsurgical procedure, lesions are destroyed, usually around the pulmonary veins, which trigger the abnormal heart rhythm thus creating an entrance and exit block from triggering foci. To help reduce the risk of a CVA warfarin or newer anticoagulants (in non-valvular AF) are used. The CHADS2 and CHA2DS2-VASc are tools to help determine a patient’s risk of developing a clot who have AF, by simply answering yes or no to certain risk factors. 3

    References:

    1 Brady WJ, Laughrey TS, Ghaemmaghami CA. Cardiac Rhythm Disturbances. In: Tintinalli JE, Stapczynski J, Ma O, Yealy DM, Meckler GD, Cline DM. eds. Tintinalli’s Emergency Medicine: A Comprehensive Study Guide, 8e. New York, NY: McGraw-Hill; 2016. http://accessmedicine.mhmedical.com.ezproxymcp.flo.org/content.aspx?bookid=1658&Sectionid=109426644. Accessed April 11, 2016.

    2 Prystowsky EN, Padanilam BJ, Waldo AL. Chapter 40. Atrial Fibrillation, Atrial Flutter, and Atrial Tachycardia. In: Fuster V, Walsh RA, Harrington RA. eds. Hurst’s The Heart, 13e. New York, NY: McGraw-Hill; 2011. http://accessmedicine.mhmedical.com.ezproxymcp.flo.org/content.aspx?bookid=376&Sectionid=40279769. Accessed April 11, 2016.

    3 Scheinman MM. Chapter 12. Atrial Fibrillation. In: Crawford MH. eds. Current Diagnosis & Treatment: Cardiology, 4e. New York, NY: McGraw-Hill; 2014. http://accessmedicine.mhmedical.com.ezproxymcp.flo.org/content.aspx?bookid=715&Sectionid=48214544. Accessed April 11, 2016.

    Incorrect

    Correct answer is B; V-paced pacer should be installed. Answer A is incorrect because AV pacing would be indicated if the patient’s underlying rhythm was sinus, not AF. 2Answer B is correct because of the patient’s underlying rhythm of AF. The AV conduction is not intact which would indicate V-pacing. 2 Answer C is incorrect because ICD’s are used for tachydysrhythmias such as ventricular tachycardia or ventricular fibrillation which this patient does not have. A pace-maker is indicated which differs from an ICD. 2 Answer D is incorrect because AV conduction is not intact during AF with pause, which is indicated for A-pacing. 2

    Discussion:

    Atrial fibrillation (AF) is defined as disorganized, irregular and rapid atrial activation resulting in loss of effective atrial contractility and an irregularly irregular ventricular response.2 AF is the most common sustained arrhythmia, see Figure 14. When looking at an EKG you will see a lack of discrete P waves and an irregularly irregular ventricular response (QRS complex). Etiology includes heart failure, hypertension, coronary artery disease, valvular disease, acute hyperthyroidism, acute alcohol intoxication, atrial enlargement, post-operative after a major surgery. AF is seen more in the age group over 65, men verse women, and Whites more than Blacks, Hispanics, or Asians. There are different types of AF, paroxysmal (returning to normal baseline within 7 days), persistent (lasting longer than 7 days and will require treatment), permanent (condition lasting indefinitely, and no further attempts to restore normal rhythm have been discussed). 1

    Symptoms vary in AF and include, fatigue, dizziness, SOB, weakness, rapid and irregular heartbeat, thumping in chest, and chest pain/pressure.2,3 Taking a full cardiac history of any patient suspected with AF is very important. Knowing all medications, heart disease, valvular disease, alcohol use are all important questions to ask. Upon examination, watching the vital signs, especially heart rate is extremely important. Patients will have an irregularly irregular pulse and commonly be tachycardiac. There may be a displaced PMI or S3 which would suggest ventricular enlargement and elevated LV pressure. Head and neck exam may show elevated JVP, cyanosis, carotid bruits, thyromegaly, or exophthalmos. Heart failure may be presents with rales and pleural effusions.2

    To diagnosis AF, an EKG is the golden standard. An EKG will reveal an irregularly irregular rhythm, no discrete P waves and narrow QRS complexes. An echocardiogram may be ordered to show any structural problems. CBC, BMP, TSH are also usually ordered to see if there is an underlying cause. Heart monitors can also be worn by the patient if they go in and out of AF to try and see is there is a cause correlated. 1,2,3

    Treatment and management for AF has three goals: control the rate, restore sinus rhythm (if possible), and decrease risk of clot formation which could lead to CVA.2 Pharmacological treatments include calcium channel blockers, beta blockers, and amiodarone for rate control. Amiodarone is used in refractory cases and better for those in heart failure. Chemical cardioversion using IV Ibutilide has seen to be about 50% successful.2,3 Direct current cardioversion is used in patient with an onset of >48 hours to restore a normal sinus rhythm.1,3 Cardiac ablation is also an option. During this nonsurgical procedure, lesions are destroyed, usually around the pulmonary veins, which trigger the abnormal heart rhythm thus creating an entrance and exit block from triggering foci. To help reduce the risk of a CVA warfarin or newer anticoagulants (in non-valvular AF) are used. The CHADS2 and CHA2DS2-VASc are tools to help determine a patient’s risk of developing a clot who have AF, by simply answering yes or no to certain risk factors. 3

    References:

    1 Brady WJ, Laughrey TS, Ghaemmaghami CA. Cardiac Rhythm Disturbances. In: Tintinalli JE, Stapczynski J, Ma O, Yealy DM, Meckler GD, Cline DM. eds. Tintinalli’s Emergency Medicine: A Comprehensive Study Guide, 8e. New York, NY: McGraw-Hill; 2016. http://accessmedicine.mhmedical.com.ezproxymcp.flo.org/content.aspx?bookid=1658&Sectionid=109426644. Accessed April 11, 2016.

    2 Prystowsky EN, Padanilam BJ, Waldo AL. Chapter 40. Atrial Fibrillation, Atrial Flutter, and Atrial Tachycardia. In: Fuster V, Walsh RA, Harrington RA. eds. Hurst’s The Heart, 13e. New York, NY: McGraw-Hill; 2011. http://accessmedicine.mhmedical.com.ezproxymcp.flo.org/content.aspx?bookid=376&Sectionid=40279769. Accessed April 11, 2016.

    3 Scheinman MM. Chapter 12. Atrial Fibrillation. In: Crawford MH. eds. Current Diagnosis & Treatment: Cardiology, 4e. New York, NY: McGraw-Hill; 2014. http://accessmedicine.mhmedical.com.ezproxymcp.flo.org/content.aspx?bookid=715&Sectionid=48214544. Accessed April 11, 2016.

  59. Question 59 of 220
    59. Question

    A 29 year old woman presents to the clinic this morning with sudden onset right-sided chest pain and mild shortness of breath after waking. She is a well appearing, athletic woman with no other complaints. She denies recent trauma, recent illness, cough, recent extended travel or history of DVT. She is a non-smoker and is not currently on any medications including OCP.   Her vital signs are: T 98.6 HR 80, BP 120/80, RR 18 and her O2 sat is 99% on room air. Chest x-ray and EKG are normal. Which algorithm would be the first choice to use if PE is in your differential diagnosis?

    Correct

     Answer: D. The PERC Criteria, developed in 2008, should be the first choice to guide your medical decision-making regarding a possible PE. It involves eight questions: Age >50?, HR >100?, SaO2 <95%?, hemoptysis?, estrogen use?, surgery/trauma w/hospitalization in last 4wks?, prior VTE?, and unilateral leg swelling?

    The PERC Criteria does not give specific guidance about when to order further testing but instead gives a general impression about the likelihood of a diagnosis of PE. If you can answer no to all the above questions and your impression is low based on your gestalt, the PE miss rate is found be only 1.8%. This has been deemed an acceptable level of error based on the risk/benefit of additional testing. If any question is answered yes and suspicion is increased, proceed to the Well’s Criteria as the next step in evaluation.

     Beck’s Triad are the three clinical signs associated with acute cardiac tamponade: low arterial blood pressure, distended neck veins, and distant, muffled heart sounds.

     Charcot’s Triad are the three clinical signs associated with acute cholangitis: jaundice, fever and right upper quadrant abdominal pain.

     Well’s Criteria would be the second step in evaluation if PERC Criteria had raised suspicion of PE. The Well’s Criteria provides a numeric grading scale based on four clinical questions similar to the PERC questions. The tally of the score is used to determine the usefulness of ordering a D-dimer test on the patient. The D-dimer test is sensitive (95-98%) but not specific (40-55%) for PE. A high Well’s score significantly increases the positive predictive value of a (+) D-dimer test, bringing the probability of PE to 79.6%.   This information will justify ordering the next level of testing, a CTA which is ~95% specific for PE.

    Reference

    Kosowsky, J., Zane, R., Pocket Emergency Medicine 3rd Edition, Wolters Kluwer, 2015, p. 1-16-18

     

     

    Incorrect

     Answer: D. The PERC Criteria, developed in 2008, should be the first choice to guide your medical decision-making regarding a possible PE. It involves eight questions: Age >50?, HR >100?, SaO2 <95%?, hemoptysis?, estrogen use?, surgery/trauma w/hospitalization in last 4wks?, prior VTE?, and unilateral leg swelling?

    The PERC Criteria does not give specific guidance about when to order further testing but instead gives a general impression about the likelihood of a diagnosis of PE. If you can answer no to all the above questions and your impression is low based on your gestalt, the PE miss rate is found be only 1.8%. This has been deemed an acceptable level of error based on the risk/benefit of additional testing. If any question is answered yes and suspicion is increased, proceed to the Well’s Criteria as the next step in evaluation.

     Beck’s Triad are the three clinical signs associated with acute cardiac tamponade: low arterial blood pressure, distended neck veins, and distant, muffled heart sounds.

     Charcot’s Triad are the three clinical signs associated with acute cholangitis: jaundice, fever and right upper quadrant abdominal pain.

     Well’s Criteria would be the second step in evaluation if PERC Criteria had raised suspicion of PE. The Well’s Criteria provides a numeric grading scale based on four clinical questions similar to the PERC questions. The tally of the score is used to determine the usefulness of ordering a D-dimer test on the patient. The D-dimer test is sensitive (95-98%) but not specific (40-55%) for PE. A high Well’s score significantly increases the positive predictive value of a (+) D-dimer test, bringing the probability of PE to 79.6%.   This information will justify ordering the next level of testing, a CTA which is ~95% specific for PE.

    Reference

    Kosowsky, J., Zane, R., Pocket Emergency Medicine 3rd Edition, Wolters Kluwer, 2015, p. 1-16-18

  60. Question 60 of 220
    60. Question

    47 year old female presents to the ED for the second time in two days complaining of severe pain in her left middle finger.  She was prescribed cephalexin (Keflex) by an urgent care provider after cutting her finger with a cooking knife one week ago.  She presented to the ED yesterday because, after a short period of improvement, the pain and swelling began to increase again.  An x-ray of the finger was negative. She was given 1 gm ceftriaxone (Rocephin) IM with the instructions to return to the ED if her condition did not improve or got worse.  Unable to sleep because of the pain’s increasing intensity, she returns today complaining of newer symptoms.  Upon evaluation, there is fusiform swelling of the finger; it is held in slight flexion, and there is point tenderness along the flexor tendon sheath.  She complains of exquisite pain with passive extension.  A CBC reveals an elevated WBC with a slight left shift and her ESR is elevated.  Given this patient’s likely diagnosis, which of the following is the most appropriate next immediate step in her care?

    Correct

    Answer C. This patient has infectious flexor tenosynovitis, which is considered an orthopedic emergency.  The most appropriate next step would be to consult an orthopedic surgeon now for possible surgical intervention. The incidence of infectious tenosynovitis has been difficult to determine1; however, there is an increased risk of a concomitant infection when a penetrating injury potentially inoculates the tendon sheath.1

    As it is with our patient, the majority of cases of infectious tenosynovitis are the result of some kind of penetrating trauma.1,3 In all likelihood, our patient’s current medical condition is a direct result of the cut to her finger more than a week prior to her presenting to the ED. Other causes, though, may also be from surgery or dissemination from the blood.2

    The most common pathogens involved are Staphylococcus aureus and streptococci, with S. aureus being the more common. It is not unusual, however, for a combination of the two to cause infection.1,2,3 Other potential agents are pseudomonas and mycobacterium.1,3

    Infection is most often the result of direct inoculation or colonized S. aureus around damaged skin gaining access into tissue through a wound,1,2,3  a likely scenario for our patient given her history of a cut to her finger.  The bacteria then attach to and further colonize the exposed damaged tissue.2 Specialized molecules on the cell wall of S. aureus enable the bacteria to adhere to different tissue surfaces, and with the aid of elaborating enzymes like proteases and lipases, the bacteria are then able to spread locally.2 It is likely that over time the infection worked its way to the patient’s tendon sheath, where its supply of synovial fluid provides a rich medium for bacterial growth.

    Our patient has recently sustained a penetrating injury to her left middle finger, and now the finger demonstrates Kenaval’s four cardinal signs of flexor tenosynovitis:  fusiform swelling, tenderness along the flexor tendon sheath, finger held in slightly flexed position, and exquisite pain upon passive extension.  These findings are pathognomonic for infectious flexor tenosynovitis.3

    An x-ray of the finger obtained the day before ruled out the presence of a foreign body or any bony involvement.  The results of the complete blood count (CBC) and the erythrocyte sedimentation rate (ESR) confirmed the presence of an inflammatory response, which helps support the suspicion of an infectious cause.

    Infectious tenosynovitis is considered an orthopedic emergency,1,3 so surgical consult with subsequent admission to the OR is the appropriate next immediate step in this patient’s care. The non-operative approaches also listed might be appropriate if the case had presented and been diagnosed earlier in the process, but the fact that antibiotic treatment for this patient has already failed is an indication for surgical drainage.1

    References

    Foster, MR. Tenosynovitis. http://emedicine.medscape.com/article/2189339. Updated 10/2015. Accessed 03/10/2016.

    Lowy, FD. Staphylococcal infections. In Longo, DL [et al], eds. Harrison’s principles of internal medicine-eighteenth edition. McGraw Hill: New York. 2012: 1160-70.

    Tsai, E; Failla, JM. Hand infections in the trauma patient. Hand clinics.  1999, May; 15(2): 373-86.

    Incorrect

    Answer C. This patient has infectious flexor tenosynovitis, which is considered an orthopedic emergency.  The most appropriate next step would be to consult an orthopedic surgeon now for possible surgical intervention. The incidence of infectious tenosynovitis has been difficult to determine1; however, there is an increased risk of a concomitant infection when a penetrating injury potentially inoculates the tendon sheath.1

    As it is with our patient, the majority of cases of infectious tenosynovitis are the result of some kind of penetrating trauma.1,3 In all likelihood, our patient’s current medical condition is a direct result of the cut to her finger more than a week prior to her presenting to the ED. Other causes, though, may also be from surgery or dissemination from the blood.2

    The most common pathogens involved are Staphylococcus aureus and streptococci, with S. aureus being the more common. It is not unusual, however, for a combination of the two to cause infection.1,2,3 Other potential agents are pseudomonas and mycobacterium.1,3

    Infection is most often the result of direct inoculation or colonized S. aureus around damaged skin gaining access into tissue through a wound,1,2,3  a likely scenario for our patient given her history of a cut to her finger.  The bacteria then attach to and further colonize the exposed damaged tissue.2 Specialized molecules on the cell wall of S. aureus enable the bacteria to adhere to different tissue surfaces, and with the aid of elaborating enzymes like proteases and lipases, the bacteria are then able to spread locally.2 It is likely that over time the infection worked its way to the patient’s tendon sheath, where its supply of synovial fluid provides a rich medium for bacterial growth.

    Our patient has recently sustained a penetrating injury to her left middle finger, and now the finger demonstrates Kenaval’s four cardinal signs of flexor tenosynovitis:  fusiform swelling, tenderness along the flexor tendon sheath, finger held in slightly flexed position, and exquisite pain upon passive extension.  These findings are pathognomonic for infectious flexor tenosynovitis.3

    An x-ray of the finger obtained the day before ruled out the presence of a foreign body or any bony involvement.  The results of the complete blood count (CBC) and the erythrocyte sedimentation rate (ESR) confirmed the presence of an inflammatory response, which helps support the suspicion of an infectious cause.

    Infectious tenosynovitis is considered an orthopedic emergency,1,3 so surgical consult with subsequent admission to the OR is the appropriate next immediate step in this patient’s care. The non-operative approaches also listed might be appropriate if the case had presented and been diagnosed earlier in the process, but the fact that antibiotic treatment for this patient has already failed is an indication for surgical drainage.1

    References

    Foster, MR. Tenosynovitis. http://emedicine.medscape.com/article/2189339. Updated 10/2015. Accessed 03/10/2016.

    Lowy, FD. Staphylococcal infections. In Longo, DL [et al], eds. Harrison’s principles of internal medicine-eighteenth edition. McGraw Hill: New York. 2012: 1160-70.

    Tsai, E; Failla, JM. Hand infections in the trauma patient. Hand clinics.  1999, May; 15(2): 373-86.

  61. Question 61 of 220
    61. Question

    An 82 year old male with a history of aortic stenosis status post bio prosthetic AVR on long term anticoagulation with warfarin sodium presents to the ER after a witnessed fall 4 hours ago.  He states he struck his head when he fell but denies loss of consciousness or neck pain. A short time after the fall he noticed intermittent weakness of the right upper extremity that began about 2 hours ago & has become increasingly worse.  His INR is 2.5 and therapeutic. Remainder of labs are within normal range.  He denies similar symptoms prior to his most recent fall. Physical exam reveals GCS of 15, right upper extremity strength is 0/5, but otherwise the rest of his exam is normal. Based on the information given above, which of the following is the most appropriate diagnostic imaging study to order?

    Correct

    Correct Answer C. CT scan without contrast is the most appropriate initial step in confirming the presence of a subdural hematoma. In patients presenting with acute head trauma, subdural hematomas have been reported to occur in 5-25% of individuals. Subdural hematomas consists of an intracranial mass of blood or calcified emulsions below the inner layer of the dura but external to the brain and arachnoid membrane.  It is the most common type of traumatic intracranial mass lesion.1 and are, also more common in the elderly population for multiple reasons including comorbidities and trauma. Subdural hematomas are typically seen as concave toward the brain and unlimited by suture lines.  Midline shifting is usually present.  Generally, acute subdural hematomas are less than 72 hours old and are hyperdense compared to the brain.

    This patient’s demographic and recent history of a fall (pertinent historical feature) with neurological deficits should elicit concern for a subdural hemorrhage (SDH).  He also presented with an therapeutic INR of 2.5, a level within normal limits for a patient on Warfarin.  This also raises the risk of developing a subdural hemorrhage (SDH).

    Choices A and B are incorrect because magnetic resonance imaging of any sort would take too much time to indicate a possibly life-threatening SDH.  Patients suspected of having a SDH should have intervention as soon as possible by means of surgery if applicable.  Choice B may be utilized if the initial choice of scan showed an isodense matter representing a subacute SDH (or if the initial image was difficult to interpret.)

    Choice D is incorrect because contrast is not necessary to visualize a SDH on a CT scan. It also carries a risk of causing a hypersensitivity reaction, and it may be contraindicated due to renal impairment – a common comorbidity of elderly patients.

    References:

    1. Meagher RJ, Young WF. Subdural Hematoma. Medscape. 2015. http://emedicine.medscape.com/article/1137207-overview. Accessed February 9, 2016.
    2. Centers for Disease Control and Prevention. Home and recreational safety: important facts about falls. http://www.cdc.gov/homeandrecreationalsafety/falls/adultfalls.html. Accessed February 09, 2016.
    3. Stevens JA, Corso PS, Finkelstein EA, Miller TR. The costs of fatal and nonfatal falls among older adults. Injury Prevention 2006;12:290–5.
    4. Atkinson JL, Lane JI, Aksamit AJ. MRI depiction of chronic intradural (subdural) hematoma in evolution. J Magn Reson Imaging. 2003 April 17(4):484-6.
    5. VirtualMedStudent. Subdural hematomas: diagnosis. http://www.virtualmedstudent.com/links/neurological/subdural_hematoma.html. Accessed February 9, 2016.
    6. Brain Trauma Foundation, AANS, Joint Section of Neurotrauma and Critical Care. Guidelines for the management of severe head injury. Neurotrauma. 1996 November. 13(11):641-734.
    Incorrect

    Correct Answer C. CT scan without contrast is the most appropriate initial step in confirming the presence of a subdural hematoma. In patients presenting with acute head trauma, subdural hematomas have been reported to occur in 5-25% of individuals. Subdural hematomas consists of an intracranial mass of blood or calcified emulsions below the inner layer of the dura but external to the brain and arachnoid membrane.  It is the most common type of traumatic intracranial mass lesion.1 and are, also more common in the elderly population for multiple reasons including comorbidities and trauma. Subdural hematomas are typically seen as concave toward the brain and unlimited by suture lines.  Midline shifting is usually present.  Generally, acute subdural hematomas are less than 72 hours old and are hyperdense compared to the brain.

    This patient’s demographic and recent history of a fall (pertinent historical feature) with neurological deficits should elicit concern for a subdural hemorrhage (SDH).  He also presented with an therapeutic INR of 2.5, a level within normal limits for a patient on Warfarin.  This also raises the risk of developing a subdural hemorrhage (SDH).

    Choices A and B are incorrect because magnetic resonance imaging of any sort would take too much time to indicate a possibly life-threatening SDH.  Patients suspected of having a SDH should have intervention as soon as possible by means of surgery if applicable.  Choice B may be utilized if the initial choice of scan showed an isodense matter representing a subacute SDH (or if the initial image was difficult to interpret.)

    Choice D is incorrect because contrast is not necessary to visualize a SDH on a CT scan. It also carries a risk of causing a hypersensitivity reaction, and it may be contraindicated due to renal impairment – a common comorbidity of elderly patients.

    References:

    1. Meagher RJ, Young WF. Subdural Hematoma. Medscape. 2015. http://emedicine.medscape.com/article/1137207-overview. Accessed February 9, 2016.
    2. Centers for Disease Control and Prevention. Home and recreational safety: important facts about falls. http://www.cdc.gov/homeandrecreationalsafety/falls/adultfalls.html. Accessed February 09, 2016.
    3. Stevens JA, Corso PS, Finkelstein EA, Miller TR. The costs of fatal and nonfatal falls among older adults. Injury Prevention 2006;12:290–5.
    4. Atkinson JL, Lane JI, Aksamit AJ. MRI depiction of chronic intradural (subdural) hematoma in evolution. J Magn Reson Imaging. 2003 April 17(4):484-6.
    5. VirtualMedStudent. Subdural hematomas: diagnosis. http://www.virtualmedstudent.com/links/neurological/subdural_hematoma.html. Accessed February 9, 2016.
    6. Brain Trauma Foundation, AANS, Joint Section of Neurotrauma and Critical Care. Guidelines for the management of severe head injury. Neurotrauma. 1996 November. 13(11):641-734.
  62. Question 62 of 220
    62. Question

    A 51 year old male with a history of hypertension, panic attacks, and well controlled type II diabetes presents to the Emergency Department with complaints of a moderate to severe headache that has been episodic for the last 8 hours.  He complains of an increased work of breathing as well as episodic diaphoresis.  He feels generally “not myself.” He denies any precipitating events.  Patient rates his headache as a 6/10 on a 0-10 scale at its worst and will spontaneously improve to a 4/10 on a 0-10 scale without aid of palliative measures.  The patient attempted an ice bag and peppermint oil for his headache with no relief. He denies increased work of breathing at baseline except when he has “anxiety attacks.”  He does feel constipated but denies any changes in bladder habits. He denies neck fullness, heat intolerance, diarrhea, restlessness, double vision, lethargy, or changes in hair or skin. He denies any personal or family history of cancer, weight loss, blood clots, recent travel, history of stroke, cardiac history, recent illnesses or infections, recent trauma, history of thyroid conditions, or renal disease. On exam the patient’s blood pressure is 220/126 mmHg & does not respond to IV Labetolol or Nicardipine. The patient states that he has had 2 similar episodes in the past when he was first diagnosed with hypertension. You have a high index of suspicion that this may not be primary hypertension & order further testing. Which of the following diagnostic tests will confirm your suspected diagnosis?

    Correct

    Correct Answer: C 24-hour urine catecholamine. The patient is suffering from a symptomatic tumor of the adrenal medula and sympathetic ganglion known as a pheochromocytoma.  This tumor produces norepinephrine and epinephrine (catecholamines). This can be identified in 1/500 hypertensive individuals. It is most commonly found in 40-60 year olds. Associated risk factors include MEN-II, von Hippel-Lindau disease, and family history. The classic triad includes: palpitations, headache, episodic diaphoresis.  Fifty percent of patients will present with hypertension.  Other symptoms include pallor, flushing, anxiety, weight loss, tachycardia, and hyperglycemia.

    The Mayo Clinic found that a 24 hour urine fractionated metanephrines and catecholamine collection is the most reliable case-detection method with a sensitivity of 98% and a specificity of 98%. To ensure adequate sample include a urinary creatinine.  There is an unfortunate high false positive rate due to the normal range being derived from a normotensive group of individuals. A urine screen should look for Vanillyl Mandelic Acid (VMA), metanephrines, and normetanephrine.  A serum epinephrine/norepinephrine level should also be gathered.

    The patient could also be hyperglycemic as epinephrine increases glucose while norepinephrine decreases insulin.  They could also show signs of polycythemia from volume depletion.

    Plasma fractionated metanephrines are regarded as first line if the patient falls into a high risk for pheochromocytoma.  These include family history, MEN II, previous pheo resection, and an incidental adrenal mass found on imaging showing characteristic features.  Definitive tumor localization should be found by imaging including CT, MRI, I-MIBG, PET scan, or OctreoSCan.

    A troponin is indicated during evaluation of patients suspected of having acute coronary syndrome.  The patient is not experiencing angina, nausea, or vomiting, and denies history of cardiac problems therefore making cardiac etiology less likely.   In the setting of palpitations and shortness of breath a troponin may still be indicated.

    Thyroid Stimulating Hormone (TSH) is released by the anterior pituitary gland and causes release of thyroid hormone from the thyroid.  A TSH is indicated when there is concern for hyper/hypothyroidism.  Although the patient does report “anxiety” and palpitations the the patient does not endorse other historical findings consistent with hyperthyroidism including: neck fullness, heat intolerance, diarrhea, restlessness, double vision, lethargy, or changes in hair or skin.

    A D-dimer is indicated when evaluating a patient for suspected venous thromboembolic disease.  A patient has a low pretest probability for PE/DVT and is therefore more unlikely to have a significant thromboembolic event.

    References

    Clinical presentation and diagnosis of pheochromocytoma. Clinical presentation and diagnosis of pheochromocytoma. Available at: http://www.uptodate.com/contents/clinical-presentation-and-diagnosis-of-pheochromocytoma?source=search_result. Accessed February 16, 2016.

    Mahadevan SV, Garmel GM. An Introduction To Clinical Emergency Medicine Guide for Practitioners in the Emergency Department. Cambridge: Cambridge University Press; 2005.

    Blackbourne LH. Surgical Recall. 7th ed. Philadelphia, PA: Wolters Kluwer/Lippincott Williams & Wilkins; 2009.

    Incorrect

    Correct Answer: C 24-hour urine catecholamine. The patient is suffering from a symptomatic tumor of the adrenal medula and sympathetic ganglion known as a pheochromocytoma.  This tumor produces norepinephrine and epinephrine (catecholamines). This can be identified in 1/500 hypertensive individuals. It is most commonly found in 40-60 year olds. Associated risk factors include MEN-II, von Hippel-Lindau disease, and family history. The classic triad includes: palpitations, headache, episodic diaphoresis.  Fifty percent of patients will present with hypertension.  Other symptoms include pallor, flushing, anxiety, weight loss, tachycardia, and hyperglycemia.

    The Mayo Clinic found that a 24 hour urine fractionated metanephrines and catecholamine collection is the most reliable case-detection method with a sensitivity of 98% and a specificity of 98%. To ensure adequate sample include a urinary creatinine.  There is an unfortunate high false positive rate due to the normal range being derived from a normotensive group of individuals. A urine screen should look for Vanillyl Mandelic Acid (VMA), metanephrines, and normetanephrine.  A serum epinephrine/norepinephrine level should also be gathered.

    The patient could also be hyperglycemic as epinephrine increases glucose while norepinephrine decreases insulin.  They could also show signs of polycythemia from volume depletion.

    Plasma fractionated metanephrines are regarded as first line if the patient falls into a high risk for pheochromocytoma.  These include family history, MEN II, previous pheo resection, and an incidental adrenal mass found on imaging showing characteristic features.  Definitive tumor localization should be found by imaging including CT, MRI, I-MIBG, PET scan, or OctreoSCan.

    A troponin is indicated during evaluation of patients suspected of having acute coronary syndrome.  The patient is not experiencing angina, nausea, or vomiting, and denies history of cardiac problems therefore making cardiac etiology less likely.   In the setting of palpitations and shortness of breath a troponin may still be indicated.

    Thyroid Stimulating Hormone (TSH) is released by the anterior pituitary gland and causes release of thyroid hormone from the thyroid.  A TSH is indicated when there is concern for hyper/hypothyroidism.  Although the patient does report “anxiety” and palpitations the the patient does not endorse other historical findings consistent with hyperthyroidism including: neck fullness, heat intolerance, diarrhea, restlessness, double vision, lethargy, or changes in hair or skin.

    A D-dimer is indicated when evaluating a patient for suspected venous thromboembolic disease.  A patient has a low pretest probability for PE/DVT and is therefore more unlikely to have a significant thromboembolic event.

    References

    Clinical presentation and diagnosis of pheochromocytoma. Clinical presentation and diagnosis of pheochromocytoma. Available at: http://www.uptodate.com/contents/clinical-presentation-and-diagnosis-of-pheochromocytoma?source=search_result. Accessed February 16, 2016.

    Mahadevan SV, Garmel GM. An Introduction To Clinical Emergency Medicine Guide for Practitioners in the Emergency Department. Cambridge: Cambridge University Press; 2005.

    Blackbourne LH. Surgical Recall. 7th ed. Philadelphia, PA: Wolters Kluwer/Lippincott Williams & Wilkins; 2009.

  63. Question 63 of 220
    63. Question

    A 55 year old postmenopausal, obese female presents to the ED for severe abdominal pain. She says the pain is crampy and it has been getting worse for the past week and she cannot stand it anymore. When asked specifically where the pain is located, the patient points to the lower left side of her abdomen. Patient denies dysuria, chest pain or any burning sensation in her chest and states she had been feeling constipated for a few days prior. Blood pressure is 145/68, pulse is 99, respiratory rate is 20, temperature is 101.2 degrees Fahrenheit and her oxygen saturation is 99% on room air. Physical exam is unremarkable except for tenderness over the LLQ of the abdomen. Which of the following is the most appropriate treatment for the patient’s most likely diagnosis?

    Correct

    Correct answer is B. The most likely diagnosis of this patient is acute diverticulitis. This is an obese patient presenting with a fever and crampy LLQ pain, all suggestive of the diagnosis. The patient’s pain is in the LLQ (unlikely appendicitis), denies dysuria (unlikely a UTI), denies a burning sensation (unlikely GERD) and she moved her bowels (unlikely a bowel obstruction). The most appropriate next step would be to obtain an abdominal CT to confirm the diagnosis. Option A is not the correct antibiotic for diverticulitis. Option C is not indicated for diverticulitis unless there are complications and the patient fails medical management. Option D is inappropriate as the patient needs antibiotics (and mostly likely morphine for pain management).

    Diverticulosis is the existence of diverticula, which are bulges in the colon wall, and they may or may not have symptoms. Diverticular disease is when they bleed, become inflamed or ischemic. Diverticulitis is when there is inflammation of a diverticulum and it can become complicated by abscesses, fistulas or bowel obstructions.
    Diverticulosis prevalence varies based on age. It occurs in under 20 percent of individuals under 40 years old but increases to 60 percent of people by 60 years old. Diverticulitis is present in about 4-15% of individuals who have diverticulosis. In western countries, the disease occurs most often in the left side of the colon and in Asian countries, it occurs more often on the the right side. It is more common in men before the age of 50. Between the ages of 50 and 70, women slightly predominate and over 70, women have a much higher incidence.

    The etiology of diverticular disease is unclear but there are numerous risk factors. Some studies indicate that a diet low in fiber increases the rate of the disease. However, increasing the intake of fiber does not decrease symptoms in patients already suffering from the disease. It may, though, decrease the rate of the disease progression by lowering inflammation in the intestines and changing the microflora of the gut. Another risk factor is obesity. The rate of diverticulitis was much greater in those with a higher waist circumference. Individuals who smoke are at a greater risk for the disease. Lastly, medications such as NSAIDs, steroids and opiates are correlated with an increased risk.

    Diverticula occur when there is herniation of small mucosal layers that protrude through the intestine and smooth muscle. They herniate through natural openings that have been produced by the vasa recta or vessels in the colon wall. “True” diverticulae include all layers of the GI wall while “false” diverticulae only contain submucosa or mucosa layers. These herniations can happen anywhere within the GI tract but most often occur in the colon, particularly the sigmoid colon, which has the greatest intraluminal pressure. Diverticulosis, which is when the diverticula are not inflamed, increases with age and the cause is not fully understood. Diverticulitis occurs when single or multiple diverticula become inflamed and its cause is uncertain as well. It is thought that fecal matter or food pieces that were undigested can accumulate in the pouches. An obstruction then occurs, leading to distention due to buildup of mucous and increased growth of bacteria. Vessels can become compromised and small or large perforations can follow. Another theory is that there is increased pressure in the lumen which leads to break-down of the diverticular wall. This leads to inflammation, perforation and tissue death. The disease can be mild with small perforations but when they are large, abscesses can form and in rare instances, they may lead to rupture of the intestines or peritonitis.

    The most common symptom of patients presenting with diverticulitis is left lower quadrant pain and it is present in up to 70 percent of patients. However, the symptoms will vary depending on the site of the inflamed diverticulum. Patients often note the pain is crampy and their bowel movements may be affected. Associated symptoms may include feeling nauseas, vomiting, having constipation or diarrhea, and experiencing flatulence and bloating. A very small perforation may present without any symptoms or signs of infection. However, it may lead to a more contained outgrowth and signs of systemic disease can arise. Severe forms of the disease is often seen with decreased appetite, nausea and vomiting. Patients usually note that they have been experiencing pain for several days and it is very intense and is only in one area of their abdomen. They are often constipated and a small minority note symptoms of the urinary system, including urgency and dysuria.

    There are numerous physical findings that can be present due to diverticulitis and they will correlate with the degree of inflammation and existence of any complications. In uncomplicated diverticulitis, the patient will present with fever and the abdomen is tender over a specific area. Most commonly, the left lower quadrant is tender as most diverticula are present in the sigmoid colon. If the diverticulitis is complicated with an abscess, often a palpable mass that is tender can be appreciated. If there is peritonitis, there may be generalized tenderness along with rebound tenderness or guarding. There may be distention of the abdomen and bowel sounds are often decreased or absent.

    Acute diverticulitis is often diagnosed based on the history and physical exam. However, lab tests are often helpful. These include a complete blood count which often show leukocytosis and a left shift which would demonstrate the presence of an infection. However, 20-40% of patients have a normal WBC count, especially those who are immunocompromised, elderly or in those in whom the disease is not severe. A hemoglobin level should be noted when the patient mentions blood in his or her stool. A basic metabolic panel should be obtained if the patient has been vomiting or had diarrhea in order to evaluate electrolyte disturbances. Liver function tests will help to rule out other etiologies. It is also important to get blood cultures before giving antibiotics, especially those who are very sick. Lastly, a pregnancy test should also be done on a woman of childbearing age.
    The imaging modality most often used in diagnosis is an abdominal CT. It may show pericolic fat stranding (linear areas of soft tissue around the colon), diverticula, bowel wall thickening and abscesses. If there is an abscess, the CT can help in localizing the site for drainage. Plain x-rays are not informative in diagnosing diverticulitis but it may show a bowel obstruction if one is present.
    Diverticulitis treatment can be divided into uncomplicated or complicated disease. If the disease is acute and uncomplicated, conservative treatment is successful in 70-100% or patients. Patients are normally put on a clear liquid diet along with 7-10 days of broad-spectrum antibacterial therapy, covering both anaerobic and aerobic microorganisms. Typically, ciprofloxacin and metronidazole are administered. The clear liquid diet is then slowly adjusted as the patient improves. Patients are hospitalized if they demonstrate an infection with systemic symptoms or are unsuccessful with outpatient treatment. In addition, patients are usually administered morphine for pain. It usually takes 2-3 days in the hospital before patients’ symptoms improve. In complicated diverticulitis, patients may need surgery due to uncontrolled sepsis or an intestinal obstruction.
    Once a patient has recovered, he or she is advised to have a colonoscopy in order to determine if there is a malignancy. They should also consume a high-fiber diet for the rest of their lives. The disease’s recurrence is extremely variable and is based on the illness’ severity, any complications or comorbidities.

    References

    Cunha, John P. A Picture Guide to Diverticulitis. MedicineNet.com. http://www.medicinenet.com/diverticulitis_diverticulosis_pictures_slideshow/article.htm. Reviewed December 24, 2014. Accessed March 14, 2016.

    Elmasry, Mohamed. Diverticulitis of the Sigmoid Colon with Fat Stranding on CT Scan. Radiology Spirit. Published July 28, 2010. Accessed March 14,2015.

    Pemberton, John. UpToDate. Clinical manifestations and diagnosis of acute diverticulitis in adults. Published December 15, 2015. Accessed March 14, 2016.

    Shahedi, Kamyar. Diverticulitis. Medscape. http://emedicine.medscape.com/article/173388-overview#a6. Updated January 14, 2015. Accessed March 14, 2016.18.

    Incorrect

    Correct answer is B. The most likely diagnosis of this patient is acute diverticulitis. This is an obese patient presenting with a fever and crampy LLQ pain, all suggestive of the diagnosis. The patient’s pain is in the LLQ (unlikely appendicitis), denies dysuria (unlikely a UTI), denies a burning sensation (unlikely GERD) and she moved her bowels (unlikely a bowel obstruction). The most appropriate next step would be to obtain an abdominal CT to confirm the diagnosis. Option A is not the correct antibiotic for diverticulitis. Option C is not indicated for diverticulitis unless there are complications and the patient fails medical management. Option D is inappropriate as the patient needs antibiotics (and mostly likely morphine for pain management).

    Diverticulosis is the existence of diverticula, which are bulges in the colon wall, and they may or may not have symptoms. Diverticular disease is when they bleed, become inflamed or ischemic. Diverticulitis is when there is inflammation of a diverticulum and it can become complicated by abscesses, fistulas or bowel obstructions.
    Diverticulosis prevalence varies based on age. It occurs in under 20 percent of individuals under 40 years old but increases to 60 percent of people by 60 years old. Diverticulitis is present in about 4-15% of individuals who have diverticulosis. In western countries, the disease occurs most often in the left side of the colon and in Asian countries, it occurs more often on the the right side. It is more common in men before the age of 50. Between the ages of 50 and 70, women slightly predominate and over 70, women have a much higher incidence.

    The etiology of diverticular disease is unclear but there are numerous risk factors. Some studies indicate that a diet low in fiber increases the rate of the disease. However, increasing the intake of fiber does not decrease symptoms in patients already suffering from the disease. It may, though, decrease the rate of the disease progression by lowering inflammation in the intestines and changing the microflora of the gut. Another risk factor is obesity. The rate of diverticulitis was much greater in those with a higher waist circumference. Individuals who smoke are at a greater risk for the disease. Lastly, medications such as NSAIDs, steroids and opiates are correlated with an increased risk.

    Diverticula occur when there is herniation of small mucosal layers that protrude through the intestine and smooth muscle. They herniate through natural openings that have been produced by the vasa recta or vessels in the colon wall. “True” diverticulae include all layers of the GI wall while “false” diverticulae only contain submucosa or mucosa layers. These herniations can happen anywhere within the GI tract but most often occur in the colon, particularly the sigmoid colon, which has the greatest intraluminal pressure. Diverticulosis, which is when the diverticula are not inflamed, increases with age and the cause is not fully understood. Diverticulitis occurs when single or multiple diverticula become inflamed and its cause is uncertain as well. It is thought that fecal matter or food pieces that were undigested can accumulate in the pouches. An obstruction then occurs, leading to distention due to buildup of mucous and increased growth of bacteria. Vessels can become compromised and small or large perforations can follow. Another theory is that there is increased pressure in the lumen which leads to break-down of the diverticular wall. This leads to inflammation, perforation and tissue death. The disease can be mild with small perforations but when they are large, abscesses can form and in rare instances, they may lead to rupture of the intestines or peritonitis.

    The most common symptom of patients presenting with diverticulitis is left lower quadrant pain and it is present in up to 70 percent of patients. However, the symptoms will vary depending on the site of the inflamed diverticulum. Patients often note the pain is crampy and their bowel movements may be affected. Associated symptoms may include feeling nauseas, vomiting, having constipation or diarrhea, and experiencing flatulence and bloating. A very small perforation may present without any symptoms or signs of infection. However, it may lead to a more contained outgrowth and signs of systemic disease can arise. Severe forms of the disease is often seen with decreased appetite, nausea and vomiting. Patients usually note that they have been experiencing pain for several days and it is very intense and is only in one area of their abdomen. They are often constipated and a small minority note symptoms of the urinary system, including urgency and dysuria.

    There are numerous physical findings that can be present due to diverticulitis and they will correlate with the degree of inflammation and existence of any complications. In uncomplicated diverticulitis, the patient will present with fever and the abdomen is tender over a specific area. Most commonly, the left lower quadrant is tender as most diverticula are present in the sigmoid colon. If the diverticulitis is complicated with an abscess, often a palpable mass that is tender can be appreciated. If there is peritonitis, there may be generalized tenderness along with rebound tenderness or guarding. There may be distention of the abdomen and bowel sounds are often decreased or absent.

    Acute diverticulitis is often diagnosed based on the history and physical exam. However, lab tests are often helpful. These include a complete blood count which often show leukocytosis and a left shift which would demonstrate the presence of an infection. However, 20-40% of patients have a normal WBC count, especially those who are immunocompromised, elderly or in those in whom the disease is not severe. A hemoglobin level should be noted when the patient mentions blood in his or her stool. A basic metabolic panel should be obtained if the patient has been vomiting or had diarrhea in order to evaluate electrolyte disturbances. Liver function tests will help to rule out other etiologies. It is also important to get blood cultures before giving antibiotics, especially those who are very sick. Lastly, a pregnancy test should also be done on a woman of childbearing age.
    The imaging modality most often used in diagnosis is an abdominal CT. It may show pericolic fat stranding (linear areas of soft tissue around the colon), diverticula, bowel wall thickening and abscesses. If there is an abscess, the CT can help in localizing the site for drainage. Plain x-rays are not informative in diagnosing diverticulitis but it may show a bowel obstruction if one is present.
    Diverticulitis treatment can be divided into uncomplicated or complicated disease. If the disease is acute and uncomplicated, conservative treatment is successful in 70-100% or patients. Patients are normally put on a clear liquid diet along with 7-10 days of broad-spectrum antibacterial therapy, covering both anaerobic and aerobic microorganisms. Typically, ciprofloxacin and metronidazole are administered. The clear liquid diet is then slowly adjusted as the patient improves. Patients are hospitalized if they demonstrate an infection with systemic symptoms or are unsuccessful with outpatient treatment. In addition, patients are usually administered morphine for pain. It usually takes 2-3 days in the hospital before patients’ symptoms improve. In complicated diverticulitis, patients may need surgery due to uncontrolled sepsis or an intestinal obstruction.
    Once a patient has recovered, he or she is advised to have a colonoscopy in order to determine if there is a malignancy. They should also consume a high-fiber diet for the rest of their lives. The disease’s recurrence is extremely variable and is based on the illness’ severity, any complications or comorbidities.

    References

    Cunha, John P. A Picture Guide to Diverticulitis. MedicineNet.com. http://www.medicinenet.com/diverticulitis_diverticulosis_pictures_slideshow/article.htm. Reviewed December 24, 2014. Accessed March 14, 2016.

    Elmasry, Mohamed. Diverticulitis of the Sigmoid Colon with Fat Stranding on CT Scan. Radiology Spirit. Published July 28, 2010. Accessed March 14,2015.

    Pemberton, John. UpToDate. Clinical manifestations and diagnosis of acute diverticulitis in adults. Published December 15, 2015. Accessed March 14, 2016.

    Shahedi, Kamyar. Diverticulitis. Medscape. http://emedicine.medscape.com/article/173388-overview#a6. Updated January 14, 2015. Accessed March 14, 2016.18.

  64. Question 64 of 220
    64. Question

    A 44 yo overweight female with h/o HTN on HCTZ presents to the emergency department with abdominal pain x 3 hours.  She states the pain is “right under my chest,” radiating to her mid-back, described as sharp, stabbing, constant, rated 9/10, and aggravated by movement.  She states she first thought the pain was due to indigestion, so she chewed two Tums, but this provided no relief.  Pain is associated with nausea, one episode of vomiting, and shortness of breath; otherwise, she denies fever/chills, diarrhea, reflux/burning, excessive regurgitation, bloody or dark stools, constipation, weight loss, diaphoresis, or hx of similar pain.  Vital signs are stable, and physical exam is unremarkable except for tenderness with deep palpation in the epigastric region and LUQ.  Based on the history and exam, what is the most likely etiology for this patient’s symptoms?

    Correct

    The correct answer is D, acute pancreatitis, likely drug induced, as she is taking HCTZ, a drug with high potential for inducing acute pancreatitis .  She also has epigastric pain radiating to the back with nausea, and tenderness to the epigastric and LUQ regions; this is a typical picture of acute pancreatitis, as will be discussed below.  Options A, B, and C are distracters.  Acute cholecystitis is a possible diagnosis, as this is an overweight female in her forties with epigastric pain.  However, she does not have a fever, and PE is benign with no indication of Murphy’s sign; these are two main factors to consider when making the diagnosis.   Acute MI is a distracter as this is a woman with epigastric pain, but it is incorrect as the patient would not have reproducible pain on PE.   GERD is a distracter as it is a common diagnosis with epigastric pain in overweight adults and can radiate to the back; however, nausea is not usual in GERD, and the patient does not have typical GERD symptoms of burning, dysphagia, and regurgitation.

    Acute pancreatitis is an inflammatory condition of the pancreas that lasts for several hours and heals over the course of several days.  The incidence of the disease has been increasing in recent years, with approximately 40 cases per year per 100,000 adults.  Incidence is greater in men than woman; however, this does depend on etiology, as alcohol related disease occurs more commonly in men, and women tend to present more as biliary disease.  Idiopathic pancreatitis has no sex-related patterns.  Etiology and pathophysiology of acute pancreatitis are vast.  Thirty to forty percent of cases occur due to cholelithiasis; if a stone from the gallbladder obstructs the ampulla, bile can flow backwards into the pancreatic duct, causing an inflammation.1  Thirty percent of cases in the US is due to alcohol; therefore, this must be one of the pertinent questions asked during the patient interview.  The exact pathophysiology of alcoholic pancreatitis is unclear, but proposed mechanisms include ethanol increasing the protein content of pancreatic juice, thereby eventually leading to protein plugs in the ducts that block outflow.  This blockage leads to inflammation.  Acute pancreatitis can also develop after ERCP to treat choledocholithiasis.5  Other etiology include triglyceride levels above 1000, abdominal trauma, infections such as Hepatitis B and Varicella, and genetic mutations.  There is an extensive list of medications that are known to trigger episodes of acute pancreatitis, including diuretics, atorvastatin, metformin, and Lisinopril.  Pancreas divisum is an anatomic abnormality where two pancreatic ducts are formed, causing an overflow obstruction, as the minor papilla are not able to manage the volume of pancreatic juice flowing into it; this can lead to an inflamed state.  If none of these etiologies are present in a patient, idiopathic acute pancreatitis becomes the answer, as in the case presented above.1

    Historical findings in a patient with acute pancreatitis correlate to the array of etiologies mentioned above.  In addition, these patients will usually present with severe, constant epigastric pain with or without radiation to the left upper quadrant.  Pain also radiates to the back and is relieved when sitting leaning forward, as in the case above.  Nausea, vomiting, and dyspnea are associated symptoms.  On physical examination, tenderness to the epigastric region is present.  Hepatomegaly is commonly present in alcoholic pancreatitis.  In gallstone pancreatitis, jaundice and scleral icterus is present if the stone has dislodged and caused an obstruction in the common bile duct.  In severe cases, abdominal distention and hypoactive bowel sounds should raise red flags for ileus.
    There are certain laboratory markers to indicate acute pancreatitis.  To make the diagnosis, serum amylase or lipase levels must be elevated three times or greater than the upper limit of normal (or, a combination of epigastric pain radiating to the back and evidence on imaging must be present to make the diagnosis).  An elevated AST, ALT, ALP, and total bilirubin are suggestive of gallstone pancreatitis.  Elevated cholesterol and triglycerides would suggest hyperlipidemia pancreatitis.  Additionally, IgG 4 levels can be checked to determine if an autoimmune pancreatitis is occurring; this marker will be elevated.  Abdominal ultrasound is the first imaging test of choice, as it can detect gallstones.  A CT scan would be necessary if the pain has been present for several days, and a walled-off necrosis or pseudocyst is suspected.6  An MRCP can also be performed if choledocholithiasis is a concern.5

    Treatment of acute pancreatitis includes keeping the patient NPO, administering IV fluids, and providing adequate pain relief.  If gallstones or biliary sludge is noted on imaging, a cholecystectomy is performed during the same admission, as future episodes of pancreatitis are now more likely to be triggered.  As the patient’s pain begins to lessen, clear liquids to full liquids to eventual regular diet is followed; once caloric intake is adequate and serum lipase/amylase have trended down to near normal, the patient can be discharged.  Any complications noted on imaging require intensive care admission.  The patient should be followed as an outpatient within 7-10 days to be certain his/her symptoms have resolved.5

    References

    1. Vege, Santhi Swaroop. Etiology of acute pancreatitis. UpToDate website. http://www.uptodate.com/contents/etiology-of-acute-pancreatitis?source=preview&search=acute+pancreatitis&language=en-US&anchor=H13#H13. Updated May 7, 2015. Accessed June 5, 2016.

    2. Afdhal, Nezam, Zakko, Salam. Acute cholecystitis: Pathogenesis, clinical features, and diagnosis. UpToDate website. http://www.uptodate.com/contents/acute-cholecystitis-pathogenesis-clinical-features-and-diagnosis?source=machineLearning&search=acute+cholecystitis&selectedTitle=1~77&sectionRank=1&anchor=H15#H15. Updated August 26, 2015. Accessed June 5, 2016.

    3. Cayley, William. Diagnosing the Cause of Chest pain. American Family Physician website. http://www.aafp.org/afp/2005/1115/p2012.html. Updated November 15, 2005. Accessed June 5, 2016

    4. Kahrilas, Peter. Clinical manifestations and diagnosis of gastroesophageal reflux in adults. UpToDate website. http://www.uptodate.com/contents/clinical-manifestations-and-diagnosis-of-gastroesophageal-reflux-in-adults?source=machineLearning&search=gerd&selectedTitle=4~150&sectionRank=2&anchor=H13575781#H13575781. Updated March 11, 2015. Accessed June 5, 2016.

    5. Gardner, Timothy B. Acute Pancreatitis. Medscape website. http://emedicine.medscape.com/article/181364-overview#a1. Updated April 1, 2015. Accessed June 5, 2016.

    6. Vege, Santhi Swaroop. Clinical manifestations and diagnosis of acute pancreatitis. UpToDate website. http://www.uptodate.com/contents/clinical-manifestations-and-diagnosis-of-acute-pancreatitis?source=search_result&search=acute+pancreatitis&selectedTitle=2~150. Updated December 14, 2015. Accessed June 7, 2016.

    7. Kirkman-Oh, Myriam. Biliary tract obstruction caused by a gallstone in the distal common bile duct. American Family Physician Website. http://www.aafp.org/afp/2000/0701/p164.html. Updated July 1, 2000. Accessed June 7, 2016

    Incorrect

    The correct answer is D, acute pancreatitis, likely drug induced, as she is taking HCTZ, a drug with high potential for inducing acute pancreatitis .  She also has epigastric pain radiating to the back with nausea, and tenderness to the epigastric and LUQ regions; this is a typical picture of acute pancreatitis, as will be discussed below.  Options A, B, and C are distracters.  Acute cholecystitis is a possible diagnosis, as this is an overweight female in her forties with epigastric pain.  However, she does not have a fever, and PE is benign with no indication of Murphy’s sign; these are two main factors to consider when making the diagnosis.   Acute MI is a distracter as this is a woman with epigastric pain, but it is incorrect as the patient would not have reproducible pain on PE.   GERD is a distracter as it is a common diagnosis with epigastric pain in overweight adults and can radiate to the back; however, nausea is not usual in GERD, and the patient does not have typical GERD symptoms of burning, dysphagia, and regurgitation.

    Acute pancreatitis is an inflammatory condition of the pancreas that lasts for several hours and heals over the course of several days.  The incidence of the disease has been increasing in recent years, with approximately 40 cases per year per 100,000 adults.  Incidence is greater in men than woman; however, this does depend on etiology, as alcohol related disease occurs more commonly in men, and women tend to present more as biliary disease.  Idiopathic pancreatitis has no sex-related patterns.  Etiology and pathophysiology of acute pancreatitis are vast.  Thirty to forty percent of cases occur due to cholelithiasis; if a stone from the gallbladder obstructs the ampulla, bile can flow backwards into the pancreatic duct, causing an inflammation.1  Thirty percent of cases in the US is due to alcohol; therefore, this must be one of the pertinent questions asked during the patient interview.  The exact pathophysiology of alcoholic pancreatitis is unclear, but proposed mechanisms include ethanol increasing the protein content of pancreatic juice, thereby eventually leading to protein plugs in the ducts that block outflow.  This blockage leads to inflammation.  Acute pancreatitis can also develop after ERCP to treat choledocholithiasis.5  Other etiology include triglyceride levels above 1000, abdominal trauma, infections such as Hepatitis B and Varicella, and genetic mutations.  There is an extensive list of medications that are known to trigger episodes of acute pancreatitis, including diuretics, atorvastatin, metformin, and Lisinopril.  Pancreas divisum is an anatomic abnormality where two pancreatic ducts are formed, causing an overflow obstruction, as the minor papilla are not able to manage the volume of pancreatic juice flowing into it; this can lead to an inflamed state.  If none of these etiologies are present in a patient, idiopathic acute pancreatitis becomes the answer, as in the case presented above.1

    Historical findings in a patient with acute pancreatitis correlate to the array of etiologies mentioned above.  In addition, these patients will usually present with severe, constant epigastric pain with or without radiation to the left upper quadrant.  Pain also radiates to the back and is relieved when sitting leaning forward, as in the case above.  Nausea, vomiting, and dyspnea are associated symptoms.  On physical examination, tenderness to the epigastric region is present.  Hepatomegaly is commonly present in alcoholic pancreatitis.  In gallstone pancreatitis, jaundice and scleral icterus is present if the stone has dislodged and caused an obstruction in the common bile duct.  In severe cases, abdominal distention and hypoactive bowel sounds should raise red flags for ileus.
    There are certain laboratory markers to indicate acute pancreatitis.  To make the diagnosis, serum amylase or lipase levels must be elevated three times or greater than the upper limit of normal (or, a combination of epigastric pain radiating to the back and evidence on imaging must be present to make the diagnosis).  An elevated AST, ALT, ALP, and total bilirubin are suggestive of gallstone pancreatitis.  Elevated cholesterol and triglycerides would suggest hyperlipidemia pancreatitis.  Additionally, IgG 4 levels can be checked to determine if an autoimmune pancreatitis is occurring; this marker will be elevated.  Abdominal ultrasound is the first imaging test of choice, as it can detect gallstones.  A CT scan would be necessary if the pain has been present for several days, and a walled-off necrosis or pseudocyst is suspected.6  An MRCP can also be performed if choledocholithiasis is a concern.5

    Treatment of acute pancreatitis includes keeping the patient NPO, administering IV fluids, and providing adequate pain relief.  If gallstones or biliary sludge is noted on imaging, a cholecystectomy is performed during the same admission, as future episodes of pancreatitis are now more likely to be triggered.  As the patient’s pain begins to lessen, clear liquids to full liquids to eventual regular diet is followed; once caloric intake is adequate and serum lipase/amylase have trended down to near normal, the patient can be discharged.  Any complications noted on imaging require intensive care admission.  The patient should be followed as an outpatient within 7-10 days to be certain his/her symptoms have resolved.5

    References

    1. Vege, Santhi Swaroop. Etiology of acute pancreatitis. UpToDate website. http://www.uptodate.com/contents/etiology-of-acute-pancreatitis?source=preview&search=acute+pancreatitis&language=en-US&anchor=H13#H13. Updated May 7, 2015. Accessed June 5, 2016.

    2. Afdhal, Nezam, Zakko, Salam. Acute cholecystitis: Pathogenesis, clinical features, and diagnosis. UpToDate website. http://www.uptodate.com/contents/acute-cholecystitis-pathogenesis-clinical-features-and-diagnosis?source=machineLearning&search=acute+cholecystitis&selectedTitle=1~77&sectionRank=1&anchor=H15#H15. Updated August 26, 2015. Accessed June 5, 2016.

    3. Cayley, William. Diagnosing the Cause of Chest pain. American Family Physician website. http://www.aafp.org/afp/2005/1115/p2012.html. Updated November 15, 2005. Accessed June 5, 2016

    4. Kahrilas, Peter. Clinical manifestations and diagnosis of gastroesophageal reflux in adults. UpToDate website. http://www.uptodate.com/contents/clinical-manifestations-and-diagnosis-of-gastroesophageal-reflux-in-adults?source=machineLearning&search=gerd&selectedTitle=4~150&sectionRank=2&anchor=H13575781#H13575781. Updated March 11, 2015. Accessed June 5, 2016.

    5. Gardner, Timothy B. Acute Pancreatitis. Medscape website. http://emedicine.medscape.com/article/181364-overview#a1. Updated April 1, 2015. Accessed June 5, 2016.

    6. Vege, Santhi Swaroop. Clinical manifestations and diagnosis of acute pancreatitis. UpToDate website. http://www.uptodate.com/contents/clinical-manifestations-and-diagnosis-of-acute-pancreatitis?source=search_result&search=acute+pancreatitis&selectedTitle=2~150. Updated December 14, 2015. Accessed June 7, 2016.

    7. Kirkman-Oh, Myriam. Biliary tract obstruction caused by a gallstone in the distal common bile duct. American Family Physician Website. http://www.aafp.org/afp/2000/0701/p164.html. Updated July 1, 2000. Accessed June 7, 2016

  65. Question 65 of 220
    65. Question

    Which of the following is the most commonly inherited coagulation disorder that causes a hypercoagulable state predisposing patients to pulmonary embolism?

    Correct

    Correct Answer A. Activated Protein C resistance (Factor V Leiden) is the most prevalent inherited hypercoagulable disorder & is found in about 30% of all individuals presenting with deep vein thrombosis (DVT). It is primarily observed in individuals of European ancestry & approximately 5% of the European & U. S. white population are heterozygous for this mutation whereas it is essentially absent in the African-American, Native Americans & Asian populations. In this disorder the gene for Factor V has a single point mutation that makes Factor Va resistant to inhibition by activated Protein C & thus leads to an overabundant conversion of prothrombin to thrombin & more fibrin is created leading to excessive thrombus. Hypercoagulability is differentiated into whether it results from primary (inherited) or secondary (acquired) acquired causes. Primary causes include defects in proteins involved in hemostasis.

    Protein C & S deficiency are inherited in an autosomal dominant pattern but not all patients with heterozygous defects develop inappropriate thrombosis. Von Willebrands is the most common congenital bleeding disorder present in 1% of the population.

    References

    Longo, DL, Kasper D, Jameson J, Fauci A, Hauser S, Loscalzo J. Harrison’s Principles of Internal Medicine. New York: McGraw-Hill; 2012

    McCance KL, Huether SE, Brashers VL, Rote NS, Pathophysiology The Biological  Basis for Disease in Adults and Children. Missouri: Mosby Elsevier; 2010

    Papadikis, Maxine A., Stephen J. McPhee, and Michael W. Rabow. 2011 Current Medical Diagnosis & Treatment. New York: McGraw-Hill Education/Medical, 2011. Print

    Tintinalli JE, Stapczynski,JS, Ma OJ, Cline DM, Cydulka RK, Meckler GD. Emergency Medicine: A Comprehensive Study Guide. New York: McGraw-Hill Pub. Division; 2011

    Incorrect

    Correct Answer A. Activated Protein C resistance (Factor V Leiden) is the most prevalent inherited hypercoagulable disorder & is found in about 30% of all individuals presenting with deep vein thrombosis (DVT). It is primarily observed in individuals of European ancestry & approximately 5% of the European & U. S. white population are heterozygous for this mutation whereas it is essentially absent in the African-American, Native Americans & Asian populations. In this disorder the gene for Factor V has a single point mutation that makes Factor Va resistant to inhibition by activated Protein C & thus leads to an overabundant conversion of prothrombin to thrombin & more fibrin is created leading to excessive thrombus. Hypercoagulability is differentiated into whether it results from primary (inherited) or secondary (acquired) acquired causes. Primary causes include defects in proteins involved in hemostasis.

    Protein C & S deficiency are inherited in an autosomal dominant pattern but not all patients with heterozygous defects develop inappropriate thrombosis. Von Willebrands is the most common congenital bleeding disorder present in 1% of the population.

    References

    Longo, DL, Kasper D, Jameson J, Fauci A, Hauser S, Loscalzo J. Harrison’s Principles of Internal Medicine. New York: McGraw-Hill; 2012

    McCance KL, Huether SE, Brashers VL, Rote NS, Pathophysiology The Biological  Basis for Disease in Adults and Children. Missouri: Mosby Elsevier; 2010

    Papadikis, Maxine A., Stephen J. McPhee, and Michael W. Rabow. 2011 Current Medical Diagnosis & Treatment. New York: McGraw-Hill Education/Medical, 2011. Print

    Tintinalli JE, Stapczynski,JS, Ma OJ, Cline DM, Cydulka RK, Meckler GD. Emergency Medicine: A Comprehensive Study Guide. New York: McGraw-Hill Pub. Division; 2011

  66. Question 66 of 220
    66. Question

    A 33 yo male patient presents to the emergency department via EMS with a chief complaint of “bleeding from the rectum and vomiting.”  He states that he “passed out” at work yesterday evening, but has not yet been to his doctor.  His past medical history is significant only for migraine headaches.  He takes Topiramate, Hydrocodone, and Orphenadrine.  He also smokes tobacco occasionally.  He denies any recent weight loss or gain, chills, fevers, any recent physical trauma, or recent travel.  He denies having swallowed any non-food objects or inserting foreign objects into his rectum.  On physical exam you notice that he is pale, tachycardic at 126 bpm, and hypotensive with a BP of 84/50 despite having received 1.5 L IV normal saline en route.  On inspection of the contents of his emesis bag you note coffee ground-like vomitus, as well as bright red blood covering his clothing.  His abdominal exam reveals tenderness to palpation in the right upper quadrant and mild distention throughout and is negative for any sign of trauma.  His rectal exam reveals the passage of bright red blood, no foreign objects or no signs of trauma.  What would be the definitive diagnostic testing to confirm your suspected diagnosis?

    Correct

    Correct answer: B, endoscopy. Due to the profuse amount of blood loss & that the patient is severely hypotensive with limited improvement in blood pressure after IV fluid resuscitation, it is imperative that the source of his bleed be found.  Upper GI endoscopy is the gold standard as it allows for direct visualization & therapeutic intervention to treat the active bleeding site & its ability to predict the likelihood of recurrent bleeding. In cases like this of severe active bleeding endoscopy should be performed as soon as the patient has been appropriately fluid resuscitated & is hemodynamically stable.

    There is already enough information to formulate a list of suspected diagnoses, which is why continuing the history and physical exam (C) in more detail is unnecessary and may actually be detrimental to the patient’s condition. In a patient this young there are very few possible diagnoses to suspect.  While his risk factors are minimal (tobacco use, male gender, medications), his presentation is strongly suggestive of a profuse upper GI bleed, of which PUD is the most common cause.  While a urea breath test (A) could be used to detect H. pylori, this would be an inappropriate test in this setting as it would not help determine the source of the patient’s GI bleed.  Likewise, an abdominal U/S (D) may assist in differentiating a source of right upper quadrant pain, however his combined history and physical exam give no indication that this would be helpful or necessary.

    References

    1. Swartz, M. Chapter 14 the abdomen. In: Textbook of Physical Diagnosis: History and Exam. 7th ed. Philadelphia, PA: Elsevier; 2014: 429-467.
    2. Williams, DA. Chapter 3: Gastrointestinal/nutrition. In: PANCE Prep Pearls: A practical comprehensive study and review guide for the PANCE and PANRE. 1st ed. Lexington, KY: CreateSpace; 2015: 154-161.
    3. Collins, RD. Hematemesis and melena, Rectal bleeding. In; Differential Diagnosis in Primary Care. 5th ed. Philadelphia, PA: Lippincott Williams & Wilkins; 2012: 208-213,367-371.
    4. National Institute of Health. Norflex – Orphenadrine citrate tablet, extended release, Orphenadrine citrate injection. April 2007. Available from: http://dailymed.nlm.nih.gov/dailymed/search.cfm?labeltype=all&query=orphenadrine (Accessed February 2016).
    5. Papadikis, Maxine A., Stephen J. McPhee, and Michael W. Rabow. 2011 Current Medical Diagnosis & Treatment. New York: McGraw-Hill Education/Medical, 2011. Print
    6. Tintinalli JE, Stapczynski,JS, Ma OJ, Cline DM, Cydulka RK, Meckler GD. Emergency Medicine: A Comprehensive Study Guide. New York: McGraw-Hill Pub. Division; 2011
    Incorrect

    Correct answer: B, endoscopy. Due to the profuse amount of blood loss & that the patient is severely hypotensive with limited improvement in blood pressure after IV fluid resuscitation, it is imperative that the source of his bleed be found.  Upper GI endoscopy is the gold standard as it allows for direct visualization & therapeutic intervention to treat the active bleeding site & its ability to predict the likelihood of recurrent bleeding. In cases like this of severe active bleeding endoscopy should be performed as soon as the patient has been appropriately fluid resuscitated & is hemodynamically stable.

    There is already enough information to formulate a list of suspected diagnoses, which is why continuing the history and physical exam (C) in more detail is unnecessary and may actually be detrimental to the patient’s condition. In a patient this young there are very few possible diagnoses to suspect.  While his risk factors are minimal (tobacco use, male gender, medications), his presentation is strongly suggestive of a profuse upper GI bleed, of which PUD is the most common cause.  While a urea breath test (A) could be used to detect H. pylori, this would be an inappropriate test in this setting as it would not help determine the source of the patient’s GI bleed.  Likewise, an abdominal U/S (D) may assist in differentiating a source of right upper quadrant pain, however his combined history and physical exam give no indication that this would be helpful or necessary.

    References

    1. Swartz, M. Chapter 14 the abdomen. In: Textbook of Physical Diagnosis: History and Exam. 7th ed. Philadelphia, PA: Elsevier; 2014: 429-467.
    2. Williams, DA. Chapter 3: Gastrointestinal/nutrition. In: PANCE Prep Pearls: A practical comprehensive study and review guide for the PANCE and PANRE. 1st ed. Lexington, KY: CreateSpace; 2015: 154-161.
    3. Collins, RD. Hematemesis and melena, Rectal bleeding. In; Differential Diagnosis in Primary Care. 5th ed. Philadelphia, PA: Lippincott Williams & Wilkins; 2012: 208-213,367-371.
    4. National Institute of Health. Norflex – Orphenadrine citrate tablet, extended release, Orphenadrine citrate injection. April 2007. Available from: http://dailymed.nlm.nih.gov/dailymed/search.cfm?labeltype=all&query=orphenadrine (Accessed February 2016).
    5. Papadikis, Maxine A., Stephen J. McPhee, and Michael W. Rabow. 2011 Current Medical Diagnosis & Treatment. New York: McGraw-Hill Education/Medical, 2011. Print
    6. Tintinalli JE, Stapczynski,JS, Ma OJ, Cline DM, Cydulka RK, Meckler GD. Emergency Medicine: A Comprehensive Study Guide. New York: McGraw-Hill Pub. Division; 2011
  67. Question 67 of 220
    67. Question

    A 34-year-old otherwise healthy female presents to urgent care for a sore throat for the past four days. She complains of associated back pain, chills, and fevers with a Tmax of 104 degrees F.  She denies any headaches, fatigue, abdominal pain, nausea, vomiting, diarrhea, cough, ear pain, sinus congestion, gynecological or urinary symptoms.  She denies any sick contacts but admits to having a miscarriage 6 days prior and attributes her symptoms to this.  On examination, she is febrile at 102.8 degrees F, HR 90bpm and regular, and RR of 14.  Exam reveals obvious rigors, skin is warm and moist to touch without a rash, Moderate erythema to the pharynx, anterior cervical lymphadenopathy bilaterally, and tenderness to palpation along the para-lumbar musculature bilaterally is also noted.  The remainder of the exam is unremarkable. Based on the history and physical examination, what would be your first step in confirming your suspected diagnosis in this patient?

    Correct

    Correct answer: Rapid Strep Test followed by a Strep Culture if negative – This patient is presenting with the typical symptoms of streptococcal pharyngitis. Although the most common cause of pharyngitis is viral, this patient is presenting with signs and symptoms of a bacterial infection. This is most commonly caused by Group A Beta Hemolytic Streptococcus.1  The Centor Criteria for strep throat consists of: Fever (>100.4 degrees Fahrenheit), tender anterior cervical lymphadenopathy, absence of a cough, and pharyngotonsillar exudate.1 Each symptom is worth 1 point. Any patient with 2-3 points should get a throat culture.1 This patient presents with 3 of the 4 symptoms (fever, anterior cervical lymphadenopathy, and no cough), giving her a score of 3 points and therefore needing a throat culture.

    A rapid influenza test via nasopharyngeal swab is incorrect. The common symptoms for influenza do include fever, myalgias, and chills but also a cough, nasal congestion, coryza, and headaches.2  This patient is not experiencing a majority of these symptoms, so therefore a rapid influenza test would not be the best choice.

    MonoSpot test followed by Epstein Barr titers is incorrect. Mononucleosis is a good option to keep in the differential, but a key symptom that would be a negative indication of mononucleosis is “anterior” cervical lymphadenopathy. Most patients with mononucleosis will have posterior cervical lymphadenopathy.1  Although patients with mononucleosis will have a sore throat and fever, they will also complain of severe fatigue as well as possible abdominal discomfort.

    Do nothing because it’s viral, is not the correct answer. This patient has been experiencing high fevers for multiple days which would indicate more of a bacterial infection rather than viral. Most viral infections have lower grade fevers as opposed to bacterial infections where fevers tend to be higher.3 Also, in bacterial infections the fever will get worse a few days into the illness rather than getting better.3

    Resources:

    1. Williams D.A. PANCE Prep Pearls. CreateSpace. 2014.
    2. Papadakis M, McPhee S, Rabow M. Current Medical Diagnosis & Treatment 2015. 2015
    3. Duke University Health System. Is it a Bacterial Infection or Virus?. DukeMedicine. https://www.dukemedicine.org/blog/it-bacterial-infection-or-virus. Published Oct. 1, 2013. Accessed Jan. 25, 2016.
    4. Wald, E. R. Approach to diagnosis of acute infectious pharyngitis in children and adolescents. UpToDate. http://www.uptodate.com/contents/approach-to-diagnosis-of-acute-infectious-pharyngitis-in-children-and-adolescents?source=search_result. Published Aug. 11, 2015. Accessed Jan. 28, 2016.
    5. Marquez-Carrillo M.A. Bacterial Pharyngitis. Medscape. http://emedicine.medscape.com/article/225243-overview. Published Jul. 28, 2015. Accessed Feb. 1, 2016.
    Incorrect

    Correct answer: Rapid Strep Test followed by a Strep Culture if negative – This patient is presenting with the typical symptoms of streptococcal pharyngitis. Although the most common cause of pharyngitis is viral, this patient is presenting with signs and symptoms of a bacterial infection. This is most commonly caused by Group A Beta Hemolytic Streptococcus.1  The Centor Criteria for strep throat consists of: Fever (>100.4 degrees Fahrenheit), tender anterior cervical lymphadenopathy, absence of a cough, and pharyngotonsillar exudate.1 Each symptom is worth 1 point. Any patient with 2-3 points should get a throat culture.1 This patient presents with 3 of the 4 symptoms (fever, anterior cervical lymphadenopathy, and no cough), giving her a score of 3 points and therefore needing a throat culture.

    A rapid influenza test via nasopharyngeal swab is incorrect. The common symptoms for influenza do include fever, myalgias, and chills but also a cough, nasal congestion, coryza, and headaches.2  This patient is not experiencing a majority of these symptoms, so therefore a rapid influenza test would not be the best choice.

    MonoSpot test followed by Epstein Barr titers is incorrect. Mononucleosis is a good option to keep in the differential, but a key symptom that would be a negative indication of mononucleosis is “anterior” cervical lymphadenopathy. Most patients with mononucleosis will have posterior cervical lymphadenopathy.1  Although patients with mononucleosis will have a sore throat and fever, they will also complain of severe fatigue as well as possible abdominal discomfort.

    Do nothing because it’s viral, is not the correct answer. This patient has been experiencing high fevers for multiple days which would indicate more of a bacterial infection rather than viral. Most viral infections have lower grade fevers as opposed to bacterial infections where fevers tend to be higher.3 Also, in bacterial infections the fever will get worse a few days into the illness rather than getting better.3

    Resources:

    1. Williams D.A. PANCE Prep Pearls. CreateSpace. 2014.
    2. Papadakis M, McPhee S, Rabow M. Current Medical Diagnosis & Treatment 2015. 2015
    3. Duke University Health System. Is it a Bacterial Infection or Virus?. DukeMedicine. https://www.dukemedicine.org/blog/it-bacterial-infection-or-virus. Published Oct. 1, 2013. Accessed Jan. 25, 2016.
    4. Wald, E. R. Approach to diagnosis of acute infectious pharyngitis in children and adolescents. UpToDate. http://www.uptodate.com/contents/approach-to-diagnosis-of-acute-infectious-pharyngitis-in-children-and-adolescents?source=search_result. Published Aug. 11, 2015. Accessed Jan. 28, 2016.
    5. Marquez-Carrillo M.A. Bacterial Pharyngitis. Medscape. http://emedicine.medscape.com/article/225243-overview. Published Jul. 28, 2015. Accessed Feb. 1, 2016.
  68. Question 68 of 220
    68. Question

    A 25 yo female presents to the emergency department complaining of sudden onset of crampy intermittent right flank pain that radiates toward the groin for 2 hours & now with nausea & vomiting for the past 1 hour. The patient has no past medical history, takes no medications & has no allergies to medications. Her last menstrual period was one week ago. On exam the patient is anxious, unable to find a comfortable position & is writhing in pain. She is afebrile, slightly tachycardic, abdominal & pelvic exam is benign, urinalysis shows large microscopic hematuria, & urine HCG is negative. The most likely diagnosis is?

    Correct

    The correct answer is Nephrolithiasis (kidney stone) – The classic symptom complex is an acute onset of a colic like pain (crampy intermittent pain) that originates in the flank & radiates to the groin. The pain originates from the hollow viscus ureter & is visceral in nature without any associated peritoneal irritation & is due to the subsequent hydronephrosis creating pressure against Gerota fascia causing flank pain. Stones contained within the kidneys themselves are usually painless, it is once they are released into the ureter that the pain begins. In contrast to the patient with an acute abdomen the patients with renal colic are constantly moving & the pain is wave like & episodic. The pain is commonly accompanied by nausea & vomiting & the adrenergic response to the pain can result in tachycardia. Microscopic hematuria is present in approximately 85% of patients with renal colic, whereas only 30% have gross hematuria. The location of the pain correlates somewhat with the location of the stone. Patients who have a stone in the upper ureter are more likely to have flank pain, whereas those with a mid-ureter stone the pain radiates to the lower anterior quadrant of the abdomen. 75% of stones diagnosed are in the distal ureter & refer the pain to the groin. Stones positioned at the urethral vesicular junction (UVJ) can mimic a urinary tract infection (UTI) with frequency, urgency & dysuria. Confirmation, position & measurement of the size of the stone is achieved with non-contrast abdominal & pelvic CT. Treatment for symptomatic nephrolithiasis includes IV hydration & pain control. NSAID’s are the primary choice (Ketorolac 15-30 mg IV) as they have a direct action on the ureter by inhibiting prostaglandin synthesis. IV or po narcotics may also be indicated but do not affect the cause of the pain. IV antiemetics are indicated for nausea & vomiting. In most cases stones measuring < 5 mm will pass & the patient can be discharged home, if there is no associated infection, with a prescription for an oral opiate & NSAID & instructions to strain their urine & follow up with their PCP and/or urology. Those patients with a stone > 5 mm should be directly referred to urology for intervention as the stone most likely will not pass. In patients who have an associated infection with obstruction, fever & systemic illness admission is indicated & they should be started on IV antibiotics gentamycin or tobramycin 3 mg/kg/day every 8 hours in divided doses plus one of the following ampicillin 1-2 grams IV every 4 hours, piperacillin-tazobactam 3.375 g IV every 6 hours, cefepime 2 grams IV every 8 hours, ticaricillin-clavulinic acid 3.1 grams IV every 6 hours or ciprofloxacin 400 mg IV every 12 hours based on culture & sensitivities.

    Pyelonephritis is an infection of the upper urinary tract that can present in a similar fashion to nephrolithiasis but almost always presents with fever which may be absent in an uncomplicated nephrolithiasis. If CT is performed emphysematous pyelonephritis may be detected instead of the urethral stone.

    Abdominal Aortic Aneurysm (AAA) must be considered in all patients with risk factors as many times rupturing or expanding AAA is misdiagnosed as nephrolithiasis & CT will help to evaluate.

    Cholecystitis can also present similar to nephrolithiasis especially if biliary colic is present, however on exam there is usually right upper quadrant tenderness to palpation that is not normally found with nephrolithiasis.

    Pancreatitis can also present similar to nephrolithiasis but will usually have diffuse tenderness to palpation of the abdomen & will have an elevated lipase.

    References:

    Longo, DL, Kasper D, Jameson J, Fauci A, Hauser S, Loscalzo J. Harrison’s Principles of Internal Medicine. New York: McGraw-Hill; 2012

    Mahadevan SV, Garmel GM. An Introduction to Clinical Emergency Medicine. Cambridge: Cambridge University Press; 2012

    McCance KL, Huether SE, Brashers VL, Rote NS, Pathophysiology The Biological  Basis for Disease in Adults and Children. Missouri: Mosby Elsevier; 2010

    Papadikis, Maxine A., Stephen J. McPhee, and Michael W. Rabow. 2011 Current Medical Diagnosis & Treatment. New York: McGraw-Hill Education/Medical, 2011. Print

    Tintinalli JE, Stapczynski,JS, Ma OJ, Cline DM, Cydulka RK, Meckler GD. Emergency Medicine: A Comprehensive Study Guide. New York: McGraw-Hill Pub. Division; 2011

    Incorrect

    The correct answer is Nephrolithiasis (kidney stone) – The classic symptom complex is an acute onset of a colic like pain (crampy intermittent pain) that originates in the flank & radiates to the groin. The pain originates from the hollow viscus ureter & is visceral in nature without any associated peritoneal irritation & is due to the subsequent hydronephrosis creating pressure against Gerota fascia causing flank pain. Stones contained within the kidneys themselves are usually painless, it is once they are released into the ureter that the pain begins. In contrast to the patient with an acute abdomen the patients with renal colic are constantly moving & the pain is wave like & episodic. The pain is commonly accompanied by nausea & vomiting & the adrenergic response to the pain can result in tachycardia. Microscopic hematuria is present in approximately 85% of patients with renal colic, whereas only 30% have gross hematuria. The location of the pain correlates somewhat with the location of the stone. Patients who have a stone in the upper ureter are more likely to have flank pain, whereas those with a mid-ureter stone the pain radiates to the lower anterior quadrant of the abdomen. 75% of stones diagnosed are in the distal ureter & refer the pain to the groin. Stones positioned at the urethral vesicular junction (UVJ) can mimic a urinary tract infection (UTI) with frequency, urgency & dysuria. Confirmation, position & measurement of the size of the stone is achieved with non-contrast abdominal & pelvic CT. Treatment for symptomatic nephrolithiasis includes IV hydration & pain control. NSAID’s are the primary choice (Ketorolac 15-30 mg IV) as they have a direct action on the ureter by inhibiting prostaglandin synthesis. IV or po narcotics may also be indicated but do not affect the cause of the pain. IV antiemetics are indicated for nausea & vomiting. In most cases stones measuring < 5 mm will pass & the patient can be discharged home, if there is no associated infection, with a prescription for an oral opiate & NSAID & instructions to strain their urine & follow up with their PCP and/or urology. Those patients with a stone > 5 mm should be directly referred to urology for intervention as the stone most likely will not pass. In patients who have an associated infection with obstruction, fever & systemic illness admission is indicated & they should be started on IV antibiotics gentamycin or tobramycin 3 mg/kg/day every 8 hours in divided doses plus one of the following ampicillin 1-2 grams IV every 4 hours, piperacillin-tazobactam 3.375 g IV every 6 hours, cefepime 2 grams IV every 8 hours, ticaricillin-clavulinic acid 3.1 grams IV every 6 hours or ciprofloxacin 400 mg IV every 12 hours based on culture & sensitivities.

    Pyelonephritis is an infection of the upper urinary tract that can present in a similar fashion to nephrolithiasis but almost always presents with fever which may be absent in an uncomplicated nephrolithiasis. If CT is performed emphysematous pyelonephritis may be detected instead of the urethral stone.

    Abdominal Aortic Aneurysm (AAA) must be considered in all patients with risk factors as many times rupturing or expanding AAA is misdiagnosed as nephrolithiasis & CT will help to evaluate.

    Cholecystitis can also present similar to nephrolithiasis especially if biliary colic is present, however on exam there is usually right upper quadrant tenderness to palpation that is not normally found with nephrolithiasis.

    Pancreatitis can also present similar to nephrolithiasis but will usually have diffuse tenderness to palpation of the abdomen & will have an elevated lipase.

    References:

    Longo, DL, Kasper D, Jameson J, Fauci A, Hauser S, Loscalzo J. Harrison’s Principles of Internal Medicine. New York: McGraw-Hill; 2012

    Mahadevan SV, Garmel GM. An Introduction to Clinical Emergency Medicine. Cambridge: Cambridge University Press; 2012

    McCance KL, Huether SE, Brashers VL, Rote NS, Pathophysiology The Biological  Basis for Disease in Adults and Children. Missouri: Mosby Elsevier; 2010

    Papadikis, Maxine A., Stephen J. McPhee, and Michael W. Rabow. 2011 Current Medical Diagnosis & Treatment. New York: McGraw-Hill Education/Medical, 2011. Print

    Tintinalli JE, Stapczynski,JS, Ma OJ, Cline DM, Cydulka RK, Meckler GD. Emergency Medicine: A Comprehensive Study Guide. New York: McGraw-Hill Pub. Division; 2011

  69. Question 69 of 220
    69. Question

    A 34 yo mildly obese male patient presents to the emergency department complaining of left sided chest pain worse with deep inspiration & shortness of breath. He denies nausea, vomiting, diaphoresis, radiation of pain or syncope. On exam his pulse is 110 & regular, respirations are 22/minute, blood pressure is 134/84 mm Hg, lungs are clear to auscultation bilaterally, normal S1S2, rate is tachycardic without murmurs rubs or gallops & there is no pedal edema. EKG shows sinus tachycardia with right ventricular strain pattern with T-wave inversion in V1 – V4 & leads II, III & aVF. What is the most likely diagnosis?

    Correct

    Correct Answer: D. Pulmonary embolism: PE is very difficult to diagnose & you must be vigilant to ensure that you do not miss it, as the clinical presentation is variable. The clinical findings depend on both the size of the embolus & the patient’s pre-existing cardiovascular status. The utilization of the PERC criteria & Wells criteria have been very helpful in determining whether to pursue PE as a diagnosis. The PERC rule can be applied to patients where the diagnosis of PE is being considered, but the patient is deemed low-risk. A patient deemed low-risk by provider’s gestalt who is also The hallmark of PE is dyspnea unexplained by auscultory findings, ECG changes, or obvious diagnosis on chest x-ray. Dyspnea occurs in 75-85% & pain on inspiration occurs in 65-75% of patients with PE. Routine cardiopulmonary testing in the ED generally demonstrates non-specific findings. The most common findings for PE on EKG are sinus tachycardia seen in about 44% of all cases & non-specific ST-T wave changes. When the PE causes the right ventricular systolic pressure to exceed 40 mm Hg the EKG begins to manifest more specific changes including T wave inversion in V1-V4 & leads II, III, & aVF, incomplete or complete right bundle branch block.. However an S1-Q3-T3 indicative of cor pulmonale is the classic finding on EKG for PE. On chest x-ray the most common finding is clear lungs & absence of pulmonary edema. However the Westermark sign from complete lobar artery obstruction & Hampton’s Hump indicative of pulmonary infarction are relatively specific findings seen in <5% of patient’s. The definitive diagnostic test is chest CT angiography which identifies a clot as a filling defect in contrast enhanced pulmonary arteries.

    The time to treatment with therapeutic anticoagulation has an inverse relationship with outcome & anticoagulation should be initiated in any patient with high suspicion for PE while waiting for the diagnostic confirmation. The use of low molecular weight heparin (1 mg/kg BID or 1.5 mg/kg daily) as a bridge to therapeutic anticoagulation with Warfarin sodium (dosing based on the INR) has become the standard of treatment.

    References

    Slovis B. MDCalc. http://www.mdcalc.com/wells-criteria-for-pulmonary-embolism-pe/. Accessed June 16, 2016

    Slovis B. MDCalc. http://www.mdcalc.com/perc-rule-for-pulmonary-embolism/. Accessed June 16, 2016

    Longo, DL, Kasper D, Jameson J, Fauci A, Hauser S, Loscalzo J. Harrison’s Principles of Internal Medicine. New York: McGraw-Hill; 2012

    Mahadevan SV, Garmel GM. An Introduction to Clinical Emergency Medicine. Cambridge: Cambridge University Press; 2012

    Papadikis, Maxine A., Stephen J. McPhee, and Michael W. Rabow. 2011 Current Medical Diagnosis & Treatment. New York: McGraw-Hill Education/Medical, 2011. Print

    Tintinalli JE, Stapczynski,JS, Ma OJ, Cline DM, Cydulka RK, Meckler GD. Emergency Medicine: A Comprehensive Study Guide. New York: McGraw-Hill Pub. Division; 2011

    Incorrect

    Correct Answer: D. Pulmonary embolism: PE is very difficult to diagnose & you must be vigilant to ensure that you do not miss it, as the clinical presentation is variable. The clinical findings depend on both the size of the embolus & the patient’s pre-existing cardiovascular status. The utilization of the PERC criteria & Wells criteria have been very helpful in determining whether to pursue PE as a diagnosis. The PERC rule can be applied to patients where the diagnosis of PE is being considered, but the patient is deemed low-risk. A patient deemed low-risk by provider’s gestalt who is also The hallmark of PE is dyspnea unexplained by auscultory findings, ECG changes, or obvious diagnosis on chest x-ray. Dyspnea occurs in 75-85% & pain on inspiration occurs in 65-75% of patients with PE. Routine cardiopulmonary testing in the ED generally demonstrates non-specific findings. The most common findings for PE on EKG are sinus tachycardia seen in about 44% of all cases & non-specific ST-T wave changes. When the PE causes the right ventricular systolic pressure to exceed 40 mm Hg the EKG begins to manifest more specific changes including T wave inversion in V1-V4 & leads II, III, & aVF, incomplete or complete right bundle branch block.. However an S1-Q3-T3 indicative of cor pulmonale is the classic finding on EKG for PE. On chest x-ray the most common finding is clear lungs & absence of pulmonary edema. However the Westermark sign from complete lobar artery obstruction & Hampton’s Hump indicative of pulmonary infarction are relatively specific findings seen in <5% of patient’s. The definitive diagnostic test is chest CT angiography which identifies a clot as a filling defect in contrast enhanced pulmonary arteries.

    The time to treatment with therapeutic anticoagulation has an inverse relationship with outcome & anticoagulation should be initiated in any patient with high suspicion for PE while waiting for the diagnostic confirmation. The use of low molecular weight heparin (1 mg/kg BID or 1.5 mg/kg daily) as a bridge to therapeutic anticoagulation with Warfarin sodium (dosing based on the INR) has become the standard of treatment.

    References

    Slovis B. MDCalc. http://www.mdcalc.com/wells-criteria-for-pulmonary-embolism-pe/. Accessed June 16, 2016

    Slovis B. MDCalc. http://www.mdcalc.com/perc-rule-for-pulmonary-embolism/. Accessed June 16, 2016

    Longo, DL, Kasper D, Jameson J, Fauci A, Hauser S, Loscalzo J. Harrison’s Principles of Internal Medicine. New York: McGraw-Hill; 2012

    Mahadevan SV, Garmel GM. An Introduction to Clinical Emergency Medicine. Cambridge: Cambridge University Press; 2012

    Papadikis, Maxine A., Stephen J. McPhee, and Michael W. Rabow. 2011 Current Medical Diagnosis & Treatment. New York: McGraw-Hill Education/Medical, 2011. Print

    Tintinalli JE, Stapczynski,JS, Ma OJ, Cline DM, Cydulka RK, Meckler GD. Emergency Medicine: A Comprehensive Study Guide. New York: McGraw-Hill Pub. Division; 2011

  70. Question 70 of 220
    70. Question

    A 36 year old obese female presents to the emergency room with complaints of a throbbing headache rated at a 9/10 and horizontal diplopia and photopsia. The headache pain worsens with positional changes and Valsalva maneuvers. The patient’s past medical history includes gestational diabetes and polycystic ovarian syndrome controlled with oral contraceptives. The patient states she was recently treated for suspected Lyme disease with doxycycline. On physical exam, funduscopic exam reveals bilateral papilledema. The remainder of her examination is benign.  Brain MRI is negative and lumbar puncture reveals elevated pressures only.  Which of the following best describes the patients presenting diagnosis?

    Correct

    The correct answer is (C) idiopathic intracranial hypertension. This is the correct choice based on the symptoms of headache, blurry vision, papilledema, no MRI brain findings and elevated cerebrospinal fluid pressures. The patient was also recently treated with a tetracycline, is overweight and suffers from polycystic ovarian syndrome. The answer cannot be (B) tension headache because there is papilledema and no history of pulsatile headache. (D) Subarachnoid hemorrhage and (E) glioblastoma multiforme are ruled out based on the negative MRI findings. (A) Migraine headache is more difficult to rule out but typically, patients don’t have papilledema, and concerning vision symptoms, photophobia is more common than blurry vision.

    Idiopathic intracranial hypertension affects 1-2 per 100,000 yearly, it is vastly more common in the obese, especially women between the ages of 15 and 44 years old, although it can occur at any age and any habitus. 1

    Risk factors include women of childbearing age, recent weight gain of over 4 pounds in recent months. 1 Some drugs appear to have an increased risk of IIH including growth hormone, tetracycline antibiotics, hypervitaminosis A and retinoids including retinoic acid, retinol and isotretinoin. 1  Associated conditions include Addison’s disease, hypoparathyroidism, anemia, sleep apnea, Lupus, Behcet’s Syndrome, PCOS, coagulation disorders, uremia and hypovitaminosis A. 1

    The etiology is unknown, it must have an explanation as to why there is increased intracranial pressure. Some people believe there is a cerebral venous outflow abnormality in these patients with IIH. The most prominent and significant finding is the venous sinus thrombosis. Less supported theories include central obesity functioning in increasing intracranial pressure and cerebral edema, which is not supported by MRI. An inconsistent theory is based off of vitamin A intoxication, but not all patients have this feature. 1

    The most common symptom of IIH is headache, followed by visual effects that are usually transient and photopsia. Patients can also present with complaints of pain in the back and retrobulbar areas. Additionally, pulsatile tinnitus and more visual effects including diplopia or sustained visual loss. 2

    As the most common presenting symptom, headaches are usually described as more severe than normal and they may have a throbbing, pulsatile nature to the pain. Headaches can be daily or less frequent and sometimes are associated with nausea and vomiting.2 In patients who present with papilledema, many will complain of visual symptoms, including bouts of transient obstruction in their visual fields along with photopsia. These vision changes are more common when a patient changes position, with valsalva maneuvers, light and with movement of the eyeball. 2

    Tinnitus with pulsations is common in IIH in combination with a headache, the two together strongly suggest an IIH diagnosis. The sound of running water or wind can also be considered tinnitus in this case.2 Lastly, patients complain of diplopia, this is most commonly horizontal and it is associated with a sixth nerve palsy that develops from increased intracranial pressure. 2

    On physical exam papilledema, visual field loss and sixth nerve palsy are the most common presenting signs.2 Papilledema, found on fundoscopic exam, is one of the hallmark signs of IIH and it can be bilateral or unilateral. Also on fundoscopic exam one will find flame hemorrhages and macular exudates.2 In the event subretinal hemorrhages are present with papilledema a poor prognosis of visual return is likely. 2

    While testing the extraoccular eye movements, one may find a sixth never palsy when asking the patient to follow their eyes horizontally from right to left. The positive finding involves the effected eye not fully moving outward. This is commonly associated with the symptom, diplopia.2 Visual loss is a poor prognostic indicator, some patients will lack return of their vision permanently. Visual acuity is less commonly affected than visual field loss. 2

    The patient needs to have blood testing to check a complete blood count, erythrocyte sedimentation rate, serum iron and iron-binding capacity, antinuclear antigen profile and a coagulation profile.2 The ANA testing is to specifically rule out lupus erythematous. Lyme screening is also common in those with a history due to the association between the two illnesses. 2

    It is also required to run a MRI of the brain with contrast to first look for any masses that could be causing headache or diplopia and papilledema.2 In a patient with IIH, the MRI studies should be normal. You can also use ultrasound to measure the diameter of the optic nerve. 2

    Finally, the best diagnostic study is the lumbar puncture, this is indicated after a mass is ruled out using the MRI. With the lumbar puncture, the opening pressure should be increased well above the normal range of 12-20cm H2O. The cerebrospinal fluid is collected and sent for cell count, cytology, culture, glucose, protein and electrolyte counts.2 In the diagnostic radiology department, lumbar punctures are performed prone with only local lidocaine, the x-ray’s guide the needle into the spinal canal where CSF is easily drawn off.

    It is important for patients to try to make lifestyle changes that promote weight loss. If they can’t loose weight or continue to have symptoms they can try medical therapy.3 The first line pharmacologic therapy is Diamox, this therapy is best at removing the headache from IIH symptoms. Dosing can be 500 mg twice daily with room to increase until you reach 3-4 grams per day. At high doses patients experience more adverse effects from the Diamox, including paresthesias, fatigue, decreased libido and metallic tastes. If a patient can’t tolerate Diamox, the second line treatment is furosemide, but it doesn’t have the same efficacy. 3Patients can also consider bariatric surgery for their treatment in effort to lose weight. 3

    When patients lack papilledema, but present with chronic headaches, standard migraine headache prophylaxis can also help, examples are amitriptyline, Topamax and propranolol. In a patient with severe papilledema, steroids can help to reduce inflammation in the optic disc and optic nerve, this will assist with vision restoration. Patients can not go on chronic steroids for IIH, the side effects are not worth the benefit. 3

    A lumbar puncture with removal of some CSF fluid should provide immediate relief for patients with IIH. If there is a need for frequent lumbar puncture, there is an option to create a CSF shunt. Surgeons prefer to create a ventricular shunt either ventirculoperitoneal or ventriculoatrial shunts. 3 They like these positions because the valve of the shunt can be manipulated from the skin surface. The shunt creates a path from the intracranial space into the abdomen. When the pressure is to high, the shunt is told to release CSF into the abdomen for the body to absorb and destroy. 3

    Optic nerve sheath fenestration is a treatment option for patients who struggle with continued vision loss due to papilledema, the fenestration is a creation of slits in the dura around the optic nerve. The slits allow CSF to leak into the orbit where the body absorbs it. There are contraindications to fenestration including: diplopia, optic nerve injury, vessel occlusion or hemorrhage, tonic pupils and infection. 3

    Most patients tolerate treatment in the outpatient setting, in the rare case of admission a common reason why is due to visual loss that is chronic with acute worsening. These patients will receive high dose intravenous steroids. If there is no response to steroids and surgery is not an option, the second line pharmacologic treatment is intravenous mannitol, the goal here is to decrease the CSF pressure which should also create relief for the optic nerve. 3

    References:

    1. Lee, Andrew G, Wall, Michael. Idiopathic Intracranial Hypertension (pseudotumor cerebri): Epidemiology and Pathogenesis. https://www.uptodate.com/contents/idiopathic-intracranial-hypertension-pseudotumor-cerebri-epidemiology-and-pathogenesis?source=search_result&search=Idiopathic+intracranial+hypertension&selectedTitle=3%7E72. Accessed May 23, 2016.
    2. Lee, Andrew G, Wall, Michael. Idiopathic Intracranial Hypertension (pseudotumor cerebri): Clinical Features and Diagnosis. UpToDate. https://www.uptodate.com/contents/idiopathic-intracranial-hypertension-pseudotumor-cerebri-clinical-features-and-diagnosis?source=search_result&search=pseudotumor+cerebri&selectedTitle=1%7E72. Accessed May 23, 2016.
    3. Gans, Mark S. Idiopathic Intracranial Hypertension Treatment & Management. Medscape. http://emedicine.medscape.com/article/1214410-treatment#showall. Accessed May 23, 2016.
    Incorrect

    The correct answer is (C) idiopathic intracranial hypertension. This is the correct choice based on the symptoms of headache, blurry vision, papilledema, no MRI brain findings and elevated cerebrospinal fluid pressures. The patient was also recently treated with a tetracycline, is overweight and suffers from polycystic ovarian syndrome. The answer cannot be (B) tension headache because there is papilledema and no history of pulsatile headache. (D) Subarachnoid hemorrhage and (E) glioblastoma multiforme are ruled out based on the negative MRI findings. (A) Migraine headache is more difficult to rule out but typically, patients don’t have papilledema, and concerning vision symptoms, photophobia is more common than blurry vision.

    Idiopathic intracranial hypertension affects 1-2 per 100,000 yearly, it is vastly more common in the obese, especially women between the ages of 15 and 44 years old, although it can occur at any age and any habitus. 1

    Risk factors include women of childbearing age, recent weight gain of over 4 pounds in recent months. 1 Some drugs appear to have an increased risk of IIH including growth hormone, tetracycline antibiotics, hypervitaminosis A and retinoids including retinoic acid, retinol and isotretinoin. 1  Associated conditions include Addison’s disease, hypoparathyroidism, anemia, sleep apnea, Lupus, Behcet’s Syndrome, PCOS, coagulation disorders, uremia and hypovitaminosis A. 1

    The etiology is unknown, it must have an explanation as to why there is increased intracranial pressure. Some people believe there is a cerebral venous outflow abnormality in these patients with IIH. The most prominent and significant finding is the venous sinus thrombosis. Less supported theories include central obesity functioning in increasing intracranial pressure and cerebral edema, which is not supported by MRI. An inconsistent theory is based off of vitamin A intoxication, but not all patients have this feature. 1

    The most common symptom of IIH is headache, followed by visual effects that are usually transient and photopsia. Patients can also present with complaints of pain in the back and retrobulbar areas. Additionally, pulsatile tinnitus and more visual effects including diplopia or sustained visual loss. 2

    As the most common presenting symptom, headaches are usually described as more severe than normal and they may have a throbbing, pulsatile nature to the pain. Headaches can be daily or less frequent and sometimes are associated with nausea and vomiting.2 In patients who present with papilledema, many will complain of visual symptoms, including bouts of transient obstruction in their visual fields along with photopsia. These vision changes are more common when a patient changes position, with valsalva maneuvers, light and with movement of the eyeball. 2

    Tinnitus with pulsations is common in IIH in combination with a headache, the two together strongly suggest an IIH diagnosis. The sound of running water or wind can also be considered tinnitus in this case.2 Lastly, patients complain of diplopia, this is most commonly horizontal and it is associated with a sixth nerve palsy that develops from increased intracranial pressure. 2

    On physical exam papilledema, visual field loss and sixth nerve palsy are the most common presenting signs.2 Papilledema, found on fundoscopic exam, is one of the hallmark signs of IIH and it can be bilateral or unilateral. Also on fundoscopic exam one will find flame hemorrhages and macular exudates.2 In the event subretinal hemorrhages are present with papilledema a poor prognosis of visual return is likely. 2

    While testing the extraoccular eye movements, one may find a sixth never palsy when asking the patient to follow their eyes horizontally from right to left. The positive finding involves the effected eye not fully moving outward. This is commonly associated with the symptom, diplopia.2 Visual loss is a poor prognostic indicator, some patients will lack return of their vision permanently. Visual acuity is less commonly affected than visual field loss. 2

    The patient needs to have blood testing to check a complete blood count, erythrocyte sedimentation rate, serum iron and iron-binding capacity, antinuclear antigen profile and a coagulation profile.2 The ANA testing is to specifically rule out lupus erythematous. Lyme screening is also common in those with a history due to the association between the two illnesses. 2

    It is also required to run a MRI of the brain with contrast to first look for any masses that could be causing headache or diplopia and papilledema.2 In a patient with IIH, the MRI studies should be normal. You can also use ultrasound to measure the diameter of the optic nerve. 2

    Finally, the best diagnostic study is the lumbar puncture, this is indicated after a mass is ruled out using the MRI. With the lumbar puncture, the opening pressure should be increased well above the normal range of 12-20cm H2O. The cerebrospinal fluid is collected and sent for cell count, cytology, culture, glucose, protein and electrolyte counts.2 In the diagnostic radiology department, lumbar punctures are performed prone with only local lidocaine, the x-ray’s guide the needle into the spinal canal where CSF is easily drawn off.

    It is important for patients to try to make lifestyle changes that promote weight loss. If they can’t loose weight or continue to have symptoms they can try medical therapy.3 The first line pharmacologic therapy is Diamox, this therapy is best at removing the headache from IIH symptoms. Dosing can be 500 mg twice daily with room to increase until you reach 3-4 grams per day. At high doses patients experience more adverse effects from the Diamox, including paresthesias, fatigue, decreased libido and metallic tastes. If a patient can’t tolerate Diamox, the second line treatment is furosemide, but it doesn’t have the same efficacy. 3Patients can also consider bariatric surgery for their treatment in effort to lose weight. 3

    When patients lack papilledema, but present with chronic headaches, standard migraine headache prophylaxis can also help, examples are amitriptyline, Topamax and propranolol. In a patient with severe papilledema, steroids can help to reduce inflammation in the optic disc and optic nerve, this will assist with vision restoration. Patients can not go on chronic steroids for IIH, the side effects are not worth the benefit. 3

    A lumbar puncture with removal of some CSF fluid should provide immediate relief for patients with IIH. If there is a need for frequent lumbar puncture, there is an option to create a CSF shunt. Surgeons prefer to create a ventricular shunt either ventirculoperitoneal or ventriculoatrial shunts. 3 They like these positions because the valve of the shunt can be manipulated from the skin surface. The shunt creates a path from the intracranial space into the abdomen. When the pressure is to high, the shunt is told to release CSF into the abdomen for the body to absorb and destroy. 3

    Optic nerve sheath fenestration is a treatment option for patients who struggle with continued vision loss due to papilledema, the fenestration is a creation of slits in the dura around the optic nerve. The slits allow CSF to leak into the orbit where the body absorbs it. There are contraindications to fenestration including: diplopia, optic nerve injury, vessel occlusion or hemorrhage, tonic pupils and infection. 3

    Most patients tolerate treatment in the outpatient setting, in the rare case of admission a common reason why is due to visual loss that is chronic with acute worsening. These patients will receive high dose intravenous steroids. If there is no response to steroids and surgery is not an option, the second line pharmacologic treatment is intravenous mannitol, the goal here is to decrease the CSF pressure which should also create relief for the optic nerve. 3

    References:

    1. Lee, Andrew G, Wall, Michael. Idiopathic Intracranial Hypertension (pseudotumor cerebri): Epidemiology and Pathogenesis. https://www.uptodate.com/contents/idiopathic-intracranial-hypertension-pseudotumor-cerebri-epidemiology-and-pathogenesis?source=search_result&search=Idiopathic+intracranial+hypertension&selectedTitle=3%7E72. Accessed May 23, 2016.
    2. Lee, Andrew G, Wall, Michael. Idiopathic Intracranial Hypertension (pseudotumor cerebri): Clinical Features and Diagnosis. UpToDate. https://www.uptodate.com/contents/idiopathic-intracranial-hypertension-pseudotumor-cerebri-clinical-features-and-diagnosis?source=search_result&search=pseudotumor+cerebri&selectedTitle=1%7E72. Accessed May 23, 2016.
    3. Gans, Mark S. Idiopathic Intracranial Hypertension Treatment & Management. Medscape. http://emedicine.medscape.com/article/1214410-treatment#showall. Accessed May 23, 2016.
  71. Question 71 of 220
    71. Question

    A 25-year-old G4P1031 presents to the Emergency Department with severe lower abdominal pain.  She is tender to palpation in the RLQ and has adnexal fullness and cervical motion tenderness.  She states she has not had a menses for three months, but has had sporadic vaginal bleeding.  There is a 2cm mass seen on transvaginal ultrasound at the junction of the uterus and fallopian tube.  Her quantitative B-hCG was 1200 in the ED and 1400 when it was taken 48 hours later.  Which of the following is the most likely diagnosis?

    Correct

    Correct Answer: (C): An ectopic pregnancy is one in which the fertilized egg implants outside of the endometrial cavity, most commonly the ampulla of the fallopian tube.1,2 The classic triad of ectopic pregnancy is abdominal pain, amenorrhea, and vaginal bleeding.1 The B-hCG is utilized to differentiate between normal and abnormal pregnancies.1,2 This patient’s B-hCG was below the discriminatory zone of 2000 mIU/mL, which is the level in which a gestational sac can be recognized on transvaginal ultrasound imaging studies.1 In a normal pregnancy the B-hCG should double every 48-72 hours for the first 40 days of pregnancy.1,2 Since this was not the case, an ectopic pregnancy is the most likely diagnosis.  This patient should undergo a Dilation and Curettage or Methotrexate therapy as to avoid rupture of the ectopic pregnancy.1,2 If this were to be a (D) spontaneous abortion, the B-hCG should have decreased in 48 hours, and would have continued to decrease in each serum test thereafter. (B) Ovarian cyst is in the differential diagnoses because a mass measuring 2cm in early pregnancy with lower abdominal pain lateralized to one side has potential to be a corpus luteal cyst, which is formed at the site of ovulation. An ovarian cyst would likely present with adnexal tenderness rather than fullness.3 The corpus luteum cyst will start to regress around 10 weeks in pregnancy.3 (A) Pelvic Inflammatory Disease (PID) often presents with cervical motion tenderness, chronic lower abdominal pain and abnormal uterine bleeding or dysmenorrhea.4 Often times, the patient has a fever and an increased WBC count.5 The most common organisms responsible for PID are Neisseria gonorrhoeae and Chlamydia trachomitis.4,5 The patient presenting with PID would not have a positive B-hCG.

    References

    Sepilian VP. Ectopic Pregnancy. Medscape. http://emedicine.medscape.com/article/256448-clinical#b3. Updated November 15, 2015. Accessed June 28, 2016.

    Tulandi T. Clinical manifestations, diagnosis, and management of ectopic pregnancy. UpToDate. http://www.uptodate.com. 2016. Accessed June 28, 2016.

    Sharp HT. Evaluation and management of ruptured ovarian cyst. UpToDate. http://www.uptodate.com. 2016. Updated November 25, 2015. Accessed June 28, 2016.

    Shepherd SM. Pelvic Inflammatory Disease. Medscape. http://emedicine.medscape.com/article/256448-clinical#b3. Updated September 28, 2015. Accessed June 28, 2016.

    O’Connell CB. Gynecology. In: A comprehensive review for the certification and recertification examinations for physician assistants. 5th ed. Baltimore, MD: Wolters Kluwer Health; 2015:161-162.

    Incorrect

    Correct Answer: (C): An ectopic pregnancy is one in which the fertilized egg implants outside of the endometrial cavity, most commonly the ampulla of the fallopian tube.1,2 The classic triad of ectopic pregnancy is abdominal pain, amenorrhea, and vaginal bleeding.1 The B-hCG is utilized to differentiate between normal and abnormal pregnancies.1,2 This patient’s B-hCG was below the discriminatory zone of 2000 mIU/mL, which is the level in which a gestational sac can be recognized on transvaginal ultrasound imaging studies.1 In a normal pregnancy the B-hCG should double every 48-72 hours for the first 40 days of pregnancy.1,2 Since this was not the case, an ectopic pregnancy is the most likely diagnosis.  This patient should undergo a Dilation and Curettage or Methotrexate therapy as to avoid rupture of the ectopic pregnancy.1,2 If this were to be a (D) spontaneous abortion, the B-hCG should have decreased in 48 hours, and would have continued to decrease in each serum test thereafter. (B) Ovarian cyst is in the differential diagnoses because a mass measuring 2cm in early pregnancy with lower abdominal pain lateralized to one side has potential to be a corpus luteal cyst, which is formed at the site of ovulation. An ovarian cyst would likely present with adnexal tenderness rather than fullness.3 The corpus luteum cyst will start to regress around 10 weeks in pregnancy.3 (A) Pelvic Inflammatory Disease (PID) often presents with cervical motion tenderness, chronic lower abdominal pain and abnormal uterine bleeding or dysmenorrhea.4 Often times, the patient has a fever and an increased WBC count.5 The most common organisms responsible for PID are Neisseria gonorrhoeae and Chlamydia trachomitis.4,5 The patient presenting with PID would not have a positive B-hCG.

    References

    Sepilian VP. Ectopic Pregnancy. Medscape. http://emedicine.medscape.com/article/256448-clinical#b3. Updated November 15, 2015. Accessed June 28, 2016.

    Tulandi T. Clinical manifestations, diagnosis, and management of ectopic pregnancy. UpToDate. http://www.uptodate.com. 2016. Accessed June 28, 2016.

    Sharp HT. Evaluation and management of ruptured ovarian cyst. UpToDate. http://www.uptodate.com. 2016. Updated November 25, 2015. Accessed June 28, 2016.

    Shepherd SM. Pelvic Inflammatory Disease. Medscape. http://emedicine.medscape.com/article/256448-clinical#b3. Updated September 28, 2015. Accessed June 28, 2016.

    O’Connell CB. Gynecology. In: A comprehensive review for the certification and recertification examinations for physician assistants. 5th ed. Baltimore, MD: Wolters Kluwer Health; 2015:161-162.

  72. Question 72 of 220
    72. Question

    A 59 year-old female comes to primary care office complaining of cold symptoms for the past three weeks. She has productive cough, purulent nasal drainage, post nasal drip, and maxillary sinus pressure. On exam, patient has temperature of 100.4F, has tenderness to palpation and percussion on the right maxillary sinus, erythematous nasal mucosa. Her heart and lungs clear on auscultation. The patient has been taking Dextromethorphan PRN for cough with temporary relief. She denies SOB, dysphagia, sore throat, rash, abdominal tenderness. Which of the following is the most likely diagnosis?

    Correct

    The correct answer is D, acute bacterial sinusitis. Chronic sinusitis is a distracter because chronic sinusitis is defined as symptoms lasting for 12 weeks or more. Here, the patient has symptoms for only three weeks. Initial management of acute sinusitis is to observe for 7 days and treat with antibiotics only if there is no clinical improvement and the patient is febrile. Option B, Acute viral sinusitis should have been resolved within 7-10 days if it was viral in etiology. In this case, patient symptoms are going on from past 3 weeks. Option C, allergic rhinitis would cause symptoms including rhinorrhea, nasal congestion, but would not consist of purulent nasal discharge, sinus pressure or tenderness, low grade fever, or productive cough.

    In the United States, 1 out of every 7 adults have an episode of acute rhinosinusitis (ARS) each year and approximately 30 million individuals are diagnosed each year. It is more common in women than men. Highest incidence among adults is seen between ages 45y.o to 64yo. Risk factors include older age, smoking, allergies and asthma, dental disease, and immunodeficiency. Acute bacterial rhinosinusitis (ABRS) comprise of 0.5 – 2.0% of ARS episodes. The pathophysiology of acute rhinosinusitis is associated with three factors including obstruction of the ostia or sinus drainage pathways, ciliary impairment, and altered mucus quantity and quality. Allergic, non-allergic or viral insults can predispose the sinuses to obstruction and reduced ciliary function leading to inflammation and edema of the sinus and nasal mucosa. The inflammation and edema results in decreased ciliary clearance of sinus secretions and/ or obstruction of the ostium leading to retention of secretions, negative sinus pressure, and decrease in partial pressure of oxygen, thereby, allowing infection by bacteria.

    Acute bacterial rhinosinusitis (ABRS) is most commonly associated with viral URI. Additionally, it can result as a complication of trauma, granulomatous diseases, dental infection, mechanical obstruction, allergy or non-allergic rhinitis, immunodeficiency, ciliary dysfunction, cystic fibrosis, and other factors that disrupt sinus drainage. Approximately 75% cases of ABRS comprise of Streptococcus pneumoniae, and Haemophilus influenza as the pathogens. Other pathogens include Moraxella catarrhalis, Pseudomonas aeruginosa, Staphylococcus aureus, Escherichia coli, Klebsiella pneumoniae. Acute sinusitis pathogens can also be very commonly viral and rarely fungal agents. Patient can have a history of recent URI that lasts more than 7 days or worsens after 7 days, in this case, consider acute sinusitis. ABRS has three common clinical presentations including onset with persistent, severe, or worsening symptoms (nasal drainage, cough). Symptoms of ABRS include sinus pressure, tenderness over the sinus areas, hyposmia, nasal congestion, thick and purulent nasal drainage, postnasal drip, cough, sore throat, fever, fatigue, maxillary dental pain, ear fullness. On physical exam, common findings include tenderness on palpation and percussion of the maxillary and/or frontal sinuses, purulent nasal secretions, mucosal and facial erythema, periorbital edema. Transillumination may show opacification of the maxillary or frontal sinuses.
    ABRS is a clinical diagnosis and does not require any imaging or tests unless complications, alternate diagnosis or treatment failure is suspected. One of the following is required to diagnose a patient with ABRS: (1) symptoms or signs of acute rhinosinusitis are present 10 days or more beyond the onset of upper respiratory symptoms, 1 or (2) symptoms or signs of acute rhinosinusitis worsen within 10 days after an initial improvement. X-ray or CT-scan may be used to determine etiology or for confirmation of sinusitis. Sinus aspiration can be performed to determine etiology as well. For treatment or management, Antibiotics are the mainstay for treatment of ABRS, but watchful waiting can be considered for up to 7 days if follow-up is assured. Antibiotic after 7 days should be started if patient’s condition fails to improve. If a decision is made to treat ABRS with an antibiotic agent, prescribe amoxicillin with or without clavulanic acid as first-line therapy for 5 to 10 days for most adults according to current guidelines. Doxycycline or levofloxacin or moxifloxacin is recommended as alternative first line agents for patients with penicillin allergy. Other commonly used antibiotics for second line treatment include third-generation cephalosporins (eg, cefuroxime, cefpodoxime, cefdinir), macrolides (ie, clarithromycin), fluoroquinolones (eg, ciprofloxacin, levofloxacin, moxifloxacin), and clindamycin.3Additionally, recommendations include drinking plenty of fluids, rest, analgesics, topical intranasal steroids, and/or nasal saline irrigation for symptomatic relief of ABRS.

    References

    Rosenfeld RM, Piccirillo JF, Chandrasekhar SS, et al. Clinical practice guideline (update): Adult sinusitis. Official Journal of American Academy of Otolaryngology—Head and Neck Surgery Foundation. 2015; 152(2S): S1–S39. doi: 10.1177/0194599815572097

    JAMA. 2009; 301(17):1844. doi:10.1001/jama.301.17.1844.

    Brook I. Acute sinusitis. Medscape website. http://emedicine.medscape.com/article/232670-overview. Updated July 29, 2015. Accessed February 4, 2016.

    Hwang PH, Patel ZM. Acute sinusitis and rhinosinusitis in adults: Clinical manifestation and diagnosis. UpToDate website. http://www.uptodate.com/contents/acute-sinusitis-and-rhinosinusitis-in-adults-clinical-manifestations-and-diagnosis?source=search_result&search=acute+sinustis&selectedTitle=2~150. Updated January 22, 2016. Accessed February 2, 2016.

    Incorrect

    The correct answer is D, acute bacterial sinusitis. Chronic sinusitis is a distracter because chronic sinusitis is defined as symptoms lasting for 12 weeks or more. Here, the patient has symptoms for only three weeks. Initial management of acute sinusitis is to observe for 7 days and treat with antibiotics only if there is no clinical improvement and the patient is febrile. Option B, Acute viral sinusitis should have been resolved within 7-10 days if it was viral in etiology. In this case, patient symptoms are going on from past 3 weeks. Option C, allergic rhinitis would cause symptoms including rhinorrhea, nasal congestion, but would not consist of purulent nasal discharge, sinus pressure or tenderness, low grade fever, or productive cough.

    In the United States, 1 out of every 7 adults have an episode of acute rhinosinusitis (ARS) each year and approximately 30 million individuals are diagnosed each year. It is more common in women than men. Highest incidence among adults is seen between ages 45y.o to 64yo. Risk factors include older age, smoking, allergies and asthma, dental disease, and immunodeficiency. Acute bacterial rhinosinusitis (ABRS) comprise of 0.5 – 2.0% of ARS episodes. The pathophysiology of acute rhinosinusitis is associated with three factors including obstruction of the ostia or sinus drainage pathways, ciliary impairment, and altered mucus quantity and quality. Allergic, non-allergic or viral insults can predispose the sinuses to obstruction and reduced ciliary function leading to inflammation and edema of the sinus and nasal mucosa. The inflammation and edema results in decreased ciliary clearance of sinus secretions and/ or obstruction of the ostium leading to retention of secretions, negative sinus pressure, and decrease in partial pressure of oxygen, thereby, allowing infection by bacteria.

    Acute bacterial rhinosinusitis (ABRS) is most commonly associated with viral URI. Additionally, it can result as a complication of trauma, granulomatous diseases, dental infection, mechanical obstruction, allergy or non-allergic rhinitis, immunodeficiency, ciliary dysfunction, cystic fibrosis, and other factors that disrupt sinus drainage. Approximately 75% cases of ABRS comprise of Streptococcus pneumoniae, and Haemophilus influenza as the pathogens. Other pathogens include Moraxella catarrhalis, Pseudomonas aeruginosa, Staphylococcus aureus, Escherichia coli, Klebsiella pneumoniae. Acute sinusitis pathogens can also be very commonly viral and rarely fungal agents. Patient can have a history of recent URI that lasts more than 7 days or worsens after 7 days, in this case, consider acute sinusitis. ABRS has three common clinical presentations including onset with persistent, severe, or worsening symptoms (nasal drainage, cough). Symptoms of ABRS include sinus pressure, tenderness over the sinus areas, hyposmia, nasal congestion, thick and purulent nasal drainage, postnasal drip, cough, sore throat, fever, fatigue, maxillary dental pain, ear fullness. On physical exam, common findings include tenderness on palpation and percussion of the maxillary and/or frontal sinuses, purulent nasal secretions, mucosal and facial erythema, periorbital edema. Transillumination may show opacification of the maxillary or frontal sinuses.
    ABRS is a clinical diagnosis and does not require any imaging or tests unless complications, alternate diagnosis or treatment failure is suspected. One of the following is required to diagnose a patient with ABRS: (1) symptoms or signs of acute rhinosinusitis are present 10 days or more beyond the onset of upper respiratory symptoms, 1 or (2) symptoms or signs of acute rhinosinusitis worsen within 10 days after an initial improvement. X-ray or CT-scan may be used to determine etiology or for confirmation of sinusitis. Sinus aspiration can be performed to determine etiology as well. For treatment or management, Antibiotics are the mainstay for treatment of ABRS, but watchful waiting can be considered for up to 7 days if follow-up is assured. Antibiotic after 7 days should be started if patient’s condition fails to improve. If a decision is made to treat ABRS with an antibiotic agent, prescribe amoxicillin with or without clavulanic acid as first-line therapy for 5 to 10 days for most adults according to current guidelines. Doxycycline or levofloxacin or moxifloxacin is recommended as alternative first line agents for patients with penicillin allergy. Other commonly used antibiotics for second line treatment include third-generation cephalosporins (eg, cefuroxime, cefpodoxime, cefdinir), macrolides (ie, clarithromycin), fluoroquinolones (eg, ciprofloxacin, levofloxacin, moxifloxacin), and clindamycin.3Additionally, recommendations include drinking plenty of fluids, rest, analgesics, topical intranasal steroids, and/or nasal saline irrigation for symptomatic relief of ABRS.

    References

    Rosenfeld RM, Piccirillo JF, Chandrasekhar SS, et al. Clinical practice guideline (update): Adult sinusitis. Official Journal of American Academy of Otolaryngology—Head and Neck Surgery Foundation. 2015; 152(2S): S1–S39. doi: 10.1177/0194599815572097

    JAMA. 2009; 301(17):1844. doi:10.1001/jama.301.17.1844.

    Brook I. Acute sinusitis. Medscape website. http://emedicine.medscape.com/article/232670-overview. Updated July 29, 2015. Accessed February 4, 2016.

    Hwang PH, Patel ZM. Acute sinusitis and rhinosinusitis in adults: Clinical manifestation and diagnosis. UpToDate website. http://www.uptodate.com/contents/acute-sinusitis-and-rhinosinusitis-in-adults-clinical-manifestations-and-diagnosis?source=search_result&search=acute+sinustis&selectedTitle=2~150. Updated January 22, 2016. Accessed February 2, 2016.

  73. Question 73 of 220
    73. Question

    A 50-year-old female presents to your clinic with numbness and tingling in both hands for the past month. She states that she struggles typing at work where she is an administrative assistant. She has to take breaks to relax her hands because she feels “electric shocks” running through her first three fingers. She also has pain during the night that regularly wakes her from sleep. She denies any recent injury to her upper extremity or swelling of her hands. Based on the patient’s symptoms, which of the following tests would help confirm the most likely diagnosis?

    Correct

    The correct answer is B. Based on the history, this patient has classic symptoms of carpal tunnel syndrome. The Phalen’s maneuver is the only exam listed that is considered a diagnostic test for carpal tunnel syndrome. During this maneuver, the patient is asked to maximally flex their wrists together for one minute. If the patient feels tingling or numbness along the median nerve pathway (the first three fingers and the radial side of the forth finger on the palm side) the test is considered a pertinent positive for the diagnosis of CTS. The test is designed to reproduce the patient’s symptoms during an office visit.

    Answer A is the diagnostic test for DeQuervain’s tenosynovitis. This test would be performed if the patient was having pain more focused around the thumb and along the radial aspect of the wrist. This condition is not associated with any nerve entrapment, which is the contrasting feature from CTS. Finkelstein’s test involved the patient placing their thumb inside their hand to make a fist and ulnar deviating the wrist. This would elicit pain over the anatomical snuff box. Answer C is used to determine arterial circulation of the hand. The radial and ulnar arteries are held simultaneously and after the hand is blanched, one of the arteries is released to determine the patency of the vessels. This answer is incorrect in the case of this patient since the test would not reproduce her symptoms and is not related to the median nerve. Answer D is most commonly used to diagnose thoracic outlet syndrome. This answer is incorrect as the patient only has focused pain in the hands, not in the neck or proximal upper extremity.

    References

    1. Filius A, Thoreson AR, Yang T, Vanhees M, An K, Zhao C and Amadio PC. The Effect of Low- and High-Velocity Tendon Excursion on the Mechanical Properties of Human Cadaver Subsynovial Connective Tissue. 32: 1: 123-128, 2014.
    2. Bezuhly Michael, O’Brien J, Lalonde D. Nerve Entrapment Syndromes. In: Neligan P, 3rd edition. Plastic Surgery. Seattle, WA: Elsevier Inc; 2013
    3. Carpal Tunnel Syndrome. University of Maryland Medical Center. Published 2016. Accessed April 23, 2016. http://umm.edu/health/medical/reports/articles/carpal-tunnel-syndrome
    4. Gejyo F and Narita I. Current clinical and Pathogenetic Understanding of Beta2-m Amyloidosis in Long-Term Haemodialysis Patients. Nephrology (Carlton) 8 Suppl: S45-S49; 2003.
    5. Tosti R and Ilyas AM. Acute Carpal Tunnel Syndrome. Orthop.Clin.North Am. 43: 4: 459-465; 2012.
    Incorrect

    The correct answer is B. Based on the history, this patient has classic symptoms of carpal tunnel syndrome. The Phalen’s maneuver is the only exam listed that is considered a diagnostic test for carpal tunnel syndrome. During this maneuver, the patient is asked to maximally flex their wrists together for one minute. If the patient feels tingling or numbness along the median nerve pathway (the first three fingers and the radial side of the forth finger on the palm side) the test is considered a pertinent positive for the diagnosis of CTS. The test is designed to reproduce the patient’s symptoms during an office visit.

    Answer A is the diagnostic test for DeQuervain’s tenosynovitis. This test would be performed if the patient was having pain more focused around the thumb and along the radial aspect of the wrist. This condition is not associated with any nerve entrapment, which is the contrasting feature from CTS. Finkelstein’s test involved the patient placing their thumb inside their hand to make a fist and ulnar deviating the wrist. This would elicit pain over the anatomical snuff box. Answer C is used to determine arterial circulation of the hand. The radial and ulnar arteries are held simultaneously and after the hand is blanched, one of the arteries is released to determine the patency of the vessels. This answer is incorrect in the case of this patient since the test would not reproduce her symptoms and is not related to the median nerve. Answer D is most commonly used to diagnose thoracic outlet syndrome. This answer is incorrect as the patient only has focused pain in the hands, not in the neck or proximal upper extremity.

    References

    1. Filius A, Thoreson AR, Yang T, Vanhees M, An K, Zhao C and Amadio PC. The Effect of Low- and High-Velocity Tendon Excursion on the Mechanical Properties of Human Cadaver Subsynovial Connective Tissue. 32: 1: 123-128, 2014.
    2. Bezuhly Michael, O’Brien J, Lalonde D. Nerve Entrapment Syndromes. In: Neligan P, 3rd edition. Plastic Surgery. Seattle, WA: Elsevier Inc; 2013
    3. Carpal Tunnel Syndrome. University of Maryland Medical Center. Published 2016. Accessed April 23, 2016. http://umm.edu/health/medical/reports/articles/carpal-tunnel-syndrome
    4. Gejyo F and Narita I. Current clinical and Pathogenetic Understanding of Beta2-m Amyloidosis in Long-Term Haemodialysis Patients. Nephrology (Carlton) 8 Suppl: S45-S49; 2003.
    5. Tosti R and Ilyas AM. Acute Carpal Tunnel Syndrome. Orthop.Clin.North Am. 43: 4: 459-465; 2012.
  74. Question 74 of 220
    74. Question

    A 22 year-old woman presents to the health clinic with a two-day history of urinary frequency, urgency and dysuria. She denies dyspareunia, fever, abdominal pain, back pain, nausea, and vomiting. She denies vaginal discharge and itching or burning. She has no history of STDs, UTI, BV, candidiasis, or nephrolithiasis. On physical exam, her blood pressure is 174/112 mm Hg, respirations 20/min, pulse 90/min, and temperature 98.4°F. She is unsure if she could be pregnant or not. There is no CVA tenderness. Urine dipstick reveals positive leukocyte esterase and nitrites. What is the recommended pharmacologic agent used to treat this presentation?

    Correct

    Answer C.  About 80-90% of UTIs are caused by E. coli. It has been suggested that E. coli originate in the gastrointestinal flora and take a fecal-vaginal-urethral route to infection in women. The infection ascends from the urethra into the bladder. Invasion appears to be related to the adhesion process where the bacteria adhere and penetrate into bladder epithelial cells. The bacteria replicate and form a stable reservoir for bladder colonization. The other 5-10% of UTI are caused by Staph. Saprophyticus, particularly in sexually active young women. Rarely, acute cystitis can be caused by a virus or fungus. There is a wider range of causative pathogens in nosocomial infections, including E. coli (27%), Klebisella (11%), Pseudomonas (11%) and Candida albicans (9%). These are typically caused by urinary catherization where the bacteria invade through a contaminated catheter.

    Uncomplicated cystitis in women can be treated with short-term antimicrobial therapy. The suggested regime is a fluoroquinolone such as Ciprofloxacin or nitrofurantoin (Macrobid) for 3 to 5 days4. Bactrim DS can be used as an alternative to a quinolone in resistant E.Coli, however resistance to Bactrim is also becoming increasingly emergent. Since UTI is rare in men, the duration of antibiotic therapy depends on the underlying etiology. Complicated cases are treated with IV or PO fluoroquinolones or aminoglycosides.  Pregnant patient’s best respond to 7-14 days of amoxicillin or Macrobid. Bactrim and Cipro should not be taken during pregnancy.

    Increasing fluids, avoiding intercourse and hot sitz baths are helpful. There is not enough evidence to conclude that drinking cranberry juice is beneficial, although a theory exists that pure cranberry juice acidifies the urine and prevents bacteria from adhering to the bladder wall. Phenazopyridine is a urinary analgesic that cannot be used for more than 48 hours because of the side effects of methemoglobinemia and hemolytic anemia, but it is useful for symptom relief. The patient should be warned that the urine will turn dark orange. Infections typically respond quickly to therapy and failure to respond suggests further investigation.

    Future infections can be prevented with proper hygiene, such as avoiding washing with soap or using baby wipes and urinating after intercourse. Women who have more than three episodes of UTI a year are candidates for prophylatic antibiotic therapy. Hospitals should strive to only use indwelling catheters when necessary, implement prompt removal of catheters when they are no longer necessary and maintain proper insertion techniques.

    References

    1. Meng MV, Walsh TJ, Chi TD. Urologic Disorders. In: Papadakis MA, McPhee SJ, CURRENT Medical Diagnosis & Treatment. 54th ed. United States of America: McGraw-Hill Education; 2015: 931-933.
    2. Foxman, B. Epidemiology of Urinary Tract Infections: Incidence, Morbidity, and Economic Costs. Am J Med. 2002; 113(1A): 5S-13S. doi: 10.1067/mda.2003.7
    3. Salvatore S, Salvatore S, Cattoni E, et al. Urinary tract infections in women. European Journal of Obstetrics & Gynecology and Reproductive Biology. 2011;156(2):131-136. doi: http://dx.doi.org.ezproxymcp.flo.org/10.1016/j.ejogrb.2011.01.028.
    4. O’Connell CB. Urinary Tract Infection. In: Certification and Recertification Examinations for Physician Assistants. 5th ed. Baltimore, Maryland: Wolters Kluwer Health; 2015: 136.

    5 Williams, DA. Genitourinary System. In: PANCE Prep Pearls. 1st edition. United States of America: Dwayne A. Williams; 2014: 335.

    1. Cranberry xyloglucan structure and inhibition of escherichia coli adhesion to epithelial cells. – J Agric Food Chem. (- 23):- 5622. doi: – 10.1021/acs.jafc.5b00730.
    Incorrect

    Answer C.  About 80-90% of UTIs are caused by E. coli. It has been suggested that E. coli originate in the gastrointestinal flora and take a fecal-vaginal-urethral route to infection in women. The infection ascends from the urethra into the bladder. Invasion appears to be related to the adhesion process where the bacteria adhere and penetrate into bladder epithelial cells. The bacteria replicate and form a stable reservoir for bladder colonization. The other 5-10% of UTI are caused by Staph. Saprophyticus, particularly in sexually active young women. Rarely, acute cystitis can be caused by a virus or fungus. There is a wider range of causative pathogens in nosocomial infections, including E. coli (27%), Klebisella (11%), Pseudomonas (11%) and Candida albicans (9%). These are typically caused by urinary catherization where the bacteria invade through a contaminated catheter.

    Uncomplicated cystitis in women can be treated with short-term antimicrobial therapy. The suggested regime is a fluoroquinolone such as Ciprofloxacin or nitrofurantoin (Macrobid) for 3 to 5 days4. Bactrim DS can be used as an alternative to a quinolone in resistant E.Coli, however resistance to Bactrim is also becoming increasingly emergent. Since UTI is rare in men, the duration of antibiotic therapy depends on the underlying etiology. Complicated cases are treated with IV or PO fluoroquinolones or aminoglycosides.  Pregnant patient’s best respond to 7-14 days of amoxicillin or Macrobid. Bactrim and Cipro should not be taken during pregnancy.

    Increasing fluids, avoiding intercourse and hot sitz baths are helpful. There is not enough evidence to conclude that drinking cranberry juice is beneficial, although a theory exists that pure cranberry juice acidifies the urine and prevents bacteria from adhering to the bladder wall. Phenazopyridine is a urinary analgesic that cannot be used for more than 48 hours because of the side effects of methemoglobinemia and hemolytic anemia, but it is useful for symptom relief. The patient should be warned that the urine will turn dark orange. Infections typically respond quickly to therapy and failure to respond suggests further investigation.

    Future infections can be prevented with proper hygiene, such as avoiding washing with soap or using baby wipes and urinating after intercourse. Women who have more than three episodes of UTI a year are candidates for prophylatic antibiotic therapy. Hospitals should strive to only use indwelling catheters when necessary, implement prompt removal of catheters when they are no longer necessary and maintain proper insertion techniques.

    References

    1. Meng MV, Walsh TJ, Chi TD. Urologic Disorders. In: Papadakis MA, McPhee SJ, CURRENT Medical Diagnosis & Treatment. 54th ed. United States of America: McGraw-Hill Education; 2015: 931-933.
    2. Foxman, B. Epidemiology of Urinary Tract Infections: Incidence, Morbidity, and Economic Costs. Am J Med. 2002; 113(1A): 5S-13S. doi: 10.1067/mda.2003.7
    3. Salvatore S, Salvatore S, Cattoni E, et al. Urinary tract infections in women. European Journal of Obstetrics & Gynecology and Reproductive Biology. 2011;156(2):131-136. doi: http://dx.doi.org.ezproxymcp.flo.org/10.1016/j.ejogrb.2011.01.028.
    4. O’Connell CB. Urinary Tract Infection. In: Certification and Recertification Examinations for Physician Assistants. 5th ed. Baltimore, Maryland: Wolters Kluwer Health; 2015: 136.

    5 Williams, DA. Genitourinary System. In: PANCE Prep Pearls. 1st edition. United States of America: Dwayne A. Williams; 2014: 335.

    1. Cranberry xyloglucan structure and inhibition of escherichia coli adhesion to epithelial cells. – J Agric Food Chem. (- 23):- 5622. doi: – 10.1021/acs.jafc.5b00730.
  75. Question 75 of 220
    75. Question

    A 53 year old male presents complaining of visual disturbances for the past few weeks. He complains of frequent nighttime urination and extreme thirst throughout his day. He states he has not changed anything in his diet or nighttime routine. He also claims he has not started any recent medications. He has a positive family history for type 2 diabetes. Which of the following tests are NOT permitted to make a diagnosis of type 2 diabetes?

    Correct

    The answer is B. Self-monitoring glucose finger sticks are not appropriate to make a diagnosis of type 2 diabetes.  Glucometers are sometimes inaccurate in glucose readings and many times readings may fluctuate between fingers and hands. Finger stick readings are the main way for patients to log their blood sugars at home and give a clinician a good idea if a patient’s current diet and pharmacotherapy regimen is adequate.  However, in the clinical setting for an official diagnosis a finger stick is not sufficient.  Lab work is mandatory for a diagnosis.

    A: Fasting plasma glucose (FPG) is the most common and least expensive test used to assess impaired glucose metabolism. A fasting plasma glucose ≥ 126 mg/dL is considered the threshold for diabetes.  The range between 100-125 mg/dL is in the pre diabetic zone, while 70-99 mg/dL is considered normal blood glucose levels. This test needs to have the patient fasting for a minimum of eight hours prior to venipuncture to receive an accurate result. If this test shows ≥ 126 mg/dL on two separate occasions, it is diagnostic of type 2 diabetes mellitus. Another option using the FPG is a random plasma glucose. If the results are ≥ 200 mg/dL and the patient is experiencing symptoms (i.e. polydipsia, polyuria, fatigue, etc.) this also acceptable diagnostic criteria.

    C:  The OGTT, although more sensitive than the FPG test, it is less convenient to administer. In non-pregnant adults, 75 grams of a glucose beverage is consumed in over 5 minutes.  Blood is drawn 2 hours later to determine the insulin response. If the 2 hour plasma glucose reports ≥ 200mg/dL, it is diagnostic of diabetes mellitus. If the result is between 140-199 mg/dL, it is indicative of impaired glucose tolerance (IGT) or prediabetes.  Before this glucose challenge, a patient needs to be fasting a minimum of 8 hours.

    D:  Hemoglobin A1C ≥6.5% is indicative of diabetes mellitus and chronic hyperglycemia. This test measures the glycated hemoglobin levels for the past 90 days in a patient and gives the clinician an average blood sugar range for that period of time. The “pre-diabetes” range of A1C is 5.7-6.4%. A normal A1C level should be below 5.6%. Once a patient receives a diagnosis of diabetes, an A1C should be checked twice yearly at minimum. If a patient requires closer monitoring, than more frequent testing is acceptable.

    References

    Alter DN, Laposata M. Pancreatic Disorders. In: Laposata M. eds. Laboratory Medicine: The Diagnosis of Disease in the Clinical Laboratory. New York, NY: McGraw-Hill; 2014.

    Molina PE. Chapter 7. Endocrine Pancreas. In: Molina PE. eds. Endocrine Physiology, 4e. New York, NY: McGraw-Hill; 2013.

    Pagana KD, Pagana TJ. Mosby’s Manual of Diagnostic and Laboratory Tests. St. Louis, MO: Mosby/Elsevier; 2010.

    Incorrect

    The answer is B. Self-monitoring glucose finger sticks are not appropriate to make a diagnosis of type 2 diabetes.  Glucometers are sometimes inaccurate in glucose readings and many times readings may fluctuate between fingers and hands. Finger stick readings are the main way for patients to log their blood sugars at home and give a clinician a good idea if a patient’s current diet and pharmacotherapy regimen is adequate.  However, in the clinical setting for an official diagnosis a finger stick is not sufficient.  Lab work is mandatory for a diagnosis.

    A: Fasting plasma glucose (FPG) is the most common and least expensive test used to assess impaired glucose metabolism. A fasting plasma glucose ≥ 126 mg/dL is considered the threshold for diabetes.  The range between 100-125 mg/dL is in the pre diabetic zone, while 70-99 mg/dL is considered normal blood glucose levels. This test needs to have the patient fasting for a minimum of eight hours prior to venipuncture to receive an accurate result. If this test shows ≥ 126 mg/dL on two separate occasions, it is diagnostic of type 2 diabetes mellitus. Another option using the FPG is a random plasma glucose. If the results are ≥ 200 mg/dL and the patient is experiencing symptoms (i.e. polydipsia, polyuria, fatigue, etc.) this also acceptable diagnostic criteria.

    C:  The OGTT, although more sensitive than the FPG test, it is less convenient to administer. In non-pregnant adults, 75 grams of a glucose beverage is consumed in over 5 minutes.  Blood is drawn 2 hours later to determine the insulin response. If the 2 hour plasma glucose reports ≥ 200mg/dL, it is diagnostic of diabetes mellitus. If the result is between 140-199 mg/dL, it is indicative of impaired glucose tolerance (IGT) or prediabetes.  Before this glucose challenge, a patient needs to be fasting a minimum of 8 hours.

    D:  Hemoglobin A1C ≥6.5% is indicative of diabetes mellitus and chronic hyperglycemia. This test measures the glycated hemoglobin levels for the past 90 days in a patient and gives the clinician an average blood sugar range for that period of time. The “pre-diabetes” range of A1C is 5.7-6.4%. A normal A1C level should be below 5.6%. Once a patient receives a diagnosis of diabetes, an A1C should be checked twice yearly at minimum. If a patient requires closer monitoring, than more frequent testing is acceptable.

    References

    Alter DN, Laposata M. Pancreatic Disorders. In: Laposata M. eds. Laboratory Medicine: The Diagnosis of Disease in the Clinical Laboratory. New York, NY: McGraw-Hill; 2014.

    Molina PE. Chapter 7. Endocrine Pancreas. In: Molina PE. eds. Endocrine Physiology, 4e. New York, NY: McGraw-Hill; 2013.

    Pagana KD, Pagana TJ. Mosby’s Manual of Diagnostic and Laboratory Tests. St. Louis, MO: Mosby/Elsevier; 2010.

  76. Question 76 of 220
    76. Question

    A 73-year-old Caucasian female presents to the office with a 6 week history of for severe, generalized aching pain, and long lasting morning stiffness in her shoulders, arms, hips and thighs. She also has fatigue and a decreased ability to carry out some of her everyday activities. Today, the patient’s lab work revealed an elevated ESR level, an elevated CRP, negative Lyme titers, a normal creatine kinase level, and a negative rheumatoid factor. Her CBC reveals a normocytic anemia and an elevated platelet level. She denies any rashes, numbness, tingling, or noticeable joint swelling. Based on the patient’s initial presentation and lab result, which choice is the most likely diagnosis?

    Correct

    Answer is B. Polymyalgia Rheumatica (PMR) is an inflammatory disease that causes muscle pain and stiffness particularly in areas of the shoulder girdle, upper arms, neck, lower back and thighs (1). A patient’s symptoms develop very quickly and tend to be worse in the mornings (1). This condition typically affects individuals greater than 65 years old and rarely occurs in those under 50 years old (1). While this is an inflammatory condition, PMR does not cause swelling of the joints, which is one aspect that has made it increasingly hard to recognize (1). However, one way providers have been able to distinguish PMR from other similarly presenting conditions is by the quick response time patients with PMR have to a low dose course of daily corticosteroids (1).

    There are three main risk factors associated with occurrence of polymyalgia rheumatic: age, sex, and race/region (2). Most often, PMR affects older individuals above the age of 50 and is not typically seen in those any younger. The average age of PMR onset is 73 years old (2). Polymyalgia rheumatica is two times more likely to occur in females than males, even more so in Caucasian females in northern European populations (2). Due to the association found between PMR and giant cell arteritis, Caucasian women over the age of 50 are at the highest risk for developing both of these conditions (3). While women are at an increased risk for the two disorders, research has shown that men with giant cell arteritis are more likely to experience potentially worse eye involvement, often progressing to blindness (3). Both PMR and giant cell arteritis are quite prevalent conditions (4). It was determined that there are an estimated 711,000 American individuals with PMR and an estimated 228,000 with giant cell arteritis in the United States (4).

    In contrast with many other rheumatic disease states, the target of inflammation in PMR is still unknown (5). It is believed that the synovial membrane is the most likely target, as synovitis around the joints of the shoulders, hips, knees, wrists, and metacarpalphalanges have been revealed via sonographic and immunohistochemical methods (5,7). Additionally, there is an increased frequency of peripheral arthritis in patients with PMR, which further supports this theory (5). Both polymyalgia rheumatica and giant cell arteritis have a similar course of pathophysiology, with underlying “abnormalities of cellular immunity leading to vessel and systemic inflammation” (6). One major difference being, however, that in patients with PMR, it has been studied that inflammation is mainly confined to the articular and perarticular tissues (6). In those with giant cell arteritis, there is evidence of severe inflammation confined to the vessels due to the presence of large-vessel vasculitis (6). Regardless of location, cytokines, specifically IL-6, and tumor necrosis factor-alpha play a major role in the progression of inflammation in those with either condition (6). Individuals with PMR or giant cell arteritis typically have an elevated level of IL-6, which is the most likely cause of the systemic inflammatory response that occurs (7). The majority of studies done on patients with PMR have revealed that a decrease in circulating IL-6 levels is directly associated with resolution of clinical manifestations in patients (7). There is also thought to be an autoimmune correlation to PMR. There may be an “imbalance between the immunosuppressive T-regulatory lymphocytes (T-reg) and proinflammatory T-helper 17 (Th-17) cells in both PMR and giant cell arteritis (8). There has shown to be a decreased incidence of circulating T-reg cells compared to an increased number of Th-17 cells in patients with PMR (8). Unfortunately, not enough data has been gathered to support any conclusions regarding the association of other circulating cytokines (interleukins, tumor necrosis factor-alpha) to PMR (7). However, recent studies have revealed that interferon-gamma (IFN-gamma) is present in about 70% of temporal artery biopsy samples taken from individuals with giant cell arteritis (7). This interferon-gamma has not been found in individuals with pure PMR, which can lead researchers to believe that IFN-gamma may be pertinent to the development of the vasculitis so characteristically associated with giant cell arteritis (7). Findings such as these are crucial for providers to know and understand as they can assist greatly in the differentiation between two similar conditions.

    PMR is a condition of generalized aching pain and stiffness throughout an individual’s body (1,2). Typically, the first symptom that a patient experiences is an ache or pain in their shoulders (2). Aches or pains in the neck, upper arms, wrist, elbows, buttocks, hips, thighs or knees can then follow (2). In addition to the pain, a severe stiffness will present in the affected areas (2). Majority of times the signs and symptoms of PMR occur bilaterally (1), The symptoms of PMR have been found to develop as quickly as overnight, or more commonly, over the course of a couple of days to weeks (1). Most often, this stiffness will be most noticeable in the morning, lasting a minimum of 30 minutes, and/or after long periods of inactivity (1,2). Patients also experience a decreased range of motion in the affected areas (2). Aside from the musculoskeletal symptoms, there may also be some general, flu-like symptoms, which include fever, fatigue, malaise, loss of appetite, and unexpected weight loss (2). PMR can have significant effects on an individual’s ADLs (2). The characteristic pain and stiffness associated with PMR can result in a struggle to carry everyday activities such as bathing, trouble lifting one’s arms over their head, brushing or doing one’s hair, getting out of bed, getting dressed, standing up from a chair, getting in or out of a car, and even trouble sleeping (2).  Polymyalgia rhuematica is clinically diagnosed on the basis of 3 main factors – patient’s medical history, symptoms and physical exam (6). Majority of the time, physical exam findings will include pain with movement that inhibits active range of motion in the shoulder and hips (6). The patient’s passive range of motion, in addition to their muscle strength, should be normal (6). It is important to note that fundamentally muscle weakness is not a symptom of PMR (7). However, true muscle weakness can be difficult to evaluate in a patient with PMR due to the significant pain and discomfort they experience (7).

    Unfortunately, there is no single gold standard test or method of imaging that can conclusively diagnose PMR (3). However, there are a variety of lab tests now utilized by health care professionals to aid in verifying a diagnosis or ruling out a different diagnosis or cause of the patient’s symptoms (3). In patients with PMR, one of the most common lab findings is an elevated erythrocyte sedimentation rate (ESR) or sed rate (3). This is a measurement test of inflammation based on how long it takes for red blood cells to fall to the bottom of a test tube filled with unclotted blood (3). Red blood cells that fall quickly signify the presence of inflammation in the body. While this lab test is sensitive for PMR, the ESR is not specific (3). An elevated ESR level strictly indicates active inflammation in the body. The problem is that this is a finding of a number of other arthritic and rheumatic conditions (3).  Therefore, an abnormal ESR measurement cannot solely confirm a PMR diagnosis (3). For this reason, there are other specific tests used to help definitively rule out other disease presentations that can mimic that of PMR. One is the C-reactive protein (CRP), which is another measurement of inflammation (3). Like the ESR, the CRP level will be elevated in the majority of PMR patients (3).

    Polymyalgia rheumatica is a chronic, limiting condition; therefore, the primary goals of therapy for patients with PMR include managing painful myalgia, improving muscle stiffness and ultimately resolving disease progression (7). The first line treatment of choice for PMR is corticosteroid therapy, typically that of Prednisone or prednisolone (6,7). This has been the foundation of successful PMR management since 1951 (6,7). Corticosteroids have been found to not only result in total or near-total symptom resolution, but also play a role in normalizing the elevated ESR level (7). Once initiated, patients can expect to experience symptom relief within 2-3 days (7). If, however, the symptoms are not relieved or controlled within 1 week, the corticosteroid dose may be increased (7). At this point, it is imperative for a patient’s provider to consider the possibility of a giant cell arteritis diagnosis, especially if the patient is still not experiencing symptom control with 20mg of Prednisone (7). Once the patient demonstrates a sufficient response to the corticosteroids, the starting dose should be sustained for 1 months duration before gradually starting to taper downward to the lowest effective dose (6). The tapering of Prednisone in these patients plays a huge role in remission on their PMR. If started at 15mg Prednisone, it was found that a slow tapering, of less than 1mg/month, correlated with fewer occurrences of relapse (7). Once tapered down to 10mg per day, a gradual taper of Prednisone by 1 mg every 2 months until therapy cessation was found to correlate with optimal regulation of disease activity (7).

    It is important to note that it is very common for disease flare-ups to occur throughout ones corticosteroid taper and typically these require temporary increases in treatment (6). Tapering in general should be directed based on a patient’s clinical response to treatment. This includes monitoring levels of “decreased pain and stiffness, decreased morning stiffness, and decreased shoulder pain or limitation on exam” (7). In addition to patient’s responses to corticosteroid therapy, overall physical function, mental function, and normalization of patient’s inflammatory markers are other important factors to consider as measures of disease activity and treatment response (7). However, they are not factors which on their own should act as indicators for alterations to or cessation of treatment (7). Patients should expect to continue their specific corticosteroid therapy for a minimum of 1-2 years (6). In addition to possible flare-ups, the majority of patients also have a minimum of 1 relapse during their corticosteroid taper (6). “The factors associated with an increased risk of relapse include persistently high levels of IL-6 and CRP despite treatment, in addition to a rapid rate of prednisone withdrawal (6).” Relapses are most likely to happen within the first 18 months of corticosteroid treatment and then within 1 year of withdrawal (7). Therefore, it is important for all patients to be closely monitored for any signs of symptoms reappearance during their corticosteroid tapering, in addition to 12 months after therapy is discontinued (7). The next step in potential PMR treatment and management is non-steroidal anti-inflammatory drugs, or NSAIDs, such as Ibuprofen or Naproxen (6,7). If there are no contraindications for a patient’s use of NSAIDs, they may be helpful in offering additional pain relief. However, in a study of 232 patients with PMR, an association between NSAIDs and drug-related morbidity was found (7). Therefore, it is suggesting that proceeding with NSAID and corticosteroid treatment should be cautious and perhaps only considered if patients have unbearable adverse effects from corticosteroids (7). Overall, once treated polymyalgia rheumatic usually resolves as quickly as 2-3 days. With the help of a timely diagnosis and sufficient treatment, patients with PMR have a tremendous prognosis (7). Even without treatment, it will resolve on its own but may take as long as several years.

    References

    1. Docken W P. Polymyalgia Rheumatica. American College of Rheumatology. http://www.rheumatology.org/I-Am-A/Patient-Caregiver/Diseases-Conditions/Polymyalgia-Rheumatica. June 2015. Accessed March 4, 2016.

    2. Polymyalgia Rhuematica. Mayo Clinic. http://www.mayoclinic.org/diseases-conditions/polymyalgia-rheumatica/basics/symptoms/con-20023162. September 17, 2015. Accessed March 4, 2016.

    3. Polymyalgia Rheumatica and Giant Cell Arteritis. National Institute of Arthritis and Musculoskeletal and Skin Disease. http://www.niams.nih.gov/Health_Info/Polymyalgia/. April 2015. Accessed March 4, 2016.

    4. Helmick CG, Felson DT, Lawrence RC, Gabriel S, Hirsch R, Kwoh CK, et al.; National Arthritis Data Workgroup. Estimates of the Prevalence of Arthritis and Other Rheumatic Conditions in the United States, Part II. Arthritis Rheum. 2008 January ; 58(1): 26?35. doi:10.1002/art.23176

    5. Cimmino M A. Genetic and Environmental Factors In Polymyalgia Rheumatica. Annals of the Rheumatic Diseases – The Eular Journal. 1997: 56;576-577 doi: 10.1136/ard.56.10.576

    6. Clifford A, Hoffman G S. Polymyalgia Rheumatica and Giant Cell Arteritis. Cleveland Clinic – Center for Continuing Education. June 2014. Accessed March 4, 2016.

    7. Saad E R. Polymyalgia Rheumatica. Medscape. http://emedicine.medscape.com/article/330815-overview. September 23, 2015. Accessed March 4, 2016.

    8. Kermani T A, Warrington K J. Advances and Challenges In The Diagnosis and Treatment of Polymyalgia Rheumatica. Therapeutic Advances in Musculoskeletal Disease. February 2014; 6(1):8-19.

    9. Lyme Disease Tests. Lab Tests Online – AACC. https://labtestsonline.org/understanding/analytes/lyme/tab/test/.  October 16, 2015. Accessed March 4, 2016.

    Incorrect

    Answer is B. Polymyalgia Rheumatica (PMR) is an inflammatory disease that causes muscle pain and stiffness particularly in areas of the shoulder girdle, upper arms, neck, lower back and thighs (1). A patient’s symptoms develop very quickly and tend to be worse in the mornings (1). This condition typically affects individuals greater than 65 years old and rarely occurs in those under 50 years old (1). While this is an inflammatory condition, PMR does not cause swelling of the joints, which is one aspect that has made it increasingly hard to recognize (1). However, one way providers have been able to distinguish PMR from other similarly presenting conditions is by the quick response time patients with PMR have to a low dose course of daily corticosteroids (1).

    There are three main risk factors associated with occurrence of polymyalgia rheumatic: age, sex, and race/region (2). Most often, PMR affects older individuals above the age of 50 and is not typically seen in those any younger. The average age of PMR onset is 73 years old (2). Polymyalgia rheumatica is two times more likely to occur in females than males, even more so in Caucasian females in northern European populations (2). Due to the association found between PMR and giant cell arteritis, Caucasian women over the age of 50 are at the highest risk for developing both of these conditions (3). While women are at an increased risk for the two disorders, research has shown that men with giant cell arteritis are more likely to experience potentially worse eye involvement, often progressing to blindness (3). Both PMR and giant cell arteritis are quite prevalent conditions (4). It was determined that there are an estimated 711,000 American individuals with PMR and an estimated 228,000 with giant cell arteritis in the United States (4).

    In contrast with many other rheumatic disease states, the target of inflammation in PMR is still unknown (5). It is believed that the synovial membrane is the most likely target, as synovitis around the joints of the shoulders, hips, knees, wrists, and metacarpalphalanges have been revealed via sonographic and immunohistochemical methods (5,7). Additionally, there is an increased frequency of peripheral arthritis in patients with PMR, which further supports this theory (5). Both polymyalgia rheumatica and giant cell arteritis have a similar course of pathophysiology, with underlying “abnormalities of cellular immunity leading to vessel and systemic inflammation” (6). One major difference being, however, that in patients with PMR, it has been studied that inflammation is mainly confined to the articular and perarticular tissues (6). In those with giant cell arteritis, there is evidence of severe inflammation confined to the vessels due to the presence of large-vessel vasculitis (6). Regardless of location, cytokines, specifically IL-6, and tumor necrosis factor-alpha play a major role in the progression of inflammation in those with either condition (6). Individuals with PMR or giant cell arteritis typically have an elevated level of IL-6, which is the most likely cause of the systemic inflammatory response that occurs (7). The majority of studies done on patients with PMR have revealed that a decrease in circulating IL-6 levels is directly associated with resolution of clinical manifestations in patients (7). There is also thought to be an autoimmune correlation to PMR. There may be an “imbalance between the immunosuppressive T-regulatory lymphocytes (T-reg) and proinflammatory T-helper 17 (Th-17) cells in both PMR and giant cell arteritis (8). There has shown to be a decreased incidence of circulating T-reg cells compared to an increased number of Th-17 cells in patients with PMR (8). Unfortunately, not enough data has been gathered to support any conclusions regarding the association of other circulating cytokines (interleukins, tumor necrosis factor-alpha) to PMR (7). However, recent studies have revealed that interferon-gamma (IFN-gamma) is present in about 70% of temporal artery biopsy samples taken from individuals with giant cell arteritis (7). This interferon-gamma has not been found in individuals with pure PMR, which can lead researchers to believe that IFN-gamma may be pertinent to the development of the vasculitis so characteristically associated with giant cell arteritis (7). Findings such as these are crucial for providers to know and understand as they can assist greatly in the differentiation between two similar conditions.

    PMR is a condition of generalized aching pain and stiffness throughout an individual’s body (1,2). Typically, the first symptom that a patient experiences is an ache or pain in their shoulders (2). Aches or pains in the neck, upper arms, wrist, elbows, buttocks, hips, thighs or knees can then follow (2). In addition to the pain, a severe stiffness will present in the affected areas (2). Majority of times the signs and symptoms of PMR occur bilaterally (1), The symptoms of PMR have been found to develop as quickly as overnight, or more commonly, over the course of a couple of days to weeks (1). Most often, this stiffness will be most noticeable in the morning, lasting a minimum of 30 minutes, and/or after long periods of inactivity (1,2). Patients also experience a decreased range of motion in the affected areas (2). Aside from the musculoskeletal symptoms, there may also be some general, flu-like symptoms, which include fever, fatigue, malaise, loss of appetite, and unexpected weight loss (2). PMR can have significant effects on an individual’s ADLs (2). The characteristic pain and stiffness associated with PMR can result in a struggle to carry everyday activities such as bathing, trouble lifting one’s arms over their head, brushing or doing one’s hair, getting out of bed, getting dressed, standing up from a chair, getting in or out of a car, and even trouble sleeping (2).  Polymyalgia rhuematica is clinically diagnosed on the basis of 3 main factors – patient’s medical history, symptoms and physical exam (6). Majority of the time, physical exam findings will include pain with movement that inhibits active range of motion in the shoulder and hips (6). The patient’s passive range of motion, in addition to their muscle strength, should be normal (6). It is important to note that fundamentally muscle weakness is not a symptom of PMR (7). However, true muscle weakness can be difficult to evaluate in a patient with PMR due to the significant pain and discomfort they experience (7).

    Unfortunately, there is no single gold standard test or method of imaging that can conclusively diagnose PMR (3). However, there are a variety of lab tests now utilized by health care professionals to aid in verifying a diagnosis or ruling out a different diagnosis or cause of the patient’s symptoms (3). In patients with PMR, one of the most common lab findings is an elevated erythrocyte sedimentation rate (ESR) or sed rate (3). This is a measurement test of inflammation based on how long it takes for red blood cells to fall to the bottom of a test tube filled with unclotted blood (3). Red blood cells that fall quickly signify the presence of inflammation in the body. While this lab test is sensitive for PMR, the ESR is not specific (3). An elevated ESR level strictly indicates active inflammation in the body. The problem is that this is a finding of a number of other arthritic and rheumatic conditions (3).  Therefore, an abnormal ESR measurement cannot solely confirm a PMR diagnosis (3). For this reason, there are other specific tests used to help definitively rule out other disease presentations that can mimic that of PMR. One is the C-reactive protein (CRP), which is another measurement of inflammation (3). Like the ESR, the CRP level will be elevated in the majority of PMR patients (3).

    Polymyalgia rheumatica is a chronic, limiting condition; therefore, the primary goals of therapy for patients with PMR include managing painful myalgia, improving muscle stiffness and ultimately resolving disease progression (7). The first line treatment of choice for PMR is corticosteroid therapy, typically that of Prednisone or prednisolone (6,7). This has been the foundation of successful PMR management since 1951 (6,7). Corticosteroids have been found to not only result in total or near-total symptom resolution, but also play a role in normalizing the elevated ESR level (7). Once initiated, patients can expect to experience symptom relief within 2-3 days (7). If, however, the symptoms are not relieved or controlled within 1 week, the corticosteroid dose may be increased (7). At this point, it is imperative for a patient’s provider to consider the possibility of a giant cell arteritis diagnosis, especially if the patient is still not experiencing symptom control with 20mg of Prednisone (7). Once the patient demonstrates a sufficient response to the corticosteroids, the starting dose should be sustained for 1 months duration before gradually starting to taper downward to the lowest effective dose (6). The tapering of Prednisone in these patients plays a huge role in remission on their PMR. If started at 15mg Prednisone, it was found that a slow tapering, of less than 1mg/month, correlated with fewer occurrences of relapse (7). Once tapered down to 10mg per day, a gradual taper of Prednisone by 1 mg every 2 months until therapy cessation was found to correlate with optimal regulation of disease activity (7).

    It is important to note that it is very common for disease flare-ups to occur throughout ones corticosteroid taper and typically these require temporary increases in treatment (6). Tapering in general should be directed based on a patient’s clinical response to treatment. This includes monitoring levels of “decreased pain and stiffness, decreased morning stiffness, and decreased shoulder pain or limitation on exam” (7). In addition to patient’s responses to corticosteroid therapy, overall physical function, mental function, and normalization of patient’s inflammatory markers are other important factors to consider as measures of disease activity and treatment response (7). However, they are not factors which on their own should act as indicators for alterations to or cessation of treatment (7). Patients should expect to continue their specific corticosteroid therapy for a minimum of 1-2 years (6). In addition to possible flare-ups, the majority of patients also have a minimum of 1 relapse during their corticosteroid taper (6). “The factors associated with an increased risk of relapse include persistently high levels of IL-6 and CRP despite treatment, in addition to a rapid rate of prednisone withdrawal (6).” Relapses are most likely to happen within the first 18 months of corticosteroid treatment and then within 1 year of withdrawal (7). Therefore, it is important for all patients to be closely monitored for any signs of symptoms reappearance during their corticosteroid tapering, in addition to 12 months after therapy is discontinued (7). The next step in potential PMR treatment and management is non-steroidal anti-inflammatory drugs, or NSAIDs, such as Ibuprofen or Naproxen (6,7). If there are no contraindications for a patient’s use of NSAIDs, they may be helpful in offering additional pain relief. However, in a study of 232 patients with PMR, an association between NSAIDs and drug-related morbidity was found (7). Therefore, it is suggesting that proceeding with NSAID and corticosteroid treatment should be cautious and perhaps only considered if patients have unbearable adverse effects from corticosteroids (7). Overall, once treated polymyalgia rheumatic usually resolves as quickly as 2-3 days. With the help of a timely diagnosis and sufficient treatment, patients with PMR have a tremendous prognosis (7). Even without treatment, it will resolve on its own but may take as long as several years.

    References

    1. Docken W P. Polymyalgia Rheumatica. American College of Rheumatology. http://www.rheumatology.org/I-Am-A/Patient-Caregiver/Diseases-Conditions/Polymyalgia-Rheumatica. June 2015. Accessed March 4, 2016.

    2. Polymyalgia Rhuematica. Mayo Clinic. http://www.mayoclinic.org/diseases-conditions/polymyalgia-rheumatica/basics/symptoms/con-20023162. September 17, 2015. Accessed March 4, 2016.

    3. Polymyalgia Rheumatica and Giant Cell Arteritis. National Institute of Arthritis and Musculoskeletal and Skin Disease. http://www.niams.nih.gov/Health_Info/Polymyalgia/. April 2015. Accessed March 4, 2016.

    4. Helmick CG, Felson DT, Lawrence RC, Gabriel S, Hirsch R, Kwoh CK, et al.; National Arthritis Data Workgroup. Estimates of the Prevalence of Arthritis and Other Rheumatic Conditions in the United States, Part II. Arthritis Rheum. 2008 January ; 58(1): 26?35. doi:10.1002/art.23176

    5. Cimmino M A. Genetic and Environmental Factors In Polymyalgia Rheumatica. Annals of the Rheumatic Diseases – The Eular Journal. 1997: 56;576-577 doi: 10.1136/ard.56.10.576

    6. Clifford A, Hoffman G S. Polymyalgia Rheumatica and Giant Cell Arteritis. Cleveland Clinic – Center for Continuing Education. June 2014. Accessed March 4, 2016.

    7. Saad E R. Polymyalgia Rheumatica. Medscape. http://emedicine.medscape.com/article/330815-overview. September 23, 2015. Accessed March 4, 2016.

    8. Kermani T A, Warrington K J. Advances and Challenges In The Diagnosis and Treatment of Polymyalgia Rheumatica. Therapeutic Advances in Musculoskeletal Disease. February 2014; 6(1):8-19.

    9. Lyme Disease Tests. Lab Tests Online – AACC. https://labtestsonline.org/understanding/analytes/lyme/tab/test/.  October 16, 2015. Accessed March 4, 2016.

  77. Question 77 of 220
    77. Question

    An 82 year old man presents with worsening renal function since this past June. At that time his creatinine was 1.4 mg/mL. In December of this year it went up to 1.93 mg/dL. Earlier this week it was 2.39 mg/dL, and today it was 2.61mg/dL with a BUN of 107 mg/dL. He has not had any gross hematuria, although he has had a history of kidney stones in the past. He denies any edema, UTI, flank pain, fever or chills. His urinary stream is rather weak and he has urinary frequency including nocturia every 2 hours. He has a history of prostate cancer for which he was treated with radiation therapy. His past medical history consists of hypertension, hyperlipidemia, type 2 diabetes mellitus complicated by peripheral neuropathy, CAD, and monoclonal gammopathy of undetermined significance (MGUS). His medications include Atorvastatin, Accupril, and Aleve once a week. He was discontinued from his Lasix 3 days ago. His urine dipstick is unremarkable except for a trace of protein. No other abnormalities found on basic metabolic panel and CBC. Vital signs are stable. What would be the next best step to determine the cause of his worsening kidney function?

    Correct

    The answer is C. This patient has a history of MGUS which can progress to multiple myeloma, which could present as a renal injury. A urine dipstick only detects albumin in the urine and no other proteins such as Bence Jones proteins as seen with multiple myeloma. The 24 hour urine will also give a more accurate creatinine clearance. Renal biopsy is performed after other less invasive exams have been exhausted and not rendered a diagnosis for increasing renal injury. Although Aleve should be discontinued as it decreases blood flow to the kidneys and can cause renal injury, a 24-hour urine collection needs to be ordered. Tylenol would be a better option for pain management. This patient has a weak urinary stream and it is important to make sure this patient does not have an obstruction causing hydro nephrosis and kidney damage. CT has a high dose of radiation and is expensive. An ultrasound is a better option for first imaging studies and should be ordered to rule out an obstruction.

    MGUS is the most common plasma cell proliferative disorder and occurs in approximately 2-3% of the white population of greater than 50 years of age. Men are more commonly affected and it is 2-3 times more common among the black population. There is no discrimination between the white and black population when it comes to progression of MGUS to multiple myeloma (MM). There is a genetic component. Patients who have a first degree relative with MGUS or MM are at a two-to-threefold higher risk of developing MGUS. Obesity and immunosuppression also increase your risk of developing MGUS. 1

    The cause of progression from MGUS to MM are not well understood. There are, however, different types of MGUS that progress to different malignancies. The IgA and IgG variant have shown progression to MM, primary amyloidosis, plasma-cell leukemia or light-chain deposition disease. The IgM variant has an increased risk of progression to Waldenstrom macroglobulinemia, CLL, or other types of non-Hodgkin lymphoma and very rarely progresses to MM.2
    MGUS is most often asymptomatic and an incidental finding when working up an older patient with several common symptoms and laboratory abnormalities in which a serum-protein electrophoresis or immunofixation is ordered. A monoclonal (M) protein is often found in the serum or urine. Patient that are symptomatic usually have the IgM variant and present with peripheral neuropathies. This can easily be overlooked in patients already suffering from diabetic neuropathy. This finding does require a multiple myeloma work up. MGUS differs from MM in that it does not have any of the CRAB criteria for multiple myeloma. For the diagnosis of MGUS the patient cannot have hypercalcemia, renal failure, anemia and lytic bone lesions attributed to plasma-cell proliferative disorders.2

    The diagnostic tests are serum protein electrophoresis with immunofixation and measurement of serum immunoglobulins. In MGUS the patient is positive for monoclonal (M) protein of There is no treatment for MGUS and it often does not cause any complications. It often goes undetected for many years since it is asymptomatic. The practitioner should be aware of alarming symptoms that could mean the MGUS has progressed to MM or other malignant disorders and know what exams to order. In our case above, the patient came to the nephrology clinic due to an increase in his creatinine levels, indicating worsening of renal function. This patient was fairly asymptomatic except for neuropathy in his lower extremities, most likely from his diabetic neuropathy. He was taking Lasix but was taken off of it 3 days prior to the appointment with us. He was still taking Aleve. The Lasix could have made him volume depleted. The Aleve is decreasing blood flow to the kidneys. Both of these cause a pre-renal injury and cause a spike in serum creatinine levels. His urinary frequency and weak stream could be a sign of an obstruction and an ultrasound should be ordered to check for hydronephrosis. We did a urinalysis in the office and it was pretty benign with trace protein. Since he has a history of MGUS and one of the signs of progression to MM is worsening kidney functions, the urine should be evaluated for Bence Jones proteins also known as light chain proteins. We ordered a 24-hour urine collection since it gives a more accurate creatinine clearance than serum calculated levels and gives us the total protein excreted. Dip stick urinalysis are good at detecting albumin in the urine but does not do a good job at detecting smaller proteins such as light chains. We followed up with our patient after two weeks and his 24-hour urine collection showed a creatinine of 1.7 (decreased but still elevated) and an abnormally high amount of kappa light chains. This could mean that he has light chain MGUS which is benign but can progress to MM or that he has light chain deposition disease in his kidneys which has a poor prognosis. The only way to get a better understanding of what is happening in his kidneys and to determine if it is something that should be treated is to do a renal biopsy. We performed a renal biopsy on this patient but results are pending. In this case the renal biopsy will tell us if the patient has something that can be treated and prevent further renal injury or if treatment is not indicated.4

    References

    1. Greer, John P., and Maxwell Myer Wintrobe. Wintrobe’s Clinical Hematology. Philadelphia, PA: Lippincott Williams & Wilkins, 2014.

    2. Epocrates. MGUS https://online.epocrates.com/u/29611173/Monoclonal+gammopathy+of+undetermined+significance. Accessed February 10, 2016.

    3. Damon LE, Andreadis C. Blood Disorders. In: Papadakis MA, McPhee SJ, Rabow MW. eds. Current Medical Diagnosis & Treatment 2015. New York, NY: McGraw-Hill; 2014. http://accessmedicine.mhmedical.com.ezproxymcp.flo.org/content.aspx?bookid=1019&Sectionid=57668605. Accessed February 10, 2016.

    4. Watnick S, Dirkx T. Kidney Disease. In: Papadakis MA, McPhee SJ, Rabow MW. eds. Current Medical Diagnosis & Treatment 2015. New York, NY: McGraw-Hill; 2014. http://accessmedicine.mhmedical.com.ezproxymcp.flo.org/content.aspx?bookid=1019&Sectionid=57668614. Accessed February 10, 2016.

    Incorrect

    The answer is C. This patient has a history of MGUS which can progress to multiple myeloma, which could present as a renal injury. A urine dipstick only detects albumin in the urine and no other proteins such as Bence Jones proteins as seen with multiple myeloma. The 24 hour urine will also give a more accurate creatinine clearance. Renal biopsy is performed after other less invasive exams have been exhausted and not rendered a diagnosis for increasing renal injury. Although Aleve should be discontinued as it decreases blood flow to the kidneys and can cause renal injury, a 24-hour urine collection needs to be ordered. Tylenol would be a better option for pain management. This patient has a weak urinary stream and it is important to make sure this patient does not have an obstruction causing hydro nephrosis and kidney damage. CT has a high dose of radiation and is expensive. An ultrasound is a better option for first imaging studies and should be ordered to rule out an obstruction.

    MGUS is the most common plasma cell proliferative disorder and occurs in approximately 2-3% of the white population of greater than 50 years of age. Men are more commonly affected and it is 2-3 times more common among the black population. There is no discrimination between the white and black population when it comes to progression of MGUS to multiple myeloma (MM). There is a genetic component. Patients who have a first degree relative with MGUS or MM are at a two-to-threefold higher risk of developing MGUS. Obesity and immunosuppression also increase your risk of developing MGUS. 1

    The cause of progression from MGUS to MM are not well understood. There are, however, different types of MGUS that progress to different malignancies. The IgA and IgG variant have shown progression to MM, primary amyloidosis, plasma-cell leukemia or light-chain deposition disease. The IgM variant has an increased risk of progression to Waldenstrom macroglobulinemia, CLL, or other types of non-Hodgkin lymphoma and very rarely progresses to MM.2
    MGUS is most often asymptomatic and an incidental finding when working up an older patient with several common symptoms and laboratory abnormalities in which a serum-protein electrophoresis or immunofixation is ordered. A monoclonal (M) protein is often found in the serum or urine. Patient that are symptomatic usually have the IgM variant and present with peripheral neuropathies. This can easily be overlooked in patients already suffering from diabetic neuropathy. This finding does require a multiple myeloma work up. MGUS differs from MM in that it does not have any of the CRAB criteria for multiple myeloma. For the diagnosis of MGUS the patient cannot have hypercalcemia, renal failure, anemia and lytic bone lesions attributed to plasma-cell proliferative disorders.2

    The diagnostic tests are serum protein electrophoresis with immunofixation and measurement of serum immunoglobulins. In MGUS the patient is positive for monoclonal (M) protein of There is no treatment for MGUS and it often does not cause any complications. It often goes undetected for many years since it is asymptomatic. The practitioner should be aware of alarming symptoms that could mean the MGUS has progressed to MM or other malignant disorders and know what exams to order. In our case above, the patient came to the nephrology clinic due to an increase in his creatinine levels, indicating worsening of renal function. This patient was fairly asymptomatic except for neuropathy in his lower extremities, most likely from his diabetic neuropathy. He was taking Lasix but was taken off of it 3 days prior to the appointment with us. He was still taking Aleve. The Lasix could have made him volume depleted. The Aleve is decreasing blood flow to the kidneys. Both of these cause a pre-renal injury and cause a spike in serum creatinine levels. His urinary frequency and weak stream could be a sign of an obstruction and an ultrasound should be ordered to check for hydronephrosis. We did a urinalysis in the office and it was pretty benign with trace protein. Since he has a history of MGUS and one of the signs of progression to MM is worsening kidney functions, the urine should be evaluated for Bence Jones proteins also known as light chain proteins. We ordered a 24-hour urine collection since it gives a more accurate creatinine clearance than serum calculated levels and gives us the total protein excreted. Dip stick urinalysis are good at detecting albumin in the urine but does not do a good job at detecting smaller proteins such as light chains. We followed up with our patient after two weeks and his 24-hour urine collection showed a creatinine of 1.7 (decreased but still elevated) and an abnormally high amount of kappa light chains. This could mean that he has light chain MGUS which is benign but can progress to MM or that he has light chain deposition disease in his kidneys which has a poor prognosis. The only way to get a better understanding of what is happening in his kidneys and to determine if it is something that should be treated is to do a renal biopsy. We performed a renal biopsy on this patient but results are pending. In this case the renal biopsy will tell us if the patient has something that can be treated and prevent further renal injury or if treatment is not indicated.4

    References

    1. Greer, John P., and Maxwell Myer Wintrobe. Wintrobe’s Clinical Hematology. Philadelphia, PA: Lippincott Williams & Wilkins, 2014.

    2. Epocrates. MGUS https://online.epocrates.com/u/29611173/Monoclonal+gammopathy+of+undetermined+significance. Accessed February 10, 2016.

    3. Damon LE, Andreadis C. Blood Disorders. In: Papadakis MA, McPhee SJ, Rabow MW. eds. Current Medical Diagnosis & Treatment 2015. New York, NY: McGraw-Hill; 2014. http://accessmedicine.mhmedical.com.ezproxymcp.flo.org/content.aspx?bookid=1019&Sectionid=57668605. Accessed February 10, 2016.

    4. Watnick S, Dirkx T. Kidney Disease. In: Papadakis MA, McPhee SJ, Rabow MW. eds. Current Medical Diagnosis & Treatment 2015. New York, NY: McGraw-Hill; 2014. http://accessmedicine.mhmedical.com.ezproxymcp.flo.org/content.aspx?bookid=1019&Sectionid=57668614. Accessed February 10, 2016.

  78. Question 78 of 220
    78. Question

    A 72-year-old obese, female presents to your clinic with progressively worsening right knee pain. She mentions that her right knee “feels stiff” more at night than during the day. On physical exam, you note her valgus alignment. She has tenderness over the lateral aspect of her right knee and over the peripatellar region. She has no instability with varus and valgus stress. There is no joint effusion, erythema, warmth or signs of infection. You diagnose her with osteoarthritis. Due to the patient’s “knock-kneed” appearing legs, what would you most likely see on her X-ray?

    Correct

    Answer C. Right knee will show evidence of increased in narrowing in the lateral space joint compared to the medial space joint. Subchondral sclerotic bone development on the lateral aspect and osteophytes.  c) is the correct answer because patients with valgus alignment will have the most stress on the lateral aspects of their knees due to the load bearing axis being shifted to the outer compartments of their knees.3 The image to the right best represents a valgus deformity of an osteoarthritic knee on X-ray.5 There is evidence of joint space narrowing more on the lateral aspect of the knee compared to the medial aspect. This confirms that the misalignment effects the joint region and the mechanical stresses that occurs. 2

    Osteoarthritis (OA) or also known as primary OA, is a degenerative joint disease involving multiple factors which determine the progression of the disease. Compared to secondary osteoarthritis, this type involves underlying conditions that can enhance the risk of developing osteoarthritis such as congenital or bone disorders. 1 Both types of osteoarthritis are commonly diagnosed clinically. Diagnosis is based on patient’s overall age, history and physical, and radiographic findings. Idiopathic OA can be localized or general, and usually occurs in the hands, hips, knees and feet.1 Common risk factors include increasing age, female, obesity, muscle weakness, repetitive use, previous injury and genetics. 1 The patient that is mentioned in the question above has several of these risk factors: her advanced age, female and she is obese. Another risk factor for osteoarthritis is misalignment of the knees. Having a valgus or varus deformity causes a shift in neutral alignment of the hip, knee and ankle. This affects the load bearing axis causing an uneven distribution of mechanical forces. 2

    Pathophysiology: Since osteoarthritis is a chronic disease, it involves the progression of joint degeneration, articular cartilage and synovial fluid damage. It was once thought to be a non-inflammatory process but recent studies have revealed that an inflammatory process does play a role in the disease progression. 1 The degradation process includes the pro-inflammatory factors that produce enzymes that are accountable for the overall joint destruction. Other factors that are found to expedite this process include the risk factors that have been mentioned earlier. Mechanical factors such as repetitive use will result in stimulation of these pro-inflammatory factors. Also, weight bearing status has proven to be a major risk factor to increasing the progression of the disease. Patients who are obese increase their weight bearing load along their hips, knees and ankles, thus causing irritation to the joints.

    History and physical exam findings: Completing a history and physical is essential to the diagnosis. Older adult patients usually complain of localized or general joint pain that is exacerbated with activity and relieved by rest. Joint stiffness is noted to increase throughout the day and even correlated with weather changes.1 On the physical exam, these patients will have limited range of motion and tenderness to palpation on the medial and/or lateral joint line. Pertaining to the patient mentioned in the above question, this patient may have crepitus, where there is arthritic changes in the patellofemoral area.1 Also, patients with varus or valgus alignment are associated with an increase in progression of osteoarthritis and will have pain on varus or valgus stress of the joint. 2

    Diagnostics: Diagnosing osteoarthritis is primarily based on clinical findings. Imaging is pertinent to confirming and assessing the progression of the disease. Radiographic imaging is used to assess the severity and presence of the disease. Findings may include: narrowing of the joint space, subchondral sclerotic bone, osteophytes and/or subchondral cysts. 1 Magnetic resonance imaging (MRI) can be used to confirm the diagnosis by evaluating the presence of cartilage abnormalities, edema, and other pathology. However, it is not necessary to order an MRI for the diagnosis of osteoarthritis. 1 Lab values that can used for part of the patient assessment include, aspiration and evaluation of the synovial fluid, rheumatoid factor (RF) titers, and erythrocyte sedimentation rate (ESR). 1

    Management and Treatment: Treatment of this disease includes conservative management and surgical options. Initial treatment of osteoarthritis tends to favor conservative options such as acetaminophen and other non-inflammatory medications. Patients are also encouraged to participate in exercise programs and weight loss counseling to further symptom relief. 5 Another option for patients that have persistent symptoms are intra-articular injections, which can be cortisone or hyaluronic acid. When patients fail conservative treatment and continue to be symptomatic, surgical consultation is indicated. Total joint replacement is the definitive treatment option for osteoarthritis. It relieves patients’ symptoms and improves joint functionality.1

    References

    1. Kalunian, K.C., Tugwell, P, et al. Clinical manifestations of osteoarthritis. UpToDate. http://www.uptodate.com/contents/clinical-manifestations-of-osteoarthritis?source=machineLearning&search=valgus+osteoarthritis++knee&selectedTitle=4~130&sectionRank=1&anchor=H5#H5. Updated December 30, 2013. Accessed April 23, 2016.
    2. Sharma, L. The role of varus and valgus alignment in knee osteoarthritis. Arthritis and Rheumatism. http://onlinelibrary.wiley.com/doi/10.1002/art.22514/pdf. Published November 4th 2007. Accessed April 23, 2016.
    3. Eustice, C. What is Varus or valgus knee deformity? Verywell. https://www.verywell.com/what-is-varus-or-valgus-knee-deformity-2552048. Updated March 3rd, 2016. Accessed April 23, 2016.
    4. Venables, P.J.W., Maini, R.N., et al. Clinical manifestations of rheumatoid arthritis. UpToDate. http://www.uptodate.com/contents/clinical-manifestations-of-rheumatoid-arthritis?source=machineLearning&search=rheumatoid+arthritis+knee&selectedTitle=1~150&sectionRank=1&anchor=H12#H1598951. Updated November 28th, 2015. Accessed April 23, 2016.
    Incorrect

    Answer C. Right knee will show evidence of increased in narrowing in the lateral space joint compared to the medial space joint. Subchondral sclerotic bone development on the lateral aspect and osteophytes.  c) is the correct answer because patients with valgus alignment will have the most stress on the lateral aspects of their knees due to the load bearing axis being shifted to the outer compartments of their knees.3 The image to the right best represents a valgus deformity of an osteoarthritic knee on X-ray.5 There is evidence of joint space narrowing more on the lateral aspect of the knee compared to the medial aspect. This confirms that the misalignment effects the joint region and the mechanical stresses that occurs. 2

    Osteoarthritis (OA) or also known as primary OA, is a degenerative joint disease involving multiple factors which determine the progression of the disease. Compared to secondary osteoarthritis, this type involves underlying conditions that can enhance the risk of developing osteoarthritis such as congenital or bone disorders. 1 Both types of osteoarthritis are commonly diagnosed clinically. Diagnosis is based on patient’s overall age, history and physical, and radiographic findings. Idiopathic OA can be localized or general, and usually occurs in the hands, hips, knees and feet.1 Common risk factors include increasing age, female, obesity, muscle weakness, repetitive use, previous injury and genetics. 1 The patient that is mentioned in the question above has several of these risk factors: her advanced age, female and she is obese. Another risk factor for osteoarthritis is misalignment of the knees. Having a valgus or varus deformity causes a shift in neutral alignment of the hip, knee and ankle. This affects the load bearing axis causing an uneven distribution of mechanical forces. 2

    Pathophysiology: Since osteoarthritis is a chronic disease, it involves the progression of joint degeneration, articular cartilage and synovial fluid damage. It was once thought to be a non-inflammatory process but recent studies have revealed that an inflammatory process does play a role in the disease progression. 1 The degradation process includes the pro-inflammatory factors that produce enzymes that are accountable for the overall joint destruction. Other factors that are found to expedite this process include the risk factors that have been mentioned earlier. Mechanical factors such as repetitive use will result in stimulation of these pro-inflammatory factors. Also, weight bearing status has proven to be a major risk factor to increasing the progression of the disease. Patients who are obese increase their weight bearing load along their hips, knees and ankles, thus causing irritation to the joints.

    History and physical exam findings: Completing a history and physical is essential to the diagnosis. Older adult patients usually complain of localized or general joint pain that is exacerbated with activity and relieved by rest. Joint stiffness is noted to increase throughout the day and even correlated with weather changes.1 On the physical exam, these patients will have limited range of motion and tenderness to palpation on the medial and/or lateral joint line. Pertaining to the patient mentioned in the above question, this patient may have crepitus, where there is arthritic changes in the patellofemoral area.1 Also, patients with varus or valgus alignment are associated with an increase in progression of osteoarthritis and will have pain on varus or valgus stress of the joint. 2

    Diagnostics: Diagnosing osteoarthritis is primarily based on clinical findings. Imaging is pertinent to confirming and assessing the progression of the disease. Radiographic imaging is used to assess the severity and presence of the disease. Findings may include: narrowing of the joint space, subchondral sclerotic bone, osteophytes and/or subchondral cysts. 1 Magnetic resonance imaging (MRI) can be used to confirm the diagnosis by evaluating the presence of cartilage abnormalities, edema, and other pathology. However, it is not necessary to order an MRI for the diagnosis of osteoarthritis. 1 Lab values that can used for part of the patient assessment include, aspiration and evaluation of the synovial fluid, rheumatoid factor (RF) titers, and erythrocyte sedimentation rate (ESR). 1

    Management and Treatment: Treatment of this disease includes conservative management and surgical options. Initial treatment of osteoarthritis tends to favor conservative options such as acetaminophen and other non-inflammatory medications. Patients are also encouraged to participate in exercise programs and weight loss counseling to further symptom relief. 5 Another option for patients that have persistent symptoms are intra-articular injections, which can be cortisone or hyaluronic acid. When patients fail conservative treatment and continue to be symptomatic, surgical consultation is indicated. Total joint replacement is the definitive treatment option for osteoarthritis. It relieves patients’ symptoms and improves joint functionality.1

    References

    1. Kalunian, K.C., Tugwell, P, et al. Clinical manifestations of osteoarthritis. UpToDate. http://www.uptodate.com/contents/clinical-manifestations-of-osteoarthritis?source=machineLearning&search=valgus+osteoarthritis++knee&selectedTitle=4~130&sectionRank=1&anchor=H5#H5. Updated December 30, 2013. Accessed April 23, 2016.
    2. Sharma, L. The role of varus and valgus alignment in knee osteoarthritis. Arthritis and Rheumatism. http://onlinelibrary.wiley.com/doi/10.1002/art.22514/pdf. Published November 4th 2007. Accessed April 23, 2016.
    3. Eustice, C. What is Varus or valgus knee deformity? Verywell. https://www.verywell.com/what-is-varus-or-valgus-knee-deformity-2552048. Updated March 3rd, 2016. Accessed April 23, 2016.
    4. Venables, P.J.W., Maini, R.N., et al. Clinical manifestations of rheumatoid arthritis. UpToDate. http://www.uptodate.com/contents/clinical-manifestations-of-rheumatoid-arthritis?source=machineLearning&search=rheumatoid+arthritis+knee&selectedTitle=1~150&sectionRank=1&anchor=H12#H1598951. Updated November 28th, 2015. Accessed April 23, 2016.
  79. Question 79 of 220
    79. Question

    A 52 year old male presents to your clinic with a concern for coughing up blood since yesterday afternoon. He explains that his wife is just recuperating from a bout of bronchitis and he is worried that he may have caught her illness. He describes three episodes within the last 24 hours of gagging and coughing into the toilet, producing some brown mucus with bright red streaking. He admits to shortness of breath throughout the day, intermittent coughing spells, headaches, and fatigue. His father passed away unexpectedly and his family has been though much emotional strife. He admits to smoking one cigar per day and increased alcohol consumption to cope with his loss. Medications include bupropion, lorazepam, omeprazole, lisinopril, and a multivitamin. On physical exam the patient is an overweight, middle-aged male. Vitals include T 99.0F, BP 136/90, P 86, RR 15, and O2 98% saturation on room air. He is in no apparent distress. His face is flushed and he is slightly diaphoretic. Cardiovascular exam reveals a normal rate and rhythm without murmurs. Lungs are clear to auscultation bilaterally. The oropharynx is erythematous and without exudates or tonsillar enlargement. Sclerae are glassy and injected. A fine essential tremor is noted in both hands. Abdominal exam reveals a non-tender, distended abdomen. The inferior hepatic border is found to be 2 inches from the right costal margin. What is the most appropriate diagnostic test to evaluate the likely etiology of the patient’s chief complaint?

    Correct

    The correct answer is B) Esophagogastroduodenoscopy (EGD). The patient’s hemoptysis, in addition to tremor, hepatomegaly, abdominal distention, headaches, and elevated BP, are all common presenting symptoms of advanced alcoholic liver disease. He is likely experiencing an irritated, or perhaps torn/ruptured, esophageal varix resulting in blood appearing in his mucus secretions. The increased alcohol consumption is a likely trigger for the progression of an already existing problem. Upon investigating his past medical history, it would be found that this patient is an alcoholic. His lorazepam is taken as needed for withdrawal symptoms. He is on a multivitamin due to the number of nutritional deficits that may develop in alcoholics.

    If the patient also presented with fever and produced a positive lung examination, one would consider a pneumonia or other respiratory infection as the cause of his hemoptysis. If this were the case, then plain radiography and peripheral blood cultures would both be diagnostic of a lung consolidation or the potential systemic manifestations of infection, respectively. A pneumonia, other lung infection, or malignancy would also be high on the differential diagnosis if the patient were a chronic tobacco smoker or had other concomitant comorbidities that would decrease his immunity. The patient does not need an emergent abdominal CT scan as he is not presenting with acute abdominal symptoms. However, he may require further imaging of his liver in the future to screen for hepatic pathology such as hepatocellular carcinoma, a deadly complication of advanced stage liver disease.

    References

    Kazushige Beppu, MD,   Kiyoshi Inokuchi, MD, PhD, FACS,   Nobuhiro Koyanagi, MD,   Shinichi Nakayama, MD,   Hisanobu Sakata, MD,   Seigo Kitano, MD,   Michio Kobayashi, MD. Prediction of variceal hemorrhage by esophageal endoscopy. Gastrointestinal Endoscopy. Volume 27, Issue 4, November 1981, Pages 213-218.

    Dagradi, Angelo E.The Natural History of Esophageal Varices in Patients with Alcoholic Liver Cirrhosis. American Journal of Gastroenterology . Jun1972, Vol. 57 Issue 6, p520-540

    Incorrect

    The correct answer is B) Esophagogastroduodenoscopy (EGD). The patient’s hemoptysis, in addition to tremor, hepatomegaly, abdominal distention, headaches, and elevated BP, are all common presenting symptoms of advanced alcoholic liver disease. He is likely experiencing an irritated, or perhaps torn/ruptured, esophageal varix resulting in blood appearing in his mucus secretions. The increased alcohol consumption is a likely trigger for the progression of an already existing problem. Upon investigating his past medical history, it would be found that this patient is an alcoholic. His lorazepam is taken as needed for withdrawal symptoms. He is on a multivitamin due to the number of nutritional deficits that may develop in alcoholics.

    If the patient also presented with fever and produced a positive lung examination, one would consider a pneumonia or other respiratory infection as the cause of his hemoptysis. If this were the case, then plain radiography and peripheral blood cultures would both be diagnostic of a lung consolidation or the potential systemic manifestations of infection, respectively. A pneumonia, other lung infection, or malignancy would also be high on the differential diagnosis if the patient were a chronic tobacco smoker or had other concomitant comorbidities that would decrease his immunity. The patient does not need an emergent abdominal CT scan as he is not presenting with acute abdominal symptoms. However, he may require further imaging of his liver in the future to screen for hepatic pathology such as hepatocellular carcinoma, a deadly complication of advanced stage liver disease.

    References

    Kazushige Beppu, MD,   Kiyoshi Inokuchi, MD, PhD, FACS,   Nobuhiro Koyanagi, MD,   Shinichi Nakayama, MD,   Hisanobu Sakata, MD,   Seigo Kitano, MD,   Michio Kobayashi, MD. Prediction of variceal hemorrhage by esophageal endoscopy. Gastrointestinal Endoscopy. Volume 27, Issue 4, November 1981, Pages 213-218.

    Dagradi, Angelo E.The Natural History of Esophageal Varices in Patients with Alcoholic Liver Cirrhosis. American Journal of Gastroenterology . Jun1972, Vol. 57 Issue 6, p520-540

  80. Question 80 of 220
    80. Question

    A 28-year old male presents to the office with a swollen and very painful left upper eyelid for the past two days. On examination the lid is red and swollen but no discharge from the eye is present. Which choice is the most likely diagnosis?

    Correct

    The Correct answer is D. A hordeolum, also known as a stye,  is a common staphylococcal abscess characterized by an acute erythematous, swollen, and painful upper or lower eyelid margin. Warm compresses and topical antibiotic ointment are recommended initially. Incision and drainage may be recommended if swelling and pain fails to improve after 48 hours of conservative treatment.

    Entropion is characterized by an inward turning of the eyelid without swelling. The eyelid and skin may rub against the eye causing irritation and discomfort.  Entropion usually occurs in older adults as a result of degeneration of the lid fascia. Lubricating eye drops may help relieve symptoms but surgery is often necessary to prevent corneal injury.

    Blepharitis is characterized by inflammation of the eyelid causing symptoms of irritation, burning, and tearing.  The eye lids are usually thickened with crust and debris around the lashes. Inflammation may be caused by infection, surrounding seborrheic dermatitis, acne rosacea, or blocked meibomian glands. Initial treatment involves warm compresses and gentle eye washing to remove irritants.

    Dacryocystitis is an infection of the lacrimal sac characterized by pain, swelling, tenderness, and redness in the tear sac area.  Purulent discharge may be expressed from the blocked lacrimal sac. Treatment of acute dacryocystitis usually includes systemic antibiotics.

    References

    McPhee SJ, Papadakis MA. 2010 Current Medical Diagnosis & Treatment.  McGraw-Hill Companies. 2010.

    American Family Physicians. http://www.aafp.org/afp/1998/0601/p2695.html. Retrieved on 10/6/14.

    Incorrect

    The Correct answer is D. A hordeolum, also known as a stye,  is a common staphylococcal abscess characterized by an acute erythematous, swollen, and painful upper or lower eyelid margin. Warm compresses and topical antibiotic ointment are recommended initially. Incision and drainage may be recommended if swelling and pain fails to improve after 48 hours of conservative treatment.

    Entropion is characterized by an inward turning of the eyelid without swelling. The eyelid and skin may rub against the eye causing irritation and discomfort.  Entropion usually occurs in older adults as a result of degeneration of the lid fascia. Lubricating eye drops may help relieve symptoms but surgery is often necessary to prevent corneal injury.

    Blepharitis is characterized by inflammation of the eyelid causing symptoms of irritation, burning, and tearing.  The eye lids are usually thickened with crust and debris around the lashes. Inflammation may be caused by infection, surrounding seborrheic dermatitis, acne rosacea, or blocked meibomian glands. Initial treatment involves warm compresses and gentle eye washing to remove irritants.

    Dacryocystitis is an infection of the lacrimal sac characterized by pain, swelling, tenderness, and redness in the tear sac area.  Purulent discharge may be expressed from the blocked lacrimal sac. Treatment of acute dacryocystitis usually includes systemic antibiotics.

    References

    McPhee SJ, Papadakis MA. 2010 Current Medical Diagnosis & Treatment.  McGraw-Hill Companies. 2010.

    American Family Physicians. http://www.aafp.org/afp/1998/0601/p2695.html. Retrieved on 10/6/14.

  81. Question 81 of 220
    81. Question

    A 49 year-old obese women complains of bilateral lower extremity discomfort for several months. She works at the post office and is on her feet most of the day. Her leg swelling and pain seems to be worse in the evenings after work. On exam you find dilated, tortuous veins in bilateral legs. Her calves are soft and Homen’s sign is negative. What is the most likely diagnosis?

    Correct

    Answer is D. Varicose veins are present as dilated, tortuous superficial veins in the lower extremities. Risk factors for developing varicose veins includes female gender, pregnancy, a family history, prolonged standing requirements, and a history of phlebitis.  Symptoms may include dull aching and heaviness of the lower extremities.  Compression stockings and prolonged elevation is recommended initially. Surgery may be indicated if persistent or disabling pain exists.  Common procedures include vein ligation and stripping.

    Superficial thrombophlebitis is characterized by induration, redness, and tenderness along a superficial vein.  Superficial thrombophlebitis may occur with varicose veins and commonly occur with a history of trauma such as an intravenous line insertion.

    Chronic venous insufficiency can be caused by DVT, phlebitis, varicose, veins, and other conditions causing poor venous circulation.  The leg and ankles become progressively swollen and the skin is usually shiny, thin, and cyanotic.

    The cause of deep vein thrombosis (DVT) is often described by Virchow’s triad (stasis, vascular injury, and hypercoagulability).  A history of major surgery, such as total knee and hip arthroplasty, or prolonged immobilization is common.  DVT’s can present with dull aching, cramping, or pain in the back of the knee and calf. Homan’s sign, or pain on passive dorsiflexion of the ankle, is positive in 50% of cases.

    An acute arterial occlusion is a sudden severe pain in the leg caused by an obstruction to the arterial system. The skin distal to the occlusion may become pale, cold to the touch, and pulses may be weak or absent.

    References

    McPhee SJ, Papadakis MA. 2010 Current Medical Diagnosis & Treatment.  McGraw-Hill Companies. 2010.

    Samina S, Collins L. Diagnosis and Treatment of Venous Ulcers.  Am Fam Physician. 2010 Apr 15;81(8):989-996

    Incorrect

    Answer is D. Varicose veins are present as dilated, tortuous superficial veins in the lower extremities. Risk factors for developing varicose veins includes female gender, pregnancy, a family history, prolonged standing requirements, and a history of phlebitis.  Symptoms may include dull aching and heaviness of the lower extremities.  Compression stockings and prolonged elevation is recommended initially. Surgery may be indicated if persistent or disabling pain exists.  Common procedures include vein ligation and stripping.

    Superficial thrombophlebitis is characterized by induration, redness, and tenderness along a superficial vein.  Superficial thrombophlebitis may occur with varicose veins and commonly occur with a history of trauma such as an intravenous line insertion.

    Chronic venous insufficiency can be caused by DVT, phlebitis, varicose, veins, and other conditions causing poor venous circulation.  The leg and ankles become progressively swollen and the skin is usually shiny, thin, and cyanotic.

    The cause of deep vein thrombosis (DVT) is often described by Virchow’s triad (stasis, vascular injury, and hypercoagulability).  A history of major surgery, such as total knee and hip arthroplasty, or prolonged immobilization is common.  DVT’s can present with dull aching, cramping, or pain in the back of the knee and calf. Homan’s sign, or pain on passive dorsiflexion of the ankle, is positive in 50% of cases.

    An acute arterial occlusion is a sudden severe pain in the leg caused by an obstruction to the arterial system. The skin distal to the occlusion may become pale, cold to the touch, and pulses may be weak or absent.

    References

    McPhee SJ, Papadakis MA. 2010 Current Medical Diagnosis & Treatment.  McGraw-Hill Companies. 2010.

    Samina S, Collins L. Diagnosis and Treatment of Venous Ulcers.  Am Fam Physician. 2010 Apr 15;81(8):989-996

  82. Question 82 of 220
    82. Question

    A mother presents with her one-month old daughter with concerns that her baby may have been exposed to chicken pox at a recent birthday party.  The baby is formula fed and exhibits no symptoms of illness.  The mother has a history of a chickenpox infection 7 years ago.  Which class of immunoglobulins is acquired from the mother in utero and protects against this virus?

    Correct

    Answer is A. Immunoglobulins (Ig), also known as antibodies in the blood, are proteins made by the plasma cells of the immune system to fight antigens such as bacteria, virus, and toxins.  The five classes of immunoglobulins include IgG, IgM, IgA, IgD, and IgE.  IgG is the smallest of all immunoglobulins and by far the most abundant of all found throughout the body.  IgG is the only class of Ig that crosses the placenta and provides passive immunity from the mother to the newborn.  The mother has a history of chickenpox exposure and the infant is exhibiting passive immunity acquired from the mother in utero.  IgG constitutes 90% of all antibodies in the fetus due to the maternal passive immunity.  Maternal IgG provides the infant with maternal antibodies during the first 4 months of life.   Around 2-3 months after birth, the infant’s immune system begins to produce other immunoglobulins. IgM is the first Ig that appears in the infant’s circulation after immunization and is the most abundant antibody produced in neonatal period.  IgM antibodies are the first antibodies to respond to an infection.  IgA predominately protects against enteric organisms and is not produced until several weeks after birth. IgD is not well understood but seems to be involved in the allergic response and hypersensitivity reactions.  IgE antibodies are found in high levels in the lungs, skin, and mucous membranes.  IgE antibodies induce an allergic response to foreign antigens and are found in high levels in people with allergies. Breast milk also contains all five types of maternal antibodies which can also help the infant against diseases and infection.

    Reference

    Hay WW, Levin MJ, Detarding RR, Abzug MJ. Current Diagnosis and Treatment Pediatrics. 22nd Edition. McGraw-Hill Companies. 2012.

    Incorrect

    Answer is A. Immunoglobulins (Ig), also known as antibodies in the blood, are proteins made by the plasma cells of the immune system to fight antigens such as bacteria, virus, and toxins.  The five classes of immunoglobulins include IgG, IgM, IgA, IgD, and IgE.  IgG is the smallest of all immunoglobulins and by far the most abundant of all found throughout the body.  IgG is the only class of Ig that crosses the placenta and provides passive immunity from the mother to the newborn.  The mother has a history of chickenpox exposure and the infant is exhibiting passive immunity acquired from the mother in utero.  IgG constitutes 90% of all antibodies in the fetus due to the maternal passive immunity.  Maternal IgG provides the infant with maternal antibodies during the first 4 months of life.   Around 2-3 months after birth, the infant’s immune system begins to produce other immunoglobulins. IgM is the first Ig that appears in the infant’s circulation after immunization and is the most abundant antibody produced in neonatal period.  IgM antibodies are the first antibodies to respond to an infection.  IgA predominately protects against enteric organisms and is not produced until several weeks after birth. IgD is not well understood but seems to be involved in the allergic response and hypersensitivity reactions.  IgE antibodies are found in high levels in the lungs, skin, and mucous membranes.  IgE antibodies induce an allergic response to foreign antigens and are found in high levels in people with allergies. Breast milk also contains all five types of maternal antibodies which can also help the infant against diseases and infection.

    Reference

    Hay WW, Levin MJ, Detarding RR, Abzug MJ. Current Diagnosis and Treatment Pediatrics. 22nd Edition. McGraw-Hill Companies. 2012.

  83. Question 83 of 220
    83. Question

    A 57-year old obese male presents to your office to discuss treatment of his high blood pressure.  Despite trying diet and exercise his systolic blood pressures have been in the 150’s and his diastolic pressures in the 90’s.  PMH history includes type 2 diabetes, eczema, and depression.  Which of the following antihypertensive medications should be recommended?

    Correct

    Answer is D. Prolonged elevated blood sugars in diabetic patients often results in damage to the kidneys over time.  Hypertension can also damage the filtering ability of the kidneys, and if left untreated in diabetic patients, can result in kidney failure.  Diabetes is the leading cause of chronic kidney disease and renal failure in the United States. An early sign of kidney damage includes the presence protein in the urine (proteinuria) and a decreased glomerular filtration rate. Treatment of HTN in diabetics has been shown to reduce the incidence of proteinuria and improve kidney function.  ACE inhibitors, such as enalapril, are the first line medications for hypertensive diabetics. ACE inhibitors are renoprotective by reducing systemic vascular resistance and renal perfusion pressure.  ACE inhibitors also cause intrarenal vasodilation further reducing perfusion pressure which protects the filtering glomeruli.  Enalapril, an ACE inhibitor, would be the best option for this patient.

    References

    Management of Diabetic Hypertension. http://www.ncbi.nlm.nih.gov/pmc/articles/PMC3230084/. Accessed on 11/9/14.

    Kidney Disease in Diabetes. http://kidney.niddk.nih.gov/kudiseases/pubs/kdd/.  Accessed on 11/9/14.

    Incorrect

    Answer is D. Prolonged elevated blood sugars in diabetic patients often results in damage to the kidneys over time.  Hypertension can also damage the filtering ability of the kidneys, and if left untreated in diabetic patients, can result in kidney failure.  Diabetes is the leading cause of chronic kidney disease and renal failure in the United States. An early sign of kidney damage includes the presence protein in the urine (proteinuria) and a decreased glomerular filtration rate. Treatment of HTN in diabetics has been shown to reduce the incidence of proteinuria and improve kidney function.  ACE inhibitors, such as enalapril, are the first line medications for hypertensive diabetics. ACE inhibitors are renoprotective by reducing systemic vascular resistance and renal perfusion pressure.  ACE inhibitors also cause intrarenal vasodilation further reducing perfusion pressure which protects the filtering glomeruli.  Enalapril, an ACE inhibitor, would be the best option for this patient.

    References

    Management of Diabetic Hypertension. http://www.ncbi.nlm.nih.gov/pmc/articles/PMC3230084/. Accessed on 11/9/14.

    Kidney Disease in Diabetes. http://kidney.niddk.nih.gov/kudiseases/pubs/kdd/.  Accessed on 11/9/14.

  84. Question 84 of 220
    84. Question

    A 20 year-old female presents with sore throat, odynophagia, headache, and fever for the past two days.  She reports ill contacts at work only.  She complains of nausea, loss of appetite, fatigue, tinnitus, and odynophagia. On examination, she has bilaterally tender, swollen anterior cervical lymphadenopathy, bilateral tonsillar hypertrophy with exudates, and fever of 100.9°F oral. According to Centor criteria, which of the following is an essential clinical feature consistent with the diagnosis of Group A Streptococcal pharyngitis?

    Correct

    The answer is C. In the diagnosis of Group A streptococcal pharyngitis, Centor’s criteria is an important clinical tool that physicians use for the management and treatment of GABHS. Centor’s criteria consists of: anterior cervical lymphadenopathy, pharyngotonsillar exudate, a temperature of 100.9°F, and the absence of cough. Streptococcal pharyngitis is diagnosed in over 11 million patients each year1, with the highest incidence occurring among children 5 to 15 years of age.  This infection is present most commonly in the winter and the early spring.  Risk factors for developing streptococcal pharyngitis include chronic medical illnesses such as diabetes, immunodeficiency, being within crowded environments such as schools, and exposure through infected persons.

    Although viruses are the most common causes of acute pharyngitis, Group A beta-hemolytic Streptococcus (GABHS) is the most common bacterial cause of pharyngitis.  Some of the respiratory viruses that cause acute pharyngitis include adenovirus, inflenza virus, RSV, and rhinovirus, among many others.  However, GABHS accounts for about 15-30% of cases of acute pharyngitis in children and 5-20% of cases in adults.  This infection is transmitted through respiratory droplets, which means that the bacteria is spread through the droplets after an infected person coughs or sneezes.  It is important in preventing the spread of infection by practicing good hand washing techniques, refraining from drinking from the same glass as an infected person, or eating food that an infected person had eaten.

    The microorganism Group A beta-hemolytic streptococci (Streptococcus pyogenes), is a gram positive extracellular bacterial pathogen that colonizes the throat or skin.  There are few individuals that have chronic GABHS colonization, but they do not have clinical symptoms of the disease, and these individuals are at low risk of transmitting the disease to others.  In the majority of individuals, the proliferation of GABHS in the pharynx causes the clinical signs and symptoms of the infection.  This is due to the fact that GABHS has many surface proteins and it produces extracellular products that cause evasion of the immune system.  Specifically, the M protein is responsible for the virulence and clinical symptoms that are seen in infected individuals.

    In the clinical diagnosis of GABHS in patients, it is important to first differentiate the clinical differences between the viral and bacterial etiologies of acute pharyngitis.  Patients with bacterial etiology (GABHS) present with sudden onset of a sore throat, odynophagia, and fever.  In children, headache, nausea, vomiting, and abdominal pain may be present.  On examination, there is inflammation of the pharynx and tonsils with or without exudates and enlarged anterior cervical lymph nodes.  Clinicians often use the “Centor Criteria” for the diagnosis which includes temperature >100.4°F, anterior cervical lymphadenopathy, absence of a cough, and pharyngotonsillar exudate. Conversely, patients with acute pharyngitis of viral origin present with conjunctivitis, cough, hoarseness, coryza, viral exanthems, diarrhea, and the absence of a fever.  It is also important to ask the patient if they have a history of exposure to an infected person with GABHS, as this is significant information to aid in the diagnosis.

    The diagnostic tests that are useful in detecting GABHS include the rapid antigen detection test (RADT) and throat culture.  The culture of a throat swab for detecting the presence of GABHS has a sensitivity of 90-95%6, whereas the RADT for strep is slightly less sensitive.  Using the Centor criteria, clinicians identify which patients need a rapid antigen test or throat culture.  When the patients have met >2 out of the 4 criteria, these patients require further testing.  When >3 out of the 4 criteria are present, then the rapid antigen testing sensitivity is over 90%.  With less than 2 out of the 4 criteria present, GABHS is not likely.  This being said, if the patient has more than 2 out of the 4 criteria, the clinician will perform the RADT or throat culture.  If the RADT is positive, then it is not necessary to perform a throat culture.  However, if a RADT is negative in high prevalence settings and clinical suspicion is high for the infection, a throat culture should be performed.  For children and adolescents, all negative RADT results should be followed with a throat culture.  Due to the lower incidence of streptococcal infection in adults and low risk of rheumatic fever, it is not necessary to confirm the diagnosis of the infection with a throat culture following a negative RADT.

    Group A streptococcal pharyngitis is a often a self-limited disease, which means that the fever and constitutional symptoms reside in 3-4 days without the use of antibiotic therapy.  However, the use of antibiotics is indicated for those patients experiencing symptoms of the infection with a positive throat culture or rapid antigen detection test.  In addition, if there is a high index of suspicion for the infection, and the clinician is waiting for laboratory results, the patient should also be placed on antibiotics. The Centor criteria is also used in the management of the disease, thus when patients have 4 criteria met, antibiotics should be started. The antibiotics that are effective in treating GABHS include penicillin, ampicillin, amoxicillin, clindamycin, certain cephalosporin and macrolides.  The course of treatment for the antibiotics is usually 10 days.  The treatment of choice for GABHS pharyngitis remains penicillin due to its efficacy and safety, low cost, and narrow spectrum6.  However, amoxicillin can be used instead with equal effectiveness and more tolerable taste.  For non-compliant patients who are not able to finish a full 10 day course of antibiotics, it is recommended that they be treated with a single IM dose of penicillin G benzathine.  Also, treatment guidelines state that those with a penicillin allergy should be treated with erythromycin or second generation macrolides azithromycin or clarithromycin.  Rarely, there are patients with erythromycin-resistant strain of GABHS and when they are unable to tolerate beta lactam antibiotics, clindamycin is the appropriate choice.  In addition, symptomatic treatment includes acetaminophen for reducing fever, cough drops as needed for throat discomfort, rest, and plenty of fluids.

    References

    Choby, B. Diagnosis and Treatment of Streptococcal Pharyngitis. American Family Physician . 2009;79(5):383–390.

    Gerber, M, Baltimore, R, Eaton, C, et al. Prevention of Rheumatic Fever and Diagnosis and Treatment of Acute Streptococcal Pharyngitis. American Heart Association. 2009;119:1541–1551.

    Chahine, E, Chamoun, J, Sucher, A. Update on the Management of Streptococcal Pharyngitis. US Pharmacist. 2013;38(7):51–56.

    Strep Throat. National Center for Biotechnology Information. Available at: http://www.ncbi.nlm.nih.gov/pubmedhealth/pmht0024690/. Accessed March 10, 2016.

    Cunningham, M. Pathogenesis of Group A Streptococcal Infections. Clinical Microbiology Reviews. 2000;13(3):470–511.

    Alan, B., Gerber, M., Gwaltney, J., Kaplan, E., Schwartz, RH. Practice Guidelines For The Diagnosis And Management Of Group A Streptococcal Pharyngitis. Infectious Diseases in Clinical Practice. 2002;35:113–123.

    Papadakis MA, McPhee SJ, Rabow MW. 2015 Current Medical Diagnosis and Treatment. New York: McGraw-Hill Education/Medical 2015.

    Incorrect

    The answer is C. In the diagnosis of Group A streptococcal pharyngitis, Centor’s criteria is an important clinical tool that physicians use for the management and treatment of GABHS. Centor’s criteria consists of: anterior cervical lymphadenopathy, pharyngotonsillar exudate, a temperature of 100.9°F, and the absence of cough. Streptococcal pharyngitis is diagnosed in over 11 million patients each year1, with the highest incidence occurring among children 5 to 15 years of age.  This infection is present most commonly in the winter and the early spring.  Risk factors for developing streptococcal pharyngitis include chronic medical illnesses such as diabetes, immunodeficiency, being within crowded environments such as schools, and exposure through infected persons.

    Although viruses are the most common causes of acute pharyngitis, Group A beta-hemolytic Streptococcus (GABHS) is the most common bacterial cause of pharyngitis.  Some of the respiratory viruses that cause acute pharyngitis include adenovirus, inflenza virus, RSV, and rhinovirus, among many others.  However, GABHS accounts for about 15-30% of cases of acute pharyngitis in children and 5-20% of cases in adults.  This infection is transmitted through respiratory droplets, which means that the bacteria is spread through the droplets after an infected person coughs or sneezes.  It is important in preventing the spread of infection by practicing good hand washing techniques, refraining from drinking from the same glass as an infected person, or eating food that an infected person had eaten.

    The microorganism Group A beta-hemolytic streptococci (Streptococcus pyogenes), is a gram positive extracellular bacterial pathogen that colonizes the throat or skin.  There are few individuals that have chronic GABHS colonization, but they do not have clinical symptoms of the disease, and these individuals are at low risk of transmitting the disease to others.  In the majority of individuals, the proliferation of GABHS in the pharynx causes the clinical signs and symptoms of the infection.  This is due to the fact that GABHS has many surface proteins and it produces extracellular products that cause evasion of the immune system.  Specifically, the M protein is responsible for the virulence and clinical symptoms that are seen in infected individuals.

    In the clinical diagnosis of GABHS in patients, it is important to first differentiate the clinical differences between the viral and bacterial etiologies of acute pharyngitis.  Patients with bacterial etiology (GABHS) present with sudden onset of a sore throat, odynophagia, and fever.  In children, headache, nausea, vomiting, and abdominal pain may be present.  On examination, there is inflammation of the pharynx and tonsils with or without exudates and enlarged anterior cervical lymph nodes.  Clinicians often use the “Centor Criteria” for the diagnosis which includes temperature >100.4°F, anterior cervical lymphadenopathy, absence of a cough, and pharyngotonsillar exudate. Conversely, patients with acute pharyngitis of viral origin present with conjunctivitis, cough, hoarseness, coryza, viral exanthems, diarrhea, and the absence of a fever.  It is also important to ask the patient if they have a history of exposure to an infected person with GABHS, as this is significant information to aid in the diagnosis.

    The diagnostic tests that are useful in detecting GABHS include the rapid antigen detection test (RADT) and throat culture.  The culture of a throat swab for detecting the presence of GABHS has a sensitivity of 90-95%6, whereas the RADT for strep is slightly less sensitive.  Using the Centor criteria, clinicians identify which patients need a rapid antigen test or throat culture.  When the patients have met >2 out of the 4 criteria, these patients require further testing.  When >3 out of the 4 criteria are present, then the rapid antigen testing sensitivity is over 90%.  With less than 2 out of the 4 criteria present, GABHS is not likely.  This being said, if the patient has more than 2 out of the 4 criteria, the clinician will perform the RADT or throat culture.  If the RADT is positive, then it is not necessary to perform a throat culture.  However, if a RADT is negative in high prevalence settings and clinical suspicion is high for the infection, a throat culture should be performed.  For children and adolescents, all negative RADT results should be followed with a throat culture.  Due to the lower incidence of streptococcal infection in adults and low risk of rheumatic fever, it is not necessary to confirm the diagnosis of the infection with a throat culture following a negative RADT.

    Group A streptococcal pharyngitis is a often a self-limited disease, which means that the fever and constitutional symptoms reside in 3-4 days without the use of antibiotic therapy.  However, the use of antibiotics is indicated for those patients experiencing symptoms of the infection with a positive throat culture or rapid antigen detection test.  In addition, if there is a high index of suspicion for the infection, and the clinician is waiting for laboratory results, the patient should also be placed on antibiotics. The Centor criteria is also used in the management of the disease, thus when patients have 4 criteria met, antibiotics should be started. The antibiotics that are effective in treating GABHS include penicillin, ampicillin, amoxicillin, clindamycin, certain cephalosporin and macrolides.  The course of treatment for the antibiotics is usually 10 days.  The treatment of choice for GABHS pharyngitis remains penicillin due to its efficacy and safety, low cost, and narrow spectrum6.  However, amoxicillin can be used instead with equal effectiveness and more tolerable taste.  For non-compliant patients who are not able to finish a full 10 day course of antibiotics, it is recommended that they be treated with a single IM dose of penicillin G benzathine.  Also, treatment guidelines state that those with a penicillin allergy should be treated with erythromycin or second generation macrolides azithromycin or clarithromycin.  Rarely, there are patients with erythromycin-resistant strain of GABHS and when they are unable to tolerate beta lactam antibiotics, clindamycin is the appropriate choice.  In addition, symptomatic treatment includes acetaminophen for reducing fever, cough drops as needed for throat discomfort, rest, and plenty of fluids.

    References

    Choby, B. Diagnosis and Treatment of Streptococcal Pharyngitis. American Family Physician . 2009;79(5):383–390.

    Gerber, M, Baltimore, R, Eaton, C, et al. Prevention of Rheumatic Fever and Diagnosis and Treatment of Acute Streptococcal Pharyngitis. American Heart Association. 2009;119:1541–1551.

    Chahine, E, Chamoun, J, Sucher, A. Update on the Management of Streptococcal Pharyngitis. US Pharmacist. 2013;38(7):51–56.

    Strep Throat. National Center for Biotechnology Information. Available at: http://www.ncbi.nlm.nih.gov/pubmedhealth/pmht0024690/. Accessed March 10, 2016.

    Cunningham, M. Pathogenesis of Group A Streptococcal Infections. Clinical Microbiology Reviews. 2000;13(3):470–511.

    Alan, B., Gerber, M., Gwaltney, J., Kaplan, E., Schwartz, RH. Practice Guidelines For The Diagnosis And Management Of Group A Streptococcal Pharyngitis. Infectious Diseases in Clinical Practice. 2002;35:113–123.

    Papadakis MA, McPhee SJ, Rabow MW. 2015 Current Medical Diagnosis and Treatment. New York: McGraw-Hill Education/Medical 2015.

  85. Question 85 of 220
    85. Question

    A 31 year-old patient comes in for her annual exam. She has had negative Paps every year since she was 21 years old. What testing would you recommend for her today?

    Correct

    Answer is C. Cervical cancer is the third most common gynecological cancer, after endometrial and ovarian cancer. There are two types of cervical carcinoma. Squamous cell accounts for 90% of the cervical carcinomas, while adenocarcinomas account for the other 10%. The average age of diagnosis is 45 years old. HPV infection is associated with 99.7% of cervical carcinomas. The most common HPV types associated are 16, 18, 31, and 33. Risk factors include early onset of sexual activity, an increased number of sexual partners, smoking, CIN, DES exposure, immunosuppression, and a history of sexually transmitted infections.

    The current guidelines to screen for cervical cancer, are to start testing with Paps alone at age 21 regardless of sexual activity or risk factors. Paps should then be performed every 3 years until age 30. For women 30 years old and older with 10 previous, consecutive, negative Paps, the recommendation is to have a Pap with HPV co-testing and repeat in 5 years if both are negative. If the Pap is done alone the recommendation is to repeat in 3 years if negative. Women greater than 65 years old can discontinue Pap and HPV screening if they have had 3 previous negative Paps or 2 previous negative co-tests within the past 10 years with the last test being within the past 5 years. Also, they cannot have a history of CIN2 or higher in the past 20 years.

    The results typically reported after a Pap test are normal, atypical squamous cells of undetermined significance (ASC-US), low grade squamous intraepithelial lesion (LSIL), high grade squamous intraepithelial lesion (HSIL).  Any level of dysplasia found on a Pap smear more significant than ASC-US should be evaluated with a colposcopy regardless of HPV testing or prior Pap test results.  ASC-US with positive HPV would also warrant a colposcopy, while ASC-US with negative HPV would require a repeat Pap in one year.  A repeat ASC-US at that time would result in colposcopy.

    References

    Cervical Cancer. Centers for Disease Control and Prevention 2014. Available at: http://www.cdc.gov/cancer/cervical. Accessed March 9, 2016.

    Williams DA. PANCE Prep Pearls: a Practical Study Guide to Preparing for Physician Assistant National Certifying Exam (PANCE), Recertification (PANRE) &Amp; Clinical Rotations. 1st ed. Lexington, KY: CreateSpace; 2014.

    Incorrect

    Answer is C. Cervical cancer is the third most common gynecological cancer, after endometrial and ovarian cancer. There are two types of cervical carcinoma. Squamous cell accounts for 90% of the cervical carcinomas, while adenocarcinomas account for the other 10%. The average age of diagnosis is 45 years old. HPV infection is associated with 99.7% of cervical carcinomas. The most common HPV types associated are 16, 18, 31, and 33. Risk factors include early onset of sexual activity, an increased number of sexual partners, smoking, CIN, DES exposure, immunosuppression, and a history of sexually transmitted infections.

    The current guidelines to screen for cervical cancer, are to start testing with Paps alone at age 21 regardless of sexual activity or risk factors. Paps should then be performed every 3 years until age 30. For women 30 years old and older with 10 previous, consecutive, negative Paps, the recommendation is to have a Pap with HPV co-testing and repeat in 5 years if both are negative. If the Pap is done alone the recommendation is to repeat in 3 years if negative. Women greater than 65 years old can discontinue Pap and HPV screening if they have had 3 previous negative Paps or 2 previous negative co-tests within the past 10 years with the last test being within the past 5 years. Also, they cannot have a history of CIN2 or higher in the past 20 years.

    The results typically reported after a Pap test are normal, atypical squamous cells of undetermined significance (ASC-US), low grade squamous intraepithelial lesion (LSIL), high grade squamous intraepithelial lesion (HSIL).  Any level of dysplasia found on a Pap smear more significant than ASC-US should be evaluated with a colposcopy regardless of HPV testing or prior Pap test results.  ASC-US with positive HPV would also warrant a colposcopy, while ASC-US with negative HPV would require a repeat Pap in one year.  A repeat ASC-US at that time would result in colposcopy.

    References:

    Cervical Cancer. Centers for Disease Control and Prevention 2014. Available at: http://www.cdc.gov/cancer/cervical. Accessed March 9, 2016.

    Williams DA. PANCE Prep Pearls: a Practical Study Guide to Preparing for Physician Assistant National Certifying Exam (PANCE), Recertification (PANRE) &Amp; Clinical Rotations. 1st ed. Lexington, KY: CreateSpace; 2014.

  86. Question 86 of 220
    86. Question

    A 7 year-old female with a history of controlled asthma and allergies presents for her well child check. Mom mentions that she noticed some pink lesions on her daughter’s skin about 4 days ago. Mom points out some lesions that are fading on her daughter’s chest and some that are present on her back and on her buttocks. On exam you discover a couple of other lesions in her axillary area and on her chest. You note some lesions as macular while others are slightly raised plaques with central clearing and a “collarette” of a scale. The lesions vary in size and are both oval and annular in shape. Her daughter has otherwise been well prior to the onset of these lesions and suffers only from minimal pruritus. There is no personal or family history of psoriasis and no one else around the child has similar symptoms. KOH scrapings do not show hyphae. What is the most likely diagnosis?

    Correct

    The salmon colored or hyperpigmented lesions of pityriasis rosea are somewhat inflammatory, and are characterized as macular to slightly raised plaques that are oval or annular in shape. Lesions are located on the trunk and proximal regions of the extremities.  Itching is a common symptom and few experience sore throat, headache and fatigue as a prodrome. The exanthem is common in the spring and fall months, and begins with a “herald” patch measuring 2-5 cm in diameter that develops a scale. Central clearing occurs leaving behind the classic “collarette” of a scale. In a matter of days or weeks, smaller lesions cluster in the typical distribution along cleavage lines forming the characteristic “Christmas tree” pattern on the back. A v shaped pattern may arise on the chest. The lesions resolve within 4-6 weeks, however they can persist for a period of months. Atypical presentations may occur in children. For example the nature of the lesions may be more papular, vesicular, or purpuric and the distribution may include the face, scalp, groin, axillary areas and distal extremities. The absence of a herald patch due to partial resolution of the exanthem may make diagnosis more difficult. Management is expectant with the aim to control pruritus.  Effective options include the use of antihistamines, topical steroids and possibly UVB treatment.

    If a patient presents acutely with a herald patch, one may mistake this lesion for tinea corporis due to its similar annular shape, central clearing and peripheral scale. Patients manifesting multiple lesions of pityriasis rosea can be differentiated from those with tinea corporis as tinea corporis is generally more localized with fewer annular lesions that may have multiple concentric circles rather than one. Branching hyphae are seen on the KOH scraping in patients with tinea corporis. Hypopigmented or hyperpigmented macules characteristic of tinea versicolor  present on the neck and trunk may also be confused with pityriasis rosea. These lesions lack erythema and have a fine scale rather than the classic “collarette” of a scale. There is no “herald” patch present. KOH scrapings show Pityrosporum described as a “spaghetti and meatball” pattern.

    Cutaneous manifestations of secondary syphilis should be considered in sexually active patients. Pityriasis rosea spares the palms and soles while the red to brown macules of secondary syphilis do not.  Patients with syphilis do not report the appearance of a “herald” patch but may have noticed signs of a chancre.

    Guttate psoriasis is common amongst children regardless of previous psoriasis history. Patients do not recall the presence of a “herald” patch, but may report recent strep infection prior to the eruption. The lesions of guttate psoriasis also occur on the trunk as oval or round erythematous plaques which have a thick, adherent scale.

    References

    Goldstein, AO and Goldstein, BG. Pityriasis Rosea. In: UpToDate, Orfori, AO (Ed), Waltham,         MA. (Accessed on May, 11, 2016).

    Lawley LP, McCall CO, Lawley TJ. Eczema, Psoriasis, Cutaneous Infections, Acne, and Other Common Skin Disorders. In: Kasper D, Fauci A, Hauser S, Longo D, Jameson J,        Loscalzo J. eds. Harrison’s Principles of Internal Medicine, 19e. New York, NY:         McGraw-Hill; 2015.

    Usatine RP, Smith MA, Chumley HS, Mayeaux EJ, Jr.. Chapter 153. Pityriasis Rosea. In:    Usatine RP, Smith MA, Chumley HS, Mayeaux EJ, Jr.. eds. The Color Atlas of Family         Medicine, 2e. New York, NY: McGraw-Hill; 2013.

    Incorrect

    The salmon colored or hyperpigmented lesions of pityriasis rosea are somewhat inflammatory, and are characterized as macular to slightly raised plaques that are oval or annular in shape. Lesions are located on the trunk and proximal regions of the extremities.  Itching is a common symptom and few experience sore throat, headache and fatigue as a prodrome. The exanthem is common in the spring and fall months, and begins with a “herald” patch measuring 2-5 cm in diameter that develops a scale. Central clearing occurs leaving behind the classic “collarette” of a scale. In a matter of days or weeks, smaller lesions cluster in the typical distribution along cleavage lines forming the characteristic “Christmas tree” pattern on the back. A v shaped pattern may arise on the chest. The lesions resolve within 4-6 weeks, however they can persist for a period of months. Atypical presentations may occur in children. For example the nature of the lesions may be more papular, vesicular, or purpuric and the distribution may include the face, scalp, groin, axillary areas and distal extremities. The absence of a herald patch due to partial resolution of the exanthem may make diagnosis more difficult. Management is expectant with the aim to control pruritus.  Effective options include the use of antihistamines, topical steroids and possibly UVB treatment.

    If a patient presents acutely with a herald patch, one may mistake this lesion for tinea corporis due to its similar annular shape, central clearing and peripheral scale. Patients manifesting multiple lesions of pityriasis rosea can be differentiated from those with tinea corporis as tinea corporis is generally more localized with fewer annular lesions that may have multiple concentric circles rather than one. Branching hyphae are seen on the KOH scraping in patients with tinea corporis. Hypopigmented or hyperpigmented macules characteristic of tinea versicolor  present on the neck and trunk may also be confused with pityriasis rosea. These lesions lack erythema and have a fine scale rather than the classic “collarette” of a scale. There is no “herald” patch present. KOH scrapings show Pityrosporum described as a “spaghetti and meatball” pattern.

    Cutaneous manifestations of secondary syphilis should be considered in sexually active patients. Pityriasis rosea spares the palms and soles while the red to brown macules of secondary syphilis do not.  Patients with syphilis do not report the appearance of a “herald” patch but may have noticed signs of a chancre.

    Guttate psoriasis is common amongst children regardless of previous psoriasis history. Patients do not recall the presence of a “herald” patch, but may report recent strep infection prior to the eruption. The lesions of guttate psoriasis also occur on the trunk as oval or round erythematous plaques which have a thick, adherent scale.

    References

    Goldstein, AO and Goldstein, BG. Pityriasis Rosea. In: UpToDate, Orfori, AO (Ed), Waltham,         MA. (Accessed on May, 11, 2016).

    Lawley LP, McCall CO, Lawley TJ. Eczema, Psoriasis, Cutaneous Infections, Acne, and Other Common Skin Disorders. In: Kasper D, Fauci A, Hauser S, Longo D, Jameson J,        Loscalzo J. eds. Harrison’s Principles of Internal Medicine, 19e. New York, NY:         McGraw-Hill; 2015.

    Usatine RP, Smith MA, Chumley HS, Mayeaux EJ, Jr.. Chapter 153. Pityriasis Rosea. In:    Usatine RP, Smith MA, Chumley HS, Mayeaux EJ, Jr.. eds. The Color Atlas of Family         Medicine, 2e. New York, NY: McGraw-Hill; 2013.

  87. Question 87 of 220
    87. Question

    A 55-year-old caucasian male presents to the clinic with an itchy rash on bilateral axillae that has been ongoing for several weeks. He has tried treating with moisturizers and “a fungal cream from the drug store”, with no improvement. The patient lives alone, is a 20-pack year smoker, and has 4-5 drinks per week. On physical exam, several shiny, erythematous, well-demarcated plaques without scaling are located in bilateral axillae. KOH prep and Wood’s Lamp tests are both negative. Which of the following is the most likely diagnosis?

    Correct

    The answer is D: inverse psoriasis. Inverse psoriasis is a variant of psoriasis that refers to a presentation involving the intertriginous areas, including the inguinal, perineal, genital, intergluteal, axillary, or inframammary regions. This presentation is called “inverse” since it is the reverse of the typical presentation on extensor surfaces. The risk factors of age, smoking, and drinking history, as well as the chronic nature of the rash, are all common in the history of patients with psoriasis.

    Seborrheic Dermatitis (A) is characterized by erythematous plaques with greasy-looking, yellowish scales. Although Seborrheic dermatitis can appear in intertriginous areas, it normally presents on areas that are abundant in sebaceous glands: such as the scalp, face and ears. Furthermore, the patient above presents without any scale.

    Tinea (B) is due to superficial fungal infections (dermatophyte) that can cause well demarcated erythematous plaques in all parts of the body – including the axillae and groin. However, the lack of response to the anti-fungal treatment the patient has been using, as well as the negative KOH prep makes this choice unlikely

    Erythrasma (C) is a superficial infection of the skin caused by Corynebacterium minutissimum. The infection typically presents as erythematous to brown patches or thin plaques in intertriginous areas. A Wood’s lamp examination depicting coral-red fluorescence confirms the diagnosis of erythrasma; therefore, a negative wood’s lamp test would rule out the diagnosis.

    According to the National Institutes of Health (NIH), psoriasis affects approximately 2.2% of the population in the United States. Worldwide prevalence of psoriasis in adults varies significantly, with rates occurring from 0.91 to 8.5 percent in adults, and 0 to 2.1 percent in children. Geographic location has a direct influence of the likelihood of having psoriasis, with prevalence having a positive correlation with increasing distance from the equator. The incidence of psoriasis is slightly more common in women then men, as well as a higher incidence in whites (2.5%) compared to African Americans is (1.3%). Although psoriasis can begin at any age, the disease is less common in children than adults. The age of onset occurs in two peaks: one between the ages of 30 and 39 years and another between the ages of 50 and 69 years. 1,2

    Etiology

    One of the primary mechanism of action is hyperproliferation of the keratinocytes in the epidermis, which in turn, increases the epidermal cell turnover rate. The cause of hyperproliferation of the keratinocytes is unknown but is highly attributed to genetic, immunologic, and external factors. In regards to genetic factors, several studies have identified multiple susceptibility loci for psoriasis, including those that involve immune system regulation. Approximately 40% of patients with psoriasis and/or psoriatic arthritis have a family history of these disorders. The major locus identified is PSORS1 on chromosome 6p21. HLA-Cw6 and HLA-B17 are the most important alleles identified within psoriasis, with HLA-Cw6 associated with early-onset disease and HLA-B17 linked to more severe phenotype. HLA associations explain only a portion of genetic susceptibility to psoriasis. Factors that are associated with psoriasis include smoking, obesity, medications, alcohol, psychogenic stress, infections, and the koebner phenomenon. Both former and current smokers are at higher risk for development of psoriasis, specifically palmoplantar pustulosis subtype. Patients with larger BMI may develop more severe psoriasis due to increased levels of cytokines. The most common medication “offenders” include beta-blockers, lithium, and antimalarial drugs. Alcohol has been linked to psoriasis due to numerous reports of alcohol consumption involved in both the development of psoriasis, as well as worsening of psoriasis. Stress is a well-established triggering factor in psoriasis; it has been associated with initial presentations of the disease as well as flares of pre-existing psoriasis. Infections, specifically streptococcal and HIV, are highly associated with the development of psoriasis. Finally, the Koebner phenomenon (the elicitation of psoriatic lesions by injury to the skin) is observed in approximately 25% of patients with psoriasis. 2,4

    Pathophysiology

    Psoriasis is a complex immune-mediated disorder that presents as inflammatory plaques in the skin. Dysregulation of components of the immune systems, keratinocyte function, and vascular structure contribute to the manifestations of this disease Plasmacytoid and myeloid dendritic cells are key contributors to development of inflammation in psoriasis. Specifically, Plasmacytoid dendritic cells stimulates the activation of myeloid dendritic cells via IFN-alpha. Consequently, Myeloid dendritic cells produce an array of inflammatory cytokines that influence T cell activity, as well cytokines IL-23 and IL-12. IL-23 and IL-12 promote the development of Th17 and Th1 cells, respectively. Th17 appears to play a major role in the pathogenesis of psoriasis via production of an array of other inflammatory cytokines (IL22, IL17, and TNF-alpha). These particular cytokines have been the target for the various biologics developed to treat psoriasis.

    Epidermal hyperplasia is another major component of psoriasis that is driven by cytokines (IL-20 and IL-22). The typical clinical findings of erythema and scaling are the result of hyperproliferation. The hyperproliferative state is characterized by an increased number of epidermal stem cells, increased number of cells undergoing DNA synthesis, shortened cell cycle time of keratinocytes (36 hours instead of the normal 311 hours of normal skin), and decreased turnover time of the epidermis.2,5

    Pertinent historical & Physical findings

    There are multiple clinical subtypes of psoriasis. The various variants of psoriasis include chronic plaque, guttate, inverse, pustular, and erythrodermic. Patients with chronic plaque type psoriasis usually present with symmetrically distributed cutaneous plaques. The scalp, extensor elbows, knees, and back are common sites for involvement. The plaques are erythematous with sharply defined margins and a thick silvery scale. In patients with dark skin, postinflammatory hyperpigmentation may be a characteristic feature. Positive koebner phenomenon and Auspitz’s sign may be present. Guttate psoriasis is characterized by the abrupt appearance of multiple small psoriatic papules and plaques, particularly on the trunk and proximal extremities. It typically occurs as an acute eruption in a child or young adult with no previous history of psoriasis. There is a strong association with a recent streptococcal infection.

    Erythrodermic psoriasis is an uncommon manifestation that may be acute or chronic. It is characterized by generalized erythema and scaling from head to toe, with increased risk of complications such as dehydration, infection, and sepsis. Pustular psoriasis presents with the acute onset of widespread erythema, scaling, and several superficial pustules. Presentation is commonly associated with pregnancy, infection, and the withdrawal of oral glucocorticoids. These variant is high risk for complications as well, such as malaise, fever, diarrhea, leukocytosis, and hypocalcemia. Inverse psoriasis refers to a presentation involving the intertriginous areas, including the inguinal, perineal, genital, intergluteal, axillary, or inframammary regions. Other manifestations of psoriasis include nail psoriasis and psoriatic arthritis, both of which will not be discussed within this review.1,2

    Diagnostic Correlation

    Psoriasis is a clinical diagnosis that is derived from a thorough physical examination and history. If necessary, other diagnostic procedures can be completed to help diagnose psoriasis. If a guttate variant is present, a throat culture may yield previous or current streptococcal pharyngitis. Other tests include KOH prep to rule out dermatophyte infection or a Wood’s Lamp evaluation to determine presence of corynebacteria bacteria. For definitive diagnosis, a punch biopsy of an active lesion or plaque would produce histological results that are consistent with psoriasis, which would include findings such as: hyperplasia of the epidermis, elongation of the dermal papillae, dilated blood vessels, hypogranulosis, and parakeratosis.4

    Management & Treatment

    For most patients, treatment entails identifying their psoriasis as either mild to moderate or moderate to severe. Most patients will be exposed to either topical and/or systemic therapy. Mild to moderate cases may start with topical treatment only, while those with moderate to severe cases may require systemic treatment with supplementation of topical treatment as well. For many, topical therapy may provide symptomatic relief and minimize required doses of systemic medications.

    In regards to treatment options for the psoriatic patient, the range of options includes topicals, light therapy, and systemic treatment (oral vs. injectable biologics). For topical therapy, all psoriatic patients should implement an emollient into their treatment to maintain proper skin hydration, prevent irritation, and help with itching. The most effective ointments are petroleum jelly or thick creams, especially when applied immediately after bathing. Tar has been a longstanding treatment that is commonly recommended due to its low side effect profile and anti-inflammatory effects. Topical steroids remain the mainstay of topical psoriasis treatment due to its antiinflammatory, antiproliferative, and immunosuppressive actions. Steroids range from low to high potency and are chosen dependent on site, plaque thickness, and patient compliance. The “typical” topical regimen includes using emollients 1-2x/day and corticosteroids 2x/day when psoriasis is active. Other topical alternatives include Vitamin D analogs, such as calcipotriene, and Calcineurin inhibitors such as Tacrolimus. The advantage of using these alternatives are to give patients who have chronic psoriasis a “vacation” from corticosteroid, or a topical option for those who do not opt to use corticosteroids in general.

    Light therapy has long been recognized as beneficial for psoriasis. UV radiation may act via antiproliferative effects (slowing keratinization) and anti-inflammatory effects (inducing apoptosis of pathogenic T-cells in psoriatic plaques). Consequently, many patients with a new psoriasis flare or those with widespread psoriasis frequently combine UVB light therapy with topical steroid use to clear their condition. For those with localized areas, Excimer laser offers a focused treatment.

    Oral and injectable biologic therapies are normally indicated for moderate to severe psoriasis. For oral treatment, options include methotrexate, apremilast, and acitretin. Methotrexate has the most years of clinical usage and is commonly considered one of the primary treatments for moderate to severe psoriasis. The medication is taken once weekly and is titrated up until the lowest effective dose is found. Monitoring includes CBC/LFTs to monitor hepatic and immune function. Folic acid is prescribed concomitantly to mitigate common side effects of MTX. Acitretin is normally given for patients of the pustular or erythrodermic variants; monitoring for hypertriglyceridemia and hepatotoxicity are required. Aprimelast, a PDE-4 inhibitor, is the newest oral medication for the treatment of psoriasis. The dose is commonly titrated due to side effects and is relatively expensive at this time. GI side effects are common, and there is no monitoring needed.

    For patients with severe or refractory psoriasis, biologics are normally indicated. The four most common injectable treatments are etanercept, adalimumab, ustekinumab and secukinumab. Etanercept and adalimumab are more widely studied and are considered the initial choice for patients starting treatment. Entanercept and adalimumab mechanism of action is focused on inhibiting TNF-alpha, while ustekinumas and secukinumab function through the inhibition of IL-22 and IL-17, respectively. Clinicians must check for TB via quantiferon gold or T-spot prior to initiating treatment, as well as monitoring for any infection and signs of lymphoma during treatment. Injection schedule includes the following: once weekly for etanercept, once every two weeks for adalimumab, once monthly for secukinumab and once every three months for ustekinumab.3,4

    References

    1) Emedicine.medscape.com. Psoriasis: epidemiology. 2016. Available at: http://emedicine.medscape.com/article/1943419-overview. Accessed April 18, 2016

    2) UpToDate.com. Epidemiology, clinical manifestations, and diagnosis of Psoriasis. 2016. Available at: http://www.uptodate.com/contents/epidemiology-clinical-manifestations-and-diagnosis-of-psoriasis?source=search_result&search=inverse+psoriasis&selectedTitle=1~11. Accessed April 18, 2016

    3) UpToDate.com. Treatment of Psoriasis. 2016. Available at: http://www.uptodate.com/contents/treatment-of-psoriasis?source=search_result&search=inverse+psoriasis&selectedTitle=2~11. Accessed April 18, 2016

    4) Bolognia JL, Jorizzo JL, Schaffer JV. Psoriasis. In: Dermatology. 3rd ed. New York, NY: Elsevier Saunders; 2012: 137-155.

    5) UpToDate.com. Pathophysiology of Psoriasis. 2016. Available at: http://www.uptodate.com/contents/pathophysiology-of-psoriasis?source=see_link#H481234. Accessed April 18, 2016

    Incorrect

    The answer is D: inverse psoriasis. Inverse psoriasis is a variant of psoriasis that refers to a presentation involving the intertriginous areas, including the inguinal, perineal, genital, intergluteal, axillary, or inframammary regions. This presentation is called “inverse” since it is the reverse of the typical presentation on extensor surfaces. The risk factors of age, smoking, and drinking history, as well as the chronic nature of the rash, are all common in the history of patients with psoriasis.

    Seborrheic Dermatitis (A) is characterized by erythematous plaques with greasy-looking, yellowish scales. Although Seborrheic dermatitis can appear in intertriginous areas, it normally presents on areas that are abundant in sebaceous glands: such as the scalp, face and ears. Furthermore, the patient above presents without any scale.

    Tinea (B) is due to superficial fungal infections (dermatophyte) that can cause well demarcated erythematous plaques in all parts of the body – including the axillae and groin. However, the lack of response to the anti-fungal treatment the patient has been using, as well as the negative KOH prep makes this choice unlikely

    Erythrasma (C) is a superficial infection of the skin caused by Corynebacterium minutissimum. The infection typically presents as erythematous to brown patches or thin plaques in intertriginous areas. A Wood’s lamp examination depicting coral-red fluorescence confirms the diagnosis of erythrasma; therefore, a negative wood’s lamp test would rule out the diagnosis.

    According to the National Institutes of Health (NIH), psoriasis affects approximately 2.2% of the population in the United States. Worldwide prevalence of psoriasis in adults varies significantly, with rates occurring from 0.91 to 8.5 percent in adults, and 0 to 2.1 percent in children. Geographic location has a direct influence of the likelihood of having psoriasis, with prevalence having a positive correlation with increasing distance from the equator. The incidence of psoriasis is slightly more common in women then men, as well as a higher incidence in whites (2.5%) compared to African Americans is (1.3%). Although psoriasis can begin at any age, the disease is less common in children than adults. The age of onset occurs in two peaks: one between the ages of 30 and 39 years and another between the ages of 50 and 69 years. 1,2

    Etiology

    One of the primary mechanism of action is hyperproliferation of the keratinocytes in the epidermis, which in turn, increases the epidermal cell turnover rate. The cause of hyperproliferation of the keratinocytes is unknown but is highly attributed to genetic, immunologic, and external factors. In regards to genetic factors, several studies have identified multiple susceptibility loci for psoriasis, including those that involve immune system regulation. Approximately 40% of patients with psoriasis and/or psoriatic arthritis have a family history of these disorders. The major locus identified is PSORS1 on chromosome 6p21. HLA-Cw6 and HLA-B17 are the most important alleles identified within psoriasis, with HLA-Cw6 associated with early-onset disease and HLA-B17 linked to more severe phenotype. HLA associations explain only a portion of genetic susceptibility to psoriasis. Factors that are associated with psoriasis include smoking, obesity, medications, alcohol, psychogenic stress, infections, and the koebner phenomenon. Both former and current smokers are at higher risk for development of psoriasis, specifically palmoplantar pustulosis subtype. Patients with larger BMI may develop more severe psoriasis due to increased levels of cytokines. The most common medication “offenders” include beta-blockers, lithium, and antimalarial drugs. Alcohol has been linked to psoriasis due to numerous reports of alcohol consumption involved in both the development of psoriasis, as well as worsening of psoriasis. Stress is a well-established triggering factor in psoriasis; it has been associated with initial presentations of the disease as well as flares of pre-existing psoriasis. Infections, specifically streptococcal and HIV, are highly associated with the development of psoriasis. Finally, the Koebner phenomenon (the elicitation of psoriatic lesions by injury to the skin) is observed in approximately 25% of patients with psoriasis. 2,4

    Pathophysiology

    Psoriasis is a complex immune-mediated disorder that presents as inflammatory plaques in the skin. Dysregulation of components of the immune systems, keratinocyte function, and vascular structure contribute to the manifestations of this disease Plasmacytoid and myeloid dendritic cells are key contributors to development of inflammation in psoriasis. Specifically, Plasmacytoid dendritic cells stimulates the activation of myeloid dendritic cells via IFN-alpha. Consequently, Myeloid dendritic cells produce an array of inflammatory cytokines that influence T cell activity, as well cytokines IL-23 and IL-12. IL-23 and IL-12 promote the development of Th17 and Th1 cells, respectively. Th17 appears to play a major role in the pathogenesis of psoriasis via production of an array of other inflammatory cytokines (IL22, IL17, and TNF-alpha). These particular cytokines have been the target for the various biologics developed to treat psoriasis.

    Epidermal hyperplasia is another major component of psoriasis that is driven by cytokines (IL-20 and IL-22). The typical clinical findings of erythema and scaling are the result of hyperproliferation. The hyperproliferative state is characterized by an increased number of epidermal stem cells, increased number of cells undergoing DNA synthesis, shortened cell cycle time of keratinocytes (36 hours instead of the normal 311 hours of normal skin), and decreased turnover time of the epidermis.2,5

    Pertinent historical & Physical findings

    There are multiple clinical subtypes of psoriasis. The various variants of psoriasis include chronic plaque, guttate, inverse, pustular, and erythrodermic. Patients with chronic plaque type psoriasis usually present with symmetrically distributed cutaneous plaques. The scalp, extensor elbows, knees, and back are common sites for involvement. The plaques are erythematous with sharply defined margins and a thick silvery scale. In patients with dark skin, postinflammatory hyperpigmentation may be a characteristic feature. Positive koebner phenomenon and Auspitz’s sign may be present. Guttate psoriasis is characterized by the abrupt appearance of multiple small psoriatic papules and plaques, particularly on the trunk and proximal extremities. It typically occurs as an acute eruption in a child or young adult with no previous history of psoriasis. There is a strong association with a recent streptococcal infection.

    Erythrodermic psoriasis is an uncommon manifestation that may be acute or chronic. It is characterized by generalized erythema and scaling from head to toe, with increased risk of complications such as dehydration, infection, and sepsis. Pustular psoriasis presents with the acute onset of widespread erythema, scaling, and several superficial pustules. Presentation is commonly associated with pregnancy, infection, and the withdrawal of oral glucocorticoids. These variant is high risk for complications as well, such as malaise, fever, diarrhea, leukocytosis, and hypocalcemia. Inverse psoriasis refers to a presentation involving the intertriginous areas, including the inguinal, perineal, genital, intergluteal, axillary, or inframammary regions. Other manifestations of psoriasis include nail psoriasis and psoriatic arthritis, both of which will not be discussed within this review.1,2

    Diagnostic Correlation

    Psoriasis is a clinical diagnosis that is derived from a thorough physical examination and history. If necessary, other diagnostic procedures can be completed to help diagnose psoriasis. If a guttate variant is present, a throat culture may yield previous or current streptococcal pharyngitis. Other tests include KOH prep to rule out dermatophyte infection or a Wood’s Lamp evaluation to determine presence of corynebacteria bacteria. For definitive diagnosis, a punch biopsy of an active lesion or plaque would produce histological results that are consistent with psoriasis, which would include findings such as: hyperplasia of the epidermis, elongation of the dermal papillae, dilated blood vessels, hypogranulosis, and parakeratosis.4

    Management & Treatment

    For most patients, treatment entails identifying their psoriasis as either mild to moderate or moderate to severe. Most patients will be exposed to either topical and/or systemic therapy. Mild to moderate cases may start with topical treatment only, while those with moderate to severe cases may require systemic treatment with supplementation of topical treatment as well. For many, topical therapy may provide symptomatic relief and minimize required doses of systemic medications.

    In regards to treatment options for the psoriatic patient, the range of options includes topicals, light therapy, and systemic treatment (oral vs. injectable biologics). For topical therapy, all psoriatic patients should implement an emollient into their treatment to maintain proper skin hydration, prevent irritation, and help with itching. The most effective ointments are petroleum jelly or thick creams, especially when applied immediately after bathing. Tar has been a longstanding treatment that is commonly recommended due to its low side effect profile and anti-inflammatory effects. Topical steroids remain the mainstay of topical psoriasis treatment due to its antiinflammatory, antiproliferative, and immunosuppressive actions. Steroids range from low to high potency and are chosen dependent on site, plaque thickness, and patient compliance. The “typical” topical regimen includes using emollients 1-2x/day and corticosteroids 2x/day when psoriasis is active. Other topical alternatives include Vitamin D analogs, such as calcipotriene, and Calcineurin inhibitors such as Tacrolimus. The advantage of using these alternatives are to give patients who have chronic psoriasis a “vacation” from corticosteroid, or a topical option for those who do not opt to use corticosteroids in general.

    Light therapy has long been recognized as beneficial for psoriasis. UV radiation may act via antiproliferative effects (slowing keratinization) and anti-inflammatory effects (inducing apoptosis of pathogenic T-cells in psoriatic plaques). Consequently, many patients with a new psoriasis flare or those with widespread psoriasis frequently combine UVB light therapy with topical steroid use to clear their condition. For those with localized areas, Excimer laser offers a focused treatment.

    Oral and injectable biologic therapies are normally indicated for moderate to severe psoriasis. For oral treatment, options include methotrexate, apremilast, and acitretin. Methotrexate has the most years of clinical usage and is commonly considered one of the primary treatments for moderate to severe psoriasis. The medication is taken once weekly and is titrated up until the lowest effective dose is found. Monitoring includes CBC/LFTs to monitor hepatic and immune function. Folic acid is prescribed concomitantly to mitigate common side effects of MTX. Acitretin is normally given for patients of the pustular or erythrodermic variants; monitoring for hypertriglyceridemia and hepatotoxicity are required. Aprimelast, a PDE-4 inhibitor, is the newest oral medication for the treatment of psoriasis. The dose is commonly titrated due to side effects and is relatively expensive at this time. GI side effects are common, and there is no monitoring needed.

    For patients with severe or refractory psoriasis, biologics are normally indicated. The four most common injectable treatments are etanercept, adalimumab, ustekinumab and secukinumab. Etanercept and adalimumab are more widely studied and are considered the initial choice for patients starting treatment. Entanercept and adalimumab mechanism of action is focused on inhibiting TNF-alpha, while ustekinumas and secukinumab function through the inhibition of IL-22 and IL-17, respectively. Clinicians must check for TB via quantiferon gold or T-spot prior to initiating treatment, as well as monitoring for any infection and signs of lymphoma during treatment. Injection schedule includes the following: once weekly for etanercept, once every two weeks for adalimumab, once monthly for secukinumab and once every three months for ustekinumab.3,4

    References

    1) Emedicine.medscape.com. Psoriasis: epidemiology. 2016. Available at: http://emedicine.medscape.com/article/1943419-overview. Accessed April 18, 2016

    2) UpToDate.com. Epidemiology, clinical manifestations, and diagnosis of Psoriasis. 2016. Available at: http://www.uptodate.com/contents/epidemiology-clinical-manifestations-and-diagnosis-of-psoriasis?source=search_result&search=inverse+psoriasis&selectedTitle=1~11. Accessed April 18, 2016

    3) UpToDate.com. Treatment of Psoriasis. 2016. Available at: http://www.uptodate.com/contents/treatment-of-psoriasis?source=search_result&search=inverse+psoriasis&selectedTitle=2~11. Accessed April 18, 2016

    4) Bolognia JL, Jorizzo JL, Schaffer JV. Psoriasis. In: Dermatology. 3rd ed. New York, NY: Elsevier Saunders; 2012: 137-155.

    5) UpToDate.com. Pathophysiology of Psoriasis. 2016. Available at: http://www.uptodate.com/contents/pathophysiology-of-psoriasis?source=see_link#H481234. Accessed April 18, 2016

  88. Question 88 of 220
    88. Question

    A 31-year-old female from Thailand presents to her PCP for some mild dyspnea on exertion. She denies SOB at rest, chest pain, palpitations, confusion, headaches or fatigue. Her labs return the following results: WBC 8.79, Hgb 7.4, Hct 26.2, MCV 52.9 (nl 80-96), Plt 252 (nl 150-450), Ferritin 775 (normal 11-307) , TIBC 215 (nl 240-250), Serum Iron 211 (normal 40-155), Fe Saturation 98%, TSH 3.02 and blood smear shows hypochromia, target cells, microcytosis and poikilocytosis. What is the most appropriate treatment for this patient?

    Correct

    The correct answer is (A) Chelation therapy. This patients Ferritin 775 (normal 11-307) and Serum Iron 211 (normal 40-155) so she needs chelation therapy to avoid build-up of iron in her liver, heart and endocrine glands. Iron becomes toxic to organs in large amounts so it is best to start chelation therapy as soon as the serum levels of ferritin and iron are out of range. Answer (D) No treatment is incorrect because of the high levels of serum ferritin and serum iron as well as the Liver Iron Index. This patient is clearly has anemia and is iron overloaded. Answer (B) PRBC transfusion is incorrect because this patient denies all other symptoms of anemia except for DOE. In addition, adding more RBCs to the system will cause an increase in system iron content, which this patient clearly doesn’t need. Answer (C) Iron supplementation is not correct because the patient’s system is already far too high in iron content. Any additional iron added to the system will cause more build up in the organs and cause further toxicity.

    Alpha thalassemias are commonly inherited and autosomal recessive in nature. There is no predilection for men or women, they are affected equally. Alpha thalassemias are most frequently seen in persons of Chinese or southeast Asian descent. In the U.S., these conditions have been considered somewhat uncommon. However, with the immigration of populations into the U.S. it is becoming more common and seen more often.

    There are 4 alpha globin genes in total, they are housed in pairs on each copy of chromosome 16. Each type of alpha thalassemia is determined by how many alpha globin genes are either deleted or contain a defect that causes them to make less alpha globin chain than they should. Gene deletion is the most common source of the alpha thalassemias. There are four types of alpha thalassemias that can be present: (1) Alpha Thalassemia Silent Carrier, (2) Alpha Thalassemia Trait, (3) Alpha Thalassemia Intermedia/Hemoglobin H Disease and (4) Thalassemia Major with Hemoglobin Bart’s, which causes the fatal condition called Fatal Hydrops Fetalis.

    The deletion of one alpha globin gene causes Alpha Thalassemia Silent Carrier (2). The silent carrier is asymptomatic and has no hematologic abnormalities (1) . Since there are no hematologic abnormalities there is no need for treatment or long-term monitoring. There is no change to the life expectancy for individuals that have this form of thalassemia.

    The deletion of two alpha globin genes, either homozygous or heterozygous, causes Alpha Thalassemia Trait (2). This condition does not exhibit anemia but they do exhibit microcytosis. The blood smear in this case will show hypochromia and microcytes as well as a few target cells and acanthocytes (1). Since people with thalassemia trait do not usually have iron deficiency anemia there is no need for treatment or long-term monitoring. Again, there is no change in life expectancy for this form of thalassemia.  Should someone will alpha thalassemia trait decide to have children genetic counseling should be considered since two partners that both carry some form of alpha thalassemia have a higher probability of having a child a more severe form of the condition.

    The deletion of three alpha globin genes causes Alpha Thalassemia Intermedia, which is also called Hemoglobin H Disease (2). In this case there is a shortage of alpha globin chains that allows for an over-abundance of beta globin chains. These beta globin chains form beta4 tetramers, or Hemoglobin H. These patients are not necessarily transfusion dependent. Often they only require sporadic transfusions for symptomatic anemia. Hemolysis due to viral infection or exposure to oxidative drugs can trigger an acute episode where transfusion may be needed. The decision to transfuse is generally based on the severity of their clinical condition, not just lab numbers. These patients will often have a baseline hemoglobin lower than the expected normal values. Patients with this condition will exhibit hematologic abnormalities such as microcytic anemia, splenomegaly and hemolysis. The blood smear will show microcytosis, hypochromia, target cells and poikilocytosis (1). As discussed with alpha thalassemia trait above, genetic counseling should be considered should child bearing be desired.

    The deletion of all four alpha globin genes results in Thalassemia Alpha with Hemoglobin Bart’s which will lead to fatal Hydrops Fetalis (2). It can be determined if a fetus has this type of thalassemia by use of chorionic villus sampling. Should the fetus have thalassemia alpha with hemoglobin Bart’s the risk of toxemia and post-partum bleeding is increased (2). Hemoglobin Bart’s has no available effective treatment (3).

    Iron overload is more often associated with beta thalassemia patients because they are more dependent on blood transfusions. However, it can occur with alpha thalassemias as well. In iron overload, excess iron in the body is deposited in visceral organs including the liver and heart. The patients will often have labs that show increased ferritin levels, low TIBC levels and high iron saturation levels as well as microcytic anemias. In the alpha thalassemias the iron overload is more commonly seen in the liver and can be triggered by events such as increased absorption of dietary iron due to anemia or hemolysis. In order to prevent toxicity to the visceral organs, patients with iron overload are usually treated with chelation therapy. There are two therapies that are currently available – Deferoxamine IV or subcutaneously or Exjade/Deferasirox, which has oral dosing (2). Chelation therapy is recommended if Liver Iron Concentration (LIC) > 5 mg/g or if ferritin levels are > 800 ng/L (4). In addition to chelation therapy, iron levels are checked every 6 months and monthly checks are done for CBC, CMP, Phos and Ferritin.

    References

    1. Papadakis, M., McPhee S. CURRENT Medical Diagnosis & Treatment 2015. New York: McGraw-Hill Education Medical; 2015.
    2. Munchie, M.D. H.L., Campbell, M.D. J.S. Alpha and Beta Thalassemias. American Family Physician. 2009; 80(4):339-344.
    3. Galanello, M.D. R., Cao M.D. A. Alpha-thalassemia. Genetics in Medicine. 2011; 13(2): 83-87.
    4. Ricchi P., Marsella M. Profile of Deferasirox for the Treatment of Patients with Non-Transfusion Dependent Thalassemia Syndromes. Drug Des Devel Ther. 2015; 16(9):6475-82.
    Incorrect

    The correct answer is (A) Chelation therapy. This patients Ferritin 775 (normal 11-307) and Serum Iron 211 (normal 40-155) so she needs chelation therapy to avoid build-up of iron in her liver, heart and endocrine glands. Iron becomes toxic to organs in large amounts so it is best to start chelation therapy as soon as the serum levels of ferritin and iron are out of range. Answer (D) No treatment is incorrect because of the high levels of serum ferritin and serum iron as well as the Liver Iron Index. This patient is clearly has anemia and is iron overloaded. Answer (B) PRBC transfusion is incorrect because this patient denies all other symptoms of anemia except for DOE. In addition, adding more RBCs to the system will cause an increase in system iron content, which this patient clearly doesn’t need. Answer (C) Iron supplementation is not correct because the patient’s system is already far too high in iron content. Any additional iron added to the system will cause more build up in the organs and cause further toxicity.

    Alpha thalassemias are commonly inherited and autosomal recessive in nature. There is no predilection for men or women, they are affected equally. Alpha thalassemias are most frequently seen in persons of Chinese or southeast Asian descent. In the U.S., these conditions have been considered somewhat uncommon. However, with the immigration of populations into the U.S. it is becoming more common and seen more often.

    There are 4 alpha globin genes in total, they are housed in pairs on each copy of chromosome 16. Each type of alpha thalassemia is determined by how many alpha globin genes are either deleted or contain a defect that causes them to make less alpha globin chain than they should. Gene deletion is the most common source of the alpha thalassemias. There are four types of alpha thalassemias that can be present: (1) Alpha Thalassemia Silent Carrier, (2) Alpha Thalassemia Trait, (3) Alpha Thalassemia Intermedia/Hemoglobin H Disease and (4) Thalassemia Major with Hemoglobin Bart’s, which causes the fatal condition called Fatal Hydrops Fetalis.

    The deletion of one alpha globin gene causes Alpha Thalassemia Silent Carrier (2). The silent carrier is asymptomatic and has no hematologic abnormalities (1) . Since there are no hematologic abnormalities there is no need for treatment or long-term monitoring. There is no change to the life expectancy for individuals that have this form of thalassemia.

    The deletion of two alpha globin genes, either homozygous or heterozygous, causes Alpha Thalassemia Trait (2). This condition does not exhibit anemia but they do exhibit microcytosis. The blood smear in this case will show hypochromia and microcytes as well as a few target cells and acanthocytes (1). Since people with thalassemia trait do not usually have iron deficiency anemia there is no need for treatment or long-term monitoring. Again, there is no change in life expectancy for this form of thalassemia.  Should someone will alpha thalassemia trait decide to have children genetic counseling should be considered since two partners that both carry some form of alpha thalassemia have a higher probability of having a child a more severe form of the condition.

    The deletion of three alpha globin genes causes Alpha Thalassemia Intermedia, which is also called Hemoglobin H Disease (2). In this case there is a shortage of alpha globin chains that allows for an over-abundance of beta globin chains. These beta globin chains form beta4 tetramers, or Hemoglobin H. These patients are not necessarily transfusion dependent. Often they only require sporadic transfusions for symptomatic anemia. Hemolysis due to viral infection or exposure to oxidative drugs can trigger an acute episode where transfusion may be needed. The decision to transfuse is generally based on the severity of their clinical condition, not just lab numbers. These patients will often have a baseline hemoglobin lower than the expected normal values. Patients with this condition will exhibit hematologic abnormalities such as microcytic anemia, splenomegaly and hemolysis. The blood smear will show microcytosis, hypochromia, target cells and poikilocytosis (1). As discussed with alpha thalassemia trait above, genetic counseling should be considered should child bearing be desired.

    The deletion of all four alpha globin genes results in Thalassemia Alpha with Hemoglobin Bart’s which will lead to fatal Hydrops Fetalis (2). It can be determined if a fetus has this type of thalassemia by use of chorionic villus sampling. Should the fetus have thalassemia alpha with hemoglobin Bart’s the risk of toxemia and post-partum bleeding is increased (2). Hemoglobin Bart’s has no available effective treatment (3).

    Iron overload is more often associated with beta thalassemia patients because they are more dependent on blood transfusions. However, it can occur with alpha thalassemias as well. In iron overload, excess iron in the body is deposited in visceral organs including the liver and heart. The patients will often have labs that show increased ferritin levels, low TIBC levels and high iron saturation levels as well as microcytic anemias. In the alpha thalassemias the iron overload is more commonly seen in the liver and can be triggered by events such as increased absorption of dietary iron due to anemia or hemolysis. In order to prevent toxicity to the visceral organs, patients with iron overload are usually treated with chelation therapy. There are two therapies that are currently available – Deferoxamine IV or subcutaneously or Exjade/Deferasirox, which has oral dosing (2). Chelation therapy is recommended if Liver Iron Concentration (LIC) > 5 mg/g or if ferritin levels are > 800 ng/L (4). In addition to chelation therapy, iron levels are checked every 6 months and monthly checks are done for CBC, CMP, Phos and Ferritin.

    References

    1. Papadakis, M., McPhee S. CURRENT Medical Diagnosis & Treatment 2015. New York: McGraw-Hill Education Medical; 2015.
    2. Munchie, M.D. H.L., Campbell, M.D. J.S. Alpha and Beta Thalassemias. American Family Physician. 2009; 80(4):339-344.
    3. Galanello, M.D. R., Cao M.D. A. Alpha-thalassemia. Genetics in Medicine. 2011; 13(2): 83-87.
    4. Ricchi P., Marsella M. Profile of Deferasirox for the Treatment of Patients with Non-Transfusion Dependent Thalassemia Syndromes. Drug Des Devel Ther. 2015; 16(9):6475-82.
  89. Question 89 of 220
    89. Question

    A 23 year-old female presents to your office with a chief complaint of exertional dyspnea. She admits to increased fatigue for the last 3-4 months, and has also recently experienced a syncopal episode. These complaints are all new to her. Past medical history is unremarkable, but she does have an older sister with “a hole in her heart.”  Which of the following physical exam findings would increase your suspicion for diagnosis of an ASD?

    Correct

    Answer, D. Wide, fixed-split S2, is pathognomonic for ASD.  The test of choice to confirm this diagnosis is an echocardiogram.Atrial Septal Defects are the second most common congenital structural heart condition in the US. ASD occurs in roughly 1 in 1000 live births and appears more frequently in female versus male gender. Primary etiology is congenital, and can arise as one of four variants: Ostium primum, Ostium Secundum (most common), sinus venosus and coronary sinus. After birth, the majority of the population will undergo a fusion of the septum primum and secundum giving rise to the septal wall between the left and right atria. Persisting communication between these two chambers resulting from maladhesion of the septae, is the pathophysiologic catalyst of atrial septal defects. Clinical presentation is dependent on the size and location of the defect in addition to presence of other comorbidities. Typically, symptoms of ASD present during the patients 20’s-30’s.

    The most pertinent finding with respect to history and physical, is the exclusive murmur (fixed, split S2). Symptoms that support this finding are right heart related in nature, most commonly dyspnea on exertion. Test of choice for ASD is echo. Typically, a transthoracic echo is suffice, and it is less invasive. While TEE are considered superior by some clinicians, it is typically reserved for cases in which diagnosis via TTE are uncertain. Echocardiography determines location, size, and additional defects of the heart if present.

    Treatment is based on patient symptoms (which suggest RV volume overload), overall heart function, and severity of the defect +/- complications. The main indications for surgical repair (closure) of an ASD are patients with clinically severe complications such as a paradoxical emboli, arrhythmias, significant shunting of blood between the atrial chambers, and RV dysfunction. It may be noted however, that if patients have progressed to shunt reversal, ASD closure is an absolute contraindication.

    Practice Question Recap: Pertinent findings from the history include: dyspnea on exertion, increased fatigue, and family history (mild familial patterns exist). Thus, choice a, can be eliminated as it represents patent ductus arteriosus. Most PDA patients are asymptomatic, and if symptoms to present they are typically associated with Left Heart pathology. All of the choices aside from letter D, can be eliminated with physical exam findings, specifically, auscultatory findings. Paradoxical split S2 represents severe Aortic Stenosis. The holosystolic murmur represents a Ventricular Septal Defect, which has a few similar presenting symptoms, however the primary symptoms are left heart related in nature with VSD, and in addition, these patients are at an elevated risk of presenting with Eisenmenger’s Syndrome.

    References

    Toy, E; Faulx, M. Case Files: Cardiology .  NewYork, New York. Mcgraw Hill, 2015

    Williams, D. PANCE PREP PEARLS  Lexington,KY; CreateSpace an Amazon company, 2016

    Connolly, H. MD; Indications for Closure and Medical Management of Atrial Septal Defects in Adults. Up to Date, Jan. 2015. Accessed June 3, 2016.

    Incorrect

    Answer, D. Wide, fixed-split S2, is pathognomonic for ASD.  The test of choice to confirm this diagnosis is an echocardiogram.Atrial Septal Defects are the second most common congenital structural heart condition in the US. ASD occurs in roughly 1 in 1000 live births and appears more frequently in female versus male gender. Primary etiology is congenital, and can arise as one of four variants: Ostium primum, Ostium Secundum (most common), sinus venosus and coronary sinus. After birth, the majority of the population will undergo a fusion of the septum primum and secundum giving rise to the septal wall between the left and right atria. Persisting communication between these two chambers resulting from maladhesion of the septae, is the pathophysiologic catalyst of atrial septal defects. Clinical presentation is dependent on the size and location of the defect in addition to presence of other comorbidities. Typically, symptoms of ASD present during the patients 20’s-30’s.

    The most pertinent finding with respect to history and physical, is the exclusive murmur (fixed, split S2). Symptoms that support this finding are right heart related in nature, most commonly dyspnea on exertion. Test of choice for ASD is echo. Typically, a transthoracic echo is suffice, and it is less invasive. While TEE are considered superior by some clinicians, it is typically reserved for cases in which diagnosis via TTE are uncertain. Echocardiography determines location, size, and additional defects of the heart if present.

    Treatment is based on patient symptoms (which suggest RV volume overload), overall heart function, and severity of the defect +/- complications. The main indications for surgical repair (closure) of an ASD are patients with clinically severe complications such as a paradoxical emboli, arrhythmias, significant shunting of blood between the atrial chambers, and RV dysfunction. It may be noted however, that if patients have progressed to shunt reversal, ASD closure is an absolute contraindication.

    Practice Question Recap: Pertinent findings from the history include: dyspnea on exertion, increased fatigue, and family history (mild familial patterns exist). Thus, choice a, can be eliminated as it represents patent ductus arteriosus. Most PDA patients are asymptomatic, and if symptoms to present they are typically associated with Left Heart pathology. All of the choices aside from letter D, can be eliminated with physical exam findings, specifically, auscultatory findings. Paradoxical split S2 represents severe Aortic Stenosis. The holosystolic murmur represents a Ventricular Septal Defect, which has a few similar presenting symptoms, however the primary symptoms are left heart related in nature with VSD, and in addition, these patients are at an elevated risk of presenting with Eisenmenger’s Syndrome.

    References

    Toy, E; Faulx, M. Case Files: Cardiology .  NewYork, New York. Mcgraw Hill, 2015

    Williams, D. PANCE PREP PEARLS  Lexington,KY; CreateSpace an Amazon company, 2016

    Connolly, H. MD; Indications for Closure and Medical Management of Atrial Septal Defects in Adults. Up to Date, Jan. 2015. Accessed June 3, 2016.

  90. Question 90 of 220
    90. Question

    A 57 year-old female patient with a history of controlled hypertension and vertigo complains of dizziness with movement, “heavy-headedness”, and fatigue. Her usual dose of Meclizine provided no relief. She has been taking HCTZ and Atenolol without any changes in dosage in the past 2 years. On exam, she shows a positive Romberg, positive Dix-Hallpike, and 4/5 UE motor strength, while the remainder of her neurological exam is unremarkable. How would you treat her vertigo?

    Correct

    Correct answer: D. 1-2 sessions of in-office Canalith repositioning. Canalith repositioning (CRP) is the treatment of choice for confirmed BPPV by positive Dix-Hallpike. The most common CRP is the Epley maneuver which general principle is to return the particles in the semi-circular canal back to its original place so that the vestibular system is no longer out of balance.

    Incorrect answers:

    1. Decrease dosage of HCTZ. The patient has not had a change in dosage in this medication for the past 2 years. Any cardiogenic vertigo would have presented earlier and closer to the time of a dosage change.
    2. Increase dosage of Meclizine. It is possible that a higher dose may treat her vertigo however her normal dose provided no relief when it has in the past. There is a better treatment option that provides immediate relief without an increased risk of side effects.
    3. Immediate ENT surgical consult. The surgical route is reserved for patients whose vertigo is unresolved with CRP. Surgery is not recommended until all other options are exhausted because it is invasive and poses a risk of nerve damage.

    Epidemiology: BPPV is prevalent among every 64 per 100,000 people. Gender distribution is slightly higher towards females (64%). BBPV occurs in an older population with an average age of 51-57 years of age. It is a rare incident in patients younger than 35 years without a history of prior head trauma. There is insignificant data correlating racial affinity. Some predisposing factors for BPPV include inactivity, acute alcoholism, major surgery, and central nervous system disease. Many patients may have concurrent ear pathology including: idiopathic, trauma, ear diseases, otitis media, vestibular neuritis, Meniere disease, otosclerosis, sudden sensorineural hearing loss, vestibular basilar insuffiency, acoustic neuroma, and cervical vertigo.

    Etiology: BPPV is an abnormal sensation that is evoked by provocative positions that usually trigger certain eye movements like nystagmus. The cause of BPPV is categorized into 2 groups: canalithiasis and cupulolithiasis. Canalithiasis or canal rocks occur when particles are present in the canal of the semi-circular canals. These rock particles are free floating and mobile leading to vertigo by exertion of force. Cupulolithiasis, or cupula rocks, are affixed to the cupula of the crista ampullaris. In contrast, these densities are non-mobile present in the ampulla of the semi-circular canals.

    Pathophysiology: In the cupulolithiasis explanation, basophilic particles are attached to the cupula rendering the posterior semicircular canal (PSC) overly sensitive to gravity. The abnormally attached structures can impinge on the cupula. The additional weight of the cupula rocks makes the pole unstable and more difficult to maintain a neutral position. The unstable cupula is prone to stay in one direction depending on the tilt of the head. Once a position is made, the weight of the cupula rocks keeps the cupula from recoiling back to a neutral position. This results in persistent nystagmus and dizziness, especially when a patient is in a specific position, ie tilted backwards.

    In the canalithiasis explanation, symptoms of BPPV are related to the mobile canaliths in the posterior semi-circular canal. When the head is upright, the densities are at the most gravity-dependent position. When the head is tilted back supine, the particles circumduct approximately 90 degrees up along the arc of the PSC. After a transient lag, the particles move back down the arc by gravitational force. The downward movement of the particles causes the endolymph to flow away from the ampulla and the cupula to be deflected. The bending of the cupula results in nystagmus. When the head returns to an upright position, the rotation and cupular deflection is reversed, thus dizziness with nystagmus flickering in the opposite direction occurs. The canalithiasis explanation is analogous to pebbles inside a tire. As the tire rotates, the pebbles are picked up temporarily then fall back down with gravity. The tumbling results in dizziness. Reversal of the tire rotation makes the flow and dizziness go in opposite direction. Canalithiasis explains the delay (latency), transient nystagmus and reversal on return to upright.

    Pertinent historical and physical findings: The onset of BPPV is sudden. Most patients will awake with these symptoms, often noticing vertigo while trying to sit up suddenly. Positional vertigo is variable from days and weeks to even months or years. Symptoms may resolve then recur. Severity is also variable. In extreme instances, the faintest head movement will elicit nausea and vomiting. There can be strong nystagmus, while others will be unfazed. Sensation of dizziness is not constant. Head movement triggers severe dizziness. In between episodes, patients will have few or no symptoms. Some patients may complain of a foggy or cloudy sensation.

    Classic BPPV symptoms are triggered by suddenly moving from an erect position to supine position while the head is angled 45 degrees towards the side of the affected ear. After reaching the provocative position, a delay of a few seconds occurs before the dizzy spell strikes. The patient feels like they are suddenly thrown into a rolling spin toward the side of the affected ear. Symptoms begin very violently and usually resolve within 20 to 30 seconds. The sensation is triggered once again when returning to sitting erect, but the nystagmus is in the opposite direction. Generally physical findings are unremarkable. Neurological exam findings are normal except those from the Dix-Hallpike maneuver. However, abnormal neurological exam findings do not rule out a BPPV diagnosis.

    Diagnostic correlations (labs/imaging): The standard clinical test for BPPV is the Dix-Hallpike maneuver. A patient with classic rotatory nystagmus with latency and limited duration is also considered diagnostic for BPPV. In the Dix-Hallpike maneuver, the patient is moved rapidly from a sitting position to a supine position with the head 45 degrees to one side. In approximately 20 to 30 seconds, the patient is then returned to the sitting position. If there is an absence of nystagmus, the maneuver is performed with the head turned to the opposite side.

    Management/treatment: Medical care includes watchful waiting, vestibulosuppressant medication, vestibular rehabilitation, canalith repositioning, and surgery. BPPV can resolve with watchful watching, however it may take weeks to months for symptoms-leaving the patient in discomfort and vertigo until resolution. In addition, patients may be at risk for a fall or other events from a vertigo episode. Vestibulosuppressant medication does not control vertigo but it may provide minimal relief of symptoms. Patients may feel groggy or sleepy with this medication. Vestibular rehabilitation is noninvasive therapy that can be successful only after a considerable period of time. While the therapeutic maneuvers are performed, vertigo is repeatedly stimulated.

    Canalith repositioning procedure (CRP) is a first choice treatment option due to its high benefit-to-risk ratio and its simple noninvasive treatment to cure BPPV in 1-2 sessions. There are 2 methods in canalith repositioning therapy: Epley maneuver and Semont maneuver. Both options involve head repositioning to rearrange the displaced particles, however the Epley is gentler.

    In the Epley maneuver the patient begins in a sitting position with the head turned 45 degrees towards the affected side. In position 1, the patient slowly reclines to the supine position. The rate of recline is so that no nystagmus is elicited- usually about 30 seconds. In position 2, the patient continues to recline supine until 15 degrees Trendelenburg is reached and head turned 45 degrees toward affected side. It takes about 10 seconds to recline with another 20 seconds in Trendelenburg position. In position 3, the body is still in supine 15 degrees Trendelenburg with the head turned 45 degrees towards the contralateral side. In position 4, the body is turned from position 3 so that the shoulders are perpendicular to the floor with the affected ear facing upwards. The head is turned farther so that the nose extends 45 degrees below the horizon. This takes 40 seconds. In position 5, the patient is raised back to sitting with the head still turned away from affected side (at least 90-135 degrees toward contralateral side). In position 6, the head is returned to midline.

    After the completion of the Epley maneuver, a Dix-Hallpike test is performed immediately. If nystagmus is observed, the Epley maneuver is repeated. After CRP treatment, patients should avoid lying down completely flat for 24-48 hours. They should sleep with the head elevated. They are also advised to avoid aggravating activities such as jumping or flipping. Patients may also want to adjust their sleeping position to prevent recurrence.

    If CRP fails, surgical options are considered. They are invasive and hold risks of complications (hearing loss and facial nerve damage). Surgical options for vertigo control include: labyrinthectomy, posterior canal occlusion, singular neurectomy, vestibular nerve resection, and transtympanic aminoglycoside application. Labyrinthectomy and vestibular nerve resection are only considered in extreme cases. Singular neurectomy is difficult and has been mastered by only a select number of surgeons. The most practical surgical option is posterior canal occlusion. The posterior canal is collapsed and therefore the particles are immobilized in the canal. This procedure takes the mastoidectomy approach- the hard bone is drilled through with diamond burrs to expose the labyrinth without spilled excessive perilymphatic fluid. The labyrinth is compressed so that the flow of endolymphatic fluid and movement of particles are disrupted.

    References

    Anagnostou E, Kouzi I, Spengos K. Diagnosis and Treatment of Anterior-Canal Benign Paroxysmal Positional Vertigo: A Systematic Review. J Clin Neurol. 2015;11(3):262-7. http://reference.medscape.com/medline/abstract/26022461. Accessed             May 26th, 2016.

    Huppert D, Strupp M, Mückter H, Brandt T. Which medication do I need to manage dizzy patients?. Acta Oto-laryngologica. 2011;131(3):228-41.  https://www.ncbi.nlm.nih.gov/pubmed/21142898. Accessed May 26th, 2016.

    Incorrect

    Correct answer: D. 1-2 sessions of in-office Canalith repositioning. Canalith repositioning (CRP) is the treatment of choice for confirmed BPPV by positive Dix-Hallpike. The most common CRP is the Epley maneuver which general principle is to return the particles in the semi-circular canal back to its original place so that the vestibular system is no longer out of balance.

    Incorrect answers:

    1. Decrease dosage of HCTZ. The patient has not had a change in dosage in this medication for the past 2 years. Any cardiogenic vertigo would have presented earlier and closer to the time of a dosage change.
    2. Increase dosage of Meclizine. It is possible that a higher dose may treat her vertigo however her normal dose provided no relief when it has in the past. There is a better treatment option that provides immediate relief without an increased risk of side effects.
    3. Immediate ENT surgical consult. The surgical route is reserved for patients whose vertigo is unresolved with CRP. Surgery is not recommended until all other options are exhausted because it is invasive and poses a risk of nerve damage.

    Epidemiology: BPPV is prevalent among every 64 per 100,000 people. Gender distribution is slightly higher towards females (64%). BBPV occurs in an older population with an average age of 51-57 years of age. It is a rare incident in patients younger than 35 years without a history of prior head trauma. There is insignificant data correlating racial affinity. Some predisposing factors for BPPV include inactivity, acute alcoholism, major surgery, and central nervous system disease. Many patients may have concurrent ear pathology including: idiopathic, trauma, ear diseases, otitis media, vestibular neuritis, Meniere disease, otosclerosis, sudden sensorineural hearing loss, vestibular basilar insuffiency, acoustic neuroma, and cervical vertigo.

    Etiology: BPPV is an abnormal sensation that is evoked by provocative positions that usually trigger certain eye movements like nystagmus. The cause of BPPV is categorized into 2 groups: canalithiasis and cupulolithiasis. Canalithiasis or canal rocks occur when particles are present in the canal of the semi-circular canals. These rock particles are free floating and mobile leading to vertigo by exertion of force. Cupulolithiasis, or cupula rocks, are affixed to the cupula of the crista ampullaris. In contrast, these densities are non-mobile present in the ampulla of the semi-circular canals.

    Pathophysiology: In the cupulolithiasis explanation, basophilic particles are attached to the cupula rendering the posterior semicircular canal (PSC) overly sensitive to gravity. The abnormally attached structures can impinge on the cupula. The additional weight of the cupula rocks makes the pole unstable and more difficult to maintain a neutral position. The unstable cupula is prone to stay in one direction depending on the tilt of the head. Once a position is made, the weight of the cupula rocks keeps the cupula from recoiling back to a neutral position. This results in persistent nystagmus and dizziness, especially when a patient is in a specific position, ie tilted backwards.

    In the canalithiasis explanation, symptoms of BPPV are related to the mobile canaliths in the posterior semi-circular canal. When the head is upright, the densities are at the most gravity-dependent position. When the head is tilted back supine, the particles circumduct approximately 90 degrees up along the arc of the PSC. After a transient lag, the particles move back down the arc by gravitational force. The downward movement of the particles causes the endolymph to flow away from the ampulla and the cupula to be deflected. The bending of the cupula results in nystagmus. When the head returns to an upright position, the rotation and cupular deflection is reversed, thus dizziness with nystagmus flickering in the opposite direction occurs. The canalithiasis explanation is analogous to pebbles inside a tire. As the tire rotates, the pebbles are picked up temporarily then fall back down with gravity. The tumbling results in dizziness. Reversal of the tire rotation makes the flow and dizziness go in opposite direction. Canalithiasis explains the delay (latency), transient nystagmus and reversal on return to upright.

    Pertinent historical and physical findings: The onset of BPPV is sudden. Most patients will awake with these symptoms, often noticing vertigo while trying to sit up suddenly. Positional vertigo is variable from days and weeks to even months or years. Symptoms may resolve then recur. Severity is also variable. In extreme instances, the faintest head movement will elicit nausea and vomiting. There can be strong nystagmus, while others will be unfazed. Sensation of dizziness is not constant. Head movement triggers severe dizziness. In between episodes, patients will have few or no symptoms. Some patients may complain of a foggy or cloudy sensation.

    Classic BPPV symptoms are triggered by suddenly moving from an erect position to supine position while the head is angled 45 degrees towards the side of the affected ear. After reaching the provocative position, a delay of a few seconds occurs before the dizzy spell strikes. The patient feels like they are suddenly thrown into a rolling spin toward the side of the affected ear. Symptoms begin very violently and usually resolve within 20 to 30 seconds. The sensation is triggered once again when returning to sitting erect, but the nystagmus is in the opposite direction. Generally physical findings are unremarkable. Neurological exam findings are normal except those from the Dix-Hallpike maneuver. However, abnormal neurological exam findings do not rule out a BPPV diagnosis.

    Diagnostic correlations (labs/imaging): The standard clinical test for BPPV is the Dix-Hallpike maneuver. A patient with classic rotatory nystagmus with latency and limited duration is also considered diagnostic for BPPV. In the Dix-Hallpike maneuver, the patient is moved rapidly from a sitting position to a supine position with the head 45 degrees to one side. In approximately 20 to 30 seconds, the patient is then returned to the sitting position. If there is an absence of nystagmus, the maneuver is performed with the head turned to the opposite side.

    Management/treatment: Medical care includes watchful waiting, vestibulosuppressant medication, vestibular rehabilitation, canalith repositioning, and surgery. BPPV can resolve with watchful watching, however it may take weeks to months for symptoms-leaving the patient in discomfort and vertigo until resolution. In addition, patients may be at risk for a fall or other events from a vertigo episode. Vestibulosuppressant medication does not control vertigo but it may provide minimal relief of symptoms. Patients may feel groggy or sleepy with this medication. Vestibular rehabilitation is noninvasive therapy that can be successful only after a considerable period of time. While the therapeutic maneuvers are performed, vertigo is repeatedly stimulated.

    Canalith repositioning procedure (CRP) is a first choice treatment option due to its high benefit-to-risk ratio and its simple noninvasive treatment to cure BPPV in 1-2 sessions. There are 2 methods in canalith repositioning therapy: Epley maneuver and Semont maneuver. Both options involve head repositioning to rearrange the displaced particles, however the Epley is gentler.

    In the Epley maneuver the patient begins in a sitting position with the head turned 45 degrees towards the affected side. In position 1, the patient slowly reclines to the supine position. The rate of recline is so that no nystagmus is elicited- usually about 30 seconds. In position 2, the patient continues to recline supine until 15 degrees Trendelenburg is reached and head turned 45 degrees toward affected side. It takes about 10 seconds to recline with another 20 seconds in Trendelenburg position. In position 3, the body is still in supine 15 degrees Trendelenburg with the head turned 45 degrees towards the contralateral side. In position 4, the body is turned from position 3 so that the shoulders are perpendicular to the floor with the affected ear facing upwards. The head is turned farther so that the nose extends 45 degrees below the horizon. This takes 40 seconds. In position 5, the patient is raised back to sitting with the head still turned away from affected side (at least 90-135 degrees toward contralateral side). In position 6, the head is returned to midline.

    After the completion of the Epley maneuver, a Dix-Hallpike test is performed immediately. If nystagmus is observed, the Epley maneuver is repeated. After CRP treatment, patients should avoid lying down completely flat for 24-48 hours. They should sleep with the head elevated. They are also advised to avoid aggravating activities such as jumping or flipping. Patients may also want to adjust their sleeping position to prevent recurrence.

    If CRP fails, surgical options are considered. They are invasive and hold risks of complications (hearing loss and facial nerve damage). Surgical options for vertigo control include: labyrinthectomy, posterior canal occlusion, singular neurectomy, vestibular nerve resection, and transtympanic aminoglycoside application. Labyrinthectomy and vestibular nerve resection are only considered in extreme cases. Singular neurectomy is difficult and has been mastered by only a select number of surgeons. The most practical surgical option is posterior canal occlusion. The posterior canal is collapsed and therefore the particles are immobilized in the canal. This procedure takes the mastoidectomy approach- the hard bone is drilled through with diamond burrs to expose the labyrinth without spilled excessive perilymphatic fluid. The labyrinth is compressed so that the flow of endolymphatic fluid and movement of particles are disrupted.

    References

    Anagnostou E, Kouzi I, Spengos K. Diagnosis and Treatment of Anterior-Canal Benign Paroxysmal Positional Vertigo: A Systematic Review. J Clin Neurol. 2015;11(3):262-7. http://reference.medscape.com/medline/abstract/26022461. Accessed             May 26th, 2016.

    Huppert D, Strupp M, Mückter H, Brandt T. Which medication do I need to manage dizzy patients?. Acta Oto-laryngologica. 2011;131(3):228-41.  https://www.ncbi.nlm.nih.gov/pubmed/21142898. Accessed May 26th, 2016.

  91. Question 91 of 220
    91. Question

    A 50 year old male, presents to your office complaining of a “rough, red spot” on his left cheek. He notes it has been there for several weeks and he finally decided to be evaluated after his wife noticed that he was constantly picking at his face. He is a fair-skinned individual and after taking his history, you learn that he has worked as an exterior house painter for the last 25 years and he rarely uses sunscreen. On physical exam, you note a 6mm erythematous scaly patch on the left maxilla and no other lesions on the face are noted. What are you most concerned about the lesion on this patients face?

    Correct

    Correct Answer: B. The patient has actinic keratosis which is often considered to be an early form of squamous cell carcinoma. The chief complaint of a “rough, red spot” on a fair-skinned male of an advanced age who works outside and rarely uses sunscreen protection are all classic for a patient presenting with AKs.2 With option A, it is unlikely JM already has invasive skin cancer based on the presentation (non-tender) and squamous cell carcinoma has a low mortality rate. However, a biopsy should be performed. Option C is inappropriate has all AKs should be treated due to their ability to evolve into SCC. With option D, rosacea can appear erythematous and scaly on the cheeks but should be disregarded as it is not normally one solitary lesion and patient’s history does not support the diagnosis.

    Actinic keratosis (AK) (sometimes referred to as solar keratosis) is a lesion of the skin that presents due to excess growth of atypical epidermal keratinocytes and can sometimes develop into squamous cell carcinoma. AKs are one of the most common reasons why a patient would present to a dermatologist. Fourteen percent of dermatology office visit in the United States between 1990 and 1999 were due to AKs. People with lighter skin are more likely to develop the lesions.1 The prevalence in Australia is believed to be between 40-60% of individuals older than 40.2 Other risk factors include ultraviolet radiation (causes mutations in the p53 tumor suppressor gene), extensive sun exposure (particularly individuals who work outdoors), a history of more than six painful sunburns in a lifetime, male sex and an increase in age. The immunosuppressed are at in increased risk and HPV may also play a role but further studies are still needed.1

    Histologically, AKs share many similarities with squamous cell carcinoma. AKs are lesions of the skin characterized by collections of atypical keratinocytes (keratin-producing cells) at the skin’s basal layer which ascends to include the granular and cornified layers of the skin. The epidermis will begin to show abnormal structures, such as acanthosis (thickening of the skin), parakeratosis (nuclei remain in the stratum corneum and causes thinning of the granular layer) and dyskeratosis (abnormal keratinization). The cells appear atypical and keratinocytes will be various sizes and shape. Often, an inflammatory component is present. It is argued that AKs are early signs of squamous cell carcinoma but others believe calling them “pre-cancerous” unnecessarily alarms patients.2

    Patients who present with AKs are almost always white and sun-sensitive. The incident of the lesions increases with age and men have a slight increased risk. Individuals often have had extensive UV exposure, particular those with occupations that require them to work outdoors. The lesions will often be located on the patient’s body where there has been long-term sun exposure such as the face, ears, scalp (in bald men), forearms and hands. They will often start as small spots that feel rough (sandpaper texture) and are often more easily felt than visualized. Over time, the lesions will increase in size and develop into a red and scaly lesion (usually 3-10mm but they can be larger).2

    In order to confirm that the lesion is an AK, a skin biopsy is advised in order to rule out squamous cell carcinoma or another advanced skin lesion. A biopsy should also be performed if the lesion is recurring or it does not respond to treatment. Under the microscope, AKs will show dysplasia (proliferation of abnormal cells) and the cell architecture will appear disordered. The keratinocytes will be abnormal in both size and shape. These abnormalities will often ascend into a thinning granular layer. A Wood lamp may also be used as the lesions will emit a pink fluorescence.2

    AKs that are untreated may stay the same, they may spontaneously resolve or they can progressive to squamous cell carcinoma. There is no way to determine their course. The chance of progression to SCC is relatively low (about 10%) but if the patient has multiple lesions, the chance increases. In addition, AKs are often difficult to differentiate from other skin malignancies and thus all lesions should be treated. Treatment is determined based on the how many lesions are present and the effectiveness of the treatment. Both medical and surgical treatments are available. With medical treatment, the patient should be educated to limit sun exposure and use sunscreen and protective clothing at all times. With medical therapy, large areas are able to be treated. However, the treatment course is long and involves irritating and uncomfortable medications. The most common topical therapy is 5-FU which acts by inhibiting the enzyme thymidylate synthetase and leads to cell destruction of actively proliferating cells. During treatment, the lesions will become red and uncomfortable. If the treatment is successfully completed, though, the patient’s complexion will become smooth and there will be substantial improvement of the AKs.2 Another medical option is photodynamic therapy (PDT). A light-sensitizing compound builds up in AK cells and is then activated by a specific wavelength of light. It is then converted by enzymes and oxygen free radicals are produced which leads to cell death. The therapy a good cosmetic outcome, however, a major drawback is that the technique is very painful.2,4

    Surgical therapy involves total destruction of the AK but limiting damage to the healthy skin surrounding the lesion. If the diagnosis is uncertain, a biopsy should be performed, though it will usually leave a scar. Cryosurgery is used which utilizes cryogen to decrease the temperature of the skin and cause cell death. Liquid nitrogen is most commonly used.  The low temperature destroys keratinocytes but other structures surrounding it are spared as they are more resistant to the cold. Other surgical options include cosmetic resurfacing, which includes deep chemical peels and dermabrasion. However, insurances do not normally cover the cost of these procedures.2

    All patients should be educated to use sunscreen with an SPF of at least 30 and to avoid sun exposure as much as possible. They should follow-up regularly with their dermatologist, especially if any new lesions appear. If the lesion does not resolve, a biopsy may need to be performed at a later date if not already done prior to treatment.2

    References:

    1. Padilla, Steven. Epidemiology, natural history, and diagnosis of actinic keratosis. UpToDate. 2015. https://www.uptodate.com/contents/epidemiology-natural-history-and-diagnosis-of-actinic-keratosis?source=search_result&search=actinic+keratosis&selectedTitle=2%7E79. Accessed May 21, 2016.
    2. Spencer, J. Actinic Keratosis. Medscape. 2016. http://emedicine.medscape.com/article/1099775-overview. Accessed May 21 2016.
    3. American Academy of Dermatology. Actinic Keratosis. https://www.aad.org/public/diseases/scaly-skin/actinic-keratosis. Accessed May 21 2016.
    4. Kessels, JP. Laser-mediated Photodynamic Therapy: An Alternative Treatment for Actinic Keratosis? Acts Derm Venereol. 2016. https://www.ncbi.nlm.nih.gov/pubmed/26551377. Accessed May 21 2016.
    Incorrect

    Correct Answer: B. The patient has actinic keratosis which is often considered to be an early form of squamous cell carcinoma. The chief complaint of a “rough, red spot” on a fair-skinned male of an advanced age who works outside and rarely uses sunscreen protection are all classic for a patient presenting with AKs.2 With option A, it is unlikely JM already has invasive skin cancer based on the presentation (non-tender) and squamous cell carcinoma has a low mortality rate. However, a biopsy should be performed. Option C is inappropriate has all AKs should be treated due to their ability to evolve into SCC. With option D, rosacea can appear erythematous and scaly on the cheeks but should be disregarded as it is not normally one solitary lesion and patient’s history does not support the diagnosis.

    Actinic keratosis (AK) (sometimes referred to as solar keratosis) is a lesion of the skin that presents due to excess growth of atypical epidermal keratinocytes and can sometimes develop into squamous cell carcinoma. AKs are one of the most common reasons why a patient would present to a dermatologist. Fourteen percent of dermatology office visit in the United States between 1990 and 1999 were due to AKs. People with lighter skin are more likely to develop the lesions.1 The prevalence in Australia is believed to be between 40-60% of individuals older than 40.2 Other risk factors include ultraviolet radiation (causes mutations in the p53 tumor suppressor gene), extensive sun exposure (particularly individuals who work outdoors), a history of more than six painful sunburns in a lifetime, male sex and an increase in age. The immunosuppressed are at in increased risk and HPV may also play a role but further studies are still needed.1

    Histologically, AKs share many similarities with squamous cell carcinoma. AKs are lesions of the skin characterized by collections of atypical keratinocytes (keratin-producing cells) at the skin’s basal layer which ascends to include the granular and cornified layers of the skin. The epidermis will begin to show abnormal structures, such as acanthosis (thickening of the skin), parakeratosis (nuclei remain in the stratum corneum and causes thinning of the granular layer) and dyskeratosis (abnormal keratinization). The cells appear atypical and keratinocytes will be various sizes and shape. Often, an inflammatory component is present. It is argued that AKs are early signs of squamous cell carcinoma but others believe calling them “pre-cancerous” unnecessarily alarms patients.2

    Patients who present with AKs are almost always white and sun-sensitive. The incident of the lesions increases with age and men have a slight increased risk. Individuals often have had extensive UV exposure, particular those with occupations that require them to work outdoors. The lesions will often be located on the patient’s body where there has been long-term sun exposure such as the face, ears, scalp (in bald men), forearms and hands. They will often start as small spots that feel rough (sandpaper texture) and are often more easily felt than visualized. Over time, the lesions will increase in size and develop into a red and scaly lesion (usually 3-10mm but they can be larger).2

    In order to confirm that the lesion is an AK, a skin biopsy is advised in order to rule out squamous cell carcinoma or another advanced skin lesion. A biopsy should also be performed if the lesion is recurring or it does not respond to treatment. Under the microscope, AKs will show dysplasia (proliferation of abnormal cells) and the cell architecture will appear disordered. The keratinocytes will be abnormal in both size and shape. These abnormalities will often ascend into a thinning granular layer. A Wood lamp may also be used as the lesions will emit a pink fluorescence.2

    AKs that are untreated may stay the same, they may spontaneously resolve or they can progressive to squamous cell carcinoma. There is no way to determine their course. The chance of progression to SCC is relatively low (about 10%) but if the patient has multiple lesions, the chance increases. In addition, AKs are often difficult to differentiate from other skin malignancies and thus all lesions should be treated. Treatment is determined based on the how many lesions are present and the effectiveness of the treatment. Both medical and surgical treatments are available. With medical treatment, the patient should be educated to limit sun exposure and use sunscreen and protective clothing at all times. With medical therapy, large areas are able to be treated. However, the treatment course is long and involves irritating and uncomfortable medications. The most common topical therapy is 5-FU which acts by inhibiting the enzyme thymidylate synthetase and leads to cell destruction of actively proliferating cells. During treatment, the lesions will become red and uncomfortable. If the treatment is successfully completed, though, the patient’s complexion will become smooth and there will be substantial improvement of the AKs.2 Another medical option is photodynamic therapy (PDT). A light-sensitizing compound builds up in AK cells and is then activated by a specific wavelength of light. It is then converted by enzymes and oxygen free radicals are produced which leads to cell death. The therapy a good cosmetic outcome, however, a major drawback is that the technique is very painful.2,4

    Surgical therapy involves total destruction of the AK but limiting damage to the healthy skin surrounding the lesion. If the diagnosis is uncertain, a biopsy should be performed, though it will usually leave a scar. Cryosurgery is used which utilizes cryogen to decrease the temperature of the skin and cause cell death. Liquid nitrogen is most commonly used.  The low temperature destroys keratinocytes but other structures surrounding it are spared as they are more resistant to the cold. Other surgical options include cosmetic resurfacing, which includes deep chemical peels and dermabrasion. However, insurances do not normally cover the cost of these procedures.2

    All patients should be educated to use sunscreen with an SPF of at least 30 and to avoid sun exposure as much as possible. They should follow-up regularly with their dermatologist, especially if any new lesions appear. If the lesion does not resolve, a biopsy may need to be performed at a later date if not already done prior to treatment.2

    References:

    1. Padilla, Steven. Epidemiology, natural history, and diagnosis of actinic keratosis. UpToDate. 2015. https://www.uptodate.com/contents/epidemiology-natural-history-and-diagnosis-of-actinic-keratosis?source=search_result&search=actinic+keratosis&selectedTitle=2%7E79. Accessed May 21, 2016.
    2. Spencer, J. Actinic Keratosis. Medscape. 2016. http://emedicine.medscape.com/article/1099775-overview. Accessed May 21 2016.
    3. American Academy of Dermatology. Actinic Keratosis. https://www.aad.org/public/diseases/scaly-skin/actinic-keratosis. Accessed May 21 2016.
    4. Kessels, JP. Laser-mediated Photodynamic Therapy: An Alternative Treatment for Actinic Keratosis? Acts Derm Venereol. 2016. https://www.ncbi.nlm.nih.gov/pubmed/26551377. Accessed May 21 2016.
  92. Question 92 of 220
    92. Question

    A 60 year-old postmenopausal white female presents with 3 week history of severe vulvar itching and dyspareunia. She denies multiple sex partners, pelvic pain, or discharge. On exam vulvar skin is erythematous with white scar looking plaques and excoriation from constant itching. Some maceration noted along labia but no other lesions noted on patient’s body. Biopsy is taken and histology shows thinning of hyperkeratosis and epithelial layers with flattening papillae. What is the most likely diagnosis?

    Correct

    Answer is C. Lichen sclerosis is most common in postmenopausal women causing signs and symptoms of pruritus and vulvar pain. Skin is typically erythematous with white wrinkled lesions along the vulva and can include perianal region. Often times maceration and lichenification will result. Lichen sclerosis is diagnosed by biopsy that shows thin hyperkeratotic and epithelial layers, flattened papillae with deep lymphocytic infiltrations. Stopping the itch-scratch cycle is the best way to manage the chronic condition of lichen sclerosus. If the patient stops scratching, the tissue can heal and prevent excoriation and maceration. It is important to educate patient on loose wearing undergarments, cleaning daily with gentle soap, and drying the vulvar area with blow dryer. Oral antihistamine can also be used to help with itching. Patients with lichen sclerosis have a higher rate of squamous cell cancer. Although it is important to biopsy all new lesions it does not manage the symptoms of lichen sclerosis. Cancer actually develops mainly in those who continue to suffer from vulvar itching. Intralesional injections of steroids are used as a last option for those who do not find relief from topical steroids such as clobestasole. Although use of water based lubricants during sexual intercourse may help some with the dyspareunia, it does not help to manage the lichen sclerosis condition itself.

    (A) Vitiligo is common along the vulva but may also include other areas of the skin. It does not cause pruritus or pain. (B) Lichen simplex chronicus causes similar symptoms but on biopsy, tissue is thickened and papillae are elongated. This is typically a result of chronic irritation. (D) Psoriasis is considered a red lesion when it involves the vulva because it often times lacks the white scales and usually shows as erythematous lesions and can look like candidal infections. Patient lacks other lesions on her extensor surfaces or scalp which also helps to rule psoriasis.

    References

    Decherney A, Nathan L, Laufer N, Roman A. Current Diagnosis & Treatment Obstetrics & Gynecology. 11th ed. United States: McGraw-Hill Companies; 2013.

    Cooper S, Arnold S. Vulvar lichen sclerosus. UpToDate.com Web site. http://www.uptodate.com/contents/vulvar-lichen-sclerosus. Accessed May 15, 2016

    We Are Macmillan. Cancer Support Web site. http://www.macmillan.org.uk Accessed May 15, 2016.

    Incorrect

    Answer is C. Lichen sclerosis is most common in postmenopausal women causing signs and symptoms of pruritus and vulvar pain. Skin is typically erythematous with white wrinkled lesions along the vulva and can include perianal region. Often times maceration and lichenification will result. Lichen sclerosis is diagnosed by biopsy that shows thin hyperkeratotic and epithelial layers, flattened papillae with deep lymphocytic infiltrations. Stopping the itch-scratch cycle is the best way to manage the chronic condition of lichen sclerosus. If the patient stops scratching, the tissue can heal and prevent excoriation and maceration. It is important to educate patient on loose wearing undergarments, cleaning daily with gentle soap, and drying the vulvar area with blow dryer. Oral antihistamine can also be used to help with itching. Patients with lichen sclerosis have a higher rate of squamous cell cancer. Although it is important to biopsy all new lesions it does not manage the symptoms of lichen sclerosis. Cancer actually develops mainly in those who continue to suffer from vulvar itching. Intralesional injections of steroids are used as a last option for those who do not find relief from topical steroids such as clobestasole. Although use of water based lubricants during sexual intercourse may help some with the dyspareunia, it does not help to manage the lichen sclerosis condition itself.

    (A) Vitiligo is common along the vulva but may also include other areas of the skin. It does not cause pruritus or pain. (B) Lichen simplex chronicus causes similar symptoms but on biopsy, tissue is thickened and papillae are elongated. This is typically a result of chronic irritation. (D) Psoriasis is considered a red lesion when it involves the vulva because it often times lacks the white scales and usually shows as erythematous lesions and can look like candidal infections. Patient lacks other lesions on her extensor surfaces or scalp which also helps to rule psoriasis.

    References

    Decherney A, Nathan L, Laufer N, Roman A. Current Diagnosis & Treatment Obstetrics & Gynecology. 11th ed. United States: McGraw-Hill Companies; 2013.

    Cooper S, Arnold S. Vulvar lichen sclerosus. UpToDate.com Web site. http://www.uptodate.com/contents/vulvar-lichen-sclerosus. Accessed May 15, 2016

    We Are Macmillan. Cancer Support Web site. http://www.macmillan.org.uk Accessed May 15, 2016.

  93. Question 93 of 220
    93. Question

    A 71-year-old male presents to the office with complaints of pre-syncopal events on numerous occasions when mowing lawn. He admits to shortness of breath and swelling in legs. He denies any chest pain or orthopnea. He has a history of asymptomatic aortic stenosis (AS) for the past 11 years. He is s/p stent in 2002 after anginal chest pain and MI with no residual blockages. He has long standing HTN and 20 pack year smoking history. What test is most specific to evaluate the severity of his AS?

    Correct

    Answer: A. Echocardiogram with Doppler is test of choice for evaluating severity and progression of AS by assessing LVEF (heart’s ability to pump blood past stenotic valve). It can also assess LV wall mass as consequence of patient’s long standing HTN and pressure gradient across leaflets. Echo also looks at valves for motion abnormalities and can measure pressure needed to open leaflets enough to pass blood. (B) Exercise stress testing is contraindicated for symptomatic AS patients because of possible exacerbation of hemodynamic instability. It is appropriate modality for asymptomatic patients as a low-risk procedure and can show abnormal BP changes. (C)BNP does not assess severity of AS. It is used a prognostic evaluation to predict symptom onset for someone with severe AS by increasing BNP levels. However, they are already symptomatic so it is apparent that treatment is needed. (D)EKG is not reliable to assess AS. In this case it might show LV hypertrophy but critical AS cannot be ruled out if there is no LVH. Also, because of patient’s history of MI, voltage may be abnormal already. This modality could be used to r/o ischemia or arrhythmias, but he is not complaining of it so it is not specific enough.
    Aortic stenosis (AS) is a valvar heart disease characterized by obstruction of blood flow across the aortic valve during systole when blood is pumped from the left ventricle into the ascending aorta and out into the peripheral vascular system. The problem with a stenotic aortic valve is that it becomes resistant to smooth blood flow and causes some back up of blood leading to increased pressures in the heart. The increased pressure within the left ventricle because vastly different from the aorta leading to a pressure gradient. The left ventricle is designed to be resistant to this increased pressure and compensates by thickening thus becoming hypertrophied.
    Initially, as with most patients, the left ventricle is still able to contract and pump blood appropriately for many years. Eventually, patients may notice DOE and other symptoms with exercise due to a fall in cardiac output. With this, what is left in the left ventricle after contraction, the end-diastolic pressure, will be more causing a backup in the heart and into the pulmonary capillary beds, also known as systolic dysfunction. This may ultimately lead to HF and heart failure symptoms.

    The pathophysiology behind aortic stenosis will likely involve diastolic dysfunction as it progresses as noted above. The hypertrophic walls of the left ventricle have a more difficult time relaxing and thus the atria will need to work harder to fill. This extra struggle that the atria has may eventually wear out the atria, causing atrial dilation, and other potential valvular disease, such as insufficiencies, as the entire system begins to get congested little by little.
    Unfortunately, as the heart is now having to work harder as the AS progresses, oxygen requirements of the heart increase leading to reduced coronary flow reserve. Once myocardial perfusion is compromised, angina is now possible1. With that, myocardial ischemia and infarction could be likely.

    AS occurs from two different pathways, either acquired or congenital. Unicuspid, bicuspid, tricuspid, and quadricuspid valves are possible, bicuspid being the most common in adults and unicuspid valves being the most common cause of symptomatic AS in adolescents and infants. It is not the bicuspid valves themselves that cause the obstruction to blood flow but rather the constant turbulent flow of blood across the valve that start to damage the leaflets over time and lead to fibrosis, calcification, and stenosis. Surprisingly, most adults with congenital bicuspid valves, are not aware nor are symptomatic until after 40-years-old.

    Acquired degenerative calcific AS, on the other hand, is the most common cause for aortic valve replacement (AVR). Risk factors for developing degenerative calcific AS is hypertension, hyperlipidemia, DM, and smoking. These increase the risk of AS because they put constant hemodynamic stress on the leaflets leading to limited normal motion during systole.
    There are also many other causes of AS but among them are rheumatic heart disease, often causing symptoms after 50 yo. RHD leads to commissural fusion and fibrosis causing both stenosis and regurgitation of aortic valve. It is not uncommon to have coexisting mitral valve insufficiency.
    The incidence of AS increases with age second to long standing histories of above risk factors. These environments cause sclerosing that initially is non-obstructing but degenerates to stenosis. Percentages range as high as 29% over 65-year-old to 37% over 75-years-old. Prevalence of AS is 2-9%, most commonly in 75-year-old males and older.

    Symptoms of AS gradually progress so a patient might be followed closely for as much as 10-20 years before they start to experience symptoms. Screening questions range greatly when interviewing a patient that you might suspect symptomatic AS. It is important to ask about DOE as this is often the initial complaint followed by exertional chest pain, and progressive inability to exercise. The chest pain felt with AS is often relieved with rest which could be hard to differentiate from angina pectoris of CAD.

    Symptoms for these patients are typically exacerbated by activity and they might start to notice lightheadedness and syncope with activity. It is possible to correlate to syncopal events to different etiologies. When it occurs with activity, it is likely due to systemic vasodilation or tachyarrhythmias.
    The classic triad of AS is not complete without heart failure symptoms. These heart failure symptoms can occur from either systolic or diastolic dysfunction as discussed above. To remind you, PND, orthopnea, DOE, and SOB should point to CHF.

    Most clues you might find when physically examining an AS patient will be found with either your stethoscope or finger tips. On palpation of different pulses, you might find delayed upstroke and decreased amplitude of the carotid arteries known as pulsus tardus. When examining the Jugular venous pulse, pulsus alternans may be identified. Remember, this is due to poor ability of the right ventricle to welcome more blood due to left sided and septal hypertrophy.

    When placing the stethoscope to the precordium, you may notice abnormalities in A2, which is often diminished or unheard because of its immobility. Paradoxical splitting might also be heard of S2 as A2 will close late due to calcification. Although, if pulmonary hypertension is presents as well, P2 might also be delayed. You may even hear an ejection click in children with congenital AS, heard best along LLSB after S1. Another extra heart sound you might hear is S4. If you recall, it is caused by atrial forcefully contracting to fill hypertrophied LV second to long standing hemodynamic pressures.

    Unfortunately with Aortic stenosis, you have to decode the dreaded and confusing murmurs. The classic murmur of AS is crescendo-decrescendo murmur best heard in the 2nd ICS of the right upper sternal border during midsystole. It often sounds rough or harsh in nature and can radiate to both carotid arteries. It is key to remember that it is not the loudness or intensity of the murmur that should clue to into the severity but rather the duration. Longer duration equals more severe AS. Fun fact, valsalva will decrease the intensity of the AS murmur. Why is that? Well think about it, valsalva causes less blood to enter LV so less blood is trying to push through stenotic aortic valve.
    Some patients may even have consequences of aortic regurgitation also. This murmur would be a diastolic high-pitch and blowing sound heard in the area of the apex.

    Diagnostic studies to confirm aortic stenosis always include a two-dimensional and doppler echocardiography, the modality of choice. Not only is it used to diagnose AS but it is also used to assess LV size, function, and hemodynamics, reevaluate patients with known AS with changing symptoms or even asymptomatic, and to assess severity (must use doppler echo). The structure and function of all valves can be assessed with this modality although three findings are indicative of severe AS. These include echo-dense aortic valve without cusp movement, decreased separation between cusps (< 8 mm), and unexplained left ventricular hypertrophy.

    Severity is measured using transvalvular gradient across the valve with mild being < 25 mmHg and sever being > 40 mmHg. Sometimes however, the echocardiogram may show minimal severity when in fact the AS appears clinically severe. In this case, evaluation with a cardiac catheterization may be necessary. The cardiac cath can measure the EF and the heart’s ability to pump past the stenotic valve, assess the need for AVR, among other things. A coronary arteriography is also indicated in those over 35-year-old before AVR to rule out symptomatology second to CAD3. It is known that 50% of patients over 50-years-old with AS have coexisting CAD.

    Other tests might be suggested to better assess the patient. Exercise stress testing can be used only in asymptomatic AS patients with severe aortic stenosis to assess exercise capacity, hemodynamics, and to establish if low-gradient AS is present. Chest radiography can be used to support diagnosis, though it is nonspecific to AS. Electrocardiography may be used to assess for LV hypertrophy, although if absent, severe AS cannot be ruled out. Lastly, B-type Natruietic peptide (BNP) can be used to assess prognosis. A rise in BNP could predict upcoming symptoms in an asymptomatic patient and may suggest the need for AVR.

    A big takeaway from this discussion is that the only definitive treatment for AS is aortic valve replacement (AVR). Once, symptoms of AS develop, it is pertinent that the patient be evaluated for AVR. Research shows that if your patient is asymptomatic, they can be followed closely until symptoms arise while optimizing other cardiac factors such as blood pressure, hydration, physical activity, blood sugar management, and smoking cessation. But, AVR is recommended for asymptomatic patients that have severe AS and left ventricle dysfunction.

    Fortunately, EF and LV mass improve within months of AVR. AVR 5-year survival rate for adults is 80-94%. AVR is completed with either bioprosthetic or mechanical valve placement. After surgery, endocarditis prophylaxis is no longer necessary or recommended.

    When a patient is not a good candidate for AVR or refuses, palliative treatment could be initiated. Percutaneous balloon valvuloplasty can be considered for patient with short life expectancy second to comorbidity, to bridge those critically ill until they can undergo AVR, and pregnant patients.
    Medical treatment is not possible with AS. However, managing AS can be done but it is limited. Caution should be taken with diuretics, ACE-I, beta blockers, and other vasodilators because they may cause exacerbation.

    References

    1. Xiushui (Mike) Ren, MD. Aortic Stenosis Pathophysiology. Medscape.com Web site. http://emedicine.medscape.com/article/150638-overview#a3. Accessed April 8, 2016.

    2. Papadakis M, McPhee S. Current Medical Diagnosis & Treatment. 54th ed. United States: McGraw-Hill Education; 2015.

    3. William H Gaasch, MD. Understanding history, epidemiology, and prognosis of Aortic Stenosis. Uptodate.com Web site. http://www.uptodate.com/contents/search?search=aortic+stenosis+ pathophysiology&sp=0&searchType=PLAIN_TEXT&source=USER_INPUT&searchControl=TOP_PULLDOWN&searchOffset=. Accessed April 7, 2016.

    Incorrect

    Answer: A. Echocardiogram with Doppler is test of choice for evaluating severity and progression of AS by assessing LVEF (heart’s ability to pump blood past stenotic valve). It can also assess LV wall mass as consequence of patient’s long standing HTN and pressure gradient across leaflets. Echo also looks at valves for motion abnormalities and can measure pressure needed to open leaflets enough to pass blood. (B) Exercise stress testing is contraindicated for symptomatic AS patients because of possible exacerbation of hemodynamic instability. It is appropriate modality for asymptomatic patients as a low-risk procedure and can show abnormal BP changes. (C)BNP does not assess severity of AS. It is used a prognostic evaluation to predict symptom onset for someone with severe AS by increasing BNP levels. However, they are already symptomatic so it is apparent that treatment is needed. (D)EKG is not reliable to assess AS. In this case it might show LV hypertrophy but critical AS cannot be ruled out if there is no LVH. Also, because of patient’s history of MI, voltage may be abnormal already. This modality could be used to r/o ischemia or arrhythmias, but he is not complaining of it so it is not specific enough.
    Aortic stenosis (AS) is a valvar heart disease characterized by obstruction of blood flow across the aortic valve during systole when blood is pumped from the left ventricle into the ascending aorta and out into the peripheral vascular system. The problem with a stenotic aortic valve is that it becomes resistant to smooth blood flow and causes some back up of blood leading to increased pressures in the heart. The increased pressure within the left ventricle because vastly different from the aorta leading to a pressure gradient. The left ventricle is designed to be resistant to this increased pressure and compensates by thickening thus becoming hypertrophied.
    Initially, as with most patients, the left ventricle is still able to contract and pump blood appropriately for many years. Eventually, patients may notice DOE and other symptoms with exercise due to a fall in cardiac output. With this, what is left in the left ventricle after contraction, the end-diastolic pressure, will be more causing a backup in the heart and into the pulmonary capillary beds, also known as systolic dysfunction. This may ultimately lead to HF and heart failure symptoms.

    The pathophysiology behind aortic stenosis will likely involve diastolic dysfunction as it progresses as noted above. The hypertrophic walls of the left ventricle have a more difficult time relaxing and thus the atria will need to work harder to fill. This extra struggle that the atria has may eventually wear out the atria, causing atrial dilation, and other potential valvular disease, such as insufficiencies, as the entire system begins to get congested little by little.
    Unfortunately, as the heart is now having to work harder as the AS progresses, oxygen requirements of the heart increase leading to reduced coronary flow reserve. Once myocardial perfusion is compromised, angina is now possible1. With that, myocardial ischemia and infarction could be likely.

    AS occurs from two different pathways, either acquired or congenital. Unicuspid, bicuspid, tricuspid, and quadricuspid valves are possible, bicuspid being the most common in adults and unicuspid valves being the most common cause of symptomatic AS in adolescents and infants. It is not the bicuspid valves themselves that cause the obstruction to blood flow but rather the constant turbulent flow of blood across the valve that start to damage the leaflets over time and lead to fibrosis, calcification, and stenosis. Surprisingly, most adults with congenital bicuspid valves, are not aware nor are symptomatic until after 40-years-old.

    Acquired degenerative calcific AS, on the other hand, is the most common cause for aortic valve replacement (AVR). Risk factors for developing degenerative calcific AS is hypertension, hyperlipidemia, DM, and smoking. These increase the risk of AS because they put constant hemodynamic stress on the leaflets leading to limited normal motion during systole.
    There are also many other causes of AS but among them are rheumatic heart disease, often causing symptoms after 50 yo. RHD leads to commissural fusion and fibrosis causing both stenosis and regurgitation of aortic valve. It is not uncommon to have coexisting mitral valve insufficiency.
    The incidence of AS increases with age second to long standing histories of above risk factors. These environments cause sclerosing that initially is non-obstructing but degenerates to stenosis. Percentages range as high as 29% over 65-year-old to 37% over 75-years-old. Prevalence of AS is 2-9%, most commonly in 75-year-old males and older.

    Symptoms of AS gradually progress so a patient might be followed closely for as much as 10-20 years before they start to experience symptoms. Screening questions range greatly when interviewing a patient that you might suspect symptomatic AS. It is important to ask about DOE as this is often the initial complaint followed by exertional chest pain, and progressive inability to exercise. The chest pain felt with AS is often relieved with rest which could be hard to differentiate from angina pectoris of CAD.

    Symptoms for these patients are typically exacerbated by activity and they might start to notice lightheadedness and syncope with activity. It is possible to correlate to syncopal events to different etiologies. When it occurs with activity, it is likely due to systemic vasodilation or tachyarrhythmias.
    The classic triad of AS is not complete without heart failure symptoms. These heart failure symptoms can occur from either systolic or diastolic dysfunction as discussed above. To remind you, PND, orthopnea, DOE, and SOB should point to CHF.

    Most clues you might find when physically examining an AS patient will be found with either your stethoscope or finger tips. On palpation of different pulses, you might find delayed upstroke and decreased amplitude of the carotid arteries known as pulsus tardus. When examining the Jugular venous pulse, pulsus alternans may be identified. Remember, this is due to poor ability of the right ventricle to welcome more blood due to left sided and septal hypertrophy.

    When placing the stethoscope to the precordium, you may notice abnormalities in A2, which is often diminished or unheard because of its immobility. Paradoxical splitting might also be heard of S2 as A2 will close late due to calcification. Although, if pulmonary hypertension is presents as well, P2 might also be delayed. You may even hear an ejection click in children with congenital AS, heard best along LLSB after S1. Another extra heart sound you might hear is S4. If you recall, it is caused by atrial forcefully contracting to fill hypertrophied LV second to long standing hemodynamic pressures.

    Unfortunately with Aortic stenosis, you have to decode the dreaded and confusing murmurs. The classic murmur of AS is crescendo-decrescendo murmur best heard in the 2nd ICS of the right upper sternal border during midsystole. It often sounds rough or harsh in nature and can radiate to both carotid arteries. It is key to remember that it is not the loudness or intensity of the murmur that should clue to into the severity but rather the duration. Longer duration equals more severe AS. Fun fact, valsalva will decrease the intensity of the AS murmur. Why is that? Well think about it, valsalva causes less blood to enter LV so less blood is trying to push through stenotic aortic valve.
    Some patients may even have consequences of aortic regurgitation also. This murmur would be a diastolic high-pitch and blowing sound heard in the area of the apex.

    Diagnostic studies to confirm aortic stenosis always include a two-dimensional and doppler echocardiography, the modality of choice. Not only is it used to diagnose AS but it is also used to assess LV size, function, and hemodynamics, reevaluate patients with known AS with changing symptoms or even asymptomatic, and to assess severity (must use doppler echo). The structure and function of all valves can be assessed with this modality although three findings are indicative of severe AS. These include echo-dense aortic valve without cusp movement, decreased separation between cusps (< 8 mm), and unexplained left ventricular hypertrophy.

    Severity is measured using transvalvular gradient across the valve with mild being < 25 mmHg and sever being > 40 mmHg. Sometimes however, the echocardiogram may show minimal severity when in fact the AS appears clinically severe. In this case, evaluation with a cardiac catheterization may be necessary. The cardiac cath can measure the EF and the heart’s ability to pump past the stenotic valve, assess the need for AVR, among other things. A coronary arteriography is also indicated in those over 35-year-old before AVR to rule out symptomatology second to CAD3. It is known that 50% of patients over 50-years-old with AS have coexisting CAD.

    Other tests might be suggested to better assess the patient. Exercise stress testing can be used only in asymptomatic AS patients with severe aortic stenosis to assess exercise capacity, hemodynamics, and to establish if low-gradient AS is present. Chest radiography can be used to support diagnosis, though it is nonspecific to AS. Electrocardiography may be used to assess for LV hypertrophy, although if absent, severe AS cannot be ruled out. Lastly, B-type Natruietic peptide (BNP) can be used to assess prognosis. A rise in BNP could predict upcoming symptoms in an asymptomatic patient and may suggest the need for AVR.

    A big takeaway from this discussion is that the only definitive treatment for AS is aortic valve replacement (AVR). Once, symptoms of AS develop, it is pertinent that the patient be evaluated for AVR. Research shows that if your patient is asymptomatic, they can be followed closely until symptoms arise while optimizing other cardiac factors such as blood pressure, hydration, physical activity, blood sugar management, and smoking cessation. But, AVR is recommended for asymptomatic patients that have severe AS and left ventricle dysfunction.

    Fortunately, EF and LV mass improve within months of AVR. AVR 5-year survival rate for adults is 80-94%. AVR is completed with either bioprosthetic or mechanical valve placement. After surgery, endocarditis prophylaxis is no longer necessary or recommended.

    When a patient is not a good candidate for AVR or refuses, palliative treatment could be initiated. Percutaneous balloon valvuloplasty can be considered for patient with short life expectancy second to comorbidity, to bridge those critically ill until they can undergo AVR, and pregnant patients.
    Medical treatment is not possible with AS. However, managing AS can be done but it is limited. Caution should be taken with diuretics, ACE-I, beta blockers, and other vasodilators because they may cause exacerbation.

    References

    1. Xiushui (Mike) Ren, MD. Aortic Stenosis Pathophysiology. Medscape.com Web site. http://emedicine.medscape.com/article/150638-overview#a3. Accessed April 8, 2016.

    2. Papadakis M, McPhee S. Current Medical Diagnosis & Treatment. 54th ed. United States: McGraw-Hill Education; 2015.

    3. William H Gaasch, MD. Understanding history, epidemiology, and prognosis of Aortic Stenosis. Uptodate.com Web site. http://www.uptodate.com/contents/search?search=aortic+stenosis+ pathophysiology&sp=0&searchType=PLAIN_TEXT&source=USER_INPUT&searchControl=TOP_PULLDOWN&searchOffset=. Accessed April 7, 2016.

  94. Question 94 of 220
    94. Question

    A 34 year-old woman presents to urgent care with a sore throat and fatigue for four days. She is also complains of back pain, chills, and fever. She denies any sick contacts but did admit to having a completed miscarriage 6 days prior and thought her symptoms may be a result from that. She denies abdominal pain or pelvic pain, cough, vomiting, rash, or dysuria. On physical exam, she is dressed in layers with rigors. Temp 104 ˚F, orally. She has a mildly erythematous pharynx with exudate and tonsillar hypertrophy. She has tender bilateral anterior lymphadenopathy without nuchal rigidity. Lung sounds and cardiac exam normal. Abdomen and mid thoracic back mildly tender to palpation without organomegaly. The remainder of the exam is unremarkable. Based on the information provided, which of the following diagnostic tests would be the most appropriate next step?

    Correct

    The correct answer is C. This patient is presenting with the typical symptoms of streptococcal pharyngitis. Although the most common cause of strep is viral, this patient has a bacterial infection. This is most commonly caused by Group A Beta Hemolytic Streptococcus.1  The Centor Criteria for strep throat consists of: Fever (>100.4 degrees Fahrenheit), tender anterior cervical lymphadenopathy, absence of a cough, and pharyngotonsillar exudate.1 Each symptom is worth 1 point. Any patient with 2-3 points should get a throat culture.1 This patient presents with 3 of the 4 symptoms (fever, anterior cervical lymphadenopathy, and no cough), giving her a score of 3 points and therefore needing a throat culture.

    Choice A, a rapid influenza test via nasopharyngeal swab is incorrect. The common symptoms for influenza do include fever, myalgias, and chills but also a cough, nasal congestion, coryza, and headaches.2  This patient is not experiencing a majority of these symptoms, so therefore a rapid influenza test would not be the best choice.

    Choice B, MonoSpot test followed by Epstein Barr titers is incorrect. Mononucleosis is a good option to keep in the differential, but a key symptom that would be a negative indication of mononucleosis is “anterior” cervical lymphadenopathy. Most patients with mononucleosis will have posterior cervical lymphadenopathy.1  Although patients with mononucleosis will have a sore throat and fever, they will also complain of severe fatigue as well as abdominal discomfort. This patient denied these symptoms. The abdominal pain is due to the inflammation of the spleen and liver that occurs during the disease. Patients who have an enlarged spleen or liver should avoid contact sports and trauma for at least 1 month in order to avoid possible rupture.1 On physical exam, bilateral lid edema known as Hoagland sign and conjunctival hemorrhaging may be present as well.2

    Choice D, do nothing because it’s viral, is not the correct answer. This patient has been experiencing high fevers for multiple days which would indicate more of a bacterial infection rather than viral. Most viral infections have lower grade fevers as opposed to bacterial infections where fevers tend to be higher.3 Also, in bacterial infections the fever will get worse a few days into the illness rather than getting better.3

    Resources

    1. Williams D.A. PANCE Prep Pearls. CreateSpace. 2014.
    2. Papadakis M, McPhee S, Rabow M. Current Medical Diagnosis & Treatment 2015. 2015
    3. Duke University Health System. Is it a Bacterial Infection or Virus?. DukeMedicine. https://www.dukemedicine.org/blog/it-bacterial-infection-or-virus. Published Oct. 1, 2013. Accessed Jan. 25, 2016.
    4. Wald, E. R. Approach to diagnosis of acute infectious pharyngitis in children and adolescents. UpToDate. http://www.uptodate.com/contents/approach-to-diagnosis-of-acute-infectious-pharyngitis-in-children-and-adolescents?source=search_result. Published Aug. 11, 2015. Accessed Jan. 28, 2016.
    5. Marquez-Carrillo M.A. Bacterial Pharyngitis. Medscape. http://emedicine.medscape.com/article/225243-overview. Published Jul. 28, 2015. Accessed Feb. 1, 2016.
    Incorrect

    The correct answer is C. This patient is presenting with the typical symptoms of streptococcal pharyngitis. Although the most common cause of strep is viral, this patient has a bacterial infection. This is most commonly caused by Group A Beta Hemolytic Streptococcus.1  The Centor Criteria for strep throat consists of: Fever (>100.4 degrees Fahrenheit), tender anterior cervical lymphadenopathy, absence of a cough, and pharyngotonsillar exudate.1 Each symptom is worth 1 point. Any patient with 2-3 points should get a throat culture.1 This patient presents with 3 of the 4 symptoms (fever, anterior cervical lymphadenopathy, and no cough), giving her a score of 3 points and therefore needing a throat culture.

    Choice A, a rapid influenza test via nasopharyngeal swab is incorrect. The common symptoms for influenza do include fever, myalgias, and chills but also a cough, nasal congestion, coryza, and headaches.2  This patient is not experiencing a majority of these symptoms, so therefore a rapid influenza test would not be the best choice.

    Choice B, MonoSpot test followed by Epstein Barr titers is incorrect. Mononucleosis is a good option to keep in the differential, but a key symptom that would be a negative indication of mononucleosis is “anterior” cervical lymphadenopathy. Most patients with mononucleosis will have posterior cervical lymphadenopathy.1  Although patients with mononucleosis will have a sore throat and fever, they will also complain of severe fatigue as well as abdominal discomfort. This patient denied these symptoms. The abdominal pain is due to the inflammation of the spleen and liver that occurs during the disease. Patients who have an enlarged spleen or liver should avoid contact sports and trauma for at least 1 month in order to avoid possible rupture.1 On physical exam, bilateral lid edema known as Hoagland sign and conjunctival hemorrhaging may be present as well.2

    Choice D, do nothing because it’s viral, is not the correct answer. This patient has been experiencing high fevers for multiple days which would indicate more of a bacterial infection rather than viral. Most viral infections have lower grade fevers as opposed to bacterial infections where fevers tend to be higher.3 Also, in bacterial infections the fever will get worse a few days into the illness rather than getting better.3

    Resources

    1. Williams D.A. PANCE Prep Pearls. CreateSpace. 2014.
    2. Papadakis M, McPhee S, Rabow M. Current Medical Diagnosis & Treatment 2015. 2015
    3. Duke University Health System. Is it a Bacterial Infection or Virus?. DukeMedicine. https://www.dukemedicine.org/blog/it-bacterial-infection-or-virus. Published Oct. 1, 2013. Accessed Jan. 25, 2016.
    4. Wald, E. R. Approach to diagnosis of acute infectious pharyngitis in children and adolescents. UpToDate. http://www.uptodate.com/contents/approach-to-diagnosis-of-acute-infectious-pharyngitis-in-children-and-adolescents?source=search_result. Published Aug. 11, 2015. Accessed Jan. 28, 2016.
    5. Marquez-Carrillo M.A. Bacterial Pharyngitis. Medscape. http://emedicine.medscape.com/article/225243-overview. Published Jul. 28, 2015. Accessed Feb. 1, 2016.
  95. Question 95 of 220
    95. Question

    A 73 year old male presents to the clinic with complaints of severe left knee and ankle pain. The patient has a history peptic ulcer disease, Wegener granulomatosis, hyperlipidemia, gout, chronic kidney disease, and hypertension. He reports a recent fall after “drinking too much” at a seafood and BBQ festival. The patient denies chest pain, cough, shortness of breath, recent travel. On physical exam you note erythema and an effusion to the knee joint. The patient is afebrile and has a serum urate level of 8.2. You aspirate synovial fluid of one of the involved joints and under polarizing light microscopy yellow urate crystals are observed. What would the best oral treatment to manage this patient’s acute joint pain?

    Correct

    The Answer is C. Gout is an inflammatory arthritis condition caused by the interaction between phagocytes and monosodium urate (MSU). Gout can be acute or chronic and involve one or multiple joints. MSU crystals are deposited in the joints and periarticular tissues. A gout flare can be triggered by a release of uncoated crystals or precipitation of crystals in a supersaturated microenvironment. The MSU crystals interact with intracellular and surface receptors of local immune system cells. This interaction in turn incites signals to activate parts of the pro-inflammatory innate immune system. This condition is predominantly a disease of adult men. Gout is rarely seen in pre-menopausal women since estrogen is uricosuric. The incidence of gout is increasing and the prevalence of gout is approximately 9-14 per 1000 (Mikuls T et all 2005).

    Hyperuricemia can occur as a result of overproduction, under-excretion, or both of uric acid. Overproduction of uric acid can be caused by rapid cell turnover, excessive dietary purine intake, or inherited disorders of purine nucleotide biosynthesis. Under-excretion of uric acid by the kidney is the most common cause of hyperuricemia. Mixed overproduction and under-excretion of uric acid, most commonly results from excessive alcohol consumption. Acute flares are often precipitated by trauma, initiation of diuretics, dehydration, dietary, systemic illness/surgery resulting in bed rest, and progressive renal insufficiency. It should be noted that not all patients with hyperuricemia have gout.

    An acute gout flare typically occurs as a rapid onset with a peak at about 12-24 hours after onset. The patient may present with a very painful, swollen, red, mono-articular joint. Common sites of acute flares are olecranon bursa, elbow, wrist, fingers, knee, ankle, subtalar, and midfoot. Patients with chronic gout may experience about 1-2 gout attacks per year. Whereas for others, gout can be a relapsing problem with multiple attacks without complete resolution.  Advanced gout is characterized by tophi, chronic pain, and deformities.

    Diagnosis of gout can be made from uric acid crystals in synovial fluid, elevated serum uric acid level, or the presence of tophi. During an acute gout attack, the presence of uric acid crystals in aspirated synovial joint fluid or tophi is definitive diagnostic finding.  For analysis, the fluid is examined under a polarized light microscope. Serum uric acid concentration is a supportive diagnostic feature. MSU crystallization occurs when the serum level of urate is greater than 6.8 mg/dL, which is the physiologic limit of solubility.  However, serum uric acid levels can be normal during a gout flare. Radiographic findings comprise of soft tissue swelling early in the disease process compared to overhanging edges and asymmetric erosions in the mid to late stage of the disease process.

    The goal of treating acute gout flares is rapid relief of pain and inflammation. Response to treatment typically occurs within the first 24-36 hours of the attack. Medical treatment of acute gout flares comprise of oral NSAIDs, Colchicine, or Corticosteroids. Management of chronic gout involves the use of uric acid lowering medications. These medications are indicated in those with several attacks per year, tophi, urate kidney stones, and x-ray changes of gout. Uric acid lowering medications can reduce the frequency of gout attacks over time, tophi formation, and the risk of joint destruction. Medication options for uric acid lowering include: Probenecid, Allopurinol, Febuxostat, and Pegloticase.  Patients with chronic gout should also be advised to avoid foods high in purines such as shellfish, red meat, anchovies, kidneys, yeast, and trout.

    References

    Aromdee E, Michet C, Crowson C, O’Fallon M, Gabriel S. Epidemiology of gout: is the incidence rising? J Rheumatology 2002; 29:2403–6.

    Fravel MA, Ernst ME. Management of gout in the older adult. American Journal of Geriatric Pharmacotherapy. 2011 Oct. 9(5):271-85

    Martinon F, Glimcher LH. Gout: new insights into an old disease. The Journal of Clinical Investigation. 2006;116(8):2073-2075. doi:10.1172/JCI29404.

    Mikuls T, Farrar J, Bilker W, Fernandes S, Schumacher Jr. H, Saag K. Gout epidemiology: results from the UK General Practice Research Database 1990-1999. Ann Rheum Dis. 2005; 64:2 267-272. doi:10.1136/ard.2004.024091.

    Singh JA, Reddy SG, Kundukulam J. Risk factors for gout and prevention: a systematic review of the literature. Current Opinion in Rheumatology. 2011 Mar. 23(2):192-202.

    Incorrect

    The Answer is C. Gout is an inflammatory arthritis condition caused by the interaction between phagocytes and monosodium urate (MSU). Gout can be acute or chronic and involve one or multiple joints. MSU crystals are deposited in the joints and periarticular tissues. A gout flare can be triggered by a release of uncoated crystals or precipitation of crystals in a supersaturated microenvironment. The MSU crystals interact with intracellular and surface receptors of local immune system cells. This interaction in turn incites signals to activate parts of the pro-inflammatory innate immune system. This condition is predominantly a disease of adult men. Gout is rarely seen in pre-menopausal women since estrogen is uricosuric. The incidence of gout is increasing and the prevalence of gout is approximately 9-14 per 1000 (Mikuls T et all 2005).

    Hyperuricemia can occur as a result of overproduction, under-excretion, or both of uric acid. Overproduction of uric acid can be caused by rapid cell turnover, excessive dietary purine intake, or inherited disorders of purine nucleotide biosynthesis. Under-excretion of uric acid by the kidney is the most common cause of hyperuricemia. Mixed overproduction and under-excretion of uric acid, most commonly results from excessive alcohol consumption. Acute flares are often precipitated by trauma, initiation of diuretics, dehydration, dietary, systemic illness/surgery resulting in bed rest, and progressive renal insufficiency. It should be noted that not all patients with hyperuricemia have gout.

    An acute gout flare typically occurs as a rapid onset with a peak at about 12-24 hours after onset. The patient may present with a very painful, swollen, red, mono-articular joint. Common sites of acute flares are olecranon bursa, elbow, wrist, fingers, knee, ankle, subtalar, and midfoot. Patients with chronic gout may experience about 1-2 gout attacks per year. Whereas for others, gout can be a relapsing problem with multiple attacks without complete resolution.  Advanced gout is characterized by tophi, chronic pain, and deformities.

    Diagnosis of gout can be made from uric acid crystals in synovial fluid, elevated serum uric acid level, or the presence of tophi. During an acute gout attack, the presence of uric acid crystals in aspirated synovial joint fluid or tophi is definitive diagnostic finding.  For analysis, the fluid is examined under a polarized light microscope. Serum uric acid concentration is a supportive diagnostic feature. MSU crystallization occurs when the serum level of urate is greater than 6.8 mg/dL, which is the physiologic limit of solubility.  However, serum uric acid levels can be normal during a gout flare. Radiographic findings comprise of soft tissue swelling early in the disease process compared to overhanging edges and asymmetric erosions in the mid to late stage of the disease process.

    The goal of treating acute gout flares is rapid relief of pain and inflammation. Response to treatment typically occurs within the first 24-36 hours of the attack. Medical treatment of acute gout flares comprise of oral NSAIDs, Colchicine, or Corticosteroids. Management of chronic gout involves the use of uric acid lowering medications. These medications are indicated in those with several attacks per year, tophi, urate kidney stones, and x-ray changes of gout. Uric acid lowering medications can reduce the frequency of gout attacks over time, tophi formation, and the risk of joint destruction. Medication options for uric acid lowering include: Probenecid, Allopurinol, Febuxostat, and Pegloticase.  Patients with chronic gout should also be advised to avoid foods high in purines such as shellfish, red meat, anchovies, kidneys, yeast, and trout.

    References

    Aromdee E, Michet C, Crowson C, O’Fallon M, Gabriel S. Epidemiology of gout: is the incidence rising? J Rheumatology 2002; 29:2403–6.

    Fravel MA, Ernst ME. Management of gout in the older adult. American Journal of Geriatric Pharmacotherapy. 2011 Oct. 9(5):271-85

    Martinon F, Glimcher LH. Gout: new insights into an old disease. The Journal of Clinical Investigation. 2006;116(8):2073-2075. doi:10.1172/JCI29404.

    Mikuls T, Farrar J, Bilker W, Fernandes S, Schumacher Jr. H, Saag K. Gout epidemiology: results from the UK General Practice Research Database 1990-1999. Ann Rheum Dis. 2005; 64:2 267-272. doi:10.1136/ard.2004.024091.

    Singh JA, Reddy SG, Kundukulam J. Risk factors for gout and prevention: a systematic review of the literature. Current Opinion in Rheumatology. 2011 Mar. 23(2):192-202.

  96. Question 96 of 220
    96. Question

    A 60-year-old postmenopausal G3P3 female presents with bloating for 2 months accompanied by nausea for one week. She denies breast, pelvic, and abdominal pain. She explains that she has had to wear elastic waistbands the past few weeks due to her expanding waistline. She denies changes in her breasts. She reports that she has not changed her diet or activity level recently. She denies any bowel or urinary changes in the past 2 months. She reports mild urine leakage with coughing and sneezing for the past several years, which is not bothersome to her at this time. She reports that she wears a panty liner during the day in case she has urine leakage. She says that she hasn’t been sleeping well the past few weeks and feels more fatigued than her normal. She has woken up in the morning with nausea for the past week. She denies shortness of breath, chest pain, palpitations, or dizziness. What is the test you want to order first for the presentation above?

    Correct

    The correct answer is C. Pelvic ultrasound (transvaginal and abdominal) is the first test you would order when trying to rule out ovarian pathology. Other reasonable answers would have included a serum marker CA125, or a pelvic x-ray, both of which were not answer options provided but are often included in the workup in conjunction with orders for a pelvic ultrasound.

    Abdominal CT scan is more often indicated in the evaluation of abdominal pain, of which this patient does not present with. Abdominal CT can be used in the evaluation of ovarian pathology, although it is not the standard first-line test, partly for the reason that CT scans subject the patient to radiation and ultrasound testing does not. Occasionally, abdominal or pelvic CT is needed to help clarify ultrasound findings.

    Abdominal ultrasound is partially correct, as it is often used in conjunction with a transvaginal ultrasound to evaluate for ovarian pathology. Abdominal ultrasound alone would not be the best choice given above, as the transvaginal combined with the abdominal ultrasound provides more information about the uterus and ovaries.

    DXA scan is used in the evaluation of bone mineral density and the screening and diagnosis of osteoporosis. The patient presentation in this case is note related to symptoms and findings related to bone density.

    Most women with epithelial ovarian cancer have pelvic or abdominal symptoms prior to their diagnosis. Women with ovarian pathology most typically present with symptoms related to the abdomen (77% of cases), gastrointestinal (70%), pain (58%), constitutional (50%), urinary (34%), and pelvic (26%).   The most common symptoms are bloating, urinary urgency or frequency, difficulty eating or feeling full, and/or abdominal or pelvic pain. Women who complain of these symptoms should be evaluated for a thorough history that includes frequency and severity of each symptom. Persistent abdominal distention, rather than fluctuating, can be more specific for ovarian pathology.

    A symptom index has been developed to aid clinicians in evaluating women for early symptoms of epithelial ovarian cancer (although it is not recommended at this time for clinical use). The symptom index is considered to be positive if a woman reports any of the following symptoms that are: new to her within the past year and occur more than 12 times per month: pelvic or abdominal pain, increased abdominal size or abdominal bloating, or difficulty eating or feeling full quickly. Physical exam findings suggestive of epithelial ovarian cancer include adnexal mass, abdominal ascites, “omental cake” or mass in the mid to left upper abdomen, pleural effusion, and groin or supraclavicular lymphadenopathy.

    Cancer of the ovary most often requires surgery for diagnosis, staging, and initial management. Most of the time, this involves a surgical total hysterectomy with salpingo-oophorectomy, where the doctor removes the ovaries, the fallopian tubes, and the uterus. Staging and initial surgical management should be performed by a gynecologic oncologist whenever possible to ensure better outcomes. Further treatment will depend a lot on the stage of the cancer, a woman’s age, and her other medical problems. Some women might not need any further treatment after surgery. Other women might need chemotherapy.

    Risk assessment for ovarian cancer depends upon age and family history. Ovarian cancer occurs most often in women ages 50 to 65. Ovarian cancer is the second most common gynecologic malignancy and the most common cause of death among women with gynecologic cancer. Most women are diagnosed at an advanced stage, while early stages of the disease are potentially curable. Screening programs for epithelial ovarian cancer using pelvic imaging or tumor markers have not yet been successful. The identification of epithelial ovarian cancer symptoms to aid early detection has become a focus of clinical research. Epithelial carcinoma of the ovaries, fallopian tubes, and peritoneum are clinically similar. Evidence suggests that these diseases have a common pathogenesis and may be initiated in the fallopian tubes. Almost 80 percent of women have lymph node or distant metastases at the time of diagnosis. Five-year survival rates for advanced disease are 19 to 32 percent.

    References

    http://www.uptodate.com/contents/screening-for-ovarian-cancer?source=machineLearning&search=ovarian+cancer&selectedTitle=2~150&sectionRank=1&anchor=H9#H9

    http://www.uptodate.com/contents/epithelial-carcinoma-of-the-ovary-fallopian-tube-and-peritoneum-clinical-features-and-diagnosis source=search_result&search=ovarian+cancer&selectedTitle=1~150

    http://www.uptodate.com/contents/overview-of-dual-energy-x-ray-absorptiometry?source=machineLearning&search=dexa+scan&selectedTitle=1~123&sectionRank=1&anchor=H5#H6

    http://www.uptodate.com/contents/search?search=clinical+presentation+ovarian+cancer&sp=0&searchType=PLAIN_TEXT&source=USER_INPUT&searchControl=TOP_PULLDOWN&searchOffset=

    http://www.uptodate.com/contents/evaluation-of-the-adult-with-abdominal-pain?source=search_result&search=abdominal+ct+scan&selectedTitle=1~150

     

    Incorrect

    The correct answer is C. Pelvic ultrasound (transvaginal and abdominal) is the first test you would order when trying to rule out ovarian pathology. Other reasonable answers would have included a serum marker CA125, or a pelvic x-ray, both of which were not answer options provided but are often included in the workup in conjunction with orders for a pelvic ultrasound.

    Abdominal CT scan is more often indicated in the evaluation of abdominal pain, of which this patient does not present with. Abdominal CT can be used in the evaluation of ovarian pathology, although it is not the standard first-line test, partly for the reason that CT scans subject the patient to radiation and ultrasound testing does not. Occasionally, abdominal or pelvic CT is needed to help clarify ultrasound findings.

    Abdominal ultrasound is partially correct, as it is often used in conjunction with a transvaginal ultrasound to evaluate for ovarian pathology. Abdominal ultrasound alone would not be the best choice given above, as the transvaginal combined with the abdominal ultrasound provides more information about the uterus and ovaries.

    DXA scan is used in the evaluation of bone mineral density and the screening and diagnosis of osteoporosis. The patient presentation in this case is note related to symptoms and findings related to bone density.

    Most women with epithelial ovarian cancer have pelvic or abdominal symptoms prior to their diagnosis. Women with ovarian pathology most typically present with symptoms related to the abdomen (77% of cases), gastrointestinal (70%), pain (58%), constitutional (50%), urinary (34%), and pelvic (26%).   The most common symptoms are bloating, urinary urgency or frequency, difficulty eating or feeling full, and/or abdominal or pelvic pain. Women who complain of these symptoms should be evaluated for a thorough history that includes frequency and severity of each symptom. Persistent abdominal distention, rather than fluctuating, can be more specific for ovarian pathology.

    A symptom index has been developed to aid clinicians in evaluating women for early symptoms of epithelial ovarian cancer (although it is not recommended at this time for clinical use). The symptom index is considered to be positive if a woman reports any of the following symptoms that are: new to her within the past year and occur more than 12 times per month: pelvic or abdominal pain, increased abdominal size or abdominal bloating, or difficulty eating or feeling full quickly. Physical exam findings suggestive of epithelial ovarian cancer include adnexal mass, abdominal ascites, “omental cake” or mass in the mid to left upper abdomen, pleural effusion, and groin or supraclavicular lymphadenopathy.

    Cancer of the ovary most often requires surgery for diagnosis, staging, and initial management. Most of the time, this involves a surgical total hysterectomy with salpingo-oophorectomy, where the doctor removes the ovaries, the fallopian tubes, and the uterus. Staging and initial surgical management should be performed by a gynecologic oncologist whenever possible to ensure better outcomes. Further treatment will depend a lot on the stage of the cancer, a woman’s age, and her other medical problems. Some women might not need any further treatment after surgery. Other women might need chemotherapy.

    Risk assessment for ovarian cancer depends upon age and family history. Ovarian cancer occurs most often in women ages 50 to 65. Ovarian cancer is the second most common gynecologic malignancy and the most common cause of death among women with gynecologic cancer. Most women are diagnosed at an advanced stage, while early stages of the disease are potentially curable. Screening programs for epithelial ovarian cancer using pelvic imaging or tumor markers have not yet been successful. The identification of epithelial ovarian cancer symptoms to aid early detection has become a focus of clinical research. Epithelial carcinoma of the ovaries, fallopian tubes, and peritoneum are clinically similar. Evidence suggests that these diseases have a common pathogenesis and may be initiated in the fallopian tubes. Almost 80 percent of women have lymph node or distant metastases at the time of diagnosis. Five-year survival rates for advanced disease are 19 to 32 percent.

    References

    http://www.uptodate.com/contents/screening-for-ovarian-cancer?source=machineLearning&search=ovarian+cancer&selectedTitle=2~150&sectionRank=1&anchor=H9#H9

    http://www.uptodate.com/contents/epithelial-carcinoma-of-the-ovary-fallopian-tube-and-peritoneum-clinical-features-and-diagnosis source=search_result&search=ovarian+cancer&selectedTitle=1~150

    http://www.uptodate.com/contents/overview-of-dual-energy-x-ray-absorptiometry?source=machineLearning&search=dexa+scan&selectedTitle=1~123&sectionRank=1&anchor=H5#H6

    http://www.uptodate.com/contents/search?search=clinical+presentation+ovarian+cancer&sp=0&searchType=PLAIN_TEXT&source=USER_INPUT&searchControl=TOP_PULLDOWN&searchOffset=

    http://www.uptodate.com/contents/evaluation-of-the-adult-with-abdominal-pain?source=search_result&search=abdominal+ct+scan&selectedTitle=1~150

  97. Question 97 of 220
    97. Question

    A 55-year-old female presents to the office as a new patient. You review her past medical history and discover that she suffered rheumatic fever as a child, but has otherwise been healthy. On exam you hear a diastolic rumble preceded by a opening snap at the heart’s apex. Which of the following heart murmurs best matches this patients findings?

    Correct

    The correct answer is B. The most common cause of mitral stenosis is rheumatic heart disease, which this patient has a history of. Also, mitral stenosis is a diastolic murmur. The mitral valve should close during diastole, but since the valve is insufficient the patient gets back flow into the left atria causing the murmur. An opening snap is also characteristic for mitral stenosis. Aortic regurgitation is also a diastolic murmur and can be a result of rheumatic fever, but it will not have an opening snap. The murmur will also be best heard at the left upper sternal border rather than the apex, and it will have a decrescendo or blowing sound. Mitral valve prolapse (MVP) is most common in patients with connective tissue disorders, such as Marfan’s or Ehlers Danlos. The characteristic murmur for MVP is a mid systolic ejection click heard best at the apex. Aortic stenosis is a systolic murmur heard best at the right upper sternal border and will have a crescendo-decrescendo sound. Some other exam findings you may find with aortic stenosis are radiation to the carotids, narrow pulse pressure, and a weak/delayed pulse. Patent ductus arteriosus or PDA is a machine-like murmur found in neonates and/or pediatric patients and is incorrect. It occurs when the ductus arteriosis does not close after birth. Symptoms include poor weight gain and shortness of breath.

    References

    Williams D.A. PANCE Prep Pearls. CreateSpace. 2014.

    Mitral stenosis. Chandrashekhar Y, Westaby S, Narula J. Lancet. 2009 Oct;374(9697):1271-83. Epub 2009 Sep 9.

    Incorrect

    The correct answer is B. The most common cause of mitral stenosis is rheumatic heart disease, which this patient has a history of. Also, mitral stenosis is a diastolic murmur. The mitral valve should close during diastole, but since the valve is insufficient the patient gets back flow into the left atria causing the murmur. An opening snap is also characteristic for mitral stenosis. Aortic regurgitation is also a diastolic murmur and can be a result of rheumatic fever, but it will not have an opening snap. The murmur will also be best heard at the left upper sternal border rather than the apex, and it will have a decrescendo or blowing sound. Mitral valve prolapse (MVP) is most common in patients with connective tissue disorders, such as Marfan’s or Ehlers Danlos. The characteristic murmur for MVP is a mid systolic ejection click heard best at the apex. Aortic stenosis is a systolic murmur heard best at the right upper sternal border and will have a crescendo-decrescendo sound. Some other exam findings you may find with aortic stenosis are radiation to the carotids, narrow pulse pressure, and a weak/delayed pulse. Patent ductus arteriosus or PDA is a machine-like murmur found in neonates and/or pediatric patients and is incorrect. It occurs when the ductus arteriosis does not close after birth. Symptoms include poor weight gain and shortness of breath.

    References

    Williams D.A. PANCE Prep Pearls. CreateSpace. 2014.

    Mitral stenosis. Chandrashekhar Y, Westaby S, Narula J. Lancet. 2009 Oct;374(9697):1271-83. Epub 2009 Sep 9.

  98. Question 98 of 220
    98. Question

    A 65-year-old man with a long history of alcohol abuse was admitted to the ICU for hepato-renal syndrome. At this stage of his disease his only treatment option is a liver transplant. The provider would like to calculate his MELD score to asses his mortality risk and priority for a liver transplant. Which of the following lab values is NOT needed to calculate his MELD score?

    Correct

    The correct answer is D. Serum bilirubin, sodium, and INR all are necessary to calculate the MELD score. Other information needed would be serum creatinine and whether or not the patient received dialysis in the past week. MELD scores are intended to help providers predict 3 month survival rates of patients who have end stage liver disease. The United Network of Organ Sharing adopted the MELD score as a way to prioritize patients awaiting liver transplantation. Historically, before the 1990s, priority was given to patients who had been on the transplant list the longest, rather than to those who were the most critical. A more equitable system was sought out though, and MELD scores allowed for organs to be allocated the sickest patients first. The scoring range is from 6-40, with 6 being less ill and 40 being severely ill. Labs should be checked every 7 days for a score greater or equal to 25, every 30 days for a score 24-19, every 90 days for a score 18-11, and every year for a score equal or less than 10. By adopting the MELD score as a way of distributing available organs for transplantation, some unintended consequences occurred. The number of patients who not only received a liver but also a renal transplant increased because of the emphasis on Creatinine in the equation. Women somehow became disadvantaged in the MELD system, and have a greater likelihood of dying or becoming too ill for transplantation. It is suggested that this may have to do with their lower Creatinine levels when compared with men. Additionally, MELD scoring removed race from impacting who receives a liver transplant and who is more likely to die while awaiting surgery.

    MELD and the Waiting List for Liver Transplant.Sutter Health CPMC. http://www.cpmc.org/advanced/liver/patients/topics/MELD.html#What%20is%20MELD?%20How%20will%20it%20be%20used?. Accessed May 11, 2016.

    https://optn.transplant.hrsa.gov/search-results?q=MELD (Accessed on January 24, 2014).

    The new liver allocation system: moving toward evidence-based transplantation policy.
    Freeman RB Jr, Wiesner RH, Harper A, McDiarmid SV, Lake J, Edwards E, Merion R, Wolfe R, Turcotte J, Teperman L, UNOS/OPTN Liver Disease Severity Score, UNOS/OPTN Liver and Intestine, and UNOS/OPTN Pediatric Transplantation Committees. Liver Transpl. 2002;8(9):851.

    Incorrect

    The correct answer is D. Serum bilirubin, sodium, and INR all are necessary to calculate the MELD score. Other information needed would be serum creatinine and whether or not the patient received dialysis in the past week. MELD scores are intended to help providers predict 3 month survival rates of patients who have end stage liver disease. The United Network of Organ Sharing adopted the MELD score as a way to prioritize patients awaiting liver transplantation. Historically, before the 1990s, priority was given to patients who had been on the transplant list the longest, rather than to those who were the most critical. A more equitable system was sought out though, and MELD scores allowed for organs to be allocated the sickest patients first. The scoring range is from 6-40, with 6 being less ill and 40 being severely ill. Labs should be checked every 7 days for a score greater or equal to 25, every 30 days for a score 24-19, every 90 days for a score 18-11, and every year for a score equal or less than 10. By adopting the MELD score as a way of distributing available organs for transplantation, some unintended consequences occurred. The number of patients who not only received a liver but also a renal transplant increased because of the emphasis on Creatinine in the equation. Women somehow became disadvantaged in the MELD system, and have a greater likelihood of dying or becoming too ill for transplantation. It is suggested that this may have to do with their lower Creatinine levels when compared with men. Additionally, MELD scoring removed race from impacting who receives a liver transplant and who is more likely to die while awaiting surgery.

    MELD and the Waiting List for Liver Transplant.Sutter Health CPMC. http://www.cpmc.org/advanced/liver/patients/topics/MELD.html#What%20is%20MELD?%20How%20will%20it%20be%20used?. Accessed May 11, 2016.

    https://optn.transplant.hrsa.gov/search-results?q=MELD (Accessed on January 24, 2014).

    The new liver allocation system: moving toward evidence-based transplantation policy.
    Freeman RB Jr, Wiesner RH, Harper A, McDiarmid SV, Lake J, Edwards E, Merion R, Wolfe R, Turcotte J, Teperman L, UNOS/OPTN Liver Disease Severity Score, UNOS/OPTN Liver and Intestine, and UNOS/OPTN Pediatric Transplantation Committees. Liver Transpl. 2002;8(9):851.

  99. Question 99 of 220
    99. Question

    A 45-year-old female presents to your clinic having recently returned from a vacation overseas. She complains of a right swollen calf. Her past medical history is remarkable for hypertension, (for which she takes Norvasc) obesity and type II diabetes. Her vital signs are stable and she denies any chest pain. You examine the leg and it is red and swollen medially. She has a palpable cord and a positive Homan’s sign. You should order which of the following?

    Correct

    The correct answer is C. This patient likely has a right lower extremity deep vein thrombosis, DVT. A palpable cord is indicative of a thrombosed vein. Homan’s sign is when the patient experiences pain in the calf or behind the knee with forced dorsiflexion of the foot. It should be noted that a positive Homan’s sign isn’t necessarily indicative of DVT and a negative sign does not completely rule it out either. Compression ultrasonography is highly sensitive and specific for DVT and is noninvasive. An abdominal CT might be indicated for bilateral lower extremity swelling in order to rule out something such as a mass. Bilateral lower extremity edema might occur if something were obstructing venous return from the legs. In this case however, the patient’s symptoms are unilateral. Plain films such as an x-ray of the lower leg would better inform you of an injury or deformity and would not be indicated here. Peripheral vascular resistance testing is done if there is suspected arterial or venous insufficiency. As a provider, you would see ulcerations, or dusky skin discoloration on your patient. The limb might be cool to the touch and pulseless. Some lower extremity DVTs have been confused with cellulitis due to the similar skin changes that can occur for both conditions. Certainly antibiotics are appropriate if cellulitis is present, but there is signifiant morbidity and mortality associated if a DVT is missed.

    References

    ABC of arterial and venous disease. Non-invasive methods of arterial and venous assessment.
    Donnelly R, Hinwood D, London NJ. BMJ. 2000;320(7236):698.

    Real-time ultrasound for diagnosis of symptomatic venous thrombosis and for screening of patients at risk: correlation with ascending conventional venography. Monreal M, Montserrat E, Salvador R, Bechini J, Donoso L, MaCallejas J, Foz M. Angiology. 1989;40(6):527.

    Incorrect

    The correct answer is C. This patient likely has a right lower extremity deep vein thrombosis, DVT. A palpable cord is indicative of a thrombosed vein. Homan’s sign is when the patient experiences pain in the calf or behind the knee with forced dorsiflexion of the foot. It should be noted that a positive Homan’s sign isn’t necessarily indicative of DVT and a negative sign does not completely rule it out either. Compression ultrasonography is highly sensitive and specific for DVT and is noninvasive. An abdominal CT might be indicated for bilateral lower extremity swelling in order to rule out something such as a mass. Bilateral lower extremity edema might occur if something were obstructing venous return from the legs. In this case however, the patient’s symptoms are unilateral. Plain films such as an x-ray of the lower leg would better inform you of an injury or deformity and would not be indicated here. Peripheral vascular resistance testing is done if there is suspected arterial or venous insufficiency. As a provider, you would see ulcerations, or dusky skin discoloration on your patient. The limb might be cool to the touch and pulseless. Some lower extremity DVTs have been confused with cellulitis due to the similar skin changes that can occur for both conditions. Certainly antibiotics are appropriate if cellulitis is present, but there is signifiant morbidity and mortality associated if a DVT is missed.

    References

    ABC of arterial and venous disease. Non-invasive methods of arterial and venous assessment.
    Donnelly R, Hinwood D, London NJ. BMJ. 2000;320(7236):698.

    Real-time ultrasound for diagnosis of symptomatic venous thrombosis and for screening of patients at risk: correlation with ascending conventional venography. Monreal M, Montserrat E, Salvador R, Bechini J, Donoso L, MaCallejas J, Foz M. Angiology. 1989;40(6):527.

  100. Question 100 of 220
    100. Question

    A 52-year-old male presents to your hospital with complaints of falls and weakness. He has had no loss of consciousness or obvious trauma. You immediately notice on physical exam though, that his abdomen is distended. He admits a longstanding history of alcohol abuse but lately has been “too sick to drink”. His blood pressure is low at 110/52 and he has a low grade fever of 38 degrees celsius. You obtain labs which reveal a leukocytosis. The patient’s WBC is 18,000. He has significant ascites on ultrasound examination. You suspect spontaneous bacterial peritonitis. The patient denies any allergies. The most appropriate initial treatment is?

    Correct

    The correct answer is B. Spontaneous bacterial peritonitis is an infection of the ascitic fluid present in the abdomen without any other identifiable surgical source. It almost always occurs in patients who have a history of cirrhosis. Typical symptoms include abdominal distention and pain, fever, general malaise, and often altered mentation due to metabolic encephalopathy. Gut pathogens are usually implicated including E.coli. Other causative agents include Klebsiella, Staph and Strep. Trimethoprim-sulfamethoxazole is an appropriate treatment for SBP prophylaxis but is not indicated in the case of active infection. The patient in this vignette requires hospitalization and close monitoring to evaluate for the possibility of decompensation. Cefotaxime is the best choice because it achieves high blood and ascitic fluid levels with IV dosing. Treatment should commence as quickly as possible once paracentesis has been performed to secure a fluid sample. The antibiotics can then be appropriately tailored to whatever culture results are obtained. Cipro does cover intestinal pathogens, but because it is an oral medication, it is not appropriate in this instance. SBP can be life threatening and is only indicated for patients who are asymptomatic. This particular patient is symptomatic given his weakness, fever, and abdominal pain. Levaquin can be used for SBP but is not the best choice because it can be nephrotoxic. The kidneys are already underperfused in the context of liver cirrhosis and there are better alternatives to Levaquin. Additionally, it does not have good penetration of ascitic fluid. Answer E is incorrect because once SBP has developed, patients have a greater mortality risk, when maintained on a nonselective beta-blocker. Nadolol should be discontinued to prevent complications including a prolonged hospital stay and precipitation and/or worsening of hepatorenal syndrome.

    References

    Cabrera J, Arroyo V, Ballesta AM, Rimola A, Gual J, Elena M, Rodes J. Aminoglycoside nephrotoxicity in cirrhosis. Value of urinary beta 2-microglobulin to discriminate functional renal failure from acute tubular damage. Gastroenterology. 1982;82(1):97.

    Terg R, Cobas S, Fassio E, Landeira G, Ríos B, Vasen W, Abecasis R, Ríos H, Guevara M. J Hepatol. Oral ciprofloxacin after a short course of intravenous ciprofloxacin in the treatment of pontaneous bacterial peritonitis: results of a multicenter, randomized study.
    2000;33(4):564.

    Mandorfer M, Bota S, Schwabl P, Bucsics T, Pfisterer N, Kruzik M, Hagmann M, Blacky A, Ferlitsch A, Sieghart W, Trauner M, Peck-Radosavljevic M, Reiberger T. Nonselectiveβblockers increase risk for hepatorenal syndrome and death in patients with cirrhosis and spontaneous bacterial peritonitis. Gastroenterology. 2014;146(7):1680.

    Incorrect

    The correct answer is B. Spontaneous bacterial peritonitis is an infection of the ascitic fluid present in the abdomen without any other identifiable surgical source. It almost always occurs in patients who have a history of cirrhosis. Typical symptoms include abdominal distention and pain, fever, general malaise, and often altered mentation due to metabolic encephalopathy. Gut pathogens are usually implicated including E.coli. Other causative agents include Klebsiella, Staph and Strep. Trimethoprim-sulfamethoxazole is an appropriate treatment for SBP prophylaxis but is not indicated in the case of active infection. The patient in this vignette requires hospitalization and close monitoring to evaluate for the possibility of decompensation. Cefotaxime is the best choice because it achieves high blood and ascitic fluid levels with IV dosing. Treatment should commence as quickly as possible once paracentesis has been performed to secure a fluid sample. The antibiotics can then be appropriately tailored to whatever culture results are obtained. Cipro does cover intestinal pathogens, but because it is an oral medication, it is not appropriate in this instance. SBP can be life threatening and is only indicated for patients who are asymptomatic. This particular patient is symptomatic given his weakness, fever, and abdominal pain. Levaquin can be used for SBP but is not the best choice because it can be nephrotoxic. The kidneys are already underperfused in the context of liver cirrhosis and there are better alternatives to Levaquin. Additionally, it does not have good penetration of ascitic fluid. Answer E is incorrect because once SBP has developed, patients have a greater mortality risk, when maintained on a nonselective beta-blocker. Nadolol should be discontinued to prevent complications including a prolonged hospital stay and precipitation and/or worsening of hepatorenal syndrome.

    References

    Cabrera J, Arroyo V, Ballesta AM, Rimola A, Gual J, Elena M, Rodes J. Aminoglycoside nephrotoxicity in cirrhosis. Value of urinary beta 2-microglobulin to discriminate functional renal failure from acute tubular damage. Gastroenterology. 1982;82(1):97.

    Terg R, Cobas S, Fassio E, Landeira G, Ríos B, Vasen W, Abecasis R, Ríos H, Guevara M. J Hepatol. Oral ciprofloxacin after a short course of intravenous ciprofloxacin in the treatment of pontaneous bacterial peritonitis: results of a multicenter, randomized study.
    2000;33(4):564.

    Mandorfer M, Bota S, Schwabl P, Bucsics T, Pfisterer N, Kruzik M, Hagmann M, Blacky A, Ferlitsch A, Sieghart W, Trauner M, Peck-Radosavljevic M, Reiberger T. Nonselectiveβblockers increase risk for hepatorenal syndrome and death in patients with cirrhosis and spontaneous bacterial peritonitis. Gastroenterology. 2014;146(7):1680.

  101. Question 101 of 220
    101. Question

    A 76 year old male with a history DM2, HTN, hyperlipidemia and CHF was admitted to the ICU for pneumonia and CHF exacerbation, requiring intubation and sedation. Overnight, the patient became progressively hypoxic (O2 sat 86%) un-responsive to 100% O2 administration with a PaO2/FiO2 ratio of 150mm Hg and BP 170/85. You order a CXR which reveals diffuse bilateral pulmonary infiltrates and is described as a “white out pattern” sparing the costophrenic angles, on report. Understanding the patients underlying pathology, which of the following treatment strategies will provide the most favorable outcome?

    Correct

    Answer is D. The above clinical vignette displays the classic scenario of Acute Respiratory Distress Syndrome (ARDS). The Berlin Definition defines ARDS as an acute (less than one week) onset of symptoms, bilateral opacities not explained by effusions classically described as a “white out pattern” sparing the costophrenic angels, respiratory failure refractory to 100% oxygen, which is not secondary to cardiac failure or fluid overload as proven by clinical examination or a PCWP > 18cm H2O, and a PaO2/FiO2 ratio Although acute respiratory distress syndrome is an inflammatory process defined as diffuse alveolar damage, IV steroids are not indicated as first line treatment and have not been found to be beneficial as standard of care in ARDs patients. 6

    While Furosemide is beneficial in the treatment of CHF exacerbation, the given CXR showed a “white out pattern” which spared the costophrenic angles. This description is not typical of a patient in congestive heart failure, steering the reader away from this pathology. The radiograph of a patient with cardiogenic CHF may show cardiomegaly, pulmonary hypertension and pleural effusions, unlike in non-cardiac cases of CHF, where those are absent. The edema is also diffuse and does NOT spare the periphery of the mid or upper lungs. 5

    Prone positioning IS a standard therapy in the treatment of ARDS due to recent evidence of improving oxygenation in patients with clinically diagnosed ARDS, however, a 12 to 24 hour stabilization period prior to the initiation of proning is recommended, along with maintaining prone ventilation for greater than 12 to 18 hours. 4, 6

    Discussion:
    Acute respiratory distress syndrome (ARDS) is a form of pulmonary edema, non-cardiac in origin which is characterized by diffuse alveolar injury and carries an average mortality rate of 30%. According to Sweeny and McAuley, “7% of patients in the intensive care unit and 16% of those receiving mechanical ventilation, have acute respiratory distress syndrome”. The cause ARDS can be classified into two broad categories: pulmonary or systemic (non-pulmonary in origin). Underneath the pulmonary category lies the two biggest risk factors for ARDS which also carry the highest mortality rate, pneumonia and aspiration. Other common causes include sepsis, trauma, multiple blood transfusions, pancreatitis, inhalation injury and certain types of drug toxicity. 1, 2, 6

    ARDS occurs in response to a primary illness, either a direct pulmonary or indirect extrapulmonary insult, which causes severe alveolar inflammation and injury to the epithelial of lung capillaries. Due to increased epithelial and endothelial permeability, cytokines, plasma proteins and macrophages flood the damaged alveolar epithelium, causing an inflammatory response which results in loss of surfactant production, epithelial ion channels, disrupted gas diffusion and increased permeability which all lead to alterations in vasomotor tone, the formation of microthrombi and subsequent pulmonary hypertension. As a result of pulmonary hypertension, there is an increase in right ventricular afterload which can be worsened by fluid overload and mechanical ventilation. 6

    In 2012, The Berlin Definition for acute respiratory distress syndrome was established. This states that in order for the diagnosis of ARDS to be met, ALL of the following criteria must be present: Within one week of a known clinical insult, or new/worsening respiratory symptoms, bilateral opacities not explained by effusions, lobar/lung collapse or nodules on chest radiography/CT, respiratory failure is not secondary to cardiac failure or fluid overload as proven by objective measures (ECHO, lung ultrasonography), and lastly, hypoxemia with severity determined by the PaO2/FiO2 ratio: Mild (200-300 mmHg), Moderate (100-200 mmHg), and Severe ( The management of ARDS is focused on assessing the underlying condition as well as specific targeted therapy. The current recommendations include protective mechanical ventilation with a low tidal volume of 6ml/kg IBW, Pplat

    References

    1. Ventilation with Lower Tidal Volumes as Compared with Traditional Tidal Volumes for Acute Lung Injury and the Acute Respiratory Distress Syndrome. New England Journal of Medicine. 2000.

    2. Kallet R, Campbell A, Dicker R, Mackersie R. TIDAL VOLUME (VT) AND WORK OF BREATHING (WOB) DURING LUNG-PROTECTIVE VENTILATION (LPV) IN ACUTE RESPIRATORY DISTRESS SYNDROME (ARDS). Critical Care Medicine. 2004.

    3. Williams D. PANCE Prep Pearls. Lexington, KY: CreateSpace; 2014.

    4. Marino PSutin K. The ICU Book. Philadelphia: Lippincott Williams & Wilkins; 2007.

    5. Singh V. Congestive Heart Failure Imaging: Overview, Radiography, Computed Tomography. Emedicinemedscapecom. 2016. Available at: http://emedicine.medscape.com/article/354666-overview. Accessed May 24, 2016.

    6. Mac Sweeny R. Acute respiratory distress syndrome. The Lancet. 2016.

    Incorrect

    Answer is D. The above clinical vignette displays the classic scenario of Acute Respiratory Distress Syndrome (ARDS). The Berlin Definition defines ARDS as an acute (less than one week) onset of symptoms, bilateral opacities not explained by effusions classically described as a “white out pattern” sparing the costophrenic angels, respiratory failure refractory to 100% oxygen, which is not secondary to cardiac failure or fluid overload as proven by clinical examination or a PCWP > 18cm H2O, and a PaO2/FiO2 ratio Although acute respiratory distress syndrome is an inflammatory process defined as diffuse alveolar damage, IV steroids are not indicated as first line treatment and have not been found to be beneficial as standard of care in ARDs patients. 6

    While Furosemide is beneficial in the treatment of CHF exacerbation, the given CXR showed a “white out pattern” which spared the costophrenic angles. This description is not typical of a patient in congestive heart failure, steering the reader away from this pathology. The radiograph of a patient with cardiogenic CHF may show cardiomegaly, pulmonary hypertension and pleural effusions, unlike in non-cardiac cases of CHF, where those are absent. The edema is also diffuse and does NOT spare the periphery of the mid or upper lungs. 5

    Prone positioning IS a standard therapy in the treatment of ARDS due to recent evidence of improving oxygenation in patients with clinically diagnosed ARDS, however, a 12 to 24 hour stabilization period prior to the initiation of proning is recommended, along with maintaining prone ventilation for greater than 12 to 18 hours. 4, 6

    Discussion:
    Acute respiratory distress syndrome (ARDS) is a form of pulmonary edema, non-cardiac in origin which is characterized by diffuse alveolar injury and carries an average mortality rate of 30%. According to Sweeny and McAuley, “7% of patients in the intensive care unit and 16% of those receiving mechanical ventilation, have acute respiratory distress syndrome”. The cause ARDS can be classified into two broad categories: pulmonary or systemic (non-pulmonary in origin). Underneath the pulmonary category lies the two biggest risk factors for ARDS which also carry the highest mortality rate, pneumonia and aspiration. Other common causes include sepsis, trauma, multiple blood transfusions, pancreatitis, inhalation injury and certain types of drug toxicity. 1, 2, 6

    ARDS occurs in response to a primary illness, either a direct pulmonary or indirect extrapulmonary insult, which causes severe alveolar inflammation and injury to the epithelial of lung capillaries. Due to increased epithelial and endothelial permeability, cytokines, plasma proteins and macrophages flood the damaged alveolar epithelium, causing an inflammatory response which results in loss of surfactant production, epithelial ion channels, disrupted gas diffusion and increased permeability which all lead to alterations in vasomotor tone, the formation of microthrombi and subsequent pulmonary hypertension. As a result of pulmonary hypertension, there is an increase in right ventricular afterload which can be worsened by fluid overload and mechanical ventilation. 6

    In 2012, The Berlin Definition for acute respiratory distress syndrome was established. This states that in order for the diagnosis of ARDS to be met, ALL of the following criteria must be present: Within one week of a known clinical insult, or new/worsening respiratory symptoms, bilateral opacities not explained by effusions, lobar/lung collapse or nodules on chest radiography/CT, respiratory failure is not secondary to cardiac failure or fluid overload as proven by objective measures (ECHO, lung ultrasonography), and lastly, hypoxemia with severity determined by the PaO2/FiO2 ratio: Mild (200-300 mmHg), Moderate (100-200 mmHg), and Severe ( The management of ARDS is focused on assessing the underlying condition as well as specific targeted therapy. The current recommendations include protective mechanical ventilation with a low tidal volume of 6ml/kg IBW, Pplat

    References

    1. Ventilation with Lower Tidal Volumes as Compared with Traditional Tidal Volumes for Acute Lung Injury and the Acute Respiratory Distress Syndrome. New England Journal of Medicine. 2000.

    2. Kallet R, Campbell A, Dicker R, Mackersie R. TIDAL VOLUME (VT) AND WORK OF BREATHING (WOB) DURING LUNG-PROTECTIVE VENTILATION (LPV) IN ACUTE RESPIRATORY DISTRESS SYNDROME (ARDS). Critical Care Medicine. 2004.

    3. Williams D. PANCE Prep Pearls. Lexington, KY: CreateSpace; 2014.

    4. Marino PSutin K. The ICU Book. Philadelphia: Lippincott Williams & Wilkins; 2007.

    5. Singh V. Congestive Heart Failure Imaging: Overview, Radiography, Computed Tomography. Emedicinemedscapecom. 2016. Available at: http://emedicine.medscape.com/article/354666-overview. Accessed May 24, 2016.

    6. Mac Sweeny R. Acute respiratory distress syndrome. The Lancet. 2016.

  102. Question 102 of 220
    102. Question

    A 21 year old patient has been admitted to the emergency room following a drug overdose. One of your assessments includes determining if the patient has acidosis and hypoxemia. Which of the following tests would you order?

    Correct

    A Comprehensive Metabolic Panel (CMP) refers to a group of blood tests that look at overall chemical balance and metabolism of the body. The blood tests look at organ function, such as that of the liver and kidneys. It also measures serum electrolyte levels, such as sodium, potassium, and chloride. Serum glucose levels and protein levels are also measured through a CMP. Abnormal results typically indicate the presence of conditions such as liver or kidney disease or diabetic complications.

    Blood Count with Differential (CBC w/diff) is used to look at overall health status and the presence of conditions such as anemia, leukemia, and infections. It can also be used to detect the presence of other life threatening conditions such as agranulocytosis, which is often associated with atypical antipsychotics, most commonly Clozaril. A CBC w/diff looks at various components of the blood including red and white blood cells, hematocrit and hemoglobin, and platelet count. Increases or decreases in cell counts may be indicative of an underlying disease process.

    Arterial Blood Gases (ABGs) are a means to measure the amount of oxygen and carbon dioxide in the blood, as well as the pH level to determine acidosis or alkalosis. The blood used for ABGs is taken directly from an artery. Red blood cells (RBCs) transport oxygen and carbon dioxide throughout the body. The blood passes through the lungs were oxygen and carbon dioxide are exchanged-the oxygen enters the blood through the lungs while the carbon dioxide flows out of the blood and into the lungs to be exhaled. Low levels of oxygen in the blood are indicative of hypoxemia. High pH levels are indicative of a process called alkalosis, while low pH levels are associated with acidosis. Abnormal levels of oxygen and carbon dioxide, as well as pH imbalance, can help detect the presents of liver and kidney disease as well as various respiratory diseases, such as asthma and COPD. You may also see alterations in any of the above as a result of drug overdose, chemical poisoning, hemorrhage, organ failure, or shock.

    A Lipid Profile is used to measure the level of cholesterol and triglycerides in the blood. Monitoring cholesterol levels can help determine the presence of plaque build-up in the arteries, which leads to narrowing and or occlusion of vessels. Elevated cholesterol and triglyceride levels do not cause any symptoms, which is why blood tests are so important, and are a known risk factor for development of heart disease. A lipid profile measures total cholesterol level and then further breaks it down to level of High Density Lipoprotein (HDL, “good” cholesterol), Low Density Lipoprotein (LDL, “bad” cholesterol), and triglycerides (fat in the blood).

    References

    Blood Gas Test. (n.d.). Retrieved May 23, 2016, from http://www.healthline.com/health/blood-gases#Overview1. Cholesterol test. (2016). Retrieved May 23, 2016, from http://www.mayoclinic.org/tests-procedures/cholesterol-test/home/ovc20169526?_ga=1.242328459.866837476.1462205784

    Complete blood count (CBC). (n.d.). Retrieved May 23, 2016, from http://www.mayoclinic.org/tests-procedures/complete-blood-count/basics/definition/prc-20014088. National Institute of Health, U.S. National Library of Medicine (2015).

    Comprehensive Metabolic Panel. Retrieved May 23, 2016, from https://www.nlm.nih.gov/medlineplus/ency/article/003468.htm

    Incorrect

    A Comprehensive Metabolic Panel (CMP) refers to a group of blood tests that look at overall chemical balance and metabolism of the body. The blood tests look at organ function, such as that of the liver and kidneys. It also measures serum electrolyte levels, such as sodium, potassium, and chloride. Serum glucose levels and protein levels are also measured through a CMP. Abnormal results typically indicate the presence of conditions such as liver or kidney disease or diabetic complications.

    Blood Count with Differential (CBC w/diff) is used to look at overall health status and the presence of conditions such as anemia, leukemia, and infections. It can also be used to detect the presence of other life threatening conditions such as agranulocytosis, which is often associated with atypical antipsychotics, most commonly Clozaril. A CBC w/diff looks at various components of the blood including red and white blood cells, hematocrit and hemoglobin, and platelet count. Increases or decreases in cell counts may be indicative of an underlying disease process.

    Arterial Blood Gases (ABGs) are a means to measure the amount of oxygen and carbon dioxide in the blood, as well as the pH level to determine acidosis or alkalosis. The blood used for ABGs is taken directly from an artery. Red blood cells (RBCs) transport oxygen and carbon dioxide throughout the body. The blood passes through the lungs were oxygen and carbon dioxide are exchanged-the oxygen enters the blood through the lungs while the carbon dioxide flows out of the blood and into the lungs to be exhaled. Low levels of oxygen in the blood are indicative of hypoxemia. High pH levels are indicative of a process called alkalosis, while low pH levels are associated with acidosis. Abnormal levels of oxygen and carbon dioxide, as well as pH imbalance, can help detect the presents of liver and kidney disease as well as various respiratory diseases, such as asthma and COPD. You may also see alterations in any of the above as a result of drug overdose, chemical poisoning, hemorrhage, organ failure, or shock.

    A Lipid Profile is used to measure the level of cholesterol and triglycerides in the blood. Monitoring cholesterol levels can help determine the presence of plaque build-up in the arteries, which leads to narrowing and or occlusion of vessels. Elevated cholesterol and triglyceride levels do not cause any symptoms, which is why blood tests are so important, and are a known risk factor for development of heart disease. A lipid profile measures total cholesterol level and then further breaks it down to level of High Density Lipoprotein (HDL, “good” cholesterol), Low Density Lipoprotein (LDL, “bad” cholesterol), and triglycerides (fat in the blood).

    References

    Blood Gas Test. (n.d.). Retrieved May 23, 2016, from http://www.healthline.com/health/blood-gases#Overview1. Cholesterol test. (2016). Retrieved May 23, 2016, from http://www.mayoclinic.org/tests-procedures/cholesterol-test/home/ovc20169526?_ga=1.242328459.866837476.1462205784

    Complete blood count (CBC). (n.d.). Retrieved May 23, 2016, from http://www.mayoclinic.org/tests-procedures/complete-blood-count/basics/definition/prc-20014088. National Institute of Health, U.S. National Library of Medicine (2015).

    Comprehensive Metabolic Panel. Retrieved May 23, 2016, from https://www.nlm.nih.gov/medlineplus/ency/article/003468.htm

  103. Question 103 of 220
    103. Question

    image 4

    A 36 year old female is brought to the emergency department after being found “wandering in her group home” and acting very confused.  She has a history of diabetes mellitus Type I, gastroparesis, neuropathy, depression, anxiety, and GERD. The above EKG is obtained.  The abnormalities on this EKG are consistent with which diagnosis?

    Correct

    Answer:  Hypothermia.  Leads V4 and V5 demonstrate classic Osborne (J) waves, defined as an additional upward deflection to the terminal portion of the QRS complex.  Other findings on this EKG consistent with hypothermia are baseline artifact related to shivering, inverted T waves and prolonged QT interval.  Atrial flutter is characterized by the classic “saw tooth pattern” and should have a faster rate than this EKG with atrial rates of 240-400 bpm.  Hyperkalemia EKG findings are usually:  bradycardia, flattened P waves, widened QRS complex, biphasic QRS-T complexes, upright peaked T waves and ST elevations.  Ischemia would be represented by ST depressions and pulmonary embolus EKG findings are usually tachycardia and/or the “S1Q3T3” pattern.

    Hypothermia is generally defined as a core temperature of less than 35 degrees C or 95.8 degrees F with severe hypothermia at a temp of less than 82.4 degrees F.  Rectal or esophageal temperatures are most accurate.  In addition to a prolonged exposure to a cold environment (primary hypothermia), secondary causes of hypothermia include endocrine disorders, spinal cord transection leading to neurogenic shock, hypothalamic dysfunction, toxins, diabetic or alcoholic ketoacidosis, and sepsis.  This particular patient had underlying diabetic ketoacidosis with a pH of 7.10 and an anion gap of 28.

    History and physical exam may reveal irritability, lethargy to confusion to coma, reduced respiratory rate, bradycardia, and/or signs of frostbite (hyperemia, edema, vesicles, hemorrhagic blisters, or necrosis).  EKG findings include Osborne waves (present in approximately 80% of cases), baseline artifacts, QT prolongation, bradycardia, atrial fibrillation with slow ventricular response, premature ventricular beats, ventricular tachycardia or fibrillation, and asystole.  The Osborne, or “J”, wave size usually corresponds to the severity of the hypothermia and will diminish in size as the patient’s core temperature is warmed.  Ventricular tachycardia and fibrillation are most commonly found during the rewarming process.

    Treatment of hypothermia is targeted at rewarming as well as addressing any potential secondary causes that may be responsible.  Rewarming should occur somewhat slowly to avoid inducing potential arrhythmias unless the patient is profoundly hypothermic, in which case a more aggressive rewarming strategy has been suggested to get the patient to 30-32 degrees C (86-89.6 degrees F).  Rewarming techniques once the patient has been removed from a cold environment include warm blankets or more commonly a forced warm air blanket found in most hospitals.  With more profound hypothermia, more invasive techniques are required.  These include:  warm IV fluids, humidified oxygen, GI, peritoneal, pleural or bladder lavage, and even cardiac bypass. Frostbite should be rapidly rewarmed in warm water baths once the chance of re-freezing has passed.

    References:

    Slovis C. ABC of clinical electrocardiography: Conditions not primarily affecting the heart. Bmj 2002;324(7349):1320–1323.

    Longo DL, Kasper D, Jameson J, Fauci A, Hauser S, Loscalzo J. Harrison’s Principles of Internal Medicine. New York: McGraw-Hill; 2012.

    Tintinalli JE, Kelen GD, Stapczynski JS. Emergency Medicine: a Comprehensive Study Guide. New York: McGraw-Hill, Medical Pub. Division; 2004.

    Accidental hypothermia. In DynaMed Plus® – the next-generation clinical information resource designed to decrease time to answer. DynaMed Plus. Available at: http://www.dynamed.com/home/. Accessed March 16, 2016.

    Incorrect

    Answer:  Hypothermia.  Leads V4 and V5 demonstrate classic Osborne (J) waves, defined as an additional upward deflection to the terminal portion of the QRS complex.  Other findings on this EKG consistent with hypothermia are baseline artifact related to shivering, inverted T waves and prolonged QT interval.  Atrial flutter is characterized by the classic “saw tooth pattern” and should have a faster rate than this EKG with atrial rates of 240-400 bpm.  Hyperkalemia EKG findings are usually:  bradycardia, flattened P waves, widened QRS complex, biphasic QRS-T complexes, upright peaked T waves and ST elevations.  Ischemia would be represented by ST depressions and pulmonary embolus EKG findings are usually tachycardia and/or the “S1Q3T3” pattern.

    Hypothermia is generally defined as a core temperature of less than 35 degrees C or 95.8 degrees F with severe hypothermia at a temp of less than 82.4 degrees F.  Rectal or esophageal temperatures are most accurate.  In addition to a prolonged exposure to a cold environment (primary hypothermia), secondary causes of hypothermia include endocrine disorders, spinal cord transection leading to neurogenic shock, hypothalamic dysfunction, toxins, diabetic or alcoholic ketoacidosis, and sepsis.  This particular patient had underlying diabetic ketoacidosis with a pH of 7.10 and an anion gap of 28.

    History and physical exam may reveal irritability, lethargy to confusion to coma, reduced respiratory rate, bradycardia, and/or signs of frostbite (hyperemia, edema, vesicles, hemorrhagic blisters, or necrosis).  EKG findings include Osborne waves (present in approximately 80% of cases), baseline artifacts, QT prolongation, bradycardia, atrial fibrillation with slow ventricular response, premature ventricular beats, ventricular tachycardia or fibrillation, and asystole.  The Osborne, or “J”, wave size usually corresponds to the severity of the hypothermia and will diminish in size as the patient’s core temperature is warmed.  Ventricular tachycardia and fibrillation are most commonly found during the rewarming process.

    Treatment of hypothermia is targeted at rewarming as well as addressing any potential secondary causes that may be responsible.  Rewarming should occur somewhat slowly to avoid inducing potential arrhythmias unless the patient is profoundly hypothermic, in which case a more aggressive rewarming strategy has been suggested to get the patient to 30-32 degrees C (86-89.6 degrees F).  Rewarming techniques once the patient has been removed from a cold environment include warm blankets or more commonly a forced warm air blanket found in most hospitals.  With more profound hypothermia, more invasive techniques are required.  These include:  warm IV fluids, humidified oxygen, GI, peritoneal, pleural or bladder lavage, and even cardiac bypass. Frostbite should be rapidly rewarmed in warm water baths once the chance of re-freezing has passed.

    References:

    Slovis C. ABC of clinical electrocardiography: Conditions not primarily affecting the heart. Bmj 2002;324(7349):1320–1323.

    Longo DL, Kasper D, Jameson J, Fauci A, Hauser S, Loscalzo J. Harrison’s Principles of Internal Medicine. New York: McGraw-Hill; 2012.

    Tintinalli JE, Kelen GD, Stapczynski JS. Emergency Medicine: a Comprehensive Study Guide. New York: McGraw-Hill, Medical Pub. Division; 2004.

    Accidental hypothermia. In DynaMed Plus® – the next-generation clinical information resource designed to decrease time to answer. DynaMed Plus. Available at: http://www.dynamed.com/home/. Accessed March 16, 2016.

  104. Question 104 of 220
    104. Question

    image 3

    In addition to a right bundle branch block (RBBB), what other abnormality is noted on the EKG above?

     

    Correct

    Answer:  Second-degree AV block – Mobitz Type II.  This EKG shows a fixed prolongation of the PR interval with a non-conducted P wave resulting in an absence of a QRS complex.  A first-degree block could appear similar but all P waves should result in a corresponding QRS.  Mobitz I blocks have a PR interval which becomes more prolonged with each successive beat, eventually resulting in a missed QRS complex.  Third-degree, or complete, heart block shows a constant interval between P waves but no association to the QRS complexes.

    These various blocks are disorders of the electrical conduction through the atrioventricular node caused by numerous potential etiologies including:  congenital anomalies, ischemia, infectious/inflammatory diseases, metabolic abnormalities, medications, cardiac procedures, and post-radiation, to name a few.

    The patient may present with complaints of dizziness, syncope, fatigue, chest pain, dyspnea or be completely asymptomatic.  History should include inquiry about current medications, tick or other possible infectious disease exposures, angina-like symptoms, or symptoms consistent with previously undiagnosed conditions such as:  sarcoidosis, amyloidosis, lupus, rheumatoid arthritis, scleroderma, and even sleep apnea.

    The diagnosis is usually made with a 12 lead EKG, however, an intermittent block might be more likely diagnosed using a Holter or event monitor, exercise testing, or with an electrophysiological (EP) study.  A first-degree block has a PR interval of > 200 milliseconds and every P is followed by a QRS.  Second-degree type I blocks have either a progressive lengthening of the PR interval eventually resulting in a missed QRS complex (Wenckebach) or a fixed PR interval with 2 P waves before a QRS is conducted.  Second-degree type II blocks usually have a fixed prolongation of the PR interval with an intermittent non-conducted QRS complex, however, they can also have a 2:1 or greater pattern similar to the second-degree type I mention but usually found in conjunction with an intraventricular conduction delay.  Third-degree blocks are characterized by a complete absence of AV node or His-Purkinje system conduction of the P wave impulse.  This requires an area from within the AV node or the ventricle itself to initiate the QRS complex resulting in a complete disassociation between the P and QRS waves.  Mixed second-degree type II and third-degree can also occur.

    Treatment involves addressing underlying systemic causes if any are present.  Temporary or permanent pacing is indicated for almost all patients with a second-degree type II or higher with biventricular pacing for those also associated with heart failure.  Atropine may be of short-term benefit.

    References:

    Zipes DP. Second-degree atrioventricular block. Circulation 1979;60(3):465–472.

    Longo DL, Kasper D, Jameson J, Fauci A, Hauser S, Loscalzo J. Harrison’s Principles of Internal Medicine. New York: McGraw-Hill; 2012.

    Atrioventricular conduction disorders. In DynaMed Plus® – the next-generation clinical information resource designed to decrease time to answer. DynaMed Plus. Available at: http://www.dynamed.com/home/. Accessed March 10, 2016.

    Incorrect

    Answer:  Second-degree AV block – Mobitz Type II.  This EKG shows a fixed prolongation of the PR interval with a non-conducted P wave resulting in an absence of a QRS complex.  A first-degree block could appear similar but all P waves should result in a corresponding QRS.  Mobitz I blocks have a PR interval which becomes more prolonged with each successive beat, eventually resulting in a missed QRS complex.  Third-degree, or complete, heart block shows a constant interval between P waves but no association to the QRS complexes.

    These various blocks are disorders of the electrical conduction through the atrioventricular node caused by numerous potential etiologies including:  congenital anomalies, ischemia, infectious/inflammatory diseases, metabolic abnormalities, medications, cardiac procedures, and post-radiation, to name a few.

    The patient may present with complaints of dizziness, syncope, fatigue, chest pain, dyspnea or be completely asymptomatic.  History should include inquiry about current medications, tick or other possible infectious disease exposures, angina-like symptoms, or symptoms consistent with previously undiagnosed conditions such as:  sarcoidosis, amyloidosis, lupus, rheumatoid arthritis, scleroderma, and even sleep apnea.

    The diagnosis is usually made with a 12 lead EKG, however, an intermittent block might be more likely diagnosed using a Holter or event monitor, exercise testing, or with an electrophysiological (EP) study.  A first-degree block has a PR interval of > 200 milliseconds and every P is followed by a QRS.  Second-degree type I blocks have either a progressive lengthening of the PR interval eventually resulting in a missed QRS complex (Wenckebach) or a fixed PR interval with 2 P waves before a QRS is conducted.  Second-degree type II blocks usually have a fixed prolongation of the PR interval with an intermittent non-conducted QRS complex, however, they can also have a 2:1 or greater pattern similar to the second-degree type I mention but usually found in conjunction with an intraventricular conduction delay.  Third-degree blocks are characterized by a complete absence of AV node or His-Purkinje system conduction of the P wave impulse.  This requires an area from within the AV node or the ventricle itself to initiate the QRS complex resulting in a complete disassociation between the P and QRS waves.  Mixed second-degree type II and third-degree can also occur.

    Treatment involves addressing underlying systemic causes if any are present.  Temporary or permanent pacing is indicated for almost all patients with a second-degree type II or higher with biventricular pacing for those also associated with heart failure.  Atropine may be of short-term benefit.

    References:

    Zipes DP. Second-degree atrioventricular block. Circulation 1979;60(3):465–472.

    Longo DL, Kasper D, Jameson J, Fauci A, Hauser S, Loscalzo J. Harrison’s Principles of Internal Medicine. New York: McGraw-Hill; 2012.

    Atrioventricular conduction disorders. In DynaMed Plus® – the next-generation clinical information resource designed to decrease time to answer. DynaMed Plus. Available at: http://www.dynamed.com/home/. Accessed March 10, 2016.

  105. Question 105 of 220
    105. Question

    A 20 year-old female with a history of smoking presents to the office with anorexia, intermittent crampy abdominal pain, diarrhea and a painful perianal mass for the past 3-4 weeks. She also reports frequent right sided abdominal pain. She denies any constitutional symptoms other than a generalized feeling of malaise. Of the following choices, which is the most likely diagnosis?

    Correct

    Inflammatory bowel disease (IBD), specifically Crohn’s disease, is a complex, chronic autoimmune disease of unknown etiology characterized by relapsing and remitting inflammation anywhere in the GI tract from mouth to anus. The disease has a bimodal age peak: 15-30 and 60-80 years. Symptoms of Crohn’s generally include anorexia, malnutrition, diarrhea (from epithelial damage), abdominal pains and cramps, possibly obstructive symptoms and sometimes perianal manifestations (eg, skin tags, abscess, fistula, anal ulcers, anal stenosis). Ulcerative colitis (UC) is the other manifestation of IBD, but there is generally no perianal involvement.

    Irritable bowel syndrome (IBS) is a functional disorder, marked by nonspecific abdominal symptoms (eg, pain, flatulence, cramps) in association with altered bowel habits (eg, constipation, diarrhea or both). Unlike IBD, IBS has no morphologic or biochemical abnormalities. Instead, it is thought to result from disturbance of the brain/gut interaction, resulting in abnormal processing of physiologic stimuli, which triggers altered motility, increased contractility/spasticity, and visceral hypersensitivity, and is associated with a reduced intestinal volume tolerance. There is no perianal disease component to this disorder.

    Celiac disease is an immunologic disorder of the small bowel, resulting in enhanced immunogenic response to gliadin (alcohol-soluble portion of gluten). This results in intense local inflammation of the villi of the small intestine, followed by villous atrophy and significantly decreased absorptive surface area. While many cases are asymptomatic, symptoms can include diarrhea, fatigue, weight loss, abdominal distention, excessive flatus, or large, bulky, foul smelling stools. Like IBS, there is no associated perianal manifestation of this disease, as there is in Crohn’s. Symptoms are related to dietary gluten intake. Management involves eliminating gluten from the diet.

    Lactose intolerance is the inability to digest milk and some dairy products, leading to abdominal bloating, cramping and diarrhea. The enzyme lactase is generally present in the small intestine at birth, but levels start to decline by the age of 5 years, and 95% of birth lactase levels are lost by early childhood. Lactase continues to decrease with aging, and as a result patients can become increasingly symptomatic. Treatment is aimed at limiting consumption of milk and other lactose-containing foods. There is no perianal manifestation of lactose intolerance.

    References

    1. Lichtenstein GR, et al. Practice Parameters Committee of American College of Gastroenterology. Management of Crohn’s disease in adults. Am J Gastroenterol 2009; 104:465.
    2. Kamp EJ, et al. Irritable Bowel Syndrome and Microscopic Colitis: A Systematic Review and Meta-analysis. Clin Gastroenterol Hepatol 2016; 14:659.
    3. Mattar, et al. Lactose intolerance: diagnosis, genetic, and clinical factors. Clin Exp Gastroenterol. 2012; 5: 113–121.

    Incorrect

    Inflammatory bowel disease (IBD), specifically Crohn’s disease, is a complex, chronic autoimmune disease of unknown etiology characterized by relapsing and remitting inflammation anywhere in the GI tract from mouth to anus. The disease has a bimodal age peak: 15-30 and 60-80 years. Symptoms of Crohn’s generally include anorexia, malnutrition, diarrhea (from epithelial damage), abdominal pains and cramps, possibly obstructive symptoms and sometimes perianal manifestations (eg, skin tags, abscess, fistula, anal ulcers, anal stenosis). Ulcerative colitis (UC) is the other manifestation of IBD, but there is generally no perianal involvement.

    Irritable bowel syndrome (IBS) is a functional disorder, marked by nonspecific abdominal symptoms (eg, pain, flatulence, cramps) in association with altered bowel habits (eg, constipation, diarrhea or both). Unlike IBD, IBS has no morphologic or biochemical abnormalities. Instead, it is thought to result from disturbance of the brain/gut interaction, resulting in abnormal processing of physiologic stimuli, which triggers altered motility, increased contractility/spasticity, and visceral hypersensitivity, and is associated with a reduced intestinal volume tolerance. There is no perianal disease component to this disorder.

    Celiac disease is an immunologic disorder of the small bowel, resulting in enhanced immunogenic response to gliadin (alcohol-soluble portion of gluten). This results in intense local inflammation of the villi of the small intestine, followed by villous atrophy and significantly decreased absorptive surface area. While many cases are asymptomatic, symptoms can include diarrhea, fatigue, weight loss, abdominal distention, excessive flatus, or large, bulky, foul smelling stools. Like IBS, there is no associated perianal manifestation of this disease, as there is in Crohn’s. Symptoms are related to dietary gluten intake. Management involves eliminating gluten from the diet.

    Lactose intolerance is the inability to digest milk and some dairy products, leading to abdominal bloating, cramping and diarrhea. The enzyme lactase is generally present in the small intestine at birth, but levels start to decline by the age of 5 years, and 95% of birth lactase levels are lost by early childhood. Lactase continues to decrease with aging, and as a result patients can become increasingly symptomatic. Treatment is aimed at limiting consumption of milk and other lactose-containing foods. There is no perianal manifestation of lactose intolerance.

    References

    1. Lichtenstein GR, et al. Practice Parameters Committee of American College of Gastroenterology. Management of Crohn’s disease in adults. Am J Gastroenterol 2009; 104:465.
    2. Kamp EJ, et al. Irritable Bowel Syndrome and Microscopic Colitis: A Systematic Review and Meta-analysis. Clin Gastroenterol Hepatol 2016; 14:659.
    3. Mattar, et al. Lactose intolerance: diagnosis, genetic, and clinical factors. Clin Exp Gastroenterol. 2012; 5: 113–121.

  106. Question 106 of 220
    106. Question

    A 65 year old male who was treated for a complicated urinary tract infection one month ago with a 10-day course of antibiotics presents to your clinic complaining of profuse diarrhea, nausea, anorexia and mild abdominal cramping for the past 2 days. Labs reveal mild dehydration, and leukocytosis of 11,000. You suspect Clostridium difficile infection. Of the following, which is the most appropriate next step in managing the patient?

    Correct

    C. diff., or pseudomembranous colitis, presents a wide clinical spectrum from mild diarrhea to fulminant and potentially fatal colitis. The infection is most often associated with antibiotic use and resulting overgrowth of the commensal anaerobe C. difficile. It can occur from a few days to up to 10 weeks after antibiotic use. For mild symptoms, treatment consists of, first, discontinuing the offending agent; then, initiation of a 10-14 day course of oral antibiotics. Metronidazole 250mg-500mg po TID or vancomycin 125mg-250mg QID are first line agents. For severe cases or patients with significant comorbidities, IV metronidazole may be used.

    Antidiarrheal agents are contraindicated as long as the infection is uncontrolled.

    The indications for operative treatment of C. diff. are fulminant disease, toxic megacolon, colonic perforation or failure to improve with medical therapy.

    References

    1. Bagdasarian N, et al. Diagnosis and treatment of Clostridium difficile in adults: a systematic review. JAMA 2015; 313:398.

    2. Cohen SH, et al. Clinical practice guidelines for Clostridium difficile infection in adults: 2010 update by the society for healthcare epidemiology of America (SHEA) and the infectious diseases society of America (IDSA). Infect Control Hosp Epidemiol 2010; 31:431.

    Incorrect

    C. diff., or pseudomembranous colitis, presents a wide clinical spectrum from mild diarrhea to fulminant and potentially fatal colitis. The infection is most often associated with antibiotic use and resulting overgrowth of the commensal anaerobe C. difficile. It can occur from a few days to up to 10 weeks after antibiotic use. For mild symptoms, treatment consists of, first, discontinuing the offending agent; then, initiation of a 10-14 day course of oral antibiotics. Metronidazole 250mg-500mg po TID or vancomycin 125mg-250mg QID are first line agents. For severe cases or patients with significant comorbidities, IV metronidazole may be used.

    Antidiarrheal agents are contraindicated as long as the infection is uncontrolled.

    The indications for operative treatment of C. diff. are fulminant disease, toxic megacolon, colonic perforation or failure to improve with medical therapy.

    References

    1. Bagdasarian N, et al. Diagnosis and treatment of Clostridium difficile in adults: a systematic review. JAMA 2015; 313:398.

    2. Cohen SH, et al. Clinical practice guidelines for Clostridium difficile infection in adults: 2010 update by the society for healthcare epidemiology of America (SHEA) and the infectious diseases society of America (IDSA). Infect Control Hosp Epidemiol 2010; 31:431.

  107. Question 107 of 220
    107. Question

    A patient presents to your clinic reporting acute onset pain following a large, hard bowel movement earlier in the day, with bright red blood per rectum dripping into the water. There is no prolapse, and no mass that she can feel at the anal outlet. Since the bowel movement, she has not been able to sit without discomfort, however. Which of the following treatments would not be appropriate for this patient?

    Correct

    This patient has most likely developed an anal fissure. A fissure is a longitudinal tear/wound in the anus, typically located in the posterior midline, and is associated with high anal sphincter tone. The main risk factors for fissure are constipation and chronic diarrhea, but fissure can also occur with normal bowel movements. Acute fissure will heal in 40-60% of patients with appropriate improved stool management and decreasing anal sphincter tone. A daily fiber supplement will help keep stool soft, bulky and easy to pass. Sitz baths will provide comfort and relaxation of the sphincter muscles. Topical nitroglycerine (0.2%) is a vasodilator that acts to increase bloodflow to the wound to aid in healing, but also has a relaxing effect on the sphincter muscles.

    Rubber band ligation is used to treat Grade I – II hemorrhoids, and has no role in the treatment of fissures.

    References

    1.  Zaghiyan KN, Fleshner P. Anal fissure. Clin Colon Rectal Surg 2011; 24:22.Perry WB, et al. Practice parameters for the management of anal fissures (3rd revision). Dis Colon Rectum 2010; 53:1110.

    2. Perry WB, et al. Practice parameters for the management of anal fissures (3rd revision). Dis Colon Rectum 2010; 53:1110.

    Incorrect

    This patient has most likely developed an anal fissure. A fissure is a longitudinal tear/wound in the anus, typically located in the posterior midline, and is associated with high anal sphincter tone. The main risk factors for fissure are constipation and chronic diarrhea, but fissure can also occur with normal bowel movements. Acute fissure will heal in 40-60% of patients with appropriate improved stool management and decreasing anal sphincter tone. A daily fiber supplement will help keep stool soft, bulky and easy to pass. Sitz baths will provide comfort and relaxation of the sphincter muscles. Topical nitroglycerine (0.2%) is a vasodilator that acts to increase bloodflow to the wound to aid in healing, but also has a relaxing effect on the sphincter muscles.

    Rubber band ligation is used to treat Grade I – II hemorrhoids, and has no role in the treatment of fissures.

    References

    1.  Zaghiyan KN, Fleshner P. Anal fissure. Clin Colon Rectal Surg 2011; 24:22.Perry WB, et al. Practice parameters for the management of anal fissures (3rd revision). Dis Colon Rectum 2010; 53:1110.

    2. Perry WB, et al. Practice parameters for the management of anal fissures (3rd revision). Dis Colon Rectum 2010; 53:1110.

  108. Question 108 of 220
    108. Question

    An otherwise healthy 73 year old female has recently undergone resection of part of her left colon due to a diagnosis of adenocarcinoma. She recovered well from the surgery, and readily participates in routine oncologic follow-up with your office. Which of the following blood tests will be most helpful in monitoring her for disease recurrence?

    Correct

    Carcinoembryonic antigen (CEA) is used to monitor people who have been diagnosed with colorectal cancer. An initial CEA test is typically ordered prior to treatment as a “baseline” value. If the level is elevated, then the test can be used to monitor a patient’s response to therapy and to determine whether the cancer has progressed or recurred. The test in not diagnostic, but can be used as an adjunct in colorectal cancer diagnosis, staging and post-treatment surveillance.

    A complete blood count (CBC) will provide useful information regarding white and red blood cell counts, but is not routinely used to monitor recurrence of colorectal cancer; it can provide information on anemia, infection, inflammation or a bleeding disorder. Cancer antigen 125 (CA-125) is used as a marker for cancers such as ovarian, breast or cervical. Alpha-fetoprotein (AFP) is used to monitor for hepatocellular carcinoma, testicular cancer, pancreatic cancer or gastric cancer.

     References

    1.https://www.fascrs.org/sites/default/files/downloads/publication/practice_guideline_for_the_surveillance_of

    Incorrect

    Carcinoembryonic antigen (CEA) is used to monitor people who have been diagnosed with colorectal cancer. An initial CEA test is typically ordered prior to treatment as a “baseline” value. If the level is elevated, then the test can be used to monitor a patient’s response to therapy and to determine whether the cancer has progressed or recurred. The test in not diagnostic, but can be used as an adjunct in colorectal cancer diagnosis, staging and post-treatment surveillance.

    A complete blood count (CBC) will provide useful information regarding white and red blood cell counts, but is not routinely used to monitor recurrence of colorectal cancer; it can provide information on anemia, infection, inflammation or a bleeding disorder. Cancer antigen 125 (CA-125) is used as a marker for cancers such as ovarian, breast or cervical. Alpha-fetoprotein (AFP) is used to monitor for hepatocellular carcinoma, testicular cancer, pancreatic cancer or gastric cancer.

     References

    1.https://www.fascrs.org/sites/default/files/downloads/publication/practice_guideline_for_the_surveillance_of.1.pdf

  109. Question 109 of 220
    109. Question

    At an annual physical exam, a 60 year old female with a history of alcoholism and ascites mentions concern about hematemesis. While taking her history, you learn that this has happened a couple times in the past year, but has always been a small amount of blood, and has spontaneously resolved within 1 day. The patient is clinically stable at the moment. The last time she remembers hematemesis was 4 or 5 months ago. Which of the following is the most appropriate next step in caring for this patient?

    Correct

    Esophageal varices are dilations of the veins of the esophagus, generally at the distal end. The most common cause in adults is cirrhosis, either from alcohol abuse or chronic viral hepatitis. Varices are generally asymptomatic until they bleed, at which point they can become life-threatening. In a stable patient with no recent bleeding, upper endoscopic exam is the preferred method to screen for esophageal varices, looking for dilated veins, red streaks and red spots, all of which may indicate a significant risk of bleeding.

    In this patient, evidence of recent bleeding is unlikely given her history of no symptoms for the past 4 or 5 months. Treatment can be undertaken to prevent further bleeding. Use of NSAIDS can exacerbate bleeding, so should be avoided. Beta blockers can be used to reduce pressure in the portal vein, thereby decreasing the risk of bleeding.

    Octreotide is a pharmacologic vasoconstrictor, and would be a treatment option if the patient were actively bleeding. It is commonly used with endoscopic therapy to slow the flow of blood from internal organs to the portal vein. Octreotide is typically continued for 5 days following a bleeding episode.

    References

    1. Garcia-Tsao G, et al. Prevention and management of gastroesophageal varices and variceal hemorrhage in cirrhosis. Hepatology 2007; 46:922.

    2. Sanyal AJ, Shiffman ML. The pharmacologic treatment of portal hypertension. Annu Rev Gastrointest Pharmacol 1996; :242.

    3. Bañares R, et al. Endoscopic treatment versus endoscopic plus pharmacologic treatment for acute variceal bleeding: a meta-analysis. Hepatology 2002; 35:609.

    Incorrect

    Esophageal varices are dilations of the veins of the esophagus, generally at the distal end. The most common cause in adults is cirrhosis, either from alcohol abuse or chronic viral hepatitis. Varices are generally asymptomatic until they bleed, at which point they can become life-threatening. In a stable patient with no recent bleeding, upper endoscopic exam is the preferred method to screen for esophageal varices, looking for dilated veins, red streaks and red spots, all of which may indicate a significant risk of bleeding.

    In this patient, evidence of recent bleeding is unlikely given her history of no symptoms for the past 4 or 5 months. Treatment can be undertaken to prevent further bleeding. Use of NSAIDS can exacerbate bleeding, so should be avoided. Beta blockers can be used to reduce pressure in the portal vein, thereby decreasing the risk of bleeding.

    Octreotide is a pharmacologic vasoconstrictor, and would be a treatment option if the patient were actively bleeding. It is commonly used with endoscopic therapy to slow the flow of blood from internal organs to the portal vein. Octreotide is typically continued for 5 days following a bleeding episode.

    References

    1. Garcia-Tsao G, et al. Prevention and management of gastroesophageal varices and variceal hemorrhage in cirrhosis. Hepatology 2007; 46:922.

    2. Sanyal AJ, Shiffman ML. The pharmacologic treatment of portal hypertension. Annu Rev Gastrointest Pharmacol 1996; :242.

    3. Bañares R, et al. Endoscopic treatment versus endoscopic plus pharmacologic treatment for acute variceal bleeding: a meta-analysis. Hepatology 2002; 35:609.

  110. Question 110 of 220
    110. Question

    A 71 year old overweight female with PMHx of well-controlled hypertension is admitted to the surgical floor following colon cancer resection surgery earlier in the day. She is recovering well. She is not yet ambulatory. Based on this information, which of the following is not indicated for DVT prophylaxis for this patient?

    Correct

    Early mobilization of patients as soon as possible after surgery is thought to reduce the chances of venous thrombosis because it stimulates calf muscles and puts pressure on the calf and leg veins, thus discouraging stasis and venous pooling of blood in the lower extremities. Likewise, sequential compression with pneumatic leg compression enhances blood flow in the deep veins of the legs. Lovenox is a low molecular weight heparin. It is an excellent therapeutic choice in high-risk VTE patients (i.e., patients over 40 years of age, patients undergoing major surgery, patients with concurrent risk factors for VTE). Prophylactic low-molecular-weight heparin should be given subcutaneously once or twice daily until the patient is ambulating well.

    An IVC filter can be considered when anticoagulation is contraindicated (eg, due to heightened bleeding risk), but is generally not first line therapy.

     References

    1. Hemmila MR, et al. Prophylactic Inferior Vena Cava Filter Placement Does Not Result in a Survival Benefit for Trauma Patients. Ann Surg 2015; 262:577.

    2. Snow V, et al. Management of venous thromboembolism: a clinical practice guideline from the American College of Physicians and the American Academy of Family Physicians. Ann Fam Med 2007; 5:74.

    3. Blann AD, Lip GY. Venous thromboembolism. BMJ 2006; 332:215.

    4. Morris RJ, Woodcock JP. Intermittent pneumatic compression or graduated compression stockings for deep vein thrombosis prophylaxis? A systematic review of direct clinical comparisons. Ann Surg 2010; 251:393.

    Incorrect

    Early mobilization of patients as soon as possible after surgery is thought to reduce the chances of venous thrombosis because it stimulates calf muscles and puts pressure on the calf and leg veins, thus discouraging stasis and venous pooling of blood in the lower extremities. Likewise, sequential compression with pneumatic leg compression enhances blood flow in the deep veins of the legs. Lovenox is a low molecular weight heparin. It is an excellent therapeutic choice in high-risk VTE patients (i.e., patients over 40 years of age, patients undergoing major surgery, patients with concurrent risk factors for VTE). Prophylactic low-molecular-weight heparin should be given subcutaneously once or twice daily until the patient is ambulating well.

    An IVC filter can be considered when anticoagulation is contraindicated (eg, due to heightened bleeding risk), but is generally not first line therapy.

     References

    1. Hemmila MR, et al. Prophylactic Inferior Vena Cava Filter Placement Does Not Result in a Survival Benefit for Trauma Patients. Ann Surg 2015; 262:577.

    2. Snow V, et al. Management of venous thromboembolism: a clinical practice guideline from the American College of Physicians and the American Academy of Family Physicians. Ann Fam Med 2007; 5:74.

    3. Blann AD, Lip GY. Venous thromboembolism. BMJ 2006; 332:215.

    4. Morris RJ, Woodcock JP. Intermittent pneumatic compression or graduated compression stockings for deep vein thrombosis prophylaxis? A systematic review of direct clinical comparisons. Ann Surg 2010; 251:393.

  111. Question 111 of 220
    111. Question

    A 23 year old overweight male presents with a history of recurrent peptic ulcer disease. In the past, symptoms have responded well to Ranitidine and Omeprazole, but always seem to recur once the medication is discontinued. In addition, the severity of symptoms seems to be worse now than when they started, and are often accompanied by diarrhea. Patient is otherwise in good health. He has undergone no testing to date. He wonders if you can prescribe him a stronger medication to get rid of symptoms. Which of the following is the most appropriate advice to give the patient regarding his symptoms?

    Correct

    This patient most likely has Zollinger-Ellinson syndrome (ZES). In ZES, a gastrin-secreting tumor (gastrinoma) causes hypergastrinemia, which results in refractory peptic ulcer disease (PUD). The excess gastric acid leads to peptic ulcers, acid reflux, abdominal pain, diarrhea and other symptoms. The clinical presentation is indistinguishable from PUD, except that ZES is typically more advanced or refractory to PUD treatment (eg, PPIs and H2 blockers), as with this patient.

    In this case, the patient should be advised that symptoms may be due to a tumor, and that further work-up is advised. A fasting gastrin lab should be obtained—a level >150pg/mL indicates hypergastrinemia. Follow with a secretin test to confirm the diagnosis.

    Dietary modifications and weight loss will not change the course of the gastrinoma, so will not likely ameliorate symptoms.

    Endoscopy, CT or MRI may be helpful to localize the tumor.

    References

    1. Ellison EC, Johnson JA. The Zollinger-Ellison syndrome: a comprehensive review of historical, scientific, and clinical considerations. Curr Probl Surg 2009; 46:13.

    2. Metz DC. Diagnosis of the Zollinger–Ellison syndrome. Clin Gastroenterol Hepatol 2012; 10:126.

    Incorrect

    This patient most likely has Zollinger-Ellinson syndrome (ZES). In ZES, a gastrin-secreting tumor (gastrinoma) causes hypergastrinemia, which results in refractory peptic ulcer disease (PUD). The excess gastric acid leads to peptic ulcers, acid reflux, abdominal pain, diarrhea and other symptoms. The clinical presentation is indistinguishable from PUD, except that ZES is typically more advanced or refractory to PUD treatment (eg, PPIs and H2 blockers), as with this patient.

    In this case, the patient should be advised that symptoms may be due to a tumor, and that further work-up is advised. A fasting gastrin lab should be obtained—a level >150pg/mL indicates hypergastrinemia. Follow with a secretin test to confirm the diagnosis.

    Dietary modifications and weight loss will not change the course of the gastrinoma, so will not likely ameliorate symptoms.

    Endoscopy, CT or MRI may be helpful to localize the tumor.

    References

    1. Ellison EC, Johnson JA. The Zollinger-Ellison syndrome: a comprehensive review of historical, scientific, and clinical considerations. Curr Probl Surg 2009; 46:13.

    2. Metz DC. Diagnosis of the Zollinger–Ellison syndrome. Clin Gastroenterol Hepatol 2012; 10:126.

  112. Question 112 of 220
    112. Question

    What is the treatment of choice for eradication of H. pylori disease (in a patient with no penicillin allergy)?

    Correct

    In the United States, the recommended primary therapies for H. pylori infection include: a PPI twice daily, clarithromycin 500mg BID, and amoxicillin1,000mg BID. For patients allergic to penicillin, metronidazole can be substituted for amoxicillin. Treatment duration of 14 days has typically been employed. When given at the recommended doses, most recent studies report eradication rates in the range of 70–80%.

    H. pylori is naturally resistant to several commonly used antibiotics, including vancomycin, trimethoprim and sulfonamides.

    Antibiotics are required to eradicate H. pylori infection; fiber powder and probiotics will not cure the infection.

     References

    1. http://gi.org/guideline/management-of-helicobacter-pylori-infection/

    2. Almeida et al. Triple Therapy with High-Dose Proton-Pump Inhibitor, Amoxicillin, and Doxycycline Is Useless for Helicobacter pylori

    Eradication: A Proof-of-Concept Study. Helicobacter 19: 90–97 (2014).

    Incorrect

    In the United States, the recommended primary therapies for H. pylori infection include: a PPI twice daily, clarithromycin 500mg BID, and amoxicillin1,000mg BID. For patients allergic to penicillin, metronidazole can be substituted for amoxicillin. Treatment duration of 14 days has typically been employed. When given at the recommended doses, most recent studies report eradication rates in the range of 70–80%.

    H. pylori is naturally resistant to several commonly used antibiotics, including vancomycin, trimethoprim and sulfonamides.

    Antibiotics are required to eradicate H. pylori infection; fiber powder and probiotics will not cure the infection.

     References

    1. http://gi.org/guideline/management-of-helicobacter-pylori-infection/

    2. Almeida et al. Triple Therapy with High-Dose Proton-Pump Inhibitor, Amoxicillin, and Doxycycline Is Useless for Helicobacter pylori

    Eradication: A Proof-of-Concept Study. Helicobacter 19: 90–97 (2014).

  113. Question 113 of 220
    113. Question

    A 57 year old female presents for colon cancer surveillance. She underwent resection of part of the transverse colon 1 year ago, and is having a CT of the abdomen and pelvis as part of her cancer surveillance. The study shows no abnormalities other than three small (~1cm) stones within the gallbladder, with no gallbladder wall thickening or pericholecystic fluid accumulation. The patient says she has not had any symptoms. What is the most appropriate therapy for this patient?

    Correct

    The presence of gallstones (cholelithiasis) is common, particularly in Western populations. In the United States, gallstones are seen in approximately 6 percent of men and 9 percent of women. Most individuals with gallstones are asymptomatic throughout their life and are referred to as having “incidental” gallstones. Prophylactic cholecystectomy is not indicated for most patients with asymptomatic gallstones. Adult patients with silent or incidental gallstones should be observed and managed expectantly, with few exceptions.

    Decision analysis models have shown no benefit with prophylactic cholecystectomy, whether laparoscopic or open. Waiting until a patient becomes symptomatic before performing cholecystectomy prevents unnecessary surgery since the majority of patients with incidental gallstones will never develop biliary colic. However, prophylactic cholecystectomy should be performed in patients at high risk of gallbladder carcinoma. The specific groups at high risk of gallbladder cancer include patients with asymptomatic gallstones who are Pima Indians or who have a calcified gall-bladder, gallbladder polyps greater than 10 mm, gallstones greater than 2.5 cm or anomalous pancreaticobiliary ductal junction, and carriers of Salmonella typhosa.

    ERCP to evaluate the biliary tree is not indicated for asymptomatic gallstones.

    References 

    1. Ahmed A, et al. Management of Gallstones and Their Complications. Am Fam Physician. 2000 Mar 15;61(6):1673-1680.

    2. Ransohoff DF, et al. Prophylactic cholecystectomy or expectant management for silent gallstones. A decision analysis to assess survival. Ann Intern Med. 1983;99(2):199.

    3. Gracie WA, Ransohoff DF. The natural history of silent gallstones: the innocent gallstone is not a myth. N Engl J Med. 1982;307:798–800.

    Incorrect

    The presence of gallstones (cholelithiasis) is common, particularly in Western populations. In the United States, gallstones are seen in approximately 6 percent of men and 9 percent of women. Most individuals with gallstones are asymptomatic throughout their life and are referred to as having “incidental” gallstones. Prophylactic cholecystectomy is not indicated for most patients with asymptomatic gallstones. Adult patients with silent or incidental gallstones should be observed and managed expectantly, with few exceptions.

    Decision analysis models have shown no benefit with prophylactic cholecystectomy, whether laparoscopic or open. Waiting until a patient becomes symptomatic before performing cholecystectomy prevents unnecessary surgery since the majority of patients with incidental gallstones will never develop biliary colic. However, prophylactic cholecystectomy should be performed in patients at high risk of gallbladder carcinoma. The specific groups at high risk of gallbladder cancer include patients with asymptomatic gallstones who are Pima Indians or who have a calcified gall-bladder, gallbladder polyps greater than 10 mm, gallstones greater than 2.5 cm or anomalous pancreaticobiliary ductal junction, and carriers of Salmonella typhosa.

    ERCP to evaluate the biliary tree is not indicated for asymptomatic gallstones.

    References 

    1. Ahmed A, et al. Management of Gallstones and Their Complications. Am Fam Physician. 2000 Mar 15;61(6):1673-1680.

    2. Ransohoff DF, et al. Prophylactic cholecystectomy or expectant management for silent gallstones. A decision analysis to assess survival. Ann Intern Med. 1983;99(2):199.

    3. Gracie WA, Ransohoff DF. The natural history of silent gallstones: the innocent gallstone is not a myth. N Engl J Med. 1982;307:798–800.

  114. Question 114 of 220
    114. Question

    A patient with ulcerative colitis underwent protcolectomy (resection of the colon and rectum) with ileal pouch anal anastomosis 5 years ago. Within the past year, he has had 2 separate occasions of a painful swollen mass in the perianal area that drains pus, then he feels much better and the mass goes away. Each time the swelling has occurred, it has been in the same location. He can feel the symptoms starting again, and comes in for evaluation. Of the following diagnoses, which is the most likely for this patient?

     

    Correct

    Crohn’s disease is characterized by transmural inflammation that can affect any part of the gastrointestinal tract, from the mouth to the anus. It is associated with segmental involvement with skip lesions. Strictures and perianal disease (eg, abscess, fistula) are also seen with Crohn’s disease but are exceedingly rare in ulcerative colitis. In patients who have undergone removal of the colon and rectum to treat presumed ulcerative colitis but who later develop symptoms elsewhere in the GI tract, including the perianal area, Crohn’s disease must be considered. The patient in this scenario is experiencing recurrent perianal abscess with likely fistula in ano, which is not a characteristic of ulcerative colitis.

    Ulcerative colitis is characterized by inflammation that is confined to the mucosal and submucosal layers of the colon. It generally starts in the rectum and progresses proximally. Surgical excision of the colon and rectum cures ulcerative colitis, though not the extra-intestinal manifestations of the disease.

    Perianal abscess is not a feature of irritable bowel syndrome.

    Anal cancer typically presents with signs of bleeding from the anus, a lump near the anus or pain, pressure, itching or discharge near the anus. Symptoms are generally not cyclic in nature.

    References

    1. Zoeten E, et al. Diagnosis and Treatment of Perianal Crohn Disease: NASPGHAN Clinical Report and Consensus Statement. JPGN Volume 57, Number 3, September 2013.

    2. Thornton M, Solomon MJ. Long-term indwelling seton for complex anal fistulas in Crohn’s disease. Dis Colon Rectum 2005; 48:459.

    Incorrect

    Crohn’s disease is characterized by transmural inflammation that can affect any part of the gastrointestinal tract, from the mouth to the anus. It is associated with segmental involvement with skip lesions. Strictures and perianal disease (eg, abscess, fistula) are also seen with Crohn’s disease but are exceedingly rare in ulcerative colitis. In patients who have undergone removal of the colon and rectum to treat presumed ulcerative colitis but who later develop symptoms elsewhere in the GI tract, including the perianal area, Crohn’s disease must be considered. The patient in this scenario is experiencing recurrent perianal abscess with likely fistula in ano, which is not a characteristic of ulcerative colitis.

    Ulcerative colitis is characterized by inflammation that is confined to the mucosal and submucosal layers of the colon. It generally starts in the rectum and progresses proximally. Surgical excision of the colon and rectum cures ulcerative colitis, though not the extra-intestinal manifestations of the disease.

    Perianal abscess is not a feature of irritable bowel syndrome.

    Anal cancer typically presents with signs of bleeding from the anus, a lump near the anus or pain, pressure, itching or discharge near the anus. Symptoms are generally not cyclic in nature.

    References

    1. Zoeten E, et al. Diagnosis and Treatment of Perianal Crohn Disease: NASPGHAN Clinical Report and Consensus Statement. JPGN Volume 57, Number 3, September 2013.

    2. Thornton M, Solomon MJ. Long-term indwelling seton for complex anal fistulas in Crohn’s disease. Dis Colon Rectum 2005; 48:459.

  115. Question 115 of 220
    115. Question

    A 47 year old patient with past medical history of ulcerative colitis presents with a complaint of fatigue, malaise and weight loss. She has also noticed a yellow tint to her skin, which is increasingly itchy. If work-up confirms the suspected diagnosis of primary sclerosing cholangitis, management options include all of the following except:

    Correct

    Primary sclerosing cholangitis (PSC) is a chronic thickening of the bile duct walls of unknown etiology, although 80% of cases are associated with inflammatory bowel disease, generally ulcerative colitis. Endoscopic retrograde cholangiopancreatography (ERCP) is the mainstay for accurate assessment of the hepatobiliary tree to establish a diagnosis of PSC. Management is multidisciplinary. Antihistamines can be used for pruritus. Bile that backs up in narrowed or blocked ducts causes frequent bacterial infections. To prevent and treat these infections, people with PSC may take repeated courses of antibiotics. Liver transplant is the only known cure for PSC.

    Resecting the pancreas (pancreatectomy) in PSC is not indicated unless signs and symptoms of pancreatitis are present.

     References

    1. Lindor, K. et al. High Dose Ursodeoxycholic Acid for the Treatment of Primary Sclerosing Cholangitis. Hepatology. 2009 Sep; 50(3): 808–814.

    2. Tharian B, et al. What is the current role of endoscopy in primary sclerosing cholangitis? World J Gastrointest Endosc. 2015 Aug 10; 7(10): 920–927.

    3. Koro N, Alkaade S. Role of Endoscopy in Primary Sclerosing Cholangitis. Curr Gastroenterol Rep (2013) 15:361.

    Incorrect

    Primary sclerosing cholangitis (PSC) is a chronic thickening of the bile duct walls of unknown etiology, although 80% of cases are associated with inflammatory bowel disease, generally ulcerative colitis. Endoscopic retrograde cholangiopancreatography (ERCP) is the mainstay for accurate assessment of the hepatobiliary tree to establish a diagnosis of PSC. Management is multidisciplinary. Antihistamines can be used for pruritus. Bile that backs up in narrowed or blocked ducts causes frequent bacterial infections. To prevent and treat these infections, people with PSC may take repeated courses of antibiotics. Liver transplant is the only known cure for PSC.

    Resecting the pancreas (pancreatectomy) in PSC is not indicated unless signs and symptoms of pancreatitis are present.

     References

    1. Lindor, K. et al. High Dose Ursodeoxycholic Acid for the Treatment of Primary Sclerosing Cholangitis. Hepatology. 2009 Sep; 50(3): 808–814.

    2. Tharian B, et al. What is the current role of endoscopy in primary sclerosing cholangitis? World J Gastrointest Endosc. 2015 Aug 10; 7(10): 920–927.

    3. Koro N, Alkaade S. Role of Endoscopy in Primary Sclerosing Cholangitis. Curr Gastroenterol Rep (2013) 15:361.

  116. Question 116 of 220
    116. Question

    A 45 year old female has been diagnosed with stage 3 colon cancer. She recalls that one of her grandmothers died of colon cancer, and she has an older sister who has had breast cancer. She has 2 children, ages 23 and 25. At what age should the children have their first colonoscopy?

    Correct

    Recommendations for colorectal cancer (CRC) screening when there is a single first-degree with CRC or advanced adenoma diagnosed at age <60 years is colonoscopy every 5 years beginning at age 40 years or 10 years younger than age at diagnosis of the youngest affected relative.

    References

    1. http://gi.org/guideline/colorectal-cancer-screening/

    Incorrect

    Recommendations for colorectal cancer (CRC) screening when there is a single first-degree with CRC or advanced adenoma diagnosed at age

    References

    1. http://gi.org/guideline/colorectal-cancer-screening/

  117. Question 117 of 220
    117. Question

    An otherwise healthy 54 year old female returns to your office today for post-op follow-up. She underwent abdominal surgery 2 weeks prior, with a 6cm vertical midline abdominal wound that was closed with absorbable sutures under the skin and staples on top of the skin. You uncover the wound and notice that there is a small amount of erythema at the staples, but no fluctuance or purulent discharge. The staples are still in place. The wound has dehisced (edges are not approximated other than directly under each staple), but it does not appear infected. The skin is separated and subcutaneous fat is visible, but all deeper layers seem intact. There is no necrotic or devitalized tissue. Patient denies fevers, chills or abdominal pain and is recovering well otherwise. What is the appropriate next step in caring for the surgical wound?

    Correct

    Wound dehiscence can range from splitting open of the skin layers to complete breakdown of the muscle and fascia, exposing internal organs. Occasionally, the skin layers remain intact, but deeper layers break down resulting in an incisional hernia.

    Many factors are associated with surgical dehiscence. Age, malnutrition, male sex and long-term steroid use are thought to influence the incidence of abdominal dehiscence. Smoking, diabetes and rheumatoid arthritis can impair healing by affecting the microcirculation. Obesity can influence wound breakdown due to reduced tissue oxygenation, increased subcutaneous dead space rendering the patient more susceptible to hematoma and seroma formation and increased incidence of infection.

    Once wound infection has been ruled out, treatment of the dehisced wound will depend on the depth of tissue interruption. In this case, the only breakdown is at the level of the skin and subcutaneous tissue. For such superficial dehiscence, remove any staples that have been in place for the required amount of time (depending on location), usually 12-14 days for abdominal incisions. There is no added benefit to leaving the staples when the wound has clearly dehisced. Explore the wound to see how deep it goes, assessing if the fascia is intact. Do not restaple or suture the dehisced wound; instead, allow it to heal by secondary intention using wet to dry packing of the wound.

    There is no indication in this scenario for returning to the operating room for abdominal washout in the absence of intra-abdominal contamination or infection.

     

    References

    1. Orsted H, et al. Best Practice Recommendations for the Prevention and Management of Open Surgical Wounds. Wound Care Canada, Volume 8, Number 1, 2010, p.6-34.

    Incorrect

    Wound dehiscence can range from splitting open of the skin layers to complete breakdown of the muscle and fascia, exposing internal organs. Occasionally, the skin layers remain intact, but deeper layers break down resulting in an incisional hernia.

    Many factors are associated with surgical dehiscence. Age, malnutrition, male sex and long-term steroid use are thought to influence the incidence of abdominal dehiscence. Smoking, diabetes and rheumatoid arthritis can impair healing by affecting the microcirculation. Obesity can influence wound breakdown due to reduced tissue oxygenation, increased subcutaneous dead space rendering the patient more susceptible to hematoma and seroma formation and increased incidence of infection.

    Once wound infection has been ruled out, treatment of the dehisced wound will depend on the depth of tissue interruption. In this case, the only breakdown is at the level of the skin and subcutaneous tissue. For such superficial dehiscence, remove any staples that have been in place for the required amount of time (depending on location), usually 12-14 days for abdominal incisions. There is no added benefit to leaving the staples when the wound has clearly dehisced. Explore the wound to see how deep it goes, assessing if the fascia is intact. Do not restaple or suture the dehisced wound; instead, allow it to heal by secondary intention using wet to dry packing of the wound.

    There is no indication in this scenario for returning to the operating room for abdominal washout in the absence of intra-abdominal contamination or infection.

     

    References

    1. Orsted H, et al. Best Practice Recommendations for the Prevention and Management of Open Surgical Wounds. Wound Care Canada, Volume 8, Number 1, 2010, p.6-34.

  118. Question 118 of 220
    118. Question

    A patient presents with a complaint of hemorrhoids. On exam, you note that internal hemorrhoids are enlarged and prolapse to the outside of the body on Valsalva, but then spontaneously recede. There is scant bleeding. Patient reports that at home on the commode, this is exactly what happens with bowel movements—tissue protrudes, then goes back inside once the bowel movement is complete, and there is a little bit of blood on the toilet paper when he wipes.

    What stage of hemorrhoids does this patient have?

    Correct

    Internal hemorrhoids are classified by their degree of prolapse, which helps determine management:

    Grade One:     No prolapse

    Grade Two:     Prolapse that goes back in on its own

    Grade Three: Prolapse that must be pushed back in by the patient

    Grade Four:   Prolapse that cannot be pushed back in by the patient (often very painful).

    Bleeding attributed to internal hemorrhoids is usually bright red and can be quite brisk. It may be found when wiping, dripping into the toilet bowl, or streaked on the bowel movement (BM) itself. Not all patients with symptomatic internal hemorrhoids will have significant bleeding. Instead, prolapse may be the main or only symptom. Prolapsing tissue may result in significant irritation and itching around the anus. Patients may also complain of mucus discharge, difficulty with cleaning themselves after a BM, or a sense that their stool is “stuck” at the anus with BMs. Patients without significant symptoms from internal hemorrhoids do not require treatment based on their appearance alone.

    References

    1. Hemorrhoids: Expanded Version. American Society of Colon and Rectal Surgeons, https://www.fascrs.org/patients/disease-condition/hemorrhoids-expanded-version.

    Incorrect

    Internal hemorrhoids are classified by their degree of prolapse, which helps determine management:

    Grade One:     No prolapse

    Grade Two:     Prolapse that goes back in on its own

    Grade Three: Prolapse that must be pushed back in by the patient

    Grade Four:   Prolapse that cannot be pushed back in by the patient (often very painful).

    Bleeding attributed to internal hemorrhoids is usually bright red and can be quite brisk. It may be found when wiping, dripping into the toilet bowl, or streaked on the bowel movement (BM) itself. Not all patients with symptomatic internal hemorrhoids will have significant bleeding. Instead, prolapse may be the main or only symptom. Prolapsing tissue may result in significant irritation and itching around the anus. Patients may also complain of mucus discharge, difficulty with cleaning themselves after a BM, or a sense that their stool is “stuck” at the anus with BMs. Patients without significant symptoms from internal hemorrhoids do not require treatment based on their appearance alone.

    References

    1. Hemorrhoids: Expanded Version. American Society of Colon and Rectal Surgeons, https://www.fascrs.org/patients/disease-condition/hemorrhoids-expanded-version.

  119. Question 119 of 220
    119. Question

    A patient presents with a small bleeding lesion at the anal outlet. On physical exam, you see that the lesion involves the anus and the adjacent perianal skin. You take a large biopsy, and final pathology report reveals invasive squamous cell carcinoma. Which of the following is the most appropriate treatment for this condition?

    Correct

    Squamous cell carcinoma of the anus is an uncommon tumor, representing only 1.5% of gastrointestinal tract tumors. Concurrent chemoradiotherapy is the current standard of treatment for anal cancer. Until the mid-1970s, anal cancer was most commonly treated with radical surgery consisting of abdominoperineal resection (APR), which involved removal of the anal sphincters and placement of a permanent colostomy. The use of chemoradiotherapy was introduced in 1974, initially as neoadjuvant therapy preceding APR. However, with the finding that many patients were rendered free of cancer by this pre-operative regimen, CRT became the widely accepted definitive therapeutic option, sparing the patient the need for permanent colostomy, and replacing radical surgery as the primary treatment of choice for anal cancer.

    Topical immunomodulators (eg, Aldara, Condylox) and cryotherapy are treatment for anal warts, not anal cancer.

    References

    1. Spithoff K, et al. Chemoradiotherapy for Squamous Cell Carcinoma of the Anal Canal: A Systematic Review. Clinical Oncology 26 (2014) 473-487.

    2. http://www.cdc.gov/std/treatment/2010/genital-warts.htm

    Incorrect

    Squamous cell carcinoma of the anus is an uncommon tumor, representing only 1.5% of gastrointestinal tract tumors. Concurrent chemoradiotherapy is the current standard of treatment for anal cancer. Until the mid-1970s, anal cancer was most commonly treated with radical surgery consisting of abdominoperineal resection (APR), which involved removal of the anal sphincters and placement of a permanent colostomy. The use of chemoradiotherapy was introduced in 1974, initially as neoadjuvant therapy preceding APR. However, with the finding that many patients were rendered free of cancer by this pre-operative regimen, CRT became the widely accepted definitive therapeutic option, sparing the patient the need for permanent colostomy, and replacing radical surgery as the primary treatment of choice for anal cancer.

    Topical immunomodulators (eg, Aldara, Condylox) and cryotherapy are treatment for anal warts, not anal cancer.

    References

    1. Spithoff K, et al. Chemoradiotherapy for Squamous Cell Carcinoma of the Anal Canal: A Systematic Review. Clinical Oncology 26 (2014) 473-487.

    2. http://www.cdc.gov/std/treatment/2010/genital-warts.htm

  120. Question 120 of 220
    120. Question

    Which of the following is the most common reason for liver transplant in the U.S.?

    Correct

    In the U.S., the most common reason for needing a liver transplant is cirrhosis caused by chronic viral hepatitis C infection, followed by cirrhosis caused by long-term alcohol abuse. Primary biliary cirrhosis and autoimmune hepatitis are also possible causes, though not as common.

     References

    1. https://www.niddk.nih.gov/health-information/liver-disease/liver-transplant

    2. Burra P, et al. Hepatitis C virus and liver transplantation: where do we stand? Transpl Int. 2016 Feb;29(2):135-52.

    Incorrect

    In the U.S., the most common reason for needing a liver transplant is cirrhosis caused by chronic viral hepatitis C infection, followed by cirrhosis caused by long-term alcohol abuse. Primary biliary cirrhosis and autoimmune hepatitis are also possible causes, though not as common.

     References

    1. https://www.niddk.nih.gov/health-information/liver-disease/liver-transplant

    2. Burra P, et al. Hepatitis C virus and liver transplantation: where do we stand? Transpl Int. 2016 Feb;29(2):135-52.

  121. Question 121 of 220
    121. Question

    The most common cause of acute hepatitis is:

    Correct

    Hepatitis is inflammation of the liver, a vital organ that processes nutrients, filters the blood, and fights infections. When the liver is inflamed or damaged, its function can be affected. Heavy alcohol use, toxins, some medications, and certain medical conditions (eg, Wilson’s disease, autoimmune hepatitis) can cause hepatitis. However, hepatitis is most often caused by a virus. In the United States, the most common types of viral hepatitis are Hepatitis A, Hepatitis B, and Hepatitis C.

    References 

    1. http://www.cdc.gov/hepatitis/abc/

    Incorrect

    Hepatitis is inflammation of the liver, a vital organ that processes nutrients, filters the blood, and fights infections. When the liver is inflamed or damaged, its function can be affected. Heavy alcohol use, toxins, some medications, and certain medical conditions (eg, Wilson’s disease, autoimmune hepatitis) can cause hepatitis. However, hepatitis is most often caused by a virus. In the United States, the most common types of viral hepatitis are Hepatitis A, Hepatitis B, and Hepatitis C.

    References 

    1. http://www.cdc.gov/hepatitis/abc/

  122. Question 122 of 220
    122. Question

    A 50 year old previously healthy female presents to the Emergency Department with 2 days of progressive abdominal pain, now severe, abdominal distention, diarrhea and fever. No history of recent travel. No history of inflammatory bowel disease, though her younger brother has a long history of “bowel troubles.” She has never undergone colonoscopy. You suspect toxic megacolon. Which of the following is the most appropriate first step in this patient’s care?

    Correct

    Toxic megacolon is a rare but potentially fatal complication of colitis, most commonly associated with inflammatory bowel disease. It is defined as a segmental or total colonic dilatation of >6cm in the presence of acute colitis and signs of systemic toxicity. Early detection and intervention have a profound impact on survival. Toxic megacolon can occur in any patient with colitis, regardless of age, gender or race. Toxic megacolon is a diagnosis based on clinical signs of systemic toxicity in combination with radiographic evidence of colonic dilatation. The radiologic diagnosis is made using plain films of the abdomen, assessing for dilatation > 6cm.

    Toxic megacolon features found by plain film may indicate the need for subsequent CT to determine the cause of the megacolon, and is an important tool in the diagnosis of abdominal complications such as colonic perforation.

    Stool studies and culture should be sent for patients with a history of antibiotic use or chemotherapy, assessing for C. diff as the etiology of the megacolon.

    Colonoscopy to assess the inside of the colon should not be performed in suspected toxic megacolon due to the increased risk of colonic perforation.

    References

    1. Woodhouse, E. Toxic Megacolon: A Reviewfor Emergency Department Clinicians. Journal of Emergency Nursing, Volume 42, Issue 6 (2016), p. 481-486.

    2. Autenrieth D, Baumgart D. Toxic Megacolon. Inflamm Bowel Dis 2012;18: 584-591 (2012).

    Incorrect

    Toxic megacolon is a rare but potentially fatal complication of colitis, most commonly associated with inflammatory bowel disease. It is defined as a segmental or total colonic dilatation of >6cm in the presence of acute colitis and signs of systemic toxicity. Early detection and intervention have a profound impact on survival. Toxic megacolon can occur in any patient with colitis, regardless of age, gender or race. Toxic megacolon is a diagnosis based on clinical signs of systemic toxicity in combination with radiographic evidence of colonic dilatation. The radiologic diagnosis is made using plain films of the abdomen, assessing for dilatation > 6cm.

    Toxic megacolon features found by plain film may indicate the need for subsequent CT to determine the cause of the megacolon, and is an important tool in the diagnosis of abdominal complications such as colonic perforation.

    Stool studies and culture should be sent for patients with a history of antibiotic use or chemotherapy, assessing for C. diff as the etiology of the megacolon.

    Colonoscopy to assess the inside of the colon should not be performed in suspected toxic megacolon due to the increased risk of colonic perforation.

    References

    1. Woodhouse, E. Toxic Megacolon: A Reviewfor Emergency Department Clinicians. Journal of Emergency Nursing, Volume 42, Issue 6 (2016), p. 481-486.

    2. Autenrieth D, Baumgart D. Toxic Megacolon. Inflamm Bowel Dis 2012;18: 584-591 (2012).

  123. Question 123 of 220
    123. Question

    A 33 year old female first-year PA student presents to your clinic with a complaint of abdominal pain and discomfort intermittently for the past 3 to 4 months. The pain tends to be generalized over the abdomen, occurs at least once a week, and improves with defecation. Her stools alternate between constipation and diarrhea, on an unpredictable basis. The patient denies blood in her stools. Symptoms do not seem related to any particular dietary choices. She will have more frequent diarrhea when she feels increased stress. Your physical exam is unremarkable, as are the labs you during the workup (ESR, CRP, CBC, CMP). Of the following, which is the most likely diagnosis?

    Correct

    Irritable bowel syndrome (IBS), one of the most common gastrointestinal (GI) disorders in the world, is characterized by abdominal pain, discomfort, bloating, and disturbed defecation. It has four types: IBS with diarrhea (IBS-D), IBS with constipation (IBS-C), mixed IBS (IBS-M), and unsubtyped IBS (IBS-U).

    Because IBS lacks characteristic imaging features and has no diagnostic biomarkers, symptom-based criteria (Rome III) are recommended for its diagnosis, defined as: Recurrent abdominal pain or discomfort at least 3 days/month in the last 3 months associated with two or more of the following:

    • Improvement with defecation.
    • Onset associated with a change in frequency of stool.

    o   Onset associated with a change in form (appearance) of stool.

    Lactose intolerance and celiac disease are both related to dietary intake, which is not indicated in this scenario.

    Inflammatory bowel disease is characterized by inflammation of all or part of the GI tract, and usually presents with severe diarrhea, pain, fatigue and weight loss, which is not indicated in this scenario.

    References

    1. Ohgo H, et al. Irritable bowel syndrome evaluation using computed tomography colonography. World J Gastroenterol 2016 November 14; 22(42): 9394-9399.

    Incorrect

    Irritable bowel syndrome (IBS), one of the most common gastrointestinal (GI) disorders in the world, is characterized by abdominal pain, discomfort, bloating, and disturbed defecation. It has four types: IBS with diarrhea (IBS-D), IBS with constipation (IBS-C), mixed IBS (IBS-M), and unsubtyped IBS (IBS-U).

    Because IBS lacks characteristic imaging features and has no diagnostic biomarkers, symptom-based criteria (Rome III) are recommended for its diagnosis, defined as: Recurrent abdominal pain or discomfort at least 3 days/month in the last 3 months associated with two or more of the following:

    • Improvement with defecation.
    • Onset associated with a change in frequency of stool.

    o   Onset associated with a change in form (appearance) of stool.

    Lactose intolerance and celiac disease are both related to dietary intake, which is not indicated in this scenario.

    Inflammatory bowel disease is characterized by inflammation of all or part of the GI tract, and usually presents with severe diarrhea, pain, fatigue and weight loss, which is not indicated in this scenario.

    References

    1. Ohgo H, et al. Irritable bowel syndrome evaluation using computed tomography colonography. World J Gastroenterol 2016 November 14; 22(42): 9394-9399.

  124. Question 124 of 220
    124. Question

    Which of the following patients is at the highest risk of post-operative venous thromboembolism (VTE)?

    Correct

    VTE is a common cause of preventable death in hospitalized patients. Approximately one-third of the 150,000 to 200,000 VTE-related deaths per year in the United States occur following surgery. In patients undergoing general and abdominal-pelvic surgery, the risk of VTE varies depending on both patient-specific and procedure-specific factors. Examples of relatively low-risk procedures include laparoscopic cholecystectomy, appendectomy, transurethral prostatectomy, inguinal herniorrhaphy, and unilateral or bilateral mastectomy. Open abdominal and open-pelvic procedures are associated with a higher risk of VTE. VTE risk appears to be highest for patients undergoing abdominal or pelvic surgery for cancer. Patient-specific factors also determine the risk of VTE, including age >60, prior VTE, anesthesia > 2 hours, bedrest > 4 days and sepsis.

     References 

    1. Gould MK, et al. Prevention of VTE in nonorthopedic surgical patients: Antithrombotic Therapy and Prevention of Thrombosis, 9th ed: American College of Chest Physicians Evidence-Based Clinical Practice Guidelines. Chest. 2012 Feb;141(2 Suppl):e227S-77S.

    Incorrect

    VTE is a common cause of preventable death in hospitalized patients. Approximately one-third of the 150,000 to 200,000 VTE-related deaths per year in the United States occur following surgery. In patients undergoing general and abdominal-pelvic surgery, the risk of VTE varies depending on both patient-specific and procedure-specific factors. Examples of relatively low-risk procedures include laparoscopic cholecystectomy, appendectomy, transurethral prostatectomy, inguinal herniorrhaphy, and unilateral or bilateral mastectomy. Open abdominal and open-pelvic procedures are associated with a higher risk of VTE. VTE risk appears to be highest for patients undergoing abdominal or pelvic surgery for cancer. Patient-specific factors also determine the risk of VTE, including age >60, prior VTE, anesthesia > 2 hours, bedrest > 4 days and sepsis.

     References 

    1. Gould MK, et al. Prevention of VTE in nonorthopedic surgical patients: Antithrombotic Therapy and Prevention of Thrombosis, 9th ed: American College of Chest Physicians Evidence-Based Clinical Practice Guidelines. Chest. 2012 Feb;141(2 Suppl):e227S-77S.

  125. Question 125 of 220
    125. Question

    A 57 year-old male smoker with PMH of COPD on home oxygen, HTN, HLD, DM, CKD, brought to the ER by ambulance with a chief complaint of “difficulty breathing and weakness.” Vital signs on presentation were: BP 84/21, pulse 132, sat 02 86% on CPAP, RR36. On exam he has peribuccal cyanosis, diaphoresis, diffuse rhonchi on auscultation of lungs, normal heart sounds, no MRG, diminished bowel sounds with abdominal distention and moderate, and diffuse tenderness on palpation of the abdomen to the right flank. He takes albuterol 2 puffs PRN, enalapril, fluoxetine, baby aspirin, labetalol HCL, lyrica, metformin, nifedipine. Two days earlier he was started on prednisone 50md daily, oxycodone10mg q6hours, oxycontin ER 40mg q8hours for low back pain. Patient was subsequently intubated and an OGT placed. Coffee ground emesis was seen on suction. CT scan shows free air and intraperitoneal fluid. His white blood cell count is 14 and hemoglobin of 8.  What is the best treatment option for this patient?

    Correct

     

    Answer is B. Given this patients vitals, physical exam, values showing blood loss and leukocytosis and CT scan with free air and fluid this patient has a perforation somewhere in his abdomen, mostly likely due to a peptic ulcer The patients age, sex, history of tobacco use, and daily ASA use put him at increased risk of developing peptic ulcers (1,2,3,4,5). GI bleeds and gastrointestinal perforations are common complications of peptic ulcers (1, 2, 3, 4, 5). Due to the fact that he is hemodynamically unstable, and he has free air and fluid on the CT scan, just giving a unit of pRBC and IVF and monitoring him would not be adequate care. The best treatment option for this patient would be emergent exploratory surgery to locate the perforation and repair it (4). Given the above, admitting him for observation and repeating the CT in the morning is not the right answer.

     

     

     

    Case Discussion:

    The above case and question represent a common complication associated with peptic ulcer disease. Peptic ulcers are the result of a defect in the gastric or duodenal mucosa that extends through the muscularis mucosa.1,2 Peptic ulcers occur when the protective mucosal factors decrease and damaging factors such as acid and pepsin increase.1 Duodenal ulcers are 5 times more common than gastric ulcers and tend to occur in patients between the ages of 30-55,1,5 whereas gastric ulcers occur in patients between the ages of 55-70.1 However, ulcers can occur at any age.1 Ulcers in general are slightly more common in men than women with a ratio of 1.3:1.1 Some common risk factors associated with people who have ulcers are smoking and long term use of NSAIDs.

    Two major etiologies associated with peptic ulcer disease is Helicobacter pylori (H. pylori) infection and the long term use of nonsteroidal anti-inflammatory drugs (NSAIDs) such as aspirin, indomethacin, ketorolac, Motrin, Aleve.1,2 Less common causes are acid hypersecretory states like Zollinger Ellison syndrome.1

    Mucosal injury occurs when the balance between the protective factors like mucous, bicarbonate, prostaglandins, and blood flow and the destructive factors like pepsin and acid are disrupted.1,3 The imbalance leads to increased acid exposure to the mucosal lining of the stomach or the duodenum causing gastric metaplasia.1

    NSAID-induced ulcers occur due to suppression of gastric prostaglandin synthesis.4 Prostaglandins are important for mucosal integrity. Once inhibited, it makes the mucosal lining more susceptible to destruction in the setting of increased pepsin and acid. The most common presenting symptom in patients with peptic ulcer disease is dyspepsia.2 However, most patients may not have any symptoms until they develop complications of the ulcers like obstruction or perforation. According to Vakil, “80 percent of patients with endoscopically diagnosed ulcers have epigastric pain.”2

    Those that complain of gastrointestinal symptoms describe the pain as dull, gnawing, aching, localized to the epigastrium area.1 The pain is typically related to eating.1,2,3,4,5 Duodenal ulcers usually occurs a couple hours after meals and in the middle of the night.1,2 While patients with peptic ulcers have pain with eating.1,2,3,4,5 Other gastrointestinal symptoms include: belching, epigastric fullness, early satiety, nausea and occasional vomiting.1 Patients with uncomplicated peptic ulcer decease usually have nonspecific findings on physical exam.1 They may or may not have mild epigastric tenderness on palpation and guaiac-positive stools.1

    Complications of peptic ulcers include bleeding, gastric outlet obstruction, fistulization and perforation.2 Patients who present with perforation usually have sudden, severe pain with peritonitis and free air on CT scan.2,4,5 They may also present with signs and symptoms of septic shock, like tachycardia and hypotension.4

    If peptic ulcer disease is suspected an upper endoscopy is the procedure of choice for diagnosis.1 Endoscopy allows for direct visualization of the ulcer and biopsies to test for H. pylori infection and malignancy.1 On endoscopic exam, peptic ulcers appear as discrete mucosal lesions with a punched out smooth ulcer base, with a whitish fribrinoid exudate in the middle.4

     

     

    References

    1. Papadakis M, McPhee S, Rabow M. Current Medical Diagnosis & Treatment 2016.
    2. Peptic ulcer disease: Clinical manifestations and diagnosis. Uptodatecom. 2016. Available at: http://www.uptodate.com/contents/peptic-ulcer-disease-clinical-manifestations-and-diagnosis?topicKey=GAST%2F26&elapsedTimeMs=6&source=search_result&searchTerm=peptic+ulcer+disesase&selectedTitle=1%7E150&view=print&displayedView=full. Accessed July 12, 2016.
    3. Williams D. PANCE Prep Pearls. Lexington, KY: CreateSpace; 2014.
    4. Peptic Ulcer Disease: Background, Anatomy, Pathphysiology. Emedicinemedscapecom. 2016. Available at: http://emedicine.medscape.com/article/181753-overview. Accessed July 12, 2016.
    5. Hcriz C. Stomach Disorders [powerpoint] Manchester, NH: MCPHS Physician Assistant Program; 2015
    Incorrect

    Answer is B. Given this patients vitals, physical exam, values showing blood loss and leukocytosis and CT scan with free air and fluid this patient has a perforation somewhere in his abdomen, mostly likely due to a peptic ulcer The patients age, sex, history of tobacco use, and daily ASA use put him at increased risk of developing peptic ulcers (1,2,3,4,5). GI bleeds and gastrointestinal perforations are common complications of peptic ulcers (1, 2, 3, 4, 5). Due to the fact that he is hemodynamically unstable, and he has free air and fluid on the CT scan, just giving a unit of pRBC and IVF and monitoring him would not be adequate care. The best treatment option for this patient would be emergent exploratory surgery to locate the perforation and repair it (4). Given the above, admitting him for observation and repeating the CT in the morning is not the right answer.

     

     

     

    Case Discussion:

    The above case and question represent a common complication associated with peptic ulcer disease. Peptic ulcers are the result of a defect in the gastric or duodenal mucosa that extends through the muscularis mucosa.1,2 Peptic ulcers occur when the protective mucosal factors decrease and damaging factors such as acid and pepsin increase.1 Duodenal ulcers are 5 times more common than gastric ulcers and tend to occur in patients between the ages of 30-55,1,5 whereas gastric ulcers occur in patients between the ages of 55-70.1 However, ulcers can occur at any age.1 Ulcers in general are slightly more common in men than women with a ratio of 1.3:1.1 Some common risk factors associated with people who have ulcers are smoking and long term use of NSAIDs.

    Two major etiologies associated with peptic ulcer disease is Helicobacter pylori (H. pylori) infection and the long term use of nonsteroidal anti-inflammatory drugs (NSAIDs) such as aspirin, indomethacin, ketorolac, Motrin, Aleve.1,2 Less common causes are acid hypersecretory states like Zollinger Ellison syndrome.1

    Mucosal injury occurs when the balance between the protective factors like mucous, bicarbonate, prostaglandins, and blood flow and the destructive factors like pepsin and acid are disrupted.1,3 The imbalance leads to increased acid exposure to the mucosal lining of the stomach or the duodenum causing gastric metaplasia.1

    NSAID-induced ulcers occur due to suppression of gastric prostaglandin synthesis.4 Prostaglandins are important for mucosal integrity. Once inhibited, it makes the mucosal lining more susceptible to destruction in the setting of increased pepsin and acid. The most common presenting symptom in patients with peptic ulcer disease is dyspepsia.2 However, most patients may not have any symptoms until they develop complications of the ulcers like obstruction or perforation. According to Vakil, “80 percent of patients with endoscopically diagnosed ulcers have epigastric pain.”2

    Those that complain of gastrointestinal symptoms describe the pain as dull, gnawing, aching, localized to the epigastrium area.1 The pain is typically related to eating.1,2,3,4,5 Duodenal ulcers usually occurs a couple hours after meals and in the middle of the night.1,2 While patients with peptic ulcers have pain with eating.1,2,3,4,5 Other gastrointestinal symptoms include: belching, epigastric fullness, early satiety, nausea and occasional vomiting.1 Patients with uncomplicated peptic ulcer decease usually have nonspecific findings on physical exam.1 They may or may not have mild epigastric tenderness on palpation and guaiac-positive stools.1

    Complications of peptic ulcers include bleeding, gastric outlet obstruction, fistulization and perforation.2 Patients who present with perforation usually have sudden, severe pain with peritonitis and free air on CT scan.2,4,5 They may also present with signs and symptoms of septic shock, like tachycardia and hypotension.4

    If peptic ulcer disease is suspected an upper endoscopy is the procedure of choice for diagnosis.1 Endoscopy allows for direct visualization of the ulcer and biopsies to test for H. pylori infection and malignancy.1 On endoscopic exam, peptic ulcers appear as discrete mucosal lesions with a punched out smooth ulcer base, with a whitish fribrinoid exudate in the middle.4

     

     

    References

    1. Papadakis M, McPhee S, Rabow M. Current Medical Diagnosis & Treatment 2016.
    2. Peptic ulcer disease: Clinical manifestations and diagnosis. Uptodatecom. 2016. Available at: http://www.uptodate.com/contents/peptic-ulcer-disease-clinical-manifestations-and-diagnosis?topicKey=GAST%2F26&elapsedTimeMs=6&source=search_result&searchTerm=peptic+ulcer+disesase&selectedTitle=1%7E150&view=print&displayedView=full. Accessed July 12, 2016.
    3. Williams D. PANCE Prep Pearls. Lexington, KY: CreateSpace; 2014.
    4. Peptic Ulcer Disease: Background, Anatomy, Pathphysiology. Emedicinemedscapecom. 2016. Available at: http://emedicine.medscape.com/article/181753-overview. Accessed July 12, 2016.
    5. Hcriz C. Stomach Disorders [powerpoint] Manchester, NH: MCPHS Physician Assistant Program; 2015
  126. Question 126 of 220
    126. Question

    A 60 year old male with a history of GERD presents to the Emergency Department with LLQ abdominal pain and a fever of 100.2°F. CT scan and a CBC reveal a left-sided diverticulitis and mild leukocytosis, respectively.  He has no documented history of diverticular disease and is otherwise healthy. Management of this patient should include which of the following?

    Correct

    1. This is the correct answer. Uncomplicated diverticulitis can be treated on an outpatient basis when the patient’s abdominal pain is not severe and the fever does not exceed 39°C (102.5°F).
    2. While it is possible that this patient will need surgery in the future if there is recurrence, he currently meets the criteria for outpatient management.
    3. Diverticulitis patients who should be admitted include those with significant comorbidities, the elderly and immunosuppressed, as well as anybody with a fever >39°C (102.5°F) or significant leukocytosis. This patient does not meet any of those criteria.
    4. While this patient can be discharged with oral antibiotics, he should remain on a liquid diet until he experiences noticeable improvement. Only then should he slowly advance to a high-fiber, low-residue diet.
      References:

      1. Klingensmith, Mary E., Aziz, Abdulhameed, Bharat, Ankit, Fox, Amy C., Porembka, Matthew R. (2012). The Washington Manual of Surgery. Philadelphia: Lippincott Williams & Wilkins.
      2. Young-Fadok, T & Pemberton, JH. Nonoperative management of acute uncomplicated diverticulitis. In: UpToDate, Post TW (Ed.), UpToDate, Waltham, MA. (Accessed on April 12, 2016.)

      3. Young-Fadok, T & Pemberton, JH. Clinical Manifestations and diagnosis of acute diverticulitis in adults. In: UpToDate, Post TW (Ed.), UpToDate, Waltham, MA. (Accessed on April 12, 2016.)

    Incorrect

    1. This is the correct answer. Uncomplicated diverticulitis can be treated on an outpatient basis when the patient’s abdominal pain is not severe and the fever does not exceed 39°C (102.5°F).
    2. While it is possible that this patient will need surgery in the future if there is recurrence, he currently meets the criteria for outpatient management.
    3. Diverticulitis patients who should be admitted include those with significant comorbidities, the elderly and immunosuppressed, as well as anybody with a fever >39°C (102.5°F) or significant leukocytosis. This patient does not meet any of those criteria.
    4. While this patient can be discharged with oral antibiotics, he should remain on a liquid diet until he experiences noticeable improvement. Only then should he slowly advance to a high-fiber, low-residue diet.
      References:

      1. Klingensmith, Mary E., Aziz, Abdulhameed, Bharat, Ankit, Fox, Amy C., Porembka, Matthew R. (2012). The Washington Manual of Surgery. Philadelphia: Lippincott Williams & Wilkins.
      2. Young-Fadok, T & Pemberton, JH. Nonoperative management of acute uncomplicated diverticulitis. In: UpToDate, Post TW (Ed.), UpToDate, Waltham, MA. (Accessed on April 12, 2016.)

      3. Young-Fadok, T & Pemberton, JH. Clinical Manifestations and diagnosis of acute diverticulitis in adults. In: UpToDate, Post TW (Ed.), UpToDate, Waltham, MA. (Accessed on April 12, 2016.)

  127. Question 127 of 220
    127. Question

    A 25 year old female has been referred to your office from her gynecologist who discovered a new breast mass during her clinical breast exam. Ultrasound shows a solid oval, non-lobular mass that is well circumscribed and hypoechoic. On exam, breasts are symmetric with no noted skin changes, nipple discharge, or nipple inversion. No noted axillary lymphadenopathy. You note a single, 3 cm diameter, round rubbery mass with sharp edges that is mobile and nontender. The mass is located in the upper outer quadrant of the right breast at 10 o’clock, 2cm from the nipple. Patient has no family history of breast cancer and reports no change in breast mass since her last visit.

    Based on physical exam and ultrasound findings, what is the most likely diagnosis?

    Correct

    Physical exam findings and ultrasound findings are most consistent with a fibroadenoma. Recognized frequently in adolescence and premenopausal women, fibroadenomas result from developmental abnormalities of a breast lobule. Diagnosis is confirmed with the gold standard triple test (physical exam findings, core-needle biopsy and ultrasound). If patients are certain they want the fibroadenoma removed they can forego biopsy and jump right to surgical excision, cryoablation or ultrasound-guided vacuum assisted biopsy. Conservative management is adequate (especially in young women) if they continue to follow up every 3-6 months with repeat ultrasound and breast exam. Suspected fibroadenoma in women 30 and older should be biopsied. If a breast mass is diagnosed as a fibroadenoma with a triple test, the patient may elect to leave the mass alone. If the patient develops symptoms or the mass grows greater than 20 percent in diameter in 6 months then excision becomes necessary to rule out malignancy.

    Lipomas of the breast are benign and consist of fat cells which form a discrete, soft , nontender mass. Indications for their excision include enlarging mass or diagnostic uncertainty.

    Physical exam findings concerning for malignancy include retraction signs, abnormal contours, skin dimpling, nipple retraction and deviation, edema of the skin and Paget’s disease of the nipple. A single firm, nontender mass is usually present with an irregular or stellate shape. The mass may be hard to delineate from surrounding tissue and may be fixed to skin or underlying tissue making it nonmobile. Mammography studies showing a spiculated focal mass is most specific for malignancy. If the mass is non-calcified, its density is telling of malignancy.Linear or branching microcalcifications and clusters of microcalcifications also may indicate malignancy.

    Cysts tend to occur in women between the ages of 30 to 50. Cysts arise from the terminal duct lobular unit of the breast due to blockage or distension of this unit. Women with cysts generally have single or multiple cysts which may fluctuate (by hormonal influence) in size, number and symptoms. Pain and tenderness is more likely prior to onset of menses and often persists throughout menses. Cysts are characterized as fluid filled, round, soft to firm, well-delineated , mobile masses that may be tender. Ultrasound has the capability to differentiate a solid mass from a cyst and is useful in the evaluation of breast masses. Cysts can also be identified by aspiration.

     

     

     

    Resources

    Bickly, Lynn. Bates’ Guide to Physical Examination. 11th ed. N.p.: Lippincott Williams &            Wilkins, 2008. Print.

    Elsamaloty H, Al-Natour M, Haswah N, Holz S, Elsayes KM. Introduction to Women’s    Imaging. In: Elsayes KM, Oldham SA. eds. Introduction to Diagnostic Radiology. New York, NY: McGraw-Hill;             2015.http://accessmedicine.mhmedical.com.une.idm.oclc.org/content.aspx?bookid=1562            &Sectionid=95880501. Accessed March 03, 2016.

    Giuliano AE, Hurvitz SA. Breast Disorders. In: Papadakis MA, McPhee SJ, Rabow MW. eds.     Current Medical Diagnosis & Treatment 2016. New York, NY: McGraw-Hill; 2016.             http://accessmedicine.mhmedical.com.une.idm.oclc.org/content.aspx?bookid=1585&Sect            ionid=97189113. Accessed March 03, 2016.

    Hoffman BL, Schorge JO, Schaffer JI, Halvorson LM, Bradshaw KD, Cunningham F, Calver      LE. Chapter 12. Breast Disease. In: Hoffman BL, Schorge JO, Schaffer JI, Halvorson               LM, Bradshaw KD, Cunningham F, Calver LE. eds. Williams Gynecology, 2e. New      York, NY: McGraw-Hill; 2012.             http://accessmedicine.mhmedical.com.une.idm.oclc.org/content.aspx?bookid=399&Secti            onid=41722300. Accessed March 03, 2016

    Hunt KK, Robertson JR, Bland KI. The Breast. In: Brunicardi F, Andersen DK, Billiar TR, Dunn            DL, Hunter JG, Matthews JB, Pollock RE. eds. Schwartz’s Principles of Surgery,          10e. New York, NY: McGraw-Hill; 2014.http://accessmedicine.mhmedical.com.une.idm.oclc.org/content.aspx?bookid=980&            Sectionid=59610859. Accessed March 03, 2016.

    Laronga, C and Tollin, S and Mooney, B. Breast Cysts: Clincal Manifestations, Diagnosis and             Management. In:UpToDate, Chen W (Ed), UpToDate, Waltham, MA. (Accessed on         March 3 2016).             http://www.uptodate.com/contents/breast-cysts-clinical-manifestations-diagnosis-and-            management?source=search_result&search=breast+cysts&selectedTitle=1~8

    Sabel, MS.Overview of Benign Breast Disease. In: UpToDate, Chen W (Ed),        UpToDate, Waltham, MA. (Accessed on March 3,           2016.)http://www.uptodate.com/contents/overview-of-benign-breast-            disease?source=search_result&search=fibroadenoma&selectedTitle=1~17

    Venkataraman, S and Slanetz, P. Breast Imaging for Cancer Screening: Mammography and             Ultrasonography. In:UpToDate, Sokol N (Ed), UpToDate, Waltham, MA. (Accessed on   March 3, 2016). http://www.uptodate.com/contents/breast-imaging-for-cancer-screening-     mammography-and-            ultrasonography?source=search_result&search=breast+cancer+ultrasound+findings&sele            ctedTitle=2~150#

     

    Incorrect

    Physical exam findings and ultrasound findings are most consistent with a fibroadenoma. Recognized frequently in adolescence and premenopausal women, fibroadenomas result from developmental abnormalities of a breast lobule. Diagnosis is confirmed with the gold standard triple test (physical exam findings, core-needle biopsy and ultrasound). If patients are certain they want the fibroadenoma removed they can forego biopsy and jump right to surgical excision, cryoablation or ultrasound-guided vacuum assisted biopsy. Conservative management is adequate (especially in young women) if they continue to follow up every 3-6 months with repeat ultrasound and breast exam. Suspected fibroadenoma in women 30 and older should be biopsied. If a breast mass is diagnosed as a fibroadenoma with a triple test, the patient may elect to leave the mass alone. If the patient develops symptoms or the mass grows greater than 20 percent in diameter in 6 months then excision becomes necessary to rule out malignancy.

    Lipomas of the breast are benign and consist of fat cells which form a discrete, soft , nontender mass. Indications for their excision include enlarging mass or diagnostic uncertainty.

    Physical exam findings concerning for malignancy include retraction signs, abnormal contours, skin dimpling, nipple retraction and deviation, edema of the skin and Paget’s disease of the nipple. A single firm, nontender mass is usually present with an irregular or stellate shape. The mass may be hard to delineate from surrounding tissue and may be fixed to skin or underlying tissue making it nonmobile. Mammography studies showing a spiculated focal mass is most specific for malignancy. If the mass is non-calcified, its density is telling of malignancy.Linear or branching microcalcifications and clusters of microcalcifications also may indicate malignancy.

    Cysts tend to occur in women between the ages of 30 to 50. Cysts arise from the terminal duct lobular unit of the breast due to blockage or distension of this unit. Women with cysts generally have single or multiple cysts which may fluctuate (by hormonal influence) in size, number and symptoms. Pain and tenderness is more likely prior to onset of menses and often persists throughout menses. Cysts are characterized as fluid filled, round, soft to firm, well-delineated , mobile masses that may be tender. Ultrasound has the capability to differentiate a solid mass from a cyst and is useful in the evaluation of breast masses. Cysts can also be identified by aspiration.

    Resources

    Bickly, Lynn. Bates’ Guide to Physical Examination. 11th ed. N.p.: Lippincott Williams &            Wilkins, 2008. Print.

    Elsamaloty H, Al-Natour M, Haswah N, Holz S, Elsayes KM. Introduction to Women’s    Imaging. In: Elsayes KM, Oldham SA. eds. Introduction to Diagnostic Radiology. New York, NY: McGraw-Hill;             2015.http://accessmedicine.mhmedical.com.une.idm.oclc.org/content.aspx?bookid=1562            &Sectionid=95880501. Accessed March 03, 2016.

    Giuliano AE, Hurvitz SA. Breast Disorders. In: Papadakis MA, McPhee SJ, Rabow MW. eds.     Current Medical Diagnosis & Treatment 2016. New York, NY: McGraw-Hill; 2016.             http://accessmedicine.mhmedical.com.une.idm.oclc.org/content.aspx?bookid=1585&Sect            ionid=97189113. Accessed March 03, 2016.

    Hoffman BL, Schorge JO, Schaffer JI, Halvorson LM, Bradshaw KD, Cunningham F, Calver      LE. Chapter 12. Breast Disease. In: Hoffman BL, Schorge JO, Schaffer JI, Halvorson               LM, Bradshaw KD, Cunningham F, Calver LE. eds. Williams Gynecology, 2e. New      York, NY: McGraw-Hill; 2012.             http://accessmedicine.mhmedical.com.une.idm.oclc.org/content.aspx?bookid=399&Secti            onid=41722300. Accessed March 03, 2016

    Hunt KK, Robertson JR, Bland KI. The Breast. In: Brunicardi F, Andersen DK, Billiar TR, Dunn            DL, Hunter JG, Matthews JB, Pollock RE. eds. Schwartz’s Principles of Surgery,          10e. New York, NY: McGraw-Hill; 2014.http://accessmedicine.mhmedical.com.une.idm.oclc.org/content.aspx?bookid=980&            Sectionid=59610859. Accessed March 03, 2016.

    Laronga, C and Tollin, S and Mooney, B. Breast Cysts: Clincal Manifestations, Diagnosis and             Management. In:UpToDate, Chen W (Ed), UpToDate, Waltham, MA. (Accessed on         March 3 2016).             http://www.uptodate.com/contents/breast-cysts-clinical-manifestations-diagnosis-and-            management?source=search_result&search=breast+cysts&selectedTitle=1~8

    Sabel, MS.Overview of Benign Breast Disease. In: UpToDate, Chen W (Ed),        UpToDate, Waltham, MA. (Accessed on March 3,           2016.)http://www.uptodate.com/contents/overview-of-benign-breast-            disease?source=search_result&search=fibroadenoma&selectedTitle=1~17

    Venkataraman, S and Slanetz, P. Breast Imaging for Cancer Screening: Mammography and             Ultrasonography. In:UpToDate, Sokol N (Ed), UpToDate, Waltham, MA. (Accessed on   March 3, 2016). http://www.uptodate.com/contents/breast-imaging-for-cancer-screening-     mammography-and-            ultrasonography?source=search_result&search=breast+cancer+ultrasound+findings&sele            ctedTitle=2~150#

  128. Question 128 of 220
    128. Question

    46 year old male presents to the ER with an acute onset of abdominal pain. The patient reports vomiting what looks like coffee grounds, and has had dark tarry stools. The patient seems to be a bit confused, and reports his heart feels like it is racing. On exam you have a difficult time hearing bowel sounds, he has a positive peritoneal sign, his abdomen is tender to palpation and there is tympanic sound when the abdomen is percussed. The abdominal x-ray shows fee air under the diaphragm. The labs that are obtained stat, reveal elevated WBC, elevated amylase and low hgb. What is the initial treatment for this patient?

    Correct

    A perforated gastric ulcer usually has an acute onset of abdominal pain in the upper part of the abdomen. Patients will report vomiting coffee grounds, which is blood that has accumulated in the stomach due to the bleeding ulcer and will also report black tarry stools. Many times on exam, bowel sounds are not able to be heard, along with a tympanic sound on percussion due to the perforation of the gastric wall. Some patients may also exhibit signs of AMS, hypotension and tachycardia due to their fluid loss. An abdominal x-ray will show free air, but the diagnostic test of choice is an upper endoscopy.

     

    Though the patient will need urgent surgery, the most important initial step is to start to correct their fluid deficit, initiate a proton pump inhibitor, and use nasogastric suctioning to determine if there is blood in the stomach, which strongly suggests a perforated gastric ulcer. However, an upper endoscopy is the diagnostic test that is performed to determine where the bleeding is originating from. A foley catheter is placed when a patient has their surgery, however it is not the initial treatment. Antibiotics and PPI’s are usually administered, however again, it is not the initial treatment for the patient. At the time the patient presents and your diagnosis is a perforated gastric ulcer, fluid resuscitation and gastric lavage are the initial treatment.

    References

    1. Peptic Ulcer. http://www.medbullets.com. Accessed on 1/15/2017.

     

     

    Incorrect

    A perforated gastric ulcer usually has an acute onset of abdominal pain in the upper part of the abdomen. Patients will report vomiting coffee grounds, which is blood that has accumulated in the stomach due to the bleeding ulcer and will also report black tarry stools. Many times on exam, bowel sounds are not able to be heard, along with a tympanic sound on percussion due to the perforation of the gastric wall. Some patients may also exhibit signs of AMS, hypotension and tachycardia due to their fluid loss. An abdominal x-ray will show free air, but the diagnostic test of choice is an upper endoscopy.

     

    Though the patient will need urgent surgery, the most important initial step is to start to correct their fluid deficit, initiate a proton pump inhibitor, and use nasogastric suctioning to determine if there is blood in the stomach, which strongly suggests a perforated gastric ulcer. However, an upper endoscopy is the diagnostic test that is performed to determine where the bleeding is originating from. A foley catheter is placed when a patient has their surgery, however it is not the initial treatment. Antibiotics and PPI’s are usually administered, however again, it is not the initial treatment for the patient. At the time the patient presents and your diagnosis is a perforated gastric ulcer, fluid resuscitation and gastric lavage are the initial treatment.

    References

    1. Peptic Ulcer. http://www.medbullets.com. Accessed on 1/15/2017.

  129. Question 129 of 220
    129. Question

    Your 56 year old patient sees you for a regular medication follow up. Two months ago, you started him on Parnate (tranylcypromine), a monamine oxidase inhibitor (MAOI) for the treatment of depression, after failure to respond to various other antidepressant medication trial. While reviewing his diet since starting the Parnate , he tells you he has been drinking beer and red wine a few times a week. He reports he continues to eat bacon for breakfast, loves his hot dogs, and cannot give up cheese, particularly the aged cheeses. You are most concerned that if the patient does not stop consuming the above mentioned foods and beverages, he will experience what?

    Correct

    Serotonin Syndrome is associated with the use of SNRIs/SNRIS/MAOIs and is characterized by abdominal pain, diarrhea, diaphoresis, fever, tachycardia, elevated blood pressure, delirium, muscle spasms, increased motor activity, irritability/mood swings, hostility, irrational thought process, and seizures. In severe cases, fever can become dangerously high and there is risk of cardiovascular shock and even death. Serotonin Syndrome is rare but life-threatening and is the possible result of too high a dose of an antidepressant medication or interactions with other medications. Risk for Serotonin Syndrome is highest when an SSRI/SNRI is combined with an MAOI or when an MAOI is given to someone who was taking an SSRI/SNRI without at least a 14 day (but up to 5 week) washout period of the SSRI/SNRI. Serotonin syndrome is not associated with any particular food consumption during treatment with an antidepressant.

    Agranulocytosis is a blood dyscrasia. It is characterized by symptoms such as sore throat, fever, malaise, and mouth sores, as well as other possible flu like symptoms. Agranulocytosis is diagnosed using a CBC w/Differential to determine the presence of leukopenia, or agranulocytosis. Patients at risk for developing agranulocytosis are either conventional or atypical antipsychotics. If not properly diagnosed and treated, agranulocytosis can result in death. Treatment generally includes removing the offending agent, monitoring for any signs/symptoms of infection, and treating those accordingly. Agranulocytosis is not associated with antidepressant use.

    Hypothyroidism refers to an underactive thyroid, meaning it does not produce enough thyroid stimulating hormone (TSH). While hypothyroidism is physiological in nature, it can often times present like a depressive disorder. Signs and symptoms of hypothyroidism can include, but are not limited to, weight loss, fatigue, depression, poor memory, body aches, and muscle weakness. Hypothyroidism can be the result of things such as autoimmune disorders, treatment of hyperthyroidism, and medications. Hypothyroidism is usually diagnosed by obtaining a TSH level, and at times, other thyroid tests such as Free T3 and T4. Treatment is usually with a thyroid hormone replacement medication, such as levothyroxine.

    Hypertensive crisis refers to a rapid and severe rise in blood pressure, and is considered to be an emergency. In rare instances, a hypertensive crisis can lead to a hemorraghic stroke (bleeding in the brain). The mechanism of action of an MAOI is to block the enzyme that breaks down excess tyramine in the body, thus leading to a decrease in depression. If tyramine containing foods are consumed while taking an MAOI, such as Parnate, tyramine can rapidly reach dangerous levels. Tyramine can be found in small amounts in protein containing foods. However, the more aged the food, the higher the levels of tyramine. Foods that contain tyramine include red wine, tap or home brewed beer, processed, aged, or smoked meats, aged cheeses, pickled or fermented foods, dried or overripe fruits, certain sauces, etc. Symptoms of a hypertensive crisis include severe headache, tachycardia, shortness of breath, nausea and vomiting, sweating, chest pain, nose bleeds, and confusion. Treatment of a hypertensive crisis may include hospitalization and use of oral and/or intravenous antihypertensive medications.

    References:

    Depression (major depressive disorder). (n.d.). Retrieved June 10, 2016, from http://www.mayoclinic.org/diseases-conditions/depression/expert-answers/maois/faq-20058035

    Herzog, E. A. & Varcarolis, E. M. (2010). Schizophrenia. In Varcarolis, E. M. & Halter, M. J. (Eds.), Foundations of Psychiatric Mental Health Nursing: A Clinical Approach (pp. 306-343). St. Louis, MO: W.B. Saunders.

    High blood pressure (hypertension). (n.d.). Retrieved June 10, 2016, from http://www.mayoclinic.org/diseases-conditions/high-blood-pressure/expert-answers/hypertensive-crisis/faq-20058491

    Kozy, M. &Varcarolis, E. M. (2010). Depressive Disorders. In Varcarolis, E. M. & Halter, M. J. (Eds.), Foundations of Psychiatric Mental Health Nursing: A Clinical Approach (pp. 246-279). St. Louis, MO: W.B. Saunders

    Incorrect

    Serotonin Syndrome is associated with the use of SNRIs/SNRIS/MAOIs and is characterized by abdominal pain, diarrhea, diaphoresis, fever, tachycardia, elevated blood pressure, delirium, muscle spasms, increased motor activity, irritability/mood swings, hostility, irrational thought process, and seizures. In severe cases, fever can become dangerously high and there is risk of cardiovascular shock and even death. Serotonin Syndrome is rare but life-threatening and is the possible result of too high a dose of an antidepressant medication or interactions with other medications. Risk for Serotonin Syndrome is highest when an SSRI/SNRI is combined with an MAOI or when an MAOI is given to someone who was taking an SSRI/SNRI without at least a 14 day (but up to 5 week) washout period of the SSRI/SNRI. Serotonin syndrome is not associated with any particular food consumption during treatment with an antidepressant.

    Agranulocytosis is a blood dyscrasia. It is characterized by symptoms such as sore throat, fever, malaise, and mouth sores, as well as other possible flu like symptoms. Agranulocytosis is diagnosed using a CBC w/Differential to determine the presence of leukopenia, or agranulocytosis. Patients at risk for developing agranulocytosis are either conventional or atypical antipsychotics. If not properly diagnosed and treated, agranulocytosis can result in death. Treatment generally includes removing the offending agent, monitoring for any signs/symptoms of infection, and treating those accordingly. Agranulocytosis is not associated with antidepressant use.

    Hypothyroidism refers to an underactive thyroid, meaning it does not produce enough thyroid stimulating hormone (TSH). While hypothyroidism is physiological in nature, it can often times present like a depressive disorder. Signs and symptoms of hypothyroidism can include, but are not limited to, weight loss, fatigue, depression, poor memory, body aches, and muscle weakness. Hypothyroidism can be the result of things such as autoimmune disorders, treatment of hyperthyroidism, and medications. Hypothyroidism is usually diagnosed by obtaining a TSH level, and at times, other thyroid tests such as Free T3 and T4. Treatment is usually with a thyroid hormone replacement medication, such as levothyroxine.

    Hypertensive crisis refers to a rapid and severe rise in blood pressure, and is considered to be an emergency. In rare instances, a hypertensive crisis can lead to a hemorraghic stroke (bleeding in the brain). The mechanism of action of an MAOI is to block the enzyme that breaks down excess tyramine in the body, thus leading to a decrease in depression. If tyramine containing foods are consumed while taking an MAOI, such as Parnate, tyramine can rapidly reach dangerous levels. Tyramine can be found in small amounts in protein containing foods. However, the more aged the food, the higher the levels of tyramine. Foods that contain tyramine include red wine, tap or home brewed beer, processed, aged, or smoked meats, aged cheeses, pickled or fermented foods, dried or overripe fruits, certain sauces, etc. Symptoms of a hypertensive crisis include severe headache, tachycardia, shortness of breath, nausea and vomiting, sweating, chest pain, nose bleeds, and confusion. Treatment of a hypertensive crisis may include hospitalization and use of oral and/or intravenous antihypertensive medications.

    References:

    Depression (major depressive disorder). (n.d.). Retrieved June 10, 2016, from http://www.mayoclinic.org/diseases-conditions/depression/expert-answers/maois/faq-20058035

    Herzog, E. A. & Varcarolis, E. M. (2010). Schizophrenia. In Varcarolis, E. M. & Halter, M. J. (Eds.), Foundations of Psychiatric Mental Health Nursing: A Clinical Approach (pp. 306-343). St. Louis, MO: W.B. Saunders.

    High blood pressure (hypertension). (n.d.). Retrieved June 10, 2016, from http://www.mayoclinic.org/diseases-conditions/high-blood-pressure/expert-answers/hypertensive-crisis/faq-20058491

    Kozy, M. &Varcarolis, E. M. (2010). Depressive Disorders. In Varcarolis, E. M. & Halter, M. J. (Eds.), Foundations of Psychiatric Mental Health Nursing: A Clinical Approach (pp. 246-279). St. Louis, MO: W.B. Saunders

  130. Question 130 of 220
    130. Question

    A 38 year old G2P2 female presents to the office with irregular periods and excess hair growth. She has been on oral contraceptives since menarche other than the two time periods when she was trying for pregnancy. She admits to having difficulty becoming pregnant and eventually used clomiphene (a medication to help her ovulate), which was successful. She has since had a tubal ligation. She denies symptoms of menopause and admits to premenstrual symptoms such as bloating and headache, but has not menstruated for the past 3 months. Last year she had a 6 month period of amenorrhea. On exam, increased body hair is noted along the linea alba of the lower abdomen and patient admits to managing a minor amount of facial hair. Stigmata of Cushing Syndrome are absent. Which of the following tests will confirm the most likely diagnosis?

    Correct

    Answer is B. All of the above tests are necessary to narrow down the differential diagnosis in this patient. Any woman of reproductive age with irregular menses should have a pregnancy test despite using birth control measures, because no birth control measure provides a 100 percent guarantee. In addition to the history obtained in the clinical vignette, other important components of patient history should be elicited such as recent changes in diet and exercise, psychological well-being and use of prescription medications. It is important to consider early menopause. The patient should be questioned about the age of menopause onset in their mother or siblings.

    This patient has polycystic ovarian syndrome (PCOS). PCOS associated oligomenorrhea or amenorrhea typically begins with menarche but may be difficult to detect in young women. Irregular menstrual cycles are common for the first couple of years after menarche and many young women start hormonal contraceptives during this time period which can mask symptoms.  PCOS is a cause of infertility however, discontinuation of hormonal contraceptives for desired pregnancy can lead to infertility for six months or more in some cases. Patients may present with clinical (hirsutism or acne) and or laboratory evidence of androgen excess. Associated obesity, metabolic syndrome or diabetes are typically present in women with PCOS, placing them at increased risk for heart disease. Therefore, appropriate serum chemistries should be followed regularly. FSH and estradiol levels are low or normal while LH levels are elevated. Visualizing small ovarian cysts on ultrasound helps confirm the diagnosis. Weight loss and exercise are encouraged. Hirsutism can be treated with agents such as spirinolactone, and low dose oral contraceptives help regulate cycles in addition to treating hirsutism. Metformin can assist with metabolic derangements and infertility can be treated with clomiphene as referenced in the clinical vignette.

    At this woman’s age, premature ovarian failure (women <40) is a possibility. With this condition, women are no longer able to regulate hormonal levels at the level of the hypothalamus and pituitary, disrupting the bodies feedback loop. In response to this dysregulation, LH and more reliably FSH levels are high. Generally more than one measurement of these hormones is necessary to nail down the diagnosis as ovarian function may fluctuate. Certain chromosomal abnormalities, and exposure to radiation and chemotherapy make the diagnosis more likely as these conditions can be associated with premature ovarian failure. Women with premature ovarian failure tend to present with symptoms of menopause.

    The presence of a pituitary adenoma could lead to abnormalities in TSH, prolactin, FSH and LH. Some patients with increased levels of prolactin experience galactorrhea. Patients undergoing MRI for other reasons sometimes have pituitary adenomas incidentally. Mass effect can cause neurological symptoms. TSH abnormalities as a result of hypothyroidism or hyperthyroidism have the potential to cause amenorrhea. Altered serum chemistries warrant further work up to determine the presence of toxic multinodular goiter, Graves disease or thyroiditis. Additionally, congenital adrenal hyperplasia and androgen-secreting adrenal tumors produce androgen levels sufficiently high to cause changes in ovulatory cycles.

    References

    Hall JE. Menstrual Disorders and Pelvic Pain. In: Kasper D, Fauci A, Hauser S, Longo D,      Jameson J, Loscalzo J. eds. Harrison’s Principles of Internal Medicine, 19e. New York, NY: McGraw-Hill; 2015. http://accessmedicine.mhmedical.com.une.idm.oclc.org/content.aspx?bookid=1130&Sectionid=79727068. Accessed March 01, 2016.

    Hoffman BL, Schorge JO, Schaffer JI, Halvorson LM, Bradshaw KD, Cunningham F, Calver               LE. Chapter 17. Polycystic Ovarian Syndrome and Hyperandrogenism. In: Hoffman BL, Schorge JO, Schaffer JI, Halvorson LM, Bradshaw KD, Cunningham F, Calver LE. eds. Williams Gynecology, 2e. New York, NY: McGraw-Hill; 2012.

    Nelson, LM, Clinical manifestations and evaluation of spontaneous primary ovarian         insufficiency (premature ovarian failure). In: UpToDate, Martin, KA (Ed), UpToDate,          Waltham, MA. (Accessed on March 1, 2016).

    Snyder, PJ. Causes, Presentation and Evaluation of Sellar Masses. In: UpToDate, Cooper DS (Ed), UpToDate, Waltham, MA. (Accessed on March 1, 2016).

    Welt, CK and Barbeiri, RL, Evaluation and Management of Secondary Amenorrhea. In: UpToDate, Martin, KA (Ed), UpToDate, Waltham, MA. (Accessed on March 1, 2016).

    Woo J, Armstrong AY. Gynecologic Disorders. In: Papadakis MA, McPhee SJ, Rabow MW. eds. Current Medical Diagnosis & Treatment 2016. New York, NY: McGraw-Hill; 2016.

    Incorrect

    Answer is B. All of the above tests are necessary to narrow down the differential diagnosis in this patient. Any woman of reproductive age with irregular menses should have a pregnancy test despite using birth control measures, because no birth control measure provides a 100 percent guarantee. In addition to the history obtained in the clinical vignette, other important components of patient history should be elicited such as recent changes in diet and exercise, psychological well-being and use of prescription medications. It is important to consider early menopause. The patient should be questioned about the age of menopause onset in their mother or siblings.

    This patient has polycystic ovarian syndrome (PCOS). PCOS associated oligomenorrhea or amenorrhea typically begins with menarche but may be difficult to detect in young women. Irregular menstrual cycles are common for the first couple of years after menarche and many young women start hormonal contraceptives during this time period which can mask symptoms.  PCOS is a cause of infertility however, discontinuation of hormonal contraceptives for desired pregnancy can lead to infertility for six months or more in some cases. Patients may present with clinical (hirsutism or acne) and or laboratory evidence of androgen excess. Associated obesity, metabolic syndrome or diabetes are typically present in women with PCOS, placing them at increased risk for heart disease. Therefore, appropriate serum chemistries should be followed regularly. FSH and estradiol levels are low or normal while LH levels are elevated. Visualizing small ovarian cysts on ultrasound helps confirm the diagnosis. Weight loss and exercise are encouraged. Hirsutism can be treated with agents such as spirinolactone, and low dose oral contraceptives help regulate cycles in addition to treating hirsutism. Metformin can assist with metabolic derangements and infertility can be treated with clomiphene as referenced in the clinical vignette.

    At this woman’s age, premature ovarian failure (women <40) is a possibility. With this condition, women are no longer able to regulate hormonal levels at the level of the hypothalamus and pituitary, disrupting the bodies feedback loop. In response to this dysregulation, LH and more reliably FSH levels are high. Generally more than one measurement of these hormones is necessary to nail down the diagnosis as ovarian function may fluctuate. Certain chromosomal abnormalities, and exposure to radiation and chemotherapy make the diagnosis more likely as these conditions can be associated with premature ovarian failure. Women with premature ovarian failure tend to present with symptoms of menopause.

    The presence of a pituitary adenoma could lead to abnormalities in TSH, prolactin, FSH and LH. Some patients with increased levels of prolactin experience galactorrhea. Patients undergoing MRI for other reasons sometimes have pituitary adenomas incidentally. Mass effect can cause neurological symptoms. TSH abnormalities as a result of hypothyroidism or hyperthyroidism have the potential to cause amenorrhea. Altered serum chemistries warrant further work up to determine the presence of toxic multinodular goiter, Graves disease or thyroiditis. Additionally, congenital adrenal hyperplasia and androgen-secreting adrenal tumors produce androgen levels sufficiently high to cause changes in ovulatory cycles.

    References

    Hall JE. Menstrual Disorders and Pelvic Pain. In: Kasper D, Fauci A, Hauser S, Longo D,      Jameson J, Loscalzo J. eds. Harrison’s Principles of Internal Medicine, 19e. New York, NY: McGraw-Hill; 2015. http://accessmedicine.mhmedical.com.une.idm.oclc.org/content.aspx?bookid=1130&Sectionid=79727068. Accessed March 01, 2016.

    Hoffman BL, Schorge JO, Schaffer JI, Halvorson LM, Bradshaw KD, Cunningham F, Calver               LE. Chapter 17. Polycystic Ovarian Syndrome and Hyperandrogenism. In: Hoffman BL, Schorge JO, Schaffer JI, Halvorson LM, Bradshaw KD, Cunningham F, Calver LE. eds. Williams Gynecology, 2e. New York, NY: McGraw-Hill; 2012.

    Nelson, LM, Clinical manifestations and evaluation of spontaneous primary ovarian         insufficiency (premature ovarian failure). In: UpToDate, Martin, KA (Ed), UpToDate,          Waltham, MA. (Accessed on March 1, 2016).

    Snyder, PJ. Causes, Presentation and Evaluation of Sellar Masses. In: UpToDate, Cooper DS (Ed), UpToDate, Waltham, MA. (Accessed on March 1, 2016).

    Welt, CK and Barbeiri, RL, Evaluation and Management of Secondary Amenorrhea. In: UpToDate, Martin, KA (Ed), UpToDate, Waltham, MA. (Accessed on March 1, 2016).

    Woo J, Armstrong AY. Gynecologic Disorders. In: Papadakis MA, McPhee SJ, Rabow MW. eds. Current Medical Diagnosis & Treatment 2016. New York, NY: McGraw-Hill; 2016.

  131. Question 131 of 220
    131. Question

    A 71 year old male presents to the ER with severe SOB and hemoptysis. Past medical history is significant for COPD, CHF, AF and CAD s/p CABG 19 years ago. After speaking with the patient he tells you that his SOB has been worsening over the past month. He can only walk up 2 steps at a time before having to stop to catch his breath. He also gets chest pain with activity for which he was prescribed nitroglycerin and uses daily. He denies loss of consciousness but does describe multiple episodes of feeling dizzy with standing and when doing activities. On exam the patient is sitting upright and appears to be out of breath. Heart sounds are irregular, irregular, a 5/6 murmur is noted along with an additional heart sound just before the AV valves closes. There is a slow, upward deflection of the carotids. Lower extremities reveal 2+ pitting edema. EKG shows atrial fibrillation with left ventricular hypertrophy. Which of the following murmurs is most likely present on cardiac exam?

    Correct

    The correct answer is D. A is incorrect as this is the typical murmur associated with mitral stenosis. B is incorrect as this is the typical murmur associated with coarctation. C is incorrect as this is the murmur associated with patent ductus arteriosus. E is incorrect as this is the murmur associated with aortic regurgitation.

    The above case and question represent a specific valvular heart condition known as aortic stenosis (AS). Aortic stenosis is a disease process affecting the leaflets of the aortic valve. The valves become stenotic and the opening becomes narrow, causing a left ventricular outflow obstruction of blood flow across the valve. (1) Gradually over time the outflow obstruction and pressure load on the heart muscle increases causing left ventricular dysfunction. (1) According to Lester and Abbas (2016) aortic stenosis is the most common indication for valve replacement in North America and Europe. (2)

    Two of the most common causes of AS is calcification and degeneration of a trileaflet or congenitally abnormal unicuspid or bicuspid valve, rather than tricuspid. Calcific and degenerative aortic stenosis, is thought to be related to calcium deposition due to processes similar to what occurs in atherosclerotic vascular disease. Approximately 25% of patients over age 65 years and 35% of those over age 70 years have echocardiographic evidence of aortic valve thickening (sclerosis). (4) About 10–20% of these will progress to hemodynamically significant aortic stenosis over a period of 10–15 years. (4)

    Patients with congenitally abnormal valves may develop symptoms during childhood or adolescence if the stenosis is severe enough (1). However, patients do not typically become symptomatic until their 50’s or 60’s when calcification and degeneration of the valve develops(8).
    A less common cause of AS is rheumatic valve disease. Since the development of antibiotics it has become a rare cause of valve disease in industrialized countries. However, it still remains a prevalent cause in developing countries(4). Other rare causes of AS are connective tissue diseases such as systemic lupus erythematosus and orchronosis. Some common risk factors associated with AS include hypertension, hypercholesterolemia, and smoking (8).

    Patients with AS develop symptoms gradually overtime. Initially they start out asymptomatic until the stenosis becomes more severe. The most common symptoms patients usually present with is exertional dyspnea. Two other common symptoms patients often develop is chest pain and syncope associated with exertion as well. Together these three symptoms make up, what is known as the classic triad of AS.

    The most classic physical exam finding associated with AS is the crescendo-decrescendo systolic ejection murmur heard short after S1 and just before S2(8). The murmur is a low pitched, harsh murmur heard best at the 2nd intercostal space in the right upper sternal boarder and radiates to the carotids(8). The murmur decreases with squatting and increases with valsalva maneuver (8). In mild or moderate cases where the valve is still pliable, an ejection click may be heard before the murmur. Some other symptoms that tend to be associated with more severe AS are; a palpable LV heave or thrill, a weak to absent aortic second sound, or reversed splitting of the second sound, weak and delayed carotid upstroke (pulsus parvus et tardus) and pulses alternans (8).

    The diagnostic test of choice for determining valvular heart disease and measuring the severity of the valve disease is an echocardiograph/Doppler(1,5,8). Catheterization can also be used a test to measure the severity of the valve disease and is usually reserved for cases where there is a discrepancy between symptoms and the echocardiography/doppler information(1). An electrocardiogram (EKG) may show non-specific findings of left ventricular hypertrophy or secondary repolarization changes, or it can be normal (3).

    The only definitive treatment for AS is an aortic valve replacement (AVR)(8). Due to the invasiveness of the valve replacement and the high risk involved, patients do not undergo replacement until the patients with severe AS becomes symptomatic (2). In the meantime noninvasive medical therapy is used to treat symptomatic patients that do not yet qualify for valve replacement. In these patients medical treatment is aimed at treating the patients symptoms associated with their AS.

    Patients with heart failure are typically treated cautiously with low doses of diuresis and angiotensin-converting enzyme (ACE) inhibitors(1,5,6,8). Digoxin is used for patients with left dysfunction and or atrial fibrillation (1,5,6,8). Once patients reach the point that surgical intervention is warranted patients can undergo surgical replacement or a transcatheter approach. Typically patients that have multiple comorbidities and are high risk surgical candidates undergo the transcatheter aortic valve repair TAVR). (2)

    References

    1. Bojar R. Manual Of Perioperative Care In Adult Cardiac Surgery. Chichester, UK: Wiley-Blackwell; 2011:18-26.
    2. Brecker S, Gaasch W, Aldea G. Transcatheter aortic valve replacement: Indications and outcomes. Transcatheter aortic valve replacement: Indications and outcomes. https://www.uptodate.com/contents/transcatheter-aortic-valve-replacement-indications-and-outcomes. Published October 25, 2016. Accessed October 20, 2016.
    3. Gaasch, MD W. Indications for valve replacement in aortic stenosis in adults. Up to date. 2016. Available at: https://www.uptodate.com/contents/indications-for-valve-replacement-in-aortic-stenosis-in-adults?source=search_result&search=aortic%20stenosis&selectedTitle=2~150#H1. Accessed October 20, 2016.
    4. Gaasch, MD W. Natural history, epidemiology, and prognosis of aortic stenosis. Up to date. 2016. Available at: https://www.uptodate.com/contents/natural-history-epidemiology-and-prognosis-of-aortic-stenosis?source=search_result&search=aortic%20stenosis&selectedTitle=5~150. Accessed October 20, 2016.
    5. Otto, MD C. Clinical manifestations and diagnosis of aortic stenosis in adults. Uptodate. 2016. Available at: https://www.uptodate.com/contents/clinical-manifestations-and-diagnosis-of-aortic-stenosis-in-adults?source=search_result&search=aortic%20stenosis&selectedTitle=1~150. Accessed October 20, 2016.
    6. Otto, MD Cooper, MD S. Medical management of symptomatic aortic stenosis. Up to date. 2016. Available at: https://www.uptodate.com/contents/medical-management-of-symptomatic-aortic-stenosis?source=search_result&search=aortic%20stenosis&selectedTitle=3~150#H1. Accessed October 20, 2016.
    7. Ren, MD X. Aortic Stenosis: Practice Essentials, Background, Pathophysiology. Emedicinemedscapecom. 2016. Available at: http://emedicine.medscape.com/article/150638-overview?src=refgatesrc1. Accessed October 20, 2016.
    8. McPhee SJ, Papadakis MA. Current Medical Diagnosis &Amp; Treatment 2011. New York: McGraw-Hill Medical; 2011.

    Incorrect

    The correct answer is D. A is incorrect as this is the typical murmur associated with mitral stenosis. B is incorrect as this is the typical murmur associated with coarctation. C is incorrect as this is the murmur associated with patent ductus arteriosus. E is incorrect as this is the murmur associated with aortic regurgitation.

    The above case and question represent a specific valvular heart condition known as aortic stenosis (AS). Aortic stenosis is a disease process affecting the leaflets of the aortic valve. The valves become stenotic and the opening becomes narrow, causing a left ventricular outflow obstruction of blood flow across the valve. (1) Gradually over time the outflow obstruction and pressure load on the heart muscle increases causing left ventricular dysfunction. (1) According to Lester and Abbas (2016) aortic stenosis is the most common indication for valve replacement in North America and Europe. (2)

    Two of the most common causes of AS is calcification and degeneration of a trileaflet or congenitally abnormal unicuspid or bicuspid valve, rather than tricuspid. Calcific and degenerative aortic stenosis, is thought to be related to calcium deposition due to processes similar to what occurs in atherosclerotic vascular disease. Approximately 25% of patients over age 65 years and 35% of those over age 70 years have echocardiographic evidence of aortic valve thickening (sclerosis). (4) About 10–20% of these will progress to hemodynamically significant aortic stenosis over a period of 10–15 years. (4)

    Patients with congenitally abnormal valves may develop symptoms during childhood or adolescence if the stenosis is severe enough (1). However, patients do not typically become symptomatic until their 50’s or 60’s when calcification and degeneration of the valve develops(8).
    A less common cause of AS is rheumatic valve disease. Since the development of antibiotics it has become a rare cause of valve disease in industrialized countries. However, it still remains a prevalent cause in developing countries(4). Other rare causes of AS are connective tissue diseases such as systemic lupus erythematosus and orchronosis. Some common risk factors associated with AS include hypertension, hypercholesterolemia, and smoking (8).

    Patients with AS develop symptoms gradually overtime. Initially they start out asymptomatic until the stenosis becomes more severe. The most common symptoms patients usually present with is exertional dyspnea. Two other common symptoms patients often develop is chest pain and syncope associated with exertion as well. Together these three symptoms make up, what is known as the classic triad of AS.

    The most classic physical exam finding associated with AS is the crescendo-decrescendo systolic ejection murmur heard short after S1 and just before S2(8). The murmur is a low pitched, harsh murmur heard best at the 2nd intercostal space in the right upper sternal boarder and radiates to the carotids(8). The murmur decreases with squatting and increases with valsalva maneuver (8). In mild or moderate cases where the valve is still pliable, an ejection click may be heard before the murmur. Some other symptoms that tend to be associated with more severe AS are; a palpable LV heave or thrill, a weak to absent aortic second sound, or reversed splitting of the second sound, weak and delayed carotid upstroke (pulsus parvus et tardus) and pulses alternans (8).

    The diagnostic test of choice for determining valvular heart disease and measuring the severity of the valve disease is an echocardiograph/Doppler(1,5,8). Catheterization can also be used a test to measure the severity of the valve disease and is usually reserved for cases where there is a discrepancy between symptoms and the echocardiography/doppler information(1). An electrocardiogram (EKG) may show non-specific findings of left ventricular hypertrophy or secondary repolarization changes, or it can be normal (3).

    The only definitive treatment for AS is an aortic valve replacement (AVR)(8). Due to the invasiveness of the valve replacement and the high risk involved, patients do not undergo replacement until the patients with severe AS becomes symptomatic (2). In the meantime noninvasive medical therapy is used to treat symptomatic patients that do not yet qualify for valve replacement. In these patients medical treatment is aimed at treating the patients symptoms associated with their AS.

    Patients with heart failure are typically treated cautiously with low doses of diuresis and angiotensin-converting enzyme (ACE) inhibitors(1,5,6,8). Digoxin is used for patients with left dysfunction and or atrial fibrillation (1,5,6,8). Once patients reach the point that surgical intervention is warranted patients can undergo surgical replacement or a transcatheter approach. Typically patients that have multiple comorbidities and are high risk surgical candidates undergo the transcatheter aortic valve repair TAVR). (2)

    References

    1. Bojar R. Manual Of Perioperative Care In Adult Cardiac Surgery. Chichester, UK: Wiley-Blackwell; 2011:18-26.
    2. Brecker S, Gaasch W, Aldea G. Transcatheter aortic valve replacement: Indications and outcomes. Transcatheter aortic valve replacement: Indications and outcomes. https://www.uptodate.com/contents/transcatheter-aortic-valve-replacement-indications-and-outcomes. Published October 25, 2016. Accessed October 20, 2016.
    3. Gaasch, MD W. Indications for valve replacement in aortic stenosis in adults. Up to date. 2016. Available at: https://www.uptodate.com/contents/indications-for-valve-replacement-in-aortic-stenosis-in-adults?source=search_result&search=aortic%20stenosis&selectedTitle=2~150#H1. Accessed October 20, 2016.
    4. Gaasch, MD W. Natural history, epidemiology, and prognosis of aortic stenosis. Up to date. 2016. Available at: https://www.uptodate.com/contents/natural-history-epidemiology-and-prognosis-of-aortic-stenosis?source=search_result&search=aortic%20stenosis&selectedTitle=5~150. Accessed October 20, 2016.
    5. Otto, MD C. Clinical manifestations and diagnosis of aortic stenosis in adults. Uptodate. 2016. Available at: https://www.uptodate.com/contents/clinical-manifestations-and-diagnosis-of-aortic-stenosis-in-adults?source=search_result&search=aortic%20stenosis&selectedTitle=1~150. Accessed October 20, 2016.
    6. Otto, MD Cooper, MD S. Medical management of symptomatic aortic stenosis. Up to date. 2016. Available at: https://www.uptodate.com/contents/medical-management-of-symptomatic-aortic-stenosis?source=search_result&search=aortic%20stenosis&selectedTitle=3~150#H1. Accessed October 20, 2016.
    7. Ren, MD X. Aortic Stenosis: Practice Essentials, Background, Pathophysiology. Emedicinemedscapecom. 2016. Available at: http://emedicine.medscape.com/article/150638-overview?src=refgatesrc1. Accessed October 20, 2016.
    8. McPhee SJ, Papadakis MA. Current Medical Diagnosis &Amp; Treatment 2011. New York: McGraw-Hill Medical; 2011.

  132. Question 132 of 220
    132. Question

    knee

    A 12 year-old girl presents to your office with 3 weeks of right anterior knee pain. She recently started playing basketball and the knee pain seems to increase with jumping and running. On exam you note tenderness to palpation over the tibial tubercle. Her knee pain is made worse with passive hyperflexion and active extension against resistance. She has no pain over the patellofemoral joint with repetitive flexion and there is no abnormal tracking of the patella. Bilateral AP, lateral, and merchant view x-rays are shown in figures 1, 2, and 3. What is the most likely diagnosis?

    Correct

    Osgood-Schlatter’s Disease (OSD) or traction apophysitis is an inflammation of the growth plate at the tibial tubercle where the patella tendon attaches. OSD occurs in adolescents during peak growth age, which is when skeletal growth outpaces the stretch of surrounding tendons. In OSD, the tight patella tendon pulls on the tibial apophysis causing inflammation and in some cases, partial or complete avulsion of the tibial tubercle. A partial avulsion will cause a noticeable bump on the anterior tibia which is a common finding in patients with long standing OSD.  Pain, tenderness, and swelling will be present over the tibial tubercle during palpation and pain is made worse with resisted active knee extension. The lateral radiograph of the knee is the most telling in patients with OSD, often showing irregularity and fragmentation of the tibial tubercle. 1,2

    Symptoms generally resolve with a period rest, activity modification, ice, and NSAIDs. Physical therapy can help patients stretch their quadriceps muscle to relieve tension on the tibial apophysis. OSD may cause pain for several months so a lengthy period of conservative treatment may be necessary. Athletes may play through mild pain if tolerable. An infrapatella strap may be used during sport participation to help with the pain. Severe pain may signal an impending avulsion fracture and the athlete should rest accordingly.  The pain generally subsides with rest or when skeletal growth completes, which is generally around age 14 for girls and 16 for boys. Patients should be advised however that a bump on the anterior tibia, if present, will persist through adulthood. 1

    References

    1. Pihlajamaki HK, Mattila VM, Parviainen M, Kiuru MJ, Visuri TI. Long-Term Outcome After Surgical Treatment of Unresolved Osgood-Schlatter Disease in Young Men. JBJS 2009; 91: 2350-8.
    2. Osgood-Schlatter’s disease. http://www.orthobullets.com. Accessed on 2/18/2017.
    Incorrect

    Osgood-Schlatter’s Disease (OSD) or traction apophysitis is an inflammation of the growth plate at the tibial tubercle where the patella tendon attaches. OSD occurs in adolescents during peak growth age, which is when skeletal growth outpaces the stretch of surrounding tendons. In OSD, the tight patella tendon pulls on the tibial apophysis causing inflammation and in some cases, partial or complete avulsion of the tibial tubercle. A partial avulsion will cause a noticeable bump on the anterior tibia which is a common finding in patients with long standing OSD.  Pain, tenderness, and swelling will be present over the tibial tubercle during palpation and pain is made worse with resisted active knee extension. The lateral radiograph of the knee is the most telling in patients with OSD, often showing irregularity and fragmentation of the tibial tubercle. 1,2

    Symptoms generally resolve with a period rest, activity modification, ice, and NSAIDs. Physical therapy can help patients stretch their quadriceps muscle to relieve tension on the tibial apophysis. OSD may cause pain for several months so a lengthy period of conservative treatment may be necessary. Athletes may play through mild pain if tolerable. An infrapatella strap may be used during sport participation to help with the pain. Severe pain may signal an impending avulsion fracture and the athlete should rest accordingly.  The pain generally subsides with rest or when skeletal growth completes, which is generally around age 14 for girls and 16 for boys. Patients should be advised however that a bump on the anterior tibia, if present, will persist through adulthood. 1

    References

    1. Pihlajamaki HK, Mattila VM, Parviainen M, Kiuru MJ, Visuri TI. Long-Term Outcome After Surgical Treatment of Unresolved Osgood-Schlatter Disease in Young Men. JBJS 2009; 91: 2350-8.
    2. Osgood-Schlatter’s disease. http://www.orthobullets.com. Accessed on 2/18/2017.
  133. Question 133 of 220
    133. Question

    A 56-year-old man presents to your urgent care clinic following 4 days of left eye redness, moderate pain, photophobia and blurred vision. He states he got hair product in his eye last Saturday, which has caused a significant burning sensation in his left eye. He has since been irrigating and rubbing his eye. Following instillation of fluorescein dye there is considerable uptake, and a 0.25 cm corneal abrasion is diagnosed along the inferior aspect of the pupil. What is NOT a part of the treatment plan of this patient?

    Correct

    A. Concomitant infection can slow healing of corneal abrasions so prophylactic antibiotics are often prescribed, although there is a lack of evidence to support this.  Polymyxin B/trimethroprim eye drops are a good choice for antibiotic coverage in this patient.

    B. Corneal abrasions often require frequent follow up to reevaluate and reassess, especially if healing is delayed or the epithelial defect is enlarging. This would most definitely be a part of treatment, as you want to see the patient back in your clinic to ensure his symptoms are resolving.

    C. Since this patient notes mild pain an NSAID eye drop is a good idea to alleviate some of the patient’s discomfort. This type of drop is not indicated in all types of corneal abrasion, but is however useful if the patient is in pain.

    D. Correct answer: Eye patching is not indicated for the treatment of corneal abrasion. Since this patient is presenting in mild distress and with a small corneal abrasion, it would not be indicated to include patching in this patient. Eye patching is often contraindicated in patients who are contact lens wearers, or if the mechanism of injury of the corneal abrasion involves fingernails, or vegetable matter.

    Reference
    1. Ehlers J, Shah CP. The Wills Eye Manual; Office and Emergency Room Diagnosis and Treatment of Eye Disease. Philadelphia, PA: Wolters Kluwer/Lippincott Williams & Wilkins; 2008.

     

    Incorrect

    A. Concomitant infection can slow healing of corneal abrasions so prophylactic antibiotics are often prescribed, although there is a lack of evidence to support this.  Polymyxin B/trimethroprim eye drops are a good choice for antibiotic coverage in this patient.

    B. Corneal abrasions often require frequent follow up to reevaluate and reassess, especially if healing is delayed or the epithelial defect is enlarging. This would most definitely be a part of treatment, as you want to see the patient back in your clinic to ensure his symptoms are resolving.

    C. Since this patient notes mild pain an NSAID eye drop is a good idea to alleviate some of the patient’s discomfort. This type of drop is not indicated in all types of corneal abrasion, but is however useful if the patient is in pain.

    D. Correct answer: Eye patching is not indicated for the treatment of corneal abrasion. Since this patient is presenting in mild distress and with a small corneal abrasion, it would not be indicated to include patching in this patient. Eye patching is often contraindicated in patients who are contact lens wearers, or if the mechanism of injury of the corneal abrasion involves fingernails, or vegetable matter.

    Reference
    1. Ehlers J, Shah CP. The Wills Eye Manual; Office and Emergency Room Diagnosis and Treatment of Eye Disease. Philadelphia, PA: Wolters Kluwer/Lippincott Williams & Wilkins; 2008.

     

  134. Question 134 of 220
    134. Question

    hip

    A 27 year-old female presents to your office with a few years of lower back and left hip pain. She went to a spine specialist last year who diagnosed her with a lumbar spine strain. She went through several months of physical therapy and received trigger point injections without relief. She started with a new physical therapist last week for her back who thought the pain was coming from her hip and recommended she get hip x-rays. She then called her primary care physician who ordered left hip x-rays (figure 1 and 2) and instructed her to follow-up with orthopedics. She denies any medical problems and does not drink alcohol or use tobacco. She doesn’t take any prescription medications currently but admits “I’ve taken more steroid dose packs than I can count” for her asthma and cat allergy. Which choice would be the next best test to order for this patient?

    Correct

    This patient’s x-ray findings of the left hip include flattening of the femoral head and advanced degenerative changes consistent with the diagnosis of avascular necrosis (AVN) of the femoral head. Hip AVN is a relatively common condition that is caused by a disruption of the blood supply to the femoral head. The condition is so common in fact that 10% of all total hip replacements performed in the United States are for AVN. Taking oral steroids has been linked to the disease and could potentially be the cause in this patient. AVN can occur bilaterally in up to 80% of patients so bilateral hip x-rays should always be ordered if the diagnosis has been made or is suspected. If the diagnosis of AVN is made on x-ray than no further imaging is necessary.

    Symptoms of hip and lumbar spine pathology can often overlap causing a delayed diagnosis. Knowing key physical exam findings and symptoms that differentiate the two can help prevent a delay in diagnosis and treatment. The most common symptoms that occur when the hip is the source of pathology includes groin, thigh, and buttock pain. Pain from hip pathology is usually made worse with hip flexion and internal rotation. Lumbar spine pathology usually causes back and buttock pain, and with lumbar nerve impingement, radiating pain that travels distally past the knee. Pain to palpation over the lumbar spine and a positive straight leg raise are common symptoms associated with lumbar spine pathology. This case is a great example of how a careful history and physical exam must be performed in order to make an accurate initial diagnosis. 2

    References
    1. Avascular necrosis of the Hip. http://www.orthobullets.com . Accessed on 2/14/2017.
    2. Buckland AJ, Miyamoto R, Patel RD, Slover J, Razi AE. Differentiating hip pathology from lumbar spine pathology: key points of evaluation and management. JAAOS 2017; 25: 23-24.

    Incorrect

    This patient’s x-ray findings of the left hip include flattening of the femoral head and advanced degenerative changes consistent with the diagnosis of avascular necrosis (AVN) of the femoral head. Hip AVN is a relatively common condition that is caused by a disruption of the blood supply to the femoral head. The condition is so common in fact that 10% of all total hip replacements performed in the United States are for AVN. Taking oral steroids has been linked to the disease and could potentially be the cause in this patient. AVN can occur bilaterally in up to 80% of patients so bilateral hip x-rays should always be ordered if the diagnosis has been made or is suspected. If the diagnosis of AVN is made on x-ray than no further imaging is necessary.

    Symptoms of hip and lumbar spine pathology can often overlap causing a delayed diagnosis. Knowing key physical exam findings and symptoms that differentiate the two can help prevent a delay in diagnosis and treatment. The most common symptoms that occur when the hip is the source of pathology includes groin, thigh, and buttock pain. Pain from hip pathology is usually made worse with hip flexion and internal rotation. Lumbar spine pathology usually causes back and buttock pain, and with lumbar nerve impingement, radiating pain that travels distally past the knee. Pain to palpation over the lumbar spine and a positive straight leg raise are common symptoms associated with lumbar spine pathology. This case is a great example of how a careful history and physical exam must be performed in order to make an accurate initial diagnosis. 2

    References
    1. Avascular necrosis of the Hip. http://www.orthobullets.com . Accessed on 2/14/2017.
    2. Buckland AJ, Miyamoto R, Patel RD, Slover J, Razi AE. Differentiating hip pathology from lumbar spine pathology: key points of evaluation and management. JAAOS 2017; 25: 23-24.

  135. Question 135 of 220
    135. Question

    A 17 year old female patient presents to the clinic with a 3 day history of a sore throat, fever, and fatigue for the past 3 days. She recently came back from a 2 week vacation in Utah and now has pain with swallowing. She states she “had a fever for the past 3 days and it broke last night”. She denies nausea, vomiting, cough, rhinitis, congestion, shortness of breath, trouble breathing, sputum production, abdominal pain, body aches, and rash. She is not sexually active and denies orogential contact. Her vitals today are all within normal limits. Physical exam reveals tender anterior cervical lymphadenopathy, red edematous uvula, and greyish white exudate on swollen tonsils. Her lungs are clear, there is no splenomegaly, no icterus, no eye pain, and no erythematous conjunctiva. What is the most likely causative agent for this diagnosis?

    Correct

    Group A Streptococci is correct. The patient presents with exudate on her tonsils, tender anterior cervical lymph nodes, fever, and an absent cough. All of these symptoms are suggestive of streptococcal pharyngitis.

    Incorrect: A. Mononucleosis. It is common with patients who are infected with Mono to have a prodromal period of malaise, myalgia, fatigue and low grade fever for 1-2 weeks before it becomes infectious pharyngitis. This patient has an onset of sore throat, fever, and fatigue for the past 3 days. The time course of mononucleosis is usually longer onset whereas Group A Strep is more of a “sudden onset”. B. Rhinovirus. Patients infected with rhinovirus usually present with nasal congestion, nasal discharge, sneezing, and/or cough. This patient did not have any of these symptoms. D. Gonorrhea infection of the pharynx is more commonly asymptomatic or mild symptoms. Patient is not sexually active or had any orogential contact.

    Reference
    1. Pharyngitis. http://www.medbullets.com. Accessed on 2/24/17.

    Incorrect

    Group A Streptococci is correct. The patient presents with exudate on her tonsils, tender anterior cervical lymph nodes, fever, and an absent cough. All of these symptoms are suggestive of streptococcal pharyngitis.

    Incorrect: A. Mononucleosis. It is common with patients who are infected with Mono to have a prodromal period of malaise, myalgia, fatigue and low grade fever for 1-2 weeks before it becomes infectious pharyngitis. This patient has an onset of sore throat, fever, and fatigue for the past 3 days. The time course of mononucleosis is usually longer onset whereas Group A Strep is more of a “sudden onset”. B. Rhinovirus. Patients infected with rhinovirus usually present with nasal congestion, nasal discharge, sneezing, and/or cough. This patient did not have any of these symptoms. D. Gonorrhea infection of the pharynx is more commonly asymptomatic or mild symptoms. Patient is not sexually active or had any orogential contact.

    Reference
    1. Pharyngitis. http://www.medbullets.com. Accessed on 2/24/17.

  136. Question 136 of 220
    136. Question

    vitiligo

    A 12-year-old Asian female presents to the clinic with a hypopigmented lesion present on left lateral lower back. The patient’s mother states the lesion originated one-year prior and presented originally as a small circle, approximately a few millimeters in diameter. The patient denies any symptoms, including pruritus or pain. The lesion has grown locally, but no other similar lesions have appeared thus far. The patient’s uncle had similar “white spots” on his arms and legs. Wood’s lamp examination reveals white fluorescence. Which of the following is the most appropriate first line treatment?

    Correct

    The answer is D: High-potency topical steroids. The patient presents with vitiligo that is localized (<10% TBSA) and located on the back. Clinical trials and current recommendations from the AAD advise that medium to high potency steroids be used as first line treatment for vitiligo that is involving <10% TBSA and not involving the face.

    (A) Topical antifungal is incorrect as the diagnosis is not due to a superficial fungal infection.
    (B) Low-potency steroids would not be effective; topical steroids MOA in vitiligo is via modulation of the immune response and this would not be achieved with low-potency steroids.
    (C) Topical calcineurin inhibitors are an effective treatment for vitiligo. Their indication is normally reserved for cases that involve sensitive areas such as the face, or as combination therapy with topical steroids. Head to head trials of high-potency topical steroids vs. topical calcineurin inhibitors in patients with non-facial vitiligo demonstrate a lower success rate in those who used topical calcineurin inhibitors alone.

    Epidemiology
    Worldwide occurrence of vitiligo is approximately 1-2%; with a rate of 1% in the United States alone. Vitiligo affects males and females equally, with no specific affinity to racial, ethnic, or socio-economic factors. Vitiligo may appear at any time from birth to late adulthood; peak onset occurring at 10-30 years (mean average of 20 years old). Approximately one-third of patients with vitiligo are children, and 70-80 percent of adult patients develop vitiligo prior to age 30 years.1,2

    Etiology
    The etiology of vitiligo is unknown. Patients commonly attribute the onset of their disease to specific triggering events such as physical injury or illness, sunburn, emotional stress, or pregnancy, but there are no data supporting a causative role for these factors. The frequency of comorbid autoimmune diseases is significantly elevated in patients with vitiligo and in their first-degree relatives, suggesting an autoimmune etiology for this disorder. Finally, family clustering of vitiligo suggests a genetic basis for the disease.2

    Pathophysiology
    Vitiligo is a multifactorial disorder related to various factors. It is generally agreed that there is an absence of functional melanocytes in vitiligo skin. Genetic studies indicate a multifactorial, polygenic inheritance pattern; with 25-50 percent of patients with affected relatives. Genetic studies have identified approximately 36 susceptibility loci; with the majority of the susceptibility genes encode either immunoregulatory proteins or melanocyte proteins. Specifically, AIS1 is a major susceptibility locus in Caucasians, while chromosome 4q13-q21 appears to be the main locus in Chinese families.2

    Historically, vitiligo has been associated with several autoimmune diseases, including Hashimoto’s thyroiditis, Graves’ disease, type 1 diabetes mellitus, alopecia areata, pernicious anemia, rheumatoid arthritis, and psoriasis. Multiple cytokines have also been implicated in the destruction of melanocytes in vitiligo. Several studies have documented increased expression of tumor necrosis factor-alpha, interferon-gamma, interleukin (IL)-10, and IL-17. Many hypotheses exist regarding the true pathophysiology behind vitiligo; including, but not limited to: the melanocyte self-destruction hypothesis, oxidative stress, neural hypothesis, and convergence theory.2,4

    Pertinent historical & Physical findings
    Vitiligo most commonly presents as totally amelanotic macules or patches surrounded by normal skin. The lesions characteristically have discrete margins, and can be round, oval, irregular or linear in shape. The borders are usually convex, as if the depigmenting process were “invading” the surrounding normal skin. Progressive lesions enlarge over time, at a slow or rapid rate, with macules and patches range from millimeters to centimeters in diameter and often vary in size within areas of involvement.4

    The current accepted classification system for vitiligo divides the disease into three types: localized, generalized, and universal. Localized vitiligo is described as focal, segmental, or mucosal. Focal is defined as one or more macules in a single area but not in a segmental distribution. Segmental is one or more macule involving a unilateral segment of the body, but stops at midline. Mucosal involves mucous membranes alone. Generalized category can be vulgaris, acrofacial, or mixed. Vulgaris are scattered patches that are widely distributed, while acrofacial affects only distal extremities and the face. Mixed is described as a various combination of segmental, acrofacial and/or vulgaris. Finally, universal vitiligo is complete or nearly complete depigmentation of the skin surface.4

    Other dermatological findings have been strongly correlated with vitiligo. Poliosis, a decrease or absence of color in head hair, eyebrows, and/or eyelashes, is frequently found prior to or simultaneously with the classic amelanotic macules/patches of vitiligo. Halo nevi have been identified as a precursor for vitiligo in up to 25% of patients. Lastly, friction and other types of physical trauma such as scratching, chronic pressure, or cuts along with allergic or irritant contact reactions may trigger vitiligo on areas such as the neck, elbows, and ankles. This is known as the Koebner phenomenon and has been reported in 20-60 percent of vitiligo patients.2,4

    Diagnostic Correlation
    The standard for diagnosis of vitiligo is through the use of a Wood’s lamp. The lamp is a handheld device emitting UVA light at 365 nm wavelength. Under the Wood’s light, the depigmented areas emit a bright blue-white fluorescence and appear sharply demarcated, confirming amelanosis. Although not necessary, a skin biopsy can also be used to distinguish between hypopigmentation and depigmentation. On histology, vitiligo would present as complete loss of melanin pigment in the epidermis and absence of melanocytes, with occasional lymphocytes at the advancing border of the lesions.2

    Management & Treatment
    The primary goals of vitiligo treatment are stabilization of the depigmentation and subsequent re-pigmentation. The choice of therapy depends on the extent, location, and activity of disease. In general, a period of at least 2–3 months is required to determine whether a particular treatment is effective. The areas of the body that typically have the best response to medical therapy are the face, neck, mid extremities and trunk, while the distal extremities and lips are the most resistant to treatment. 4

    For patients who have progressive vitiligo, stabilization is the primary goal and can be achieved with oral prednisone 5-10 mg per day in children and 10-20 mg per day in adults for a maximum of two weeks. If needed, treatment can be repeated in four to six weeks. Those who have stabilized and are seeking re-pigmentation have various treatment options depending on the extent of the disease. 3

    In patients who have less than 10% TBSA (total body surface area), mid or high potency topical steroids are considered first line therapy. Topical calcineurin inhibitors are considered first line alternatives if the depigmentation occurs on areas that are at risk for skin atrophy, such as the face or genitals. Targeted UVB phototherapy (bi-weekly) can be used as adjunct therapy for those who do not respond to topical treatment or would like to be treated more aggressively. For patients with greater than 10% TBSA affected, UVB therapy 2-3x/week is considered first line treatment, with supplemental topical steroid and/or topical calcineurin inhibitors. Assessment of response to therapy should be done after 15-20 NB-UVB treatments. 3,4

    Surgical therapies involving the autologous transplantation of healthy melanocytes in depigmented areas are an option for patients with localized or segmental vitiligo that has been stable for a minimum of 6 months. These procedures primarily are made up of different types of grafts, melanocyte cultures, and epidermal suspensions that transfer a reservoir of healthy melanocytes to affected skin for proliferation and migration into areas of depigmentation.3

    References

    1) Emedicine.medscape.com. Vitiligo: An overview. 2016. Available at: http://emedicine.medscape.com/article/1068962-overview. Accessed September 18, 2016

    2) UpToDate.com. Vitiligo: pathogenesis, clinical features, and diagnosis. 2016. Available at: https://www.uptodate.com/contents/vitiligo-pathogenesis-clinical-features-and-diagnosis?source=search_result&search=Vitiligo&selectedTitle=3~115#H14676582. Accessed September 18, 2016

    3) UpToDate.com. Vitiligo: Management and Prognosis. 2016. Available at: https://www.uptodate.com/contents/vitiligo-management-and-prognosis?source=search_result&search=Vitiligo&selectedTitle=1~115#H4083146597. Accessed September 18, 2016

    4) Bolognia JL, Jorizzo JL, Schaffer JV. Vitiligo. In: Dermatology. 3rd ed. New York, NY: Elsevier Saunders; 2012: 1023-1030.

    Incorrect

    The answer is D: High-potency topical steroids. The patient presents with vitiligo that is localized (<10% TBSA) and located on the back. Clinical trials and current recommendations from the AAD advise that medium to high potency steroids be used as first line treatment for vitiligo that is involving <10% TBSA and not involving the face.

    (A) Topical antifungal is incorrect as the diagnosis is not due to a superficial fungal infection.
    (B) Low-potency steroids would not be effective; topical steroids MOA in vitiligo is via modulation of the immune response and this would not be achieved with low-potency steroids.
    (C) Topical calcineurin inhibitors are an effective treatment for vitiligo. Their indication is normally reserved for cases that involve sensitive areas such as the face, or as combination therapy with topical steroids. Head to head trials of high-potency topical steroids vs. topical calcineurin inhibitors in patients with non-facial vitiligo demonstrate a lower success rate in those who used topical calcineurin inhibitors alone.

    Epidemiology
    Worldwide occurrence of vitiligo is approximately 1-2%; with a rate of 1% in the United States alone. Vitiligo affects males and females equally, with no specific affinity to racial, ethnic, or socio-economic factors. Vitiligo may appear at any time from birth to late adulthood; peak onset occurring at 10-30 years (mean average of 20 years old). Approximately one-third of patients with vitiligo are children, and 70-80 percent of adult patients develop vitiligo prior to age 30 years.1,2

    Etiology
    The etiology of vitiligo is unknown. Patients commonly attribute the onset of their disease to specific triggering events such as physical injury or illness, sunburn, emotional stress, or pregnancy, but there are no data supporting a causative role for these factors. The frequency of comorbid autoimmune diseases is significantly elevated in patients with vitiligo and in their first-degree relatives, suggesting an autoimmune etiology for this disorder. Finally, family clustering of vitiligo suggests a genetic basis for the disease.2

    Pathophysiology
    Vitiligo is a multifactorial disorder related to various factors. It is generally agreed that there is an absence of functional melanocytes in vitiligo skin. Genetic studies indicate a multifactorial, polygenic inheritance pattern; with 25-50 percent of patients with affected relatives. Genetic studies have identified approximately 36 susceptibility loci; with the majority of the susceptibility genes encode either immunoregulatory proteins or melanocyte proteins. Specifically, AIS1 is a major susceptibility locus in Caucasians, while chromosome 4q13-q21 appears to be the main locus in Chinese families.2

    Historically, vitiligo has been associated with several autoimmune diseases, including Hashimoto’s thyroiditis, Graves’ disease, type 1 diabetes mellitus, alopecia areata, pernicious anemia, rheumatoid arthritis, and psoriasis. Multiple cytokines have also been implicated in the destruction of melanocytes in vitiligo. Several studies have documented increased expression of tumor necrosis factor-alpha, interferon-gamma, interleukin (IL)-10, and IL-17. Many hypotheses exist regarding the true pathophysiology behind vitiligo; including, but not limited to: the melanocyte self-destruction hypothesis, oxidative stress, neural hypothesis, and convergence theory.2,4

    Pertinent historical & Physical findings
    Vitiligo most commonly presents as totally amelanotic macules or patches surrounded by normal skin. The lesions characteristically have discrete margins, and can be round, oval, irregular or linear in shape. The borders are usually convex, as if the depigmenting process were “invading” the surrounding normal skin. Progressive lesions enlarge over time, at a slow or rapid rate, with macules and patches range from millimeters to centimeters in diameter and often vary in size within areas of involvement.4

    The current accepted classification system for vitiligo divides the disease into three types: localized, generalized, and universal. Localized vitiligo is described as focal, segmental, or mucosal. Focal is defined as one or more macules in a single area but not in a segmental distribution. Segmental is one or more macule involving a unilateral segment of the body, but stops at midline. Mucosal involves mucous membranes alone. Generalized category can be vulgaris, acrofacial, or mixed. Vulgaris are scattered patches that are widely distributed, while acrofacial affects only distal extremities and the face. Mixed is described as a various combination of segmental, acrofacial and/or vulgaris. Finally, universal vitiligo is complete or nearly complete depigmentation of the skin surface.4

    Other dermatological findings have been strongly correlated with vitiligo. Poliosis, a decrease or absence of color in head hair, eyebrows, and/or eyelashes, is frequently found prior to or simultaneously with the classic amelanotic macules/patches of vitiligo. Halo nevi have been identified as a precursor for vitiligo in up to 25% of patients. Lastly, friction and other types of physical trauma such as scratching, chronic pressure, or cuts along with allergic or irritant contact reactions may trigger vitiligo on areas such as the neck, elbows, and ankles. This is known as the Koebner phenomenon and has been reported in 20-60 percent of vitiligo patients.2,4

    Diagnostic Correlation
    The standard for diagnosis of vitiligo is through the use of a Wood’s lamp. The lamp is a handheld device emitting UVA light at 365 nm wavelength. Under the Wood’s light, the depigmented areas emit a bright blue-white fluorescence and appear sharply demarcated, confirming amelanosis. Although not necessary, a skin biopsy can also be used to distinguish between hypopigmentation and depigmentation. On histology, vitiligo would present as complete loss of melanin pigment in the epidermis and absence of melanocytes, with occasional lymphocytes at the advancing border of the lesions.2

    Management & Treatment
    The primary goals of vitiligo treatment are stabilization of the depigmentation and subsequent re-pigmentation. The choice of therapy depends on the extent, location, and activity of disease. In general, a period of at least 2–3 months is required to determine whether a particular treatment is effective. The areas of the body that typically have the best response to medical therapy are the face, neck, mid extremities and trunk, while the distal extremities and lips are the most resistant to treatment. 4

    For patients who have progressive vitiligo, stabilization is the primary goal and can be achieved with oral prednisone 5-10 mg per day in children and 10-20 mg per day in adults for a maximum of two weeks. If needed, treatment can be repeated in four to six weeks. Those who have stabilized and are seeking re-pigmentation have various treatment options depending on the extent of the disease. 3

    In patients who have less than 10% TBSA (total body surface area), mid or high potency topical steroids are considered first line therapy. Topical calcineurin inhibitors are considered first line alternatives if the depigmentation occurs on areas that are at risk for skin atrophy, such as the face or genitals. Targeted UVB phototherapy (bi-weekly) can be used as adjunct therapy for those who do not respond to topical treatment or would like to be treated more aggressively. For patients with greater than 10% TBSA affected, UVB therapy 2-3x/week is considered first line treatment, with supplemental topical steroid and/or topical calcineurin inhibitors. Assessment of response to therapy should be done after 15-20 NB-UVB treatments. 3,4

    Surgical therapies involving the autologous transplantation of healthy melanocytes in depigmented areas are an option for patients with localized or segmental vitiligo that has been stable for a minimum of 6 months. These procedures primarily are made up of different types of grafts, melanocyte cultures, and epidermal suspensions that transfer a reservoir of healthy melanocytes to affected skin for proliferation and migration into areas of depigmentation.3

    References

    1) Emedicine.medscape.com. Vitiligo: An overview. 2016. Available at: http://emedicine.medscape.com/article/1068962-overview. Accessed September 18, 2016

    2) UpToDate.com. Vitiligo: pathogenesis, clinical features, and diagnosis. 2016. Available at: https://www.uptodate.com/contents/vitiligo-pathogenesis-clinical-features-and-diagnosis?source=search_result&search=Vitiligo&selectedTitle=3~115#H14676582. Accessed September 18, 2016

    3) UpToDate.com. Vitiligo: Management and Prognosis. 2016. Available at: https://www.uptodate.com/contents/vitiligo-management-and-prognosis?source=search_result&search=Vitiligo&selectedTitle=1~115#H4083146597. Accessed September 18, 2016

    4) Bolognia JL, Jorizzo JL, Schaffer JV. Vitiligo. In: Dermatology. 3rd ed. New York, NY: Elsevier Saunders; 2012: 1023-1030.

  137. Question 137 of 220
    137. Question

    A 22-year-old female with a history of asthma and seasonal allergic rhinitis presents to the office with increased wheezing and shortness of breath with normal activity. She states her symptoms require daily use of her Albuterol inhaler. She also reports that she has nighttime coughing fits awakening her from sleep more than once per week. Which of the following represent the most appropriate treatment regimen based on the severity of this patient’s symptoms?

    Correct

    Answer is C. This patient presents with moderate-persistent asthma in which the preferred treatment is with a low-dose inhaled glucocorticoid and a long-acting beta-2 agonist (LABA) in addition to the Albuterol inhaler PRN. The treatment plan in Choice A is for a patient with intermittent asthma. Choice B represents the treatment for a patient with mild-persistent asthma and choice D is for a patient with severe-persistent asthma. Classification of severity is based on the frequency of symptoms, nighttime awakenings, use of a short-acting beta-2 agonist (SABA), interference with normal activity, and lung function tests.

    Asthma Case Discussion:

    Epidemiology:
    Asthma affects millions of patients worldwide. In the U.S. it has been found that asthma is more prevalent in African Americans than Caucasians. Recently, there has been an increase in the prevalence and morbidity of asthma which is likely due to air pollution, smoking, and exposure to environmental allergens. Asthma is also more common in boys during childhood years and is greater in females after puberty. The majority of cases are diagnosed before adulthood and about half of all children with asthma have less severe symptoms by early adulthood although symptoms may develop at any age.

    Etiology & Pathophysiology:
    Asthma is a chronic inflammatory disorder that involves airway hyper-responsiveness as well as increased mucus production which ultimately leads to decreased respiratory airflow. However, the exact cause of asthma is not completely understood as it is a very complex disorder. Environmental exposures, genetic factors, allergens, foods, as well as infectious processes are among the strongest risk factors. There are several prenatal risk factors which include maternal smoking, diet, stress, and even the use of antibiotics. Childhood risk factors include allergic sensitization, environmental tobacco smoke, animal dander, gender, and even socioeconomic status. Allergens plays one of the largest role in asthmatics, particularly in those with a history of atopy (allergies, atopic dermatitis, and allergic rhinitis). In fact, total serum IgE levels, markers for allergen sensitivity, are often associated with a higher incidence of asthma as IgE is closely linked with airway hyperresponsiveness. There is increasing evidence that indoor allergens play a large role in the development of asthma and recurrent wheezing in children. Such allergens include house dust mites, animal proteins, cockroaches, and fungi. In adult onset asthma, the most common etiologies include occupational exposures such as chemicals used by hairdressers, paint used by mechanics, as well as commercial cleaning solutions. Smoking may also contribute to asthma-like symptoms in the adult population.

    History and Physical Exam Findings:
    Patients usually present with a typical history of respiratory symptoms that occurred following exposure to certain triggers that usually revolve after avoiding the stimuli or with asthma medication. The most common triggers include exercise, allergen exposure, or recent viral infection. Patients often report the most common symptoms of asthma which include wheeze, cough (worse at night), and shortness of breath. Typically, patients also have a personal history of atopy which includes atopic dermatitis, seasonal allergic rhinitis as well as asthma.
    On physical exam, patients often have diffuse high-pitched expiratory wheezes on auscultation. In patients with severe airflow obstruction, physical exam findings may include tachypnea, tachycardia, prolonged expiration, use of accessory muscles, and the patient may be in the “tripod” position. Other findings in asthmatic patients include pale or swollen nasal mucus membranes, nasal polyps, or atopic dermatitis.

    Diagnostic Labs/Imaging:
    Laboratory testing and chest imaging are not routinely used in the diagnosis of asthma, but often used to rule out other differential diagnoses. Pulse oximetry may be useful for assessing the severity and response to treatment. Pulmonary function testing (PFT) is the diagnostic test of choice in asthma. PFTs test airflow limitation and are useful in identifying airway hyper-responsiveness. Spirometry testing, in particular, measures the amount your lungs can inhale, exhale, and how quickly you do so. This testing measures forced expiratory volume in one second (FEV1) and forced vital capacity (FVC). When there is a suspected diagnosis of asthma, it is important that providers obtain baseline results to have a comparison for when there is a possible asthma exacerbation or uncontrolled symptoms. Compared to baseline, a reduced FEV1/FVC ratio represents the presence of airway obstruction. Spirometry can be used to determine the amount of obstruction present, assess the response to bronchodilators, characterize the severity of airflow limitation, and identify any restrictive patterns.

    Other testing includes the methacholine/histamine challenge or bronchoprovocation testing. It is most commonly used to assess airway hyper-responsiveness in patients with atypical presentations or normal spirometry results. During the test, methacholine or histamine (provocative substances) are used to cause constriction of the bronchioles. Asthmatics will be more sensitive to these stimuli than patients without asthma. Peak expiratory flow is a simple and inexpensive test that is measured during forceful expiration. It is more useful in monitoring patients with asthma rather than diagnosing asthma. A reduced peak flow value at the time a patient is experiencing respiratory symptoms is suggestive of asthma or an exacerbation. Also, a 20% or greater improvement in peak flow after administration of a SABA is also indicative of asthma.

    Management & Treatment:
    The most important components in the management of asthma are routine monitoring of lung function, education, trigger avoidance, control of comorbid conditions, as well as pharmacologic therapy. Aside from avoiding triggers, the pharmacologic treatment of asthma is multi-faceted and depends on severity of symptoms. The mainstay of pharmacologic management includes the use of short-acting beta-2 agonists, long-acting beta-2 agonists, inhaled corticosteroids, leukotriene receptor antagonists, and omalizumab. Of course, effective management requires prevention of symptoms and exacerbations and when the asthma is not well-controlled, therapy should be “stepped-up” or “stepped-down” when symptoms are well-controlled. Almost all asthmatics should have a prescription for an Albuterol inhaler (SABA) to use as needed for symptoms. However, the use of Albuterol inhalers more than two days per week for symptom relief typically represents inadequate symptom control and the need to step up treatment. Severe asthma exacerbations are often treated with nebulized albuterol, intravenous/oral steroids, and occasionally helium-oxygen.

    Work Cited:

    1. Zhang Y, McConnell R, Gilliland F, Berhane K. Ethnic differences in the effect of asthma on pulmonary function in children. Am J Respir Crit Care Med. 2011 Mar 1. 183(5):596-603. [Medline]
    2.Subbarao P, Mandhane PJ, Sears MR. Asthma: epidemiology, etiology and risk factors. CMAJ. 2009;181(9):E181-90.
    3.Litonjua A et al. Risk Factors For Asthma. UpToDate. 2016. Available at: https://www.uptodate.com/contents/risk-factors-for-asthma?source=search_result&search=asthma+epidemiology&selectedTitle=4%7E150.
    Accessed September 12, 2016.
    4.Fanta M.D. C. Diagnosis of Asthma in Adolescents and Adults. UpToDate. 2016. Available at: https://www.uptodate.com/contents/diagnosis-of-asthma-in-adolescents-and-adults?source=search_result&search=asthma+history+and+physical&selectedTitle=3%7E150. Accessed September 15, 2016.
    5.Fanta C. An Overview of Asthma Management. UpToDate. 2016. Available at: https://www.uptodate.com/contents/an-overview-of-asthma-management?source=search_result&search=asthma+treatment&selectedTitle=1%7E150. Accessed September 15, 2016.

    Incorrect

    Answer is C. This patient presents with moderate-persistent asthma in which the preferred treatment is with a low-dose inhaled glucocorticoid and a long-acting beta-2 agonist (LABA) in addition to the Albuterol inhaler PRN. The treatment plan in Choice A is for a patient with intermittent asthma. Choice B represents the treatment for a patient with mild-persistent asthma and choice D is for a patient with severe-persistent asthma. Classification of severity is based on the frequency of symptoms, nighttime awakenings, use of a short-acting beta-2 agonist (SABA), interference with normal activity, and lung function tests.

    Asthma Case Discussion:

    Epidemiology:
    Asthma affects millions of patients worldwide. In the U.S. it has been found that asthma is more prevalent in African Americans than Caucasians. Recently, there has been an increase in the prevalence and morbidity of asthma which is likely due to air pollution, smoking, and exposure to environmental allergens. Asthma is also more common in boys during childhood years and is greater in females after puberty. The majority of cases are diagnosed before adulthood and about half of all children with asthma have less severe symptoms by early adulthood although symptoms may develop at any age.

    Etiology & Pathophysiology:
    Asthma is a chronic inflammatory disorder that involves airway hyper-responsiveness as well as increased mucus production which ultimately leads to decreased respiratory airflow. However, the exact cause of asthma is not completely understood as it is a very complex disorder. Environmental exposures, genetic factors, allergens, foods, as well as infectious processes are among the strongest risk factors. There are several prenatal risk factors which include maternal smoking, diet, stress, and even the use of antibiotics. Childhood risk factors include allergic sensitization, environmental tobacco smoke, animal dander, gender, and even socioeconomic status. Allergens plays one of the largest role in asthmatics, particularly in those with a history of atopy (allergies, atopic dermatitis, and allergic rhinitis). In fact, total serum IgE levels, markers for allergen sensitivity, are often associated with a higher incidence of asthma as IgE is closely linked with airway hyperresponsiveness. There is increasing evidence that indoor allergens play a large role in the development of asthma and recurrent wheezing in children. Such allergens include house dust mites, animal proteins, cockroaches, and fungi. In adult onset asthma, the most common etiologies include occupational exposures such as chemicals used by hairdressers, paint used by mechanics, as well as commercial cleaning solutions. Smoking may also contribute to asthma-like symptoms in the adult population.

    History and Physical Exam Findings:
    Patients usually present with a typical history of respiratory symptoms that occurred following exposure to certain triggers that usually revolve after avoiding the stimuli or with asthma medication. The most common triggers include exercise, allergen exposure, or recent viral infection. Patients often report the most common symptoms of asthma which include wheeze, cough (worse at night), and shortness of breath. Typically, patients also have a personal history of atopy which includes atopic dermatitis, seasonal allergic rhinitis as well as asthma.
    On physical exam, patients often have diffuse high-pitched expiratory wheezes on auscultation. In patients with severe airflow obstruction, physical exam findings may include tachypnea, tachycardia, prolonged expiration, use of accessory muscles, and the patient may be in the “tripod” position. Other findings in asthmatic patients include pale or swollen nasal mucus membranes, nasal polyps, or atopic dermatitis.

    Diagnostic Labs/Imaging:
    Laboratory testing and chest imaging are not routinely used in the diagnosis of asthma, but often used to rule out other differential diagnoses. Pulse oximetry may be useful for assessing the severity and response to treatment. Pulmonary function testing (PFT) is the diagnostic test of choice in asthma. PFTs test airflow limitation and are useful in identifying airway hyper-responsiveness. Spirometry testing, in particular, measures the amount your lungs can inhale, exhale, and how quickly you do so. This testing measures forced expiratory volume in one second (FEV1) and forced vital capacity (FVC). When there is a suspected diagnosis of asthma, it is important that providers obtain baseline results to have a comparison for when there is a possible asthma exacerbation or uncontrolled symptoms. Compared to baseline, a reduced FEV1/FVC ratio represents the presence of airway obstruction. Spirometry can be used to determine the amount of obstruction present, assess the response to bronchodilators, characterize the severity of airflow limitation, and identify any restrictive patterns.

    Other testing includes the methacholine/histamine challenge or bronchoprovocation testing. It is most commonly used to assess airway hyper-responsiveness in patients with atypical presentations or normal spirometry results. During the test, methacholine or histamine (provocative substances) are used to cause constriction of the bronchioles. Asthmatics will be more sensitive to these stimuli than patients without asthma. Peak expiratory flow is a simple and inexpensive test that is measured during forceful expiration. It is more useful in monitoring patients with asthma rather than diagnosing asthma. A reduced peak flow value at the time a patient is experiencing respiratory symptoms is suggestive of asthma or an exacerbation. Also, a 20% or greater improvement in peak flow after administration of a SABA is also indicative of asthma.

    Management & Treatment:
    The most important components in the management of asthma are routine monitoring of lung function, education, trigger avoidance, control of comorbid conditions, as well as pharmacologic therapy. Aside from avoiding triggers, the pharmacologic treatment of asthma is multi-faceted and depends on severity of symptoms. The mainstay of pharmacologic management includes the use of short-acting beta-2 agonists, long-acting beta-2 agonists, inhaled corticosteroids, leukotriene receptor antagonists, and omalizumab. Of course, effective management requires prevention of symptoms and exacerbations and when the asthma is not well-controlled, therapy should be “stepped-up” or “stepped-down” when symptoms are well-controlled. Almost all asthmatics should have a prescription for an Albuterol inhaler (SABA) to use as needed for symptoms. However, the use of Albuterol inhalers more than two days per week for symptom relief typically represents inadequate symptom control and the need to step up treatment. Severe asthma exacerbations are often treated with nebulized albuterol, intravenous/oral steroids, and occasionally helium-oxygen.

    Work Cited:

    1. Zhang Y, McConnell R, Gilliland F, Berhane K. Ethnic differences in the effect of asthma on pulmonary function in children. Am J Respir Crit Care Med. 2011 Mar 1. 183(5):596-603. [Medline]
    2.Subbarao P, Mandhane PJ, Sears MR. Asthma: epidemiology, etiology and risk factors. CMAJ. 2009;181(9):E181-90.
    3.Litonjua A et al. Risk Factors For Asthma. UpToDate. 2016. Available at: https://www.uptodate.com/contents/risk-factors-for-asthma?source=search_result&search=asthma+epidemiology&selectedTitle=4%7E150.
    Accessed September 12, 2016.
    4.Fanta M.D. C. Diagnosis of Asthma in Adolescents and Adults. UpToDate. 2016. Available at: https://www.uptodate.com/contents/diagnosis-of-asthma-in-adolescents-and-adults?source=search_result&search=asthma+history+and+physical&selectedTitle=3%7E150. Accessed September 15, 2016.
    5.Fanta C. An Overview of Asthma Management. UpToDate. 2016. Available at: https://www.uptodate.com/contents/an-overview-of-asthma-management?source=search_result&search=asthma+treatment&selectedTitle=1%7E150. Accessed September 15, 2016.

  138. Question 138 of 220
    138. Question

    A 64-year-old male patient with a history of Marfan Syndrome presents complaining of four days of mild left-sided chest pain accompanied by shortness of breath, fatigue, and swelling in the legs and feet. He has no prior history of cardiovascular disorders. Physical exam reveals a mid-systolic click heard over the fifth intercostal space at the left midclavicular line and 2+ pitting edema of both lower legs and feet. EKG shows no new changes or abnormalities, and cardiac enzymes are all within normal ranges with the exception of a slightly elevated BNP. Which of the following valvular diseases is most likely to be found on echocardiogram?

    Correct

    Answer is D. Mitral valve prolapse is a disease where the leaflets of the mitral valve project back into the left atrium with systolic pressure. Patients with connective tissue disorders such as Marfan Syndrome are at increased risk for the disease.1 Most patients are asymptomatic, but those with symptoms can have left sided chest pain, sweats, dyspnea, orthopnea, fatigue, and edema.3 The classic murmur for mitral valve prolapse is a midsystolic click over the mitral valve area (fifth intercostal space left midclavicular line).6 Echocardiogram is the definitive diagnostic tool for this condition.1 The remaining choices can be ruled out because tricuspid regurgitation presents with a blowing holosystolic murmur over the left sternal border, mitral stenosis presents with a diastolic low, rumbling decrescendo murmur heard at the apex, pulmonary stenosis presents with a diastolic, crescendo-decrescendo murmur at the left second intercostal space, and aortic regurgitation presents with a decrescendo, high-pitched diastolic murmur over the right parasternal third intercostal space.6

    References
    1. Problem: Mitral Valve Prolapse. http://www.heart.org/HEARTORG/Conditions/More/Heart
    ValveProblemsandDisease/Problem-Mitral-Valve-prolapse_UCM_450441_
    Article.jsp#.WIodtYWcHIU. Published October 16, 2016. Accessed January 26, 2017.
    2.How is Mitral Valve Prolapse Treated? National Institutes of Health.
    https://www.nhlbi.nih.gov/health/health-topics/topics/mvp/treatment. Published June 22,
    2016. Accessed January 26, 2017.
    3. Mitral valve prolapse. Mayo Clinic. http://www.mayoclinic.org/diseases-conditions/mitral-valve-
    prolapse/basics/definition/con-20024748. Published April 5, 2014. Accessed January 26, 2017.
    4. The Evolution of Mitral Valve Prolapse: Insights from the Framingham Heart Study. Circulation American Heart Association. 2016;37(1):327-335. doi:10.1353/crb.0.0100.Epidemiology and
    5. Pathophysiology of Mitral Valve Prolapse. Contemporary Reviews in Cardiovascular Medicine. 2014;37(1):327-335. doi:10.1353/crb.0.0100
    6.Types of Heart Murmurs. The Society for Cardiovascular Angiography and Interventions. http://www.secondscount.org/pediatric-center/pediatric-detail-2/types-of-heart-murmurs#.WI0P5YWcHIU. Published January 12, 2015. Accessed January 28, 2017.

    Incorrect

    Answer is D. Mitral valve prolapse is a disease where the leaflets of the mitral valve project back into the left atrium with systolic pressure. Patients with connective tissue disorders such as Marfan Syndrome are at increased risk for the disease.1 Most patients are asymptomatic, but those with symptoms can have left sided chest pain, sweats, dyspnea, orthopnea, fatigue, and edema.3 The classic murmur for mitral valve prolapse is a midsystolic click over the mitral valve area (fifth intercostal space left midclavicular line).6 Echocardiogram is the definitive diagnostic tool for this condition.1 The remaining choices can be ruled out because tricuspid regurgitation presents with a blowing holosystolic murmur over the left sternal border, mitral stenosis presents with a diastolic low, rumbling decrescendo murmur heard at the apex, pulmonary stenosis presents with a diastolic, crescendo-decrescendo murmur at the left second intercostal space, and aortic regurgitation presents with a decrescendo, high-pitched diastolic murmur over the right parasternal third intercostal space.6

    References
    1. Problem: Mitral Valve Prolapse. http://www.heart.org/HEARTORG/Conditions/More/Heart
    ValveProblemsandDisease/Problem-Mitral-Valve-prolapse_UCM_450441_
    Article.jsp#.WIodtYWcHIU. Published October 16, 2016. Accessed January 26, 2017.
    2.How is Mitral Valve Prolapse Treated? National Institutes of Health.
    https://www.nhlbi.nih.gov/health/health-topics/topics/mvp/treatment. Published June 22,
    2016. Accessed January 26, 2017.
    3. Mitral valve prolapse. Mayo Clinic. http://www.mayoclinic.org/diseases-conditions/mitral-valve-
    prolapse/basics/definition/con-20024748. Published April 5, 2014. Accessed January 26, 2017.
    4. The Evolution of Mitral Valve Prolapse: Insights from the Framingham Heart Study. Circulation American Heart Association. 2016;37(1):327-335. doi:10.1353/crb.0.0100.Epidemiology and
    5. Pathophysiology of Mitral Valve Prolapse. Contemporary Reviews in Cardiovascular Medicine. 2014;37(1):327-335. doi:10.1353/crb.0.0100
    6.Types of Heart Murmurs. The Society for Cardiovascular Angiography and Interventions. http://www.secondscount.org/pediatric-center/pediatric-detail-2/types-of-heart-murmurs#.WI0P5YWcHIU. Published January 12, 2015. Accessed January 28, 2017.

  139. Question 139 of 220
    139. Question

    Which of the following choices is the gold standard screening tool for osteoporosis?

    Correct

    Incorrect

  140. Question 140 of 220
    140. Question

    A DXA should be ordered to screen for osteoporosis in which of the following patients?

    Correct

    Incorrect

  141. Question 141 of 220
    141. Question

    Which choice describes a patient that should be started on a prescription medication to treat osteoporosis?

    Correct

    Incorrect

  142. Question 142 of 220
    142. Question

    What is the current recommendations for daily vitamin D intake in patients over 50 years old

    Correct

    Incorrect

  143. Question 143 of 220
    143. Question

    Which choice describes the mechanism of action of bisphosphonates?

    Correct

    Incorrect

  144. Question 144 of 220
    144. Question

    A potential adverse reaction of bisphosphonates include all of the following except?

    Correct

    Incorrect

  145. Question 145 of 220
    145. Question

    Which choice below offers patients the best reduction in future fracture risk?

    Correct

    Incorrect

  146. Question 146 of 220
    146. Question

    What is the current recommendations for daily calcium intake in patients over 50 years old?

    Correct

    Incorrect

  147. Question 147 of 220
    147. Question

    Which choice below is an indication for a drug holiday?

    Correct

    Incorrect

  148. Question 148 of 220
    148. Question

    A 32 year old women presents to your office with a history of a femoral neck stress fracture. You order bone density testing to screen for low bone density. Which bone density score below would suggest a work-up for metabolic bone disease is necessary?

    Correct

    Incorrect

  149. Question 149 of 220
    149. Question

    A 72 year old male comes to the ED presenting with an episode of chest pain that began 3 hours ago after shoveling snow outside his front yard. He describes the pain as a substernal “tightness” that radiated to his left upper jaw. At the time, he noticed a sudden shortness of breath in addition to the pain. He did not experience nausea/vomiting, diaphoresis, palpitations, pre-syncope, or syncope. En route to the ED, EMT gave him aspirin, NTG, and high flow O2. EKG showed ST depression of 2mm in leads V1-V4. Labs show elevations in CK-MB and troponin levels. His past medical history is significant for hypertension, hyperlipidemia, & obesity. He does not have any prior history for coronary artery disease. He is a current smoker with a 40-pack year history. Family history is significant for hypertension, hyperlipidemia and diabetes. Which criteria/scoring system would best determine the next step for treatment of this patient?

    Correct

    Answer is D. The patient in the clinical vignette above is presenting with an NSTEMI. The TIMI Score (aka. Thrombolysis in Myocardial Infarction Score) is used to estimate the risk of death, MI or need for urgent cardiac catheterization in patients presenting with unstable angina or NSTEMI.4 It will risk stratify patients to low risk, intermediate risk, or high risk based on the number of risk factors a patient is positive for. There are 7 components, with each risk factor worth one point for a maximum score of 7.4 Typically, the higher the score, the more likely one of the above events will occur. The criteria includes:
    Age ≥ 65
    Aspirin use in the past 7 days
    Severe angina (≥2 episodes within 24 hrs)
    Positive cardiac biomarkers
    ≥3 risk factors for coronary artery disease (HTN, hypercholesterolemia, family hx of CAD, DM, smoking)
    Previous history of coronary artery disease (stenosis of ≥ 50% stenosis)
    ST changes ≥0.5mm on EKG

    The scoring system is also used to determine whether a patient should be managed conservatively with drug therapy or with more invasive therapy such as early coronary angiography and revascularization. Low risk patients (score of 0-2) are typically followed with drug therapy. Intermediate patients (score of 3-4) and high risk patients (score of 5-7) are typically followed with a regimen consisting of multiple drugs and early coronary intervention. Our patient above has a TIMI Score of 5 (1 point for his age, 1 point for aspirin use within 7 days, 1 point for positive cardiac biomarkers, 1 point for having ≥ 3 risk factors for CAD, and 1 point for ST changes ≥0.5mm on EKG). Thus, he would be managed with a more invasive regimen, such as a cardiac catheterization. (A)Well’s Criteria is used for risk of DVT/PE. (B)Ranson’s Criteria is used to determine the prognosis and severity of patients with pancreatitis. (C)CHADS VASc Score is utilized for the risk of stroke in patients with atrial fibrillation. (E)CURB 65 is used to determine the risk of death in patients with community acquired pneumonia. The above patient does not present with a chief complaint that would require the utilization of these other scoring systems.

    References:
    1. Abbott JD, Oettgen P, eds. Acute Coronary Syndromes. DynaMed. http://web.a.ebscohost.com.ezproxymcp.flo.org/dynamed/detail?sid=7b9cfc99-5e55-46f9-a275-4bbe5f0afe0e%40sessionmgr4006&vid=0&hid=4104&bdata=JnNpdGU9ZHluYW1lZC1saXZlJnNjb3BlPXNpdGU%3d#AN=116779&db=dme. Published October 27, 2016. Accessed February 4, 2017.

    2. NSTEMI vs. STEMI. NSTEMI.org. http://nstemi.org/nstemi-vs-stemi/. Published January 12, 2017. Accessed February 4, 2017.

    3. Papadakis MA, McPhee SJ, Rabow MW. Current Medical Diagnosis & Treatment 2015. 54th ed. McGraw-Hill Education; 2014.

    4. TIMI Risk Score for UA/NSTEMI Mnemonic. Internalize Medicine. http://www.internalizemedicine.com/2012/08/timi-risk-score-for-uanstemi-mnemonic.html. Published August 11, 2012. Accessed February 4, 2017.

    5. Wilson PW, Douglas PS. Epidemiology of Coronary Heart Disease. UpToDate. http://www.uptodate.com/contents/epidemiology-of-coronary-heart-disease. Published January 23, 2015. Accessed February 4, 2017.

    Incorrect

    Answer is D. The patient in the clinical vignette above is presenting with an NSTEMI. The TIMI Score (aka. Thrombolysis in Myocardial Infarction Score) is used to estimate the risk of death, MI or need for urgent cardiac catheterization in patients presenting with unstable angina or NSTEMI.4 It will risk stratify patients to low risk, intermediate risk, or high risk based on the number of risk factors a patient is positive for. There are 7 components, with each risk factor worth one point for a maximum score of 7.4 Typically, the higher the score, the more likely one of the above events will occur. The criteria includes:
    Age ≥ 65
    Aspirin use in the past 7 days
    Severe angina (≥2 episodes within 24 hrs)
    Positive cardiac biomarkers
    ≥3 risk factors for coronary artery disease (HTN, hypercholesterolemia, family hx of CAD, DM, smoking)
    Previous history of coronary artery disease (stenosis of ≥ 50% stenosis)
    ST changes ≥0.5mm on EKG

    The scoring system is also used to determine whether a patient should be managed conservatively with drug therapy or with more invasive therapy such as early coronary angiography and revascularization. Low risk patients (score of 0-2) are typically followed with drug therapy. Intermediate patients (score of 3-4) and high risk patients (score of 5-7) are typically followed with a regimen consisting of multiple drugs and early coronary intervention. Our patient above has a TIMI Score of 5 (1 point for his age, 1 point for aspirin use within 7 days, 1 point for positive cardiac biomarkers, 1 point for having ≥ 3 risk factors for CAD, and 1 point for ST changes ≥0.5mm on EKG). Thus, he would be managed with a more invasive regimen, such as a cardiac catheterization. (A)Well’s Criteria is used for risk of DVT/PE. (B)Ranson’s Criteria is used to determine the prognosis and severity of patients with pancreatitis. (C)CHADS VASc Score is utilized for the risk of stroke in patients with atrial fibrillation. (E)CURB 65 is used to determine the risk of death in patients with community acquired pneumonia. The above patient does not present with a chief complaint that would require the utilization of these other scoring systems.

    References:
    1. Abbott JD, Oettgen P, eds. Acute Coronary Syndromes. DynaMed. http://web.a.ebscohost.com.ezproxymcp.flo.org/dynamed/detail?sid=7b9cfc99-5e55-46f9-a275-4bbe5f0afe0e%40sessionmgr4006&vid=0&hid=4104&bdata=JnNpdGU9ZHluYW1lZC1saXZlJnNjb3BlPXNpdGU%3d#AN=116779&db=dme. Published October 27, 2016. Accessed February 4, 2017.

    2. NSTEMI vs. STEMI. NSTEMI.org. http://nstemi.org/nstemi-vs-stemi/. Published January 12, 2017. Accessed February 4, 2017.

    3. Papadakis MA, McPhee SJ, Rabow MW. Current Medical Diagnosis & Treatment 2015. 54th ed. McGraw-Hill Education; 2014.

    4. TIMI Risk Score for UA/NSTEMI Mnemonic. Internalize Medicine. http://www.internalizemedicine.com/2012/08/timi-risk-score-for-uanstemi-mnemonic.html. Published August 11, 2012. Accessed February 4, 2017.

    5. Wilson PW, Douglas PS. Epidemiology of Coronary Heart Disease. UpToDate. http://www.uptodate.com/contents/epidemiology-of-coronary-heart-disease. Published January 23, 2015. Accessed February 4, 2017.

  150. Question 150 of 220
    150. Question

    A 60 year old female with a PMH significant for asymptomatic dilated cardiomyopathy presents to the ED with a chief complaint of new-onset dyspnea at rest and with exertion, LE edema, and fatigue, but denies heart palpitations and syncope. Physical examination reveals a grade 3/6 blowing holosystolic murmur at the apex, radiating to the left axilla, with no associated click. CXR reveals interstitial and alveolar edema with bilateral pleural effusions. EKG reveals an irregularly irregular rhythm at a rate of 95bpm. What is the most likely cause of this patient’s presentation?

    Correct

    Answer: (C) Mitral regurgitation is a common valvular disorder that results in the backflow of blood from the left ventricle to the left atrium due to incomplete closure of the mitral valve. It results from an abnormality of the mitral valve apparatus, ischemia, or enlargement of the LV with systolic dysfunction. A common cause of mitral regurgitation is dilated cardiomyopathy, which is classified by enlargement of the LV, LA, and sometimes the RA and RV if it becomes severe enough. The patient in this example has a PMH of dilated cardiomyopathy, though she has been asymptomatic in the past. Patients can be asymptomatic, or they can exhibit signs of HF (e.g. LE edema, DOE, fatigue, etc.) as it becomes more chronic. A blowing holosystolic murmur at the apex radiating to the left axilla is classic for MR. The EKG reveals atrial fibrillation, which is a common complication due to years of left atrial enlargement.

    (A) Mitral valve prolapse is a valve disorder most common in women, and in patients with connective tissue disorders (e.g. Marfan’s syndrome/Ehlers-Danlos syndrome) or hypertrophic cardiomyopathy, not dilated cardiomyopathy. It results from mitral valve leaflets entering the left atrium during systole. It often presents with heart palpitations or syncope and is associated with a mid-systolic click heard at the apex, without radiation. It can progress to symptomatic mitral regurgitation, with pulmonary edema and signs of heart failure. (B) Tricuspid regurgitation results from incomplete closure of the tricuspid valve and consequently, backflow of blood from the RV to the RA. It can be a severe complication of chronic MR. It commonly results from left-sided HF that progresses to RV dilation and right-sided HF. However, the classic murmur of tricuspid regurgitation is a holosystolic murmur at the lower left sternal border (not the apex) without radiation. Although (D) CHF is the current clinical picture, the most likely cause of the patient’s clinical presentation is mitral regurgitation. The clinical picture is of decompensated MR, which manifests as heart failure. Even though the CXR reveals interstitial and alveolar edema with bilateral pleural effusions which is indicative of HF, the physical exam reveals the murmur characteristic of MR as well as atrial fibrillation on the EKG. Therefore, although the patient is presenting with CHF, the most likely cause of the CHF is MR.

    Reference: Otto CM. Clinical manifestations and diagnosis of chronic mitral regurgitation. UpToDate. http://www.uptodate.com/contents/clinical-manifestations-and-diagnosis-of-chronic-mitral-regurgitation. Updated January 29, 2016. Accessed January 31, 2017.

    Incorrect

    Answer: (C) Mitral regurgitation is a common valvular disorder that results in the backflow of blood from the left ventricle to the left atrium due to incomplete closure of the mitral valve. It results from an abnormality of the mitral valve apparatus, ischemia, or enlargement of the LV with systolic dysfunction. A common cause of mitral regurgitation is dilated cardiomyopathy, which is classified by enlargement of the LV, LA, and sometimes the RA and RV if it becomes severe enough. The patient in this example has a PMH of dilated cardiomyopathy, though she has been asymptomatic in the past. Patients can be asymptomatic, or they can exhibit signs of HF (e.g. LE edema, DOE, fatigue, etc.) as it becomes more chronic. A blowing holosystolic murmur at the apex radiating to the left axilla is classic for MR. The EKG reveals atrial fibrillation, which is a common complication due to years of left atrial enlargement.

    (A) Mitral valve prolapse is a valve disorder most common in women, and in patients with connective tissue disorders (e.g. Marfan’s syndrome/Ehlers-Danlos syndrome) or hypertrophic cardiomyopathy, not dilated cardiomyopathy. It results from mitral valve leaflets entering the left atrium during systole. It often presents with heart palpitations or syncope and is associated with a mid-systolic click heard at the apex, without radiation. It can progress to symptomatic mitral regurgitation, with pulmonary edema and signs of heart failure. (B) Tricuspid regurgitation results from incomplete closure of the tricuspid valve and consequently, backflow of blood from the RV to the RA. It can be a severe complication of chronic MR. It commonly results from left-sided HF that progresses to RV dilation and right-sided HF. However, the classic murmur of tricuspid regurgitation is a holosystolic murmur at the lower left sternal border (not the apex) without radiation. Although (D) CHF is the current clinical picture, the most likely cause of the patient’s clinical presentation is mitral regurgitation. The clinical picture is of decompensated MR, which manifests as heart failure. Even though the CXR reveals interstitial and alveolar edema with bilateral pleural effusions which is indicative of HF, the physical exam reveals the murmur characteristic of MR as well as atrial fibrillation on the EKG. Therefore, although the patient is presenting with CHF, the most likely cause of the CHF is MR.

    Reference: Otto CM. Clinical manifestations and diagnosis of chronic mitral regurgitation. UpToDate. http://www.uptodate.com/contents/clinical-manifestations-and-diagnosis-of-chronic-mitral-regurgitation. Updated January 29, 2016. Accessed January 31, 2017.

  151. Question 151 of 220
    151. Question

    A 78 year old patient presents to the ED with a chief complaint of shortness of breath. Vitals signs show a BP 130/60 mmHg, HR 134, Temp 98.8 F. Physical examination reveals an elevated jugular venous pressure (JVP) with S3 gallop heard upon auscultation, and 2+ edema in the legs. EKG shows a tachycardic normal sinus rhythm with no abnormal morphological changes. Echocardiogram reveals an ejection fraction of 20%. You diagnose the patient with heart failure. Which of the following medications is likely to be part of this patient’s medication regimen to have the best efficacy in treating his HF?

    Correct

    Correct: The correct answer is D: Coreg (carvedilol), Prinivil (Lisinopril). Only 3 beta blockers have been approved by the FDA for the treatment of heart failure patients which have shown to reduce mortality. They are carvedilol, metoprolol, and bisoprolol. Guidelines say the highest level of evidence to reduce mortality in a patient with reduced ejection fraction is to start them on a recommended beta blocker plus an ACE-I such as Lisinopril2. This is why answer choice (D) is correct.

    Incorrect: The answer choice (A) Veletri (epoprostenol), Aldactone (spironolactone) is incorrect because Epoprostenol is a drug that is used to treat pulmonary arterial hypertension. Epoprostenol is contraindicated in patient with left ventricular heart failure. The answer choice (B) Inderal XL (propranolol), Epaned (enalapril) is incorrect because though an ACE-I is the proper treatment for HF, Propanol is not one of the 3 beta blockers that is used in treatment of HF2. The answer choice (C) Nitrostat (nitroglycerin) is generally used for angina and Eliquis (apixaban) is incorrect because it is used for irregular heart rhythms such as atrial fibrillation to prevent blood clots.

    References
    1. Su V, Chang Y, Hu Y et al. Carvedilol, Bisoprolol, and Metoprolol Use in Patients With Coexistent Heart Failure and Chronic Obstructive Pulmonary Disease. Medicine. 2016;95(5):e2427. doi:10.1097/md.0000000000002427.
    2. Gheorghiade M. beta-Blockers in Chronic Heart Failure. Circulation. 2003;107(12):1570-1575. doi:10.1161/01.cir.0000065187.80707.18.
    3. Mozaffarian D, Benjamin E, Go A et al. Heart Disease and Stroke Statistics—2016 Update. Circulation. 2015;133(4):e38-e360. doi:10.1161/cir.0000000000000350.
    4. Kannel WBelanger A. Epidemiology of heart failure. American Heart Journal. 1991;121(3):951-957. doi:10.1016/0002-8703(91)90225-7.

    Incorrect

    Correct: The correct answer is D: Coreg (carvedilol), Prinivil (Lisinopril). Only 3 beta blockers have been approved by the FDA for the treatment of heart failure patients which have shown to reduce mortality. They are carvedilol, metoprolol, and bisoprolol. Guidelines say the highest level of evidence to reduce mortality in a patient with reduced ejection fraction is to start them on a recommended beta blocker plus an ACE-I such as Lisinopril2. This is why answer choice (D) is correct.

    Incorrect: The answer choice (A) Veletri (epoprostenol), Aldactone (spironolactone) is incorrect because Epoprostenol is a drug that is used to treat pulmonary arterial hypertension. Epoprostenol is contraindicated in patient with left ventricular heart failure. The answer choice (B) Inderal XL (propranolol), Epaned (enalapril) is incorrect because though an ACE-I is the proper treatment for HF, Propanol is not one of the 3 beta blockers that is used in treatment of HF2. The answer choice (C) Nitrostat (nitroglycerin) is generally used for angina and Eliquis (apixaban) is incorrect because it is used for irregular heart rhythms such as atrial fibrillation to prevent blood clots.

    References
    1. Su V, Chang Y, Hu Y et al. Carvedilol, Bisoprolol, and Metoprolol Use in Patients With Coexistent Heart Failure and Chronic Obstructive Pulmonary Disease. Medicine. 2016;95(5):e2427. doi:10.1097/md.0000000000002427.
    2. Gheorghiade M. beta-Blockers in Chronic Heart Failure. Circulation. 2003;107(12):1570-1575. doi:10.1161/01.cir.0000065187.80707.18.
    3. Mozaffarian D, Benjamin E, Go A et al. Heart Disease and Stroke Statistics—2016 Update. Circulation. 2015;133(4):e38-e360. doi:10.1161/cir.0000000000000350.
    4. Kannel WBelanger A. Epidemiology of heart failure. American Heart Journal. 1991;121(3):951-957. doi:10.1016/0002-8703(91)90225-7.

  152. Question 152 of 220
    152. Question

    A 68-year-old female presents with difficulty swallowing that has gotten progressively worse over the last couple of months. She feels like food gets stuck in her throat when eating. She also gets frequent heartburn. The patient notices the most difficulty when she eats meats. She does not struggle to swallow liquids. The patient has a history of COPD for which she is on a steroid inhaler. She has no pertinent physical exam findings. Which diagnosis is the most likely cause of the patient’s dysphagia?

    Correct

    The correct answer is C. Schatzki/esophageal ring. Esophageal ring is the correct diagnosis based on the patient’s presentation and lack of physical exam findings. The patient has intermittent dysphagia to solid foods only and exhibits “steakhouse syndrome” as she has the most dysphagia when eating steak and bread with others.4 The patient has uncontrolled heartburn and reflux, which has been linked to the development of Schatzki rings. An esophageal motility disorder would be incorrect in this scenario because the patient does not have dysphagia to liquids. With motility disorders, the patient will have difficulty swallowing both liquid and solid foods. Candidiasis is possible in individuals who take inhaled steroids and can cause dysphagia and chest discomfort. Despite this, Candida esophagitis is not correct in this scenario. The patient had no evidence of candidiasis on physical exam. There were no fluffy white patches in her mouth or oropharynx.4 Lastly, GERD is not the correct answer. The patient exhibits symptoms of GERD, but it is unlikely that it is the primary reason for the patient’s dysphagia. GERD has been linked to the development of esophageal strictures, but in this case it would not be the most correct diagnosis.

    References
    1.Kafrouni M. Schatzki’s Ring. Memorial Hermann. http://www.memorialhermann.org/digestive/schatzkis-ring/. Published July 8, 2015. Accessed February 6, 2017.
    2.Liu JJ, Kahrilas PJ. Pharyngeal and esophageal diverticula, rings, and webs. GI Motility Online. May 2006. doi:10.1038/gimo41.
    3.Lundell L. Reflux esophagitis and peptic strictures. GI Motility Online. May 2006. doi:10.1038/gimo43.
    4.Schatzki Ring. Schatzki Ring: Background, Pathophysiology, Epidemiology. http://emedicine.medscape.com/article/182647-overview. Accessed February 1, 2017.
    5.Smith MS. Diagnosis and Management of Esophageal Rings and Webs. Gastroenterology & Hepatology. https://www.ncbi.nlm.nih.gov/pmc/articles/PMC3033540/. Published November 2010. Accessed February 5, 2017.

    Incorrect

    The correct answer is C. Schatzki/esophageal ring. Esophageal ring is the correct diagnosis based on the patient’s presentation and lack of physical exam findings. The patient has intermittent dysphagia to solid foods only and exhibits “steakhouse syndrome” as she has the most dysphagia when eating steak and bread with others.4 The patient has uncontrolled heartburn and reflux, which has been linked to the development of Schatzki rings. An esophageal motility disorder would be incorrect in this scenario because the patient does not have dysphagia to liquids. With motility disorders, the patient will have difficulty swallowing both liquid and solid foods. Candidiasis is possible in individuals who take inhaled steroids and can cause dysphagia and chest discomfort. Despite this, Candida esophagitis is not correct in this scenario. The patient had no evidence of candidiasis on physical exam. There were no fluffy white patches in her mouth or oropharynx.4 Lastly, GERD is not the correct answer. The patient exhibits symptoms of GERD, but it is unlikely that it is the primary reason for the patient’s dysphagia. GERD has been linked to the development of esophageal strictures, but in this case it would not be the most correct diagnosis.

    References
    1.Kafrouni M. Schatzki’s Ring. Memorial Hermann. http://www.memorialhermann.org/digestive/schatzkis-ring/. Published July 8, 2015. Accessed February 6, 2017.
    2.Liu JJ, Kahrilas PJ. Pharyngeal and esophageal diverticula, rings, and webs. GI Motility Online. May 2006. doi:10.1038/gimo41.
    3.Lundell L. Reflux esophagitis and peptic strictures. GI Motility Online. May 2006. doi:10.1038/gimo43.
    4.Schatzki Ring. Schatzki Ring: Background, Pathophysiology, Epidemiology. http://emedicine.medscape.com/article/182647-overview. Accessed February 1, 2017.
    5.Smith MS. Diagnosis and Management of Esophageal Rings and Webs. Gastroenterology & Hepatology. https://www.ncbi.nlm.nih.gov/pmc/articles/PMC3033540/. Published November 2010. Accessed February 5, 2017.

  153. Question 153 of 220
    153. Question

    A 74-year-old Hispanic female patient comes in complaining of a “rash” on her right cheek, which erupted a year ago and has gotten worse since. Physical exam shows a gray-black hyperpigmented macular lesion. The lesion is not pruritic, erythematous, or scaly. The patient has been using hydroquinone 2% cream daily on the lesion for the past year with no sign of improvement. Her medications include allopurinol, lisinopril and atorvastatin, all of which she has been taking for four years. Which of the following is the most likely diagnosis?

    Correct

    The answer is B. Exogenous ochronosis is a hyperpigmentation condition caused my chronic use of a variety of topical products. One of the most common offending agents is hydroquinone 2% cream, which the patient has used for about a year. The lesion is not symptomatic and presents as a worsening gray-black macule. A drug eruption can be a possibility due to the patients medication list. However, the presentation is not consistent with that of a morbiliform drug eruption. The onset of the lesion cannot be linked to the start of any of her medication and no blisters, erythema, angioedema, pruritus, necrosis or purpura is present. Melasma is a hypermelanosis of widely sun-exposed skin. The condition presents as hyper-pigmented macules, which are tan-brown rather than gray-black. Melasma also tends to be symmetrical and has a correlation with estrogen activity such as pregnancy and oral contraceptive pills. Because the patient has not had any recent hormonal changes, it is unlikely that she has Melasma. Post inflammatory hyperpigmentation (PIH) is a temporary pigmentation disorder that is caused by prior trauma, inflammatory skin conditions such as dermatitis, infections or certain medications. Melanocytes are increased due to inflammation of the epidermis and are transferred to the surrounding keratinocytes, resulting in hyperpigmentation (Ngan, 2015). The lesions are tan-brown or black in color and can become more prominent with sun exposure. It is unlikely that the patient has PIH due to the fact that she does not have a history of any inflammatory skin conditions in the area of the lesions, or is not taking a medication that is known to cause PIH.

    References
    Burkhart, C. G. (2015, August 19). Ochronosis Treatment & Management. Retrieved February 05, 2017, from http://emedicine.medscape.com/article/1104184-treatment#d6

    Dyall-Smith, D. (2016, January). DermNet New Zealand. Retrieved February 05, 2017, from https://www.dermnetnz.org/topics/morbilliform-drug-reaction

    Gandhi, V., Verma, P., & Naik, G. (2012). Exogenous ochronosis After Prolonged Use of Topical Hydroquinone (2%) in a 50-Year-Old Indian Female. Indian Journal of Dermatology, 57(5), 394–395. http://doi.org/10.4103/0019-5154.100498

    Incorrect

    The answer is B. Exogenous ochronosis is a hyperpigmentation condition caused my chronic use of a variety of topical products. One of the most common offending agents is hydroquinone 2% cream, which the patient has used for about a year. The lesion is not symptomatic and presents as a worsening gray-black macule. A drug eruption can be a possibility due to the patients medication list. However, the presentation is not consistent with that of a morbiliform drug eruption. The onset of the lesion cannot be linked to the start of any of her medication and no blisters, erythema, angioedema, pruritus, necrosis or purpura is present. Melasma is a hypermelanosis of widely sun-exposed skin. The condition presents as hyper-pigmented macules, which are tan-brown rather than gray-black. Melasma also tends to be symmetrical and has a correlation with estrogen activity such as pregnancy and oral contraceptive pills. Because the patient has not had any recent hormonal changes, it is unlikely that she has Melasma. Post inflammatory hyperpigmentation (PIH) is a temporary pigmentation disorder that is caused by prior trauma, inflammatory skin conditions such as dermatitis, infections or certain medications. Melanocytes are increased due to inflammation of the epidermis and are transferred to the surrounding keratinocytes, resulting in hyperpigmentation (Ngan, 2015). The lesions are tan-brown or black in color and can become more prominent with sun exposure. It is unlikely that the patient has PIH due to the fact that she does not have a history of any inflammatory skin conditions in the area of the lesions, or is not taking a medication that is known to cause PIH.

    References
    Burkhart, C. G. (2015, August 19). Ochronosis Treatment & Management. Retrieved February 05, 2017, from http://emedicine.medscape.com/article/1104184-treatment#d6

    Dyall-Smith, D. (2016, January). DermNet New Zealand. Retrieved February 05, 2017, from https://www.dermnetnz.org/topics/morbilliform-drug-reaction

    Gandhi, V., Verma, P., & Naik, G. (2012). Exogenous ochronosis After Prolonged Use of Topical Hydroquinone (2%) in a 50-Year-Old Indian Female. Indian Journal of Dermatology, 57(5), 394–395. http://doi.org/10.4103/0019-5154.100498

  154. Question 154 of 220
    154. Question

    An 85 year-old male presents with a 4 month history of severe right upper extremity pain. He denies prior injury to the neck or shoulder. The pain is worse over the right scapula, posterior arm, and seems to radiate to the ulnar side of the forearm. Conservative treatment with ice and NSAIDs has not helped. He has also been feeling unstable with gait and his balance seems off. AP and lateral x-rays of his cervical spine show multilevel degenerative changes. There is significant degenerative disc disease with disc height loss and anterior and posterior osteophyte formation at C5-6 and C6-7. MRI shoes significant bilateral foraminal stenosis, right worse than left and most pronounced at C5-6 and C6-7. Which physical exam finding would most likely indicate the patient has cervical myelopathy?

    Correct

    Cervical myelopathy is a compressive injury to the spinal cord that most commonly occurs due to degenerative spondylosis in patients over 50 years of age. With aging the cervical discs desiccate and lose their height, the ligamentum flavum hypertrophies, osteophytes form, and the posterior longitudinal ligament may ossify. This degenerative cascade leads to compression of the cord. Symptoms can vary considerably and usually don’t follow a dermatomal pattern like cervical radiculopathy. Subtle changes in gait and balance are often the first signs of early myelopathy. Complaints of neck pain, loss of fine motor control of the upper extremity or “clumsy hands”, and paresthesia’s are common. Patients will often complain of increasing difficulty with hand writing over a few weeks to months. The proximal motor groups of the legs are more commonly affected than the distal motor groups in cervical myelopathy (which is opposite to the findings of lumbar stenosis). Changes is bowel and bladder function are rarely found with cervical myelopathy.

    Several physical exam tests of the hands exist to screen for cervical myelopathy. For the finger escape sign, the patient is asked to extend and adduct the fingers. If the small and ring fingers flex and abduct after 30-60 seconds then cervical myelopathy is considered. Slow or clumsy hands may be evident by asking the patient to continuously make a fist and release 20 times in 10 seconds. Other upper motor neuron signs often present in patients with moderate to severe cervical myelopathy include hyperreflexia, a positive Hoffman’s sign, sustained clonus, and a positive Babanski’s. A Hoffman’s sign involves snapping the distal tip of the patient’s extended middle finger which leads to spontaneous flexion of the other fingers in patients with cervical myelopathy.

    References
    1. Sanford EM. Cervical Spondylotic Myelopathy: Diagnosis and Treatment. JAAOS 2001; 9: 376-388.
    2. Rao R. Neck pain, Cervical Radiculopathy, and Cervical Myelopathy: Pathophysiology, Natural History, and Clinical Evaluation. AAOS Instruction Course Lecture. JBJS 2012. 84. 1871-1881.

    Incorrect

    Cervical myelopathy is a compressive injury to the spinal cord that most commonly occurs due to degenerative spondylosis in patients over 50 years of age. With aging the cervical discs desiccate and lose their height, the ligamentum flavum hypertrophies, osteophytes form, and the posterior longitudinal ligament may ossify. This degenerative cascade leads to compression of the cord. Symptoms can vary considerably and usually don’t follow a dermatomal pattern like cervical radiculopathy. Subtle changes in gait and balance are often the first signs of early myelopathy. Complaints of neck pain, loss of fine motor control of the upper extremity or “clumsy hands”, and paresthesia’s are common. Patients will often complain of increasing difficulty with hand writing over a few weeks to months. The proximal motor groups of the legs are more commonly affected than the distal motor groups in cervical myelopathy (which is opposite to the findings of lumbar stenosis). Changes is bowel and bladder function are rarely found with cervical myelopathy.

    Several physical exam tests of the hands exist to screen for cervical myelopathy. For the finger escape sign, the patient is asked to extend and adduct the fingers. If the small and ring fingers flex and abduct after 30-60 seconds then cervical myelopathy is considered. Slow or clumsy hands may be evident by asking the patient to continuously make a fist and release 20 times in 10 seconds. Other upper motor neuron signs often present in patients with moderate to severe cervical myelopathy include hyperreflexia, a positive Hoffman’s sign, sustained clonus, and a positive Babanski’s. A Hoffman’s sign involves snapping the distal tip of the patient’s extended middle finger which leads to spontaneous flexion of the other fingers in patients with cervical myelopathy.

    References
    1. Sanford EM. Cervical Spondylotic Myelopathy: Diagnosis and Treatment. JAAOS 2001; 9: 376-388.
    2. Rao R. Neck pain, Cervical Radiculopathy, and Cervical Myelopathy: Pathophysiology, Natural History, and Clinical Evaluation. AAOS Instruction Course Lecture. JBJS 2012. 84. 1871-1881.

  155. Question 155 of 220
    155. Question

    A 32-year-old European female with a history of 1 prior DVT presents to your hematology practice in the first trimester of her second pregnancy.  She recently underwent genetic testing after her sister was diagnosed with the most common inherited cause of hypercoagulability. The patients test results showed she contained 2 copies of the F5 mutation. She is not obese, denies recent travel, does not smoke, and is not planning on undergoing any surgery soon. She denies history of oral contraceptive use or hormone replacement therapy. Given the suspected diagnosis, what is the first line treatment for this patient?

    Correct

    Factor V Leiden Thrombophilia poses an increased risk of abnormal clot formation to diagnosed patients, especially younger individuals and of European ancestry. An inherited mutation of the F5 gene is present. This mutation prevents argon plasma coagulation (APC) in the coagulation pathway from inactivating Factor V which allows more time for clot formation and bigger, abnormal clots to occur. Risk of clot formation increases with 1 copy of the mutation but is even greater with 2 copies present.

    A major risk factor for this condition and predisposes one to genetic diagnostic testing is having a close family member diagnosed with this disorder. Other risk factors include pregnancy, obesity, smoking, recent travel (especially on airplane), oral contraceptive pills, and hormone replacement therapy. Symptoms for this disorder include recurrent PE or DVT, occurrence of either at a young age, loss of pregnancy in the second or third trimester that is unexplained, or venous thrombosis in an unusual organ.

    Choice B is the correct answer to the question given. Factor V Leiden Thrombophilia is not a benign condition as it increases the risk of abnormal clot formation. Given the patients’ previous DVT diagnosis and current pregnancy, clinical observation and risk factor counseling would not be appropriate as sole therapy. The patient should receive treatment with anticoagulant for the duration of the pregnancy to decrease risk of abnormal clot formation. She should not be given just prophylaxis as she did have a previous DVT and is currently pregnant which is a risk factor.

    Discussion: Factor V Leiden Thrombophilia
    Etiology: Factor V Leiden Thrombophilia is an inherited condition in which a gene mutation is present that leads to a state of increased risk of clot formation. It is the most common inherited disorder that causes a hypercoagulable state. For this reason, the initial MD in the case described above took extra precautions in terms of directing the workup the patient received.
    Epidemiology: This mutation is more common in individuals of European ancestry. Individuals with 2 copies of the F5 mutation have an increased chance of up to 80 in 1,000 people developing abnormal clot formation3. Individuals with 1 copy of the F5 mutation have a 4-8 in 1,000 increased chance of abnormal clot formation. Normal risk for elevated clot formation is 1 in 1,000 individuals per year.

    Pathophysiology: Factor V protein is produced primarily in the liver from instructions provided by the F5 gene. Until the coagulation pathway is stimulated by a blood vessel injury, factor V protein rotates in the bloodstream in its inactive form. Factor V plays an important role in the coagulation pathway. It is activated by factor X2. Once triggered, Factor V works as the activator of prothrombin which converts to thrombin when stimulated. Thrombin then activates fibrinogen which becomes the clot forming material known as fibrin2. Another role of factor V in the coagulation system is being inactivated by activated protein C (APC) which stops clots from becoming too big. In the condition of Factor V Leiden Thrombophilia, a genetic mutation of the F5 gene is present and causes the hypercoagulability as Factor V can no longer be inactivated by APC3. Thus, clots more time to form and can become bigger leading to abnormalities. Individuals with 2 copies of the inherited mutation (one from father and one from mother) have the greatest risk of increased clot formation.

    History and physical exam: Individuals with a family member who has Factor V Leiden Thrombophilia should consider being tested as this is a risk factor for the disorder4. In the case presentation above, the patient was diagnosed after she was urged to undergo test by her primary care physician once her sister was diagnosed with the condition. Components in a patient history with Factor V Leiden Thrombophilia that will increase out of the ordinary clot formation are surgery, pregnancy, smoking, obesity, oral contraceptive pills, obesity, hormone replacement therapy, containing another mutation in the coagulation pathway and increasing age3. Symptoms include a DVT or PE that keep occurring, DVT before age 50 or age a young age, pregnancy loss in the second or third trimester that cannot be explained, or have a venous thrombosis take place in an out of the ordinary site4. Physical exam findings that may indicate a PE include dyspnea, tachypnea, and pain on inspiration5. Physical exam findings indicating a DVT include a warm to touch, unilateral extremity or area, swelling, and pain5. Also, the Wells criteria for predication of PE is very useful in the clinical setting5. However, in both a DVT and PE, there may be no presenting symptoms.

    Diagnostic: Genetic testing where the individual’s DNA is analyzed for the F5 mutation or APC resistance and/or a coagulation screening test is used for diagnosis. Testing is strongly recommended if there is a family history of the disorder.

    Management and treatment: Treatment and management is performed on a case by case basis as it depends on the individuals history and risks. Patients who are at low risk and have never had a PE or DVT may be treated with clinical observation as well as counseling on eliminating other risk factors and presenting symptoms to monitor oneself for4. If an individual only had one PE or DVT, lifelong anticoagulation is typically not necessary unless that person has other risks present4. Anticoagulation may be given as prophylaxis in these individuals in procedures that are high risk such as major surgery. At UMASS, all surgical patients are placed on some type of DVT prophylaxis prior to surgery as witnessed during the general surgery rotation. Individuals who are high risk, such as those with recurrent PE or DVT, should be placed on indefinite anticoagulation using agents such as Heparin4.

    References
    1. Reference G. F5 gene. Genetics Home Reference. 2017. Available at: https://ghr.nlm.nih.gov/gene/F5. Accessed March 16, 2017.
    2. Berkowitz A. Clinical Pathophysiology Made Ridiculously Simple. 1st ed. Miami Florida: MedMaster Inc.; 2007:129-132.
    3. Reference G. factor V Leiden thrombophilia. Genetics Home Reference. 2017. Available at: https://ghr.nlm.nih.gov/condition/factor-v-leiden-thrombophilia#synonyms. Accessed March 16, 2017.
    4. Learning about Factor V Leiden Thrombophilia. National Human Genome Research Institute (NHGRI). 2017. Available at: https://www.genome.gov/15015167/learning-about-factor-v-leiden-thrombophilia/. Accessed March 16, 2017.
    5. Papadakis M, McPhee S, Rabow M. Current Medical Diagnosis & Treatment 2015. 1st ed. San Francisco: McGraw Hill Medical; :293-297.

    Incorrect

    Factor V Leiden Thrombophilia poses an increased risk of abnormal clot formation to diagnosed patients, especially younger individuals and of European ancestry. An inherited mutation of the F5 gene is present. This mutation prevents argon plasma coagulation (APC) in the coagulation pathway from inactivating Factor V which allows more time for clot formation and bigger, abnormal clots to occur. Risk of clot formation increases with 1 copy of the mutation but is even greater with 2 copies present.

    A major risk factor for this condition and predisposes one to genetic diagnostic testing is having a close family member diagnosed with this disorder. Other risk factors include pregnancy, obesity, smoking, recent travel (especially on airplane), oral contraceptive pills, and hormone replacement therapy. Symptoms for this disorder include recurrent PE or DVT, occurrence of either at a young age, loss of pregnancy in the second or third trimester that is unexplained, or venous thrombosis in an unusual organ.

    Choice B is the correct answer to the question given. Factor V Leiden Thrombophilia is not a benign condition as it increases the risk of abnormal clot formation. Given the patients’ previous DVT diagnosis and current pregnancy, clinical observation and risk factor counseling would not be appropriate as sole therapy. The patient should receive treatment with anticoagulant for the duration of the pregnancy to decrease risk of abnormal clot formation. She should not be given just prophylaxis as she did have a previous DVT and is currently pregnant which is a risk factor.

    Discussion: Factor V Leiden Thrombophilia
    Etiology: Factor V Leiden Thrombophilia is an inherited condition in which a gene mutation is present that leads to a state of increased risk of clot formation. It is the most common inherited disorder that causes a hypercoagulable state. For this reason, the initial MD in the case described above took extra precautions in terms of directing the workup the patient received.
    Epidemiology: This mutation is more common in individuals of European ancestry. Individuals with 2 copies of the F5 mutation have an increased chance of up to 80 in 1,000 people developing abnormal clot formation3. Individuals with 1 copy of the F5 mutation have a 4-8 in 1,000 increased chance of abnormal clot formation. Normal risk for elevated clot formation is 1 in 1,000 individuals per year.

    Pathophysiology: Factor V protein is produced primarily in the liver from instructions provided by the F5 gene. Until the coagulation pathway is stimulated by a blood vessel injury, factor V protein rotates in the bloodstream in its inactive form. Factor V plays an important role in the coagulation pathway. It is activated by factor X2. Once triggered, Factor V works as the activator of prothrombin which converts to thrombin when stimulated. Thrombin then activates fibrinogen which becomes the clot forming material known as fibrin2. Another role of factor V in the coagulation system is being inactivated by activated protein C (APC) which stops clots from becoming too big. In the condition of Factor V Leiden Thrombophilia, a genetic mutation of the F5 gene is present and causes the hypercoagulability as Factor V can no longer be inactivated by APC3. Thus, clots more time to form and can become bigger leading to abnormalities. Individuals with 2 copies of the inherited mutation (one from father and one from mother) have the greatest risk of increased clot formation.

    History and physical exam: Individuals with a family member who has Factor V Leiden Thrombophilia should consider being tested as this is a risk factor for the disorder4. In the case presentation above, the patient was diagnosed after she was urged to undergo test by her primary care physician once her sister was diagnosed with the condition. Components in a patient history with Factor V Leiden Thrombophilia that will increase out of the ordinary clot formation are surgery, pregnancy, smoking, obesity, oral contraceptive pills, obesity, hormone replacement therapy, containing another mutation in the coagulation pathway and increasing age3. Symptoms include a DVT or PE that keep occurring, DVT before age 50 or age a young age, pregnancy loss in the second or third trimester that cannot be explained, or have a venous thrombosis take place in an out of the ordinary site4. Physical exam findings that may indicate a PE include dyspnea, tachypnea, and pain on inspiration5. Physical exam findings indicating a DVT include a warm to touch, unilateral extremity or area, swelling, and pain5. Also, the Wells criteria for predication of PE is very useful in the clinical setting5. However, in both a DVT and PE, there may be no presenting symptoms.

    Diagnostic: Genetic testing where the individual’s DNA is analyzed for the F5 mutation or APC resistance and/or a coagulation screening test is used for diagnosis. Testing is strongly recommended if there is a family history of the disorder.

    Management and treatment: Treatment and management is performed on a case by case basis as it depends on the individuals history and risks. Patients who are at low risk and have never had a PE or DVT may be treated with clinical observation as well as counseling on eliminating other risk factors and presenting symptoms to monitor oneself for4. If an individual only had one PE or DVT, lifelong anticoagulation is typically not necessary unless that person has other risks present4. Anticoagulation may be given as prophylaxis in these individuals in procedures that are high risk such as major surgery. At UMASS, all surgical patients are placed on some type of DVT prophylaxis prior to surgery as witnessed during the general surgery rotation. Individuals who are high risk, such as those with recurrent PE or DVT, should be placed on indefinite anticoagulation using agents such as Heparin4.

    References
    1. Reference G. F5 gene. Genetics Home Reference. 2017. Available at: https://ghr.nlm.nih.gov/gene/F5. Accessed March 16, 2017.
    2. Berkowitz A. Clinical Pathophysiology Made Ridiculously Simple. 1st ed. Miami Florida: MedMaster Inc.; 2007:129-132.
    3. Reference G. factor V Leiden thrombophilia. Genetics Home Reference. 2017. Available at: https://ghr.nlm.nih.gov/condition/factor-v-leiden-thrombophilia#synonyms. Accessed March 16, 2017.
    4. Learning about Factor V Leiden Thrombophilia. National Human Genome Research Institute (NHGRI). 2017. Available at: https://www.genome.gov/15015167/learning-about-factor-v-leiden-thrombophilia/. Accessed March 16, 2017.
    5. Papadakis M, McPhee S, Rabow M. Current Medical Diagnosis & Treatment 2015. 1st ed. San Francisco: McGraw Hill Medical; :293-297.

  156. Question 156 of 220
    156. Question

    duypatryns images
    A 52 year-old right hand dominant male presents to your office with an increasing contracture to his right ring finger. He was diagnosed with Dupuytren’s a year ago and has been watching his contracture closely. His father and brother both have Dupuytren’s with deformities much worse than his. On exam he has a large pretendinous cord in the palm in line with the ring finger (figure 1). He has an estimated 30 degree flexion contracture with good flexion strength. Table top test is shown in figure 2. He has full range of motion of adjacent fingers. What is the next best treatment option for this patient?

    Correct

    Dupuytren’s contracture is an abnormal thickening of the fibrous tissue under the skin in the hand that causes flexion of the fingers. The idiopathic condition is more common in men and strongly associated with a family history. Dupuytrens’s generally starts as a firm lump or pit in the palm as patients complain of difficulty placing the hand flat on a surface. The lump then progresses causing tissue under the skin to form bands and contract. The fingers then begin to bend (most commonly the 4th and 5th digits) and can no longer fully extend. This progressive finger flexion makes it difficult for patients to do certain things like wear gloves, put the hand in pocket, and hygiene.

    The first line treatment of Dupuytren’s is observation as the contracture may be non-progressive. Splinting and physical therapy are not effective for initial treatment, although therapy may be used after injections or surgical treatment. Collagenase clostridium (Xiaflex) is an injectable enzyme that causes the collagen to degrade within connective tissue which helps release the contracture. Xialfex is injected into the contracture and the patient is instructed to return to the office in 24-72 hours for passive extension manipulation of the involved fingers under local anesthesia. After the manipulation has been performed in the office the patient is often sent to physical therapy to be fitted for an extension night splint and to work on finger extension. The extension splint is typically worn at night for 4 weeks. The patient may have Xiaflex repeated at 6 weeks from the initial injection should a flexion contracture remain. Surgical treatments such as resection or fasciectomy are generally used when nonoperative treatments fail or when the flexion contracture progresses.

    References

    1. Black EM, Blazar PE. Dupuytren Disease: An Evolving Understanding of an Age Old Disease. JAAOS 2011; 19: 746-757.

    2. Dupuytren’s Disease. http://www.orthobullets.com. Accessed on 2/18/2017.

    Incorrect

    Dupuytren’s contracture is an abnormal thickening of the fibrous tissue under the skin in the hand that causes flexion of the fingers. The idiopathic condition is more common in men and strongly associated with a family history. Dupuytrens’s generally starts as a firm lump or pit in the palm as patients complain of difficulty placing the hand flat on a surface. The lump then progresses causing tissue under the skin to form bands and contract. The fingers then begin to bend (most commonly the 4th and 5th digits) and can no longer fully extend. This progressive finger flexion makes it difficult for patients to do certain things like wear gloves, put the hand in pocket, and hygiene.

    The first line treatment of Dupuytren’s is observation as the contracture may be non-progressive. Splinting and physical therapy are not effective for initial treatment, although therapy may be used after injections or surgical treatment. Collagenase clostridium (Xiaflex) is an injectable enzyme that causes the collagen to degrade within connective tissue which helps release the contracture. Xialfex is injected into the contracture and the patient is instructed to return to the office in 24-72 hours for passive extension manipulation of the involved fingers under local anesthesia. After the manipulation has been performed in the office the patient is often sent to physical therapy to be fitted for an extension night splint and to work on finger extension. The extension splint is typically worn at night for 4 weeks. The patient may have Xiaflex repeated at 6 weeks from the initial injection should a flexion contracture remain. Surgical treatments such as resection or fasciectomy are generally used when nonoperative treatments fail or when the flexion contracture progresses.

    References

    1. Black EM, Blazar PE. Dupuytren Disease: An Evolving Understanding of an Age Old Disease. JAAOS 2011; 19: 746-757.

    2. Dupuytren’s Disease. http://www.orthobullets.com. Accessed on 2/18/2017.

  157. Question 157 of 220
    157. Question

    A 34 year old woman presents to her primary care complaining of episodic hearing loss and vertigo for the past year. Once every couple of weeks she gets “dizzy spells” where she needs to lie down for about an hour until they pass. She thought they were related to her migraine headaches but she hasn’t had a migraine in many months and still has been experiencing the above symptoms. She also states her right ear sometimes feels full and she often has to turn her head to the left side to understand what people are saying. She has some nausea with the dizzy spells but denies any fever/chills, head trauma, or recent infection/illness. Her PCP sends her to get an audiogram which shows a low frequency hearing loss in her R ear with normal hearing in the left, as well as an MRI which shows no abnormalities. The provider also attempted to lay the patient on her back, head turned to the side and lower than the body but observed no signs of any nystagmus and it did not provoke symptoms. What is the most likely diagnosis?

    Correct

    Although all of the above answers can cause peripheral vertigo, the correct answer in the above vignette after eliminating the other choices is choice B, Meniere’s Disease. The classic presentation for this illness consists of episodic vertigo and hearing loss that tends to fluctuate (and often is low or high frequency loss), as well as tinnitus. Some data has also shown it can be more common in people who have a history of migraines.

    Choice A (BPPV) would also cause episodic periods of vertigo, but it is not linked to any hearing loss. These periods of vertigo are also most commonly 10-20 seconds long, rarely as long as a minute. In addition, the provider performed a Dix-Hallpike Maneuver during the visit, which with BPPV would likely cause some fatigable nystagmus, of which the patient had none.
    Choices C and D are diseases involving inflammation of the vestibular portion of eighth cranial nerve thought to be associated with viral illnesses. These would also cause dizziness, but in this case the symptoms of vertigo would be more constant instead of fluctuating. In addition, acoustic neuritis is not associated with hearing loss until the inflammation involves the bony labyrinth of the inner ear, when it becomes labyrinthitis.

    Choice E is unlikely as well, since the patient underwent an MRI which did not show any sign of mass suggesting an acoustic neuroma. Also, the hearing loss associated with an acoustic neuroma is irreversible and thus would not be coming and going as the above patient described.

    Discussion:
    Meniere’s Disease is a frustrating and complicated diagnosis, and while it may lack life-threatening qualities it can absolutely have a devastating effect on the patient’s quality of life. The typical patient presents with Meniere’s Disease in the 3rd-5th decade of life but people of all ages can be affected.1 Children with Meniere’s Disease will often have congenital problems, specifically malformations of the cochlea and vestibular organs of the inner ear. It affects as many as 150 out of 100,000 people in the general population, although it is difficult to definitively measure due to nonstandard and obscure reporting of cases.

    The inner ear is made up of not only the cochlea, which is responsible for interpreting air waves conducted through the external and middle ear and transmitting them to the brain, but also the vestibular organs which include the semicircular canals, the utricle, and the saccule. All of these together make up the bony labyrinth of the inner ear. These structures contain endolymph within them which facilitates the various vestibular and auditory functions in order to properly hear and balance. It has been demonstrated that patients who suffer from Meniere’s Disease display “endolymphatic hydrops,” or excess fluid build-up within these inner ear structures.1 Unfortunately the cause of this imbalance is not clearly understood. It is possible that there is some sort of blockage of the endolymphatic duct or narrowing of the vestibular aqueduct that causes the back-up, and other various immunologic, genetic, viral, and vascular etiologies hypotheses exist as well.1
    The clinical triad associated with Meniere’s Disease is episodic vertigo, sensorineural hearing loss, and tinnitus.1 It is very important to distinguish true vertigo from other types of “dizziness” during a patient interview. When a patient reports dizziness, the true complaint can range from vertigo (a spinning sensation with central and peripheral causes), presyncope (associated with vascular causes), disequilibrium (musculoskeletal, neuropathies, or cerebellar causes), or even vague/nonspecific causes (associated with psychiatric disorders/anxiety).2 While aural fullness, nausea, and tinnitus may or may not be present with this condition, it is necessary for diagnosis to have episodic vertigo as well as hearing loss.1 This defines the condition from various differentials, such as BPPV which would have similar episodic vertigo without any hearing loss, and vestibular neuritis/labyrinthitis, which would have more continuous symptoms of vertigo. The hearing loss experienced usually comes and goes and is primarily a lower frequency loss that progresses over the course of the disease. The intensity of hearing loss and vertigo vary from patient to patient.

    It is almost impossible to measure the amount of endolymph within the inner ear without risking permanent damage, and imaging leaves a lot to be desired. Often it can only be truly assessed post-mortem, which is little help to the patient. Therefore, the diagnosis of Meniere’s Disease is almost always a clinical one. Often the first diagnostic test that will be done is an audiogram to assess the patient’s level of hearing loss and differentiate between a sensorineural or conductive hearing loss. Again, this is commonly a low frequency loss, or occasionally high frequency with normal mid-frequency hearing.1 Additionally vestibular testing can be done, such as an electronystagmogram. This test measures inner ear response to caloric stimulation and various movements which can provide insight towards the source of the vertigo. The ear is irrigated with warm air followed by cold while the patient lies down, and as the temperature change triggers movement of fluid in the semicircular canals, nystagmic movement of the eyes follows.3 A normal test would result in equal nystagmic response in each eye. With Meniere’s, often times the affected side will display a lesser response to testing, while in contrast some sort of inner ear lesion or irritation may result in an increased response.3 Labs and imaging studies such as an MRI for diagnosis of Meniere’s Disease are primarily used to rule out other possible causes, most importantly brain lesions that can cause similar symptoms of vertigo. Some controversial tests attempt to assess endolymphatic hydrops such as glycerine, urea, and sorbitol stress tests but are not widely used due to their low specificity and sensitivity.1

    The treatment for Meniere’s usually doesn’t aim to cure the disease but rather to control the chronic symptoms associated with it. Hearing loss connected with this diagnosis does not have any treatment. However, the vertigo spells have several options to reduce severity and frequency. First line treatments for vertigo spells are anti-emetics (Zofran or Promethazine) to control nausea, and motion sickness medications (Valium or Antivert) to reduce spinning sensations. The most non-invasive option for long-term control is strict diet control. Often Meniere’s patients will be affected by certain dietary triggers, and it usually helps to restrict salt consumption as well as caffeine, alcohol, and nicotine, which reduce blood flow to the inner ear and also cause imbalances to fluid and electrolytes.1 Diuretics such as acetazolamide may also reduce the frequency of vertigo attacks.4 There are also several more “destructive” options to control refractory vertigo. These consist of injecting dexamethasone, a steroid, or gentamicin, an ototoxic antibiotic, into the inner ear. Gentamicin can be especially damaging to the cochlea and semicircular canals but may alleviate some chronic symptoms of Meniere’s. Most times it has the added complication of further reducing hearing ability. The most invasive options include surgical routes of endolymphatic sac procedures, vestibular nerve sections, or even a total labyrinthectomy, which would only be performed if the hearing loss in the affected ear was already completely lost.

    References:

    1. Moskowitz HS et al. Meniere Disease. UpToDate. https://www.uptodate.com/contents/meniere-disease?source=search_result&search=meniere%20disease&selectedTitle=1~29#H4. Updated 3/22/16. Accessed 4/22/17.
    2. Branch WT et al. Approach to the patient with dizziness. UpToDate. https://www.uptodate.com/contents/approach-to-the-patient-with-dizziness?source=search_result&search=disequilibrium&selectedTitle=1~79#H7. Updated 10/20/14. Accessed 4/22/17.
    3. Li JC. Meniere Disease (Idiopathic Endolymphatic Hydrops) Workup. UpToDate. http://emedicine.medscape.com/article/1159069-workup#c1. Updated 11/29/16. Accessed 4/22/17.
    4. Mayo Clinic Staff. Meniere’s Disease – Treatment and drugs. Mayo Clinic. http://www.mayoclinic.org/diseases-conditions/menieres-disease/basics/treatment/con-20028251. Published 11/26/15. Accessed 4/22/17.

    Incorrect

    Although all of the above answers can cause peripheral vertigo, the correct answer in the above vignette after eliminating the other choices is choice B, Meniere’s Disease. The classic presentation for this illness consists of episodic vertigo and hearing loss that tends to fluctuate (and often is low or high frequency loss), as well as tinnitus. Some data has also shown it can be more common in people who have a history of migraines.

    Choice A (BPPV) would also cause episodic periods of vertigo, but it is not linked to any hearing loss. These periods of vertigo are also most commonly 10-20 seconds long, rarely as long as a minute. In addition, the provider performed a Dix-Hallpike Maneuver during the visit, which with BPPV would likely cause some fatigable nystagmus, of which the patient had none.
    Choices C and D are diseases involving inflammation of the vestibular portion of eighth cranial nerve thought to be associated with viral illnesses. These would also cause dizziness, but in this case the symptoms of vertigo would be more constant instead of fluctuating. In addition, acoustic neuritis is not associated with hearing loss until the inflammation involves the bony labyrinth of the inner ear, when it becomes labyrinthitis.

    Choice E is unlikely as well, since the patient underwent an MRI which did not show any sign of mass suggesting an acoustic neuroma. Also, the hearing loss associated with an acoustic neuroma is irreversible and thus would not be coming and going as the above patient described.

    Discussion:
    Meniere’s Disease is a frustrating and complicated diagnosis, and while it may lack life-threatening qualities it can absolutely have a devastating effect on the patient’s quality of life. The typical patient presents with Meniere’s Disease in the 3rd-5th decade of life but people of all ages can be affected.1 Children with Meniere’s Disease will often have congenital problems, specifically malformations of the cochlea and vestibular organs of the inner ear. It affects as many as 150 out of 100,000 people in the general population, although it is difficult to definitively measure due to nonstandard and obscure reporting of cases.

    The inner ear is made up of not only the cochlea, which is responsible for interpreting air waves conducted through the external and middle ear and transmitting them to the brain, but also the vestibular organs which include the semicircular canals, the utricle, and the saccule. All of these together make up the bony labyrinth of the inner ear. These structures contain endolymph within them which facilitates the various vestibular and auditory functions in order to properly hear and balance. It has been demonstrated that patients who suffer from Meniere’s Disease display “endolymphatic hydrops,” or excess fluid build-up within these inner ear structures.1 Unfortunately the cause of this imbalance is not clearly understood. It is possible that there is some sort of blockage of the endolymphatic duct or narrowing of the vestibular aqueduct that causes the back-up, and other various immunologic, genetic, viral, and vascular etiologies hypotheses exist as well.1
    The clinical triad associated with Meniere’s Disease is episodic vertigo, sensorineural hearing loss, and tinnitus.1 It is very important to distinguish true vertigo from other types of “dizziness” during a patient interview. When a patient reports dizziness, the true complaint can range from vertigo (a spinning sensation with central and peripheral causes), presyncope (associated with vascular causes), disequilibrium (musculoskeletal, neuropathies, or cerebellar causes), or even vague/nonspecific causes (associated with psychiatric disorders/anxiety).2 While aural fullness, nausea, and tinnitus may or may not be present with this condition, it is necessary for diagnosis to have episodic vertigo as well as hearing loss.1 This defines the condition from various differentials, such as BPPV which would have similar episodic vertigo without any hearing loss, and vestibular neuritis/labyrinthitis, which would have more continuous symptoms of vertigo. The hearing loss experienced usually comes and goes and is primarily a lower frequency loss that progresses over the course of the disease. The intensity of hearing loss and vertigo vary from patient to patient.

    It is almost impossible to measure the amount of endolymph within the inner ear without risking permanent damage, and imaging leaves a lot to be desired. Often it can only be truly assessed post-mortem, which is little help to the patient. Therefore, the diagnosis of Meniere’s Disease is almost always a clinical one. Often the first diagnostic test that will be done is an audiogram to assess the patient’s level of hearing loss and differentiate between a sensorineural or conductive hearing loss. Again, this is commonly a low frequency loss, or occasionally high frequency with normal mid-frequency hearing.1 Additionally vestibular testing can be done, such as an electronystagmogram. This test measures inner ear response to caloric stimulation and various movements which can provide insight towards the source of the vertigo. The ear is irrigated with warm air followed by cold while the patient lies down, and as the temperature change triggers movement of fluid in the semicircular canals, nystagmic movement of the eyes follows.3 A normal test would result in equal nystagmic response in each eye. With Meniere’s, often times the affected side will display a lesser response to testing, while in contrast some sort of inner ear lesion or irritation may result in an increased response.3 Labs and imaging studies such as an MRI for diagnosis of Meniere’s Disease are primarily used to rule out other possible causes, most importantly brain lesions that can cause similar symptoms of vertigo. Some controversial tests attempt to assess endolymphatic hydrops such as glycerine, urea, and sorbitol stress tests but are not widely used due to their low specificity and sensitivity.1

    The treatment for Meniere’s usually doesn’t aim to cure the disease but rather to control the chronic symptoms associated with it. Hearing loss connected with this diagnosis does not have any treatment. However, the vertigo spells have several options to reduce severity and frequency. First line treatments for vertigo spells are anti-emetics (Zofran or Promethazine) to control nausea, and motion sickness medications (Valium or Antivert) to reduce spinning sensations. The most non-invasive option for long-term control is strict diet control. Often Meniere’s patients will be affected by certain dietary triggers, and it usually helps to restrict salt consumption as well as caffeine, alcohol, and nicotine, which reduce blood flow to the inner ear and also cause imbalances to fluid and electrolytes.1 Diuretics such as acetazolamide may also reduce the frequency of vertigo attacks.4 There are also several more “destructive” options to control refractory vertigo. These consist of injecting dexamethasone, a steroid, or gentamicin, an ototoxic antibiotic, into the inner ear. Gentamicin can be especially damaging to the cochlea and semicircular canals but may alleviate some chronic symptoms of Meniere’s. Most times it has the added complication of further reducing hearing ability. The most invasive options include surgical routes of endolymphatic sac procedures, vestibular nerve sections, or even a total labyrinthectomy, which would only be performed if the hearing loss in the affected ear was already completely lost.

    References:

    1. Moskowitz HS et al. Meniere Disease. UpToDate. https://www.uptodate.com/contents/meniere-disease?source=search_result&search=meniere%20disease&selectedTitle=1~29#H4. Updated 3/22/16. Accessed 4/22/17.
    2. Branch WT et al. Approach to the patient with dizziness. UpToDate. https://www.uptodate.com/contents/approach-to-the-patient-with-dizziness?source=search_result&search=disequilibrium&selectedTitle=1~79#H7. Updated 10/20/14. Accessed 4/22/17.
    3. Li JC. Meniere Disease (Idiopathic Endolymphatic Hydrops) Workup. UpToDate. http://emedicine.medscape.com/article/1159069-workup#c1. Updated 11/29/16. Accessed 4/22/17.
    4. Mayo Clinic Staff. Meniere’s Disease – Treatment and drugs. Mayo Clinic. http://www.mayoclinic.org/diseases-conditions/menieres-disease/basics/treatment/con-20028251. Published 11/26/15. Accessed 4/22/17.

  158. Question 158 of 220
    158. Question

    A 71-year-old female presents with a new diagnosis of a breast mass and is complaining of shortness of breath and non-productive cough. Her shortness of breath has been worsening over the weekend and is exacerbated by light activity, so she came into the Emergency Department. It is typically alleviated by supplement oxygen, but not lately. She denies any chest pain. Her vital signs are: temperature 98°F, heart rate at 82 bpm, blood pressure 119/67 RUE, respirations at 20, and PO2 at 94% on 4 liters of oxygen. Her Laboratory results include a BMP within normal limits, CBC with hemoglobin at 10.9 and hematocrit at 33.2, white blood cell count and platelets within normal range, and an LDH elevated at 796. A chest x-ray was obtained in the Emergency Department which showed blunting of the left costophrenic angle and a hyperdense area in the left apical lung. Given all relevant clinical information, what would be the most likely diagnosis?

    Correct

    The answer is A. Exudative pleural effusions form due to an underlying disease. This patient presented with shortness of breath, which is one of the most common symptoms patients present with when they have a pleural effusion. This patient also has newly diagnosed breast cancer, which was indicated by her breast mass stated in the question and supported by her chest x-ray. Effusions can oftentimes occur in patients who have malignancies because of changes in the permeability of pleural membranes secondary to malignancy (10). A diagnosis of an exudative effusion can then be made confirmed if because the pleural fluid LDH is greater than two thirds of the upper limit of normal for serum LDH (8).

    Choice B is incorrect because transudative pleural effusions are usually caused by disease processes such as congestive heart failure, cirrhosis or pneumonia renal failure (9). Evaluation of the pleural fluid is used to confirm the diagnosis.

    Choice C is incorrect because patients with TB have a productive cough of bloody sputum and chest pain. Although she has a cough, it is non-productive and she denies chest pain. The classic chest x-ray finding of tuberculosis is an upper lobe cavitary opacification. Another common symptom is fever, but she is afebrile because her vitals show her temperature at 98°F (7).

    Choice D is incorrect because, like TB, patients with bacterial pneumonia usually have a fever, productive cough, and chest pain. This patient denies these symptoms. Her chest x-ray showed blunting of the left costophrenic angle and a hyperdense area in the left apical lung. These findings suggest a cancerous mass and a pleural effusion in the base of the left lung, and not a lobar infiltrate commonly seen on a chest x-ray that is positive for bacterial pneumonia (6)

    References
    1. Pleural Effusion Clinical Presentation: History, Physical Examination. Emedicinemedscapecom. 2017. Available at: http://emedicine.medscape.com/article/299959-clinical#showall. Accessed April 24, 2017.
    2. What Are Pleural Effusions?. WebMD. 2017. Available at: http://www.webmd.com/lung/pleural-effusion-symptoms-causes-treatments#1-4. Accessed April 24, 2017.
    3. Management of the Patient With a Malignant Pleural Effusion. Medscape. 2017. Available at: http://www.medscape.com/viewarticle/736939_3. Accessed April 24, 2017.
    4. Management of Malignant Pleural Effusions. Uptodatecom. 2017. Available at: https://www.uptodate.com/contents/management-of-malignant-pleural-effusions. Accessed April 24, 2017.
    5. Pleural Effusion: Background, Anatomy, Etiology. Emedicinemedscapecom. 2017. Available at: http://emedicine.medscape.com/article/299959-overview#a6. Accessed April 24, 2017.
    6. Pneumonia-Exams and Tests. WebMD. 2017. Available at: http://www.webmd.com/lung/tc/pneumonia-exams-and-tests. Accessed April 24, 2017.
    7. CDC | TB | Basic TB Facts | Signs & Symptoms. Cdcgov. 2017. Available at: https://www.cdc.gov/tb/topic/basics/signsandsymptoms.htm. Accessed April 24, 2017.
    8. Light J. Diagnostic Approach to Pleural Effusion in Adults – American Family Physician. Aafporg. 2017. Available at: http://www.aafp.org/afp/2006/0401/p1211.html. Accessed April 24, 2017.
    9. Light’s Criteria: Light’s Criteria. Emedicinemedscapecom. 2017. Available at: http://emedicine.medscape.com/article/2172232-overview#a1. Accessed April 24, 2017.
    10. Fluid Around the Lungs or Malignant Pleural Effusion. CancerNet. 2017. Available at: http://www.cancer.net/navigating-cancer-care/side-effects/fluid-around-lungs-or-malignant-pleural-effusion. Accessed April 24, 2017.

    Incorrect

    The answer is A. Exudative pleural effusions form due to an underlying disease. This patient presented with shortness of breath, which is one of the most common symptoms patients present with when they have a pleural effusion. This patient also has newly diagnosed breast cancer, which was indicated by her breast mass stated in the question and supported by her chest x-ray. Effusions can oftentimes occur in patients who have malignancies because of changes in the permeability of pleural membranes secondary to malignancy (10). A diagnosis of an exudative effusion can then be made confirmed if because the pleural fluid LDH is greater than two thirds of the upper limit of normal for serum LDH (8).

    Choice B is incorrect because transudative pleural effusions are usually caused by disease processes such as congestive heart failure, cirrhosis or pneumonia renal failure (9). Evaluation of the pleural fluid is used to confirm the diagnosis.

    Choice C is incorrect because patients with TB have a productive cough of bloody sputum and chest pain. Although she has a cough, it is non-productive and she denies chest pain. The classic chest x-ray finding of tuberculosis is an upper lobe cavitary opacification. Another common symptom is fever, but she is afebrile because her vitals show her temperature at 98°F (7).

    Choice D is incorrect because, like TB, patients with bacterial pneumonia usually have a fever, productive cough, and chest pain. This patient denies these symptoms. Her chest x-ray showed blunting of the left costophrenic angle and a hyperdense area in the left apical lung. These findings suggest a cancerous mass and a pleural effusion in the base of the left lung, and not a lobar infiltrate commonly seen on a chest x-ray that is positive for bacterial pneumonia (6)

    References
    1. Pleural Effusion Clinical Presentation: History, Physical Examination. Emedicinemedscapecom. 2017. Available at: http://emedicine.medscape.com/article/299959-clinical#showall. Accessed April 24, 2017.
    2. What Are Pleural Effusions?. WebMD. 2017. Available at: http://www.webmd.com/lung/pleural-effusion-symptoms-causes-treatments#1-4. Accessed April 24, 2017.
    3. Management of the Patient With a Malignant Pleural Effusion. Medscape. 2017. Available at: http://www.medscape.com/viewarticle/736939_3. Accessed April 24, 2017.
    4. Management of Malignant Pleural Effusions. Uptodatecom. 2017. Available at: https://www.uptodate.com/contents/management-of-malignant-pleural-effusions. Accessed April 24, 2017.
    5. Pleural Effusion: Background, Anatomy, Etiology. Emedicinemedscapecom. 2017. Available at: http://emedicine.medscape.com/article/299959-overview#a6. Accessed April 24, 2017.
    6. Pneumonia-Exams and Tests. WebMD. 2017. Available at: http://www.webmd.com/lung/tc/pneumonia-exams-and-tests. Accessed April 24, 2017.
    7. CDC | TB | Basic TB Facts | Signs & Symptoms. Cdcgov. 2017. Available at: https://www.cdc.gov/tb/topic/basics/signsandsymptoms.htm. Accessed April 24, 2017.
    8. Light J. Diagnostic Approach to Pleural Effusion in Adults – American Family Physician. Aafporg. 2017. Available at: http://www.aafp.org/afp/2006/0401/p1211.html. Accessed April 24, 2017.
    9. Light’s Criteria: Light’s Criteria. Emedicinemedscapecom. 2017. Available at: http://emedicine.medscape.com/article/2172232-overview#a1. Accessed April 24, 2017.
    10. Fluid Around the Lungs or Malignant Pleural Effusion. CancerNet. 2017. Available at: http://www.cancer.net/navigating-cancer-care/side-effects/fluid-around-lungs-or-malignant-pleural-effusion. Accessed April 24, 2017.

  159. Question 159 of 220
    159. Question

    A 29- year old man, with a recent history of bipolar disorder, presents to his PCP with a chief complaint of a “new rash”. The patient states that the rash began approximately 4 weeks ago and seems to be getting worse. The patient describes the rash as itchy and irritating. Physical Exam findings show numerous erythematous plaques with silvery scale located on the elbows, forearms, knees, and posterior scalp. The patient states that he has used moisturizer on the affected area due to its dry appearance, which has not seemed to help. What is the most likely diagnosis?

    Correct

    Answer is D. Psoriasis is a chronic, multisystem inflammatory, immune-mediated disorder. Plaque psoriasis of the skin is the most common type of psoriasis. It is caused by epidermal proliferation that results in a thickened stratum corneum layer of the skin. Research has shown that uncontrolled T-cell activation and cytokine release causes immature skin cells to mature rapidly- up to 7 times the normal rate resulting in epidermal thickening (James 2011). The etiology of psoriasis is still unknown, although many factors can increase a person’s risk of developing the disorder. Evidence has shown a genetic disposition in which 30-50% of patients with the disease have a positive family history of psoriasis. Many factors can trigger psoriasis including stress, infections, obesity, skin trauma (known as Koebner phenomenon), and certain medications, such as Lithium, NSAIDs, and beta blockers James 2011). Psoriasis can affect all ages, but most commonly it occurs in the 3rd decade of life. The onset can be gradual or sudden. The hallmark sign of psoriasis is a well-demarcated erythematous plaque with silvery scale. The psoriatic plaques most often affect the extensor surfaces of the body, and most commonly occur on the elbows, knees, and scalp. Symptoms include pruritus, pain, and arthralgia (Papadakis 2016).

    There are many types of psoriasis including psoriatic arthritis, in which 30% of people suffering from psoriasis develop joint pain and deformation. Nail psoriasis affects the nail bed causing nail pitting and discoloration. Guttate psoriasis usually follows a strep infection and is characterized by multiple small plaques peppered throughout the body. Pustular psoriasis is characterized by small pustules on the hands and feet (Williams 2014). Palmar/ Plantar psoriasis symptoms are confined to the hands and feet. Inverse psoriasis symptoms occur mainly in skin folds and are often mistaken for a tinea infection (James 2011).

    Treatment for Psoriasis varies depending on the severity, location, symptoms, and patient preference. There is no cure for psoriasis. The goal of treatment is to control the symptoms to a manageable level for the patient. Treatments range from topical to systemic (James 2011). Topical treatment includes topic steroids, which decrease inflammation. Calcipotriene (Dovonex) is a vitamin D analog that binds to keratinocyte receptors, which alters keratinocyte proliferation. Tazarotene (Tazorac) is a topical retinoid that alters epidermal proliferation and decreases scale. Systemic therapy includes Methotrexate, which is an immunosuppressive drug that inhibits cellular proliferation. Hepatotoxicity is a common side effect, along with GI symptoms. Newer medications such as biologics, like Enbrel and Humira are weekly injectables that suppress inflammation through T cell interference. Increased risk of infection and lymphoma are adverse effects to be considered with these drugs (James 2011).

    In question 1 the patient is 29-years old (psoriasis onset is usually in the 3rd decade of life) with a recent history of bipolar disorder. Lithium is a common medication for bipolar disorder and has been a known trigger of psoriasis. Although it is not stated in the vignette, the provider should ask what medications the patient is currently taking for bipolar disorder. The patient’s physical exam shows erythematous plaques with silvery scale, which is the hallmark sign of psoriasis. The plaques are located on the patient’s scalp and extensor surfaces, which is also a classic finding of psoriasis.
    Lichen Planus is an idiopathic cell-mediated immune response commonly associated Hepatitis C. It is characterized by pruritic flat-topped violaceous papules (James 2011). Fine white streaks, called Wickman’s striae, are commonly found within the lesions. Lichen Planus can affect the scalp, nails, genitals, and mucous membranes. It most commonly occurs in the flexor creases, unlike psoriasis, which normally affects the extensor surfaces. The oral lesions associated with Lichen Planus are described as lacy white patches found on the buccal mucosa. Treatment includes topical steroids, systemic steroids for severe cases, and phototherapy. The rash can resolve within 8-12 months with treatment (Williams 2014).

    Atopic dermatitis, or eczema, is an altered immune reaction (increased IgE production) that usually has a childhood onset, unlike psoriasis. Patients with atopic dermatitis usually display the atopic triad of symptoms including eczema, allergies, and asthma. It is characterized by chronic tiny erythematous, edematous, pruritic, ill-defined blisters, that can produce scale (similar to psoriasis) from the “itch-scratch cycle” (Williams 2014). The symptoms usually occur in the flexor creases, unlike psoriasis. Treatment usually consists of moisturizers, topical steroids, systemic steroids for severe acute flares, and Calcineurin Inhibitors (Protopic), which can trigger an immune response to potentially clear up the eczema. Antihistamines can be utilized for pruritus (Papadakis 2016).
    Lichen Simplex Chronicus is characterized by skin thickening in patients secondary to repetitive rubbing and/ or scratching. It is caused by the chronic itch scratch cycle suffered by patients with conditions such as atopic dermatitis. The affected areas are described as scaly, well-demarcated, rough papules, with exaggerated skin lines. Treatment usually consists of potent topical steroids to decrease inflammation, antihistamines for pruritus relief, and behavior modification (Papadakis 2016).

    References:
    1. James W, Elston D, Berger T, Andrews G. Andrews’ Diseases Of The Skin. [London]: Saunders/ Elsevier; 2011.
    2. Papadakis MA, McPhee SJ. Current medical diagnosis and treatment 2016. New York: McGraw-Hill Education; 2016.
    3. Williams D. PANCE Prep Pearls. Lexington, KY: CreateSpace; 2014.

    Incorrect

    Answer is D. Psoriasis is a chronic, multisystem inflammatory, immune-mediated disorder. Plaque psoriasis of the skin is the most common type of psoriasis. It is caused by epidermal proliferation that results in a thickened stratum corneum layer of the skin. Research has shown that uncontrolled T-cell activation and cytokine release causes immature skin cells to mature rapidly- up to 7 times the normal rate resulting in epidermal thickening (James 2011). The etiology of psoriasis is still unknown, although many factors can increase a person’s risk of developing the disorder. Evidence has shown a genetic disposition in which 30-50% of patients with the disease have a positive family history of psoriasis. Many factors can trigger psoriasis including stress, infections, obesity, skin trauma (known as Koebner phenomenon), and certain medications, such as Lithium, NSAIDs, and beta blockers James 2011). Psoriasis can affect all ages, but most commonly it occurs in the 3rd decade of life. The onset can be gradual or sudden. The hallmark sign of psoriasis is a well-demarcated erythematous plaque with silvery scale. The psoriatic plaques most often affect the extensor surfaces of the body, and most commonly occur on the elbows, knees, and scalp. Symptoms include pruritus, pain, and arthralgia (Papadakis 2016).

    There are many types of psoriasis including psoriatic arthritis, in which 30% of people suffering from psoriasis develop joint pain and deformation. Nail psoriasis affects the nail bed causing nail pitting and discoloration. Guttate psoriasis usually follows a strep infection and is characterized by multiple small plaques peppered throughout the body. Pustular psoriasis is characterized by small pustules on the hands and feet (Williams 2014). Palmar/ Plantar psoriasis symptoms are confined to the hands and feet. Inverse psoriasis symptoms occur mainly in skin folds and are often mistaken for a tinea infection (James 2011).

    Treatment for Psoriasis varies depending on the severity, location, symptoms, and patient preference. There is no cure for psoriasis. The goal of treatment is to control the symptoms to a manageable level for the patient. Treatments range from topical to systemic (James 2011). Topical treatment includes topic steroids, which decrease inflammation. Calcipotriene (Dovonex) is a vitamin D analog that binds to keratinocyte receptors, which alters keratinocyte proliferation. Tazarotene (Tazorac) is a topical retinoid that alters epidermal proliferation and decreases scale. Systemic therapy includes Methotrexate, which is an immunosuppressive drug that inhibits cellular proliferation. Hepatotoxicity is a common side effect, along with GI symptoms. Newer medications such as biologics, like Enbrel and Humira are weekly injectables that suppress inflammation through T cell interference. Increased risk of infection and lymphoma are adverse effects to be considered with these drugs (James 2011).

    In question 1 the patient is 29-years old (psoriasis onset is usually in the 3rd decade of life) with a recent history of bipolar disorder. Lithium is a common medication for bipolar disorder and has been a known trigger of psoriasis. Although it is not stated in the vignette, the provider should ask what medications the patient is currently taking for bipolar disorder. The patient’s physical exam shows erythematous plaques with silvery scale, which is the hallmark sign of psoriasis. The plaques are located on the patient’s scalp and extensor surfaces, which is also a classic finding of psoriasis.
    Lichen Planus is an idiopathic cell-mediated immune response commonly associated Hepatitis C. It is characterized by pruritic flat-topped violaceous papules (James 2011). Fine white streaks, called Wickman’s striae, are commonly found within the lesions. Lichen Planus can affect the scalp, nails, genitals, and mucous membranes. It most commonly occurs in the flexor creases, unlike psoriasis, which normally affects the extensor surfaces. The oral lesions associated with Lichen Planus are described as lacy white patches found on the buccal mucosa. Treatment includes topical steroids, systemic steroids for severe cases, and phototherapy. The rash can resolve within 8-12 months with treatment (Williams 2014).

    Atopic dermatitis, or eczema, is an altered immune reaction (increased IgE production) that usually has a childhood onset, unlike psoriasis. Patients with atopic dermatitis usually display the atopic triad of symptoms including eczema, allergies, and asthma. It is characterized by chronic tiny erythematous, edematous, pruritic, ill-defined blisters, that can produce scale (similar to psoriasis) from the “itch-scratch cycle” (Williams 2014). The symptoms usually occur in the flexor creases, unlike psoriasis. Treatment usually consists of moisturizers, topical steroids, systemic steroids for severe acute flares, and Calcineurin Inhibitors (Protopic), which can trigger an immune response to potentially clear up the eczema. Antihistamines can be utilized for pruritus (Papadakis 2016).
    Lichen Simplex Chronicus is characterized by skin thickening in patients secondary to repetitive rubbing and/ or scratching. It is caused by the chronic itch scratch cycle suffered by patients with conditions such as atopic dermatitis. The affected areas are described as scaly, well-demarcated, rough papules, with exaggerated skin lines. Treatment usually consists of potent topical steroids to decrease inflammation, antihistamines for pruritus relief, and behavior modification (Papadakis 2016).

    References:
    1. James W, Elston D, Berger T, Andrews G. Andrews’ Diseases Of The Skin. [London]: Saunders/ Elsevier; 2011.
    2. Papadakis MA, McPhee SJ. Current medical diagnosis and treatment 2016. New York: McGraw-Hill Education; 2016.
    3. Williams D. PANCE Prep Pearls. Lexington, KY: CreateSpace; 2014.

  160. Question 160 of 220
    160. Question

    A 34-year-old man is evaluated for hypercalcemia incidentally discovered on laboratory studies performed for another indication. He reports no hypercalcemia-related symptoms such as upset stomach, confusion, fatigue, or palpitations. Medical history is significant for gastroesophageal reflux disease. Family history is notable for a brother who has a “calcium” problem. The only medication he is taking is omeprazole. On physical examination, temperature is 36.2 °C (97.2 °F), blood pressure is 124/70 mm Hg, pulse rate is 76/min, and respiration rate is 12/min. BMI is 25. Chest, heart, and abdominal examinations are normal, as is the remainder of his examination. Laboratory studies are significant for a serum calcium level of 11.1 mg/dL (2.8 mmol/L), parathyroid hormone level of 55 pg/mL (55 ng/L), and 25-hydroxyvitamin D level of 35 ng/mL (87.4 nmol/L). Kidney and thyroid function studies are normal. Which of the following is the most appropriate next step in management?

    Correct

    Answer B. This patient’s urine calcium and creatinine levels should be measured. His laboratory values are consistent with hypercalcemia, and her parathyroid hormone level is toward the upper end of the normal range. In his age group and with a family member with a suspicious history, it is important to distinguish between primary hyperparathyroidism and familial Hypocalciuric hypercalcemia (FHH). The distinction between primary hyperparathyroidism and FHH can be made by a 24-hour urine collection for calcium and creatinine, which will establish the amount of kidney calcium excretion and will allow evaluation of the calcium-creatinine ratio. Total urine calcium of less than 200 mg/24 h and a calcium-creatinine ratio less than 0.01 are highly suggestive of FHH. Bone densitometry is not indicated in this age group in the absence of fragility fractures or other risk factors such as long-term high-dose glucocorticoid use or primary hyperparathyroidism. A parathyroid sestamibi scan is a very useful nuclear imaging study for localization of adenomas in patients with primary hyperparathyroidism or parathyroid cancer. However, primary hyperparathyroidism has not been confirmed in this patient with suspicion for FHH, making this study premature. The need for surgical treatment in this patient has also not been established. Therefore, surgical referral for parathyroidectomy would not be appropriate.

    References:
    1. Mcmurtry CT, Schranck FW, Walkenhorst DA, et al. Significant developmental elevation in serum parathyroid hormone levels in a large kindred with familial benign (hypocalciuric) hypercalcemia. The American Journal of Medicine. 1992;93(3):247-258. doi:10.1016/0002-9343(92)90229-5.
    2. Marx SJ, Attie MF, Levine MA, Spiegel AM, Downs RW, Lasker RD. The Hypocalciuric or Benign Variant of Familial Hypercalcemia: Clinical and Biochemical Features in Fifteen Kindreds. Medicine. 1981;60(6):397-412. doi:10.1097/00005792-198111000-00002.
    3. Shinall M, Dahir K, Broome J. Differentiating Familial Hypocalciuric Hypercalcemia from Primary Hyperparathyroidism. Endocrine Practice. 2013;19(4):697-702. doi:10.4158/ep12284.ra.
    4. Law WM. Familial Benign Hypercalcemia (Hypocalciuric Hypercalcemia). Annals of Internal Medicine. 1985;102(4):511. doi:10.7326/0003-4819-102-4-511.
    5. Marx SJ. Familial Hypocalciuric Hypercalcemia: Recognition Among Patients Referred After Unsuccessful Parathyroid Exploration. Annals of Internal Medicine. 1980;92(3):351. doi:10.7326/0003-4819-92-3-351.

    Incorrect

    Answer B. This patient’s urine calcium and creatinine levels should be measured. His laboratory values are consistent with hypercalcemia, and her parathyroid hormone level is toward the upper end of the normal range. In his age group and with a family member with a suspicious history, it is important to distinguish between primary hyperparathyroidism and familial Hypocalciuric hypercalcemia (FHH). The distinction between primary hyperparathyroidism and FHH can be made by a 24-hour urine collection for calcium and creatinine, which will establish the amount of kidney calcium excretion and will allow evaluation of the calcium-creatinine ratio. Total urine calcium of less than 200 mg/24 h and a calcium-creatinine ratio less than 0.01 are highly suggestive of FHH. Bone densitometry is not indicated in this age group in the absence of fragility fractures or other risk factors such as long-term high-dose glucocorticoid use or primary hyperparathyroidism. A parathyroid sestamibi scan is a very useful nuclear imaging study for localization of adenomas in patients with primary hyperparathyroidism or parathyroid cancer. However, primary hyperparathyroidism has not been confirmed in this patient with suspicion for FHH, making this study premature. The need for surgical treatment in this patient has also not been established. Therefore, surgical referral for parathyroidectomy would not be appropriate.

    References:
    1. Mcmurtry CT, Schranck FW, Walkenhorst DA, et al. Significant developmental elevation in serum parathyroid hormone levels in a large kindred with familial benign (hypocalciuric) hypercalcemia. The American Journal of Medicine. 1992;93(3):247-258. doi:10.1016/0002-9343(92)90229-5.
    2. Marx SJ, Attie MF, Levine MA, Spiegel AM, Downs RW, Lasker RD. The Hypocalciuric or Benign Variant of Familial Hypercalcemia: Clinical and Biochemical Features in Fifteen Kindreds. Medicine. 1981;60(6):397-412. doi:10.1097/00005792-198111000-00002.
    3. Shinall M, Dahir K, Broome J. Differentiating Familial Hypocalciuric Hypercalcemia from Primary Hyperparathyroidism. Endocrine Practice. 2013;19(4):697-702. doi:10.4158/ep12284.ra.
    4. Law WM. Familial Benign Hypercalcemia (Hypocalciuric Hypercalcemia). Annals of Internal Medicine. 1985;102(4):511. doi:10.7326/0003-4819-102-4-511.
    5. Marx SJ. Familial Hypocalciuric Hypercalcemia: Recognition Among Patients Referred After Unsuccessful Parathyroid Exploration. Annals of Internal Medicine. 1980;92(3):351. doi:10.7326/0003-4819-92-3-351.

  161. Question 161 of 220
    161. Question

    A 74 yo male presents to the emergency department with complaints of frequent falls and headache. On physical examination, his head is normocephalic, atraumatic. All other physical examination findings were within normal limits except during the neurological exam the patient exhibits gait instability. The patient also admits that he had difficulty maintaining his balance for the last couple of weeks, which made him fall multiple times. The patient denies dizziness, changes in vision, nausea or vomiting, and extremity pain. On CT scan of the head, you note a crescent shaped hematoma crossing a suture line. Which of the following is the most likely diagnosis?

    Correct

    Correct Answer: B – subdural hematoma. A is incorrect because epidural hematomas do not cross suture lines and moreover epidural hematomas are “convex” (lens) in shape. C is incorrect because intraparenchymal hematomas do not follow suture lines, as IPHs occur in the brain tissue. Finally, answer D is incorrect since the patient did not complain of sudden onset of headache and subarachnoid hemorrhages normally present with sudden onset of “thunder-clap”, “worst headache of my life”. B is the most likely diagnosis because the CT scan showed a hematoma crossing suture lines and the hematoma was “crescent”-shaped, which is indicative of a subdural hematoma. However, in chronic SDH, the shape can change to biconvex.

    Subdural hematoma

    The brain is enclosed within a closed and fixed space and is surrounded by cerebrospinal fluid (CSF), which acts as a shock absorbent. The brain is surrounded by 3 layers: the pia is the innermost layer, the subarachnoid is the middle layer, and the dura is the outermost fibrous membrane – the extremely tough layer of the three meningeal layers. The skull encompasses all of these layers. Brains have fixed volume, in which 80% is brain tissue, 10% is CSF, and 10% is blood (Kaye et al). Increases in intracranial pressure (ICP) is fatal for brain function. Brain hemorrhage is the most likely cause of increased intracranial pressure (Kaye et al). There are two type of brain hemorrhage: intracranial, which includes intraventricular and intraparenchymal hemorrhages, and extracranial, which includes subarachnoid, subdural, and epidural hemorrhages.

    In subdural hematomas (SDH), bleeding occurs in-between the dura and subarachnoid membrane. SDH is further divided into subtypes based on time of injury. Acute is 1 week from injury. SDHs can also be mixed in nature, as acute injuries can turn into chronic SDHs. Acute SDHs are the most common type of traumatic intracranial hemorrhage. Acute SDHs can also present with other brain injuries such as subarachnoid hemorrhage, diffuse axonal injury, and so on. 72% of SDH occur following fall or assaults and 24% are associated with MVA. Chronic SDHs more commonly occur in the elderly population and are the most common type of treatable cause of dementia. Chronic SDHs are more common in males and the incidence increases with age. SDHs can also present in children. Bilateral presentation of SDHs in children is associated with underlying physiology, while interhemispheric presentation is associated with child abuse, also known as shaken baby syndrome (Meagher et al).

    Subacute
    As the clot ages and protein degradation occurs, the density starts to drop. At some point between 3 and 21 days (typically 10-14 days), the density will drop to ~ 30 HU and become isodense to the adjacent cortex, making identification potentially tricky, especially if subdural collections are bilateral 4. Contrast-enhanced CT is often useful in this instance if MRI is unavailable. The key to identification is visualising a number of indirect signs, including:
    • CSF filled sulci do not reach the skull but rather fade out into the subdural
    • mass effect including sulcal effacement (distortion) and midline shift
    • apparent thickening of the cortex

    Chronic
    By definition it is at least 3 weeks old,
    The subdural collection becomes hypodense and can reach ~0HU and be isodense to CSF, and mimic subdural hygromas.
    Shape from crescentic may change to biconvex.

    Acute on chronic
    Acute on chronic subdural hematomas refers to a second episode of acute haemorrhage into a pre-existing chronic subdural hematoma. It typically appears as a hypodense collection with a haematocrit level (located posteriorly). A similar appearance can be seen in patients with clotting disorders or on anticoagulants 4.

    While trauma is the most common cause of SDH, atrophy, ruptured aneurysms, tumors, alcoholism, anticoagulants, seizure disorders, intracranial hypotension, CSF shunts, and intravenous drug use are also other etiologies. Many of these etiologies can lead to either acute or chronic SDHs. Though, atrophy of the brain is more commonly associated with chronic SDH, which is also associated with alcoholism.

    The prognosis of acute SDHs is variable with mortality ranging between 36-79%of cases. Only 40% of acute SDHs results in favorable outcomes. The reason for this fatal prognosis is due to its anatomy and pathophysiology. As veins exit the pia matter, they bridge into the dura membrane. Those bridging veins drain into the cerebral sinus and then drain into jugular vein. SDHs occur when those bridging veins tear. In the setting of acute trauma, the space between the dura and arachnoid fills up with blood very rapidly, which causes increased ICP and compression of the cortex.

    The rapid increase in ICP is the most common reason for mortality. It has been shown that acceleration injuries from assaults and falls cause more damage than energy absorption trauma as seen in dashboard injuries from MVAs. When initial insult leads to increases in ICP, the compression of arteries causes infarct in various parts of the brain. It also results in other brain injuries, like cerebral edema, which further increases ICP. If it is not treated, it can cause the brain to herniate downward. Downward herniation of the brain causes compression of major blood supplies, such as the Circle of Willis. Herniation of the brain cuts off blood supply to the brainstem, which results in death. Acute SDHs can also change the brain’s metabolism and cause secondary brain injury, according to Meagher. Tissue under the SDH does not respond to oxygen metabolism and switches to anaerobic metabolism, which increases the levels of lactate, which is toxic for the brain.

    In the setting of chronic SDHs, cerebral atrophy increases the tension on bridging veins and even with minor trauma those veins can rupture. Sometimes acute small SDHs fail to dissolve and form membranes surrounding it, which can turn into chronic expansion. An imbalance in the osmotic gradient can also cause fluid to shift into the subdural space. However, often, the cause of chronic SDHs is idiopathic.

    Most of the patients, about 50%, with acute SDHs are comatose due to other brain injuries. Neurologically intact will progressively deteriorate with time. Patients with chronic SDHs present with various neurological problems, such as insidious onset of headaches, light-headedness, apathy, dementia, hemiparesis, and ataxia, like in the case of NM. Symptoms related to increased ICP like nausea, vomiting, and changes in vision can also be present. Seizures can also be seen in patients with SDHs (McBride et al).

    Due to the high fatality rates, patients who present following traumatic injuries, with sudden onset of symptoms described above, or status post falls warrant a head CT scan. Head CT scans of acute SDHs show hyperdensed crescent lesions that cross the suture lines. Due to the nature of injuries, acute SDH may also present with IPHs and SAHs. Midline shifts can also be seen on head CT. Alternatively, head CTs of chronic SDHs is hypodense due to degraded blood and the liquefaction effect. But the shape of chronic SDHs is also crescent, as seen in acute. Brain MRIs are more sensitive in picking up small SDHs, however the availability and time-consuming process limit its use in the evaluation of traumatic SDHs (McBride et al).

    Acute (symptomatic) SDHs are medical emergencies, hence it requires surgical innervations. Such injuries need to be evaluated by neurosurgeons. According to McBride et al, surgical decision is made based on the GCS score, head CT, clot thickness, midline shift, neurologic exam, and comorbidities. If the patient is stable and the head CT shows a bleeding area

    References
    1. Kaye AH. Brain Tumors: An Encyclopedic Approach, 2nd, Churchill Livingstone, New York 2001. p.205
    2. McBride W, Biller J. Subdural hematoma in adults: Prognosis and management. Uptodate. https://www.uptodate.com/contents/subdural-hematoma-in-adults-prognosis-and-management?source=search_result&search=subdural%20hematoma&selectedTitle=1~150#H1
    3. Meagher RJ, Young VM. Subdural Hematoma. Subdural Hematoma. http://emedicine.medscape.com/article/1137207-overview#a3. Published January 6, 2017. Accessed May 21, 2017.
    4. Victor M, Ropper A. Craniocerebral trauma. In: Adams and Victor’s Principles of Neurology, 7th ed, Victor M, Ropper A. (Eds), McGraw-Hill, New York 2001. p.925

    Incorrect

    Correct Answer: B – subdural hematoma. A is incorrect because epidural hematomas do not cross suture lines and moreover epidural hematomas are “convex” (lens) in shape. C is incorrect because intraparenchymal hematomas do not follow suture lines, as IPHs occur in the brain tissue. Finally, answer D is incorrect since the patient did not complain of sudden onset of headache and subarachnoid hemorrhages normally present with sudden onset of “thunder-clap”, “worst headache of my life”. B is the most likely diagnosis because the CT scan showed a hematoma crossing suture lines and the hematoma was “crescent”-shaped, which is indicative of a subdural hematoma. However, in chronic SDH, the shape can change to biconvex.

    Subdural hematoma

    The brain is enclosed within a closed and fixed space and is surrounded by cerebrospinal fluid (CSF), which acts as a shock absorbent. The brain is surrounded by 3 layers: the pia is the innermost layer, the subarachnoid is the middle layer, and the dura is the outermost fibrous membrane – the extremely tough layer of the three meningeal layers. The skull encompasses all of these layers. Brains have fixed volume, in which 80% is brain tissue, 10% is CSF, and 10% is blood (Kaye et al). Increases in intracranial pressure (ICP) is fatal for brain function. Brain hemorrhage is the most likely cause of increased intracranial pressure (Kaye et al). There are two type of brain hemorrhage: intracranial, which includes intraventricular and intraparenchymal hemorrhages, and extracranial, which includes subarachnoid, subdural, and epidural hemorrhages.

    In subdural hematomas (SDH), bleeding occurs in-between the dura and subarachnoid membrane. SDH is further divided into subtypes based on time of injury. Acute is 1 week from injury. SDHs can also be mixed in nature, as acute injuries can turn into chronic SDHs. Acute SDHs are the most common type of traumatic intracranial hemorrhage. Acute SDHs can also present with other brain injuries such as subarachnoid hemorrhage, diffuse axonal injury, and so on. 72% of SDH occur following fall or assaults and 24% are associated with MVA. Chronic SDHs more commonly occur in the elderly population and are the most common type of treatable cause of dementia. Chronic SDHs are more common in males and the incidence increases with age. SDHs can also present in children. Bilateral presentation of SDHs in children is associated with underlying physiology, while interhemispheric presentation is associated with child abuse, also known as shaken baby syndrome (Meagher et al).

    Subacute
    As the clot ages and protein degradation occurs, the density starts to drop. At some point between 3 and 21 days (typically 10-14 days), the density will drop to ~ 30 HU and become isodense to the adjacent cortex, making identification potentially tricky, especially if subdural collections are bilateral 4. Contrast-enhanced CT is often useful in this instance if MRI is unavailable. The key to identification is visualising a number of indirect signs, including:
    • CSF filled sulci do not reach the skull but rather fade out into the subdural
    • mass effect including sulcal effacement (distortion) and midline shift
    • apparent thickening of the cortex

    Chronic
    By definition it is at least 3 weeks old,
    The subdural collection becomes hypodense and can reach ~0HU and be isodense to CSF, and mimic subdural hygromas.
    Shape from crescentic may change to biconvex.

    Acute on chronic
    Acute on chronic subdural hematomas refers to a second episode of acute haemorrhage into a pre-existing chronic subdural hematoma. It typically appears as a hypodense collection with a haematocrit level (located posteriorly). A similar appearance can be seen in patients with clotting disorders or on anticoagulants 4.

    While trauma is the most common cause of SDH, atrophy, ruptured aneurysms, tumors, alcoholism, anticoagulants, seizure disorders, intracranial hypotension, CSF shunts, and intravenous drug use are also other etiologies. Many of these etiologies can lead to either acute or chronic SDHs. Though, atrophy of the brain is more commonly associated with chronic SDH, which is also associated with alcoholism.

    The prognosis of acute SDHs is variable with mortality ranging between 36-79%of cases. Only 40% of acute SDHs results in favorable outcomes. The reason for this fatal prognosis is due to its anatomy and pathophysiology. As veins exit the pia matter, they bridge into the dura membrane. Those bridging veins drain into the cerebral sinus and then drain into jugular vein. SDHs occur when those bridging veins tear. In the setting of acute trauma, the space between the dura and arachnoid fills up with blood very rapidly, which causes increased ICP and compression of the cortex.

    The rapid increase in ICP is the most common reason for mortality. It has been shown that acceleration injuries from assaults and falls cause more damage than energy absorption trauma as seen in dashboard injuries from MVAs. When initial insult leads to increases in ICP, the compression of arteries causes infarct in various parts of the brain. It also results in other brain injuries, like cerebral edema, which further increases ICP. If it is not treated, it can cause the brain to herniate downward. Downward herniation of the brain causes compression of major blood supplies, such as the Circle of Willis. Herniation of the brain cuts off blood supply to the brainstem, which results in death. Acute SDHs can also change the brain’s metabolism and cause secondary brain injury, according to Meagher. Tissue under the SDH does not respond to oxygen metabolism and switches to anaerobic metabolism, which increases the levels of lactate, which is toxic for the brain.

    In the setting of chronic SDHs, cerebral atrophy increases the tension on bridging veins and even with minor trauma those veins can rupture. Sometimes acute small SDHs fail to dissolve and form membranes surrounding it, which can turn into chronic expansion. An imbalance in the osmotic gradient can also cause fluid to shift into the subdural space. However, often, the cause of chronic SDHs is idiopathic.

    Most of the patients, about 50%, with acute SDHs are comatose due to other brain injuries. Neurologically intact will progressively deteriorate with time. Patients with chronic SDHs present with various neurological problems, such as insidious onset of headaches, light-headedness, apathy, dementia, hemiparesis, and ataxia, like in the case of NM. Symptoms related to increased ICP like nausea, vomiting, and changes in vision can also be present. Seizures can also be seen in patients with SDHs (McBride et al).

    Due to the high fatality rates, patients who present following traumatic injuries, with sudden onset of symptoms described above, or status post falls warrant a head CT scan. Head CT scans of acute SDHs show hyperdensed crescent lesions that cross the suture lines. Due to the nature of injuries, acute SDH may also present with IPHs and SAHs. Midline shifts can also be seen on head CT. Alternatively, head CTs of chronic SDHs is hypodense due to degraded blood and the liquefaction effect. But the shape of chronic SDHs is also crescent, as seen in acute. Brain MRIs are more sensitive in picking up small SDHs, however the availability and time-consuming process limit its use in the evaluation of traumatic SDHs (McBride et al).

    Acute (symptomatic) SDHs are medical emergencies, hence it requires surgical innervations. Such injuries need to be evaluated by neurosurgeons. According to McBride et al, surgical decision is made based on the GCS score, head CT, clot thickness, midline shift, neurologic exam, and comorbidities. If the patient is stable and the head CT shows a bleeding area

    References
    1. Kaye AH. Brain Tumors: An Encyclopedic Approach, 2nd, Churchill Livingstone, New York 2001. p.205
    2. McBride W, Biller J. Subdural hematoma in adults: Prognosis and management. Uptodate. https://www.uptodate.com/contents/subdural-hematoma-in-adults-prognosis-and-management?source=search_result&search=subdural%20hematoma&selectedTitle=1~150#H1
    3. Meagher RJ, Young VM. Subdural Hematoma. Subdural Hematoma. http://emedicine.medscape.com/article/1137207-overview#a3. Published January 6, 2017. Accessed May 21, 2017.
    4. Victor M, Ropper A. Craniocerebral trauma. In: Adams and Victor’s Principles of Neurology, 7th ed, Victor M, Ropper A. (Eds), McGraw-Hill, New York 2001. p.925

  162. Question 162 of 220
    162. Question

    A mother brings her 1 year old son for hematologic evaluation due to laboratory results revealing asymptomatic, intermittent decreased hemoglobin. Pertinent medical history includes neonatal jaundice treated with phototherapy and cystic fibrosis with recurrent infections treated with Bactrim. His physical exam and CBC and peripheral smear are normal at today’s visit. What findings might be expected if a peripheral smear was obtained simultaneously with a CBC showing low hemoglobin?

    Correct

    Anemia, defined by decreased hemoglobin or red blood cells, can occur due to many etiologies. G6PD deficiency anemia is caused by a genetic mutation leading to quantitative and qualitative changes in the enzyme, which has a protective function against oxidation. This disorder may be asymptomatic with incidentally found intermittent anemia. If a patient presents clinically it is most commonly with jaundice in a newborn and episodes of hemolytic anemia later in life due to illness or exposure to foods and medications that increase oxidative stress. Most notably are fava beans and sulfa drugs (Bactrim).2 Peripheral smear is a helpful diagnostic tool that can be used in determining the cause of anemia. G6PD deficiency anemia is caused by acute hemolysis when red blood cells are exposed to oxidative stress which appear as bite cells with Heinz bodies (B) on peripheral smear. Microcytic hypochromic red blood cells (A) are characteristic of iron deficiency anemia which would not present with intermittent decrease in hemoglobin. All children born in the United States are screened for sickle cell anemia at birth, suggested by thin, elongated sickle shaped cells and Howell-Jolly Bodies (C & D) on peripheral smear. It is also unlikely that a peripheral smear and CBC would be normal at any point in a one year old patient. Megaloblastic RBCs and hyper-segmented neutrophils (E) are present in macrocytic anemias commonly caused by B12 deficiency and is associated with an insidious onset of gastrointestinal and neurologic symptoms.

    References
    1. Luzzatto L. Glucose 6-phosphate dehydrogenase deficiency: from genotype to phenotype. Haematologica/the hematology journal. 2006; 91(10):1303-1306.
    2. Luzzatto L, Nannelli C, & Notaro R. Glucose-6-phosphate dehydrogenase deficiency. Hematology/oncology clinics of North America. 2016;30(2):373-393.
    3. Luzzatto L, Seneca E. G6PD deficiency: a classic example of pharmacogenetics with on-going clinical implications. British Journal of Haematology. 2013;164(4):469-480. doi:10.1111/bjh.12665.

    Incorrect

    Anemia, defined by decreased hemoglobin or red blood cells, can occur due to many etiologies. G6PD deficiency anemia is caused by a genetic mutation leading to quantitative and qualitative changes in the enzyme, which has a protective function against oxidation. This disorder may be asymptomatic with incidentally found intermittent anemia. If a patient presents clinically it is most commonly with jaundice in a newborn and episodes of hemolytic anemia later in life due to illness or exposure to foods and medications that increase oxidative stress. Most notably are fava beans and sulfa drugs (Bactrim).2 Peripheral smear is a helpful diagnostic tool that can be used in determining the cause of anemia. G6PD deficiency anemia is caused by acute hemolysis when red blood cells are exposed to oxidative stress which appear as bite cells with Heinz bodies (B) on peripheral smear. Microcytic hypochromic red blood cells (A) are characteristic of iron deficiency anemia which would not present with intermittent decrease in hemoglobin. All children born in the United States are screened for sickle cell anemia at birth, suggested by thin, elongated sickle shaped cells and Howell-Jolly Bodies (C & D) on peripheral smear. It is also unlikely that a peripheral smear and CBC would be normal at any point in a one year old patient. Megaloblastic RBCs and hyper-segmented neutrophils (E) are present in macrocytic anemias commonly caused by B12 deficiency and is associated with an insidious onset of gastrointestinal and neurologic symptoms.

    References
    1. Luzzatto L. Glucose 6-phosphate dehydrogenase deficiency: from genotype to phenotype. Haematologica/the hematology journal. 2006; 91(10):1303-1306.
    2. Luzzatto L, Nannelli C, & Notaro R. Glucose-6-phosphate dehydrogenase deficiency. Hematology/oncology clinics of North America. 2016;30(2):373-393.
    3. Luzzatto L, Seneca E. G6PD deficiency: a classic example of pharmacogenetics with on-going clinical implications. British Journal of Haematology. 2013;164(4):469-480. doi:10.1111/bjh.12665.

  163. Question 163 of 220
    163. Question

    Figure 2
    An 82 year-old male presents to the office with right knee pain for several years. He has been seen in the past for right knee osteoarthritis and steroid injections have worked well. You plan on giving him another steroid injection but notice an incidental finding of a calcified mass in the back of the knee on lateral x-ray above. The lateral x-ray shows a 2.5 by 2 cm well circumscribed oval calcification adjacent to the popliteal artery and posterior to the distal femur. You order an MRI to differentiate between a soft tissue mass vs. a popliteal aneurysm. MRI confirms the diagnosis of a popliteal aneurysm. What is the best treatment option for this patient?

    Correct

    Popliteal artery aneurysm (PAA) is an abnormal dilation of the arterial wall that can cause life and limb threatening complications if the aneurysm burst or clots. Thromboembolic events are the most frequent complication (occurring in 42-77% of patients), which can lead to amputation in up to 20% of patients. PAA’s account for 70-80% of all peripheral aneurysms and occur bilaterally in 50% of patients. Although the exact etiology is unknown, the presences of arteriosclerosis is thought be the most significant contributor. Hypertension also adds mechanical stresses to the artery which can play a role in aneurysm formation. Popliteal aneurysms will naturally expand over time, on average about 10% a year. Large PAA’s expand faster than smaller ones. PAA’s are often asymptomatic but as they enlarge can cause pain, swelling, pulsation, and distal edema and venous thrombosis (if the aneurysm compresses the popliteal vein). The rate of complications is greatest with PAA’s greater than 2 cm and increases with time (24% rate in 1 year, 35% rate in 4 years, and 74% in at 5 years). PAA’s that are symptomatic and greater than 2 cm should be treated surgically. Anticoagulants have no effect on the growth of PAA’s and ischemia complications cannot be prevented by observation. Current surgical options include open vs. endovascular repair, with similar success rates between the two. 1, 2

    References
    1. Ghotbi R, Sotiriou A, Schönhofer S, Zikos D, Schips K, Westermeier W. Stent-Graft Placement in Popliteal Artery Aneurysms: Midterm Results. Vascular Disease Management. E-edition. 9/5/08
    2. Dawson I, Brand R. Popliteal artery aneruysm. Longer term follow-up of aneurysmal disease and results of surgical treatment. Journal of Vascular Surgery 1991. Volume 13 (3). 398-407.

    Incorrect

    Popliteal artery aneurysm (PAA) is an abnormal dilation of the arterial wall that can cause life and limb threatening complications if the aneurysm burst or clots. Thromboembolic events are the most frequent complication (occurring in 42-77% of patients), which can lead to amputation in up to 20% of patients. PAA’s account for 70-80% of all peripheral aneurysms and occur bilaterally in 50% of patients. Although the exact etiology is unknown, the presences of arteriosclerosis is thought be the most significant contributor. Hypertension also adds mechanical stresses to the artery which can play a role in aneurysm formation. Popliteal aneurysms will naturally expand over time, on average about 10% a year. Large PAA’s expand faster than smaller ones. PAA’s are often asymptomatic but as they enlarge can cause pain, swelling, pulsation, and distal edema and venous thrombosis (if the aneurysm compresses the popliteal vein). The rate of complications is greatest with PAA’s greater than 2 cm and increases with time (24% rate in 1 year, 35% rate in 4 years, and 74% in at 5 years). PAA’s that are symptomatic and greater than 2 cm should be treated surgically. Anticoagulants have no effect on the growth of PAA’s and ischemia complications cannot be prevented by observation. Current surgical options include open vs. endovascular repair, with similar success rates between the two. 1, 2

    References
    1. Ghotbi R, Sotiriou A, Schönhofer S, Zikos D, Schips K, Westermeier W. Stent-Graft Placement in Popliteal Artery Aneurysms: Midterm Results. Vascular Disease Management. E-edition. 9/5/08
    2. Dawson I, Brand R. Popliteal artery aneruysm. Longer term follow-up of aneurysmal disease and results of surgical treatment. Journal of Vascular Surgery 1991. Volume 13 (3). 398-407.

  164. Question 164 of 220
    164. Question

    Figure 1
    A 46 year-old female presents to your office with left foot pain for 3 weeks. The pain is located over the lateral midfoot and she has tenderness to palpation over an apparent ossicle adjacent to the lateral cuboid (seen in an oblique x-ray of the foot above. What is the name of this ossicle?

    Correct

    Accessory ossicles are boney structures (normal congenital variant) that separate from adjacent bone and are usually embedded within tendons. Over 40 different types of accessory ossicles have been known to occur in the foot. The most common types of ossicles in the foot include an os trigonum, os peroneum, and accessory navicular. It is important to have a general knowledge of the common accessory ossicles of the foot as they are often found incidentally on x-ray and can be confused with avulsion fractures. Accessory ossicles can also cause painful syndromes that are usually associated with tendinitis of the surrounding tendon. 1

    Os peroneum is an ossicle located just lateral to the cuboid and is embedded within the peroneus brevis tendon. Several sources of os peroneum pathology can cause pain in the peroneus brevis tendon including fracture of the ossicle, rupture or tearing of the tendon around the ossicle, and stenosing tenosynovitis. Os peroneum syndrome generally causes symptoms of pain over the lateral cuboid and pain during palpation of the ossicle. Treatment of os peroneum syndrome is the same as for peroneal tendinitis, which includes a period of NSAIDS and immobilization. 1,2
    Os trigonum is located posterior to the talus and may be a source of pain due to posterior ankle impingement. Os versalianum is located proximal to the base of the 5th metatarsal and is often confused with an avulsion fracture of the tuberosity. Os naviculare, or accessory navicular, is a large accessory ossicle adjacent to the medial side of the navicular. 1

    References
    1. Nwawka OK, Hayashi D, Diaz LE, Goud AR, Arndt WF, Roemer FW, Malguria N, Guermazi A. Sesamoids and accessory ossicles of the foot: anatomical variability and related pathology. Insights Imaging. 2013 Oct; 4(5): 581–593.
    2. Heckman DS, Reddy S, Pedowitz D, Wapner KL, Parekh SG. Current Concepts Review: Operative Treatment for Peroneal Tendon Disorders. JBJS 2008; 90: 404-418

    Incorrect

    Accessory ossicles are boney structures (normal congenital variant) that separate from adjacent bone and are usually embedded within tendons. Over 40 different types of accessory ossicles have been known to occur in the foot. The most common types of ossicles in the foot include an os trigonum, os peroneum, and accessory navicular. It is important to have a general knowledge of the common accessory ossicles of the foot as they are often found incidentally on x-ray and can be confused with avulsion fractures. Accessory ossicles can also cause painful syndromes that are usually associated with tendinitis of the surrounding tendon. 1

    Os peroneum is an ossicle located just lateral to the cuboid and is embedded within the peroneus brevis tendon. Several sources of os peroneum pathology can cause pain in the peroneus brevis tendon including fracture of the ossicle, rupture or tearing of the tendon around the ossicle, and stenosing tenosynovitis. Os peroneum syndrome generally causes symptoms of pain over the lateral cuboid and pain during palpation of the ossicle. Treatment of os peroneum syndrome is the same as for peroneal tendinitis, which includes a period of NSAIDS and immobilization. 1,2
    Os trigonum is located posterior to the talus and may be a source of pain due to posterior ankle impingement. Os versalianum is located proximal to the base of the 5th metatarsal and is often confused with an avulsion fracture of the tuberosity. Os naviculare, or accessory navicular, is a large accessory ossicle adjacent to the medial side of the navicular. 1

    References
    1. Nwawka OK, Hayashi D, Diaz LE, Goud AR, Arndt WF, Roemer FW, Malguria N, Guermazi A. Sesamoids and accessory ossicles of the foot: anatomical variability and related pathology. Insights Imaging. 2013 Oct; 4(5): 581–593.
    2. Heckman DS, Reddy S, Pedowitz D, Wapner KL, Parekh SG. Current Concepts Review: Operative Treatment for Peroneal Tendon Disorders. JBJS 2008; 90: 404-418

  165. Question 165 of 220
    165. Question

    A 65 year-old patient presents to your office with a painful erythematous macular rash all over her body for the past 2 weeks. Patient states she has been recently hospitalized for the flu, fever of 102.4, dehydration, UTI, and diminished kidney function. The patient states she is currently prescribed Allopurinol for her Gout that she began 4 weeks ago. Blood work was ordered which revealed a decrease in lymphocytes, increased creatinine, and lowered GFR. Patient denies use of new lotions, soaps, or detergents. What is the most likely diagnosis?

    Correct

    The correct answer is C (Drug Reaction with Eosinophilia and Systemic Symptoms (DRESS) Syndrome). Allopurinol is a common drug that causes DRESS syndrome.

    DRESS Syndrome:

    DRESS Syndrome is an acronym for Drug Reaction with Eosinophilia and Systemic Symptoms. It is a drug-induced condition, which is characterized by a widespread rash, lymphadenopathy, fever, hematologic abnormalities, involvement of the kidneys, lungs, pancreas, or heart, and hepatitis. It is an idiosyncratic adverse effect from particular drugs and could be fatal. Medications including anticonvulsants, antibiotics (particularly beta-lactams), and allopurinol are a cause of DRESS. Other medications, but less commonly cause DRESS syndrome include non-steroidal anti-inflammatory drugs, captopril, mood stabilizers, and antiretroviral. This syndrome arises from 1 in 1,000 to 1 in 10,000 drug exposures and generally occurs with initial exposure to a drug. The risk of DRESS induced by allopurinol is increased in patients who have renal impairment and use of thiazide diuretics. The mortality rate for DRESS is 10-20% with most deaths resulting from liver failure. Genetics can also be a factor in this syndrome. Patients have a risk as high as 25% if they have a first degree relative who has experienced this syndrome.

    The pathogenesis of DRESS syndrome is thought to be multifactorial and include both nonimmunologic and immunologic factors, but not fully understood yet. It is considered an idiosyncratic reaction with three potential factors that are identified as a factor: 1. reactivation of human herpesvirus 6 (HHV-6), human herpesvirus 7 (HHV-7), Epstein-Barr virus (EBV), or cytomegalovirus (CMV), 2. a genetic susceptibility that alters the immune response, 3. a defect in drug metabolism causing the failure to eliminate toxic reactive intermediates.

    The hallmarks of DRESS syndrome include a long latency period between initiation of the provoking medication and onset of the reaction (2-3 weeks), which consists of fever, rash and at least one involvement of internal organ system. Rash and Fever are the most common clinical manifestations and occur in 75% and 85% of cases respectively. Central facial erythema and edema are common, which can serve as an important clue when it comes to the diagnosis. Other common findings include leukocytosis, abnormal liver function tests, generalized lymphadenopathy, and peripheral eosinophilia. The most common organ involved is the liver, and fulminant hepatitis is major cause of deaths due to DRESS. Cutaneous and visceral inflammation may remain for weeks to months after the drug is discontinued.

    DRESS syndrome can be difficult to diagnose because of the variable presentations of the syndrome. The European Registry of Severe Cutaneous Adverse Reaction to Drugs and Collection of Biological Samples (RegiSCAR) has produced diagnostic criteria to aid in the diagnosis of DRESS. The diagnostic criteria include three requirements (acute rash, hospitalization, and suspicion of a drug related reaction) PLUS at least three of the four systemic features: 1. Fever > 38C, 2. lymphadenopathy involving at least two sites, 3. involvement of at least one internal organ (heart, liver, kidney, etc.) and 4. Hematologic abnormalities, including lymphocyte count above or below the normal limits and eosinophil count higher than normal limits or platelet count below normal limits.

    Treatment of DRESS syndrome includes immediate discontinuation of the drug that is prompting the reaction. Supportive care is recommended for patients. Systemic corticosteroids are used to treat DRESS as first line therapy. Patients are tapered over a prolonged period of time. Systemic steroids are continued for several months to avoid flare. Intravenous immunoglobulin has also been used and reported being effected, but there is no data to support this claim.

    References:

    Buck ML. DRESS Syndrome. Medscape. http://www.medscape.com/viewarticle/776164_1. Published 2012. Accessed June 1, 2017.

    DRESS Syndrome: remember to look under the skin. Medsafe. http://www.medsafe.govt.nz/profs/PUArticles/DRESSsyndromeJune2011.htm. Published June 2011. Accessed June 1, 2017.

    Khetarpal S, Fernandez A. Dermatological Emergencies. Disease Management. http://www.clevelandclinicmeded.com/medicalpubs/diseasemanagement/dermatology/dermatological-emergencies/. Published August 2014. Accessed June 1, 2017.

    Incorrect

    The correct answer is C (Drug Reaction with Eosinophilia and Systemic Symptoms (DRESS) Syndrome). Allopurinol is a common drug that causes DRESS syndrome.

    DRESS Syndrome:

    DRESS Syndrome is an acronym for Drug Reaction with Eosinophilia and Systemic Symptoms. It is a drug-induced condition, which is characterized by a widespread rash, lymphadenopathy, fever, hematologic abnormalities, involvement of the kidneys, lungs, pancreas, or heart, and hepatitis. It is an idiosyncratic adverse effect from particular drugs and could be fatal. Medications including anticonvulsants, antibiotics (particularly beta-lactams), and allopurinol are a cause of DRESS. Other medications, but less commonly cause DRESS syndrome include non-steroidal anti-inflammatory drugs, captopril, mood stabilizers, and antiretroviral. This syndrome arises from 1 in 1,000 to 1 in 10,000 drug exposures and generally occurs with initial exposure to a drug. The risk of DRESS induced by allopurinol is increased in patients who have renal impairment and use of thiazide diuretics. The mortality rate for DRESS is 10-20% with most deaths resulting from liver failure. Genetics can also be a factor in this syndrome. Patients have a risk as high as 25% if they have a first degree relative who has experienced this syndrome.

    The pathogenesis of DRESS syndrome is thought to be multifactorial and include both nonimmunologic and immunologic factors, but not fully understood yet. It is considered an idiosyncratic reaction with three potential factors that are identified as a factor: 1. reactivation of human herpesvirus 6 (HHV-6), human herpesvirus 7 (HHV-7), Epstein-Barr virus (EBV), or cytomegalovirus (CMV), 2. a genetic susceptibility that alters the immune response, 3. a defect in drug metabolism causing the failure to eliminate toxic reactive intermediates.

    The hallmarks of DRESS syndrome include a long latency period between initiation of the provoking medication and onset of the reaction (2-3 weeks), which consists of fever, rash and at least one involvement of internal organ system. Rash and Fever are the most common clinical manifestations and occur in 75% and 85% of cases respectively. Central facial erythema and edema are common, which can serve as an important clue when it comes to the diagnosis. Other common findings include leukocytosis, abnormal liver function tests, generalized lymphadenopathy, and peripheral eosinophilia. The most common organ involved is the liver, and fulminant hepatitis is major cause of deaths due to DRESS. Cutaneous and visceral inflammation may remain for weeks to months after the drug is discontinued.

    DRESS syndrome can be difficult to diagnose because of the variable presentations of the syndrome. The European Registry of Severe Cutaneous Adverse Reaction to Drugs and Collection of Biological Samples (RegiSCAR) has produced diagnostic criteria to aid in the diagnosis of DRESS. The diagnostic criteria include three requirements (acute rash, hospitalization, and suspicion of a drug related reaction) PLUS at least three of the four systemic features: 1. Fever > 38C, 2. lymphadenopathy involving at least two sites, 3. involvement of at least one internal organ (heart, liver, kidney, etc.) and 4. Hematologic abnormalities, including lymphocyte count above or below the normal limits and eosinophil count higher than normal limits or platelet count below normal limits.

    Treatment of DRESS syndrome includes immediate discontinuation of the drug that is prompting the reaction. Supportive care is recommended for patients. Systemic corticosteroids are used to treat DRESS as first line therapy. Patients are tapered over a prolonged period of time. Systemic steroids are continued for several months to avoid flare. Intravenous immunoglobulin has also been used and reported being effected, but there is no data to support this claim.

    References:

    Buck ML. DRESS Syndrome. Medscape. http://www.medscape.com/viewarticle/776164_1. Published 2012. Accessed June 1, 2017.

    DRESS Syndrome: remember to look under the skin. Medsafe. http://www.medsafe.govt.nz/profs/PUArticles/DRESSsyndromeJune2011.htm. Published June 2011. Accessed June 1, 2017.

    Khetarpal S, Fernandez A. Dermatological Emergencies. Disease Management. http://www.clevelandclinicmeded.com/medicalpubs/diseasemanagement/dermatology/dermatological-emergencies/. Published August 2014. Accessed June 1, 2017.

  166. Question 166 of 220
    166. Question

    A 48 year-old female presents to the office with a gradual, progressive increase of joint pain and swelling of the PIP joints bilaterally as well as bilateral wrists. She states she has morning stiffness lasting greater than 1 hour as well as low grade fevers, fatigue, and 10lb weight loss without dieting over the last month. Labs were completed prior to the office visit and she had a positive rheumatoid factor, positive anti CCP antibody, negative ANA and negative SS-A and SS-B antibodies. A physical exam was performed and ulnar deviation was present bilaterally as well as bilateral subluxation of the MCP joints. She denies any recent infections, nausea, vomiting, dry eyes, dry mouth and rashes or change in skin. What would the next appropriate step in treatment be with her assumed diagnosis?

    Correct

    The correct answer is A methotrexate. The patient described above should be diagnosed with rheumatoid arthritis. She has a positive RF and CCP antibody which would indicate seropositive RA. She also has morning stiffness lasting greater than one hour which is a hallmark sign. She also has systemic symptoms such as low grade fevers, weight loss and fatigue. It is also symmetric swelling affecting the PIP joints and wrists which are common areas that RA targets. She also has ulnar deviation indicating the RA is aggressive and has started damaging her joints. The first line treatment for moderate to severe rheumatoid arthritis is methotrexate. Hydroxychloroquine is first line treatment for systemic lupus erythematous and mild treatment for rheumatoid arthritis. Since her ANA was negative lupus can be ruled out. Pilocarpine can be used to help treat the symptoms of Sjogrens. This patient doesn’t have sjogrens because her SS-A and SS-B were negative and she didn’t have symptoms such as dry eyes or dry mouth. NSAIDS can be used to treat the pain and inflammation in osteoarthritis. You wouldn’t use NSAIDS to treat rheumatoid arthritis since it’s an autoimmune disorder. She doesn’t have osteoarthritis because it usually spares the wrists and MCP joints .Also joint pain is relieved with rest and morning stiffness lasting more than one hour is usually not characteristic of osteoarthritis. Also osteoarthritis is usually not associated with systemic symptoms such as fever, weight loss and fatigue.

    Discussion
    Rheumatoid arthritis is an autoimmune disorder that precipitates systemic inflammation within the body. This disease targets joints in a progressive and symmetrical fashion and causes significant destruction and damage to cartilage and bone that leads to detrimental and lasting joint effects.
    Although RA affects both males and females it tends to affect females at a significantly greater rate. Females are 2.5 times more likely to develop RA than males. The peak incidence usually occurs between the fourth and fifth decades of life. The average annual incidence in the US is 0.5 per 1000 persons per year.1

    Genetics significantly contribute to the development of RA and are roughly 50% the risk. 3 Chances of developing RA grossly increase with family history. The risk of developing the disease in first degree relatives of RA patients is 1.5 times higher than the general population. The PTPN22 gene is thought to play a significant role in developing RA. There is a fourfold higher risk in homozygotes that carry the polymorphism and a twofold higher chance in heterozygotes to develop RA. This polymorphism is also thought to be involved with type I diabetes and SLE. Sex and hormones are also thought to contribute to RA. Women are 2-3 times more likely to develop RA than men1. Hormones more specifically estrogen is thought to play a significant and detrimental role in the development of RA. For example, many times during pregnancy RA flares will dramatically decrease and the mother will go into remission while pregnant. Shortly after giving birth usually a few weeks the mother will have a flare. Also women on oral contraceptives have a decreased incidence of RA which is why hormones are thought to contribute in some aspect to RA3. Many other factors are also thought to contribute to RA including environmental .Tobacco can increase the risk of developing RA. Cigarette smoke can lead to a low positive CCP and also activate the innate immune system as well as cause inflammation within the body. Bacteria and viruses also play a role in the etiology of RA. They stimulate the innate immune system which eventually leads to stimulation of the adaptive immune system. Two of the main culprits are thought to be EBV and parvovirus B-19.EBV stimulates B lymphocytes and increases production of RF and Parvovirus B-19 is commonly found in RA joints and leads to infected fibroblast-like synoviocytes1.

    The pathophysiology of RA is very complicated and is thought to be a combination of cellular and molecular pathways. This combination inherently precipitates joint degeneration and destruction especially effecting joint space. Innate and adaptive immunity are hypothesized to play a significant role in RA1. Many different types of immune factors play a significant role in RA. These include CD4 Tells, mononuclear phagocytes, fibroblasts, osteoclasts, and neutrophils .There is also abnormal proliferation of cytokines, chemokines such as TNF-a, IL-1, IL-6, IL-8, transforming growth factor beta, fibroblast growth factor and platelet derived growth factor in the development of rheumatoid arthritis3. Many medications used in rheumatoid arthritis work by targeting and inhibiting these various agents1.

    In order to make an accurate diagnosis of rheumatoid arthritis it is important to gather a pertinent history and perform a focused physical exam. It is important to focus on articular symptoms, gradual or rapid onset of symptoms and perform a careful examination of which joints are affected. It’s also important to determine if the patient has morning stiffness lasting greater than one hour since this is a classic sign of rheumatoid arthritis. Since RA is a systemic disease patients may have fever, weight loss and fatigue. Joint pain and swelling is gradual and usually happens over weeks to months. The most frequent joints affected are the wrists, metacarpophalangeal joints, proximal interphalangeal joints and metatarsal phalangeal joints. As rheumatoid arthritis evolves the ankles, knees elbows and shoulders become involved. However, DIP joints and the thoracolumbar spine are usually not affected. Rheumatoid arthritis usually affects Joints symmetrically and morning stiffness lasting greater than one hour is a hallmark sign to be aware of and screen for in the patient. More often than not articular manifestations are present on physical exam. Joint swelling usually stays within the joint capsule and fusiform swelling of the PIP joint is one of the earliest findings. Hand deformities occur as the disease progresses, becomes more aggressive or is left untreated or treated incorrectly. Joint deformities can include ulnar deviation of the fingers, dorsal subluxation of the MCP joints, swan neck deformity and boutonniere deformity at the PIP. There may be also swelling of the elbows and wrists. Rheumatoid nodules may also be present and can occur over the elbows, achilles tendons, fingers, scalp and ischial tuberosities1. Extra articular manifestations can also occur and need to be monitored and checked during a physical exam. These can include rheumatoid nodules, interstitial lung disease, pleural effusion, pericarditis, splenomegaly with leukopenia, vasculitis and sjogrens syndrome. Sjogrens syndrome will manifest as dry eyes, mouth and other mucous membranes2.

    Diagnostic imaging and labs can be very helpful in diagnosing and determining the severity of rheumatoid arthritis. Laboratory findings can be useful in determining the amount of systemic inflammation going on within the body. Laboratory tests that may come back as being elevated or abnormal are inflammatory joint fluid, anemia of chronic disease, rheumatoid factor and anti CCP. Roughly about 75-80% of patients with RA will test positive for anti CCP and RF factor. This is known as seropositive RA .Elevations in CRP and ESR, which are inflammatory markers can correlate with disease activity and be an indication that active inflammation is present within the body. Imaging can also be helpful in determining joint destruction and progression of the disease. Plain film radiography will show progressive changes in the joints such as joint space narrowing, osteopenia and bone erosions. An MRI can be used to detect joint erosions and changes in joints and is more sensitive than radiography and can detect changes earlier than plain radiography. Finally, ultrasonography may also be used to detect inflammation in the joint but other methods will usually be utilized before using this method. 4

    Once diagnosed with rheumatoid arthritis, treatment should be initiated as quickly as possible to decrease the risk of developing lasting and devastating destruction of the joints. Rheumatoid arthritis should be categorized as mild, moderate or severe depending on patient presentation and lab values. Treatment should then be tailored to the degree of the disease. NSAIDS and COX-2 inhibitors are used to help relieve joint pain. Inflammation present within the body can try to be lowered by corticosteroids such as prednisone in 5mg-10mg doses. Corticosteroids can decrease systemic inflammation and slow the rate of joint destruction2. Sulfasalazine and hydroxychloroquine can be used for mild forms of RA. However, the backbone of treatment is the use of DMARDS. They are used to decrease the signs and symptoms of RA and to stop the progression of joint damage. Methotrexate is the first line treatment due to its proven benefits in slowing the progression of RA damage. Methotrexate can also be combined with other biologics such as humira or Enbrel if flares, systemic inflammation and other symptoms are present. Methotrexate has been shown to have a greater effect on reducing systemic inflammation when combined with two or three other biologics and is more effective than methotrexate alone in preventing joint damage1.

    References:
    1.Klippel JH. Primer on the rheumatic diseases. New York, NY: Springer; 2008.
    2.Papadakis MA, McPhee SJ, Rabow MW. Current medical diagnosis & treatment 2015. New York: McGraw-Hill Education; 2015.
    3.Rheumatoid Arthritis. Practice Essentials, Background, Pathophysiology. http://emedicine.medscape.com/article/331715-overview#a2. Published May 5, 2017. Accessed May 17, 2017.
    4. Venables PJW, Maini R. Clinical manifestations of rheumatoid arthritis. Clinical manifestations of rheumatoid arthritis. https://www.uptodate.com/contents/clinical-manifestations-of-rheumatoid-arthritis?source=search_result&search=rheumatoid arthritis&selectedTitle=2~150. Accessed May 17, 2017.

    Incorrect

    The correct answer is A methotrexate. The patient described above should be diagnosed with rheumatoid arthritis. She has a positive RF and CCP antibody which would indicate seropositive RA. She also has morning stiffness lasting greater than one hour which is a hallmark sign. She also has systemic symptoms such as low grade fevers, weight loss and fatigue. It is also symmetric swelling affecting the PIP joints and wrists which are common areas that RA targets. She also has ulnar deviation indicating the RA is aggressive and has started damaging her joints. The first line treatment for moderate to severe rheumatoid arthritis is methotrexate. Hydroxychloroquine is first line treatment for systemic lupus erythematous and mild treatment for rheumatoid arthritis. Since her ANA was negative lupus can be ruled out. Pilocarpine can be used to help treat the symptoms of Sjogrens. This patient doesn’t have sjogrens because her SS-A and SS-B were negative and she didn’t have symptoms such as dry eyes or dry mouth. NSAIDS can be used to treat the pain and inflammation in osteoarthritis. You wouldn’t use NSAIDS to treat rheumatoid arthritis since it’s an autoimmune disorder. She doesn’t have osteoarthritis because it usually spares the wrists and MCP joints .Also joint pain is relieved with rest and morning stiffness lasting more than one hour is usually not characteristic of osteoarthritis. Also osteoarthritis is usually not associated with systemic symptoms such as fever, weight loss and fatigue.

    Discussion
    Rheumatoid arthritis is an autoimmune disorder that precipitates systemic inflammation within the body. This disease targets joints in a progressive and symmetrical fashion and causes significant destruction and damage to cartilage and bone that leads to detrimental and lasting joint effects.
    Although RA affects both males and females it tends to affect females at a significantly greater rate. Females are 2.5 times more likely to develop RA than males. The peak incidence usually occurs between the fourth and fifth decades of life. The average annual incidence in the US is 0.5 per 1000 persons per year.1

    Genetics significantly contribute to the development of RA and are roughly 50% the risk. 3 Chances of developing RA grossly increase with family history. The risk of developing the disease in first degree relatives of RA patients is 1.5 times higher than the general population. The PTPN22 gene is thought to play a significant role in developing RA. There is a fourfold higher risk in homozygotes that carry the polymorphism and a twofold higher chance in heterozygotes to develop RA. This polymorphism is also thought to be involved with type I diabetes and SLE. Sex and hormones are also thought to contribute to RA. Women are 2-3 times more likely to develop RA than men1. Hormones more specifically estrogen is thought to play a significant and detrimental role in the development of RA. For example, many times during pregnancy RA flares will dramatically decrease and the mother will go into remission while pregnant. Shortly after giving birth usually a few weeks the mother will have a flare. Also women on oral contraceptives have a decreased incidence of RA which is why hormones are thought to contribute in some aspect to RA3. Many other factors are also thought to contribute to RA including environmental .Tobacco can increase the risk of developing RA. Cigarette smoke can lead to a low positive CCP and also activate the innate immune system as well as cause inflammation within the body. Bacteria and viruses also play a role in the etiology of RA. They stimulate the innate immune system which eventually leads to stimulation of the adaptive immune system. Two of the main culprits are thought to be EBV and parvovirus B-19.EBV stimulates B lymphocytes and increases production of RF and Parvovirus B-19 is commonly found in RA joints and leads to infected fibroblast-like synoviocytes1.

    The pathophysiology of RA is very complicated and is thought to be a combination of cellular and molecular pathways. This combination inherently precipitates joint degeneration and destruction especially effecting joint space. Innate and adaptive immunity are hypothesized to play a significant role in RA1. Many different types of immune factors play a significant role in RA. These include CD4 Tells, mononuclear phagocytes, fibroblasts, osteoclasts, and neutrophils .There is also abnormal proliferation of cytokines, chemokines such as TNF-a, IL-1, IL-6, IL-8, transforming growth factor beta, fibroblast growth factor and platelet derived growth factor in the development of rheumatoid arthritis3. Many medications used in rheumatoid arthritis work by targeting and inhibiting these various agents1.

    In order to make an accurate diagnosis of rheumatoid arthritis it is important to gather a pertinent history and perform a focused physical exam. It is important to focus on articular symptoms, gradual or rapid onset of symptoms and perform a careful examination of which joints are affected. It’s also important to determine if the patient has morning stiffness lasting greater than one hour since this is a classic sign of rheumatoid arthritis. Since RA is a systemic disease patients may have fever, weight loss and fatigue. Joint pain and swelling is gradual and usually happens over weeks to months. The most frequent joints affected are the wrists, metacarpophalangeal joints, proximal interphalangeal joints and metatarsal phalangeal joints. As rheumatoid arthritis evolves the ankles, knees elbows and shoulders become involved. However, DIP joints and the thoracolumbar spine are usually not affected. Rheumatoid arthritis usually affects Joints symmetrically and morning stiffness lasting greater than one hour is a hallmark sign to be aware of and screen for in the patient. More often than not articular manifestations are present on physical exam. Joint swelling usually stays within the joint capsule and fusiform swelling of the PIP joint is one of the earliest findings. Hand deformities occur as the disease progresses, becomes more aggressive or is left untreated or treated incorrectly. Joint deformities can include ulnar deviation of the fingers, dorsal subluxation of the MCP joints, swan neck deformity and boutonniere deformity at the PIP. There may be also swelling of the elbows and wrists. Rheumatoid nodules may also be present and can occur over the elbows, achilles tendons, fingers, scalp and ischial tuberosities1. Extra articular manifestations can also occur and need to be monitored and checked during a physical exam. These can include rheumatoid nodules, interstitial lung disease, pleural effusion, pericarditis, splenomegaly with leukopenia, vasculitis and sjogrens syndrome. Sjogrens syndrome will manifest as dry eyes, mouth and other mucous membranes2.

    Diagnostic imaging and labs can be very helpful in diagnosing and determining the severity of rheumatoid arthritis. Laboratory findings can be useful in determining the amount of systemic inflammation going on within the body. Laboratory tests that may come back as being elevated or abnormal are inflammatory joint fluid, anemia of chronic disease, rheumatoid factor and anti CCP. Roughly about 75-80% of patients with RA will test positive for anti CCP and RF factor. This is known as seropositive RA .Elevations in CRP and ESR, which are inflammatory markers can correlate with disease activity and be an indication that active inflammation is present within the body. Imaging can also be helpful in determining joint destruction and progression of the disease. Plain film radiography will show progressive changes in the joints such as joint space narrowing, osteopenia and bone erosions. An MRI can be used to detect joint erosions and changes in joints and is more sensitive than radiography and can detect changes earlier than plain radiography. Finally, ultrasonography may also be used to detect inflammation in the joint but other methods will usually be utilized before using this method. 4

    Once diagnosed with rheumatoid arthritis, treatment should be initiated as quickly as possible to decrease the risk of developing lasting and devastating destruction of the joints. Rheumatoid arthritis should be categorized as mild, moderate or severe depending on patient presentation and lab values. Treatment should then be tailored to the degree of the disease. NSAIDS and COX-2 inhibitors are used to help relieve joint pain. Inflammation present within the body can try to be lowered by corticosteroids such as prednisone in 5mg-10mg doses. Corticosteroids can decrease systemic inflammation and slow the rate of joint destruction2. Sulfasalazine and hydroxychloroquine can be used for mild forms of RA. However, the backbone of treatment is the use of DMARDS. They are used to decrease the signs and symptoms of RA and to stop the progression of joint damage. Methotrexate is the first line treatment due to its proven benefits in slowing the progression of RA damage. Methotrexate can also be combined with other biologics such as humira or Enbrel if flares, systemic inflammation and other symptoms are present. Methotrexate has been shown to have a greater effect on reducing systemic inflammation when combined with two or three other biologics and is more effective than methotrexate alone in preventing joint damage1.

    References:
    1.Klippel JH. Primer on the rheumatic diseases. New York, NY: Springer; 2008.
    2.Papadakis MA, McPhee SJ, Rabow MW. Current medical diagnosis & treatment 2015. New York: McGraw-Hill Education; 2015.
    3.Rheumatoid Arthritis. Practice Essentials, Background, Pathophysiology. http://emedicine.medscape.com/article/331715-overview#a2. Published May 5, 2017. Accessed May 17, 2017.
    4. Venables PJW, Maini R. Clinical manifestations of rheumatoid arthritis. Clinical manifestations of rheumatoid arthritis. https://www.uptodate.com/contents/clinical-manifestations-of-rheumatoid-arthritis?source=search_result&search=rheumatoid arthritis&selectedTitle=2~150. Accessed May 17, 2017.

  167. Question 167 of 220
    167. Question

    A 67 year-old female with no personal or family history of skin cancer presents for her annual full body skin exam. On exam you notice a solitary well circumscribed 3 mm dark brown circular lesion on her left shoulder. During palpation you note that the lesion has raised borders equally throughout the lesion. When you ask her about it she states that it has been there since she was little and has never changed. She denies any fevers, chills or recent unexplained weight loss. What would the next step in the management of this lesion?

    Correct

    A: This lesion is consistent with a typical benign nevus and does not need to be biopsied at this time. B: This is the correct answer. This lesion is consistent with a typical nevus due to its symmetric shape, even border, small size, no variation in color and no evolution over time. This lesion should be observed for any changes or signs of melanoma. C: This lesion is not consistent with melanoma due to its symmetry.

    Discussion:
    Atypical or dysplastic nevi are moles that look unusual and can sometimes look like melanoma. They are benign lesions, but people who have dysplastic nevi are at an increased risk of developing melanoma. This risk of melanoma can be in the dysplastic mole itself or in another area on the body. The greater number of dysplastic moles someone has, the higher their risk of developing melanoma. A person who has more than ten dysplastic moles has a twelve times greater risk of developing melanoma.

    Dysplastic moles tend to have a genetic component and can be seen in multiple family members. These types of moles are most commonly found in Caucasians. Individuals who have dysplastic moles and a family history of melanoma are at a very high risk.

    If a physician identifies a mole as atypical or if there is a patient who develops a new mole after age 40, the lesion should be biopsied. Those with atypical moles should monitor them and seek medical attention with any changes or signs of melanoma. When looking for signs of melanoma we can use the ABCs. A is for asymmetry, B is for irregular border, C is for multi-colored, D is for diameter greater than 6mm and E is for evolution or a spot that is changing.

    The pathophysiology behind dysplastic nevi is either inherited or sporadic. UV light can play a role in the development of dysplastic nevi and can be both an initiator and promotor in the transformation of the melanocytes into atypical melanocytes or melanoma.

    A person presenting with a dysplastic nevus may have a personal and/or family history of skin cancer, particularly melanoma. They may have a personal history of increased sun exposure, a recent or rapid change of an existing nevus or the development of a new nevus.

    On physical exam the patient may have a large pigmented lesion 5-15 mm in diameter. The lesion may be asymmetric and contain an irregular, notched or ill-defined border. There may be macular and papular areas within a single lesion. The color may be highly inconstant and range from tan to dark brown to pink. These lesions are most commonly found in the back, chest, buttocks, breasts and scalp. They can be found in sun exposed areas as well as sun protected areas, so it is important to check all areas including in the mouth and between the toes.

    In the initial evaluation a dermatoscope can be used to evaluate atypical pigmentation. If a lesion is said to be atypical or dysplastic a biopsy needs to be performed for histologic confirmation. A shallow scoop saucerization with at least 2 mm margin or normal appearing skin can be performed if lesion is removed entirely.

    The management of dysplastic nevus is somewhat controversial. The Skin Cancer Foundation examined outcomes of surgical excision of dysplastic nevi and their association with melanoma in order to help find a better idea of this relationship. The results of the study lead to their clinical recommendations for the management of dysplastic nevi. DN that contain severely atypical components (moderately-to-severely and severely dysplastic nevi) showed an increased association with melanoma and increased rate of melanoma diagnosis and supports the management with excision. When looking at moderately dysplastic nevi the data suggests a generally benign behavior similar to mild dysplastic nevi. This supports management with periodic observation for both moderate and mild DN with a positive margin. Histologic classification of atypia provides some guidance to melanoma risk, but an improved classification may allow better risk stratification in the management of dysplastic nevi. Since further evidence is needed the management of each lesion should be individualized and excision should be considered if judged appropriate by both the patient and provider.

    References

    1) Goldberg L, Lebwohl M. Atypical mole (dysplastic nevi). Skin Cancer Foundation. http://www.skincancer.org/skin-cancer-information/atypical-moles. Accessed May 24, 2017.

    2) Reddy K, Rogers G. Management of dysplastic nevi: the role of complete surgical excision. The Skin Cancer Foundation. http://www.skincancer.org/publications/the-melanoma-letter/summer-2014-vol-32-no-2/management. Published 2014. Accessed May 16, 2017.

    3) Valdebran, M. Atypical mole (clark nevus or dysplastic nevus). Medscape. http://emedicine.medscape.com/article/1056283-overview#a4. Updated April 4, 2017. Accessed May 24, 2017.

    Incorrect

    A: This lesion is consistent with a typical benign nevus and does not need to be biopsied at this time. B: This is the correct answer. This lesion is consistent with a typical nevus due to its symmetric shape, even border, small size, no variation in color and no evolution over time. This lesion should be observed for any changes or signs of melanoma. C: This lesion is not consistent with melanoma due to its symmetry.

    Discussion:
    Atypical or dysplastic nevi are moles that look unusual and can sometimes look like melanoma. They are benign lesions, but people who have dysplastic nevi are at an increased risk of developing melanoma. This risk of melanoma can be in the dysplastic mole itself or in another area on the body. The greater number of dysplastic moles someone has, the higher their risk of developing melanoma. A person who has more than ten dysplastic moles has a twelve times greater risk of developing melanoma.

    Dysplastic moles tend to have a genetic component and can be seen in multiple family members. These types of moles are most commonly found in Caucasians. Individuals who have dysplastic moles and a family history of melanoma are at a very high risk.

    If a physician identifies a mole as atypical or if there is a patient who develops a new mole after age 40, the lesion should be biopsied. Those with atypical moles should monitor them and seek medical attention with any changes or signs of melanoma. When looking for signs of melanoma we can use the ABCs. A is for asymmetry, B is for irregular border, C is for multi-colored, D is for diameter greater than 6mm and E is for evolution or a spot that is changing.

    The pathophysiology behind dysplastic nevi is either inherited or sporadic. UV light can play a role in the development of dysplastic nevi and can be both an initiator and promotor in the transformation of the melanocytes into atypical melanocytes or melanoma.

    A person presenting with a dysplastic nevus may have a personal and/or family history of skin cancer, particularly melanoma. They may have a personal history of increased sun exposure, a recent or rapid change of an existing nevus or the development of a new nevus.

    On physical exam the patient may have a large pigmented lesion 5-15 mm in diameter. The lesion may be asymmetric and contain an irregular, notched or ill-defined border. There may be macular and papular areas within a single lesion. The color may be highly inconstant and range from tan to dark brown to pink. These lesions are most commonly found in the back, chest, buttocks, breasts and scalp. They can be found in sun exposed areas as well as sun protected areas, so it is important to check all areas including in the mouth and between the toes.

    In the initial evaluation a dermatoscope can be used to evaluate atypical pigmentation. If a lesion is said to be atypical or dysplastic a biopsy needs to be performed for histologic confirmation. A shallow scoop saucerization with at least 2 mm margin or normal appearing skin can be performed if lesion is removed entirely.

    The management of dysplastic nevus is somewhat controversial. The Skin Cancer Foundation examined outcomes of surgical excision of dysplastic nevi and their association with melanoma in order to help find a better idea of this relationship. The results of the study lead to their clinical recommendations for the management of dysplastic nevi. DN that contain severely atypical components (moderately-to-severely and severely dysplastic nevi) showed an increased association with melanoma and increased rate of melanoma diagnosis and supports the management with excision. When looking at moderately dysplastic nevi the data suggests a generally benign behavior similar to mild dysplastic nevi. This supports management with periodic observation for both moderate and mild DN with a positive margin. Histologic classification of atypia provides some guidance to melanoma risk, but an improved classification may allow better risk stratification in the management of dysplastic nevi. Since further evidence is needed the management of each lesion should be individualized and excision should be considered if judged appropriate by both the patient and provider.

    References

    1) Goldberg L, Lebwohl M. Atypical mole (dysplastic nevi). Skin Cancer Foundation. http://www.skincancer.org/skin-cancer-information/atypical-moles. Accessed May 24, 2017.

    2) Reddy K, Rogers G. Management of dysplastic nevi: the role of complete surgical excision. The Skin Cancer Foundation. http://www.skincancer.org/publications/the-melanoma-letter/summer-2014-vol-32-no-2/management. Published 2014. Accessed May 16, 2017.

    3) Valdebran, M. Atypical mole (clark nevus or dysplastic nevus). Medscape. http://emedicine.medscape.com/article/1056283-overview#a4. Updated April 4, 2017. Accessed May 24, 2017.

  168. Question 168 of 220
    168. Question

    A 15-year-old boy presents to the emergency room after experiencing a sudden onset of sharp, unilateral chest pain while playing basketball. His vital signs are stable with the exception of mild tachycardia and tachypnea. On exam, he has diminished breath sounds on the right, hyperresonance on percussion on the right, and decreased tactile fremitus. He has a normal S1 and S2, and no murmurs are noted. Of note, he recently went through a growth spurt and has no underlying medical conditions. Based on this information, what is the suspected diagnosis?

    Correct

    The correct answer is spontaneous pneumothorax (A). This patient demonstrates the classic presentation of an individual with this condition. A spontaneous pneumothorax is most commonly diagnosed in individuals between the ages of 10 and 40. Additionally, this patient recently went through a growth spurt, demonstrating that he is most likely tall and thin. His physical exam findings of diminished breath sounds, hyperresonance on percussion, and decreased tactile fremitus are typical of patients with this condition.1 Hypertrophic cardiomyopathy (B) is incorrect. Although this patient developed symptoms during exercise, most patients with HCM develop other symptoms such as dyspnea, presyncope, syncope, heart palpitations, and/or dizziness. The absence of a fourth heart sound (S4) or a heart murmur combined with his abnormal lung exam do not support the diagnosis of HCM.7 Pleural effusion (C) is also incorrect. While this patient’s physical exam findings of diminished breath sounds, hyperresonance on percussion, and decreased tactile fremitus support the diagnosis of a pleural effusion, the background information makes this diagnosis inaccurate. Most patients diagnosed with a pleural effusion suffer from an underlying cardiac or pulmonary disease.8 Pulmonary embolism (D) is incorrect. Although this patient experienced a sudden onset of chest pain, the remainder of his presentation does not support the diagnosis of a PE. This diagnosis is more common in overweight women rather than thin men. Most patients with a PE have an antecedent deep vein thrombosis (DVT) and experience lower extremity pain or swelling. Additionally, patients are at an increased risk of developing a PE if they meet the criteria for Virchow’s triad: venous stasis, vessel wall injury, and hypercoagulability. This patient does not meet these criteria. This patient’s history, physical exam findings, and chest x-ray findings do not support the diagnosis of a PE.9

    References
    1 Daley BJ. Pneumothorax. Medscape. http://emedicine.medscape.com/article/424547-overview. Updated July 20, 2016. Accessed April 19, 2017.
    2 Chesnutt MS, Prendergast TJ. Chapter 9: Pulmonary disorders. In: Papadakis MA, McPhee SJ, ed. Current Medical Diagnosis and Treatment. 54th ed. New York, NY: McGraw-Hill Education. 2015: 312-313.
    3 Gonzales, CA. Spontaneous pneumothorax in children. In: DynaMed [database online]. Ipswich, MA: EBSCO Information Services. http://web.b.ebscohost.com.ezproxymcp.flo.org/dynamed/detail?vid=2&sid=c01dbb52-d28a-438f-b577-5c4bc2c6c9c9%40sessionmgr101&hid=118&bdata=JnNpdGU9ZHluYW1lZC1saXZlJnNjb3BlPXNpdGU%3d#AN=306335&db=dme. Updated January 11, 2016. Accessed April 19, 2017.
    4 Patel A. Spontaneous pneumothorax in adults. In: DynaMed [database online]. Ipswich, MA: EBSCO Information Services. http://web.b.ebscohost.com.ezproxymcp.flo.org/dynamed/detail?sid=c01dbb52-d28a-438f-b577-5c4bc2c6c9c9%40sessionmgr101&vid=3&hid=118&bdata=JnNpdGU9ZHluYW1lZC1saXZlJnNjb3BlPXNpdGU%3d#AN=114714&db=dme. Updated January 9, 2017. Accessed April 19, 2017.
    5 Mayo Clinic Staff. Pneumothorax. Mayo Clinic. http://www.mayoclinic.org/diseases-conditions/pneumothorax/home/ovc-20179880. Updated January 28, 2016. Accessed April 19 2017.
    6 Gluckman W. Pediatric Pneumothorax. Medscape. http://emedicine.medscape.com/article/1003552-overview. Updated October 26, 2015. Accessed April 19, 2017.
    7 Shah SN. Hypertrophic Cardiomyopathy. Medscape. http://emedicine.medscape.com/article/152913-overview. Updated January 5, 2016. Accessed April 19, 2017.
    8 Rubins J. Pleural Effusion. Medscape. http://emedicine.medscape.com/article/299959-overview. Updated March 30, 2017. Accessed April 19, 2017.
    9 Ouellette DR. Pulmonary Embolism. Medscape. http://emedicine.medscape.com/article/300901-overview. Updated June 22, 2016. Accessed April 19, 2017.

    Incorrect

    The correct answer is spontaneous pneumothorax (A). This patient demonstrates the classic presentation of an individual with this condition. A spontaneous pneumothorax is most commonly diagnosed in individuals between the ages of 10 and 40. Additionally, this patient recently went through a growth spurt, demonstrating that he is most likely tall and thin. His physical exam findings of diminished breath sounds, hyperresonance on percussion, and decreased tactile fremitus are typical of patients with this condition.1 Hypertrophic cardiomyopathy (B) is incorrect. Although this patient developed symptoms during exercise, most patients with HCM develop other symptoms such as dyspnea, presyncope, syncope, heart palpitations, and/or dizziness. The absence of a fourth heart sound (S4) or a heart murmur combined with his abnormal lung exam do not support the diagnosis of HCM.7 Pleural effusion (C) is also incorrect. While this patient’s physical exam findings of diminished breath sounds, hyperresonance on percussion, and decreased tactile fremitus support the diagnosis of a pleural effusion, the background information makes this diagnosis inaccurate. Most patients diagnosed with a pleural effusion suffer from an underlying cardiac or pulmonary disease.8 Pulmonary embolism (D) is incorrect. Although this patient experienced a sudden onset of chest pain, the remainder of his presentation does not support the diagnosis of a PE. This diagnosis is more common in overweight women rather than thin men. Most patients with a PE have an antecedent deep vein thrombosis (DVT) and experience lower extremity pain or swelling. Additionally, patients are at an increased risk of developing a PE if they meet the criteria for Virchow’s triad: venous stasis, vessel wall injury, and hypercoagulability. This patient does not meet these criteria. This patient’s history, physical exam findings, and chest x-ray findings do not support the diagnosis of a PE.9

    References
    1 Daley BJ. Pneumothorax. Medscape. http://emedicine.medscape.com/article/424547-overview. Updated July 20, 2016. Accessed April 19, 2017.
    2 Chesnutt MS, Prendergast TJ. Chapter 9: Pulmonary disorders. In: Papadakis MA, McPhee SJ, ed. Current Medical Diagnosis and Treatment. 54th ed. New York, NY: McGraw-Hill Education. 2015: 312-313.
    3 Gonzales, CA. Spontaneous pneumothorax in children. In: DynaMed [database online]. Ipswich, MA: EBSCO Information Services. http://web.b.ebscohost.com.ezproxymcp.flo.org/dynamed/detail?vid=2&sid=c01dbb52-d28a-438f-b577-5c4bc2c6c9c9%40sessionmgr101&hid=118&bdata=JnNpdGU9ZHluYW1lZC1saXZlJnNjb3BlPXNpdGU%3d#AN=306335&db=dme. Updated January 11, 2016. Accessed April 19, 2017.
    4 Patel A. Spontaneous pneumothorax in adults. In: DynaMed [database online]. Ipswich, MA: EBSCO Information Services. http://web.b.ebscohost.com.ezproxymcp.flo.org/dynamed/detail?sid=c01dbb52-d28a-438f-b577-5c4bc2c6c9c9%40sessionmgr101&vid=3&hid=118&bdata=JnNpdGU9ZHluYW1lZC1saXZlJnNjb3BlPXNpdGU%3d#AN=114714&db=dme. Updated January 9, 2017. Accessed April 19, 2017.
    5 Mayo Clinic Staff. Pneumothorax. Mayo Clinic. http://www.mayoclinic.org/diseases-conditions/pneumothorax/home/ovc-20179880. Updated January 28, 2016. Accessed April 19 2017.
    6 Gluckman W. Pediatric Pneumothorax. Medscape. http://emedicine.medscape.com/article/1003552-overview. Updated October 26, 2015. Accessed April 19, 2017.
    7 Shah SN. Hypertrophic Cardiomyopathy. Medscape. http://emedicine.medscape.com/article/152913-overview. Updated January 5, 2016. Accessed April 19, 2017.
    8 Rubins J. Pleural Effusion. Medscape. http://emedicine.medscape.com/article/299959-overview. Updated March 30, 2017. Accessed April 19, 2017.
    9 Ouellette DR. Pulmonary Embolism. Medscape. http://emedicine.medscape.com/article/300901-overview. Updated June 22, 2016. Accessed April 19, 2017.

  169. Question 169 of 220
    169. Question

    Figure 1
    The yellow arrow in figure 1 (lateral x-ray of the foot) points to which structure?

    Correct

    Knowing the complex anatomy of the foot is crucial in differentiating potential pain generators. Over 26 bones make up three regions of the foot including the hindfoot, midfoot, and forefoot. The hindfoot contains the talus and calcaneus, the midfoot contains the navicular, cuboid, and three cuneiform bones, and the forefoot contains the metatarsals and phalanges. The hindfoot and midfoot are separated by the transverse tarsal joint (Chopart joint) and the midfoot and the forefoot by the tarsometatarsal joint. 1

    The tarsal tunnel is a fibro-osseus canal formed on one side by the medial calcaneus and on the other by the flexor retinaculum. Structures that course through the tunnel include flexor tendons and the neurovascular bundle. The posterior tibial nerve can become entrapped in the tarsal tunnel causing pain and numbness, a condition called tarsal tunnel syndrome. 1

    The sinus tarsi is a bony canal between the under surface of the talus and the superior surface of the calcaneus, often referred to as the “eye of the foot”. The sinus tarsi can be a source of painful pathology, or a condition known as sinus tarsi syndrome, where the soft tissue within the sinus tarsi can become inflamed. This can occur after an ankle sprain that causes scar tissue and inflammation to develop within the sinus tarsi. 2

    The sustentaculum tali is a horizontal shelf of the anterior medial calcaneus that helps support the talus superiorly, provides an attachment site for the plantar calcaneonavicular ligament anteriorly, provides an attachment site for the deltoid ligament medially, and forms the roof of the tarsal tunnel and flexor hallucis longus tendon posterior and inferiorly. The sustentaculum tali can become painful with such conditions as tarsal coalition and fracture. 1

    References
    1. Netter, FH. Netter’s Concise Atlas of Orthopaedic Anatomy. Elsevier, Inc. 2002. Philadelphia, PA. Pages 244-279.
    2. Sinus tarsi syndrome. http://www.radiopaedia.org. Accessed 6/13/17.

    Incorrect

    Knowing the complex anatomy of the foot is crucial in differentiating potential pain generators. Over 26 bones make up three regions of the foot including the hindfoot, midfoot, and forefoot. The hindfoot contains the talus and calcaneus, the midfoot contains the navicular, cuboid, and three cuneiform bones, and the forefoot contains the metatarsals and phalanges. The hindfoot and midfoot are separated by the transverse tarsal joint (Chopart joint) and the midfoot and the forefoot by the tarsometatarsal joint. 1

    The tarsal tunnel is a fibro-osseus canal formed on one side by the medial calcaneus and on the other by the flexor retinaculum. Structures that course through the tunnel include flexor tendons and the neurovascular bundle. The posterior tibial nerve can become entrapped in the tarsal tunnel causing pain and numbness, a condition called tarsal tunnel syndrome. 1

    The sinus tarsi is a bony canal between the under surface of the talus and the superior surface of the calcaneus, often referred to as the “eye of the foot”. The sinus tarsi can be a source of painful pathology, or a condition known as sinus tarsi syndrome, where the soft tissue within the sinus tarsi can become inflamed. This can occur after an ankle sprain that causes scar tissue and inflammation to develop within the sinus tarsi. 2

    The sustentaculum tali is a horizontal shelf of the anterior medial calcaneus that helps support the talus superiorly, provides an attachment site for the plantar calcaneonavicular ligament anteriorly, provides an attachment site for the deltoid ligament medially, and forms the roof of the tarsal tunnel and flexor hallucis longus tendon posterior and inferiorly. The sustentaculum tali can become painful with such conditions as tarsal coalition and fracture. 1

    References
    1. Netter, FH. Netter’s Concise Atlas of Orthopaedic Anatomy. Elsevier, Inc. 2002. Philadelphia, PA. Pages 244-279.
    2. Sinus tarsi syndrome. http://www.radiopaedia.org. Accessed 6/13/17.

  170. Question 170 of 220
    170. Question

    Figure 3
    A 50 year-old female presents to your office with a fingertip injury after getting her right index finger caught between a board and a cement block. X-rays taken in the ED 4 days ago show a distal phalanx fracture (tuft fracture) to the index finger. She also has a subungual hematoma involving over 50% of her nail (figure 1). She rates her pain as a 5/10 which is somewhat improved over the last few days. What is the best treatment option?

    Correct

    A subungual hematoma, or bleeding under the nail, commonly occurs after crush injuries of the fingertip. The affected nail may become swollen and significantly painful. Treatment of these injuries is controversial with conflicting recommendations among authors. Subungual hematomas involving over 50% of the nail will generally have a nail bed injury. Patients with tuft fractures (distal phalanx fractures) have nail bed injuries in 90% of cases. Several authors recommend removing the nail in these cases in order to repair the nail bed directly. However, more commonly in clinical practice the nail is left in place regardless of the size of the subungual hematoma. Removing the nail can be a traumatic experience for patients and requires a lengthy period of wound care as the nail heals back down. Leaving the nail intact helps keep the nail bed approximated and is much easier for patients to care for. Patients with severe pain with a subungual hematoma may require decompression of the hematoma. Decompression can be achieved by making 2-3 small holes in the nail (nail trephination) above the hematoma with electro cautery or a heated paper clip. Distal phalanx fractures should be treated with a splint to immobilize and protect against contact for 6 weeks. Patients can be advised that these fractures can heal slowly, especially if there is displacement or comminution, and sensitivity over the finger tip for 3-4 months is not uncommon. 1,2

    References
    1. Steven E. Roser, MD, Harris Gellman, MD. Comparison of nail bed repair versus nail trephination for subungual hematomas in children. The Journal of Hand Surgery 1999. (24) 6. 1166–1170.
    2. Wang QC, Johnson BA. Fingertip Injuries. Am Fam Physician. 2001 May 15;63(10):1961-1966.

    Incorrect

    A subungual hematoma, or bleeding under the nail, commonly occurs after crush injuries of the fingertip. The affected nail may become swollen and significantly painful. Treatment of these injuries is controversial with conflicting recommendations among authors. Subungual hematomas involving over 50% of the nail will generally have a nail bed injury. Patients with tuft fractures (distal phalanx fractures) have nail bed injuries in 90% of cases. Several authors recommend removing the nail in these cases in order to repair the nail bed directly. However, more commonly in clinical practice the nail is left in place regardless of the size of the subungual hematoma. Removing the nail can be a traumatic experience for patients and requires a lengthy period of wound care as the nail heals back down. Leaving the nail intact helps keep the nail bed approximated and is much easier for patients to care for. Patients with severe pain with a subungual hematoma may require decompression of the hematoma. Decompression can be achieved by making 2-3 small holes in the nail (nail trephination) above the hematoma with electro cautery or a heated paper clip. Distal phalanx fractures should be treated with a splint to immobilize and protect against contact for 6 weeks. Patients can be advised that these fractures can heal slowly, especially if there is displacement or comminution, and sensitivity over the finger tip for 3-4 months is not uncommon. 1,2

    References
    1. Steven E. Roser, MD, Harris Gellman, MD. Comparison of nail bed repair versus nail trephination for subungual hematomas in children. The Journal of Hand Surgery 1999. (24) 6. 1166–1170.
    2. Wang QC, Johnson BA. Fingertip Injuries. Am Fam Physician. 2001 May 15;63(10):1961-1966.

  171. Question 171 of 220
    171. Question

    A 41- year-old obese Caucasian female presents to the office with a red, dry rash under her breasts that began one week ago. She reports that the rash is very irritating, itches, and has been progressively getting worse. Patient denies pain, history of skin cancer, recent changes in medications, sick contacts, febrile illness, joint pains, or trauma. Punch biopsy confirms inverse psoriasis. Of the following choices, which is the best initial step in this patient’s management?

    Correct

    Answer is D. Triamcinolone acetonide 0.1% Cream. Inverse psoriasis is treated initially with a topical glucocorticoid which should only be applied for a short period of time to avoid atrophy of the skin. If the psoriasis is recurrent or fails to respond to treatment, systemic therapy should be considered. Watchful waiting would not be appropriate because psoriasis is a chronic disorder. Nystatin topical cream treats fungal infections such as Candidal Intertrigo. Topical 5-Fluorouracil has been FDA- approved for the treatment of basal cell carcinomas and actinic keratosis, however, it is not a recommended treatment for psoriasis.

    Discussion
    Psoriasis Vulgaris is a chronic disorder that affects 1.5-2% of the population in Western countries and has various clinical presentations.1 The types of psoriasis vulgaris include acute guttate, chronic stable plaque, palmoplantar, and inverse. Individuals may present with a few lesions in one location while other individuals may have widespread skin involvement. Most cases involve localized psoriasis, but out of the 3-5 million people in the United States diagnosed, 300,000 individuals have generalized lesions.1 The typical psoriatic lesions are scaly papules and plaques that are recurrent.1 Pustules as well as erythema at the base of the lesions are seen along with the plaques.1 Psoriasis affects men and women equally and is seen in all ages with a peak incidence at age 22.5.1 Patients who present at an earlier age tend to have more of an aggressive and longer course. Psoriasis also affects all races, but 3.6% of Caucasians have the condition compared to 1.9% of African Americans.2

    A positive family history is also strongly associated with psoriasis. Interestingly, if one parent has psoriasis the child has an 8% chance of developing the condition and a 41% chance if both parents have psoriasis.1 Triggers for psoriasis include stress, drugs such as glucocorticoids, lithium, interferon, β- adrenergic blockers, alcohol, and trauma such as rubbing or scratching the skin. 1 Guttate psoriasis is also associated with acute streptococcal infection.

    The pathophysiology of psoriasis is marked by the acceleration of the cell cycle of the keratinocytes. As a result there is an over proliferation of epidermal cells and CD8+ T cells (28 times the normal production).1 Psoriasis is treated as an autoimmune response and is considered a disease manifested by T cells.1

    A patient with psoriasis may present with the eruptive type, such as guttate psoriasis, which presents with multiple salmon-pink small plaques commonly on the trunk, and resolves spontaneously.1 However, most individuals diagnosed with psoriasis have the chronic plaque psoriasis type. These lesions present for long periods of time and minimally change. The red plaques have silvery white scales and are sharply demarcated.1 These plaques may coalesce and form larger lesions. Common sites for the chronic stable type include knees, elbows, scalp, palms, soles, and gluteal region.1 Psoriasis is usually symmetric and pruritus often times accompanies the visible plaques. Inverse psoriasis occurs in body folds such as the submammary region. The body folds create a moist and warm environment and often times scales are not seen in these regions. The inverse pattern involves a “brightly erythematous and glistening base”.1 On physical examination the clinician should examine the lesion’s appearance, size, and location. The clinician should determine if the psoriasis is mild, moderate, or severe by observing the total body surface area affected by the psoriasis. Mild is less than 3%, moderate is between 3 to 10% and severe is more than 10%.2 Well-demarcated “red plaques with silvery scales” found on extensor surfaces such as the elbows, knees, with thick scale on the scalp or nail pitting are diagnostic. 3 A skin biopsy or scraping of the body folds may be necessary and will differentiate psoriasis from other skin conditions such as candidal intertrigo.3

    There are various treatments for psoriasis and many factors need to be considered when managing this condition. Localized psoriasis may be effectively treated with topical glucocorticoids, Vitamin D analogues such as calcipotriene, or topical pimecrolimus 1%.1 Inverse psoriasis is treated with topical glucocorticoids for a short amount of time due to atrophy of the skin, and should be switched to topical Vitamin D derivatives, topical tacrolimus, or pimecrolimus.1 Systemic therapies such as oral Methotrexate or intravenous Adalimumab should be considered for psoriasis that recurs or does not respond to treatment. Psoriasis on the scalp typically involves thick scale and plaques which tar or ketoconazole shampoos are initially used and betamethasone valerate or clobestasol propionate may be applied after.1 Generalized psoriasis can also be managed with different therapies. Guttate psoriasis is treated with antibiotics and often times UVB phototherapy. Generalized chronic plaque type is managed with systemic medications such as biologics or phototherapy. They may be used in combination for greater efficacy.

    References

    1. Wolff K, Johnson R, Saavedra A, Roh E. Fitzpatrick’s Color Atlas And Synopsis Of Clinical Dermatology. 7th ed. The McGraw- Hill Companies, Inc.; :49-56.
    2. Learn about plaque psoriasis, guttate psoriasis, inverse psoriasis, and pustular psoriasis | National Psoriasis Foundation. Psoriasisorg. 2017. Available at: https://www.psoriasis.org/about-psoriasis. Accessed April 24, 2017.
    3. Papadakis M, McPhee S, Rabow M. 2015 Current Medical Diagnosis & Treatment. 1st ed. New York: McGraw-Hill Education/Medical; 2015:105-107.

    Incorrect

    Answer is D. Triamcinolone acetonide 0.1% Cream. Inverse psoriasis is treated initially with a topical glucocorticoid which should only be applied for a short period of time to avoid atrophy of the skin. If the psoriasis is recurrent or fails to respond to treatment, systemic therapy should be considered. Watchful waiting would not be appropriate because psoriasis is a chronic disorder. Nystatin topical cream treats fungal infections such as Candidal Intertrigo. Topical 5-Fluorouracil has been FDA- approved for the treatment of basal cell carcinomas and actinic keratosis, however, it is not a recommended treatment for psoriasis.

    Discussion
    Psoriasis Vulgaris is a chronic disorder that affects 1.5-2% of the population in Western countries and has various clinical presentations.1 The types of psoriasis vulgaris include acute guttate, chronic stable plaque, palmoplantar, and inverse. Individuals may present with a few lesions in one location while other individuals may have widespread skin involvement. Most cases involve localized psoriasis, but out of the 3-5 million people in the United States diagnosed, 300,000 individuals have generalized lesions.1 The typical psoriatic lesions are scaly papules and plaques that are recurrent.1 Pustules as well as erythema at the base of the lesions are seen along with the plaques.1 Psoriasis affects men and women equally and is seen in all ages with a peak incidence at age 22.5.1 Patients who present at an earlier age tend to have more of an aggressive and longer course. Psoriasis also affects all races, but 3.6% of Caucasians have the condition compared to 1.9% of African Americans.2

    A positive family history is also strongly associated with psoriasis. Interestingly, if one parent has psoriasis the child has an 8% chance of developing the condition and a 41% chance if both parents have psoriasis.1 Triggers for psoriasis include stress, drugs such as glucocorticoids, lithium, interferon, β- adrenergic blockers, alcohol, and trauma such as rubbing or scratching the skin. 1 Guttate psoriasis is also associated with acute streptococcal infection.

    The pathophysiology of psoriasis is marked by the acceleration of the cell cycle of the keratinocytes. As a result there is an over proliferation of epidermal cells and CD8+ T cells (28 times the normal production).1 Psoriasis is treated as an autoimmune response and is considered a disease manifested by T cells.1

    A patient with psoriasis may present with the eruptive type, such as guttate psoriasis, which presents with multiple salmon-pink small plaques commonly on the trunk, and resolves spontaneously.1 However, most individuals diagnosed with psoriasis have the chronic plaque psoriasis type. These lesions present for long periods of time and minimally change. The red plaques have silvery white scales and are sharply demarcated.1 These plaques may coalesce and form larger lesions. Common sites for the chronic stable type include knees, elbows, scalp, palms, soles, and gluteal region.1 Psoriasis is usually symmetric and pruritus often times accompanies the visible plaques. Inverse psoriasis occurs in body folds such as the submammary region. The body folds create a moist and warm environment and often times scales are not seen in these regions. The inverse pattern involves a “brightly erythematous and glistening base”.1 On physical examination the clinician should examine the lesion’s appearance, size, and location. The clinician should determine if the psoriasis is mild, moderate, or severe by observing the total body surface area affected by the psoriasis. Mild is less than 3%, moderate is between 3 to 10% and severe is more than 10%.2 Well-demarcated “red plaques with silvery scales” found on extensor surfaces such as the elbows, knees, with thick scale on the scalp or nail pitting are diagnostic. 3 A skin biopsy or scraping of the body folds may be necessary and will differentiate psoriasis from other skin conditions such as candidal intertrigo.3

    There are various treatments for psoriasis and many factors need to be considered when managing this condition. Localized psoriasis may be effectively treated with topical glucocorticoids, Vitamin D analogues such as calcipotriene, or topical pimecrolimus 1%.1 Inverse psoriasis is treated with topical glucocorticoids for a short amount of time due to atrophy of the skin, and should be switched to topical Vitamin D derivatives, topical tacrolimus, or pimecrolimus.1 Systemic therapies such as oral Methotrexate or intravenous Adalimumab should be considered for psoriasis that recurs or does not respond to treatment. Psoriasis on the scalp typically involves thick scale and plaques which tar or ketoconazole shampoos are initially used and betamethasone valerate or clobestasol propionate may be applied after.1 Generalized psoriasis can also be managed with different therapies. Guttate psoriasis is treated with antibiotics and often times UVB phototherapy. Generalized chronic plaque type is managed with systemic medications such as biologics or phototherapy. They may be used in combination for greater efficacy.

    References

    1. Wolff K, Johnson R, Saavedra A, Roh E. Fitzpatrick’s Color Atlas And Synopsis Of Clinical Dermatology. 7th ed. The McGraw- Hill Companies, Inc.; :49-56.
    2. Learn about plaque psoriasis, guttate psoriasis, inverse psoriasis, and pustular psoriasis | National Psoriasis Foundation. Psoriasisorg. 2017. Available at: https://www.psoriasis.org/about-psoriasis. Accessed April 24, 2017.
    3. Papadakis M, McPhee S, Rabow M. 2015 Current Medical Diagnosis & Treatment. 1st ed. New York: McGraw-Hill Education/Medical; 2015:105-107.

  172. Question 172 of 220
    172. Question

    A 72 year-old African American man hospitalized due to worsening shortness of breath, cough and chest pain is found incidentally to have an elevated calcium level of 15mg/dL. He reports a 1 year history of mild dyspnea, fatigue, and weight loss. Chest x ray with follow up CT shows bilateral hilar adenopathy and reticular opacities distributed within the upper lung lobes. Laboratory evaluation of this patient may show which of the following?

    Correct

    The answer is D. The patient’s presentation above is suggestive for sarcoidosis, a granulomatous disease that causes an increase in intestinal absorption of calcium. This is due to enhanced secretion of 1 alpha hydroxylase which leads to an increased production of the vitamin D metabolite called calcitriol (1,25 dihydroxyvitamin D) 2. In sarcoidosis, elevated serum calcium concentrations are independent of PTH, therefore intact PTH levels are low. Option A is suggestive of primary hyperparathyroidism, a PTH mediated disease leading to elevated serum and urine calcium levels. Option B is suggestive of familial hypercalciuric hypercalcemia, a genetic disorder affecting the gene that codes for the calcium sensing receptor (CaSR) of the body3. These patients present with normal to mildly elevated PTH levels with increased urinary calcium levels. Option C is suggestive of hypercalcemia of malignancy. Intact PTH levels and vitamin D metabolites are decreased while PTH-rP levels are increased due to secretion by the underlying tumor.

    References:
    1. Agraharkar M. Hypercalcemia. Medscape. http://emedicine.medscape.com/article/240681-overview#a6. Published August 2, 2016. Accessed June 1, 2017.
    2. Agus ZS. Hypercalcemia in Granulomatous Disease. UpToDate. https://www.uptodate.com/contents/hypercalcemia-in-granulomatous-diseases?source=search_result&search=pth-rp%20in%20granulomatous%20disease&selectedTitle=1~150. Published October 29, 2015. Accessed June 1, 2017.
    3. Brown EM. Disorders of the calcium-sensing receptor: Familial hypocalciuric hypercalcemia and autosomal dominant hypocalcemia. UpToDate. https://www.uptodate.com/contents/disorders-of-the-calcium-sensing-receptor-familial-hypocalciuric-hypercalcemia-and-autosomal-dominant-hypocalcemia?source=search_result&search=familial%20hypocalciuric%20hypercalcemia&selectedTitle=1~15. Published April 18, 2017. Accessed June 6, 2017.
    4. Horwitz MJ. Hypercalcemia of Malignancy: Mechanisms. UpToDate. https://www.uptodate.com/contents/hypercalcemia-of-malignancy-mechanisms?source=search_result&search=hypercalcemia%20of%20malignancy&selectedTitle=1~150. Published August 22, 2016. Accessed June 1, 2017.
    5. Shane E. Clinical Manifestations of Hypercalcemia. UpToDate. https://www.uptodate.com/contents/clinical-manifestations-of-hypercalcemia?source=see_link#H12. Published April 24, 2017. Accessed June 6, 2017.
    6. Shane E. Diagnostic Approach to Hypercalcemia. UpToDate. https://www.uptodate.com/contents/diagnostic-approach-to-hypercalcemia?source=search_result&search=hypercalcemia&selectedTitle=1~150#H10. Published November 1, 2016. Accessed June 6, 2017.

    Incorrect

    The answer is D. The patient’s presentation above is suggestive for sarcoidosis, a granulomatous disease that causes an increase in intestinal absorption of calcium. This is due to enhanced secretion of 1 alpha hydroxylase which leads to an increased production of the vitamin D metabolite called calcitriol (1,25 dihydroxyvitamin D) 2. In sarcoidosis, elevated serum calcium concentrations are independent of PTH, therefore intact PTH levels are low. Option A is suggestive of primary hyperparathyroidism, a PTH mediated disease leading to elevated serum and urine calcium levels. Option B is suggestive of familial hypercalciuric hypercalcemia, a genetic disorder affecting the gene that codes for the calcium sensing receptor (CaSR) of the body3. These patients present with normal to mildly elevated PTH levels with increased urinary calcium levels. Option C is suggestive of hypercalcemia of malignancy. Intact PTH levels and vitamin D metabolites are decreased while PTH-rP levels are increased due to secretion by the underlying tumor.

    References:
    1. Agraharkar M. Hypercalcemia. Medscape. http://emedicine.medscape.com/article/240681-overview#a6. Published August 2, 2016. Accessed June 1, 2017.
    2. Agus ZS. Hypercalcemia in Granulomatous Disease. UpToDate. https://www.uptodate.com/contents/hypercalcemia-in-granulomatous-diseases?source=search_result&search=pth-rp%20in%20granulomatous%20disease&selectedTitle=1~150. Published October 29, 2015. Accessed June 1, 2017.
    3. Brown EM. Disorders of the calcium-sensing receptor: Familial hypocalciuric hypercalcemia and autosomal dominant hypocalcemia. UpToDate. https://www.uptodate.com/contents/disorders-of-the-calcium-sensing-receptor-familial-hypocalciuric-hypercalcemia-and-autosomal-dominant-hypocalcemia?source=search_result&search=familial%20hypocalciuric%20hypercalcemia&selectedTitle=1~15. Published April 18, 2017. Accessed June 6, 2017.
    4. Horwitz MJ. Hypercalcemia of Malignancy: Mechanisms. UpToDate. https://www.uptodate.com/contents/hypercalcemia-of-malignancy-mechanisms?source=search_result&search=hypercalcemia%20of%20malignancy&selectedTitle=1~150. Published August 22, 2016. Accessed June 1, 2017.
    5. Shane E. Clinical Manifestations of Hypercalcemia. UpToDate. https://www.uptodate.com/contents/clinical-manifestations-of-hypercalcemia?source=see_link#H12. Published April 24, 2017. Accessed June 6, 2017.
    6. Shane E. Diagnostic Approach to Hypercalcemia. UpToDate. https://www.uptodate.com/contents/diagnostic-approach-to-hypercalcemia?source=search_result&search=hypercalcemia&selectedTitle=1~150#H10. Published November 1, 2016. Accessed June 6, 2017.

  173. Question 173 of 220
    173. Question

    A 4 week old Caucasian female presents to the ER with non-bilious projectile vomiting for the past 5 days. Her mother reports that after vomiting, she seems hungry again instantly. On exam, she is ill appearing. Her stomach appears distended and you observe left to right peristaltic waves in her upper abdomen. A small round mass is palpated in her right upper abdomen. Her only past medical history is an episode of pertussis at 2 weeks old for which she was treated successfully with erythromycin. What is the most likely diagnosis?

    Correct

    Pyloric stenosis is diagnosed in around 1-8 out of every 1,000 births, with a 4-5:1 male to female ratio. The average age at diagnosis is 43.1 days1. It has been found, however, that a later age of diagnosis is associated with a greater degree of prematurity2. The delayed diagnosis in premature infants is due to low birth weight and an atypical presentation3. Pyloric stenosis is more common in white infants than those of Hispanic, black, or Asian descent3. The cause is unknown; however there have been some documented risk factors including male gender, prematurity, positive family history, and early use of erythromycin4.
    Pyloric stenosis occurs when both the circular and longitudinal muscle layers of the pylorus become hypertrophic and hyperplastic. This causes a narrowing of the pyloric lumen and an inability for liquid to pass through. Although the cause is multifactorial, it is known to involve deficiency in neurons containing nitric oxide synthase, abnormal innervation of the myenteric plexus, and lack of exposure to vasoactive intestinal peptide (VIP)3. Nitric oxide synthase contributes to the relaxation of the smooth muscle in the myenteric plexus. VIP also contributes to relaxation of the stomach and can be found in breast milk. This could explain why formula-fed babies are more likely to develop pyloric stenosis3.
    A classic presentation of pyloric stenosis involves non-bilious, post-prandial projectile vomiting between 2 and 6 weeks of age. Following emesis, the infant will typically be hungry immediately and want to be fed. It is common to see a distended upper abdomen after feeding accompanied by prominent peristaltic waves traveling from left to right. In a small percentage of patients, there is a palpable “olive” shaped mass in the right upper abdomen, representing the thickening of the pylorus muscle1.
    Common lab findings include hypochloremic alkalosis and hypokalemia. Dehydration occurs in cases of advanced disease, which is reflected in elevated hemoglobin and hematocrit values1 or hyper/hyponatremia3. If the classic “olive” mass is not palpated, imaging studies can be used to arrive at a diagnosis. The first line imaging study for pyloric stenosis is an abdominal ultrasound. Positive findings include a hypoechoic muscle ring greater than 4mm thick and a pylorus length greater than 15mm1. If the US is unclear, a barium upper GI series may be performed. This would show retained contrast in the stomach and a long narrow channel with a double track of barium1.
    First line treatment is a Ramstedt pyloromyotomy, which is typically performed laparoscopically. In this procedure, the outer layer of the pylorus is incised which allows the inner lining to bulge out, allowing gastric contents to pass through the lumen4. Before an infant is brought in for surgery, they will be made NPO and be given intravenous fluids and electrolytes to correct the deficiencies sustained from vomiting3. If for some reason general anesthesia is contraindicated, the infant can be treated non-surgically with atropine sulfate, however this route has a lower success rate than a pyloromyotomy3.
    The answer to the self-assessment question is B, female gender. As stated above, pyloric stenosis is most common in males with around a 4-5:1 predominance. Caucasian infants are 25% more likely to develop pyloric stenosis than Hispanics, and almost 75% more likely than blacks or Asians3. Pyloric stenosis is most frequently diagnosed in infants between 2-6 weeks of life. Early use of erythromycin has been found to be associated with a higher incidence of pyloric stenosis1.

    References
    1. Hay WW Jr., Levin MJ, Deterding RR, Abzug MJ. Current Diagnosis & Treatment, Pediatrics. 22nd ed. Lange Medical Publications; 2014: 658-659.
    2. Costanzo CM, MD, Vinocur C, MD, Berman L, MD. Prematurity Affects Age of Presentation of Pyloric Stenosis. Clinical Pediatrics. Vol 56, Issue2, pp 127-131. May 4, 2016.
    3. Subramaniam S, MD. Pediatric Pyloric Stenosis. Medscape. Nov 20, 2016, Accessed Jun 27, 2017.
    4. Mayo Clinic Staff. Pyloric Stenosis. MayoClinic.org. Nov 24, 2015. Accessed Jun 27, 2017.

    Incorrect

    Pyloric stenosis is diagnosed in around 1-8 out of every 1,000 births, with a 4-5:1 male to female ratio. The average age at diagnosis is 43.1 days1. It has been found, however, that a later age of diagnosis is associated with a greater degree of prematurity2. The delayed diagnosis in premature infants is due to low birth weight and an atypical presentation3. Pyloric stenosis is more common in white infants than those of Hispanic, black, or Asian descent3. The cause is unknown; however there have been some documented risk factors including male gender, prematurity, positive family history, and early use of erythromycin4.
    Pyloric stenosis occurs when both the circular and longitudinal muscle layers of the pylorus become hypertrophic and hyperplastic. This causes a narrowing of the pyloric lumen and an inability for liquid to pass through. Although the cause is multifactorial, it is known to involve deficiency in neurons containing nitric oxide synthase, abnormal innervation of the myenteric plexus, and lack of exposure to vasoactive intestinal peptide (VIP)3. Nitric oxide synthase contributes to the relaxation of the smooth muscle in the myenteric plexus. VIP also contributes to relaxation of the stomach and can be found in breast milk. This could explain why formula-fed babies are more likely to develop pyloric stenosis3.
    A classic presentation of pyloric stenosis involves non-bilious, post-prandial projectile vomiting between 2 and 6 weeks of age. Following emesis, the infant will typically be hungry immediately and want to be fed. It is common to see a distended upper abdomen after feeding accompanied by prominent peristaltic waves traveling from left to right. In a small percentage of patients, there is a palpable “olive” shaped mass in the right upper abdomen, representing the thickening of the pylorus muscle1.
    Common lab findings include hypochloremic alkalosis and hypokalemia. Dehydration occurs in cases of advanced disease, which is reflected in elevated hemoglobin and hematocrit values1 or hyper/hyponatremia3. If the classic “olive” mass is not palpated, imaging studies can be used to arrive at a diagnosis. The first line imaging study for pyloric stenosis is an abdominal ultrasound. Positive findings include a hypoechoic muscle ring greater than 4mm thick and a pylorus length greater than 15mm1. If the US is unclear, a barium upper GI series may be performed. This would show retained contrast in the stomach and a long narrow channel with a double track of barium1.
    First line treatment is a Ramstedt pyloromyotomy, which is typically performed laparoscopically. In this procedure, the outer layer of the pylorus is incised which allows the inner lining to bulge out, allowing gastric contents to pass through the lumen4. Before an infant is brought in for surgery, they will be made NPO and be given intravenous fluids and electrolytes to correct the deficiencies sustained from vomiting3. If for some reason general anesthesia is contraindicated, the infant can be treated non-surgically with atropine sulfate, however this route has a lower success rate than a pyloromyotomy3.
    The answer to the self-assessment question is B, female gender. As stated above, pyloric stenosis is most common in males with around a 4-5:1 predominance. Caucasian infants are 25% more likely to develop pyloric stenosis than Hispanics, and almost 75% more likely than blacks or Asians3. Pyloric stenosis is most frequently diagnosed in infants between 2-6 weeks of life. Early use of erythromycin has been found to be associated with a higher incidence of pyloric stenosis1.

    References
    1. Hay WW Jr., Levin MJ, Deterding RR, Abzug MJ. Current Diagnosis & Treatment, Pediatrics. 22nd ed. Lange Medical Publications; 2014: 658-659.
    2. Costanzo CM, MD, Vinocur C, MD, Berman L, MD. Prematurity Affects Age of Presentation of Pyloric Stenosis. Clinical Pediatrics. Vol 56, Issue2, pp 127-131. May 4, 2016.
    3. Subramaniam S, MD. Pediatric Pyloric Stenosis. Medscape. Nov 20, 2016, Accessed Jun 27, 2017.
    4. Mayo Clinic Staff. Pyloric Stenosis. MayoClinic.org. Nov 24, 2015. Accessed Jun 27, 2017.

  174. Question 174 of 220
    174. Question

    A 45 year old female with no significant past medial history presents to the orthopedic office for evaluation of right knee pain. Two days ago the patient was involved in a head on collision while riding unrestrained in the front passenger seat of a small car. The patient states that she struck her knee against the dashboard of the car during the collision. The patient states there is pain globally throughout the right knee, but it is worse along the posterior aspect of the joint. On physical examination there is visible swelling of the joint. There is no medial or lateral joint tenderness. There is a negative Lachman test. There is negative anterior drawer test. There is 2+ posterior drawer test. There is no significant laxity to valgus or varus stress at 0 and 30 flexion of the right knee. She is able to perform a straight leg raise. Which soft tissue injury did the patient most likely suffer in the collision?

    Correct

    Answer: (b) – The dashboard mechanism of injury, posterior pain, and positive posterior drawer test indicate that the patient’s PCL is torn. (a) – Incorrect. This is not the best choice because there is no joint line tenderness and no other indications of meniscal tear such as complaints of joint locking and giving way. (c) – Incorrect. Patient is able to perform a straight leg raise. This rules out the possibility of a patella tendon rupture. (d) – Incorrect. A negative anterior drawer test and negative Lachman test rule out disruption of the ACL.
    Discussion:
    The posterior cruciate ligament is one of four ligaments that stabilize the knee by connecting the femur to the tibia1. The PCL is found inside the knee joint, originating at the posterior tibial sulcus below the articular surface and inserting anterolateral medial femoral condyle2. The PCL functions to minimize posterior translation of the tibia, thus protecting and stabilizing the knee joint1,2. The PCL also protects the knee joint from hyperextension, and stabilizes the knee joint during rotational movements3. Injury of the posterior cruciate ligament makes up only 5-20% of all knee ligamentous injuries2, and often occurs in conjunction with injuries to other structures of the knee such as ligament, cartilage, and bone1. The PCL is significantly stronger than the ACL and is therefore less likely to get injured, and also more difficult to assess for minor injuries. Injury to the PCL is classified as a sprain and there are 3 grades. A Grade 1 sprain indicates the ligament has been slightly stretched, but not compromised to the point where it can no longer stabilize the knee. A Grade 2 sprain, also known as a partial tear, indicates the ligament has become loose. A Grade 3 sprain is a full tear, or rupture, of the ligament and the knee joint is no longer stable1. Most often PCL injuries are partial tears and minimal intervention is needed.
    Injury to the PCL is most frequently sustained by a forceful direct blow to the anterior tibia while the knee is in a flexed position; often referred to as a “dashboard injury”1,2,3. The injury can also occur from forced hyperextension of the leg1,2. These injuries are commonly seen in car accidents when a person strikes his lower leg on the dashboard of the car, a tackle injury in contact sports, and from a fall forward onto a flexed knee3. It is less likely to see injuries to the PCL occur from normal activities of daily life, which are often causes of other knee soft tissue injuries.
    Excessive posterior translation of the tibia from the force sustained during the previously mentioned injuries exceeds the stabilizing capability of the PCL, 2500-3000N2, and results in injury, or sprain to the ligament1. Extent of PCL sprain is directly related to the amount of posterior subluxation of tibia with the knee at 90 flexion. A Grade 1 sprain has 1-5mm posterior translation, a grade 2 has 6-10mm posterior translation, and a grade 3 has >10mm posterior translation2.
    History obtained from the patient with suspicion of PCL injury will include some moment of trauma, whether it be the car accident, tackle, or fall with hyperextension. Symptoms associated with the injury include persistent pain and swelling immediately after injury, difficulty walking, feelings of knee instability, giving way of knee joint especially when walking downhill or descending stairs, and posterior knee pain1,2. However, these symptoms are often subtle or not noticed in isolated PCL injuries2.
    The most accurate physical exam test to diagnose a PCL injury is the posterior drawer test performed at 90 flexion. A posterior directed force applied to the proximal tibia will results in >10-15mm posterior translation of tibia in PCL injury2. Laxity at 0 flexion with varus and valgus stress to the knee can also be indicative of PCL injury. The posterior sag sign on physical exam suggest PCL injury. A posterior shift of the tibia of the involved knee is observed when a patient lies on back and flexes both knees to 902.
    First line imaging for complaints of knee pain are plan films. AP, supine lateral, and lateral stress views of the affected knee are all appropriate x-rays to order. While x-ray will not show injury to the PCL, they may show avulsion fractures due to acute rupture of the ligament1,2. The lateral stress view is becoming a more popular and frequently used tool in diagnosing PCL injury. The x-ray is taken with the knee flexed to 70 and stress applied to the anterior tibia. Asymmetric posterior tibial displacement of the two knees indicated PCL injury2.
    The confirmatory test for diagnosing PCL injury is an MRI of the affected knee. Normally the PCL appears as an uninterrupted dark band on MRI. If there has been injury to the ligament there will be disruption of this dark band evident on MRI3.
    There are both surgical and nonsurgical treatments for PCL injuries depending on the patient’s symptoms and degree of injury to the PCL and entire knee joint. Grade 1 and 2 sprains of the PCL are often treated nonsurgically. This treatment includes bracing for protected weightbearing and physical therapy for strengthening of the quadriceps. Patients are able to return to normal activity including sports in 2-4 weeks with successful conservative treatment1,2. In isolated grade 3 sprains in patients who are not athletes, nonsurgical treatment can also be used and includes immobilization in extension for 4 weeks followed by daily range of motion exercises and strengthening2.
    In combined ligamentous injuries with knee instability and isolated grade 2 or 3 PCL sprain with avulsion fracture, surgery is indicated to stabilize the knee joint. Because a torn PCL cannot be repaired, reconstruction of the ligament in the surgery of choice. Graft options for PCL reconstruction include autografts, allografts, and artificial material3. Allograft reconstruction is the most frequently used method of reconstruction and tissues include Achilles tendon, bone-patellar tendon-bone, hamstring, and anterior tibialis tendon3. Reconstruction surgery is often performed arthroscopically unless a large avulsion fracture needs repair, in which case an open approach is taken and the bone-tendon-bone graft is inlaid2,3. In the arthroscopic approach, a canal is drilled through the tibia starting on the anterolateral aspect, passing through the insertion point of the PCL and out the medial aspect of the femur. The graft is secured to the tibia with a screw, and held in place on the femur using a stabilizing button2,3. Post surgically patients are immobilized in extension. Physical therapy begins 1-4 weeks post surgery, and full recovery and return to normal activity requires between 6 and 12 months1,2,3. Surgical techniques for PCL reconstruction continue to improve, and generally patients have good success and full recovery.
    REFERENCES:
    1. Posterior Cruciate Ligament Injuries.” Posterior Cruciate Ligament Injuries-OrthoInfo – AAOS. American Orthopedic Society for Sports Medicine, 01 Feb. 2009. Web. 08 July 2017.
    2. Allen, Deborah, MD. “PCL Injury.” Orthobullets. Lineage Medical, 2011. Web. 08 July 2017.
    3. Orthopaedic Procedures | Posterior Cruciate Ligament Reconstruction.” One Health. The One Health Group, n.d. Web. 08 July 2017.

    Incorrect

    Answer: (b) – The dashboard mechanism of injury, posterior pain, and positive posterior drawer test indicate that the patient’s PCL is torn. (a) – Incorrect. This is not the best choice because there is no joint line tenderness and no other indications of meniscal tear such as complaints of joint locking and giving way. (c) – Incorrect. Patient is able to perform a straight leg raise. This rules out the possibility of a patella tendon rupture. (d) – Incorrect. A negative anterior drawer test and negative Lachman test rule out disruption of the ACL.
    Discussion:
    The posterior cruciate ligament is one of four ligaments that stabilize the knee by connecting the femur to the tibia1. The PCL is found inside the knee joint, originating at the posterior tibial sulcus below the articular surface and inserting anterolateral medial femoral condyle2. The PCL functions to minimize posterior translation of the tibia, thus protecting and stabilizing the knee joint1,2. The PCL also protects the knee joint from hyperextension, and stabilizes the knee joint during rotational movements3. Injury of the posterior cruciate ligament makes up only 5-20% of all knee ligamentous injuries2, and often occurs in conjunction with injuries to other structures of the knee such as ligament, cartilage, and bone1. The PCL is significantly stronger than the ACL and is therefore less likely to get injured, and also more difficult to assess for minor injuries. Injury to the PCL is classified as a sprain and there are 3 grades. A Grade 1 sprain indicates the ligament has been slightly stretched, but not compromised to the point where it can no longer stabilize the knee. A Grade 2 sprain, also known as a partial tear, indicates the ligament has become loose. A Grade 3 sprain is a full tear, or rupture, of the ligament and the knee joint is no longer stable1. Most often PCL injuries are partial tears and minimal intervention is needed.
    Injury to the PCL is most frequently sustained by a forceful direct blow to the anterior tibia while the knee is in a flexed position; often referred to as a “dashboard injury”1,2,3. The injury can also occur from forced hyperextension of the leg1,2. These injuries are commonly seen in car accidents when a person strikes his lower leg on the dashboard of the car, a tackle injury in contact sports, and from a fall forward onto a flexed knee3. It is less likely to see injuries to the PCL occur from normal activities of daily life, which are often causes of other knee soft tissue injuries.
    Excessive posterior translation of the tibia from the force sustained during the previously mentioned injuries exceeds the stabilizing capability of the PCL, 2500-3000N2, and results in injury, or sprain to the ligament1. Extent of PCL sprain is directly related to the amount of posterior subluxation of tibia with the knee at 90 flexion. A Grade 1 sprain has 1-5mm posterior translation, a grade 2 has 6-10mm posterior translation, and a grade 3 has >10mm posterior translation2.
    History obtained from the patient with suspicion of PCL injury will include some moment of trauma, whether it be the car accident, tackle, or fall with hyperextension. Symptoms associated with the injury include persistent pain and swelling immediately after injury, difficulty walking, feelings of knee instability, giving way of knee joint especially when walking downhill or descending stairs, and posterior knee pain1,2. However, these symptoms are often subtle or not noticed in isolated PCL injuries2.
    The most accurate physical exam test to diagnose a PCL injury is the posterior drawer test performed at 90 flexion. A posterior directed force applied to the proximal tibia will results in >10-15mm posterior translation of tibia in PCL injury2. Laxity at 0 flexion with varus and valgus stress to the knee can also be indicative of PCL injury. The posterior sag sign on physical exam suggest PCL injury. A posterior shift of the tibia of the involved knee is observed when a patient lies on back and flexes both knees to 902.
    First line imaging for complaints of knee pain are plan films. AP, supine lateral, and lateral stress views of the affected knee are all appropriate x-rays to order. While x-ray will not show injury to the PCL, they may show avulsion fractures due to acute rupture of the ligament1,2. The lateral stress view is becoming a more popular and frequently used tool in diagnosing PCL injury. The x-ray is taken with the knee flexed to 70 and stress applied to the anterior tibia. Asymmetric posterior tibial displacement of the two knees indicated PCL injury2.
    The confirmatory test for diagnosing PCL injury is an MRI of the affected knee. Normally the PCL appears as an uninterrupted dark band on MRI. If there has been injury to the ligament there will be disruption of this dark band evident on MRI3.
    There are both surgical and nonsurgical treatments for PCL injuries depending on the patient’s symptoms and degree of injury to the PCL and entire knee joint. Grade 1 and 2 sprains of the PCL are often treated nonsurgically. This treatment includes bracing for protected weightbearing and physical therapy for strengthening of the quadriceps. Patients are able to return to normal activity including sports in 2-4 weeks with successful conservative treatment1,2. In isolated grade 3 sprains in patients who are not athletes, nonsurgical treatment can also be used and includes immobilization in extension for 4 weeks followed by daily range of motion exercises and strengthening2.
    In combined ligamentous injuries with knee instability and isolated grade 2 or 3 PCL sprain with avulsion fracture, surgery is indicated to stabilize the knee joint. Because a torn PCL cannot be repaired, reconstruction of the ligament in the surgery of choice. Graft options for PCL reconstruction include autografts, allografts, and artificial material3. Allograft reconstruction is the most frequently used method of reconstruction and tissues include Achilles tendon, bone-patellar tendon-bone, hamstring, and anterior tibialis tendon3. Reconstruction surgery is often performed arthroscopically unless a large avulsion fracture needs repair, in which case an open approach is taken and the bone-tendon-bone graft is inlaid2,3. In the arthroscopic approach, a canal is drilled through the tibia starting on the anterolateral aspect, passing through the insertion point of the PCL and out the medial aspect of the femur. The graft is secured to the tibia with a screw, and held in place on the femur using a stabilizing button2,3. Post surgically patients are immobilized in extension. Physical therapy begins 1-4 weeks post surgery, and full recovery and return to normal activity requires between 6 and 12 months1,2,3. Surgical techniques for PCL reconstruction continue to improve, and generally patients have good success and full recovery.
    REFERENCES:
    1. Posterior Cruciate Ligament Injuries.” Posterior Cruciate Ligament Injuries-OrthoInfo – AAOS. American Orthopedic Society for Sports Medicine, 01 Feb. 2009. Web. 08 July 2017.
    2. Allen, Deborah, MD. “PCL Injury.” Orthobullets. Lineage Medical, 2011. Web. 08 July 2017.
    3. Orthopaedic Procedures | Posterior Cruciate Ligament Reconstruction.” One Health. The One Health Group, n.d. Web. 08 July 2017.

  175. Question 175 of 220
    175. Question

    A 23-year old married female G1 P1 presents five days postpartum, having had an uncomplicated vaginal birth. During the interview the patient often stares blankly at the clinician or the wall and is very hesitant with her responses, at times not responding at all. She does express worry about her newborn daughter, she is afraid something will happen to her and “she must protect her.” The patient describes herself as very preoccupied about consistently doing the right thing, including caring for her daughter “the right way.” She currently gets very little to no sleep out of concern for the child and for night feedings. She goes on to state there are computers at church that are recording her, she wants to be forgiven for her sins, and she thinks she’s in heaven. Her husband is present and describes a recent change since the birth of their daughter, reaffirming that his wife has become excessively religious and overly concerned about the care of her baby. She has been sleeping little, hyper-verbal at home, and is not her usual self.Her presenting condition is most likely which of the following?

    Correct

    Correct answer: (D) postpartum psychosis or brief psychotic disorder with peripartum onset. This patient is experiencing delusional thoughts of guilt or sin, paranoia, thought-blocking and appears internally preoccupied. Risk factors seen here for PP include parturition within one month and degraded sleep pattern. Collateral information from family and friends is oftentimes essential, and based on the husband’s report of hyper-verbal behavior it is possible that this is an initial presentation of (E), bipolar disorder with mania, however more time and outpatient treatment will be necessary to clench such a diagnosis. The patient may have elements of an obsessive-compulsive disorder (A) because of her fixated concern about caring for the infant, however would not be a leading diagnosis in this case at this time. There is no information in this scenario regarding the patient’s past history of a depressive disorder (B), and while it is normal for a new mother to lack sleep and feel concerned for the infant, the delusional thoughts and overprotectiveness are not a normal postpartum response (C).

    PP is a medical emergency due to the high risk of suicide and infanticide. Other risk factors include family history of postpartum depression or bipolar disorder, endocrine dysregulation, infection and sleep disturbances. Symptoms may include: delusions or hallucinations, irritability, hyperactivity, paranoia, rapid mood swings and difficulty concentrating.

    Discussion: Postpartum Psychosis
    Epidemiology
    Prevalence is 1-2 occurrences per 1,000 births. In females, a higher risk of psychosis is noted during the first few weeks, most notably between days 3-10 following birth than at any other time. A family history of mental illness, postpartum depression or psychosis increases the risk. For women with known psychiatric illnesses, the postnatal period is a period of high relapse. 1This is a medical emergency due to the increased risk of suicide and infanticide.
    Etiology: Postpartum psychosis is precipitated by childbirth, with a significant increase in risk for first time mothers. It is thought that it is due to physiological changes surrounding childbirth combined with genetic predisposition that are likely at play. Additionally, there may be endocrine, hormonal, immunological, circadian rhythm changes that contribute to the risk for PP; life events and psychosocial stressors are not considered to be triggers. 2
    A consideration in the onset of postpartum psychosis is that it may be a first break psychosis with mania in bipolar disorder that coincides with hormonal shifts after delivery. PP is a possible onset or recurrence of primary psychotic disorder such as schizophrenia. 1
    Presentation: Normally, PP presents in the postnatal period within the first month. It is often severe, with prominent confusion and fluctuating symptoms as mixed affective, schizoaffective, or manic states. 1
    Symptoms may include: Delusions or strange beliefs, Hallucinations (seeing or hearing things that aren’t there), Feeling very irritated, Hyperactivity, Decreased need for or inability to sleep, Paranoia and suspiciousness, Rapid mood swings, Difficulty communicating at times.
    Differential Diagnosis: Depressive episode with psychotic features, Schizophrenia, Mania or mixed episodes features, Obsessive compulsive disorder, Alcohol or substance withdrawal, Anti-NMDAR encephalitis, Delirium or psychosis due to including acute infection, peripartum blood loss and anemia, and exacerbation of predicting endocrine and/or autoimmune disease.

    Treatment
    Relatively little is known in the prevention and treatment, although investigations in the use of ECT, antipsychotics, mood stabilizers, hormones, and the beta-blocker propranolol have been documented.
    The treatment of postpartum psychosis ideally entails medical management with neuroleptic agents, mood stabilizers or antipsychotics. Often, inpatient psychiatric setting is warranted, and social services must be recruited and consideration given to electroconvulsive therapy (ECT) in refractory cases. The separation of mother and infant is important until the resolution of all symptoms of psychosis. Counseling services should be recruited once patient’s disorganized thought processes have been minimized, and the patient should be educated regarding their symptoms and possible recurrences, treatment and outcomes. 2
    Medications
    Postpartum psychosis is treated in the same fashion as other psychotic disorders. It is important that breastfeeding is be taken into consideration, and that the patient make an informed decision whether to continue nursing the infant. Psychotropic medications are not absolute contraindications in breastfeeding, however the infant must be monitored for sedation or extrapyramidal symptoms (EPS). EPS may include muscle spasms or contractions, akathisia (motor restlessness), parkinsonism (rigidity, bradykinesia, tremor), tardive dyskinesia (irregular, jerky movements). Aim for monotherapy.

    Monitor
    Note any exacerbation in symptoms
    Mental status to include thought content, though process, affect, and mood
    Endocrine and/or thyroid function
    Infection recurrence
    Side effects to medications

    Patient Counseling
    Rest or sleep when your baby sleeps. The housework will wait
    Seek out practical help in the house – friends, family or a volunteer
    Take short walks in the daytime
    Share responsibility for night feeds if you possibly can
    Talk to your Mental Health Team about how you are feeling. Taking medication at bedtime might reduce sleepiness in the day.

    References
    1. DynaMed Plus. Postpartum psychosis. http://www.dynamed.com/topics/dmp~AN~T116521/Postpartum-psychosis. Published 1995. Updated 2016 Sep 05 12:00 AM (ET). Accessed July 9, 2017, 2017.
    2. Bergink V, Rasgon N, Wisner KL. Postpartum psychosis: Madness, mania, and melancholia in motherhood. Am J Psychiatry. 2016;173(12):1179-1188. https://ezproxymcp.flo.org/login?url=http://ovidsp.ovid.com/ovidweb.cgi?T=JS&CSC=Y&NEWS=N&PAGE=fulltext&D=psyc13&AN=2017-04781-007; http://resolver.ebscohost.com/openurl?sid=OVID:psycdb&id=pmid:&id=doi:10.1176%2Fappi.ajp.2016.16040454&issn=0002-953X&isbn=&volume=173&issue=12&spage=1179&date=2016&title=The+American+Journal+of+Psychiatry&atitle=Postpartum+psychosis%3A+Madness%2C+mania%2C+and+melancholia+in+motherhood.&aulast=Bergink.
    3. Doucet S, Jones I, Letourneau N, Dennis C, Blackmore ER. Interventions for the prevention and treatment of postpartum psychosis: A systematic review. Archives of Women’s Mental Health. 2011;14(2):89-98. https://ezproxymcp.flo.org/login?url=http://ovidsp.ovid.com/ovidweb.cgi?T=JS&CSC=Y&NEWS=N&PAGE=fulltext&D=psyc8&AN=2011-08555-001; http://resolver.ebscohost.com/openurl?sid=OVID:psycdb&id=pmid:&id=doi:10.1007%2Fs00737-010-0199-6&issn=1434-1816&isbn=&volume=14&issue=2&spage=89&date=2011&title=Archives+of+Women%27s+Mental+Health&atitle=Interventions+for+the+prevention+and+treatment+of+postpartum+psychosis%3A+A+systematic+review.&aulast=Doucet.
    4. Action on Postpartum Psychosis. Insider guide – recovery after postpartum psychosis. 2013.

    Incorrect

    Correct answer: (D) postpartum psychosis or brief psychotic disorder with peripartum onset. This patient is experiencing delusional thoughts of guilt or sin, paranoia, thought-blocking and appears internally preoccupied. Risk factors seen here for PP include parturition within one month and degraded sleep pattern. Collateral information from family and friends is oftentimes essential, and based on the husband’s report of hyper-verbal behavior it is possible that this is an initial presentation of (E), bipolar disorder with mania, however more time and outpatient treatment will be necessary to clench such a diagnosis. The patient may have elements of an obsessive-compulsive disorder (A) because of her fixated concern about caring for the infant, however would not be a leading diagnosis in this case at this time. There is no information in this scenario regarding the patient’s past history of a depressive disorder (B), and while it is normal for a new mother to lack sleep and feel concerned for the infant, the delusional thoughts and overprotectiveness are not a normal postpartum response (C).

    PP is a medical emergency due to the high risk of suicide and infanticide. Other risk factors include family history of postpartum depression or bipolar disorder, endocrine dysregulation, infection and sleep disturbances. Symptoms may include: delusions or hallucinations, irritability, hyperactivity, paranoia, rapid mood swings and difficulty concentrating.

    Discussion: Postpartum Psychosis
    Epidemiology
    Prevalence is 1-2 occurrences per 1,000 births. In females, a higher risk of psychosis is noted during the first few weeks, most notably between days 3-10 following birth than at any other time. A family history of mental illness, postpartum depression or psychosis increases the risk. For women with known psychiatric illnesses, the postnatal period is a period of high relapse. 1This is a medical emergency due to the increased risk of suicide and infanticide.
    Etiology: Postpartum psychosis is precipitated by childbirth, with a significant increase in risk for first time mothers. It is thought that it is due to physiological changes surrounding childbirth combined with genetic predisposition that are likely at play. Additionally, there may be endocrine, hormonal, immunological, circadian rhythm changes that contribute to the risk for PP; life events and psychosocial stressors are not considered to be triggers. 2
    A consideration in the onset of postpartum psychosis is that it may be a first break psychosis with mania in bipolar disorder that coincides with hormonal shifts after delivery. PP is a possible onset or recurrence of primary psychotic disorder such as schizophrenia. 1
    Presentation: Normally, PP presents in the postnatal period within the first month. It is often severe, with prominent confusion and fluctuating symptoms as mixed affective, schizoaffective, or manic states. 1
    Symptoms may include: Delusions or strange beliefs, Hallucinations (seeing or hearing things that aren’t there), Feeling very irritated, Hyperactivity, Decreased need for or inability to sleep, Paranoia and suspiciousness, Rapid mood swings, Difficulty communicating at times.
    Differential Diagnosis: Depressive episode with psychotic features, Schizophrenia, Mania or mixed episodes features, Obsessive compulsive disorder, Alcohol or substance withdrawal, Anti-NMDAR encephalitis, Delirium or psychosis due to including acute infection, peripartum blood loss and anemia, and exacerbation of predicting endocrine and/or autoimmune disease.

    Treatment
    Relatively little is known in the prevention and treatment, although investigations in the use of ECT, antipsychotics, mood stabilizers, hormones, and the beta-blocker propranolol have been documented.
    The treatment of postpartum psychosis ideally entails medical management with neuroleptic agents, mood stabilizers or antipsychotics. Often, inpatient psychiatric setting is warranted, and social services must be recruited and consideration given to electroconvulsive therapy (ECT) in refractory cases. The separation of mother and infant is important until the resolution of all symptoms of psychosis. Counseling services should be recruited once patient’s disorganized thought processes have been minimized, and the patient should be educated regarding their symptoms and possible recurrences, treatment and outcomes. 2
    Medications
    Postpartum psychosis is treated in the same fashion as other psychotic disorders. It is important that breastfeeding is be taken into consideration, and that the patient make an informed decision whether to continue nursing the infant. Psychotropic medications are not absolute contraindications in breastfeeding, however the infant must be monitored for sedation or extrapyramidal symptoms (EPS). EPS may include muscle spasms or contractions, akathisia (motor restlessness), parkinsonism (rigidity, bradykinesia, tremor), tardive dyskinesia (irregular, jerky movements). Aim for monotherapy.

    Monitor
    Note any exacerbation in symptoms
    Mental status to include thought content, though process, affect, and mood
    Endocrine and/or thyroid function
    Infection recurrence
    Side effects to medications

    Patient Counseling
    Rest or sleep when your baby sleeps. The housework will wait
    Seek out practical help in the house – friends, family or a volunteer
    Take short walks in the daytime
    Share responsibility for night feeds if you possibly can
    Talk to your Mental Health Team about how you are feeling. Taking medication at bedtime might reduce sleepiness in the day.

    References
    1. DynaMed Plus. Postpartum psychosis. http://www.dynamed.com/topics/dmp~AN~T116521/Postpartum-psychosis. Published 1995. Updated 2016 Sep 05 12:00 AM (ET). Accessed July 9, 2017, 2017.
    2. Bergink V, Rasgon N, Wisner KL. Postpartum psychosis: Madness, mania, and melancholia in motherhood. Am J Psychiatry. 2016;173(12):1179-1188. https://ezproxymcp.flo.org/login?url=http://ovidsp.ovid.com/ovidweb.cgi?T=JS&CSC=Y&NEWS=N&PAGE=fulltext&D=psyc13&AN=2017-04781-007; http://resolver.ebscohost.com/openurl?sid=OVID:psycdb&id=pmid:&id=doi:10.1176%2Fappi.ajp.2016.16040454&issn=0002-953X&isbn=&volume=173&issue=12&spage=1179&date=2016&title=The+American+Journal+of+Psychiatry&atitle=Postpartum+psychosis%3A+Madness%2C+mania%2C+and+melancholia+in+motherhood.&aulast=Bergink.
    3. Doucet S, Jones I, Letourneau N, Dennis C, Blackmore ER. Interventions for the prevention and treatment of postpartum psychosis: A systematic review. Archives of Women’s Mental Health. 2011;14(2):89-98. https://ezproxymcp.flo.org/login?url=http://ovidsp.ovid.com/ovidweb.cgi?T=JS&CSC=Y&NEWS=N&PAGE=fulltext&D=psyc8&AN=2011-08555-001; http://resolver.ebscohost.com/openurl?sid=OVID:psycdb&id=pmid:&id=doi:10.1007%2Fs00737-010-0199-6&issn=1434-1816&isbn=&volume=14&issue=2&spage=89&date=2011&title=Archives+of+Women%27s+Mental+Health&atitle=Interventions+for+the+prevention+and+treatment+of+postpartum+psychosis%3A+A+systematic+review.&aulast=Doucet.
    4. Action on Postpartum Psychosis. Insider guide – recovery after postpartum psychosis. 2013.

  176. Question 176 of 220
    176. Question

    A 25-year-old male patients presents with erythematous, scaly annular plaques on his right hand and wrist. The rash is pruritic and “is annoying” according to the patient. It appeared three days ago and hasn’t gotten worse, but hasn’t gotten better. The patient denied trying any medications to relieve the itch and redness. The patient denies any new soaps, lotions, detergents, etc. The patient works as a computer engineer in a business office. The patient states he recently visited a friend’s farm to help with some work. What is the most likely diagnosis?

    Correct

    Correct answer: D. tinea manuum
    The correct answer here is D. tinea manuum, a tinea infection of the hand(s). Tinea infections can occur in any age and are almost always pruritic. Tinea manuum presents similarly to tinea corporis except it is located on the hand as opposed to the other parts of the body. Tinea manuum most commonly occurs unilaterally and annular. Most dermatophyte infections are erythematous with scaling involved.1 A common cause of tinea is contact with animals as well as soil.2 This patient admitted to working on his friend’s farm. Answer A. contact dermatitis could be considered with a rash on the hands due to most patients not realizing when they encounter an irritating substance. However, the patient does not recall using anything new and the rash is unilateral. Contact dermatitis also has a lower incidence in males.3 Answer B. psoriasis, has a variant that can occur on the hands. This is called pustular psoriasis and does not fit the description of the rash. The rash does not involved pustules which is the primary lesion in this type of psoriasis. The patient is also only 25 years of age and the most common age to be diagnosed with psoriasis is between 40-60.4 Answer C. dyshidrotic eczema can be considered because it leaves the hands scaly and itchy. It can also be erythematous although it does not present in an annular shape. Dyshidrotic eczema is almost always bilateral and is usually caused by sweating or excessive moisture on the hands.5 This seems unlikely with the patient’s job description. It is extremely pruritic and most commonly presents with vesicles.

    References
    1. Goldstein A, Goldstein B. Dermatophyte (tinea) infections. UpToDate. https://www.uptodate.com/contents/dermatophyte-tinea-infections?source=search_result&search=tinea manuum&selectedTitle=1~9. Published 2017. Accessed July 11, 2017.

    2. Hainer BL. Dermatophyte Infections. American Family Physician. http://www.aafp.org/afp/2003/0101/p101.html. Published January 1, 2003. Accessed July 11, 2017.

    3. Goldner R, Tuchinda P. Irritant Contact Dermatitis in Adults. Up. https://www.uptodate.com/contents/irritant-contact-dermatitis-in-adults?source=search_result&search=contact dermatitis&selectedTitle=1~150. Published 2017. Accessed July 11, 2017.

    4. Kalb R. Pustular psoriasis: pathogenesis, clinical manifestations, and diagnosis. UpToDate. https://www.uptodate.com/contents/pustular-psoriasis-pathogenesis-clinical-manifestations-and-diagnosis?source=search_result&search=psoriasis&selectedTitle=3~150. Published 2017. Accessed July 11, 2017.

    5. Adams D, Marks Jr. J. Acute palmoplantar eczema (dyshidrotic eczema). UpToDate. https://www.uptodate.com/contents/acute-palmoplantar-eczema-dyshidrotic-eczema?source=search_result&search=dyshidrotic eczema&selectedTitle=1~23. Published 2017. Accessed July 11, 2017.

    Incorrect

    Correct answer: D. tinea manuum
    The correct answer here is D. tinea manuum, a tinea infection of the hand(s). Tinea infections can occur in any age and are almost always pruritic. Tinea manuum presents similarly to tinea corporis except it is located on the hand as opposed to the other parts of the body. Tinea manuum most commonly occurs unilaterally and annular. Most dermatophyte infections are erythematous with scaling involved.1 A common cause of tinea is contact with animals as well as soil.2 This patient admitted to working on his friend’s farm. Answer A. contact dermatitis could be considered with a rash on the hands due to most patients not realizing when they encounter an irritating substance. However, the patient does not recall using anything new and the rash is unilateral. Contact dermatitis also has a lower incidence in males.3 Answer B. psoriasis, has a variant that can occur on the hands. This is called pustular psoriasis and does not fit the description of the rash. The rash does not involved pustules which is the primary lesion in this type of psoriasis. The patient is also only 25 years of age and the most common age to be diagnosed with psoriasis is between 40-60.4 Answer C. dyshidrotic eczema can be considered because it leaves the hands scaly and itchy. It can also be erythematous although it does not present in an annular shape. Dyshidrotic eczema is almost always bilateral and is usually caused by sweating or excessive moisture on the hands.5 This seems unlikely with the patient’s job description. It is extremely pruritic and most commonly presents with vesicles.

    References
    1. Goldstein A, Goldstein B. Dermatophyte (tinea) infections. UpToDate. https://www.uptodate.com/contents/dermatophyte-tinea-infections?source=search_result&search=tinea manuum&selectedTitle=1~9. Published 2017. Accessed July 11, 2017.

    2. Hainer BL. Dermatophyte Infections. American Family Physician. http://www.aafp.org/afp/2003/0101/p101.html. Published January 1, 2003. Accessed July 11, 2017.

    3. Goldner R, Tuchinda P. Irritant Contact Dermatitis in Adults. Up. https://www.uptodate.com/contents/irritant-contact-dermatitis-in-adults?source=search_result&search=contact dermatitis&selectedTitle=1~150. Published 2017. Accessed July 11, 2017.

    4. Kalb R. Pustular psoriasis: pathogenesis, clinical manifestations, and diagnosis. UpToDate. https://www.uptodate.com/contents/pustular-psoriasis-pathogenesis-clinical-manifestations-and-diagnosis?source=search_result&search=psoriasis&selectedTitle=3~150. Published 2017. Accessed July 11, 2017.

    5. Adams D, Marks Jr. J. Acute palmoplantar eczema (dyshidrotic eczema). UpToDate. https://www.uptodate.com/contents/acute-palmoplantar-eczema-dyshidrotic-eczema?source=search_result&search=dyshidrotic eczema&selectedTitle=1~23. Published 2017. Accessed July 11, 2017.

  177. Question 177 of 220
    177. Question

    A 36 yo male with history or IVDU comes into the ER with progressive weakness, arthralgias, and fever. Vital signs are as follows: T 101.7*F, BP 111/75, P130, respirations 19, O2 98% on RA. Positive physical exam findings include splinter hemorrhages, extremity weakness, and bilateral lower extremity 2+ edema. The rest of the exam is unremarkable. He has no previous hospital visits in the past 5 years. A sepsis work up is initiated, various cultures are ordered, and empiric broad-spectrum antibiotics are ordered. Which important diagnostic tests need to be done to assess the appropriate duration of antibiotic therapy?

    Correct

    Infective endocarditis (IE) was recognized as a complication of IVDU in the 1950s. It is estimated to occur at a rate of 2-4 cases per 1000 years of IVDU.1 Infective endocarditis occurs when bacteria stick to the inside of the heart and valves and replicate. There are many predisposing factors that increase the risk of IE. Injectable synthetic material causes direct damage to the endothelial tissue, drugs such as heroin may cause direct toxic damage to the valves themselves, users may inject organisms from their skin because of improper skin cleansing, and use of saliva as a diluent leads to an increased risk of infection with oropharyngeal flora.1 The most common organism isolated from IVDU associated IE is Staphylococcus aureus. Assessment of a patient with acute IE may sometimes be challenging. They usually present with an acute septic picture. Physical exam findings can include, but not be limited to; fever, weakness, signs of heart failure, a heart murmur, peripheral septic emboli leading to splinter hemorrhages, janeway lesions, conjunctival hemorrhages, or other end-organ damage. End-organ damage can manifest in bone pain, acute kidney injury with or without hematuria, altered mental status or a change in sensorium due to brain emboli, and rashes or lesions on the skin due to septic occlusion of the distal capillaries.1 Diagnosis of IE can be made using the Duke Criteria. The diagnostic tests that need to be done for the Duke Criteria are blood cultures and an echocardiogram. The other components of the diagnostic criteria are a fever >100.4*F, immunologic phenomena (glomerulonephritis, osler nodes, roth spots), vascular phenomena (arterial emboli, pulmonary infarct, janeway lesions, intracranial bleed, conjunctival hemorrhages), and predisposing conditions like IVDU can all be made clinically.2 Management of IE is antibiotic therapy dependent on the susceptibility profile of the organism isolated in blood cultures. Blood cultures may take a few days to come back so broad-spectrum antibiotics to cover gram-positive organisms including MRSA should be initiated as soon as a presumptive diagnosis is made. If valve or heart wall involvement is extensive or an abscess is formed, valve replacement therapy and tissue debridement may need to be used in the treatment process. In cases of the IE with Staphylococcus aureus, treatment should be for 4-6 weeks.3

    Explanation:
    Blood cultures are an option for all 4 choices provided. In suspected IE it is one of the first tests that should be ordered before broad-spectrum antibiotics are initiated. Blood cultures are important because it allows us to isolate an organism and find its susceptibility profile. Duration of treatment for IE is dependent on organism isolated. According to the Infectious Disease Society of America, Strep spp. can be treated for 4 weeks, while Staph spp. should be treated for 4-6 weeks.3 The next part of the question asks about imaging modalities. The main concept behind this question is that it is important to differentiate between sepsis and IE with potential vegetation in the heart or the heart valves. The treatment times for sepsis can range from 2-4 weeks while the treatment time for IE will be a minimum of 4 weeks. A chest x-ray will not provide us with any clues as to whether or not there is vegetation on one of the heart valves. An abdominal ultrasound would not be used in this scenario because it would provide little diagnostic utility for our patient’s condition. A trans-thoracic echo could potentially be done, but a trans-esophageal echo provides better visualization of the areas of the heart that could be affected, including the heart valves. A trans-esophageal echo has a high sensitivity for picking up heart valve vegetation than a trans-thoracic echo.4 The correct answer is B, blood cultures and a trans-esophageal echo should be performed to determine the duration of antibiotic treatment.

    References
    1. Sexton, MD D, Chu, MD, MHS V. Infective endocarditis in injection drug users. Uptodatecom. 2017. Available at: https://www.uptodate.com/contents/infective-endocarditis-in-injection-drug-users?source=search_result&search=ivdu%20endocarditis&selectedTitle=1~150#H6. Accessed July 5, 2017.
    2. Endocarditis Diagnostic Criteria (Duke Criteria). Referencemedscapecom. 2017. Available at: http://reference.medscape.com/calculator/endocarditis-diagnostic-criteria-duke. Accessed July 5, 2017.
    3. Baddour L. Infective Endocarditis: Diagnosis, Antimicrobial Therapy, and Management of Complications: A Statement for Healthcare Professionals From the Committee on Rheumatic Fever, Endocarditis, and Kawasaki Disease, Council on Cardiovascular Disease in the Young, and the Councils on Clinical Cardiology, Stroke, and Cardiovascular Surgery and Anesthesia, American Heart Association: Endorsed by the Infectious Diseases Society of America. Circulation. 2005;111(23):e394-e434. doi:10.1161/circulationaha.105.165564.
    4. Evangelista A. Echocardiography in infective endocarditis. Heart. 2004;90(6):614-617. doi:10.1136/hrt.2003.029868.

    Incorrect

    Infective endocarditis (IE) was recognized as a complication of IVDU in the 1950s. It is estimated to occur at a rate of 2-4 cases per 1000 years of IVDU.1 Infective endocarditis occurs when bacteria stick to the inside of the heart and valves and replicate. There are many predisposing factors that increase the risk of IE. Injectable synthetic material causes direct damage to the endothelial tissue, drugs such as heroin may cause direct toxic damage to the valves themselves, users may inject organisms from their skin because of improper skin cleansing, and use of saliva as a diluent leads to an increased risk of infection with oropharyngeal flora.1 The most common organism isolated from IVDU associated IE is Staphylococcus aureus. Assessment of a patient with acute IE may sometimes be challenging. They usually present with an acute septic picture. Physical exam findings can include, but not be limited to; fever, weakness, signs of heart failure, a heart murmur, peripheral septic emboli leading to splinter hemorrhages, janeway lesions, conjunctival hemorrhages, or other end-organ damage. End-organ damage can manifest in bone pain, acute kidney injury with or without hematuria, altered mental status or a change in sensorium due to brain emboli, and rashes or lesions on the skin due to septic occlusion of the distal capillaries.1 Diagnosis of IE can be made using the Duke Criteria. The diagnostic tests that need to be done for the Duke Criteria are blood cultures and an echocardiogram. The other components of the diagnostic criteria are a fever >100.4*F, immunologic phenomena (glomerulonephritis, osler nodes, roth spots), vascular phenomena (arterial emboli, pulmonary infarct, janeway lesions, intracranial bleed, conjunctival hemorrhages), and predisposing conditions like IVDU can all be made clinically.2 Management of IE is antibiotic therapy dependent on the susceptibility profile of the organism isolated in blood cultures. Blood cultures may take a few days to come back so broad-spectrum antibiotics to cover gram-positive organisms including MRSA should be initiated as soon as a presumptive diagnosis is made. If valve or heart wall involvement is extensive or an abscess is formed, valve replacement therapy and tissue debridement may need to be used in the treatment process. In cases of the IE with Staphylococcus aureus, treatment should be for 4-6 weeks.3

    Explanation:
    Blood cultures are an option for all 4 choices provided. In suspected IE it is one of the first tests that should be ordered before broad-spectrum antibiotics are initiated. Blood cultures are important because it allows us to isolate an organism and find its susceptibility profile. Duration of treatment for IE is dependent on organism isolated. According to the Infectious Disease Society of America, Strep spp. can be treated for 4 weeks, while Staph spp. should be treated for 4-6 weeks.3 The next part of the question asks about imaging modalities. The main concept behind this question is that it is important to differentiate between sepsis and IE with potential vegetation in the heart or the heart valves. The treatment times for sepsis can range from 2-4 weeks while the treatment time for IE will be a minimum of 4 weeks. A chest x-ray will not provide us with any clues as to whether or not there is vegetation on one of the heart valves. An abdominal ultrasound would not be used in this scenario because it would provide little diagnostic utility for our patient’s condition. A trans-thoracic echo could potentially be done, but a trans-esophageal echo provides better visualization of the areas of the heart that could be affected, including the heart valves. A trans-esophageal echo has a high sensitivity for picking up heart valve vegetation than a trans-thoracic echo.4 The correct answer is B, blood cultures and a trans-esophageal echo should be performed to determine the duration of antibiotic treatment.

    References
    1. Sexton, MD D, Chu, MD, MHS V. Infective endocarditis in injection drug users. Uptodatecom. 2017. Available at: https://www.uptodate.com/contents/infective-endocarditis-in-injection-drug-users?source=search_result&search=ivdu%20endocarditis&selectedTitle=1~150#H6. Accessed July 5, 2017.
    2. Endocarditis Diagnostic Criteria (Duke Criteria). Referencemedscapecom. 2017. Available at: http://reference.medscape.com/calculator/endocarditis-diagnostic-criteria-duke. Accessed July 5, 2017.
    3. Baddour L. Infective Endocarditis: Diagnosis, Antimicrobial Therapy, and Management of Complications: A Statement for Healthcare Professionals From the Committee on Rheumatic Fever, Endocarditis, and Kawasaki Disease, Council on Cardiovascular Disease in the Young, and the Councils on Clinical Cardiology, Stroke, and Cardiovascular Surgery and Anesthesia, American Heart Association: Endorsed by the Infectious Diseases Society of America. Circulation. 2005;111(23):e394-e434. doi:10.1161/circulationaha.105.165564.
    4. Evangelista A. Echocardiography in infective endocarditis. Heart. 2004;90(6):614-617. doi:10.1136/hrt.2003.029868.

  178. Question 178 of 220
    178. Question

    A 74-year-old male presents to a trauma center after sustaining flame burns to his abdomen (lower anterior trunk) and his entire left upper extremity from an engine explosion. Exam reveals splotchy red, slowly blanching burns to the abdomen and left arm with areas of white that are painless. Overall pain is severe at 10 out of 10. Patient denies headaches, nausea, vomiting, shortness of breath, fever or chills. What is the burn classification and total body surface area affected in this patient?

    Correct

    Answer is A. 18%, 2nd degree deep partial thickness

    Discussion
    Burns are fairly common, accounting for the 4th leading cause of trauma worldwide3. There is an increased prevalence of burns in lower socioeconomic populations and most burns occur in the home3. The three major causes of burns are by heat including flame or scalding liquid, chemicals, or electricity.
    Burns are categorized by the depth of skin and tissue evolvement. Traditionally, burns were divided into four categories 1st, 2nd, 3rd, and 4th degree burns. Today, we have transitioned into categorizing based on thickness; superficial, partial thickness, and full thickness2. Superficial is defined as affecting only the epidermis, appearing red, easily blanchable and patients have mild to moderate pain. Partial thickness can be further broken into superficial and deep. Superficial partial thickness involves the entire epidermis and only the superficial dermis. These burns appear erythematous, moist, with blisters, and patients usually have severe pain. Deep partial thickness involves the epidermis and most of the dermis with some skin appendages destroyed. These burns present as white and erythematous areas, dry, waxy and decreased blanching to pressure. In certain areas, usually where the skin appears white, their pain will be decreased. Lastly, full thickness burns affect all layers where the appendages are completely destroyed. Patients will present with white, charred skin that are dry and leathery and are not blanchable. These patient will usually be without pain4.
    It is important to obtain a full history and physical from this patient population, specifically the cause and length of exposure, degree of pain, and any other medical conditions that may worsen the prognosis or change the treatment3. If the burn was caused by fire, it is crucial to assess for symptoms of smoke inhalation. Symptoms include shortness of breath, sore throat, soot around the nares, or singed facial hair3. It is important to assess the depth of burn as described above and estimate the total area of the burn3. The rules of nine is a universal tool to calculate total body surface area and further assisting in the treatment plans for burns. The body is divided in multiples of 9, each arm 9% (4.5% front and 4.5% back), the head both front and back totaling 9%, Front of the torso 18% (upper 9%, lower 9%), Back 18%, each leg 18% (front 9%, back 9%), and genitalia 1%4.
    Diagnostics studies are dependent on the mechanism and duration of exposure. One should obtain a chest x-ray if there is question of smoke inhalation as well as carboxyhemoglobin levels and ABGs3. If the burn is of electric nature an EKG and CK levels are important to rule out any cardiovascular injury. Baseline labs are also an important tool to monitor and compare throughout the treatment process3.
    The treatment of burns is very complex and dependent on the depth and total body surface area affected. Fluid resuscitation is crucial in these patients so much so a formula was created for specifically for burn treatment. The Parkland formula is an equation to calculate the total fluid requirements for a 24-hour period. Parkland Formula: 4ml x total body surface area % x body weight (kg). Once calculated, 50% of this total is given within the first eight hours and the rest is given in the remaining 16 hours4.
    Another important aspect in managing these patient is pain control. Patients present in severe pain and require extensive and rapid pain relief. Fentanyl or Morphine are the analgesic drugs of choice for burn treatment3. Wound management is dependent on burn depth. All burns should be thoroughly cleaned with large amounts of water1. Superficial and superficial partial thickness burns heal quickly with wet dressings, silver sulfadiazine antibiotic cream or xeroform wound dressing1, 2. For deep partial thickness and full thickness burns, surgical intervention is usually necessary for treatment as these patients most likely require skin grafting1.

    References

    1.Centers WC. Partial thickness burns. Partial Thickness Burns. http://www.woundcarecenters.org/article/wound-types/partial-thickness-burns. Accessed July 12, 2017.
    2.Davidge K. Older Adults and Burns. Medscape. http://www.medscape.com/viewarticle/579832_3. Accessed July 12, 2017.
    3.DynaMed Plus [Internet]. Ipswich (MA): EBSCO Information Services. 1995 – . Record No. 902902, Burns – emergency management; [updated 2017 Jun 22, cited place cited date here]; [about 4 screens]. Available from http://www.dynamed.com/login.aspx?direct=true&site=DynaMed&id=902902. Registration and login required.

    4. Grabb W, Smith JW, Thorne CH, Chung KC, Gosain A. Grabb and Smiths plastic surgery. Philadelphia: Wolters Kluwer/Lippincott Williams & Wilkins Health; 2014

    Incorrect

    Answer is A. 18%, 2nd degree deep partial thickness

    Discussion
    Burns are fairly common, accounting for the 4th leading cause of trauma worldwide3. There is an increased prevalence of burns in lower socioeconomic populations and most burns occur in the home3. The three major causes of burns are by heat including flame or scalding liquid, chemicals, or electricity.
    Burns are categorized by the depth of skin and tissue evolvement. Traditionally, burns were divided into four categories 1st, 2nd, 3rd, and 4th degree burns. Today, we have transitioned into categorizing based on thickness; superficial, partial thickness, and full thickness2. Superficial is defined as affecting only the epidermis, appearing red, easily blanchable and patients have mild to moderate pain. Partial thickness can be further broken into superficial and deep. Superficial partial thickness involves the entire epidermis and only the superficial dermis. These burns appear erythematous, moist, with blisters, and patients usually have severe pain. Deep partial thickness involves the epidermis and most of the dermis with some skin appendages destroyed. These burns present as white and erythematous areas, dry, waxy and decreased blanching to pressure. In certain areas, usually where the skin appears white, their pain will be decreased. Lastly, full thickness burns affect all layers where the appendages are completely destroyed. Patients will present with white, charred skin that are dry and leathery and are not blanchable. These patient will usually be without pain4.
    It is important to obtain a full history and physical from this patient population, specifically the cause and length of exposure, degree of pain, and any other medical conditions that may worsen the prognosis or change the treatment3. If the burn was caused by fire, it is crucial to assess for symptoms of smoke inhalation. Symptoms include shortness of breath, sore throat, soot around the nares, or singed facial hair3. It is important to assess the depth of burn as described above and estimate the total area of the burn3. The rules of nine is a universal tool to calculate total body surface area and further assisting in the treatment plans for burns. The body is divided in multiples of 9, each arm 9% (4.5% front and 4.5% back), the head both front and back totaling 9%, Front of the torso 18% (upper 9%, lower 9%), Back 18%, each leg 18% (front 9%, back 9%), and genitalia 1%4.
    Diagnostics studies are dependent on the mechanism and duration of exposure. One should obtain a chest x-ray if there is question of smoke inhalation as well as carboxyhemoglobin levels and ABGs3. If the burn is of electric nature an EKG and CK levels are important to rule out any cardiovascular injury. Baseline labs are also an important tool to monitor and compare throughout the treatment process3.
    The treatment of burns is very complex and dependent on the depth and total body surface area affected. Fluid resuscitation is crucial in these patients so much so a formula was created for specifically for burn treatment. The Parkland formula is an equation to calculate the total fluid requirements for a 24-hour period. Parkland Formula: 4ml x total body surface area % x body weight (kg). Once calculated, 50% of this total is given within the first eight hours and the rest is given in the remaining 16 hours4.
    Another important aspect in managing these patient is pain control. Patients present in severe pain and require extensive and rapid pain relief. Fentanyl or Morphine are the analgesic drugs of choice for burn treatment3. Wound management is dependent on burn depth. All burns should be thoroughly cleaned with large amounts of water1. Superficial and superficial partial thickness burns heal quickly with wet dressings, silver sulfadiazine antibiotic cream or xeroform wound dressing1, 2. For deep partial thickness and full thickness burns, surgical intervention is usually necessary for treatment as these patients most likely require skin grafting1.

    References

    1.Centers WC. Partial thickness burns. Partial Thickness Burns. http://www.woundcarecenters.org/article/wound-types/partial-thickness-burns. Accessed July 12, 2017.
    2.Davidge K. Older Adults and Burns. Medscape. http://www.medscape.com/viewarticle/579832_3. Accessed July 12, 2017.
    3.DynaMed Plus [Internet]. Ipswich (MA): EBSCO Information Services. 1995 – . Record No. 902902, Burns – emergency management; [updated 2017 Jun 22, cited place cited date here]; [about 4 screens]. Available from http://www.dynamed.com/login.aspx?direct=true&site=DynaMed&id=902902. Registration and login required.

    4. Grabb W, Smith JW, Thorne CH, Chung KC, Gosain A. Grabb and Smiths plastic surgery. Philadelphia: Wolters Kluwer/Lippincott Williams & Wilkins Health; 2014

  179. Question 179 of 220
    179. Question

    A 65 year-old African American male presents to your office with 3 months history of progressive fatigue and back pain. Physical exam reveals no focal neurological findings. Routine lab work reveals serum Calcium of 13.1 mg/dL (nl 8.5-10.2), Creatinine of 3.5 mg/dL (nl 0.6-1.2) and Hgb of 9.9 g/dL (nl 13.5-17.5). Further work-up for new onset renal insufficiency reveals monoclonal spike in IgG kappa in both serum and urine. Skeletal survey reveals multiple lytic lesions consistent with Multiple Myeloma. Which of the following cells is expected to be elevated in the Bone Marrow biopsy that is performed as a part of the work up?

    Correct

    Correct Answer: E. Plasma Cells. Multiple Myeloma is a plasma cell dyscrasia which involves abnormal proliferation of a monoclonal population of plasma cells that commonly secrete high levels of monoclonal immunoglobulins (mostly IgG or IgA). Classic presentation of MM involves the acronym CRAB (high Calcium, Renal dysfunction, Anemia and Bone lytic lesions). Bone Marrow biopsy reveals >10% plasma cells. Options 1. NK cells, 2. Neutrophils, 3. T-lymphocytes are not associated with MM. Option 4. B-lymphocytes; Plasma cells are derived from B-lymphocytes. However, B-lymphocyte counts are not elevated in the Bone Marrow evaluation.
    Discussion
    Epidemiology: Multiple Myeloma (MM) is a plasma cell disorder where there is an abnormal proliferation of monoclonal population of plasma cells that secrete monoclonal immunoglobulins. It is the second most common hematological malignancy in US. It is a disease of an older population (median age of onset is 69 years). It affects men slightly more common than women and African Americans have twice the incidence and mortality as Caucasians.
    Etiology: MM is a malignancy of terminally differentiated B-lymphocytes; the Plasma Cells, that is not completely understood. Some of the known predisposing factors includes ionizing radiation, environmental exposures (pesticides, herbicides and occupational exposures), MGUS, hereditary and viruses (increased risk in HIV).
    Pathophysiology: A proposed model of disease progression: normal plasma cell->monoclonal gammopathy of undetermined significance (MGUS) ->MM. The major mutations associated with poor prognosis includes deletion (chromosome 13) and translocation (4; 14). Translocation (14;16) has conflicting data vs translocation (11;14) is associated with improved survival. The overall progression of MGUS toward plasma cell malignancy is at the rate of 1% per year with cumulative probability of 12 % at 10 years. Cytokines like Interleukin-6 (IL-6) is a growth factor secreted by MM cells that promotes its survival. It also secretes IL- 1 and soluble IL-6 receptor that activates osteoclast promoting bone resorption. Basic fibroblast growth factors (bFGF) and vascular endothelia growth factors (VEGF) are potent and synergistic mediators of angiogenesis that are elevated in MM and are used as markers for plasma Cell labeling index (PCLI) for evaluation of proliferative capacity of MM cells.
    Historical and physical finding: Major symptoms of MM can be remembered by acronym CRAB. HyperCalcemia (Ca of >11.5) is a presenting feature in 30 % of the patients. Renal Dysfunction (Serum Creatinine of > 1.5) seen in up to 31 % of the cases and is primarily associated with tubular damage by monoclonal light chain protein. Anemia (Hgb Diagnostic Labs/Imagings: Common workup for MM includes CBC with differential, calcium, serum creatinine, LDH, albumin, β₂ macroglobulin, Bone Marrow Aspiration with cytogenetics and PCLI, chromosomal analysis with FISH, 24 urine for protein and UPEP with immunofixation, SPEP with immunofixation, Kappa and Lambda light chains and Skeletal survey.
    Management/Treatment: Despite recent advances in treatment options, MM remains incurable. Goal of therapy remains disease control, improve quality of life and prolong survival. High dose chemotherapy followed by autologous Hematopoietic Stem cell Transplant remains the treatment of choice. Factors such as initial response to therapy, age, performance status, comorbidities, renal and pulmonary functions and active infection status defines their eligibility and success to transplant.
    The patient had a classic presentation of symptomatic Myeloma with elevated monoclonal proteins in both urine and serum. His bone marrow biopsy at the time of initial diagnosis revealed 80 % plasma cells with plasmacytoma in the bones. Also, he had positive findings for renal impairment, anemia, bone involvement and recurrent infections.
    References
    1.Clinical features, laboratory manifestations, and diagnosis of multiple myeloma. https://www.uptodate.com/contents/clinical-features-laboratory-manifestations-and-diagnosis-of-multiple-myeloma?source=search_result&search=multiple myeloma&selectedTitle=1~150#H28. Accessed July 10, 2017.
    2. Anderson KC. Multiple Myeloma: A Clinical Overview. Cancer Network . November 2011:1-8. file:///C:/Users/mamta/Downloads/Cancer_Network_-_Multiple_Myeloma_A_Clinical_Overview_-_2017-05-11.pdf. Accessed July 7, 2017.
    3. Multiple myeloma. HomePage. https://bethematch.org/for-patients-and-families/learning-about-your-disease/multiple-myeloma/. Accessed July 10, 2017.
    4. What Is Multiple Myeloma? American Cancer Society. https://www.cancer.org/cancer/multiple-myeloma/about/what-is-multiple-myeloma.
    5. Guidelines. International Myeloma Working Group. http://imwg.myeloma.org/category/guidelines-2/. Accessed July 10, 2017. html. Accessed July 10, 2017.

    Incorrect

    Correct Answer: E. Plasma Cells. Multiple Myeloma is a plasma cell dyscrasia which involves abnormal proliferation of a monoclonal population of plasma cells that commonly secrete high levels of monoclonal immunoglobulins (mostly IgG or IgA). Classic presentation of MM involves the acronym CRAB (high Calcium, Renal dysfunction, Anemia and Bone lytic lesions). Bone Marrow biopsy reveals >10% plasma cells. Options 1. NK cells, 2. Neutrophils, 3. T-lymphocytes are not associated with MM. Option 4. B-lymphocytes; Plasma cells are derived from B-lymphocytes. However, B-lymphocyte counts are not elevated in the Bone Marrow evaluation.
    Discussion
    Epidemiology: Multiple Myeloma (MM) is a plasma cell disorder where there is an abnormal proliferation of monoclonal population of plasma cells that secrete monoclonal immunoglobulins. It is the second most common hematological malignancy in US. It is a disease of an older population (median age of onset is 69 years). It affects men slightly more common than women and African Americans have twice the incidence and mortality as Caucasians.
    Etiology: MM is a malignancy of terminally differentiated B-lymphocytes; the Plasma Cells, that is not completely understood. Some of the known predisposing factors includes ionizing radiation, environmental exposures (pesticides, herbicides and occupational exposures), MGUS, hereditary and viruses (increased risk in HIV).
    Pathophysiology: A proposed model of disease progression: normal plasma cell->monoclonal gammopathy of undetermined significance (MGUS) ->MM. The major mutations associated with poor prognosis includes deletion (chromosome 13) and translocation (4; 14). Translocation (14;16) has conflicting data vs translocation (11;14) is associated with improved survival. The overall progression of MGUS toward plasma cell malignancy is at the rate of 1% per year with cumulative probability of 12 % at 10 years. Cytokines like Interleukin-6 (IL-6) is a growth factor secreted by MM cells that promotes its survival. It also secretes IL- 1 and soluble IL-6 receptor that activates osteoclast promoting bone resorption. Basic fibroblast growth factors (bFGF) and vascular endothelia growth factors (VEGF) are potent and synergistic mediators of angiogenesis that are elevated in MM and are used as markers for plasma Cell labeling index (PCLI) for evaluation of proliferative capacity of MM cells.
    Historical and physical finding: Major symptoms of MM can be remembered by acronym CRAB. HyperCalcemia (Ca of >11.5) is a presenting feature in 30 % of the patients. Renal Dysfunction (Serum Creatinine of > 1.5) seen in up to 31 % of the cases and is primarily associated with tubular damage by monoclonal light chain protein. Anemia (Hgb Diagnostic Labs/Imagings: Common workup for MM includes CBC with differential, calcium, serum creatinine, LDH, albumin, β₂ macroglobulin, Bone Marrow Aspiration with cytogenetics and PCLI, chromosomal analysis with FISH, 24 urine for protein and UPEP with immunofixation, SPEP with immunofixation, Kappa and Lambda light chains and Skeletal survey.
    Management/Treatment: Despite recent advances in treatment options, MM remains incurable. Goal of therapy remains disease control, improve quality of life and prolong survival. High dose chemotherapy followed by autologous Hematopoietic Stem cell Transplant remains the treatment of choice. Factors such as initial response to therapy, age, performance status, comorbidities, renal and pulmonary functions and active infection status defines their eligibility and success to transplant.
    The patient had a classic presentation of symptomatic Myeloma with elevated monoclonal proteins in both urine and serum. His bone marrow biopsy at the time of initial diagnosis revealed 80 % plasma cells with plasmacytoma in the bones. Also, he had positive findings for renal impairment, anemia, bone involvement and recurrent infections.
    References
    1.Clinical features, laboratory manifestations, and diagnosis of multiple myeloma. https://www.uptodate.com/contents/clinical-features-laboratory-manifestations-and-diagnosis-of-multiple-myeloma?source=search_result&search=multiple myeloma&selectedTitle=1~150#H28. Accessed July 10, 2017.
    2. Anderson KC. Multiple Myeloma: A Clinical Overview. Cancer Network . November 2011:1-8. file:///C:/Users/mamta/Downloads/Cancer_Network_-_Multiple_Myeloma_A_Clinical_Overview_-_2017-05-11.pdf. Accessed July 7, 2017.
    3. Multiple myeloma. HomePage. https://bethematch.org/for-patients-and-families/learning-about-your-disease/multiple-myeloma/. Accessed July 10, 2017.
    4. What Is Multiple Myeloma? American Cancer Society. https://www.cancer.org/cancer/multiple-myeloma/about/what-is-multiple-myeloma.
    5. Guidelines. International Myeloma Working Group. http://imwg.myeloma.org/category/guidelines-2/. Accessed July 10, 2017. html. Accessed July 10, 2017.

  180. Question 180 of 220
    180. Question

    An 81 year-old male presents to the emergency room with a 10-day history of fever, chills, night sweats, SOB, weakness and fatigue. He first noticed the symptoms after returning from vacation with his family on Cape Cod in Massachusetts. He presents with a 105.3° temperature, 90/50 BP, 115 bpm and 80% O2 sat. Physical exam reveals minor crackles in bilateral basilar lungs and a systolic ejection murmur heard best at the apex. Routine lab work reveals a hemolytic anemia, thrombocytopenia and a normal WBC count. What is the most likely cause of his current state?

    Correct

    Answer is C. Babesiosis is a tick-borne disease that is carried by the same vector as Lyme disease and found classically in the northeastern portion of the United States like Cape Cod, Massachusetts. Patients infected with Babesiosis usually present acutely with malaise, weakness, fatigue and fevers upwards of 105°F. Fever in these patients is typically followed by chills and night sweats. Hemolytic anemia and thrombocytopenia are common features of this disease and WBC counts can be low, normal or high. The Patient’s acute onset with flu-like symptoms and high-grade fever suggest an infection that effectively rules out Pulmonary Embolism. Although PE patients often present with Tachycardia, SOB, weakness, fatigue, low O2 Sat and possibly lung finding on physical examination, there is no connection between PE causing thrombocytopenia or hemolytic anemia. Endocarditis would cause high fevers, flu-like symptoms and a heart murmur, however, this patient has a history of valvular disease that would explain the murmur and Endocarditis has no correlation to hemolytic anemias or thrombocytopenia. Patients who develop endocarditis typically have a point of entry into the bloodstream such as IV drug use, recent dental surgery or open wounds which this patient did not have. Rocky Mountain spotted fever is another tick-borne illness that presents with similar symptoms and findings including thrombocytopenia and hemolytic anemia, however it is not commonly found in the northeastern United States and classically has a palmar rash associated with its presentation.

    Discussion:
    Babesiosis is an infectious parasitic protozoan that is predominantly found in the Northeastern and upper Midwestern regions of the United States.1 Other species are found in Europe; however, all species affect all ethnicities equally.1 The most common species of babesiosis is B. microti which is what our conversation will focus on. There are rarer species of Babesiosis such as B. divergens which can cause fulminant illness and are considered a medical emergency requiring blood exchange transfusion.1
    Babesiosis is carried by tick vectors most commonly Ixodes scapularis ticks (aka the Black-Legged Deer Ticks).1 Ixodes scapularis have 3 stages of development from larva to nymph to adult which all require what is called a “blood meal” to advance to the next stage.1 They obtain the parasite through multiple feedings from hosts and the protozoa multiplies in the gut wall of the tick before concentrating in the salivary glands.1 When the next feeding occurs, the parasite is transferred from the tick vector to a new host entering the bloodstream during the bite as mature trophozoites.1 The parasite infects RBCs directly and once in the cell, the trophozoites undergo asexual budding and replicated into 2 to 4 merozoites which leave the cell through direct lysis of the membrane.1 Alterations in RBC membranes can lead to excess fluid which manifests as pulmonary edema and possibly ARDS.1
    Pathophysiological manifestations of the disease can be seen as hemolytic anemia due to direct parasite-mediated lysis of red blood cells in the circulation. This leads to a low hematocrit, elevated lactate dehydrogenase, low hemoglobin and elevated total bilirubin.1 As a result of lysis, the RBC fragments can cause capillary blockage and microvascular stasis which can lead to serious CNS complications, liver, splenic or renal involvement.1
    Presentation with Babesiosis infections range from being asymptomatic to severely unstable and possibly fatal.1 The severity and course of the infection mainly depend on the species of Babesiosis, the patient’s immune status and the patient’s co-morbidities. Most patients typically become symptomatic with the first 6 weeks following a bite from an infected tick in an endemic area but more precisely it has been determined that the median interval for symptom onset is 37 days.1 Patients typically present with gradual onset of fatigue, weakness, malaise and fevers that can reach dangerously high fevers such as 105.6°F.1 Following the fever, patients will develop chills and night sweats.1 Less commonly symptoms will include headache, nausea, vomiting, diarrhea, anorexia, arthralgia, myalgias, neck stiffness, shortness of breath, dry cough, weight loss or dark urine.1
    Physical examination may reveal mild organomegaly, specifically splenomegaly, but lymphadenopathy is absent.1 As previously stated, hemolytic anemia is present in all patients but thrombocytopenia is also common.1 White blood cell counts have no significance on evaluation as they can range from decreased to normal to elevated.1
    Patients with severe babesiosis require hospitalization and are susceptible to potential complications include acute respiratory distress syndrome, disseminated intravascular coagulation, congestive heart failure, and renal failure.1 These usually occur when the anemia becomes sever and the parasitemia is high. In rare circumstances, high fever can cause a hyperadrenergic state leading to excessive orthostatic tachycardia, which in someone with preexisting heart disease, such as our individual, can precipitate lethal arrhythmias.1
    The definitive diagnosis of babesiosis should be made by examining blood with direct light microscopy and Wright-Giemsa Stain.2 Specifically, examination of thin blood smears is necessary as babesial parasites may be too small to be seen using thick blood smears. Babesiosis most commonly are seen as a pale blue ring with two red chromatic dots.2 You also can occasionally see the parasites arranged in tetrads known as “Maltese Cross.1”
    Polymerase chain reaction (PCR) is also useful for diagnosis especially in those with a low-level of parasitemia and can be used to detect any Babesia DNA levels in blood that show no parasites on blood smear.1 One downfall of PCR is that the DNA detected does not always equate to live parasites and can be detected in blood for up to 26 months post-infection.1
    Indirect immunofluorescence can be used in rare cases with high suspicion of babesiosis and negative blood smears and PCR.1 These cases are seen more in blood donors that have been implicated in transfusion-acquired disease or in asymptomatic patients who have cleared the disease without treatment but may have complications following the infection.1
    The diagnosis should be considered in patients who present with flu-like symptoms and who either live in or have traveled recently to endemic areas. Also, patients with symptom who have been previously diagnosed with Lyme disease, Anaplasmosis, Powassan or other tick-borne illnesses should also be suspected.
    Treatment should be initiated in any symptomatic patients with parasitemia confirmed via blood smear or PCR. If a patient is asymptomatic but has a confirmed diagnosis of babesiosis, treatment is not necessary until symptomatic. Asymptomatic individuals who have persistent infection for > 3months as confirmed by blood smear or PCR should also consider treatment.
    The most effective therapy is the use of antimicrobial agents, specifically atovaquone plus azithromycin or quinine plus clindamycin.3 Both options are equally efficacious but Clindamycin plus quinine is typically reserved for severe cases (requiring hospitalization) or those who are immunocompromised.1 The typical regimen is to take both drugs orally for 7 to 10 days.3 Atovaquone plus azithromycin is the preferred treatment as the combination is better tolerated with less adverse effects and those who cannot tolerate PO can be given IV quinidine and clindamycin.1 Atovaquone is better absorbed when taken with a fatty meal, however azithromycin from capsules have a 50% reduced absorption when taken with food, which is why immediate-release azithromycin suspension is preferred for efficacy.1
    The typical dosing for Atovaquone is 750mg orally every 12 hours and Azithromycin is 500mg/day orally on day 1 and 250mg/day from days 2-10.3 In the case of Clindamycin, 300-600mg is given IV every 6 hours or 600mg orally every 8 hours.3 Quinine is typically dosed at 650mg orally every 6 to 8 hours.3 The dosing is modified for different species of pathogen and for children.
    There are some serious side effects associated with these treatments. Serial ECG’s need to be monitored while on quinines as they can prolong QT intervals and precipitate Toursades de Pointes.3 It is for this reason, that we tailored our treatment toward avoiding this side effect in our patient who had already experienced ventricular arrhythmias during his work-up. Pregnant patients or those breastfeeding should not take quinine or atovaquone as it is a category C drug and leads to abnormal limb development and neurological complications.3
    If patients fail treatment with clindamycin plus quinine, there have been studies that have shown some success with azithromycin plus quinine.3 There also has been success with high doses of clindamycin plus azithromycin and doxycycline especially in immunocompromised patients with recurrent symptoms.1 Other antimalarial agents have been unsuccessful in treating babesiosis and should not be considered for alternative treatment. In cases with high parasitemia and severe hemolytic anemia, blood exchange transfusions should be considered.1

    References
    1. Gelfand, JA and Vannier, EG. Clinical manifestations, diagnosis, treatment, and prevention of babesiosis. https://www.uptodate.com/contents/clinical-manifestations-diagnosis-treatment-and-prevention-of-babesiosis?source=search_result&search=babesiosis&selectedTitle=1~7. Accessed 13 Aug. 2017
    2. CDC – DPDx – Malaria – Diagnostic Findings. https://www.cdc.gov/dpdx/malaria/dx.html. Accessed 13 Aug. 2017
    3. Babesiosis Treatment & Management: Approach Considerations, Pharmacologic Therapy, Exchange Transfusion and Supportive Care. http://emedicine.medscape.com/article/212605-treatment. Accessed 13 Aug. 2017

    Incorrect

    Answer is C. Babesiosis is a tick-borne disease that is carried by the same vector as Lyme disease and found classically in the northeastern portion of the United States like Cape Cod, Massachusetts. Patients infected with Babesiosis usually present acutely with malaise, weakness, fatigue and fevers upwards of 105°F. Fever in these patients is typically followed by chills and night sweats. Hemolytic anemia and thrombocytopenia are common features of this disease and WBC counts can be low, normal or high. The Patient’s acute onset with flu-like symptoms and high-grade fever suggest an infection that effectively rules out Pulmonary Embolism. Although PE patients often present with Tachycardia, SOB, weakness, fatigue, low O2 Sat and possibly lung finding on physical examination, there is no connection between PE causing thrombocytopenia or hemolytic anemia. Endocarditis would cause high fevers, flu-like symptoms and a heart murmur, however, this patient has a history of valvular disease that would explain the murmur and Endocarditis has no correlation to hemolytic anemias or thrombocytopenia. Patients who develop endocarditis typically have a point of entry into the bloodstream such as IV drug use, recent dental surgery or open wounds which this patient did not have. Rocky Mountain spotted fever is another tick-borne illness that presents with similar symptoms and findings including thrombocytopenia and hemolytic anemia, however it is not commonly found in the northeastern United States and classically has a palmar rash associated with its presentation.

    Discussion:
    Babesiosis is an infectious parasitic protozoan that is predominantly found in the Northeastern and upper Midwestern regions of the United States.1 Other species are found in Europe; however, all species affect all ethnicities equally.1 The most common species of babesiosis is B. microti which is what our conversation will focus on. There are rarer species of Babesiosis such as B. divergens which can cause fulminant illness and are considered a medical emergency requiring blood exchange transfusion.1
    Babesiosis is carried by tick vectors most commonly Ixodes scapularis ticks (aka the Black-Legged Deer Ticks).1 Ixodes scapularis have 3 stages of development from larva to nymph to adult which all require what is called a “blood meal” to advance to the next stage.1 They obtain the parasite through multiple feedings from hosts and the protozoa multiplies in the gut wall of the tick before concentrating in the salivary glands.1 When the next feeding occurs, the parasite is transferred from the tick vector to a new host entering the bloodstream during the bite as mature trophozoites.1 The parasite infects RBCs directly and once in the cell, the trophozoites undergo asexual budding and replicated into 2 to 4 merozoites which leave the cell through direct lysis of the membrane.1 Alterations in RBC membranes can lead to excess fluid which manifests as pulmonary edema and possibly ARDS.1
    Pathophysiological manifestations of the disease can be seen as hemolytic anemia due to direct parasite-mediated lysis of red blood cells in the circulation. This leads to a low hematocrit, elevated lactate dehydrogenase, low hemoglobin and elevated total bilirubin.1 As a result of lysis, the RBC fragments can cause capillary blockage and microvascular stasis which can lead to serious CNS complications, liver, splenic or renal involvement.1
    Presentation with Babesiosis infections range from being asymptomatic to severely unstable and possibly fatal.1 The severity and course of the infection mainly depend on the species of Babesiosis, the patient’s immune status and the patient’s co-morbidities. Most patients typically become symptomatic with the first 6 weeks following a bite from an infected tick in an endemic area but more precisely it has been determined that the median interval for symptom onset is 37 days.1 Patients typically present with gradual onset of fatigue, weakness, malaise and fevers that can reach dangerously high fevers such as 105.6°F.1 Following the fever, patients will develop chills and night sweats.1 Less commonly symptoms will include headache, nausea, vomiting, diarrhea, anorexia, arthralgia, myalgias, neck stiffness, shortness of breath, dry cough, weight loss or dark urine.1
    Physical examination may reveal mild organomegaly, specifically splenomegaly, but lymphadenopathy is absent.1 As previously stated, hemolytic anemia is present in all patients but thrombocytopenia is also common.1 White blood cell counts have no significance on evaluation as they can range from decreased to normal to elevated.1
    Patients with severe babesiosis require hospitalization and are susceptible to potential complications include acute respiratory distress syndrome, disseminated intravascular coagulation, congestive heart failure, and renal failure.1 These usually occur when the anemia becomes sever and the parasitemia is high. In rare circumstances, high fever can cause a hyperadrenergic state leading to excessive orthostatic tachycardia, which in someone with preexisting heart disease, such as our individual, can precipitate lethal arrhythmias.1
    The definitive diagnosis of babesiosis should be made by examining blood with direct light microscopy and Wright-Giemsa Stain.2 Specifically, examination of thin blood smears is necessary as babesial parasites may be too small to be seen using thick blood smears. Babesiosis most commonly are seen as a pale blue ring with two red chromatic dots.2 You also can occasionally see the parasites arranged in tetrads known as “Maltese Cross.1”
    Polymerase chain reaction (PCR) is also useful for diagnosis especially in those with a low-level of parasitemia and can be used to detect any Babesia DNA levels in blood that show no parasites on blood smear.1 One downfall of PCR is that the DNA detected does not always equate to live parasites and can be detected in blood for up to 26 months post-infection.1
    Indirect immunofluorescence can be used in rare cases with high suspicion of babesiosis and negative blood smears and PCR.1 These cases are seen more in blood donors that have been implicated in transfusion-acquired disease or in asymptomatic patients who have cleared the disease without treatment but may have complications following the infection.1
    The diagnosis should be considered in patients who present with flu-like symptoms and who either live in or have traveled recently to endemic areas. Also, patients with symptom who have been previously diagnosed with Lyme disease, Anaplasmosis, Powassan or other tick-borne illnesses should also be suspected.
    Treatment should be initiated in any symptomatic patients with parasitemia confirmed via blood smear or PCR. If a patient is asymptomatic but has a confirmed diagnosis of babesiosis, treatment is not necessary until symptomatic. Asymptomatic individuals who have persistent infection for > 3months as confirmed by blood smear or PCR should also consider treatment.
    The most effective therapy is the use of antimicrobial agents, specifically atovaquone plus azithromycin or quinine plus clindamycin.3 Both options are equally efficacious but Clindamycin plus quinine is typically reserved for severe cases (requiring hospitalization) or those who are immunocompromised.1 The typical regimen is to take both drugs orally for 7 to 10 days.3 Atovaquone plus azithromycin is the preferred treatment as the combination is better tolerated with less adverse effects and those who cannot tolerate PO can be given IV quinidine and clindamycin.1 Atovaquone is better absorbed when taken with a fatty meal, however azithromycin from capsules have a 50% reduced absorption when taken with food, which is why immediate-release azithromycin suspension is preferred for efficacy.1
    The typical dosing for Atovaquone is 750mg orally every 12 hours and Azithromycin is 500mg/day orally on day 1 and 250mg/day from days 2-10.3 In the case of Clindamycin, 300-600mg is given IV every 6 hours or 600mg orally every 8 hours.3 Quinine is typically dosed at 650mg orally every 6 to 8 hours.3 The dosing is modified for different species of pathogen and for children.
    There are some serious side effects associated with these treatments. Serial ECG’s need to be monitored while on quinines as they can prolong QT intervals and precipitate Toursades de Pointes.3 It is for this reason, that we tailored our treatment toward avoiding this side effect in our patient who had already experienced ventricular arrhythmias during his work-up. Pregnant patients or those breastfeeding should not take quinine or atovaquone as it is a category C drug and leads to abnormal limb development and neurological complications.3
    If patients fail treatment with clindamycin plus quinine, there have been studies that have shown some success with azithromycin plus quinine.3 There also has been success with high doses of clindamycin plus azithromycin and doxycycline especially in immunocompromised patients with recurrent symptoms.1 Other antimalarial agents have been unsuccessful in treating babesiosis and should not be considered for alternative treatment. In cases with high parasitemia and severe hemolytic anemia, blood exchange transfusions should be considered.1

    References
    1. Gelfand, JA and Vannier, EG. Clinical manifestations, diagnosis, treatment, and prevention of babesiosis. https://www.uptodate.com/contents/clinical-manifestations-diagnosis-treatment-and-prevention-of-babesiosis?source=search_result&search=babesiosis&selectedTitle=1~7. Accessed 13 Aug. 2017
    2. CDC – DPDx – Malaria – Diagnostic Findings. https://www.cdc.gov/dpdx/malaria/dx.html. Accessed 13 Aug. 2017
    3. Babesiosis Treatment & Management: Approach Considerations, Pharmacologic Therapy, Exchange Transfusion and Supportive Care. http://emedicine.medscape.com/article/212605-treatment. Accessed 13 Aug. 2017

  181. Question 181 of 220
    181. Question

    An 81-year-old female presents to the emergency department with progressive confusion, disorientation, headache, nausea, and vomiting for the past 24 hours. Physical exam is benign and appears euvolemic. A basic metabolic panel is run and is significant for hyponatremia with a serum sodium of 119 mEq/L. Further testing reveals a decreased serum osmolarity, increased urine osmolarity, and a normal TSH and cortisol level. Of all the following choices which is the most likely diagnosis?

    Correct

    Answer is B. The patient is presenting with signs and symptoms of hyponatremia, specifically from SIADH. The lab diagnostics are valid for SIADH with decreased serum sodium and hypo-osmolality1,3. In addition, the patient is excreting sodium due to having a water problem with ADH and not a true salt deficiency. Therefore, the patient presents with increased urine sodium and increased urine osmolality2,3.
    From the history, one may believe the patient is dehydrated explaining the signs and symptoms of hyponatremia. However, in the setting of dehydration the urine sodium would be decreased, not increased 1. Nephrotic would be an inappropriate answer for this question. The patient’s physical exam is benign. In a patient who is experiencing nephrotic syndrome, they would be edematous and third spacing fluid. In addition, this case does not offer any information pertaining to a UA, which would be needed in the diagnosis of nephrotic syndrome. Patients with nephrotic syndrome present with proteinuria1,4. Psychogenic water intoxication occurs from marked free water intake which results in a dilutional hyponatremia with low serum osmolality similar to that of SIADH1,4. Like SIADH, the patient often maintains euvolemia via renal excretion of sodium. In a patient with psychogenic water intoxication, the urine sodium is often elevated4. However, in this situation there is also an increase in free water excretion, which dilutes the urine and overwhelms the ability of ADH. In psychogenic water intoxication, the urine osmolality would be low unlike SIADH with a high urine osmolality1,4.

    Discussion:
    SIADH is a syndrome in which there is excessive ADH in a patient who is euvolemic. The excess ADH interferes with free water excretion, resulting in an imbalance of input verses output and the devilment of dilutional hyponatremia1,3. Under normal circumstances, the hypothalamus can detect that there is too much water in the blood, and less ADH would be secreted by the pituitary gland into the blood stream1. The kidneys would then absorb less water from the blood in the collecting duct, and dilute urine would be produced which would bring the blood water level back to normal1. In the case of SIADH, the increase in ADH results in increased water reabsorption by the kidneys and decreased aldosterone secretion. SIADH is the most common cause of euvolemic hyponatremia in hospitalized patients3.
    Despite being presenting with hyponatremia, SIADH is not a problem of Na deficiency but of problem with excess water. The ADH increases the water permeability of the renal collection ducts, thereby increasing renal water absorption3. Most of the water absorption is intracellular, but there is also an expansion in the extracellular volume. It is the expansion of the extracellular volume that causes excretion of sodium, and to some degree potassium excretion. This excretion of sodium occurs until a steady state of sodium excreted in the urine matches sodium intake3. The overall result is a dilutional hyponatremia. The most severe risk of SIADH is the brain’s response hypo-osmolality, which can cause brain edema3.
    SIADH can be caused by over secretion of ADH from the hypothalamus or from an ectopically. The etiology can be broken down into four main categories: neoplasia’s, nervous system disorders, pulmonary diseases, and drug induced. Pharmacologic agents that increase AVP release include: isoproterenol, MAOIs, and tricyclic antidepressants1. There are also drugs that can increase the effects of AVP action such as theophylline and some hypoglycemic agents1. Some examples of the other three broad categories include, but are by no means limited to: lung carcinoma, mesothelioma, cerebellar and cerebral atrophy, cerebrovascular accident, multiple sclerosis, asthma, COPD, pneumothorax, and cystic fibrosis3.
    Hyponatremia tends to occur more often in the hospital setting, largely due to the administration of hypotonic IV fluids in the setting of SIADH1,3. It is noted that those who have SIADH experience more hyponatremic events with increasing age over thirty years old3. Men have a higher likely hood of having hyponatremia in the hospital with SIADH than their woman counterpart3. Low body weight is also described as a risk factor for developing hyponatremia in the setting of SIADH3.
    Evaluation of someone presenting with SIADH starts with a history of new medications, changes in fluid intake, and fluid output. The H&P should help identify the etiology of SIADH. The physical exam should help make the diagnosis based on determining the patient’s volume status. Patients with SIADH often present with anorexia, nausea, and vomiting1,2,3,4. In addition, they can develop generalized muscle weakness, myoclonus, tremor, hyporeflexia, ataxia, and dysarthria3. These patients may progress to develop neurological symptoms such as headache, dulled sensorium, confusion, convulsions, and even death due to untreated brain swelling1,2,3.
    Laboratory testing is beneficial in the diagnosis of SIADH. Tests include; but are not limited to: BMP, serum osmolality, urine osmolality, serum uric acid, serum cortisol, and TSH3. In the setting of SIADH, the patient should present with serum hypo-osmolality and hyponatremia with urine hyper osmolality1,2,3. The remainder of the discussed lab values should be within normal reference ranges. Other conditions that alter volume status should be ruled out; such as: CHF, hypovolemia from any cause, and renal insufficiency.
    The rapidity of correction of hyponatremia depends on the degree of the hyponatremia, symptomatic presentation, and acute vs chronic. Extreme hyponatremia and an inappropriate approach to its treatment can both have disastrous consequences and often a nephrology consult is needed. Correcting hyponatremia too rapidly may result in central pontine myelinolysis with permanent neurologic defects1, 2. In order to avoid neurologic defects, it is advised to raise serum sodium by 0.5-1mEq/hr until a serum sodium of 125-130mEq/L is obtained2, 3, 4. However, in an acute setting with hyponatremia occurring within 48 hours 3% hypertonic saline can be used at 513mEq/L. In this situation loop diuretics with saline, vasopressin-2 receptor antagonists, and water restriction can also be effective3. In the more chronic, asymptomatic setting fluid restriction with or without vasopressin-2 receptor antagonists can be used2, 3.

    References:
    1.)Papadakis MA, McPhee SJ. Current Medical Diagnosis & Treamtent. 54th ed. Cenveo
    Publisher Services; 2015.

    2.)Pillai BP, Unnikrishnan AG, Pavithran PV. Syndrome of inappropriate antidiuretic hormone
    secretion: Revisiting a classical endocrine disorder. Indian Journal of Endocrinology and
    Metabolism. 2011;15(Suppl3):S208-S215. doi:10.4103/2230-8210.84870.

    3.)Thomas CP. Syndrome of Inappropriate Antidiuretic Hormone Secretion Workup. Syndrome
    of Inappropriate Antidiuretic Hormone Secretion Workup: Approach Considerations,
    Laboratory Tests, Volume Assessment. http://emedicine.medscape.com/article/246650- workup#c7. Published July 17, 2017. Accessed August 12, 2017.

    4.)Wiener, Longo, Fauci, et al. Harrison’s Principles of Internal Medicine. 18th ed. Cenveo
    Publisher Services; 2012.

    Incorrect

    Answer is B. The patient is presenting with signs and symptoms of hyponatremia, specifically from SIADH. The lab diagnostics are valid for SIADH with decreased serum sodium and hypo-osmolality1,3. In addition, the patient is excreting sodium due to having a water problem with ADH and not a true salt deficiency. Therefore, the patient presents with increased urine sodium and increased urine osmolality2,3.
    From the history, one may believe the patient is dehydrated explaining the signs and symptoms of hyponatremia. However, in the setting of dehydration the urine sodium would be decreased, not increased 1. Nephrotic would be an inappropriate answer for this question. The patient’s physical exam is benign. In a patient who is experiencing nephrotic syndrome, they would be edematous and third spacing fluid. In addition, this case does not offer any information pertaining to a UA, which would be needed in the diagnosis of nephrotic syndrome. Patients with nephrotic syndrome present with proteinuria1,4. Psychogenic water intoxication occurs from marked free water intake which results in a dilutional hyponatremia with low serum osmolality similar to that of SIADH1,4. Like SIADH, the patient often maintains euvolemia via renal excretion of sodium. In a patient with psychogenic water intoxication, the urine sodium is often elevated4. However, in this situation there is also an increase in free water excretion, which dilutes the urine and overwhelms the ability of ADH. In psychogenic water intoxication, the urine osmolality would be low unlike SIADH with a high urine osmolality1,4.

    Discussion:
    SIADH is a syndrome in which there is excessive ADH in a patient who is euvolemic. The excess ADH interferes with free water excretion, resulting in an imbalance of input verses output and the devilment of dilutional hyponatremia1,3. Under normal circumstances, the hypothalamus can detect that there is too much water in the blood, and less ADH would be secreted by the pituitary gland into the blood stream1. The kidneys would then absorb less water from the blood in the collecting duct, and dilute urine would be produced which would bring the blood water level back to normal1. In the case of SIADH, the increase in ADH results in increased water reabsorption by the kidneys and decreased aldosterone secretion. SIADH is the most common cause of euvolemic hyponatremia in hospitalized patients3.
    Despite being presenting with hyponatremia, SIADH is not a problem of Na deficiency but of problem with excess water. The ADH increases the water permeability of the renal collection ducts, thereby increasing renal water absorption3. Most of the water absorption is intracellular, but there is also an expansion in the extracellular volume. It is the expansion of the extracellular volume that causes excretion of sodium, and to some degree potassium excretion. This excretion of sodium occurs until a steady state of sodium excreted in the urine matches sodium intake3. The overall result is a dilutional hyponatremia. The most severe risk of SIADH is the brain’s response hypo-osmolality, which can cause brain edema3.
    SIADH can be caused by over secretion of ADH from the hypothalamus or from an ectopically. The etiology can be broken down into four main categories: neoplasia’s, nervous system disorders, pulmonary diseases, and drug induced. Pharmacologic agents that increase AVP release include: isoproterenol, MAOIs, and tricyclic antidepressants1. There are also drugs that can increase the effects of AVP action such as theophylline and some hypoglycemic agents1. Some examples of the other three broad categories include, but are by no means limited to: lung carcinoma, mesothelioma, cerebellar and cerebral atrophy, cerebrovascular accident, multiple sclerosis, asthma, COPD, pneumothorax, and cystic fibrosis3.
    Hyponatremia tends to occur more often in the hospital setting, largely due to the administration of hypotonic IV fluids in the setting of SIADH1,3. It is noted that those who have SIADH experience more hyponatremic events with increasing age over thirty years old3. Men have a higher likely hood of having hyponatremia in the hospital with SIADH than their woman counterpart3. Low body weight is also described as a risk factor for developing hyponatremia in the setting of SIADH3.
    Evaluation of someone presenting with SIADH starts with a history of new medications, changes in fluid intake, and fluid output. The H&P should help identify the etiology of SIADH. The physical exam should help make the diagnosis based on determining the patient’s volume status. Patients with SIADH often present with anorexia, nausea, and vomiting1,2,3,4. In addition, they can develop generalized muscle weakness, myoclonus, tremor, hyporeflexia, ataxia, and dysarthria3. These patients may progress to develop neurological symptoms such as headache, dulled sensorium, confusion, convulsions, and even death due to untreated brain swelling1,2,3.
    Laboratory testing is beneficial in the diagnosis of SIADH. Tests include; but are not limited to: BMP, serum osmolality, urine osmolality, serum uric acid, serum cortisol, and TSH3. In the setting of SIADH, the patient should present with serum hypo-osmolality and hyponatremia with urine hyper osmolality1,2,3. The remainder of the discussed lab values should be within normal reference ranges. Other conditions that alter volume status should be ruled out; such as: CHF, hypovolemia from any cause, and renal insufficiency.
    The rapidity of correction of hyponatremia depends on the degree of the hyponatremia, symptomatic presentation, and acute vs chronic. Extreme hyponatremia and an inappropriate approach to its treatment can both have disastrous consequences and often a nephrology consult is needed. Correcting hyponatremia too rapidly may result in central pontine myelinolysis with permanent neurologic defects1, 2. In order to avoid neurologic defects, it is advised to raise serum sodium by 0.5-1mEq/hr until a serum sodium of 125-130mEq/L is obtained2, 3, 4. However, in an acute setting with hyponatremia occurring within 48 hours 3% hypertonic saline can be used at 513mEq/L. In this situation loop diuretics with saline, vasopressin-2 receptor antagonists, and water restriction can also be effective3. In the more chronic, asymptomatic setting fluid restriction with or without vasopressin-2 receptor antagonists can be used2, 3.

    References:
    1.)Papadakis MA, McPhee SJ. Current Medical Diagnosis & Treamtent. 54th ed. Cenveo
    Publisher Services; 2015.

    2.)Pillai BP, Unnikrishnan AG, Pavithran PV. Syndrome of inappropriate antidiuretic hormone
    secretion: Revisiting a classical endocrine disorder. Indian Journal of Endocrinology and
    Metabolism. 2011;15(Suppl3):S208-S215. doi:10.4103/2230-8210.84870.

    3.)Thomas CP. Syndrome of Inappropriate Antidiuretic Hormone Secretion Workup. Syndrome
    of Inappropriate Antidiuretic Hormone Secretion Workup: Approach Considerations,
    Laboratory Tests, Volume Assessment. http://emedicine.medscape.com/article/246650- workup#c7. Published July 17, 2017. Accessed August 12, 2017.

    4.)Wiener, Longo, Fauci, et al. Harrison’s Principles of Internal Medicine. 18th ed. Cenveo
    Publisher Services; 2012.

  182. Question 182 of 220
    182. Question

    A 62 year old female is brought to your office by her family who is concerned due to her sudden inability to move the right side of her face. The patient’s family reports they were eating breakfast when they noticed that she was having difficulty chewing her food, and the right side of her face was drooping. The patient notes that her right eye has been dry, and when she woke up this morning she noticed she could no longer close it. On examination you notice right sided facial paralysis which includes sagging, loss of the nasolabial fold, inability to close the right eye, and inability to raise the forehead on the right side. She denies any visual changes, or pain. The remainder of her neurological exam is unremarkable. What is the most likely cause of this patient’s condition?

    Correct

    Case Discussion
    Bell’s Palsy (BP), which may also be known as Idiopathic Facial Paralysis (IFP), is a disorder involving cranial nerve VII.1,2 It is described as the most common cause of sudden onset unilateral facial paralysis.1,3 When BP was first diagnosed in the 1800’s the etiology was thought to be trauma to the peripheral nerve branches, but since this theory has been dispelled.1 The most common cause of BP is herpes simplex virus (HSV). There are many other, less common, etiologies such as Lyme disease, herpes zoster, syphilis, Epstein Barr virus, cytomegalovirus, HIV, inflammation, or microvascular disease.3Another hypothesis is that BP can have a genetic etiology; however, since the most common causes of BP are viral, and therefore contagious, the true etiology may not be genetic, but instead infectious in nature, and merely passed onto family members through contact.3
    BP accounts for over half of all cases of facial nerve palsy.1,3The incidence of BP is approximately 23/100,000 in the United States.3 Although there has been no link found between race, geographic location, or sex there is a slight increase of cases in females between 10-19 years old compared to males of the same age group.2,3 BP rarely occurs before age ten, and most commonly occurs after age 60.2,3 A major risk factor for BP is pregnancy, with a 3.3 times higher risk of contracting BP than nonpregnant women, with the highest rate occurring in the third trimester, and first week postpartum.1,3 Other risk factors include diabetes, obesity, and hypertension.2
    BP is a sudden onset of peripheral facial palsy of which there may be many causes, while the pathophysiology is widely debated there are a number of hypothesis. One hypothesis states that HSV causes BP when the virus is spread along the axon, and then the reactivated virus is multiplied causing inflammation, demyelination and palsy of cranial nerve VII (facial nerve).1 Another hypothesis states that the palsy is caused when there is edema and ischemia of the facial nerve which leads to compression of the nerve in the bony canal. 3 However, this theory states that the cause of the edema and ischemia are unknown. BP is called a peripheral nerve palsy because the injury to the nerve occurs peripherally compared to the nerve’s nucleus. If the injury occurs proximal to the geniculate ganglion, there is motor paralysis, gustatory, and autonomic abnormalities seen. If the lesion is between the geniculate ganglion and the chorda tympani then lacrimation is spared. Finally, if the lesion is at the stylomastoid foramen this results solely in motor abnormalities. 3
    Thorough history and physical examination should be obtained. Patients with BP usually present with an acute onset of unilateral facial paralysis that has occurred over several hours.1 Patients may complain of decreased tearing on the affected side, hyperacusis, loss of taste on the anterior 2/3 of the tongue, inability to smile, otalgia, and/or posterior auricular pain.1,3 Physical examination should include a full general and neurological examination including assessment of all cranial nerves, sensory, motor, and cerebellar testing.3 On exam you will notice inability to close the eye on the affected side along with, eyebrow dropping, facial sagging, and loss of the nasolabial fold.1 It is important to note if the forehead muscles are spared or not in order to differentiate between a central lesion or a peripheral lesion. The muscles will be spared in a central lesion. 1 All other neurological abnormalities that cannot be explained by a facial nerve palsy should be referred for further testing, such as MRI, lumbar puncture, or EMG. 3
    The diagnosis of BP is one of exclusion that can usually be made if there has been a thorough history and physical examination done. Imaging is not normally needed for patients with a typical facial nerve palsy presentation; however, as stated above, further studies and imaging should be ordered for patients who do not present in a traditional fashion.1 Serological testing for Lyme disease, HSV, etc. may be done in order to determine the etiology of the palsy.1 Additional testing may include a CBC with differential to look for signs of infection, and screening for diabetes mellitus.2
    Treatment of BP of suspected viral etiology is with oral glucocorticoids, and in some cases the addition of an antiviral. Treatment should be started within three days of the onset for the best outcomes. The guidelines suggest Prednisone 60-80mg/day for one week as the glucocorticoid of choice. For severe cases addition of an antiviral, such as Valacyclovir 1000mg three times a day for one week is recommended. BP due to other etiologies such as Lyme disease should be treated based on the cause. For patients that are unable to close their eye, eye care is of utmost importance, due to the cornea being at increased risk for damage. Using artificial tears, or other lubricating eye drops along with protective patch or goggles will decrease risk of damage. Most patients start to see improvement within the first 21 days, however some patients may have long term sequel. Education about the long term effects should be discussed with the patient.1 Referral for further investigation should be made if the palsy does not show improvements in three weeks.2

    References
    1. Ronthal M. Bell’s palsy: Pathogenesis, clinical features, and diagnosis in adults. Bell’s palsy: Pathogenesis, clinical features, and diagnosis in adults. https://www.uptodate.com/contents/bells-palsy-pathogenesis-clinical-features-and-diagnosis-in-adults?source=see_link#H69095511. Published December 1, 2016. Accessed August 8, 2017.
    2. Patel D, Levin K. Bell palsy: Clinical examination and management. MDedge. http://www.mdedge.com/ccjm/article/100703/neurology/bell-palsy-clinical-examination-and-management. Published July 2015. Accessed August 8, 2017.
    3. Taylor D. Bell Palsy Clinical Presentation. Bell Palsy Clinical Presentation: History, Physical Examination, Grading. http://emedicine.medscape.com/article/1146903-clinical#b3. Published July 19, 2017. Accessed August 8, 2017.

    Incorrect

    Case Discussion
    Bell’s Palsy (BP), which may also be known as Idiopathic Facial Paralysis (IFP), is a disorder involving cranial nerve VII.1,2 It is described as the most common cause of sudden onset unilateral facial paralysis.1,3 When BP was first diagnosed in the 1800’s the etiology was thought to be trauma to the peripheral nerve branches, but since this theory has been dispelled.1 The most common cause of BP is herpes simplex virus (HSV). There are many other, less common, etiologies such as Lyme disease, herpes zoster, syphilis, Epstein Barr virus, cytomegalovirus, HIV, inflammation, or microvascular disease.3Another hypothesis is that BP can have a genetic etiology; however, since the most common causes of BP are viral, and therefore contagious, the true etiology may not be genetic, but instead infectious in nature, and merely passed onto family members through contact.3
    BP accounts for over half of all cases of facial nerve palsy.1,3The incidence of BP is approximately 23/100,000 in the United States.3 Although there has been no link found between race, geographic location, or sex there is a slight increase of cases in females between 10-19 years old compared to males of the same age group.2,3 BP rarely occurs before age ten, and most commonly occurs after age 60.2,3 A major risk factor for BP is pregnancy, with a 3.3 times higher risk of contracting BP than nonpregnant women, with the highest rate occurring in the third trimester, and first week postpartum.1,3 Other risk factors include diabetes, obesity, and hypertension.2
    BP is a sudden onset of peripheral facial palsy of which there may be many causes, while the pathophysiology is widely debated there are a number of hypothesis. One hypothesis states that HSV causes BP when the virus is spread along the axon, and then the reactivated virus is multiplied causing inflammation, demyelination and palsy of cranial nerve VII (facial nerve).1 Another hypothesis states that the palsy is caused when there is edema and ischemia of the facial nerve which leads to compression of the nerve in the bony canal. 3 However, this theory states that the cause of the edema and ischemia are unknown. BP is called a peripheral nerve palsy because the injury to the nerve occurs peripherally compared to the nerve’s nucleus. If the injury occurs proximal to the geniculate ganglion, there is motor paralysis, gustatory, and autonomic abnormalities seen. If the lesion is between the geniculate ganglion and the chorda tympani then lacrimation is spared. Finally, if the lesion is at the stylomastoid foramen this results solely in motor abnormalities. 3
    Thorough history and physical examination should be obtained. Patients with BP usually present with an acute onset of unilateral facial paralysis that has occurred over several hours.1 Patients may complain of decreased tearing on the affected side, hyperacusis, loss of taste on the anterior 2/3 of the tongue, inability to smile, otalgia, and/or posterior auricular pain.1,3 Physical examination should include a full general and neurological examination including assessment of all cranial nerves, sensory, motor, and cerebellar testing.3 On exam you will notice inability to close the eye on the affected side along with, eyebrow dropping, facial sagging, and loss of the nasolabial fold.1 It is important to note if the forehead muscles are spared or not in order to differentiate between a central lesion or a peripheral lesion. The muscles will be spared in a central lesion. 1 All other neurological abnormalities that cannot be explained by a facial nerve palsy should be referred for further testing, such as MRI, lumbar puncture, or EMG. 3
    The diagnosis of BP is one of exclusion that can usually be made if there has been a thorough history and physical examination done. Imaging is not normally needed for patients with a typical facial nerve palsy presentation; however, as stated above, further studies and imaging should be ordered for patients who do not present in a traditional fashion.1 Serological testing for Lyme disease, HSV, etc. may be done in order to determine the etiology of the palsy.1 Additional testing may include a CBC with differential to look for signs of infection, and screening for diabetes mellitus.2
    Treatment of BP of suspected viral etiology is with oral glucocorticoids, and in some cases the addition of an antiviral. Treatment should be started within three days of the onset for the best outcomes. The guidelines suggest Prednisone 60-80mg/day for one week as the glucocorticoid of choice. For severe cases addition of an antiviral, such as Valacyclovir 1000mg three times a day for one week is recommended. BP due to other etiologies such as Lyme disease should be treated based on the cause. For patients that are unable to close their eye, eye care is of utmost importance, due to the cornea being at increased risk for damage. Using artificial tears, or other lubricating eye drops along with protective patch or goggles will decrease risk of damage. Most patients start to see improvement within the first 21 days, however some patients may have long term sequel. Education about the long term effects should be discussed with the patient.1 Referral for further investigation should be made if the palsy does not show improvements in three weeks.2

    References
    1. Ronthal M. Bell’s palsy: Pathogenesis, clinical features, and diagnosis in adults. Bell’s palsy: Pathogenesis, clinical features, and diagnosis in adults. https://www.uptodate.com/contents/bells-palsy-pathogenesis-clinical-features-and-diagnosis-in-adults?source=see_link#H69095511. Published December 1, 2016. Accessed August 8, 2017.
    2. Patel D, Levin K. Bell palsy: Clinical examination and management. MDedge. http://www.mdedge.com/ccjm/article/100703/neurology/bell-palsy-clinical-examination-and-management. Published July 2015. Accessed August 8, 2017.
    3. Taylor D. Bell Palsy Clinical Presentation. Bell Palsy Clinical Presentation: History, Physical Examination, Grading. http://emedicine.medscape.com/article/1146903-clinical#b3. Published July 19, 2017. Accessed August 8, 2017.

  183. Question 183 of 220
    183. Question

    A 63 year-old female presents to the office with left knee pain for several months. The pain is made worse with stairs and seems to be really achy at the end of the day. She denies prior injury. AP, lateral, and merchant view x-rays show mild joint space narrowing, subchondral sclerosis in the medial compartment, and significant osteophytes in the patella femoral joint (Figures 1, 2, and 3). Based on the Kellgren-Lawrence grading system for knee osteoarthritis, which grade of arthritis does this patient have?

    Correct

    Answer C. The Kellgren-Lawrence (KL) grading system is the most widely accepted assessment tool for quantifying the degree of knee osteoarthritis found on x-ray. Knowing this grading system is of increasing importance today as some insurance carriers are requiring documentation of KL grade 2 or higher knee osteoarthritis on x-ray in order to approve certain treatments like viscosupplementation injections. Viscosupplementation, or hyaluronic acid injections, are gel like injections for the knee that provide lubrication and pain relief for patients with knee arthritis.
    KL grade 1 changes include no obvious joint space narrowing with small osteophytes present, grade 2 changes include definite osteophytes with possible joint space narrowing, grade 3 changes include multiple osteophytes, moderate joint space narrowing, subchondral sclerosis, and possible bony deformity, and grade 4 changes include large osteophytes, severe joint space narrowing, subchondral sclerorsis, and definite bony deformity.
    There are several flaws in the KL grading system, however. The KL system relies on the formation of osteophytes occurring before joint space narrowing, although generally the opposite occurs. Also, several studies have looked at arthroscopic findings in patients with early articular degeneration compared to KL grade and concluded that the KL system can significantly underestimate the degree of osteoarthritis in the knee.
    References
    1. Kijowski R, Blankenbaker D, Stanton P, Fine J, De Smet A. Arthroscopic Validation of Radiographic Grading Scales of Osteoarthritis of the Tibiofemoral Joint. AJR. 2006. 187. 794-799.
    2. Kellgren-Lawrence (KL) grading system. https://radiopaedia.org. Accessed on 7/23/17.

    Incorrect

    Answer C. The Kellgren-Lawrence (KL) grading system is the most widely accepted assessment tool for quantifying the degree of knee osteoarthritis found on x-ray. Knowing this grading system is of increasing importance today as some insurance carriers are requiring documentation of KL grade 2 or higher knee osteoarthritis on x-ray in order to approve certain treatments like viscosupplementation injections. Viscosupplementation, or hyaluronic acid injections, are gel like injections for the knee that provide lubrication and pain relief for patients with knee arthritis.
    KL grade 1 changes include no obvious joint space narrowing with small osteophytes present, grade 2 changes include definite osteophytes with possible joint space narrowing, grade 3 changes include multiple osteophytes, moderate joint space narrowing, subchondral sclerosis, and possible bony deformity, and grade 4 changes include large osteophytes, severe joint space narrowing, subchondral sclerorsis, and definite bony deformity.
    There are several flaws in the KL grading system, however. The KL system relies on the formation of osteophytes occurring before joint space narrowing, although generally the opposite occurs. Also, several studies have looked at arthroscopic findings in patients with early articular degeneration compared to KL grade and concluded that the KL system can significantly underestimate the degree of osteoarthritis in the knee.
    References
    1. Kijowski R, Blankenbaker D, Stanton P, Fine J, De Smet A. Arthroscopic Validation of Radiographic Grading Scales of Osteoarthritis of the Tibiofemoral Joint. AJR. 2006. 187. 794-799.
    2. Kellgren-Lawrence (KL) grading system. https://radiopaedia.org. Accessed on 7/23/17.

  184. Question 184 of 220
    184. Question

    A 56 year-old female presents to your office with a several month history of left forefoot pain. She denies prior injury or known precipitating event. The pain seems to be located between the 2nd and 3rd toes and is made worse with tight shoes. She really notices the pain when she wears high heels and works out at the gym. On exam you note increased pain when squeezing the metatarsal heads together. AP and lateral x-rays of the left foot are shown in figures 1 and 2. Which choice below is commonly used to confirm the patient’s diagnosis?

    Correct

    Answer C. Interdigital neuritis, often referred to as Morton’s neuroma, is a compressive neuropathy that most commonly involves the 3rd interdigital nerve between the 3rd and 4th metatarsal heads. The condition is most prevalent in middle aged women. The pathophysiology of the condition is poorly understood but many believe it is due to mechanical trauma, entrapment, and/or tethering of the nerve. Symptoms include burning and paresthesia’s in the involved toes that are made worse with narrow toe box shoes or high heels. Patients may feel like they are walking on a small pebble in their shoe. The pain is often relieved with removing the shoe and taking a rest from walking. On exam there is generally no obvious deformity of the foot and pain can be palpated at the involved interspace between the metatarsal heads. Pain is often made worse by compressing the metatarsal heads together. 1,2
    The diagnosis of Morton’s neuroma is often made with history and physical alone. If there is any doubt in the diagnosis, 1 ml of lidocaine can be injected into the dorsal intermetatarsal ligament between the metatarsal heads. Temporary relief of pain after the injection confirms the diagnosis. MRI and ultrasound can identify an interdigital neuroma, but are rarely used due to the diagnostic accuracy of history and physical alone. Conservative treatment include NSAIDS, shoes with wide toe boxes, and metatarsal pads to push the metatarsal heads away from each other. Steroid injections may also provide short term relief of pain. Morton’s neuroma is generally treated surgically as only 20% of patients will get better with conservative treatment. The standard surgical treatment involves resection of the interdigital nerve (neurectomy). 1,2

    References
    1. Hidalgo-Ovejero, AM; Martinez-Grande M, Sanchez-Villares JJ, Garcia-Mata S, Lasanta P, Coughlin MJ, Pinsonneault T. Clinical Examination and Imaging Studies in the Diagnosis of Interdigital Neuroma. Journal of Bone & Joint Surgery – American Volume: July 2002 – Volume 84 – Issue 7 – p 1276–1278.
    2. Interdigital Neuroma. http://www.orthobullets.com. Accessed on 4/26/17.

    Incorrect

    Answer C. Interdigital neuritis, often referred to as Morton’s neuroma, is a compressive neuropathy that most commonly involves the 3rd interdigital nerve between the 3rd and 4th metatarsal heads. The condition is most prevalent in middle aged women. The pathophysiology of the condition is poorly understood but many believe it is due to mechanical trauma, entrapment, and/or tethering of the nerve. Symptoms include burning and paresthesia’s in the involved toes that are made worse with narrow toe box shoes or high heels. Patients may feel like they are walking on a small pebble in their shoe. The pain is often relieved with removing the shoe and taking a rest from walking. On exam there is generally no obvious deformity of the foot and pain can be palpated at the involved interspace between the metatarsal heads. Pain is often made worse by compressing the metatarsal heads together. 1,2
    The diagnosis of Morton’s neuroma is often made with history and physical alone. If there is any doubt in the diagnosis, 1 ml of lidocaine can be injected into the dorsal intermetatarsal ligament between the metatarsal heads. Temporary relief of pain after the injection confirms the diagnosis. MRI and ultrasound can identify an interdigital neuroma, but are rarely used due to the diagnostic accuracy of history and physical alone. Conservative treatment include NSAIDS, shoes with wide toe boxes, and metatarsal pads to push the metatarsal heads away from each other. Steroid injections may also provide short term relief of pain. Morton’s neuroma is generally treated surgically as only 20% of patients will get better with conservative treatment. The standard surgical treatment involves resection of the interdigital nerve (neurectomy). 1,2

    References
    1. Hidalgo-Ovejero, AM; Martinez-Grande M, Sanchez-Villares JJ, Garcia-Mata S, Lasanta P, Coughlin MJ, Pinsonneault T. Clinical Examination and Imaging Studies in the Diagnosis of Interdigital Neuroma. Journal of Bone & Joint Surgery – American Volume: July 2002 – Volume 84 – Issue 7 – p 1276–1278.
    2. Interdigital Neuroma. http://www.orthobullets.com. Accessed on 4/26/17.

  185. Question 185 of 220
    185. Question

    A 56 year old woman with a past medical history of fibrocystic breast disease diagnosed in her 20s, one child delivered by cesarean section at age 33 (G1P1), and hypertension presents to her gynecologist with a lump in her right breast. She has a family history present for a maternal grandmother diagnosed with breast cancer at age 68 and a father with lung cancer. She denies any nipple discharge or skin changes. How many risk factors for breast cancer does this patient have?

    Correct

    Correct Answer: (C) The patient has four risk factors present for breast cancer. She has a previous history of a benign breast condition (fibrocystic breast disease), she had her first child after the age of 30, she has a family history of breast cancer and she is 56 years old.
    Incorrect Answers: (A) There is more than one risk factor.
    (B) There are more than three risk factors.
    (D) There are not five risk factors; hypertension is not a risk factor for breast cancer.

    Discussion:
    Invasive ductal carcinoma also known as infiltrating ductal carcinoma is the most common type of invasive breast cancer. It accounts for 70-80% of invasive breast lesions. (1) Although it can be seen in women of any age it is most commonly seen in women over the age of 55 (3). IDC is a cancer that began growing in the milk ducts of the breast and has invaded the fatty tissue of the breast as well. Cytological features of the involved cells range from bland to highly malignant. The malignant cells prompt a fibrous response as they infiltrate the breast tissue which is what causes a clinically palpable mass, the density that can be seen radiologically, and solid sonographic characteristics of classic invasive carcinomas (1). It is not clear what actually causes the abnormal cell growth but some risk factors have been identified. These risk factors include increasing age, personal history of benign breast disease, family history of breast cancer, first pregnancy after the age of 30 or nulliparity, history of DES exposure, hormone replacement therapy, obesity and genetic factors such as the BRCA1 or BRCA2 gene (2). Clinical features of invasive ductal carcinoma usually involve a mass or lesion that is a hard, immovable, lesion with irregular borders. However, the clinical features cannot consistently distinguish a benign tumor from a malignant one. If the cancer is locally advanced the signs can include axillary adenopathy or skin findings such as erythema, or dimpling of the overlying skin known as peau d’orange. If the cancer has metastases the symptoms can greatly vary depending on what organs are involved. The most common site of metastatic involvement is the bone, followed by the liver and lungs. Signs can include back or leg pain, abdominal pain, nausea, jaundice, and shortness of breath or cough. Most commonly however, patients present due to an abnormal mammogram finding (4).
    Classic findings of breast cancer on mammogram include the presence of a soft tissue mass or density as well as clustered microcalcifications invading to surrounding tissues. Breast ultrasound is often used in younger women due to the density of their breasts as well as to distinguish a benign versus malignant lesion. Sonographic features of breast malignancy include the presence of “spiculation, hypoechogenicity, microlobulation, internal calcifications, shadowing, a lesion taller than it is wide and angular margins” (4). Magnetic resonance imaging (MRI) is typically used to screen women at high risk for breast cancer as well as to further distinguish cancers seem on mammogram or ultrasound. Even though nearly all invasive breast cancers will enhance on gadolinium contrast-enhanced MRI, it is still not specific enough to remove the need for biopsy. Normally, once a lesion or mass is seen on mammogram, ultrasound, and/or MRI and it is suspicious for malignancy, it will be biopsied either using FNA or a core needle biopsy (4).
    As far as management and treatment goes, it really depends on the stage and grade of the cancer as well as the patient characteristics. In most cases, treatment will begin with surgery not only to remove the cancer but also to determine if any lymph nodes are involved. Possible surgical options include lumpectomy or different variations of mastectomy including partial, total and radical. Sentinel lymph node and axillary dissections are also usually done during the procedure to help stage the cancer and see the extent to which it has spread to the lymphatic system. Radiation and chemotherapy are also treatment options again, depending on the stage of the cancer and the extent to which it has metastasized (3).
    References:
    Bleiweiss IJ. Pathology of Breast Cancer. Pathology of breast cancer. https://www.uptodate.com/contents/pathology-of-breast-cancer?source=search_result&search=invasive ductal carcinoma&selectedTitle=1~150. Published June 17, 2016. Accessed August 1, 2017.
    Ductal carcinoma in situ (DCIS). Mayo Clinic. http://www.mayoclinic.org/diseases-conditions/dcis/basics/definition/con-20031842. Published June 14, 2014. Accessed August 1, 2017.
    Esserman L J, Joe BN. Clinical features, diagnosis, and staging of newly diagnosed breast cancer. Clinical features, diagnosis, and staging of newly diagnosed breast cancer. https://www.uptodate.com/contents/clinical-features-diagnosis-and-staging-of-newly-diagnosed-breast-cancer?source=search_result&search=invasive infiltrating ductal breast cancer&selectedTitle=1~150#H28333270. Published February 2, 2017. Accessed August 1, 2017.
    IDC – Invasive Ductal Carcinoma. Breastcancer.org. http://www.breastcancer.org/symptoms/types/idc. Accessed August 1, 2017.

    Incorrect

    Correct Answer: (C) The patient has four risk factors present for breast cancer. She has a previous history of a benign breast condition (fibrocystic breast disease), she had her first child after the age of 30, she has a family history of breast cancer and she is 56 years old.
    Incorrect Answers: (A) There is more than one risk factor.
    (B) There are more than three risk factors.
    (D) There are not five risk factors; hypertension is not a risk factor for breast cancer.

    Discussion:
    Invasive ductal carcinoma also known as infiltrating ductal carcinoma is the most common type of invasive breast cancer. It accounts for 70-80% of invasive breast lesions. (1) Although it can be seen in women of any age it is most commonly seen in women over the age of 55 (3). IDC is a cancer that began growing in the milk ducts of the breast and has invaded the fatty tissue of the breast as well. Cytological features of the involved cells range from bland to highly malignant. The malignant cells prompt a fibrous response as they infiltrate the breast tissue which is what causes a clinically palpable mass, the density that can be seen radiologically, and solid sonographic characteristics of classic invasive carcinomas (1). It is not clear what actually causes the abnormal cell growth but some risk factors have been identified. These risk factors include increasing age, personal history of benign breast disease, family history of breast cancer, first pregnancy after the age of 30 or nulliparity, history of DES exposure, hormone replacement therapy, obesity and genetic factors such as the BRCA1 or BRCA2 gene (2). Clinical features of invasive ductal carcinoma usually involve a mass or lesion that is a hard, immovable, lesion with irregular borders. However, the clinical features cannot consistently distinguish a benign tumor from a malignant one. If the cancer is locally advanced the signs can include axillary adenopathy or skin findings such as erythema, or dimpling of the overlying skin known as peau d’orange. If the cancer has metastases the symptoms can greatly vary depending on what organs are involved. The most common site of metastatic involvement is the bone, followed by the liver and lungs. Signs can include back or leg pain, abdominal pain, nausea, jaundice, and shortness of breath or cough. Most commonly however, patients present due to an abnormal mammogram finding (4).
    Classic findings of breast cancer on mammogram include the presence of a soft tissue mass or density as well as clustered microcalcifications invading to surrounding tissues. Breast ultrasound is often used in younger women due to the density of their breasts as well as to distinguish a benign versus malignant lesion. Sonographic features of breast malignancy include the presence of “spiculation, hypoechogenicity, microlobulation, internal calcifications, shadowing, a lesion taller than it is wide and angular margins” (4). Magnetic resonance imaging (MRI) is typically used to screen women at high risk for breast cancer as well as to further distinguish cancers seem on mammogram or ultrasound. Even though nearly all invasive breast cancers will enhance on gadolinium contrast-enhanced MRI, it is still not specific enough to remove the need for biopsy. Normally, once a lesion or mass is seen on mammogram, ultrasound, and/or MRI and it is suspicious for malignancy, it will be biopsied either using FNA or a core needle biopsy (4).
    As far as management and treatment goes, it really depends on the stage and grade of the cancer as well as the patient characteristics. In most cases, treatment will begin with surgery not only to remove the cancer but also to determine if any lymph nodes are involved. Possible surgical options include lumpectomy or different variations of mastectomy including partial, total and radical. Sentinel lymph node and axillary dissections are also usually done during the procedure to help stage the cancer and see the extent to which it has spread to the lymphatic system. Radiation and chemotherapy are also treatment options again, depending on the stage of the cancer and the extent to which it has metastasized (3).
    References:
    Bleiweiss IJ. Pathology of Breast Cancer. Pathology of breast cancer. https://www.uptodate.com/contents/pathology-of-breast-cancer?source=search_result&search=invasive ductal carcinoma&selectedTitle=1~150. Published June 17, 2016. Accessed August 1, 2017.
    Ductal carcinoma in situ (DCIS). Mayo Clinic. http://www.mayoclinic.org/diseases-conditions/dcis/basics/definition/con-20031842. Published June 14, 2014. Accessed August 1, 2017.
    Esserman L J, Joe BN. Clinical features, diagnosis, and staging of newly diagnosed breast cancer. Clinical features, diagnosis, and staging of newly diagnosed breast cancer. https://www.uptodate.com/contents/clinical-features-diagnosis-and-staging-of-newly-diagnosed-breast-cancer?source=search_result&search=invasive infiltrating ductal breast cancer&selectedTitle=1~150#H28333270. Published February 2, 2017. Accessed August 1, 2017.
    IDC – Invasive Ductal Carcinoma. Breastcancer.org. http://www.breastcancer.org/symptoms/types/idc. Accessed August 1, 2017.

  186. Question 186 of 220
    186. Question

    A 17 year-old male is referred to your office by his PCP after reporting new onset of frank blood in stools after 6 weeks of intermittent diarrhea. CBC, LFT’s, BMP, and stool studies were all initiated and only Signiant lab result of note was ESR elevated at 41. Decision was made at your office for a complete GI workup complete with an upper and lower endoscopy. Endoscopy of the colon with multiple biopsy retrievals concluded the diagnosis of Ulcerative Colitis and the patient was started on Lialda (mesalamine) 3.6mg daily. Which of the following are you most likely to read on the biopsy pathology results confirming the diagnosis of Ulcerative Colitis?

    Correct

    Answer is C.

    Discussion:
    Ulcerative Colitis is an inflammatory disease that effects the colon. Ulcerative Colitis is a subcategory of Inflammatory Bowel Disease and often has a similar presentation to its counterpart, Crohn’s Disease. Ulcerative Colitis is an inflammatory process which is believed to be idiopathic, although there are many factors that are thought to influence the development of ulcerative colitis, there is no root underlying cause to the disease progression. Some factors that are believed to influence the development of ulcerative colitis are heredity, triggers to the immune system and environmental factors such as viruses 1. All of these factors are suspected to have an impact on the body’s immune system and inflammation response in the large intestine.
    Ulcerative colitis is defined as an inflammatory condition characterized by relapsing and remitting episodes of inflammation limited to the mucosal layer of the colon. While Crohn’s disease is characterized by involvement of any portion of the GI tract, Ulcerative colitis involves the colon, starting in the rectum and spreading proximally. Ulcerative Colitis commonly presents in the younger populations commonly between the ages of 15 and 35, but has also be documented in children younger than 15 years of age. According to the Crohn’s and Ulcerative Colitis Foundation over 500,000 Americans are living with Ulcerative Colitis.
    In patients diagnosed with Ulcerative Colitis, pathological changes to the normal mucosal layer of the colon include inflammatory and morphological changes. While the underlying mechanism of the direct cause of these mucosal changes are unknown, there are multiple hypotheses that result in these changes. One hypothesis is that there is an autoimmune response to a mucosal antigen which results in morphological changes to the mucosal layer of the large intestine causing inflammation. Another hypothesis explores the possibly of a dysregulation of the immune function to fight of normal bacterial and flora found in the large intestine. Finally, there is also a hypothesis that explores the immune system dysregulation to fight off and destroy viruses and in result continuous inflammation of the large intestine results. The inflammation seen in Ulcerative Colitis affects the mucosa and sometimes the submucosa, and often a clear border is seen between normal and affected tissue. Early in the disease and with mild presentation, the mucous membrane is erythematous, with loss of the normal vascular pattern and depicts a friable pattern and often with scattered hemorrhagic areas. In severe presentations morphologic changes include scattered large mucosal ulcers.
    Many patients with Ulcerative colitis may present in early stages with minor and vague abdominal complaints; such as diarrhea, gas, bloating, crampy pain and fatigue. Important history to take on a patient with these complaints and suspicion of UC are family history, recent travel, medication uses and previous radiation exposures. In more active times of disease, patients with complain of bloody stools, bouts of diarrhea, rectal urgency, fatigue and possibly fever. On exam it is important to look for signs of dehydration with multiple day history of diarrhea, such as orthostatic readings or urine analysis. Physical exam may yield unremarkable findings or lower quadrant abdominal pain to palpation. During a severe flare, patients may present with more systemic symptoms such as fever and night sweats and can have arthritis which is a frequent extra-intestinal finding.
    Patients who present with a high suspicion of these complaints should have blood work and stool studies completed. Routine blood work such as a CBC, BMP, LFTs, as well as inflammatory markers CRP and ESR should be ordered. Stool studies such as C.Diff, ova and parasite, fecal calprotectin and lactoferrin should also be considered to rule out other infectious causes2. Blood work may yield signs of anemias or elevated inflammatory markers. Low levels of Albumin and electrolyte abnormalities are also commonly seen in patients with ulcerative colitis2. While blood work may help strengthen the suspicion of UC, these markers are not diagnostic for UC. Abdominal X-Ray may be preformed in patients with vague complaints and may show evidence of mucosal thickening or “thumbprinting”2. The definitive diagnosis must be obtained through biopsies through an endoscopy of the colon. Common findings from an endoscopy of the colon are marked erythema and loss of vascular markings of the colon mucosa, as well friable tissue and bloody exudate. Inflammation most always involved the rectum in diagnosis of ulcerative colitis and is commonly an initial presentation on endoscopy. Other common and classic findings on endoscopy of the colon in a patient with UC are pseudo polyps and crypt abscesses, which represent an acute flare of disease3.
    Once definitive diagnosis has been made many patients will undergo therapy to control the flare of the disease. First line treatment for UC is a topical 5-aminosalicyclic (5-ASA) medication that comes in many different forms such as oral, enemas, and suppositories. Commonly patients will prefer oral medication but for those with more severe symptoms or recurrent flare, other combination routes can be given as well. According to UpToDate guidelines maintenance therapy is not recommended to patients who have been diagnosed with a first episode or have mild disease and have responded will to 5-ASA treatment2. Patients who do not have good results and experience many episodes of recurrence on the 5-ASA can also be placed on oral corticosteroids with a taper for symptoms. Patients are also encouraged to watch their diets and avoid foods that may be irritating to the stomach. Ulcerative colitis is a lifelong disorder and the goal is untimely reemission of symptoms and preventing flares of disease.

    References:

    1. Crohn’s Disease Diagnosis & Testing. Crohn’s and Colitis’. https://www.crohnsandcolitis.com/crohns/testing-for-crohns. Accessed August 16, 2017.

    2. Peppercorn, MD. Clinical manifestations, diagnosis, and prognosis of ulcerative colitis in adults. Post TW, ed. UpToDate. Waltham, MA: UpToDate Inc. http://www.uptodate.com

    3. Ulcerative Colitis – Gastrointestinal Disorders. Merck Manuals Professional Edition. http://www.merckmanuals.com/professional/gastrointestinal-disorders/inflammatory-bowel-disease-ibd/ulcerative-colitis. Accessed August 16, 2017.

    4. Ulcerative colitis. Ulcerative colitis (Idiopathic Proctocolitis) – treatment and medications. http://www.nmihi.com/c/ulcerative-colitis.htm. Accessed August 16, 2017.

    5. Hendrickson BA, Gokhale R, Cho JH. Clinical Aspects and Pathophysiology of Inflammatory Bowel Disease. Clinical Microbiology Reviews. 2002;15(1):79-94. doi:10.1128/CMR.15.1.79-94.2002.

    Incorrect

    Answer is C.

    Discussion:
    Ulcerative Colitis is an inflammatory disease that effects the colon. Ulcerative Colitis is a subcategory of Inflammatory Bowel Disease and often has a similar presentation to its counterpart, Crohn’s Disease. Ulcerative Colitis is an inflammatory process which is believed to be idiopathic, although there are many factors that are thought to influence the development of ulcerative colitis, there is no root underlying cause to the disease progression. Some factors that are believed to influence the development of ulcerative colitis are heredity, triggers to the immune system and environmental factors such as viruses 1. All of these factors are suspected to have an impact on the body’s immune system and inflammation response in the large intestine.
    Ulcerative colitis is defined as an inflammatory condition characterized by relapsing and remitting episodes of inflammation limited to the mucosal layer of the colon. While Crohn’s disease is characterized by involvement of any portion of the GI tract, Ulcerative colitis involves the colon, starting in the rectum and spreading proximally. Ulcerative Colitis commonly presents in the younger populations commonly between the ages of 15 and 35, but has also be documented in children younger than 15 years of age. According to the Crohn’s and Ulcerative Colitis Foundation over 500,000 Americans are living with Ulcerative Colitis.
    In patients diagnosed with Ulcerative Colitis, pathological changes to the normal mucosal layer of the colon include inflammatory and morphological changes. While the underlying mechanism of the direct cause of these mucosal changes are unknown, there are multiple hypotheses that result in these changes. One hypothesis is that there is an autoimmune response to a mucosal antigen which results in morphological changes to the mucosal layer of the large intestine causing inflammation. Another hypothesis explores the possibly of a dysregulation of the immune function to fight of normal bacterial and flora found in the large intestine. Finally, there is also a hypothesis that explores the immune system dysregulation to fight off and destroy viruses and in result continuous inflammation of the large intestine results. The inflammation seen in Ulcerative Colitis affects the mucosa and sometimes the submucosa, and often a clear border is seen between normal and affected tissue. Early in the disease and with mild presentation, the mucous membrane is erythematous, with loss of the normal vascular pattern and depicts a friable pattern and often with scattered hemorrhagic areas. In severe presentations morphologic changes include scattered large mucosal ulcers.
    Many patients with Ulcerative colitis may present in early stages with minor and vague abdominal complaints; such as diarrhea, gas, bloating, crampy pain and fatigue. Important history to take on a patient with these complaints and suspicion of UC are family history, recent travel, medication uses and previous radiation exposures. In more active times of disease, patients with complain of bloody stools, bouts of diarrhea, rectal urgency, fatigue and possibly fever. On exam it is important to look for signs of dehydration with multiple day history of diarrhea, such as orthostatic readings or urine analysis. Physical exam may yield unremarkable findings or lower quadrant abdominal pain to palpation. During a severe flare, patients may present with more systemic symptoms such as fever and night sweats and can have arthritis which is a frequent extra-intestinal finding.
    Patients who present with a high suspicion of these complaints should have blood work and stool studies completed. Routine blood work such as a CBC, BMP, LFTs, as well as inflammatory markers CRP and ESR should be ordered. Stool studies such as C.Diff, ova and parasite, fecal calprotectin and lactoferrin should also be considered to rule out other infectious causes2. Blood work may yield signs of anemias or elevated inflammatory markers. Low levels of Albumin and electrolyte abnormalities are also commonly seen in patients with ulcerative colitis2. While blood work may help strengthen the suspicion of UC, these markers are not diagnostic for UC. Abdominal X-Ray may be preformed in patients with vague complaints and may show evidence of mucosal thickening or “thumbprinting”2. The definitive diagnosis must be obtained through biopsies through an endoscopy of the colon. Common findings from an endoscopy of the colon are marked erythema and loss of vascular markings of the colon mucosa, as well friable tissue and bloody exudate. Inflammation most always involved the rectum in diagnosis of ulcerative colitis and is commonly an initial presentation on endoscopy. Other common and classic findings on endoscopy of the colon in a patient with UC are pseudo polyps and crypt abscesses, which represent an acute flare of disease3.
    Once definitive diagnosis has been made many patients will undergo therapy to control the flare of the disease. First line treatment for UC is a topical 5-aminosalicyclic (5-ASA) medication that comes in many different forms such as oral, enemas, and suppositories. Commonly patients will prefer oral medication but for those with more severe symptoms or recurrent flare, other combination routes can be given as well. According to UpToDate guidelines maintenance therapy is not recommended to patients who have been diagnosed with a first episode or have mild disease and have responded will to 5-ASA treatment2. Patients who do not have good results and experience many episodes of recurrence on the 5-ASA can also be placed on oral corticosteroids with a taper for symptoms. Patients are also encouraged to watch their diets and avoid foods that may be irritating to the stomach. Ulcerative colitis is a lifelong disorder and the goal is untimely reemission of symptoms and preventing flares of disease.

    References:

    1. Crohn’s Disease Diagnosis & Testing. Crohn’s and Colitis’. https://www.crohnsandcolitis.com/crohns/testing-for-crohns. Accessed August 16, 2017.

    2. Peppercorn, MD. Clinical manifestations, diagnosis, and prognosis of ulcerative colitis in adults. Post TW, ed. UpToDate. Waltham, MA: UpToDate Inc. http://www.uptodate.com

    3. Ulcerative Colitis – Gastrointestinal Disorders. Merck Manuals Professional Edition. http://www.merckmanuals.com/professional/gastrointestinal-disorders/inflammatory-bowel-disease-ibd/ulcerative-colitis. Accessed August 16, 2017.

    4. Ulcerative colitis. Ulcerative colitis (Idiopathic Proctocolitis) – treatment and medications. http://www.nmihi.com/c/ulcerative-colitis.htm. Accessed August 16, 2017.

    5. Hendrickson BA, Gokhale R, Cho JH. Clinical Aspects and Pathophysiology of Inflammatory Bowel Disease. Clinical Microbiology Reviews. 2002;15(1):79-94. doi:10.1128/CMR.15.1.79-94.2002.

  187. Question 187 of 220
    187. Question

    A 44 year old male presents to the emergency department complaining of abdominal pain. He states he has never felt anything like this in the past. He rates his pain as 8/10 and describes it as constant pain in his upper abdomen that radiates to his back. He says that he went out and had “a few too many drinks last night” but his stomach felt okay when he went to bed. At some point during the night his pain came on suddenly and hasn’t gone away. Labs were done on this patient and his serum amylase was 458U/L (normal value: 23-85U/L), lipase was 3948U/L (normal value: 0-160U/L), and triglycerides were 1328 (normal value: <159mg/dL). What is the best treatment option for this patient?

    Correct

    In the United States, one of the most frequent GI causes for hospitalization is pancreatitis1. Pancreatitis has many etiologies, and may be acute or chronic. The most common cause of acute pancreatitis is gallstones, followed by alcohol use. Smoking cigarettes is not a known cause on its own, but when combined with alcohol use the effect will be synergistic and the likelihood of getting acute pancreatitis is heightened1. The third most common cause of pancreatitis is hypertriglyceridemia2. Hypertriglyceridemia is determined when fasting triglycerides are greater than 150 mg/dL. Mild hypertriglyceridemia is 150-199 mg/dL, moderate is 200-999mg/dL, severe is 1000-1999mg/dL and very severe hypertriglyceridemia is greater than 2000 mg/dL. When triglyceride levels are above 500mg/dL there is a progressive risk for acute pancreatitis. Levels above 1000 mg/dL increases the risk of acute pancreatitis even further2. Drinking alcohol can also contribute to hypertriglyceridemia because it increases serum triglyceride concentrations in a “dose dependent” manner. Having hypertriglyceride-induced pancreatitis has a worse outcome than other types of pancreatitis2.
    When a person has hypertriglyceridemia, they have an increased concentration of chylomicrons3. Chylomicrons are large lipoproteins with very low density that are formed 1-3 hours after eating, and are typically cleared within 8 hours. They are a derivative of dietary fat and their job is to carry triglycerides throughout the body4. When the triglyceride level is above 1000mg/dL, the concentration of chylomicrons is always increased3. Due to their large, low density nature, they may obstruct capillaries leading to local ischemia and acidemia. This damage can expose triglycerides to pancreatic lipase which will further degrade the triglycerides to free fatty acids causing cytotoxic injury. This local injury will increase the inflammatory mediators and free radicals in the body, and eventually manifest into acute pancreatitis3.
    When working a patient up for pancreatitis it is important to ask about their history and family history of gallstones since that is the number one cause of pancreatitis. It is also important to ask about alcohol use and diet since pancreatitis is also commonly caused from excess alcohol use, and from hypertriglyceridemia. If the patient smokes cigarettes as well as drinking alcohol they are even more likely to have an episode of acute pancreatitis. The patient used in this case had a history of alcohol abuse, cigarette smoking, dyslipidemia, and coronary artery disease which are all pertinent to pancreatitis. On physical exam, his epigastric region was extremely tender to palpation, with pain rising to 10/10. He described his pain as a sudden onset, chronic pain in his upper abdomen radiating to his back, which is a very typical presentation of acute pancreatitis. His labs showed extremely high lipase at 3948U/L, and his liver enzymes were also elevated. On first draw his triglycerides and cholesterol/LDL were unable to detect due to specimen integrity, but they were estimated to be extremely elevated based on appearance. The tube showed a thick lactescent serum indicative of high triglycerides. Between that and his onset of acute pancreatitis it was estimated that his triglycerides were over 1000mg/dL. The exact level of triglycerides necessary to induce acute pancreatitis is unknown, but it is believed to happen at a level over 1000mg/dL5.
    When it comes to acute pancreatitis caused by hypertriglyceridemia, there are a few treatment options to choose from. The first line choice is apheresis, however this is not always available. Apheresis is a procedure that removes whole blood from the patient and runs it through a machine that will separate the blood into individual components. This process is allowing a specific component to be removed, and then the remaining components will be returned back into the blood stream of the patient. This process typically takes about two hours6. This is a great option for hypertriglyceridemia-induced pancreatitis because it can remove the unwanted lipids from the blood.
    Another option to decrease triglycerides is using an insulin drip. This method is preferred over apheresis if the patient has a glucose level above 500mg/dL. It is also used when apheresis is not available2. Hypertriglyceridemia is caused when there is a direct lipoprotein lipase deficiency or a decrease in lipoprotein lipase activity3. Lipoprotein lipase is an enzyme that accelerates chylomicron and VLDL metabolism to glycerol and free fatty acids2. Insulin will enhance lipoprotein lipase activity and help reduce triglyceride levels3. The protocol for using insulin to reduce triglycerides is to set up an IV drip of 0.1 – 0.3 units/kg/hour. If a patient is not hyperglycemic and an insulin drip is started, IV glucose supplementation like D5W must be added to prevent hypoglycemia. The insulin drip should be discontinued when triglycerides return to a level below 500mg/dL2.
    In the case study question of the best treatment option for our patient, “Keep this patient NPO, give IV fluids, insulin drip at 0.2 units/kg/hour and pain management” is the correct answer. The first answer is incorrect, because the role of heparin in reducing triglycerides is controversial. It has been used with some success, but the problem with using heparin is that after the initial benefit of the drug, there is a rebound effect that will increase hepatic degradation of lipoprotein lipase which further decreases plasma stores of lipoprotein lipase and ultimately results in an increase in chylomicrons2. High levels of chylomicrons contributed to the cause of acute pancreatitis in the first place, so it is best to avoid this. The second answer, “Keep the patient NPO and give pain management,” is incorrect because although you want to keep the patient NPO and relieve their pain, this is not doing enough for the patient. It is important to resuscitate with IV fluids. The final answer that includes NPO, pain management, and IV fluids is still incorrect for this particular patient. Although most cases of pancreatitis are treated by doing exactly this combination, our patient has triglycerides greater than 1000mg/dL. Without getting his triglyceride levels to decrease his pancreatitis will not resolve. The best answer is adding the insulin IV drip because that will be able to get his triglyceride level down. That in combination with NPO, IV fluids, and pain management is the treatment of choice to obtain the best possible outcome for this patient.

    References
    1. Yadav D, Lowenfels A. The Epidemiology of Pancreatitis and Pancreatic Cancer. National Center for Biotechnology Information. 2013. Available at: https://www.ncbi.nlm.nih.gov/pmc/articles/PMC3662544/. Accessed August 5, 2017.
    2. Gelrud A, Whitcomb D. Hypertriglyceridemia-induced acute pancreatitis. UpToDate. 2015. Available at: https://www.uptodate.com/contents/hypertriglyceridemia-induced-acute-pancreatitis. Accessed August 2, 2017.
    3. Frenkel D. Why Does Hypertriglyceridemia Lead to Pancreatitis? | Clinical Correlations. Clinical Correlations. 2007. Available at: http://www.clinicalcorrelations.org/?p=478. Accessed August 2, 2017.
    4. Engelking L. Chylomicron – an overview | ScienceDirect Topics. Sciencedirectcom. 2015. Available at: http://www.sciencedirect.com/topics/neuroscience/chylomicron. Accessed August 5, 2017.
    5. Tamás. Baranyai, ViktAória. Terzin, ágota. Vajda, Tibor. Wittmann & László. Czakó (2012) Hypertriglyceridemia causes more severe course of acute pancreatitis, Clinical Lipidology, 7:6, 731-736. Accessed August 5, 2017
    6. Stoppler M. Apheresis: Facts on Platelet Donation Procedures. MedicineNet. 2016. Available at: http://www.medicinenet.com/hemapheresis/article.htm. Accessed August 5, 2017.

    Incorrect

    In the United States, one of the most frequent GI causes for hospitalization is pancreatitis1. Pancreatitis has many etiologies, and may be acute or chronic. The most common cause of acute pancreatitis is gallstones, followed by alcohol use. Smoking cigarettes is not a known cause on its own, but when combined with alcohol use the effect will be synergistic and the likelihood of getting acute pancreatitis is heightened1. The third most common cause of pancreatitis is hypertriglyceridemia2. Hypertriglyceridemia is determined when fasting triglycerides are greater than 150 mg/dL. Mild hypertriglyceridemia is 150-199 mg/dL, moderate is 200-999mg/dL, severe is 1000-1999mg/dL and very severe hypertriglyceridemia is greater than 2000 mg/dL. When triglyceride levels are above 500mg/dL there is a progressive risk for acute pancreatitis. Levels above 1000 mg/dL increases the risk of acute pancreatitis even further2. Drinking alcohol can also contribute to hypertriglyceridemia because it increases serum triglyceride concentrations in a “dose dependent” manner. Having hypertriglyceride-induced pancreatitis has a worse outcome than other types of pancreatitis2.
    When a person has hypertriglyceridemia, they have an increased concentration of chylomicrons3. Chylomicrons are large lipoproteins with very low density that are formed 1-3 hours after eating, and are typically cleared within 8 hours. They are a derivative of dietary fat and their job is to carry triglycerides throughout the body4. When the triglyceride level is above 1000mg/dL, the concentration of chylomicrons is always increased3. Due to their large, low density nature, they may obstruct capillaries leading to local ischemia and acidemia. This damage can expose triglycerides to pancreatic lipase which will further degrade the triglycerides to free fatty acids causing cytotoxic injury. This local injury will increase the inflammatory mediators and free radicals in the body, and eventually manifest into acute pancreatitis3.
    When working a patient up for pancreatitis it is important to ask about their history and family history of gallstones since that is the number one cause of pancreatitis. It is also important to ask about alcohol use and diet since pancreatitis is also commonly caused from excess alcohol use, and from hypertriglyceridemia. If the patient smokes cigarettes as well as drinking alcohol they are even more likely to have an episode of acute pancreatitis. The patient used in this case had a history of alcohol abuse, cigarette smoking, dyslipidemia, and coronary artery disease which are all pertinent to pancreatitis. On physical exam, his epigastric region was extremely tender to palpation, with pain rising to 10/10. He described his pain as a sudden onset, chronic pain in his upper abdomen radiating to his back, which is a very typical presentation of acute pancreatitis. His labs showed extremely high lipase at 3948U/L, and his liver enzymes were also elevated. On first draw his triglycerides and cholesterol/LDL were unable to detect due to specimen integrity, but they were estimated to be extremely elevated based on appearance. The tube showed a thick lactescent serum indicative of high triglycerides. Between that and his onset of acute pancreatitis it was estimated that his triglycerides were over 1000mg/dL. The exact level of triglycerides necessary to induce acute pancreatitis is unknown, but it is believed to happen at a level over 1000mg/dL5.
    When it comes to acute pancreatitis caused by hypertriglyceridemia, there are a few treatment options to choose from. The first line choice is apheresis, however this is not always available. Apheresis is a procedure that removes whole blood from the patient and runs it through a machine that will separate the blood into individual components. This process is allowing a specific component to be removed, and then the remaining components will be returned back into the blood stream of the patient. This process typically takes about two hours6. This is a great option for hypertriglyceridemia-induced pancreatitis because it can remove the unwanted lipids from the blood.
    Another option to decrease triglycerides is using an insulin drip. This method is preferred over apheresis if the patient has a glucose level above 500mg/dL. It is also used when apheresis is not available2. Hypertriglyceridemia is caused when there is a direct lipoprotein lipase deficiency or a decrease in lipoprotein lipase activity3. Lipoprotein lipase is an enzyme that accelerates chylomicron and VLDL metabolism to glycerol and free fatty acids2. Insulin will enhance lipoprotein lipase activity and help reduce triglyceride levels3. The protocol for using insulin to reduce triglycerides is to set up an IV drip of 0.1 – 0.3 units/kg/hour. If a patient is not hyperglycemic and an insulin drip is started, IV glucose supplementation like D5W must be added to prevent hypoglycemia. The insulin drip should be discontinued when triglycerides return to a level below 500mg/dL2.
    In the case study question of the best treatment option for our patient, “Keep this patient NPO, give IV fluids, insulin drip at 0.2 units/kg/hour and pain management” is the correct answer. The first answer is incorrect, because the role of heparin in reducing triglycerides is controversial. It has been used with some success, but the problem with using heparin is that after the initial benefit of the drug, there is a rebound effect that will increase hepatic degradation of lipoprotein lipase which further decreases plasma stores of lipoprotein lipase and ultimately results in an increase in chylomicrons2. High levels of chylomicrons contributed to the cause of acute pancreatitis in the first place, so it is best to avoid this. The second answer, “Keep the patient NPO and give pain management,” is incorrect because although you want to keep the patient NPO and relieve their pain, this is not doing enough for the patient. It is important to resuscitate with IV fluids. The final answer that includes NPO, pain management, and IV fluids is still incorrect for this particular patient. Although most cases of pancreatitis are treated by doing exactly this combination, our patient has triglycerides greater than 1000mg/dL. Without getting his triglyceride levels to decrease his pancreatitis will not resolve. The best answer is adding the insulin IV drip because that will be able to get his triglyceride level down. That in combination with NPO, IV fluids, and pain management is the treatment of choice to obtain the best possible outcome for this patient.

    References
    1. Yadav D, Lowenfels A. The Epidemiology of Pancreatitis and Pancreatic Cancer. National Center for Biotechnology Information. 2013. Available at: https://www.ncbi.nlm.nih.gov/pmc/articles/PMC3662544/. Accessed August 5, 2017.
    2. Gelrud A, Whitcomb D. Hypertriglyceridemia-induced acute pancreatitis. UpToDate. 2015. Available at: https://www.uptodate.com/contents/hypertriglyceridemia-induced-acute-pancreatitis. Accessed August 2, 2017.
    3. Frenkel D. Why Does Hypertriglyceridemia Lead to Pancreatitis? | Clinical Correlations. Clinical Correlations. 2007. Available at: http://www.clinicalcorrelations.org/?p=478. Accessed August 2, 2017.
    4. Engelking L. Chylomicron – an overview | ScienceDirect Topics. Sciencedirectcom. 2015. Available at: http://www.sciencedirect.com/topics/neuroscience/chylomicron. Accessed August 5, 2017.
    5. Tamás. Baranyai, ViktAória. Terzin, ágota. Vajda, Tibor. Wittmann & László. Czakó (2012) Hypertriglyceridemia causes more severe course of acute pancreatitis, Clinical Lipidology, 7:6, 731-736. Accessed August 5, 2017
    6. Stoppler M. Apheresis: Facts on Platelet Donation Procedures. MedicineNet. 2016. Available at: http://www.medicinenet.com/hemapheresis/article.htm. Accessed August 5, 2017.

  188. Question 188 of 220
    188. Question

    A young woman who has been happily married for over two years was brought to the hospital after her boyfriend crashed the motorcycle that she was riding on the back of. At the hospital, the patient’s husband reports that their marriage was a smooth one until a few weeks ago when he started noticing that she has “been acting weird”. The husband reports that she has gone on lavish shopping trips maxing out their credit cards, is going out all night partying and not coming home, and has now found out she has a boyfriend. Which of the following disorders is this patient most likely to be diagnosed with?

    Correct

    Explanation: Bipolar I is characterized by extreme mania that includes symptoms such as lack of sleep, impulsivity, risky behavior activities, high volume and rate of speech, flight of ideas, elated and expansive mood, increased activity, psychomotor agitation, grandiosity, and self perceived talents. The patient in this question has been exhibiting excessive risk taking behaviors for over 7 days that include excessive spending of money, hypersexual tendencies, and increased impulsivity. Choice A- Schizophrenia Paranoid Type is incorrect because this diagnosis requires the presence of active delusions and hallucinations, disorganized behavior and speech, and negative symptoms. Choice B- Cyclothymia is incorrect because it is a less severe form of bipolar disorder, with alternating periods of hypomania and moderate depression. Symptoms also must be present for at least 2 years. Choice D- Borderline Personality Disorder is incorrect because this diagnosis requires a pattern of unstable personal relationships, self-image, and affect that are seen as efforts to avoid abandonment, unstable relationships, and recurrent suicidal behavior.
    Discussion:
    Bipolar Disorder, also formerly known as manic-depressive disorder is a mental illness that results in dramatic shifts in mood and energy that may disrupt a patient’s ability to complete their day-to day tasks (1). This mental condition causes mood swings that are more excessive than the typical ups and downs, they are characterized by extreme emotional highs known as mania or hypomania and extreme lows that are consistent with depression (2). Although the average age of onset is 25, patients’ can be diagnosed at any age, most uncommonly during childhood. Bipolar disorder is equally as common in men as in women. In the U.S 2.6% of the population has been diagnosed with bipolar disorder, although this number is thought to be widely underreported. Within the number of diagnosed cases 83% have been classified as severe. Suicide rates are currently higher in men. (3).
    Currently, there are four different basic subcategories of bipolar disorder. Bipolar I disorder is diagnosed by manic episodes that at minimum lasts 7 days, or symptoms that are severe enough to require hospitalization. Typically, depressive symptoms are also present and lasting about two weeks. In this category it is also possible for depression with mixed features to also occur which is having manic and depressive symptoms at the same time. Bipolar II disorder is diagnosed by a patient having a pattern of depressive episodes and hypomanic episodes, but absence of full blown manic episodes. Cylcothymic Disorder is diagnosed by frequent periods of hypomanic symptoms as well as frequent periods of depressive symptoms lasting for at least 2 years. The symptoms of this disorder do not meet the diagnostic criteria for a hypomanic episode or a depressive episode. The final subcategory is just a category that doesn’t fit any of the above mentioned criteria. (1)
    To date a single cause of Bipolar Disorder has not been found but there are several factors that are thought to contribute to the disorder. The first factor is genetics, it is thought that a family history of bipolar disorder increases a patient’s overall risk. The second factor is stress, it has been found that a stressful event in one’s life may trigger a manic or depressive episode. The third factor is brain structure and function, currently there isn’t a brain scan that can diagnose bipolar disorder but researchers have found that people with bipolar versus people without have subtle difference in average size and activation of some brain structures. The fourth factor is biochemical factors, there is thought to be an up regulation of monoamine neurotransmission and receptor function as well as changes in the limbic system and prefrontal cortex in patients with Bipolar Disorder. (3)
    Diagnosis of a manic state in a patient with bipolar disorder can be characterized by lack of sleep, impulsivity, risky behavior activities, high volume and rate of speech, flight of ideas, elated and expansive mood, increased activity, psychomotor agitation, grandiosity, and self perceived talents. Bipolar II is mostly characterized by Major Depressive Disorder and at least one hypomanic episode during the patient’s life. Rapid cycling may be more common in Bipolar I than Bipolar II. Differential diagnosis and etiology is very similar is Bipolar I versus Bipolar II (1).
    For Bipolar I and Bipolar II, treatment options include mood stabilizers which are the mainstay and can be used for both acute and maintenance treatment. These include lithium which is gold standard, valproic acid, and lamotrigine/lamictal. Antipsychotics such as Zyprexa, Risperdal, and Seroquel may also be used especially for mania with psychosis since it is faster acting than a mood stabilizer. Antidepressants may also be prescribed but not usually as a monotherapy. For patients on Lithium antidepressants are useful in the severely depressed. Some non pharmalogical treatment include electroconvulsive therapy, psychotherapy such as CBT, good social support, and economic stability. It has been found that psychopharmatherapeutics and psychotherapy leads to the best outcome. For these patients the prognosis varies on level of treatment and adherence, but most will have a second episode 2-4 years after the first event (2).
    References:
    Bipolar Disorder. National Institute of Mental Health. https://www.nimh.nih.gov/health/topics/bipolar-disorder/index.shtml. Accessed September 23, 2017.

    Bipolar disorder. Mayo Clinic. http://www.mayoclinic.org/diseases-conditions/bipolar-disorder/home/ovc-20307967. Published February 15, 2017. Accessed September 23, 2017.

    NAMI. Bipolar disorder | NAMI: National Alliance on Mental Illness. https://www.nami.org/Learn-More/Mental-Health-Conditions/Bipolar-Disorder. Accessed September 23, 2017.

    Incorrect

    Explanation: Bipolar I is characterized by extreme mania that includes symptoms such as lack of sleep, impulsivity, risky behavior activities, high volume and rate of speech, flight of ideas, elated and expansive mood, increased activity, psychomotor agitation, grandiosity, and self perceived talents. The patient in this question has been exhibiting excessive risk taking behaviors for over 7 days that include excessive spending of money, hypersexual tendencies, and increased impulsivity. Choice A- Schizophrenia Paranoid Type is incorrect because this diagnosis requires the presence of active delusions and hallucinations, disorganized behavior and speech, and negative symptoms. Choice B- Cyclothymia is incorrect because it is a less severe form of bipolar disorder, with alternating periods of hypomania and moderate depression. Symptoms also must be present for at least 2 years. Choice D- Borderline Personality Disorder is incorrect because this diagnosis requires a pattern of unstable personal relationships, self-image, and affect that are seen as efforts to avoid abandonment, unstable relationships, and recurrent suicidal behavior.
    Discussion:
    Bipolar Disorder, also formerly known as manic-depressive disorder is a mental illness that results in dramatic shifts in mood and energy that may disrupt a patient’s ability to complete their day-to day tasks (1). This mental condition causes mood swings that are more excessive than the typical ups and downs, they are characterized by extreme emotional highs known as mania or hypomania and extreme lows that are consistent with depression (2). Although the average age of onset is 25, patients’ can be diagnosed at any age, most uncommonly during childhood. Bipolar disorder is equally as common in men as in women. In the U.S 2.6% of the population has been diagnosed with bipolar disorder, although this number is thought to be widely underreported. Within the number of diagnosed cases 83% have been classified as severe. Suicide rates are currently higher in men. (3).
    Currently, there are four different basic subcategories of bipolar disorder. Bipolar I disorder is diagnosed by manic episodes that at minimum lasts 7 days, or symptoms that are severe enough to require hospitalization. Typically, depressive symptoms are also present and lasting about two weeks. In this category it is also possible for depression with mixed features to also occur which is having manic and depressive symptoms at the same time. Bipolar II disorder is diagnosed by a patient having a pattern of depressive episodes and hypomanic episodes, but absence of full blown manic episodes. Cylcothymic Disorder is diagnosed by frequent periods of hypomanic symptoms as well as frequent periods of depressive symptoms lasting for at least 2 years. The symptoms of this disorder do not meet the diagnostic criteria for a hypomanic episode or a depressive episode. The final subcategory is just a category that doesn’t fit any of the above mentioned criteria. (1)
    To date a single cause of Bipolar Disorder has not been found but there are several factors that are thought to contribute to the disorder. The first factor is genetics, it is thought that a family history of bipolar disorder increases a patient’s overall risk. The second factor is stress, it has been found that a stressful event in one’s life may trigger a manic or depressive episode. The third factor is brain structure and function, currently there isn’t a brain scan that can diagnose bipolar disorder but researchers have found that people with bipolar versus people without have subtle difference in average size and activation of some brain structures. The fourth factor is biochemical factors, there is thought to be an up regulation of monoamine neurotransmission and receptor function as well as changes in the limbic system and prefrontal cortex in patients with Bipolar Disorder. (3)
    Diagnosis of a manic state in a patient with bipolar disorder can be characterized by lack of sleep, impulsivity, risky behavior activities, high volume and rate of speech, flight of ideas, elated and expansive mood, increased activity, psychomotor agitation, grandiosity, and self perceived talents. Bipolar II is mostly characterized by Major Depressive Disorder and at least one hypomanic episode during the patient’s life. Rapid cycling may be more common in Bipolar I than Bipolar II. Differential diagnosis and etiology is very similar is Bipolar I versus Bipolar II (1).
    For Bipolar I and Bipolar II, treatment options include mood stabilizers which are the mainstay and can be used for both acute and maintenance treatment. These include lithium which is gold standard, valproic acid, and lamotrigine/lamictal. Antipsychotics such as Zyprexa, Risperdal, and Seroquel may also be used especially for mania with psychosis since it is faster acting than a mood stabilizer. Antidepressants may also be prescribed but not usually as a monotherapy. For patients on Lithium antidepressants are useful in the severely depressed. Some non pharmalogical treatment include electroconvulsive therapy, psychotherapy such as CBT, good social support, and economic stability. It has been found that psychopharmatherapeutics and psychotherapy leads to the best outcome. For these patients the prognosis varies on level of treatment and adherence, but most will have a second episode 2-4 years after the first event (2).
    References:
    Bipolar Disorder. National Institute of Mental Health. https://www.nimh.nih.gov/health/topics/bipolar-disorder/index.shtml. Accessed September 23, 2017.

    Bipolar disorder. Mayo Clinic. http://www.mayoclinic.org/diseases-conditions/bipolar-disorder/home/ovc-20307967. Published February 15, 2017. Accessed September 23, 2017.

    NAMI. Bipolar disorder | NAMI: National Alliance on Mental Illness. https://www.nami.org/Learn-More/Mental-Health-Conditions/Bipolar-Disorder. Accessed September 23, 2017.

  189. Question 189 of 220
    189. Question

    A 16-year-old boy presents to the emergency department stating that he just took 20 tablets of 325mg Tylenol about 2 hours ago. He states that his girlfriend broke up with him and he felt like his life was over. He currently has no symptoms at all and is feeling well physically despite his depressed mood. What is the correct treatment for this patient?

    Correct

    Correct Answer: (D) The patient is presenting admitting to an acetaminophen overdose. He is stating the timeframe and the dose that was ingested. The appropriate treatment is to get an acetaminophen concentration and treat the patient with NAC if appropriate, which it most likely is in this case due to the large amount consumed.
    Incorrect Answers: (A) Even though a psych eval and 72-hour hold may indeed be necessary this is not the correct treatment as this doesn’t treat the active issue of possible hepatoxicity.
    (B) Discharging the patient home is incorrect because we don’t know the patient’s level of toxicity and therefore can’t predict his risk of hepatoxicity.
    (C) This is incorrect because activated charcoal is usually only used within the first hour after ingestion.

    Discussion:
    Acetaminophen is one of the most widely used over the counter oral analgesic and antipyretic. It is usually well tolerated and very safe when administered in the therapeutic dose range. When the dosage exceeds the therapeutic range, hepatotoxicity can occur. This is becoming increasingly common in the United States and acetaminophen toxicity has now replaced viral hepatitis as the most common cause of acute hepatic failure (1). Acetaminophen hepatotoxicity is estimated to account for approximately 48% of all acute liver failure diagnoses. In addition, some studies have shown that 29% of patients with acute liver failure secondary to acetaminophen toxicity end up undergoing a liver transplant, and ultimately have a 28% mortality rate (3).
    On average it takes approximately 4 hours from ingestion to peak acetaminophen level in the blood, with liquid formulations peaking somewhat quicker. Even the extended-release preparations are almost entirely absorbed by the body at the 4-hour mark. Primarily the liver metabolizes acetaminophen, which is why in overdose it is hepatotoxic. The majority of the metabolism is completed via Glucuronidation and sulfation. The remaining 5%-15% is oxidized through the p450 system, largely CYP2E1 (2).
    Acetaminophen overdoses happen both intentionally and unintentionally. The signs and symptoms can be pretty difficult to catch if the patient doesn´t present admitting to the overdose. Acute ingestions of acetaminophen are single ingestions occurring within a 4-hour period. When the patient presents with a clear history and a timeframe of less than 4 hours from ingestion, the protocol for treatment is pretty clear. Chronic ingestions on the other hand are not so clear-cut. A chronic ingestion is defined as multiple ingestions or a timeframe greater than 4 hours (2). Patients are usually asymptomatic for the first 24 to 48 hours. When they do start showing clinical signs it is due to end organ toxicity and the patient´s life is in danger (1). It is important for the clinician to know the time at which the overdose occurred and how much was ingested if possible. Minimum toxic doses of acetaminophen in a single ingestion that have severe risk of hepatotoxicity are 7.5 to 10g in adults and 150 to 200mg-kg in healthy children aged 1 to 6 years (1). Poor prognostic signs include multi-organ failure, which can include cerebral edema, renal failure, profound hypoglycemia and lactic acidosis, any signs of which should prompt an immediate liver transplant evaluation. As stated before most of the signs of unintentional toxic ingestion typically present later on, when the complications and end organ damage is already apparent (3).
    Treatment for acetaminophen overdoses consists of a few different things based on the time of ingestion, the amount the patient ingested or the level in the patient’s blood and the liver function. A nomogram called the Rumack-Matthew nomogram is used to calculate PO/IV N-Acetylcysteine (NAC) dosing for acetaminophen overdose and to determine toxic 4-hour acetaminophen levels. It looks at the acetaminophen concentration and the time post ingestion to give an estimate about possible toxicity and if to treat (4). Activated charcoal can be used as a gastrointestinal decontamination agent during the immediate postingestion time, ideally within 1 hour. If the patient has an acetaminophen concentration below the “possible toxicity” line on the Rumack-Matthew nomogram, they can be discharged home after they are cleared medically. However, if the patient falls above the “possible toxicity” line, they need to be treated with NAC and hospitalized for observation. NAC is close to 100% hepatoprotective when administered within 8 hours of ingestion of acetaminophen, but can also still be beneficial to patients who present more than 24 hours post ingestion. NAC can be given in both oral and IV form. If the damage is too much for the liver to handle and the NAC to overcome, the only other option is liver transplantation (1).

    References:
    Farrell SE. Acetaminophen Toxicity. Practice Essentials, Background, Pathophysiology. http://emedicine.medscape.com/article/820200-overview?pa=Nr1y61Ko11XC7v8pvuZCnb8l1YOpVLo7yOfSsThn6h%2FAMipBdjDfyXPfOm8VaALLeiwSmQLHuFrfjJk2gj%2F4wy8HoCpv88EfsR0xVZ6JHe4%3D. Published August 17, 2017. Accessed September 12, 2017.
    Mottram AM, Kumar AM. ACEP. Focus On: Acetaminophen Toxicity and Treatment // ACEP. https://www.acep.org/Clinical—Practice-Management/Focus-On–Acetaminophen-Toxicity-and-Treatment/. Published May 2007. Accessed September 12, 2017.
    Yoon E, Babar A, Choudhary M, Kutner M, Pyrsopoulos N. Acetaminophen-Induced Hepatotoxicity: a Comprehensive Update. Journal of Clinical and Translational Hepatology. https://www.ncbi.nlm.nih.gov/pmc/articles/PMC4913076/. Published June 28, 2016. Accessed September 12, 2017.
    Rumack BM. Acetaminophen Overdose and NAC Dosing. MDCalc. https://www.mdcalc.com/acetaminophen-overdose-nac-dosing. Accessed September 13, 2017.

    Incorrect

    Correct Answer: (D) The patient is presenting admitting to an acetaminophen overdose. He is stating the timeframe and the dose that was ingested. The appropriate treatment is to get an acetaminophen concentration and treat the patient with NAC if appropriate, which it most likely is in this case due to the large amount consumed.
    Incorrect Answers: (A) Even though a psych eval and 72-hour hold may indeed be necessary this is not the correct treatment as this doesn’t treat the active issue of possible hepatoxicity.
    (B) Discharging the patient home is incorrect because we don’t know the patient’s level of toxicity and therefore can’t predict his risk of hepatoxicity.
    (C) This is incorrect because activated charcoal is usually only used within the first hour after ingestion.

    Discussion:
    Acetaminophen is one of the most widely used over the counter oral analgesic and antipyretic. It is usually well tolerated and very safe when administered in the therapeutic dose range. When the dosage exceeds the therapeutic range, hepatotoxicity can occur. This is becoming increasingly common in the United States and acetaminophen toxicity has now replaced viral hepatitis as the most common cause of acute hepatic failure (1). Acetaminophen hepatotoxicity is estimated to account for approximately 48% of all acute liver failure diagnoses. In addition, some studies have shown that 29% of patients with acute liver failure secondary to acetaminophen toxicity end up undergoing a liver transplant, and ultimately have a 28% mortality rate (3).
    On average it takes approximately 4 hours from ingestion to peak acetaminophen level in the blood, with liquid formulations peaking somewhat quicker. Even the extended-release preparations are almost entirely absorbed by the body at the 4-hour mark. Primarily the liver metabolizes acetaminophen, which is why in overdose it is hepatotoxic. The majority of the metabolism is completed via Glucuronidation and sulfation. The remaining 5%-15% is oxidized through the p450 system, largely CYP2E1 (2).
    Acetaminophen overdoses happen both intentionally and unintentionally. The signs and symptoms can be pretty difficult to catch if the patient doesn´t present admitting to the overdose. Acute ingestions of acetaminophen are single ingestions occurring within a 4-hour period. When the patient presents with a clear history and a timeframe of less than 4 hours from ingestion, the protocol for treatment is pretty clear. Chronic ingestions on the other hand are not so clear-cut. A chronic ingestion is defined as multiple ingestions or a timeframe greater than 4 hours (2). Patients are usually asymptomatic for the first 24 to 48 hours. When they do start showing clinical signs it is due to end organ toxicity and the patient´s life is in danger (1). It is important for the clinician to know the time at which the overdose occurred and how much was ingested if possible. Minimum toxic doses of acetaminophen in a single ingestion that have severe risk of hepatotoxicity are 7.5 to 10g in adults and 150 to 200mg-kg in healthy children aged 1 to 6 years (1). Poor prognostic signs include multi-organ failure, which can include cerebral edema, renal failure, profound hypoglycemia and lactic acidosis, any signs of which should prompt an immediate liver transplant evaluation. As stated before most of the signs of unintentional toxic ingestion typically present later on, when the complications and end organ damage is already apparent (3).
    Treatment for acetaminophen overdoses consists of a few different things based on the time of ingestion, the amount the patient ingested or the level in the patient’s blood and the liver function. A nomogram called the Rumack-Matthew nomogram is used to calculate PO/IV N-Acetylcysteine (NAC) dosing for acetaminophen overdose and to determine toxic 4-hour acetaminophen levels. It looks at the acetaminophen concentration and the time post ingestion to give an estimate about possible toxicity and if to treat (4). Activated charcoal can be used as a gastrointestinal decontamination agent during the immediate postingestion time, ideally within 1 hour. If the patient has an acetaminophen concentration below the “possible toxicity” line on the Rumack-Matthew nomogram, they can be discharged home after they are cleared medically. However, if the patient falls above the “possible toxicity” line, they need to be treated with NAC and hospitalized for observation. NAC is close to 100% hepatoprotective when administered within 8 hours of ingestion of acetaminophen, but can also still be beneficial to patients who present more than 24 hours post ingestion. NAC can be given in both oral and IV form. If the damage is too much for the liver to handle and the NAC to overcome, the only other option is liver transplantation (1).

    References:
    Farrell SE. Acetaminophen Toxicity. Practice Essentials, Background, Pathophysiology. http://emedicine.medscape.com/article/820200-overview?pa=Nr1y61Ko11XC7v8pvuZCnb8l1YOpVLo7yOfSsThn6h%2FAMipBdjDfyXPfOm8VaALLeiwSmQLHuFrfjJk2gj%2F4wy8HoCpv88EfsR0xVZ6JHe4%3D. Published August 17, 2017. Accessed September 12, 2017.
    Mottram AM, Kumar AM. ACEP. Focus On: Acetaminophen Toxicity and Treatment // ACEP. https://www.acep.org/Clinical—Practice-Management/Focus-On–Acetaminophen-Toxicity-and-Treatment/. Published May 2007. Accessed September 12, 2017.
    Yoon E, Babar A, Choudhary M, Kutner M, Pyrsopoulos N. Acetaminophen-Induced Hepatotoxicity: a Comprehensive Update. Journal of Clinical and Translational Hepatology. https://www.ncbi.nlm.nih.gov/pmc/articles/PMC4913076/. Published June 28, 2016. Accessed September 12, 2017.
    Rumack BM. Acetaminophen Overdose and NAC Dosing. MDCalc. https://www.mdcalc.com/acetaminophen-overdose-nac-dosing. Accessed September 13, 2017.

  190. Question 190 of 220
    190. Question

    A 50 year-old male presents to the emergency room with a chief complaint of upper abdominal pain. Upon examining him the Physician Assistant notices that he is quite jaundice and his sclera are icteric. He has an extensive drinking history, average of 3 to 4 drinks daily, and a single episode of hematemesis one week prior to presentation. Other than steroids what pharmacotherapy should be added to decrease risk on complications.

    Correct

    B is the correct answer. PPI to decrease risk of GI bleed (upper: mainly due to patients presumed varices, from portal HTN). Folic acid and Thiamine for malnutrition and as a preventative measure for Wernicke’s encephalopathy. If there is any indication of alcohol withdrawal or DTs a CIWA should be initiated and Benzodiazepines used as indicated.
    A is incorrect because the patient doesn’t have any known symptoms of acute cholecystitis or biliary cholic, cholestyramine can also be used in indicated.
    C is incorrect because these supplements are not currently necessary.
    D is incorrect because it would be used in place of prednisolone, if used at all. This drug was more common in the past but it not frequently used due to side effects. Recent studies show that this drug can prevent AKI during a hospital stay but the longer a patient is on it mortality increases.

    Alcoholic Hepatitis is due to increased alcohol consumption over decades; binge drinking is less commonly the cause. Many hepatic conditions, alcoholic fatty liver disease, alcoholic hepatitis, and cirrhosis, are caused by alcohol abuse and can present with or without steatohepatitis. The amount of alcohol intake is thought to be about 3oz to 4oz daily for decades. Currently alcohol liver disease is the most common cause of chronic and end-stage liver disease with about 10% to 35% of patients who abuse alcohol having changes in liver histology.
    On physical exam abdominal pain and jaundice are common. Hepatomegaly is occasionally observed; from fatty liver, hepatocyte injury and protein retention. A liver bruit, gynecomastia and spider angiomatas are features of severe alcoholic hepatitis. Ascites may also be present, due to underlying cirrhosis and portal hypertension, and would be an indication to preform a paracentesis for question of SBP.
    A clinical diagnosis is made if the patient presents with jaundice, moderately elevated aminotransferases, an AST:ALT ratio ≥2, an increased bilirubin and a high INR. The addition of leukocytosis or a fever only strengthens the diagnosis. There are several clinical scoring systems that can be used to judge outcomes. Discriminate function, INR & Bili, predicts prognosis and the need pharmacotherapy. A value >32 requires corticosteroids or pentoxifylline and implies a 30d mortality ranging between 35% to 45%. LILLE score, age, albumin, bili days 1 & 7, creatinine and PT, predicts mortality in patients who do not respond to 7d steroid treatment. A score >0.45 indicates a poorer outcome. Child-Pugh score, tBili, albumin, INR, ascites, and encephalopathy, determines the necessity of liver transplant. MELD score, bili, creatinine & INR, is used to predict survival post transplant.
    The main treatment is prednisolone. Prednisone is not used due to the known hepatic metabolism. If patients are unstable and corticosteroids are not improving symptoms patients should be worked up for a liver transplant. The work up is quiet extensive and can take some time. That is why it is important to get the ball rolling before the patient direly needs a transplant. Mortality rates among patients who do not receive pharmacologic therapy are variable. Risk factors for increased mortality are: age, AKI, high bilirubin, high INR, Leukocytosis, Alcohol consumption >4 oz/d, presence of infection, hepatic encephalopathy and UGIB. The primary causes of death are hepatic failure, GIB, and sepsis.
    Recurrence is a possibility but the possibility of more severe complications is more striking. This is why it is so important to attend alcohol rehab and have GI follow up; screen for cirrhosis and other such complications. The screening may include; abdominal ultrasounds/MRI every 6 months and yearly EGD for varices if the patient has portal HTN.
    Resources
    Evangelos Akriviadis, Ravi Botla, William Briggs, Steven Han, Telfer Reynolds, Obaid Shakil. “Pentoxifylline improves short-term survival in severe acute alcoholic hepatitis: A double-blind, placebo-controlled trial” Gastroenterology. Volume 119, Issue 6, December 2000, Pages 1637-1648 http://www.sciencedirect.com/science/article/pii/S0016508500511834

    Lucey, M.R., M.D., Mathurin, P., M.D., Ph.D., & Morgan, T.R., M.D. “Alcoholic Hepatitis”. New England Journal on Medicine 2009; 360:2758-2769June 25, 2009DOI: 10.1056/NEJMra0805786 p. 2758-2769. http://www.nejm.org/doi/pdf/10.1056/NEJMra0805786

    Maddrey W.C. MD., Boitnott J.K. MD., Bedine M.S. MD., Webber F.L. MD., Mezey E. MD., White R.I. MD., “Corticosteroid therapy of alcoholic hepatitis” 1978 Gastroenterology Vol 75 No 2 p193-199 http://www.gastrojournal.org/article/0016-5085(78)90401-8/pdf

    National institute on Alcohol abuse and alcoholism. “Drinking Levels Defined” https://www.niaaa.nih.gov/alcohol-health/overview-alcohol-consumption/moderate-binge-drinking

    Thurzs, M.R, MD., Richardson, P, MD., Allison, M, Ph.D., Austin, A, MD., … Forrest, E.H, MD., “Prednisolone or Pentoxifylline for Alcoholic Hepatitis”. N Engl J Med 2015; 372:1619-1628April 23, 2015DOI: 10.1056/NEJMoa1412278. http://www.nejm.org/doi/pdf/10.1056/NEJMoa1412278

    ©2017 UpToDate, Inc. All Rights Reserved. “Alcoholic Hepatitis Clinical Manifestations and Diagnosis”. Topic 86934 Version 13.0 https://www.uptodate.com/contents/alcoholic-hepatitis-clinical-manifestations-and-diagnosis?source=search_result&search=alcohol%20hepatitis&selectedTitle=2~150

    ©2017 UpToDate, Inc. All Rights Reserved. “Management and Prognosis of Alcoholic Hepatitis”. Topic 87052 Version 22.0 https://www.uptodate.com/contents/management-and-prognosis-of-alcoholic-hepatitis?source=search_result&search=alcohol%20hepatitis&selectedTitle=1~150

    Incorrect

    B is the correct answer. PPI to decrease risk of GI bleed (upper: mainly due to patients presumed varices, from portal HTN). Folic acid and Thiamine for malnutrition and as a preventative measure for Wernicke’s encephalopathy. If there is any indication of alcohol withdrawal or DTs a CIWA should be initiated and Benzodiazepines used as indicated.
    A is incorrect because the patient doesn’t have any known symptoms of acute cholecystitis or biliary cholic, cholestyramine can also be used in indicated.
    C is incorrect because these supplements are not currently necessary.
    D is incorrect because it would be used in place of prednisolone, if used at all. This drug was more common in the past but it not frequently used due to side effects. Recent studies show that this drug can prevent AKI during a hospital stay but the longer a patient is on it mortality increases.

    Alcoholic Hepatitis is due to increased alcohol consumption over decades; binge drinking is less commonly the cause. Many hepatic conditions, alcoholic fatty liver disease, alcoholic hepatitis, and cirrhosis, are caused by alcohol abuse and can present with or without steatohepatitis. The amount of alcohol intake is thought to be about 3oz to 4oz daily for decades. Currently alcohol liver disease is the most common cause of chronic and end-stage liver disease with about 10% to 35% of patients who abuse alcohol having changes in liver histology.
    On physical exam abdominal pain and jaundice are common. Hepatomegaly is occasionally observed; from fatty liver, hepatocyte injury and protein retention. A liver bruit, gynecomastia and spider angiomatas are features of severe alcoholic hepatitis. Ascites may also be present, due to underlying cirrhosis and portal hypertension, and would be an indication to preform a paracentesis for question of SBP.
    A clinical diagnosis is made if the patient presents with jaundice, moderately elevated aminotransferases, an AST:ALT ratio ≥2, an increased bilirubin and a high INR. The addition of leukocytosis or a fever only strengthens the diagnosis. There are several clinical scoring systems that can be used to judge outcomes. Discriminate function, INR & Bili, predicts prognosis and the need pharmacotherapy. A value >32 requires corticosteroids or pentoxifylline and implies a 30d mortality ranging between 35% to 45%. LILLE score, age, albumin, bili days 1 & 7, creatinine and PT, predicts mortality in patients who do not respond to 7d steroid treatment. A score >0.45 indicates a poorer outcome. Child-Pugh score, tBili, albumin, INR, ascites, and encephalopathy, determines the necessity of liver transplant. MELD score, bili, creatinine & INR, is used to predict survival post transplant.
    The main treatment is prednisolone. Prednisone is not used due to the known hepatic metabolism. If patients are unstable and corticosteroids are not improving symptoms patients should be worked up for a liver transplant. The work up is quiet extensive and can take some time. That is why it is important to get the ball rolling before the patient direly needs a transplant. Mortality rates among patients who do not receive pharmacologic therapy are variable. Risk factors for increased mortality are: age, AKI, high bilirubin, high INR, Leukocytosis, Alcohol consumption >4 oz/d, presence of infection, hepatic encephalopathy and UGIB. The primary causes of death are hepatic failure, GIB, and sepsis.
    Recurrence is a possibility but the possibility of more severe complications is more striking. This is why it is so important to attend alcohol rehab and have GI follow up; screen for cirrhosis and other such complications. The screening may include; abdominal ultrasounds/MRI every 6 months and yearly EGD for varices if the patient has portal HTN.
    Resources
    Evangelos Akriviadis, Ravi Botla, William Briggs, Steven Han, Telfer Reynolds, Obaid Shakil. “Pentoxifylline improves short-term survival in severe acute alcoholic hepatitis: A double-blind, placebo-controlled trial” Gastroenterology. Volume 119, Issue 6, December 2000, Pages 1637-1648 http://www.sciencedirect.com/science/article/pii/S0016508500511834

    Lucey, M.R., M.D., Mathurin, P., M.D., Ph.D., & Morgan, T.R., M.D. “Alcoholic Hepatitis”. New England Journal on Medicine 2009; 360:2758-2769June 25, 2009DOI: 10.1056/NEJMra0805786 p. 2758-2769. http://www.nejm.org/doi/pdf/10.1056/NEJMra0805786

    Maddrey W.C. MD., Boitnott J.K. MD., Bedine M.S. MD., Webber F.L. MD., Mezey E. MD., White R.I. MD., “Corticosteroid therapy of alcoholic hepatitis” 1978 Gastroenterology Vol 75 No 2 p193-199 http://www.gastrojournal.org/article/0016-5085(78)90401-8/pdf

    National institute on Alcohol abuse and alcoholism. “Drinking Levels Defined” https://www.niaaa.nih.gov/alcohol-health/overview-alcohol-consumption/moderate-binge-drinking

    Thurzs, M.R, MD., Richardson, P, MD., Allison, M, Ph.D., Austin, A, MD., … Forrest, E.H, MD., “Prednisolone or Pentoxifylline for Alcoholic Hepatitis”. N Engl J Med 2015; 372:1619-1628April 23, 2015DOI: 10.1056/NEJMoa1412278. http://www.nejm.org/doi/pdf/10.1056/NEJMoa1412278

    ©2017 UpToDate, Inc. All Rights Reserved. “Alcoholic Hepatitis Clinical Manifestations and Diagnosis”. Topic 86934 Version 13.0 https://www.uptodate.com/contents/alcoholic-hepatitis-clinical-manifestations-and-diagnosis?source=search_result&search=alcohol%20hepatitis&selectedTitle=2~150

    ©2017 UpToDate, Inc. All Rights Reserved. “Management and Prognosis of Alcoholic Hepatitis”. Topic 87052 Version 22.0 https://www.uptodate.com/contents/management-and-prognosis-of-alcoholic-hepatitis?source=search_result&search=alcohol%20hepatitis&selectedTitle=1~150

  191. Question 191 of 220
    191. Question

    A 55-year-old male with diabetes mellitus and hypertension presents to the Emergency Department with concern of something stuck in his throat. He doesn’t recall exactly when it started but states that his throat became sore this morning while eating breakfast. He reports drooling and progressive decline in ability to breath. A full review of systems are negative. His vitals are stable with an elevated temperature to 99.8 degrees Fahrenheit. Due to concern of rapid clinical decline, a portable lateral soft tissue neck radiograph is obtained. The diagnostic image shows thickened aryepiglottic folds and loss of epiglottic vallecular space with no signs of a foreign body. What is the first line intervention for this patient?

    Correct

    Answer is C. Supraglottitis, inflammation of the epiglottis and adjacent structures, can present with a spectrum of chief complaints from sore throat to fever and airway compromise.2,7 It is most commonly of infectious etiology and in the past two decades became more common in the adult population.5,6 In some cases, the diagnosis can be made clinically, but radiographs, ultrasound and direct visualization aid in the evaluation of this condition.3 Even a low suspicion for supraglottitis warrants further assessment, as a patient’s clinical condition can rapidly decline.8 A management plan should be decided at initial presentation and centered around airway management.3 If a patient presents with drooling and signs of respiratory distress, as in this case, emergently securing the airway is a priority.1,8 This should be done with a skilled critical care team and otolaryngologist present, and include direct endoscopic evaluation (C) if the patient is currently able to maintain their own away.8 With potential for difficulty and complications, bedside intubation (B) should be avoided in cases of suproglottitis and the procedure should take place in a well-controlled, closely monitored environment.5 All patients presenting with infectious supraglottitis should receive empiric antibacterial coverage with IV ceftriaxone (A), however administration of this medication should not prolong initiation of airway intervention.3,5 Some studies advise the use of nebulized racemic epinephrine (D), IV steroids and supplemental oxygen but recent literature suggests that it has no effect on clinical outcome or length of ICU stay.3,4,8

    References
    1. Bizaki AJ, Numminen J, Vasama J-P, Laranne J, Rautiainen M. Acute supraglottitis in adults in Finland: Review and analysis of 308 cases. The Laryngoscope. 2011; 121(10):2107-2113.
    2. Galitz YS, Shoffel-Havakuk H, Cohen O, Halperin D, Lahav Y. Adult acute supraglottitis: Analysis of 358 patients for predictors of airway intervention. The Laryngoscope. 2017; 127(9):2106-2112.
    3. Lindquist B. Adult Epiglottitis: A Case Series. The Permanente Journal. November 2016.
    4. Navaratnam AV, Smith ME, Majeed A, Mcferran DJ. Adult supraglottitis: a potential airway emergency that can present in primary care. British Journal of General Practice. 2015; 65(631):99-100.
    5. Abdallah C. Acute epiglottitis: Trends, diagnosis and management. Saudi Journal of Anaesthesia. 2012; 6(3):279.
    6. Woods, CR. Epiglottitis (supraglottitis): Clinical features and diagnosis. Up to Date. May 2017. Retrieved from https://www.uptodate.com/contents/epiglottitis-supraglottitis-clinical-features-and-diagnosis?source=search_result&search=epiglottitis&selectedTitle=1~42
    7. Takata M, Fujikawa T, Goto R. Thumb sign: acute epiglottitis. BMJ Case Reports. 2016.
    8. Woods, CR. Epiglottitis (supraglottitis): Management. Up to Date. September 2017. Retrieved from https://www.uptodate.com/contents/epiglottitis-supraglottitis-management?source=search_result&search=epiglottitis&selectedTitle=2~42

    Incorrect

    Answer is C. Supraglottitis, inflammation of the epiglottis and adjacent structures, can present with a spectrum of chief complaints from sore throat to fever and airway compromise.2,7 It is most commonly of infectious etiology and in the past two decades became more common in the adult population.5,6 In some cases, the diagnosis can be made clinically, but radiographs, ultrasound and direct visualization aid in the evaluation of this condition.3 Even a low suspicion for supraglottitis warrants further assessment, as a patient’s clinical condition can rapidly decline.8 A management plan should be decided at initial presentation and centered around airway management.3 If a patient presents with drooling and signs of respiratory distress, as in this case, emergently securing the airway is a priority.1,8 This should be done with a skilled critical care team and otolaryngologist present, and include direct endoscopic evaluation (C) if the patient is currently able to maintain their own away.8 With potential for difficulty and complications, bedside intubation (B) should be avoided in cases of suproglottitis and the procedure should take place in a well-controlled, closely monitored environment.5 All patients presenting with infectious supraglottitis should receive empiric antibacterial coverage with IV ceftriaxone (A), however administration of this medication should not prolong initiation of airway intervention.3,5 Some studies advise the use of nebulized racemic epinephrine (D), IV steroids and supplemental oxygen but recent literature suggests that it has no effect on clinical outcome or length of ICU stay.3,4,8

    References
    1. Bizaki AJ, Numminen J, Vasama J-P, Laranne J, Rautiainen M. Acute supraglottitis in adults in Finland: Review and analysis of 308 cases. The Laryngoscope. 2011; 121(10):2107-2113.
    2. Galitz YS, Shoffel-Havakuk H, Cohen O, Halperin D, Lahav Y. Adult acute supraglottitis: Analysis of 358 patients for predictors of airway intervention. The Laryngoscope. 2017; 127(9):2106-2112.
    3. Lindquist B. Adult Epiglottitis: A Case Series. The Permanente Journal. November 2016.
    4. Navaratnam AV, Smith ME, Majeed A, Mcferran DJ. Adult supraglottitis: a potential airway emergency that can present in primary care. British Journal of General Practice. 2015; 65(631):99-100.
    5. Abdallah C. Acute epiglottitis: Trends, diagnosis and management. Saudi Journal of Anaesthesia. 2012; 6(3):279.
    6. Woods, CR. Epiglottitis (supraglottitis): Clinical features and diagnosis. Up to Date. May 2017. Retrieved from https://www.uptodate.com/contents/epiglottitis-supraglottitis-clinical-features-and-diagnosis?source=search_result&search=epiglottitis&selectedTitle=1~42
    7. Takata M, Fujikawa T, Goto R. Thumb sign: acute epiglottitis. BMJ Case Reports. 2016.
    8. Woods, CR. Epiglottitis (supraglottitis): Management. Up to Date. September 2017. Retrieved from https://www.uptodate.com/contents/epiglottitis-supraglottitis-management?source=search_result&search=epiglottitis&selectedTitle=2~42

  192. Question 192 of 220
    192. Question

    A 62 year-old male with history of pulmonary embolus presents to his primary care provider with frequent headaches, fatigue, early satiety and pruritus after his morning shower. He denies any history of smoking. He denies all other constitutional symptoms. Lab results show Hct 58, Hgb 19, platelets 545,000 and WBC 15.1. Physical exam shows a flushed face. Of the following choices, which is the most likely diagnosis?

    Correct

    Answer is C. Polycythemia vera is a myeloproliferative disorder with an increase in Hgb, Hct, WBC’s and plateletes. It is most commonly seen in men ages 50-60 yrs old. Signs/symptoms are due to increase in RBC mass and can also include headaches, dizziness, blurred vision, pruritis after hot bath, fatigue, early saiety, flushed face and splenomegaly.
    Essential thrombocythemia is also a myeloproliferative disorder that can have an increase in WBC and marked elevation in platelets (>600,000), but no increase in Hct. Symptoms include headache and dizziness.
    Chronic myeloid leukemia is another myeloproliferative disorder with a large increase in WBC (>30,000/mcL). Patients may also have an increase in platelets, but a normal Hct. Symptoms include fatigue, SOB, weight loss, or abdominal discomfort from splenomegaly.
    Secondary polycythemia is caused by hypoxia (ie: smoking). It results in an increase in Hct with normal WBC’s and platelets. Symptoms are usually related to the underlying cause. Physical exam may show cyanosis and clubbing. Splenomegaly would be absent.1
    Reference: Hammond DB. Myeloproliferative Disorder, Bone Marrow Failure & the Myelodysplastic Syndrome. MCPHS University: Clinical Medicine III. October 2016.

    DISCUSSION
    Polycythemia vera (PCV) is an acquired chronic myeloproliferative disorder.2 Myeloproliferative disorders are caused by clonal defects of hematopoietic stem cells. These stem cells produce myeloid, erythroid, and platelet cells. Polycythemia vera causes an overproduction of all three of these cell lines, with erythrocytes, or red blood cells (RBC), being the most prominent.3 Red blood cells are produced from bone marrow when erythropoietin is secreted by the kidneys, in response to cellular hypoxia. The exact etiology of PCV is unknown, however most believe it is caused by a mutation in exon 14 of Janus Kinase (JAK2 V617F), which is a signling molecule found in 95% of PCV cases.4 JAK2 may play a role in causing an increase in the sensitivity of erythroid cells to erythropoietin.3 PCV occurs in all ages. It is commonly seen in 50-60 year old patients, with the average age of diagnosis 60 years. Sixty percent of patients are men. It occurs in all populations. The incidence is lower in Japan when compared to the United States and Europe. The incidence is 2/100,000 – 18/100,000 in adults and increases with age.1
    PCV is also known as primary erythrocytosis, which means an increase in hematocrit or red blood cell mass in the absence of hypoxia. Patients should have a normal oxygen saturation on exam.3 This is different from secondary erythrocytosis, which is an increase in hematocrit from a response to another process (ie: hypoxia from smoking).2 Majority of patients with PCV are discovered incidentally. Symptoms occur from the increase in red blood cell mass. This includes hyperviscosity or thrombosis. Also headache, dizziness, tinnitus, blurred vision, early satiety, fatigue, epistaxis, easy bruising, or abdominal discomfort due to splenomegaly.3 Pruritis is a common symptom, especially after a hot bath or shower. This is due to the increase in WBC’s, specifically the basophils releasing histamine.4 Physical exam findings include splenomegaly, facial flushing/ruddy complexion, engorged retinal veins or injection of the conjunctiva, excoriation of the skin, prior thrombotic event (ie: stroke, DVT/PE, myocardial infarction, mesentaric or hepatic venous thrombosis, superficial thrombophlebitis).1
    PCV should be suspected in any patient with an elevated hemoglobin/hematocrit or red blood cell mass and a normal O2 saturation. A hemoglobin >18.5 g/dL (or hematocrit >56%) in men or a hemoglobin >16.5 g/dL (or hematocrit >50%) in women is considered an elevated red cell mass. There is also a mild increase in the WBC’s and platelets.4 The white blood cell count is usually elevated to 10,000-20,000/mcL and the platelet count is variably increased.1 Upon obtaining elevated blood counts, it is important to repeat a CBC to see if the hemoglobin and hematocrit return to normal.3 If the level remain elevated, a serum erytropoietin level can be obtained. This level should be low because the bone marrow is overproducing RBC’s in the absence of erythropoietin. Low erythropoietin levels is highly specific for PCV. A splenic ultrasound can also be done to show splenomegaly.4 Peripheral blood should be tested for the JAK2 mutation, which is on chromosome 9p24. A bone marrow biopsy would show hypercellularity in all three cell lines.3 The diagnostic criteria for PCV is as followed: Major criteria includes increase in RBC mass demonstrated by an increase in hemoglobin and hematocrit, bone marrow biopsy shows hypercellularity, and JAK2 mutation presence. Minor criteria includes a decrease in serum erythropoietin levels. A proper diagnosis is required to have all three major or the first 2 major and the minor.2 It is also important to rule out other causes of erythrocytosis, such as hypoxia, familial polycythemic disorder, damaged EPO receptors, and tumor producing EPO inappropriately.3
    There is no cure for PCV at this time. However, it is important to reduce the risk of thrombosis, decrease the symptoms, and lower the transition to AML/MDS.1 Therapeutic phlebotomy is the management of choice. It is done until a hematocrit of less than 45 is obtained.4 A standard 1 unit phlebotomy (500 mL) should decrease the hematocrit by 3%. Men can lose 1.5 – 2 units per week and women can lose 0.5 units per week. Phlebotomies are repeated as often as it is necessary to maintain a normal hematocrit level. Patients are at risk of venous thrombosis. Low-dose aspirin should be given to all patients, unless contraindicated, to help prevent thrombosis. High dose aspirin should be avoided due to increase in GI bleed. If thrombosis has occurred, than anticoagulation should be started. Anagrelide can be helpful in thrombocythemia, to reduce the amount of platelets. However, it is not first line treatment for PCV because it causes an increased risk of arterial thrombosis and bleeding. Myelosuppression therapy is also part of treatment for high-risk PCV patient or those who cannot tolerate phlebotomies (ie: poor venous access). High-risk patients are >60 years old or have history of thrombosis. Most commonly used is hydroxyurea.5 It inhibits cells with a high division rate.2 It works by inhibiting ribonucleotide reductase that interferes with DNA repair. Side effects from hydroxyurea include cytopenias, mucositis, diarrhea, peripheral neuropathy, and skin cancer. Also can use interferon-alpha, an immunotherapy, as an alternative. Interferon-alpha may be considered better for younger patients and those who want to get pregnant. For patients who are still symptomatic (ie: extreme pruritus and symptomatic splenomegaly) despite phlebotomy, aspirin, and myelosuppression therapy, ruxolitinib may be offered. Ruxolitinib is a JAK inhibitor, therefore it can only be used with patient who have the JAK2 mutation. It causes a decrease in platelets and help reduce the size of the spleen. It may cause severe anemia. For the pruritis, patients can try antihistamines, moisturizing cream, lowering water temperature while bathing, and patting skin dry rather than rubbing. It is also important to advise patients the importance of weight loss, diet, blood pressure control, and smoking cessation.5 Prognosis is good for patients with PCV. Median survival is 15 years. The major cause of death is venous and arterial thrombosis.4 PCV may transition to myelofibrosis, AML, or myelodysplastic syndrome which can also limit life expectancy.5

    References
    Hammond DB. Myeloproliferative Disorder, Bone Marrow Failure & the Myelodysplastic Syndrome. MCPHS University: Clinical Medicine III. October 2016.
    Williams DA. PANCE Prep Pearls: a medical study and review guide for the PANCE, PANRE & medical examinations. 2nd ed. North Charleston, SC: CreateSpace Independent Publishing Platform; 2017.
    Tefferi A. Clinical manifestations and diagnosis of polycythemia vera. UpToDate. https://www.uptodate.com/contents/clinical-manifestations-and-diagnosis-of-polycythemia-vera?source=search_result&search=polycythemia vera&selectedTitle=1~104. Accessed September 27, 2017.
    Papadakis MA, McPhee SJ, Rabow MW. 2015 Current Medical Diagnosis & Treatment. New York: McGraw-Hill Education/Medical; 2015.
    Tefferi A. Clinical manifestations and diagnosis of polycythemia vera. UpToDate. https://www.uptodate.com/contents/clinical-manifestations-and-diagnosis-of-polycythemia-vera?source=search_result&search=polycythemia vera&selectedTitle=1~104. Accessed September 27, 2017.
    Tefferi A. Prognosis and treatment of polycythemia vera. UpToDate. https://www.uptodate.com/contents/prognosis-and-treatment-of-polycythemia-vera?source=search_result&search=polycythemia vera&selectedTitle=2~104. Accessed September 27, 2017.

    Incorrect

    Answer is C. Polycythemia vera is a myeloproliferative disorder with an increase in Hgb, Hct, WBC’s and plateletes. It is most commonly seen in men ages 50-60 yrs old. Signs/symptoms are due to increase in RBC mass and can also include headaches, dizziness, blurred vision, pruritis after hot bath, fatigue, early saiety, flushed face and splenomegaly.
    Essential thrombocythemia is also a myeloproliferative disorder that can have an increase in WBC and marked elevation in platelets (>600,000), but no increase in Hct. Symptoms include headache and dizziness.
    Chronic myeloid leukemia is another myeloproliferative disorder with a large increase in WBC (>30,000/mcL). Patients may also have an increase in platelets, but a normal Hct. Symptoms include fatigue, SOB, weight loss, or abdominal discomfort from splenomegaly.
    Secondary polycythemia is caused by hypoxia (ie: smoking). It results in an increase in Hct with normal WBC’s and platelets. Symptoms are usually related to the underlying cause. Physical exam may show cyanosis and clubbing. Splenomegaly would be absent.1
    Reference: Hammond DB. Myeloproliferative Disorder, Bone Marrow Failure & the Myelodysplastic Syndrome. MCPHS University: Clinical Medicine III. October 2016.

    DISCUSSION
    Polycythemia vera (PCV) is an acquired chronic myeloproliferative disorder.2 Myeloproliferative disorders are caused by clonal defects of hematopoietic stem cells. These stem cells produce myeloid, erythroid, and platelet cells. Polycythemia vera causes an overproduction of all three of these cell lines, with erythrocytes, or red blood cells (RBC), being the most prominent.3 Red blood cells are produced from bone marrow when erythropoietin is secreted by the kidneys, in response to cellular hypoxia. The exact etiology of PCV is unknown, however most believe it is caused by a mutation in exon 14 of Janus Kinase (JAK2 V617F), which is a signling molecule found in 95% of PCV cases.4 JAK2 may play a role in causing an increase in the sensitivity of erythroid cells to erythropoietin.3 PCV occurs in all ages. It is commonly seen in 50-60 year old patients, with the average age of diagnosis 60 years. Sixty percent of patients are men. It occurs in all populations. The incidence is lower in Japan when compared to the United States and Europe. The incidence is 2/100,000 – 18/100,000 in adults and increases with age.1
    PCV is also known as primary erythrocytosis, which means an increase in hematocrit or red blood cell mass in the absence of hypoxia. Patients should have a normal oxygen saturation on exam.3 This is different from secondary erythrocytosis, which is an increase in hematocrit from a response to another process (ie: hypoxia from smoking).2 Majority of patients with PCV are discovered incidentally. Symptoms occur from the increase in red blood cell mass. This includes hyperviscosity or thrombosis. Also headache, dizziness, tinnitus, blurred vision, early satiety, fatigue, epistaxis, easy bruising, or abdominal discomfort due to splenomegaly.3 Pruritis is a common symptom, especially after a hot bath or shower. This is due to the increase in WBC’s, specifically the basophils releasing histamine.4 Physical exam findings include splenomegaly, facial flushing/ruddy complexion, engorged retinal veins or injection of the conjunctiva, excoriation of the skin, prior thrombotic event (ie: stroke, DVT/PE, myocardial infarction, mesentaric or hepatic venous thrombosis, superficial thrombophlebitis).1
    PCV should be suspected in any patient with an elevated hemoglobin/hematocrit or red blood cell mass and a normal O2 saturation. A hemoglobin >18.5 g/dL (or hematocrit >56%) in men or a hemoglobin >16.5 g/dL (or hematocrit >50%) in women is considered an elevated red cell mass. There is also a mild increase in the WBC’s and platelets.4 The white blood cell count is usually elevated to 10,000-20,000/mcL and the platelet count is variably increased.1 Upon obtaining elevated blood counts, it is important to repeat a CBC to see if the hemoglobin and hematocrit return to normal.3 If the level remain elevated, a serum erytropoietin level can be obtained. This level should be low because the bone marrow is overproducing RBC’s in the absence of erythropoietin. Low erythropoietin levels is highly specific for PCV. A splenic ultrasound can also be done to show splenomegaly.4 Peripheral blood should be tested for the JAK2 mutation, which is on chromosome 9p24. A bone marrow biopsy would show hypercellularity in all three cell lines.3 The diagnostic criteria for PCV is as followed: Major criteria includes increase in RBC mass demonstrated by an increase in hemoglobin and hematocrit, bone marrow biopsy shows hypercellularity, and JAK2 mutation presence. Minor criteria includes a decrease in serum erythropoietin levels. A proper diagnosis is required to have all three major or the first 2 major and the minor.2 It is also important to rule out other causes of erythrocytosis, such as hypoxia, familial polycythemic disorder, damaged EPO receptors, and tumor producing EPO inappropriately.3
    There is no cure for PCV at this time. However, it is important to reduce the risk of thrombosis, decrease the symptoms, and lower the transition to AML/MDS.1 Therapeutic phlebotomy is the management of choice. It is done until a hematocrit of less than 45 is obtained.4 A standard 1 unit phlebotomy (500 mL) should decrease the hematocrit by 3%. Men can lose 1.5 – 2 units per week and women can lose 0.5 units per week. Phlebotomies are repeated as often as it is necessary to maintain a normal hematocrit level. Patients are at risk of venous thrombosis. Low-dose aspirin should be given to all patients, unless contraindicated, to help prevent thrombosis. High dose aspirin should be avoided due to increase in GI bleed. If thrombosis has occurred, than anticoagulation should be started. Anagrelide can be helpful in thrombocythemia, to reduce the amount of platelets. However, it is not first line treatment for PCV because it causes an increased risk of arterial thrombosis and bleeding. Myelosuppression therapy is also part of treatment for high-risk PCV patient or those who cannot tolerate phlebotomies (ie: poor venous access). High-risk patients are >60 years old or have history of thrombosis. Most commonly used is hydroxyurea.5 It inhibits cells with a high division rate.2 It works by inhibiting ribonucleotide reductase that interferes with DNA repair. Side effects from hydroxyurea include cytopenias, mucositis, diarrhea, peripheral neuropathy, and skin cancer. Also can use interferon-alpha, an immunotherapy, as an alternative. Interferon-alpha may be considered better for younger patients and those who want to get pregnant. For patients who are still symptomatic (ie: extreme pruritus and symptomatic splenomegaly) despite phlebotomy, aspirin, and myelosuppression therapy, ruxolitinib may be offered. Ruxolitinib is a JAK inhibitor, therefore it can only be used with patient who have the JAK2 mutation. It causes a decrease in platelets and help reduce the size of the spleen. It may cause severe anemia. For the pruritis, patients can try antihistamines, moisturizing cream, lowering water temperature while bathing, and patting skin dry rather than rubbing. It is also important to advise patients the importance of weight loss, diet, blood pressure control, and smoking cessation.5 Prognosis is good for patients with PCV. Median survival is 15 years. The major cause of death is venous and arterial thrombosis.4 PCV may transition to myelofibrosis, AML, or myelodysplastic syndrome which can also limit life expectancy.5

    References
    Hammond DB. Myeloproliferative Disorder, Bone Marrow Failure & the Myelodysplastic Syndrome. MCPHS University: Clinical Medicine III. October 2016.
    Williams DA. PANCE Prep Pearls: a medical study and review guide for the PANCE, PANRE & medical examinations. 2nd ed. North Charleston, SC: CreateSpace Independent Publishing Platform; 2017.
    Tefferi A. Clinical manifestations and diagnosis of polycythemia vera. UpToDate. https://www.uptodate.com/contents/clinical-manifestations-and-diagnosis-of-polycythemia-vera?source=search_result&search=polycythemia vera&selectedTitle=1~104. Accessed September 27, 2017.
    Papadakis MA, McPhee SJ, Rabow MW. 2015 Current Medical Diagnosis & Treatment. New York: McGraw-Hill Education/Medical; 2015.
    Tefferi A. Clinical manifestations and diagnosis of polycythemia vera. UpToDate. https://www.uptodate.com/contents/clinical-manifestations-and-diagnosis-of-polycythemia-vera?source=search_result&search=polycythemia vera&selectedTitle=1~104. Accessed September 27, 2017.
    Tefferi A. Prognosis and treatment of polycythemia vera. UpToDate. https://www.uptodate.com/contents/prognosis-and-treatment-of-polycythemia-vera?source=search_result&search=polycythemia vera&selectedTitle=2~104. Accessed September 27, 2017.

  193. Question 193 of 220
    193. Question

    Which nerve is most commonly injured during an anterior shoulder dislocation?

    Correct

    Answer D.
    The most common nerve injured during an anterior shoulder dislocation is the axillary nerve. The axillary nerve branches off the posterior cord of the brachial plexus and C5 and C6 nerve roots. The nerve then courses anterior to the subscapularis and exits out axilla posteriorly through the quadrangular space. The nerve then divides into the anterior, posterior, and articular branches. The anterior branch wraps around the surgical neck of the humerus beneath the deltoid muscle. The posterior branch innervates the teres minor and posterior deltoid muscle. The smaller articular branch enters the shoulder behind the subscaupularis muscle and innervates the shoulder joint itself. 1, 2
    The mechanism of injury during a low velocity shoulder dislocation is traction on the axillary nerve wrapped around the humerus as the nerve is pulled forward beyond physiological limits. Symptomatic axillary nerve injury occurs in around 10% of patients after an anterior shoulder dislocation. However, electromyography studies have reported much higher rates of nerve injury with some studies approaching 55%. Symptoms of an axillary nerve injury include numbness and a dull achy pain over the lateral and posterior deltoid. A nerve injury should be suspected with any numbness to the lateral arm after a shoulder dislocation. In patients over 40 years old who sustain a dislocation, it is important to know that weak arm abduction is more likely related to a torn rotator cuff than a nerve injury. Axillary nerve injuries have a very good prognosis with full recovery expected between 12 and 45 weeks. Electromyography studies are generally not warranted as the prognosis is straight forward. Physical therapy should be started early after the injury to help regain motion and strength of the shoulder. 1
    References
    1. Visser CPJ, Coene LNJEM, Brand R, Tavy. The Incidence of Nerve Injury in Anterior Dislocation of the Shoulder and its Influence of Functional Recovery. JBJS (Br). Vol 81-B, No. 4. July 1999. 679-685.
    2. Cetik O, Uslu M, Acar HI, Comert A, Tekdemir I, Cift H. Is there a safe area for the axillary nerve in the deltoid muscle? A cadaveric study. JBJS. 2006. 88-A (11). 2395-2399.

    Incorrect

    Answer D.
    The most common nerve injured during an anterior shoulder dislocation is the axillary nerve. The axillary nerve branches off the posterior cord of the brachial plexus and C5 and C6 nerve roots. The nerve then courses anterior to the subscapularis and exits out axilla posteriorly through the quadrangular space. The nerve then divides into the anterior, posterior, and articular branches. The anterior branch wraps around the surgical neck of the humerus beneath the deltoid muscle. The posterior branch innervates the teres minor and posterior deltoid muscle. The smaller articular branch enters the shoulder behind the subscaupularis muscle and innervates the shoulder joint itself. 1, 2
    The mechanism of injury during a low velocity shoulder dislocation is traction on the axillary nerve wrapped around the humerus as the nerve is pulled forward beyond physiological limits. Symptomatic axillary nerve injury occurs in around 10% of patients after an anterior shoulder dislocation. However, electromyography studies have reported much higher rates of nerve injury with some studies approaching 55%. Symptoms of an axillary nerve injury include numbness and a dull achy pain over the lateral and posterior deltoid. A nerve injury should be suspected with any numbness to the lateral arm after a shoulder dislocation. In patients over 40 years old who sustain a dislocation, it is important to know that weak arm abduction is more likely related to a torn rotator cuff than a nerve injury. Axillary nerve injuries have a very good prognosis with full recovery expected between 12 and 45 weeks. Electromyography studies are generally not warranted as the prognosis is straight forward. Physical therapy should be started early after the injury to help regain motion and strength of the shoulder. 1
    References
    1. Visser CPJ, Coene LNJEM, Brand R, Tavy. The Incidence of Nerve Injury in Anterior Dislocation of the Shoulder and its Influence of Functional Recovery. JBJS (Br). Vol 81-B, No. 4. July 1999. 679-685.
    2. Cetik O, Uslu M, Acar HI, Comert A, Tekdemir I, Cift H. Is there a safe area for the axillary nerve in the deltoid muscle? A cadaveric study. JBJS. 2006. 88-A (11). 2395-2399.

  194. Question 194 of 220
    194. Question

    A 67 year-old male presents to the office with end-stage arthritis of the left hip. He has failed conservative treatment and wishes to have a total hip replacement. He had a right hip replacement 5 years ago using the posterior approach. Although he recently heard about the anterior approach and asks if this is the best approach for him now. Which choice below is true regarding direct anterior approach for total hip replacement as compared to the posterior approach?

    Correct

    Answer A.
    The anterior approach is an increasingly popular approach to total hip replacement. Recent advances in operating room tables and implant designs have made the anterior approach more surgeon friendly. The anterior approach is unique to the posterior approach in that the anterior approach advances through a true internervous and intermuscular plane. The posterior approach requires cutting through muscles to get to the joint capsule which can increase postoperative pain and slow recovery time as compared to the anterior approach. Patients who undergo the anterior approach tend to have shorter hospitals stays, less postoperative narcotic use, less postoperative hip precautions, and a quicker return to walking independently without a walker or crutches. Several studies have shown a lower postoperative dislocation rate with the anterior approach, although other studies have refuted this data with equal rates for both anterior and posterior.
    One disadvantage of the anterior approach is the steep learning curve it takes for orthopedic surgeons to master the technique. This learning curve often leads to increased operating room times and operative blood loss. It generally takes 50-100 operative cases before orthopedic surgeons are proficient with the anterior approach. The anterior approach is often avoided in obese patients as a large pannus can fold over the anterior incision and increase infection potential.

    References
    1. Zawadsky MW, Paulus MC, Murray PJ, Johansen MA. Early Outcome Comparison Between the Direct Anterior Approach and the Mini-Incision Posterior Approach for Primary Total Hip Arthroplasty: 150 Consecutive Cases. June 2014Volume 29, Issue 6, Pages 1256–1260.
    2. Higgins BT, Barlow DR, Heagerty NE, Lin TJ. Anterior vs. Posterior Approach for Total Hip Arthroplasty, a Systematic Review and Meta-analysis. The Journal of Arthroplasty. Volume 30, Issue 3, March 2015, Pages 419–434

    Incorrect

    Answer A.
    The anterior approach is an increasingly popular approach to total hip replacement. Recent advances in operating room tables and implant designs have made the anterior approach more surgeon friendly. The anterior approach is unique to the posterior approach in that the anterior approach advances through a true internervous and intermuscular plane. The posterior approach requires cutting through muscles to get to the joint capsule which can increase postoperative pain and slow recovery time as compared to the anterior approach. Patients who undergo the anterior approach tend to have shorter hospitals stays, less postoperative narcotic use, less postoperative hip precautions, and a quicker return to walking independently without a walker or crutches. Several studies have shown a lower postoperative dislocation rate with the anterior approach, although other studies have refuted this data with equal rates for both anterior and posterior.
    One disadvantage of the anterior approach is the steep learning curve it takes for orthopedic surgeons to master the technique. This learning curve often leads to increased operating room times and operative blood loss. It generally takes 50-100 operative cases before orthopedic surgeons are proficient with the anterior approach. The anterior approach is often avoided in obese patients as a large pannus can fold over the anterior incision and increase infection potential.

    References
    1. Zawadsky MW, Paulus MC, Murray PJ, Johansen MA. Early Outcome Comparison Between the Direct Anterior Approach and the Mini-Incision Posterior Approach for Primary Total Hip Arthroplasty: 150 Consecutive Cases. June 2014Volume 29, Issue 6, Pages 1256–1260.
    2. Higgins BT, Barlow DR, Heagerty NE, Lin TJ. Anterior vs. Posterior Approach for Total Hip Arthroplasty, a Systematic Review and Meta-analysis. The Journal of Arthroplasty. Volume 30, Issue 3, March 2015, Pages 419–434

  195. Question 195 of 220
    195. Question

    A 70-year-old caucasian male with a PMH of hypertension and emphysema arrives at the emergency room with worsening left sided chest pain and dyspnea over the past 2 hours. EKG shows sinus tachycardia. A CXR is obtained and patient is found to have a right-sided tension pneumothorax. What radiological finding would NOT be consistent with this diagnosis?

    Correct

    X-ray findings for a tension pneumothorax would show increased radiolucency on the side of the pneumothorax where the air is collecting in the pleural space, tracheal and mediastinal deviation to the contralateral side of the pneumothorax due to the compression caused by the air leak. Because of this, answer A is correct, as it is not a radiologic finding consistent with a tension pneumothorax. Choices B, C, and D are all radiologic findings in a tension pneumothorax.

    DISCUSSION:
    The patient in the above case was diagnosed with a spontaneous tension pneumothorax. A pneumothorax occurs when there is a presence of air in the pleural cavity and can occur for a variety of different reasons and can have different levels of severity. A tension pneumothorax is “a life-threatening condition that develops when air is trapped in the pleural cavity under positive pressure, displacing mediastinal structures and compromising cardiopulmonary function” (Daley, 2016). Patients are at the highest risk for developing a pneumothorax during the neonatal period. The incidence in neonates is 1-2% (Daley, 2016). Neonates who are male, preterm, delivered by c-section, low birth weight, low 1 minute Apgar scores, require resuscitation at birth, have RDS, PIE and receive ventilator treatment are at high risk for developing pneumothoraxes as represented by the patient in the above case. Other risk factors include receiving positive pressure ventilation or CPAP, vacuum extraction, pneumonia, pulmonary hypoplasia, CPR, perinatal asphyxia and urinary tract anomalies. The incidence of pneumothorax development has increased since the start of ventilator use in neonates. In those receiving positive pressure ventilation, the risk is 15-30% (Gomella, 2013). The incidence of tension pneumothorax is unknown.
    Pneumothorax can either be spontaneous or due to trauma. There are two types of spontaneous pneumothorax (primary and secondary). Primary spontaneous pneumothoraxes are without known pulmonary disease and usually occur in tall, thin younger males. They are more at risk if they are smokers, or have a positive family history however; this cause is rare during the neonatal period. A secondary spontaneous pneumothorax occurs in the presence of pulmonary disease, such as Pulmonary Interstitial Emphysema or RDS as seen in the above case. PIE is seen almost exclusively in very low birth weight infants on ventilatory support like Twin Boy B and usually develops in the first 48-72 hours of life (Gomella, 2013). In adults, diseases such as COPD put you at increased risk for a pneumothorax. A pneumothorax can also be iatrogenic from ventilation use or traumatic.
    In a pneumothorax, air gets displaced into the pleural space due to the pressure gradient of either the alveoli and pleural space or the pleural space and the atmospheric pressure (traumatic). Alveoli are under a greater pressure than the intrapleural space, with the pressure in the intrapleural space being less than atmospheric pressure. Normally, a pneumothorax will “seal itself off” when the pressure between the two is equalized. Tension pneumothorax can result from any of these when the intrapleural pressure exceeds the atmospheric or alveolar pressure (Epocrates, 2017). This acts as a one-way valve and does not allow for air to escape (back into the lung or through a trauma induced puncture) with exhalation (Gamella, 2013). The pressure in your thoracic cavity increases causing collapse of the lung tissue/volume. If it is a tension pneumothorax, the pressure continues to collapse the lung and causes a shift of the trachea and mediastinum to the contralateral side. This further causes an increase in pulmonary vascular resistance and central venous pressure. With the heart compressed, cardiac output and the venous return to the heart can become decreased. The result is hypoxia and acute respiratory failure (Gamella, 2013).
    As for history and physical findings, it becomes more challenging to catch with neonatal patients. Generally, a tension pneumothorax presents itself as a medical emergency with sudden and rapid deterioration of the patient. Especially in neonates, this is not always that apparent and can present more subtly as with the patient’s presentation in the above case. Signs to look for in the infant include grunting, irritability, and/or restlessness. Most commonly, a patient will present with worsening chest pain, dyspnea, anxiety or fatigue and occasionally epigastric pain. However, neonates cannot provide you with this information so you have to rely on physical exam. Physical exam findings include cyanosis, hypoxia, tachypnea, increased work of breathing, retractions, diminished or absent breath sounds, and an asymmetric chest rise. If mechanically ventilated, you may see an increase in oxygen demand. The cardiovascular exam can show tachycardia or bradycardia, an increase in blood pressure with narrowed pulse pressure and eventually hypotension (Gomella, 2013). The PMI will be displaced to the contralateral side. Infants may present with a “cyanotic upper half and pale lower half” of their body (Gomella, 2013). Not as commonly in patients you may see JVD and tracheal shift and have hyper resonance on percussion (Epocrates 2016). Pneumothoraxes are found incidentally in 1-2% of neonates without any suggesting signs or symptoms (Gomella 2013). When looking at labs, a blood gas may show respiratory acidosis as seen with the above patient giving a decreased PaO2 and and increased PCO2.
    Imaging that is useful in the diagnosis of a pneumothorax include ultrasound, xray and on rare occasions CT if patient is hemodynamically stable with a high index of suspision but a pneumothorax cannot be ruled out by CXR/US. In neonates, transillumination of the thorax may also be used. Ultrasound will show an absence of lung sliding/absence of the normal “sea shore” sign. In a tension pneumothorax a CXR will show a shift in the mediastinum and trachea to the contralateral side of the air leak, depression of the diaphragm on the ipsilateral side, a radiolucent band of air with a lack of lung markings on the ipsilateral side of the air leak and decreased radiolucency on the contralateral side of the air leak where the lungs are being collapsed (Gorrochategui, 2017).
    Treatment differs for a tension vs. nontension pneumothorax. If suspecting a tension pneumothorax, do not delay for confirmation on CXR. If the air is not evacuated immediately it can be fatal. Treatment includes a needle decompression followed by chest tube placement on the side of the air leak with a confirmatory CXR after decompression/tube placement. Chest tube placement is not always necessary for pneumothorax following needle decompression, especially if the patient is not using any form of mechanical ventialation. The needle can be placed at either the second intercostal space – midclavicular line or the fourth intercostal space – anterior axillary line with needle insertion on top of the rib. Removal of the chest tube requires pain medication and can be done once there is no longer bubbling of the underwater seal or presence of air on xray for 24-48 hours. If hemodynamicaly stable and a non-tension pneumothorax, conservative measures such as oxygen supplementation may be all that is necessary. If patient is currently on or requiring mechanical ventilation, High Frequency Oscillatory Ventilation is preferred as it has lower mean airway pressures and is a gentler mode of ventilation for these patients (Gomella, 2013). If patient is hyperinflated or has atelectasis after reinflation, bronchodilators can be used as well as proper positioning of the infant (hyperinflated side down/atelectatic side up).
    With the above case the differential diagnosis included pneumothorax, displaced ET tube, pneumonia, pneumomediastinum, pleural effusion, pericardial effusion, pneumopericardium, anemia. Although pneumothorax was high on suspicion, due to baby’s risk factors of RDS, PIE and mechanical ventilation, the diagnosis of tension pneumothorax was quite surprising. The infant had subtle signs of respiratory distress but was not at all hemodynamically unstable. Bacteremia/sepsis would have been considered however this could be ruled out with blood cultures showing no growth for 48 hours. Obstruction of the infant’s nares by a mucus plug was ruled out after no relief after suction by the nurse. The displaced ET tube was the most likely and most easily correctable diagnosis second to a pneumothorax but this was ruled out with the XRAY along with the pleural effusion, pericardial effusion and pneumopericardium. Anemia could have contributed to some of the symptoms but not all and could not be the cause of the pneumothorax; this was eventually corrected with a blood transfusion. Pneumomediastinum would have resulted in an elevated appearance of the thymus with air outlining the heart. Pneumopericardium and tension pneumothoraxes can both present with rapid deterioration of the patient although the patient did not present in this way. Pneumothorax is the more common of these diagnoses and the diagnosis can be differentiated on CXR. Pneumonia was a likely diagnosis before the XRAY as the patient had multiple risk factors and had a subtle presentation of symptoms. Infants with pneumonia may also have decreased breath sounds, rales, and tachypnea but may also present with increased respiratory secretions, fever/hypothermia and an altered white blood cell count.

    References
    Daley BJ. Pneumothorax. http://emedicine.medscape.com/article/424547-overview. Published August 18, 2017. Accessed September 24, 2017.
    Gomella TL, Cunningham MD, Eyal FG. Neonatology: management, procedures, on-Call problems, diseases, and drugs. 7th ed. New York, NY: McGraw-Hill Education Medical; 2013.
    Pneumothorax History & Exam – Epocrates Online. https://online.epocrates.com/diseases/50433/Pneumothorax/History-Exam. Accessed September 24, 2017.
    Gorrochategui M. Pneumothorax | Radiology Reference Article. Radiopaedia.org. https://radiopaedia.org/articles/pneumothorax. Accessed September 24, 2017.

    Incorrect

    X-ray findings for a tension pneumothorax would show increased radiolucency on the side of the pneumothorax where the air is collecting in the pleural space, tracheal and mediastinal deviation to the contralateral side of the pneumothorax due to the compression caused by the air leak. Because of this, answer A is correct, as it is not a radiologic finding consistent with a tension pneumothorax. Choices B, C, and D are all radiologic findings in a tension pneumothorax.

    DISCUSSION:
    The patient in the above case was diagnosed with a spontaneous tension pneumothorax. A pneumothorax occurs when there is a presence of air in the pleural cavity and can occur for a variety of different reasons and can have different levels of severity. A tension pneumothorax is “a life-threatening condition that develops when air is trapped in the pleural cavity under positive pressure, displacing mediastinal structures and compromising cardiopulmonary function” (Daley, 2016). Patients are at the highest risk for developing a pneumothorax during the neonatal period. The incidence in neonates is 1-2% (Daley, 2016). Neonates who are male, preterm, delivered by c-section, low birth weight, low 1 minute Apgar scores, require resuscitation at birth, have RDS, PIE and receive ventilator treatment are at high risk for developing pneumothoraxes as represented by the patient in the above case. Other risk factors include receiving positive pressure ventilation or CPAP, vacuum extraction, pneumonia, pulmonary hypoplasia, CPR, perinatal asphyxia and urinary tract anomalies. The incidence of pneumothorax development has increased since the start of ventilator use in neonates. In those receiving positive pressure ventilation, the risk is 15-30% (Gomella, 2013). The incidence of tension pneumothorax is unknown.
    Pneumothorax can either be spontaneous or due to trauma. There are two types of spontaneous pneumothorax (primary and secondary). Primary spontaneous pneumothoraxes are without known pulmonary disease and usually occur in tall, thin younger males. They are more at risk if they are smokers, or have a positive family history however; this cause is rare during the neonatal period. A secondary spontaneous pneumothorax occurs in the presence of pulmonary disease, such as Pulmonary Interstitial Emphysema or RDS as seen in the above case. PIE is seen almost exclusively in very low birth weight infants on ventilatory support like Twin Boy B and usually develops in the first 48-72 hours of life (Gomella, 2013). In adults, diseases such as COPD put you at increased risk for a pneumothorax. A pneumothorax can also be iatrogenic from ventilation use or traumatic.
    In a pneumothorax, air gets displaced into the pleural space due to the pressure gradient of either the alveoli and pleural space or the pleural space and the atmospheric pressure (traumatic). Alveoli are under a greater pressure than the intrapleural space, with the pressure in the intrapleural space being less than atmospheric pressure. Normally, a pneumothorax will “seal itself off” when the pressure between the two is equalized. Tension pneumothorax can result from any of these when the intrapleural pressure exceeds the atmospheric or alveolar pressure (Epocrates, 2017). This acts as a one-way valve and does not allow for air to escape (back into the lung or through a trauma induced puncture) with exhalation (Gamella, 2013). The pressure in your thoracic cavity increases causing collapse of the lung tissue/volume. If it is a tension pneumothorax, the pressure continues to collapse the lung and causes a shift of the trachea and mediastinum to the contralateral side. This further causes an increase in pulmonary vascular resistance and central venous pressure. With the heart compressed, cardiac output and the venous return to the heart can become decreased. The result is hypoxia and acute respiratory failure (Gamella, 2013).
    As for history and physical findings, it becomes more challenging to catch with neonatal patients. Generally, a tension pneumothorax presents itself as a medical emergency with sudden and rapid deterioration of the patient. Especially in neonates, this is not always that apparent and can present more subtly as with the patient’s presentation in the above case. Signs to look for in the infant include grunting, irritability, and/or restlessness. Most commonly, a patient will present with worsening chest pain, dyspnea, anxiety or fatigue and occasionally epigastric pain. However, neonates cannot provide you with this information so you have to rely on physical exam. Physical exam findings include cyanosis, hypoxia, tachypnea, increased work of breathing, retractions, diminished or absent breath sounds, and an asymmetric chest rise. If mechanically ventilated, you may see an increase in oxygen demand. The cardiovascular exam can show tachycardia or bradycardia, an increase in blood pressure with narrowed pulse pressure and eventually hypotension (Gomella, 2013). The PMI will be displaced to the contralateral side. Infants may present with a “cyanotic upper half and pale lower half” of their body (Gomella, 2013). Not as commonly in patients you may see JVD and tracheal shift and have hyper resonance on percussion (Epocrates 2016). Pneumothoraxes are found incidentally in 1-2% of neonates without any suggesting signs or symptoms (Gomella 2013). When looking at labs, a blood gas may show respiratory acidosis as seen with the above patient giving a decreased PaO2 and and increased PCO2.
    Imaging that is useful in the diagnosis of a pneumothorax include ultrasound, xray and on rare occasions CT if patient is hemodynamically stable with a high index of suspision but a pneumothorax cannot be ruled out by CXR/US. In neonates, transillumination of the thorax may also be used. Ultrasound will show an absence of lung sliding/absence of the normal “sea shore” sign. In a tension pneumothorax a CXR will show a shift in the mediastinum and trachea to the contralateral side of the air leak, depression of the diaphragm on the ipsilateral side, a radiolucent band of air with a lack of lung markings on the ipsilateral side of the air leak and decreased radiolucency on the contralateral side of the air leak where the lungs are being collapsed (Gorrochategui, 2017).
    Treatment differs for a tension vs. nontension pneumothorax. If suspecting a tension pneumothorax, do not delay for confirmation on CXR. If the air is not evacuated immediately it can be fatal. Treatment includes a needle decompression followed by chest tube placement on the side of the air leak with a confirmatory CXR after decompression/tube placement. Chest tube placement is not always necessary for pneumothorax following needle decompression, especially if the patient is not using any form of mechanical ventialation. The needle can be placed at either the second intercostal space – midclavicular line or the fourth intercostal space – anterior axillary line with needle insertion on top of the rib. Removal of the chest tube requires pain medication and can be done once there is no longer bubbling of the underwater seal or presence of air on xray for 24-48 hours. If hemodynamicaly stable and a non-tension pneumothorax, conservative measures such as oxygen supplementation may be all that is necessary. If patient is currently on or requiring mechanical ventilation, High Frequency Oscillatory Ventilation is preferred as it has lower mean airway pressures and is a gentler mode of ventilation for these patients (Gomella, 2013). If patient is hyperinflated or has atelectasis after reinflation, bronchodilators can be used as well as proper positioning of the infant (hyperinflated side down/atelectatic side up).
    With the above case the differential diagnosis included pneumothorax, displaced ET tube, pneumonia, pneumomediastinum, pleural effusion, pericardial effusion, pneumopericardium, anemia. Although pneumothorax was high on suspicion, due to baby’s risk factors of RDS, PIE and mechanical ventilation, the diagnosis of tension pneumothorax was quite surprising. The infant had subtle signs of respiratory distress but was not at all hemodynamically unstable. Bacteremia/sepsis would have been considered however this could be ruled out with blood cultures showing no growth for 48 hours. Obstruction of the infant’s nares by a mucus plug was ruled out after no relief after suction by the nurse. The displaced ET tube was the most likely and most easily correctable diagnosis second to a pneumothorax but this was ruled out with the XRAY along with the pleural effusion, pericardial effusion and pneumopericardium. Anemia could have contributed to some of the symptoms but not all and could not be the cause of the pneumothorax; this was eventually corrected with a blood transfusion. Pneumomediastinum would have resulted in an elevated appearance of the thymus with air outlining the heart. Pneumopericardium and tension pneumothoraxes can both present with rapid deterioration of the patient although the patient did not present in this way. Pneumothorax is the more common of these diagnoses and the diagnosis can be differentiated on CXR. Pneumonia was a likely diagnosis before the XRAY as the patient had multiple risk factors and had a subtle presentation of symptoms. Infants with pneumonia may also have decreased breath sounds, rales, and tachypnea but may also present with increased respiratory secretions, fever/hypothermia and an altered white blood cell count.

    References
    Daley BJ. Pneumothorax. http://emedicine.medscape.com/article/424547-overview. Published August 18, 2017. Accessed September 24, 2017.
    Gomella TL, Cunningham MD, Eyal FG. Neonatology: management, procedures, on-Call problems, diseases, and drugs. 7th ed. New York, NY: McGraw-Hill Education Medical; 2013.
    Pneumothorax History & Exam – Epocrates Online. https://online.epocrates.com/diseases/50433/Pneumothorax/History-Exam. Accessed September 24, 2017.
    Gorrochategui M. Pneumothorax | Radiology Reference Article. Radiopaedia.org. https://radiopaedia.org/articles/pneumothorax. Accessed September 24, 2017.

  196. Question 196 of 220
    196. Question

    A 32-year-old male comes into the office for hair loss at the back of his head. The patient has noticed that the hair loss began after his divorce to his wife was finalized a month ago. He noticed that the hair loss began as a small patch and has progressed to two large circular areas, each around the size of a quarter. He denies any flaking, redness, or itching of the area. Upon examination, the lesions are smooth with hairs easily pulled from the periphery of the lesion. Which finding is most consistent with the likely diagnosis?

    Correct

    Alopecia areata is usually diagnosed clinically, with a biopsy rarely being needed. Exclamation hairs, which are short broken off hairs that are narrower closer to the scalp, are pathognomonic for Alopecia areata, but are not always seen in the disease (2,4). While Tinea capitis is a cause of alopecia, it has a slightly different presentation then that of alopecia areata. Tinea capitis tends to begin as small erythematous papules that become scaly and typically become ring-like in form. This presentation would warrant an KOH prep examination (2). Telogen effluvium presents with increased shedding and the patients hair will appear thinner, however, there will be no areas of total alopecia in patients with this condition. Telogen effluvium may have a metabolic cause so it is important to check common causes such as hypothyroidism or iron deficiency anemia, which are both common and easily corrected (2). Systemic Lupus Erythematosus is another cause of alopecia, as it is an autoimmune disorder that can affect any organ system. It is most common to have hair loss on the scalp that thins gradually, however, some people may lose hair in clumps. If lupus is suspected, it is important to get labs checking the presence of Antinuclear antibodies (1).

    Case Discussion
    Epidemiology
    The prevalence of Alopecia areata in the general population is .1-.2% with a lifetime risk of developing the condition of 2.1% (1,3). It effects approximately 6.8 million people in the U.S. alone (1). There is noted to be a slight female predominance of the condition, but all races seem to be effected equally. This condition can occur at any age, however, peak incidence is between 15-29 years of life (1,2).
    Etiology and Pathophysiology
    The cause of Alopecia areata is unknown, however there are several hypotheses on what may contribute to this condition, the most supported being that it is an autoimmune mediated process. Studies have found that this is likely a T cell mediated disease, with biopsies showing lymphocytic infiltrate around the hair follicles of effected patients (2). It is unclear what triggers the autoimmune reaction to the hair follicle.
    It is also apparent that genetics play a role in the development in this condition as well. It has been shown that in patients with alopecia areata, around 10-20% of the patients have family members with the same condition (2). Studies have focused primarily on Human leukocyte antigen DQ3 , which has been found to be in more than 80% of patients with alopecia areata (2,3). However, this disease is said to be a “polygenic disease,” meaning multiple genes contribute to the expression of the condition (1).
    Research has been completed targeting nerve and blood supply to the area of scalp affected by this condition , as it is believed to be correlated to symptoms of itching and pain in these patients, however it is unclear whether the findings are significant (2,3). Other studies have been conducted to determine if there is an infectious cause of this condition, however, most studies have been negative.
    History and Physical exam
    Patients with alopecia areata can often present with a precipitating factor in their history, such as a major life event, febrile illness, drugs, pregnancy, or trauma (2). These patients are often asymptomatic other than the discreet patches of hair loss, which are smooth and without scarring (4). A majority of patients have a single patch. On occasion, some patients complain of burning or pruritis in the area. This condition is primarily found on the scalp but can affect any hair bearing areas, including beard, eyebrows and eyelashes (4,2). There are several patterns used to classify alopecia areata, which include reticular, ophiasis, sisaipho, and alopecia universalis. Extensive alopecia areata is less common but involves >50% of hair, which can often lead to total hair loss (alopecia totalis). Patients with alopecia areata also have the presence of small hairs 2-3mm in length called “exclamation hairs” which are pathognomonic but not always found (4). They often border new patches of alopecia areata and indicate that the patch is expanding. Telogen hairs, or hairs in the resting phase, can be easily dislodged with the “Pull test” in this condition.
    Diagnostics
    In most cases alopecia areata is diagnosed clinically. On occasion, a scalp biopsy can be taken when the diagnosis in unclear, however, this is rarely needed (2). On biopsy, peribulbar lymphocytic infiltrate is seen, often resembling a “swarm of bees.” An increase in vellus hairs versus terminal hairs is often seen as well (2).
    Management
    Patients with alopecia areata have several options available to them. However, treatment is not necessary as the condition is benign and spontaneous remissions are also common. The goal of treatment is to stimulate hair growth in the affected areas. The type of treatment varies based on presentation and extent of hair involved.
    Intralesional steroid injections, usually with Kenalog, are often used in the treatment of alopecia areata and are first line in patients with localized lesions (3,4). Regrowth is often seen within 4-6 weeks in patients that respond to this treatment. Topical steroids are often useful in patients who cannot tolerate injections, especially in children (4). Topical immunotherapy is another treatment option that involves stimulating the area with an antigen that can initiate an allergic response which may hopefully trigger regrowth (2,3). Minoxidil is effective in patients with extensive hair loss. It is well tolerated and regrowth can be seen within 12 weeks.
    Systemic treatments are also an option for patients with extensive disease. This includes UV therapy, systemic steroids, cyclosporine, and methotrexate. Additionally, interleukin 2, which targets T regulatory cells, and ruxolitinib, which inhibits JAK are new treatments that may help with immune modulation (4). Unfortunately, alopecia areata is a highly unpredictable disease and these treatments are no guarantee for a cure, thus it is important to counsel these patients on support groups and camouflage techniques that are used as coping mechanisms for living with the disease (2,1).
     
    References

    1. Alopecia Areata | National Alopecia Areata Foundation. Naaforg. 2017. Available at: https://www.naaf.org/alopecia-areata. Accessed October 23, 2017.
    2. Bolduc, MD C. Alopecia Areata: Practice Essentials, Background, Pathophysiology. Emedicinemedscapecom. 2017. Available at: https://emedicine.medscape.com/article/1069931-overview. Accessed October 23, 2017.
    3. Dainichi T, Kabashima K. Alopecia areata: What’s new in epidemiology, pathogenesis, diagnosis, and therapeutic options?. Journal of Dermatological Science. 2017;86(1):3-12. doi:10.1016/j.jdermsci.2016.10.004.
    4. McPhee S, Papadakis M. Current Medical Diagnosis & Treatment 2016. 1st ed. New York: McGraw-Hill Medical; 2016.

    Incorrect

    Alopecia areata is usually diagnosed clinically, with a biopsy rarely being needed. Exclamation hairs, which are short broken off hairs that are narrower closer to the scalp, are pathognomonic for Alopecia areata, but are not always seen in the disease (2,4). While Tinea capitis is a cause of alopecia, it has a slightly different presentation then that of alopecia areata. Tinea capitis tends to begin as small erythematous papules that become scaly and typically become ring-like in form. This presentation would warrant an KOH prep examination (2). Telogen effluvium presents with increased shedding and the patients hair will appear thinner, however, there will be no areas of total alopecia in patients with this condition. Telogen effluvium may have a metabolic cause so it is important to check common causes such as hypothyroidism or iron deficiency anemia, which are both common and easily corrected (2). Systemic Lupus Erythematosus is another cause of alopecia, as it is an autoimmune disorder that can affect any organ system. It is most common to have hair loss on the scalp that thins gradually, however, some people may lose hair in clumps. If lupus is suspected, it is important to get labs checking the presence of Antinuclear antibodies (1).

    Case Discussion
    Epidemiology
    The prevalence of Alopecia areata in the general population is .1-.2% with a lifetime risk of developing the condition of 2.1% (1,3). It effects approximately 6.8 million people in the U.S. alone (1). There is noted to be a slight female predominance of the condition, but all races seem to be effected equally. This condition can occur at any age, however, peak incidence is between 15-29 years of life (1,2).
    Etiology and Pathophysiology
    The cause of Alopecia areata is unknown, however there are several hypotheses on what may contribute to this condition, the most supported being that it is an autoimmune mediated process. Studies have found that this is likely a T cell mediated disease, with biopsies showing lymphocytic infiltrate around the hair follicles of effected patients (2). It is unclear what triggers the autoimmune reaction to the hair follicle.
    It is also apparent that genetics play a role in the development in this condition as well. It has been shown that in patients with alopecia areata, around 10-20% of the patients have family members with the same condition (2). Studies have focused primarily on Human leukocyte antigen DQ3 , which has been found to be in more than 80% of patients with alopecia areata (2,3). However, this disease is said to be a “polygenic disease,” meaning multiple genes contribute to the expression of the condition (1).
    Research has been completed targeting nerve and blood supply to the area of scalp affected by this condition , as it is believed to be correlated to symptoms of itching and pain in these patients, however it is unclear whether the findings are significant (2,3). Other studies have been conducted to determine if there is an infectious cause of this condition, however, most studies have been negative.
    History and Physical exam
    Patients with alopecia areata can often present with a precipitating factor in their history, such as a major life event, febrile illness, drugs, pregnancy, or trauma (2). These patients are often asymptomatic other than the discreet patches of hair loss, which are smooth and without scarring (4). A majority of patients have a single patch. On occasion, some patients complain of burning or pruritis in the area. This condition is primarily found on the scalp but can affect any hair bearing areas, including beard, eyebrows and eyelashes (4,2). There are several patterns used to classify alopecia areata, which include reticular, ophiasis, sisaipho, and alopecia universalis. Extensive alopecia areata is less common but involves >50% of hair, which can often lead to total hair loss (alopecia totalis). Patients with alopecia areata also have the presence of small hairs 2-3mm in length called “exclamation hairs” which are pathognomonic but not always found (4). They often border new patches of alopecia areata and indicate that the patch is expanding. Telogen hairs, or hairs in the resting phase, can be easily dislodged with the “Pull test” in this condition.
    Diagnostics
    In most cases alopecia areata is diagnosed clinically. On occasion, a scalp biopsy can be taken when the diagnosis in unclear, however, this is rarely needed (2). On biopsy, peribulbar lymphocytic infiltrate is seen, often resembling a “swarm of bees.” An increase in vellus hairs versus terminal hairs is often seen as well (2).
    Management
    Patients with alopecia areata have several options available to them. However, treatment is not necessary as the condition is benign and spontaneous remissions are also common. The goal of treatment is to stimulate hair growth in the affected areas. The type of treatment varies based on presentation and extent of hair involved.
    Intralesional steroid injections, usually with Kenalog, are often used in the treatment of alopecia areata and are first line in patients with localized lesions (3,4). Regrowth is often seen within 4-6 weeks in patients that respond to this treatment. Topical steroids are often useful in patients who cannot tolerate injections, especially in children (4). Topical immunotherapy is another treatment option that involves stimulating the area with an antigen that can initiate an allergic response which may hopefully trigger regrowth (2,3). Minoxidil is effective in patients with extensive hair loss. It is well tolerated and regrowth can be seen within 12 weeks.
    Systemic treatments are also an option for patients with extensive disease. This includes UV therapy, systemic steroids, cyclosporine, and methotrexate. Additionally, interleukin 2, which targets T regulatory cells, and ruxolitinib, which inhibits JAK are new treatments that may help with immune modulation (4). Unfortunately, alopecia areata is a highly unpredictable disease and these treatments are no guarantee for a cure, thus it is important to counsel these patients on support groups and camouflage techniques that are used as coping mechanisms for living with the disease (2,1).
     
    References

    1. Alopecia Areata | National Alopecia Areata Foundation. Naaforg. 2017. Available at: https://www.naaf.org/alopecia-areata. Accessed October 23, 2017.
    2. Bolduc, MD C. Alopecia Areata: Practice Essentials, Background, Pathophysiology. Emedicinemedscapecom. 2017. Available at: https://emedicine.medscape.com/article/1069931-overview. Accessed October 23, 2017.
    3. Dainichi T, Kabashima K. Alopecia areata: What’s new in epidemiology, pathogenesis, diagnosis, and therapeutic options?. Journal of Dermatological Science. 2017;86(1):3-12. doi:10.1016/j.jdermsci.2016.10.004.
    4. McPhee S, Papadakis M. Current Medical Diagnosis & Treatment 2016. 1st ed. New York: McGraw-Hill Medical; 2016.

  197. Question 197 of 220
    197. Question

    A 32 year old female presents to outpatient clinic after 1 day of red and irritated right eye with clear discharge. She describes the constant feeling of “sand in the eye” but denies visual change, photosensitivity, flashes or floaters. She also admits to recent contact with “pink eye.” During physical exam, severely injected right eye is discovered with non-purulent discharge. EOMs are intact. Pupils are round and reactive to light and accommodation. What is the best choice for treatment of this patient?

    Correct

    A) Ciprofloxacin would be a good choice for broad spectrum coverage of acute bacterial conjunctivitis. Although this patient does exhibit unilateral injection, her recent contact with “pink eye” is more indicative of a viral, highly contageous, conjunctivitis. She also has clear discharge instead of the purulent discharge, which would be expected with bacterial etiology. 1

    C) Emergent ophthalmologist referral is essential in the setting of acute angle closure glaucoma. However, for this diagnosis we would expect sudden decrease in vision, in addition to a red and painful eye. Discharge would not be as likely and physical exam would find a mid-dialated and non reactive pupil. More concerning finding such as sudden nausea and vomiting would also be expected.1

    D) Artificial tears and lubricating ointment are the treatent of choice for Keratoconjunctivits sica. However, a more insidious onset would be expected with only mild scleral injection. Foreign body sensation is consistent with this diagnosis, however discharge is not.1

    B) Cold compress and good hygeine are the best choice in the setting of viral conjunctivitis. This patient’s presentation of injected, painful eyes with clear discharge is most consistent with viral etiology. Recent infectious contact and lack of purulent discharge make the diagnosis of bacterial conjunctivitis less likely. The sudden onset and severe injection are not consistent with Keratoconjunctivits sicca. Finally, because this patient’s vision is intact, and physical exam finds PERRLA, the diagnosis of acute angle closure Glaucoma is presumptively ruled out. 1

    Conjunctivits refers to a broad category of disorders which involve inflammation of the conjunctiva. These may be infectious or non infectious, acute or chronic. Infectious etiologies may be viral or bacterial. Non-infectious subtypes include allergic, mechanical (irritative or toxic), immune mediated or even neoplastic causes. Acute conjunctivitis is present for 3 to 4 weeks, whereas chronic disease must be present for greater than 4 weeks. 2
    Much like the common cold, viral conjunctivitis is so common that statistics pertaining to epidemiology are often under reported and inaccurate. Outbreaks are particularly common in schools, families, and the military. Acute bacterial conjunctivitis accounts for 1% of all primary care visits in the US with 135 cases per 10,000 people. 3
    The most common etiology for viral conjunctivitis is Adenovirus, although other causes such as Herpes Simplex, Herpes Zoster and Molluscum Contagiosum are possible. Bacterial etiologies include non gonococcal (Staphylococci and Streptococci) and gonococcal, as well as Chlamydia infection. 2
    The epithelial layer of conjunctiva is a crucial barrier against infection. If this defence is disrupted, dangerous alterations in flora or viral contact may occur. Swimming, contact lense use, or self inoculation are all possible mechanisms of contamination. Infection may spread from the adjacent eye, especially in the case of viral disease, or remain isolated on one side. 3
    In the setting of acute viral conjunctivitis, the patient commonly complains of sudden onset of eye irritation, redness, and photophobia. These symptoms may be unilateral or bilateral. Recent contact with conjunctivitis or URI may be noted. On physical exam, the provider is likely to find conjunctival injection, eyelid edema, erythema and watery discharge. Lymphadenopathy and subconjunctival hemorrhage are rare findings. In contrast, bacterial conjunctivitis is associated with purulent drainage and is usually unilateral. 2
    Diagnosis is based heavily on clinical findings. Although secondary bacterial infection is rare in the setting of viral conjunctivitis, culture may be indicated. Culture is also helpful to confirm the diagnosis and guide treatment in simple bacterial infection. 2
    Treatment for viral conjunctivitis consists largely of supportive care. This includes cold compress, artificial tears, and in some cases Anti-histamines. Hand and eye hygiene are also very important elements of patient education. Contact lenses and cosmetics should be temporarily discontinued. Individuals who work with food, young children, or in the health care setting should refrain from work until symptoms resolve. 2
    Topical antibiotic may be considered if bacterial etiology is suspected. These include Ciprofloxacin, Azithromycin, Gentamicin and other broad spectrum choices. Close follow-up is recommended, in four days to one week, but this is typically a self-limiting disease. Ophthalmology referral should be obtained if symptoms do not begin to improve in one week, or more severe disease is suspected.

    References:
    1) Agabi SS. Step up to medicine. Philadelphia, PA: Lippincott Williams & Wilkins; 2004
    2) Dynamed.com.
    http://www.dynamed.com/topics/dmp~AN~T116741/Infectious-conjunctivitis. Accessed October 20, 2017.
    3) Silverman M, Bessman E. Acute Conjunctivitis (Pink Eye). Overview, Clinical Evaluation, Bacterial Conjunctivitis. https://emedicine.medscape.com/article/797874-overview. Published August 17, 2017. Accessed October 20, 2017.

    Incorrect

    A) Ciprofloxacin would be a good choice for broad spectrum coverage of acute bacterial conjunctivitis. Although this patient does exhibit unilateral injection, her recent contact with “pink eye” is more indicative of a viral, highly contageous, conjunctivitis. She also has clear discharge instead of the purulent discharge, which would be expected with bacterial etiology. 1

    C) Emergent ophthalmologist referral is essential in the setting of acute angle closure glaucoma. However, for this diagnosis we would expect sudden decrease in vision, in addition to a red and painful eye. Discharge would not be as likely and physical exam would find a mid-dialated and non reactive pupil. More concerning finding such as sudden nausea and vomiting would also be expected.1

    D) Artificial tears and lubricating ointment are the treatent of choice for Keratoconjunctivits sica. However, a more insidious onset would be expected with only mild scleral injection. Foreign body sensation is consistent with this diagnosis, however discharge is not.1

    B) Cold compress and good hygeine are the best choice in the setting of viral conjunctivitis. This patient’s presentation of injected, painful eyes with clear discharge is most consistent with viral etiology. Recent infectious contact and lack of purulent discharge make the diagnosis of bacterial conjunctivitis less likely. The sudden onset and severe injection are not consistent with Keratoconjunctivits sicca. Finally, because this patient’s vision is intact, and physical exam finds PERRLA, the diagnosis of acute angle closure Glaucoma is presumptively ruled out. 1

    Conjunctivits refers to a broad category of disorders which involve inflammation of the conjunctiva. These may be infectious or non infectious, acute or chronic. Infectious etiologies may be viral or bacterial. Non-infectious subtypes include allergic, mechanical (irritative or toxic), immune mediated or even neoplastic causes. Acute conjunctivitis is present for 3 to 4 weeks, whereas chronic disease must be present for greater than 4 weeks. 2
    Much like the common cold, viral conjunctivitis is so common that statistics pertaining to epidemiology are often under reported and inaccurate. Outbreaks are particularly common in schools, families, and the military. Acute bacterial conjunctivitis accounts for 1% of all primary care visits in the US with 135 cases per 10,000 people. 3
    The most common etiology for viral conjunctivitis is Adenovirus, although other causes such as Herpes Simplex, Herpes Zoster and Molluscum Contagiosum are possible. Bacterial etiologies include non gonococcal (Staphylococci and Streptococci) and gonococcal, as well as Chlamydia infection. 2
    The epithelial layer of conjunctiva is a crucial barrier against infection. If this defence is disrupted, dangerous alterations in flora or viral contact may occur. Swimming, contact lense use, or self inoculation are all possible mechanisms of contamination. Infection may spread from the adjacent eye, especially in the case of viral disease, or remain isolated on one side. 3
    In the setting of acute viral conjunctivitis, the patient commonly complains of sudden onset of eye irritation, redness, and photophobia. These symptoms may be unilateral or bilateral. Recent contact with conjunctivitis or URI may be noted. On physical exam, the provider is likely to find conjunctival injection, eyelid edema, erythema and watery discharge. Lymphadenopathy and subconjunctival hemorrhage are rare findings. In contrast, bacterial conjunctivitis is associated with purulent drainage and is usually unilateral. 2
    Diagnosis is based heavily on clinical findings. Although secondary bacterial infection is rare in the setting of viral conjunctivitis, culture may be indicated. Culture is also helpful to confirm the diagnosis and guide treatment in simple bacterial infection. 2
    Treatment for viral conjunctivitis consists largely of supportive care. This includes cold compress, artificial tears, and in some cases Anti-histamines. Hand and eye hygiene are also very important elements of patient education. Contact lenses and cosmetics should be temporarily discontinued. Individuals who work with food, young children, or in the health care setting should refrain from work until symptoms resolve. 2
    Topical antibiotic may be considered if bacterial etiology is suspected. These include Ciprofloxacin, Azithromycin, Gentamicin and other broad spectrum choices. Close follow-up is recommended, in four days to one week, but this is typically a self-limiting disease. Ophthalmology referral should be obtained if symptoms do not begin to improve in one week, or more severe disease is suspected.

    References:
    1) Agabi SS. Step up to medicine. Philadelphia, PA: Lippincott Williams & Wilkins; 2004
    2) Dynamed.com.
    http://www.dynamed.com/topics/dmp~AN~T116741/Infectious-conjunctivitis. Accessed October 20, 2017.
    3) Silverman M, Bessman E. Acute Conjunctivitis (Pink Eye). Overview, Clinical Evaluation, Bacterial Conjunctivitis. https://emedicine.medscape.com/article/797874-overview. Published August 17, 2017. Accessed October 20, 2017.

  198. Question 198 of 220
    198. Question

    A 60 year old Asian female presents to primary care provider due to worsening dyspnea on exertion and dry, nonproductive cough for the past few months. The patient denies history of smoking, second hand smoke or asthma. She has worked in a laundromat and dry cleaners for the past 40 years using unknown chemicals. She does not wear a mask at work. She denies fever, chills, weight loss, fatigue, myalgias or chest pain. Physical exam shows fine bibasilar inspiratory crackles. Pulmonary function test (PFT) shows TLC reduced and FEV1/FVC increased, FEV1 increased, FVC is reduced. What is the most likely diagnosis of the patient?

    Correct

    A. Is incorrect because COPD is a obstructive breathing issue and the PFT test shows results for a restrictive breathing issue.
    B. Asthma is incorrect because most asthma is diagnosed as a child and not in 60 year olds. Also, asthma is an obstructive breathing issue and the PFT test shows results for a restrictive breathing issue.
    C. Pneumonia is not the answer because symptoms have been going on for a few months with no signs of infection, fever, fatigue, or myalgias.
    D. Pleural effusion is not the answer because on physical exam you would find decreased or absent breath sounds at the bases from the fluid collection.
    E. Interstitial pulmonary fibrosis is the correct answer because of the worsening symptoms, the work place environment that could be causing her to breath in pollutants and chemicals for the past 40 years. Also, IPF is a restrictive lung disease which is consistent with the PFT findings.

    Interstitial lung disease (ILD) is a large umbrella term for many different lung issues. The most common type of ILD is idiopathic pulmonary fibrosis (IPF) which accounts for 40% of all ILDs. IPF is a common cause of restrictive lung disease that causes scar tissue formation in the parenchymal of the lungs (Meltzer and Noble, 2008). It can be caused by inhalation of harmful substances, drugs, infections, radiation or autoimmune conditions (Martino 2016). One of these come in contact with the lungs causing a hypersensitivity reaction which causes inflammation. Our bodies are in charge of wound healing and if something goes wrong in the process our lungs can develop fibrosis or scarring to the tissue. The lung fibrosis then has an issue with expanding causing decreased ventilation, hypoxemia and a restrictive lung disease.

    The prevalence of IPF is 20 cases per 100,000 persons for males and 13 cases per 100,000 persons for female. 66% of patients who are diagnosed with IPF are 60 years of age or older and have a poor prognosis estimating 2-5 years to live from diagnosis (Martino 2016). When these patients arrive to their primary providers most of the symptoms are nonspecific. The most common symptoms are dyspnea on exertion and or a nonproductive cough. The most important part of diagnosis IPF is getting a complete history. The patient could have a social history of smoking, travel history, occupational history of exposure to pollutants, exposure history of asbestos in a house or medications that cause lung fibrosis years after use like Amiodarone, Bleomycin and Nitrofurantoin. Other history needs to be ruled out also such as tuberculosis and human immunodeficiency virus. The physical exam on IPF patients could be positive for fine bibasilar inspiratory crackles which is called Velcro crackles in IPF. The patient should be evaluated for digital clubbing which could occur slowly overtime and found in 50% of IPF patients (Meltzer and Noble, 2008).

    The most important work up for IPF is a pulmonary function test which will show restrictive lung disease. The Total Lung Capacity (TLC) will be reduced, Forced Expiratory Volume in 1 second (FEV1) will be increased, Forced Vital Capacity (FVC) will be reduced and FEV1/FVC will be increased (Martino 2016). Chest x-rays are also useful which will show small irregular opacities less than 1.5mm in diameter, ground-glass opacities, and honey combing which is small translucency in the lungs. The more superior option to diagnosing IPF is by using a High-Resolution Computed Tomography (HRCT). This will imaging tool will diagnosis, assesses disease severity and can find ILD in patients with a negative chest x-ray. The next step in IPF is getting a lung biopsy. Depending on the location of the IPF biopsies can be taken by bronchoscopy, bronchoalveolar lavage or thoracoscopic wedge biopsy.

    The definitive treatment for IPF is lung transplant. There are some new medications that can help with the disease if the patient is not a lung transplant candidate which is Tyrosine Kinase Inhibitors and Antifibrotic Agents. Tyrosine Kinase Inhibitor called Nintedanib was FDA approved for the treatment of IPF in 2014 after it found improvement of FVC in patients and less acute exacerbations (Godfrey, 2017). The common side effect of the medication is diarrhea which caused less than 5% of patients to stop treatment. Nintedanib targets platelet derived growth factor receptors, vascular endothelial growth factor receptors and fibroblast growth factor. The FDA also approved an Antifibrotic Agent called Pirfenidone for IPF in 2014 after it was found to improve FVC over a 52 week time span and less progression of the disease and longer survival (Raghu, 2017). Complications of IPF are increased pulmonary pressures causing pulmonary hypertension or cor pulmonale, repeat acute exacerbations of pulmonary fibrosis or pneumonia (Martino 2016). All IPF patients need to be evaluated for obstructive sleep apnea and lung cancer. If the patient is a current smoker they should be given smoking cessation. These patients need to receive the influenza and pneumococcal vaccine due to their high risk of development with an underlying lung condition. These patients need to stay active, exercise regularly, loss weight if needed, eat a healthy diet. Their provider should start by prescribing pulmonary rehab to help them.

    References

    Godfrey, Amanda M K. Emedicine.medscape.com. (2017). Idiopathic Pulmonary Fibrosis Medication: Tyrosine Kinase Inhibitors, Antifibrotic Agents, Corticosteroid, Systemic, Immunosuppressant Agent. [online] Available at: https://emedicine.medscape.com/article/301226-medication [Accessed 25 Oct. 2017].
    Martino, Linda. Idiopathic pulmonary fibrosis. MCPHS PowerPoint Lecture on April 4, 2016.
    Meltzer, E. and Noble, P. (2008). Idiopathic pulmonary fibrosis. Orphanet Journal of Rare Diseases, [online] 3(1), p.8. Available at: https://ojrd.biomedcentral.com/articles/10.1186/1750-1172-3-8 [Accessed 25 Oct. 2017].
    Raghu, G. Selman, M. Atsjournals.org. (2017). Nintedanib and Pirfenidone. New Antifibrotic Treatments Indicated for Idiopathic Pulmonary Fibrosis Offer Hopes and Raises Questions | American Journal of Respiratory and Critical Care Medicine. [online] Available at: http://www.atsjournals.org/doi/full/10.1164/rccm.201411-2044ED [Accessed 25 Oct. 2017].

    Incorrect

    A. Is incorrect because COPD is a obstructive breathing issue and the PFT test shows results for a restrictive breathing issue.
    B. Asthma is incorrect because most asthma is diagnosed as a child and not in 60 year olds. Also, asthma is an obstructive breathing issue and the PFT test shows results for a restrictive breathing issue.
    C. Pneumonia is not the answer because symptoms have been going on for a few months with no signs of infection, fever, fatigue, or myalgias.
    D. Pleural effusion is not the answer because on physical exam you would find decreased or absent breath sounds at the bases from the fluid collection.
    E. Interstitial pulmonary fibrosis is the correct answer because of the worsening symptoms, the work place environment that could be causing her to breath in pollutants and chemicals for the past 40 years. Also, IPF is a restrictive lung disease which is consistent with the PFT findings.

    Interstitial lung disease (ILD) is a large umbrella term for many different lung issues. The most common type of ILD is idiopathic pulmonary fibrosis (IPF) which accounts for 40% of all ILDs. IPF is a common cause of restrictive lung disease that causes scar tissue formation in the parenchymal of the lungs (Meltzer and Noble, 2008). It can be caused by inhalation of harmful substances, drugs, infections, radiation or autoimmune conditions (Martino 2016). One of these come in contact with the lungs causing a hypersensitivity reaction which causes inflammation. Our bodies are in charge of wound healing and if something goes wrong in the process our lungs can develop fibrosis or scarring to the tissue. The lung fibrosis then has an issue with expanding causing decreased ventilation, hypoxemia and a restrictive lung disease.

    The prevalence of IPF is 20 cases per 100,000 persons for males and 13 cases per 100,000 persons for female. 66% of patients who are diagnosed with IPF are 60 years of age or older and have a poor prognosis estimating 2-5 years to live from diagnosis (Martino 2016). When these patients arrive to their primary providers most of the symptoms are nonspecific. The most common symptoms are dyspnea on exertion and or a nonproductive cough. The most important part of diagnosis IPF is getting a complete history. The patient could have a social history of smoking, travel history, occupational history of exposure to pollutants, exposure history of asbestos in a house or medications that cause lung fibrosis years after use like Amiodarone, Bleomycin and Nitrofurantoin. Other history needs to be ruled out also such as tuberculosis and human immunodeficiency virus. The physical exam on IPF patients could be positive for fine bibasilar inspiratory crackles which is called Velcro crackles in IPF. The patient should be evaluated for digital clubbing which could occur slowly overtime and found in 50% of IPF patients (Meltzer and Noble, 2008).

    The most important work up for IPF is a pulmonary function test which will show restrictive lung disease. The Total Lung Capacity (TLC) will be reduced, Forced Expiratory Volume in 1 second (FEV1) will be increased, Forced Vital Capacity (FVC) will be reduced and FEV1/FVC will be increased (Martino 2016). Chest x-rays are also useful which will show small irregular opacities less than 1.5mm in diameter, ground-glass opacities, and honey combing which is small translucency in the lungs. The more superior option to diagnosing IPF is by using a High-Resolution Computed Tomography (HRCT). This will imaging tool will diagnosis, assesses disease severity and can find ILD in patients with a negative chest x-ray. The next step in IPF is getting a lung biopsy. Depending on the location of the IPF biopsies can be taken by bronchoscopy, bronchoalveolar lavage or thoracoscopic wedge biopsy.

    The definitive treatment for IPF is lung transplant. There are some new medications that can help with the disease if the patient is not a lung transplant candidate which is Tyrosine Kinase Inhibitors and Antifibrotic Agents. Tyrosine Kinase Inhibitor called Nintedanib was FDA approved for the treatment of IPF in 2014 after it found improvement of FVC in patients and less acute exacerbations (Godfrey, 2017). The common side effect of the medication is diarrhea which caused less than 5% of patients to stop treatment. Nintedanib targets platelet derived growth factor receptors, vascular endothelial growth factor receptors and fibroblast growth factor. The FDA also approved an Antifibrotic Agent called Pirfenidone for IPF in 2014 after it was found to improve FVC over a 52 week time span and less progression of the disease and longer survival (Raghu, 2017). Complications of IPF are increased pulmonary pressures causing pulmonary hypertension or cor pulmonale, repeat acute exacerbations of pulmonary fibrosis or pneumonia (Martino 2016). All IPF patients need to be evaluated for obstructive sleep apnea and lung cancer. If the patient is a current smoker they should be given smoking cessation. These patients need to receive the influenza and pneumococcal vaccine due to their high risk of development with an underlying lung condition. These patients need to stay active, exercise regularly, loss weight if needed, eat a healthy diet. Their provider should start by prescribing pulmonary rehab to help them.

    References

    Godfrey, Amanda M K. Emedicine.medscape.com. (2017). Idiopathic Pulmonary Fibrosis Medication: Tyrosine Kinase Inhibitors, Antifibrotic Agents, Corticosteroid, Systemic, Immunosuppressant Agent. [online] Available at: https://emedicine.medscape.com/article/301226-medication [Accessed 25 Oct. 2017].
    Martino, Linda. Idiopathic pulmonary fibrosis. MCPHS PowerPoint Lecture on April 4, 2016.
    Meltzer, E. and Noble, P. (2008). Idiopathic pulmonary fibrosis. Orphanet Journal of Rare Diseases, [online] 3(1), p.8. Available at: https://ojrd.biomedcentral.com/articles/10.1186/1750-1172-3-8 [Accessed 25 Oct. 2017].
    Raghu, G. Selman, M. Atsjournals.org. (2017). Nintedanib and Pirfenidone. New Antifibrotic Treatments Indicated for Idiopathic Pulmonary Fibrosis Offer Hopes and Raises Questions | American Journal of Respiratory and Critical Care Medicine. [online] Available at: http://www.atsjournals.org/doi/full/10.1164/rccm.201411-2044ED [Accessed 25 Oct. 2017].

  199. Question 199 of 220
    199. Question

    A 72 year-old obese male with a history HTN, Hyperlipidemia and Atrial fibrillation presents to the emergency room via EMS for acute onset of sub-sternal chest pain that radiated down his right arm. Pain occurred when he was outside doing yard work. On physical exam, you notice the patient is diaphoretic, using accessory muscles to breathe and clenching his chest with his fist. Initial troponins are not elevated and EKG shows depressed ST segments in leads II, III and avF. Which of the following choices in management should you avoid in this patient to prevent harm?

    Correct

    Choice (B) is correct. Since this patient is possibly having an Inferior wall NSTEMI, you should avoid Nitroglycerin and Morphine because it decreases preload. The right side of the heart is more dependent on preload to maintain cardiac output. Therefore, if this patient is actually having an NSTEMI and preload is decreased, it could be fatally harmful for this patient.
    (A) is not harmful because per ACLS guidelines, a patient with suspected MI should receive full dose ASA and Oxygen and present to the cath lab with a time window of 90 minutes.
    (C) is not harmful because if this patient presented within an hour of onset, initial cardiac enzymes would not be elevated. They begin to elevate in 4-6 hours and peak in 12-24 hours.
    (D) is not harmful because if this patient had ischemia/infarct to their myocardium, abnormal wall motions would show on ECHO. ECHO is useful because it is a noninvasive way to assess the function of the heart.

    NSTEMI
    Epidemiology: Approximately one-third of deaths in individuals over the age of 35 is attributed to coronary heart disease (CHD). A total of 16.5 million people in the United States live with CHD. Males are more common than women and risk increases with age proportionately. (2)
    Etiology: CHD is due to accumulation of plaques as each individual ages. Unstable plaques have properties within them that can cause platelets to activate and aggregate together and cause adhesions. This leads to a coagulation cascade which can cause a thrombus to form and dislodge and cause a transient or near-complete occlusion. (1)
    Pathophysiology: NSTEMI occurs when myocardial oxygen demand exceeds oxygen supply, most likely due to a decrease perfusion due to a partially obstructed vessel. In addition, decreased perfusion to the myocardium leads to ischemia. Ischemia over a period of time can cause elevations in troponins. Severity of STEMI depends on duration of ischemia, extent of occlusion, degree of underlying atherosclerosis and presence of other comorbidities. (1)
    Pertinent historical findings: Generally, most symptoms include chest pain with exertion or at rest. The pain can radiate to their jaw, shoulder or down their arm. They may experience nausea, diaphoresis, dizziness and lightheadedness. Symptoms vary from person to person, especially in women and diabetics, who are likely to have atypical variations of symptoms. Many patients will admit to progressively worsening and more intermittent chest pain with exertion. (3)
    Pertinent physical findings: Physical findings can also vary from person to person depending on degree of severity. Typically, these patients are in acute chest pain, clenching their chest with their first. They can use accessory muscles to breath and be short of breath. They can also be diaphoretic. However, other patients can present asymptomatically and have stable vital signs and be in no acute distress. The presentation really varies from person to person. (3)
    Diagnostic Correlation (Labs/Imaging): Findings depend on the time of onset of symptoms to presentation to the hospital setting. Initiate labs can be negative therefore it is important to continue to trend cardiac enzymes and monitor for an elevation indicating ischemia or infarct. Cardiac enzymes include CK-MB and troponins. EKG can also vary but typically, NSTEMI presents with depressed ST segments. EKG can also show T wave flattening or inversion and new Q waves not previously present on prior EKGs. ECHOs are also a noninvasive imaging technique that is useful to show wall abnormalities due to wall infarct or ischemia. Stress tests are useful in stable patients who present with a concerning history of chest pain worsening with exertion. Different stress tests include exercise stress test but those who cannot tolerate exercise stress tests will be given a pharmacological stress test and an ECHO can be used to show whether ischemia is reversible or not. The gold standard of imaging is a cardiac catherization which can outline and show occlusions to the vessel. A cardiac catherization is considered diagnostic and interventional management as well. (2)
    Management/Treatment: Once it is determined that a patient is having ACS, specifically a NSTEMI, the patient should be given ASA 324mg, Oxygen if O2 sat <94% and Nitrates. If nitrates do not relieve the pain, IV Morphine is added onto the regimen. Aside from relieving the pain, Morphine is also a vasodilator that increases the perfusion of blood. The patient should also be started on antithrombotics such as Heparin or Enoxaparin. Ideally, the protocol to the cath lab from entering the emergency room should be less than 90minutes. Within the cath lab, the coronary blockages are identified and if one of two vessels are involved that does not involve the main LCA, a drug eluting stent or bare metal stent is placed. If more than three vessels are involved or the left main coronary artery is involved, the ideal situation is to go to for CABG where the occluded coronary arteries are bypassed. Patients should go home with ASA 81mg, Clopidagrel 75mg, Nitroglycerin SL prn for chest pain, Beta blockers and a high intensity statin such as Atorvastatin or Rosuvastatin. If the EF <40%, an ACE-I/ARB/ARNI should be initiated to decrease progression of heart failure and provide neurohormonal blockade. The patient should follow up with his cardiologist when discharged. (2)

    References
    1. Non-ST Elevation Myocardial Infartion. https://online.epocrates.com/diseases/15124/Non-ST-elevation-myocardial-infarction/Etiology. Accessed 23 October 2017.
    2. Overview of the acute management of Non-ST elevation acute coronary syndromes. https://www.uptodate.com/contents/overview-of-the-acute-management-of-non-st-elevation-acute-coronary-syndromes?source=search_result&search=nstemi&selectedTitle=1~150#H2. Accessed 23 October 2017.
    3. Papadakis MA, McPhee SJ, Rabow MW. 2015 current medical diagnosis & treatment. New York: McGraw-Hill Education/Medical; 2015.

    Incorrect

    Choice (B) is correct. Since this patient is possibly having an Inferior wall NSTEMI, you should avoid Nitroglycerin and Morphine because it decreases preload. The right side of the heart is more dependent on preload to maintain cardiac output. Therefore, if this patient is actually having an NSTEMI and preload is decreased, it could be fatally harmful for this patient.
    (A) is not harmful because per ACLS guidelines, a patient with suspected MI should receive full dose ASA and Oxygen and present to the cath lab with a time window of 90 minutes.
    (C) is not harmful because if this patient presented within an hour of onset, initial cardiac enzymes would not be elevated. They begin to elevate in 4-6 hours and peak in 12-24 hours.
    (D) is not harmful because if this patient had ischemia/infarct to their myocardium, abnormal wall motions would show on ECHO. ECHO is useful because it is a noninvasive way to assess the function of the heart.

    NSTEMI
    Epidemiology: Approximately one-third of deaths in individuals over the age of 35 is attributed to coronary heart disease (CHD). A total of 16.5 million people in the United States live with CHD. Males are more common than women and risk increases with age proportionately. (2)
    Etiology: CHD is due to accumulation of plaques as each individual ages. Unstable plaques have properties within them that can cause platelets to activate and aggregate together and cause adhesions. This leads to a coagulation cascade which can cause a thrombus to form and dislodge and cause a transient or near-complete occlusion. (1)
    Pathophysiology: NSTEMI occurs when myocardial oxygen demand exceeds oxygen supply, most likely due to a decrease perfusion due to a partially obstructed vessel. In addition, decreased perfusion to the myocardium leads to ischemia. Ischemia over a period of time can cause elevations in troponins. Severity of STEMI depends on duration of ischemia, extent of occlusion, degree of underlying atherosclerosis and presence of other comorbidities. (1)
    Pertinent historical findings: Generally, most symptoms include chest pain with exertion or at rest. The pain can radiate to their jaw, shoulder or down their arm. They may experience nausea, diaphoresis, dizziness and lightheadedness. Symptoms vary from person to person, especially in women and diabetics, who are likely to have atypical variations of symptoms. Many patients will admit to progressively worsening and more intermittent chest pain with exertion. (3)
    Pertinent physical findings: Physical findings can also vary from person to person depending on degree of severity. Typically, these patients are in acute chest pain, clenching their chest with their first. They can use accessory muscles to breath and be short of breath. They can also be diaphoretic. However, other patients can present asymptomatically and have stable vital signs and be in no acute distress. The presentation really varies from person to person. (3)
    Diagnostic Correlation (Labs/Imaging): Findings depend on the time of onset of symptoms to presentation to the hospital setting. Initiate labs can be negative therefore it is important to continue to trend cardiac enzymes and monitor for an elevation indicating ischemia or infarct. Cardiac enzymes include CK-MB and troponins. EKG can also vary but typically, NSTEMI presents with depressed ST segments. EKG can also show T wave flattening or inversion and new Q waves not previously present on prior EKGs. ECHOs are also a noninvasive imaging technique that is useful to show wall abnormalities due to wall infarct or ischemia. Stress tests are useful in stable patients who present with a concerning history of chest pain worsening with exertion. Different stress tests include exercise stress test but those who cannot tolerate exercise stress tests will be given a pharmacological stress test and an ECHO can be used to show whether ischemia is reversible or not. The gold standard of imaging is a cardiac catherization which can outline and show occlusions to the vessel. A cardiac catherization is considered diagnostic and interventional management as well. (2)
    Management/Treatment: Once it is determined that a patient is having ACS, specifically a NSTEMI, the patient should be given ASA 324mg, Oxygen if O2 sat <94% and Nitrates. If nitrates do not relieve the pain, IV Morphine is added onto the regimen. Aside from relieving the pain, Morphine is also a vasodilator that increases the perfusion of blood. The patient should also be started on antithrombotics such as Heparin or Enoxaparin. Ideally, the protocol to the cath lab from entering the emergency room should be less than 90minutes. Within the cath lab, the coronary blockages are identified and if one of two vessels are involved that does not involve the main LCA, a drug eluting stent or bare metal stent is placed. If more than three vessels are involved or the left main coronary artery is involved, the ideal situation is to go to for CABG where the occluded coronary arteries are bypassed. Patients should go home with ASA 81mg, Clopidagrel 75mg, Nitroglycerin SL prn for chest pain, Beta blockers and a high intensity statin such as Atorvastatin or Rosuvastatin. If the EF <40%, an ACE-I/ARB/ARNI should be initiated to decrease progression of heart failure and provide neurohormonal blockade. The patient should follow up with his cardiologist when discharged. (2)

    References
    1. Non-ST Elevation Myocardial Infartion. https://online.epocrates.com/diseases/15124/Non-ST-elevation-myocardial-infarction/Etiology. Accessed 23 October 2017.
    2. Overview of the acute management of Non-ST elevation acute coronary syndromes. https://www.uptodate.com/contents/overview-of-the-acute-management-of-non-st-elevation-acute-coronary-syndromes?source=search_result&search=nstemi&selectedTitle=1~150#H2. Accessed 23 October 2017.
    3. Papadakis MA, McPhee SJ, Rabow MW. 2015 current medical diagnosis & treatment. New York: McGraw-Hill Education/Medical; 2015.

  200. Question 200 of 220
    200. Question


    A 58 year-old male presents to the emergency department with a chainsaw injury to the left forearm. He was trimming some trees when he lost control of the saw. On exam you note a deep laceration at the mid-forearm (figure 1). Based on the location and depth of the wound, which nerve is most likely to be injured?

    Correct

    An understanding of anatomy is crucial for early recognition of injuries to tendons, muscles, and nerves. An accurate assessment of the injury and physical exam findings is also necessary to determine if the patient can be washed out and sent home from the ED or if urgent nerve, artery, and/or tendon repair is necessary.
    On the flexor or palmar aspect of the forearm the most superficial layer is made of 4 muscles that arise from the medial epicondyle and fan out across the forearm. From a radial to ulna direction these muscles include the pronator teres, flexor carpi radialis, palmaris longus, and flexor carpi ulnaris. The “mobile wad” of muscle that arises from the lateral epicondyle on the lateral and palmar aspect of the forearm is made of the brachioradialis and the extensor carpi radialis longus and brevis. 1,2
    The location of the nerves and vessels of the anterior forearm is relatively simple to remember. The radial nerve runs down the radial or lateral side of the forearm with the radial artery running along the medial side of the nerve at the distal half of the forearm. The ulnar nerve runs down the unlar side of the forearm with the ulnar artery running along the lateral boarder of the nerve at the distal half of the forearm. The median nerve courses down the middle of the forearm. The anterior interosseous nerve, a branch of the median nerve, courses deep to the median nerve in the mid forearm. 1,2
    The depth and location of this patient’s laceration suggest that the superficial muscles of the mid forearm may have been injured including the brachioradialis, flexor carpi radialis, palmaris longus, and the flexor carpi ulnaris muscles. The wound is deepest in the middle of the forearm which suggest the median nerve is at the highest risk of being injured.
    References
    1. Flexor Tendon Injuries. http://www.orthobullets.com. Accessed 7/23/17.
    2. Thompson JC. Netter’s Concise Atlas of Orhtopedic Anatomy. Elsevier, Inc. Philadelphia, PA. 2002. Pages 108-114.

    Incorrect

    An understanding of anatomy is crucial for early recognition of injuries to tendons, muscles, and nerves. An accurate assessment of the injury and physical exam findings is also necessary to determine if the patient can be washed out and sent home from the ED or if urgent nerve, artery, and/or tendon repair is necessary.
    On the flexor or palmar aspect of the forearm the most superficial layer is made of 4 muscles that arise from the medial epicondyle and fan out across the forearm. From a radial to ulna direction these muscles include the pronator teres, flexor carpi radialis, palmaris longus, and flexor carpi ulnaris. The “mobile wad” of muscle that arises from the lateral epicondyle on the lateral and palmar aspect of the forearm is made of the brachioradialis and the extensor carpi radialis longus and brevis. 1,2
    The location of the nerves and vessels of the anterior forearm is relatively simple to remember. The radial nerve runs down the radial or lateral side of the forearm with the radial artery running along the medial side of the nerve at the distal half of the forearm. The ulnar nerve runs down the unlar side of the forearm with the ulnar artery running along the lateral boarder of the nerve at the distal half of the forearm. The median nerve courses down the middle of the forearm. The anterior interosseous nerve, a branch of the median nerve, courses deep to the median nerve in the mid forearm. 1,2
    The depth and location of this patient’s laceration suggest that the superficial muscles of the mid forearm may have been injured including the brachioradialis, flexor carpi radialis, palmaris longus, and the flexor carpi ulnaris muscles. The wound is deepest in the middle of the forearm which suggest the median nerve is at the highest risk of being injured.
    References
    1. Flexor Tendon Injuries. http://www.orthobullets.com. Accessed 7/23/17.
    2. Thompson JC. Netter’s Concise Atlas of Orhtopedic Anatomy. Elsevier, Inc. Philadelphia, PA. 2002. Pages 108-114.

  201. Question 201 of 220
    201. Question

    A 22 year old, otherwise healthy, female presents to your family medicine office with 3 months of diffuse, cramping abdominal pain. The stomach pain is intermittent with episodes lasting from 2 minutes to 20 minutes. Patient admits to 2-3 bowel movements a week with small hard stools. She denies diarrhea or bloody stools. Patient is sitting comfortably and in no acute distress. Physical exam reveals a non-distended diffusely tender abdomen with predominantly left lower quadrant tenderness. Patient denies fever, tenesmus, nausea, or vomiting. She denies any medication use, significant past medical history, or past surgeries. The patient also has a guaiac negative stool test. What would be your initial treatment of choice for this patient?

    Correct

    A. This medication would be used to induce remission in a patient with Ulcerative Colitis. Ulcerative colitis is a chronic inflammatory disease of the colon or rectal mucosa that usually begins in adolescence or young adulthood. Abdominal pain and left lower quadrant tenderness are present in this patient, but she denies hematochezia and tenesmus, two common symptoms of UC. Therefore, UC is unlikely the cause of symptoms in this patient’s case.1

    B. This treatment plan would be appropriate for a patient experiencing paralytic ileus, which is decreased or absent peristalsis. Common causes of paralytic ileus include medications such as narcotics and anticholinergics. Other common causes include hospitalization, immobilization, shock, and spinal cord injury. This patient is an otherwise healthy female who denies any medication use, significant past medical history, or past surgeries so paralytic ileus is highly unlikely.1

    C. This patient admits to 2-3 bowel movements a week with small hard stools and episodes of diffuse cramping abdominal pain. These symptoms are consistent with constipation. Modification of lifestyle factors is the first line approach to symptomatic constipation. Patients should increase fiber in their diet, and reduce consumption of red meat, fried and fatty foods, and dairy. It is also recommended that patients increase their water intake to 1.5-2 liters a day. Patient should also work to increase their daily physical activity.3

    D. This medication would be used to treat a patient with Ogilvie’s Syndrome, specifically with a cecal diameter > 12 cm. Ogilvie’s Syndrome is a colonic pseudo-obstruction, meaning dilation of the colon in the absence of mechanical obstruction. Symptoms include constipation and abdominal pain. However, the hallmark of Ogilvie’s Syndrome is abdominal distention. This patient was found to have a non-distended abdomen upon physical exam so Ogilvie’s Syndrome is a doubtful diagnosis.4

    Constipation is defined as unsatisfactory defecation with infrequent or difficult to pass stools, or both.3 It is a symptom, rather than a disease. It is the most common digestive complaint in the US, affecting 2-27% of the North American population.2,3 Risk factors for constipation include female sex, age older than 65 years, a low fiber diet, lack of physical activity, or having an endocrine or neuromuscular disorder.
    Constipation can be divided into two main groups, primary and secondary constipation. There are three types of primary constipation: functional constipation, slow transit constipation, and outlet dysfunction constipation. Functional constipation is either idiopathic or caused by irritable bowel syndrome. Slow transit constipation is caused by idiopathic colonic inertia or delayed movements. Outlet dysfunction constipation is most commonly due to the inability of pelvic floor and anal muscles to relax during straining. Other causes of outlet dysfunction constipation include Hirschsprung disease or impaired rectal sensation.3 Secondary constipation can be due to dietary issues and reduced levels of exercise.2 Many medications can contribute to constipation such as narcotic analgesics, antacids, calcium channel blockers, and anticholinergics. Structural causes of secondary constipation include anal fissures, thrombosed hemorrhoids, and obstructing tumors. Systemic diseases such as scleroderma, hypothyroidism, hypercalcemia, and diabetic autonomic neuropathy can also cause secondary constipation.1
    The Rome criteria were initially introduced in 1988 to become the research-standard definition of constipation. It has been modified twice since then, and is now the Rome III criteria. In order to meet the Rome III criteria for constipation, a patient must have experienced at least two of the follow symptoms over the preceding three months: fewer than 3 bowel movements per week, straining during bowel movements, lumpy or hard stools, sensation of anorectal obstruction or incomplete defecation, or manually facilitating defecations. Additionally, the patient should not meet criteria for irritable bowel syndrome and loose stools are rarely present without laxatives. 2
    Review the patient’s current medication list, ask about past abdominal and pelvic surgeries, and medical conditions with increased risk for constipation. Always review potentially modifiable risk factors for constipation such as daily fiber, fluid intake, and level of physical activity. It is important to ask the patient about “red flag” signs and symptoms that may be suggestive of a colorectal malignancy. These signs and symptoms include rectal bleeding, change in the caliber of stool, bloody stools, weight loss, anemia, or family history of colorectal cancer.3
    Physical exam findings may include a distended abdomen with tenderness upon palpation. Palpation of the abdomen may also reveal a palpable colon with hard stool, nonfecal masses, and scars from previous abdominal or pelvis surgeries. External anorectal exam may reveal fistulas, fissures, hemorrhoids, or rectal mucosal prolapse. Other physical exams may include assessment of the anal wink, perineal descent, and digital rectal exam. Evaluate the patient’s anal wink by stroking the surrounding anal skin and looking for a reflexive external sphincter contraction. Assess patient’s perineal descent by observing the patient’s perineum both at rest and while bearing down. Normal perineal descent is 1.0-3.5 cm. A descent < 1 cm may indicate the inability to relax the pelvic floor muscles. A descent >3.5 cm may indicate laxity of perineum. A digital rectal exam may reveal the presence of a mass of stool impaction. Also feel for possible strictures, assess sphincter tone, and note any tenderness. Ask women to perform a Valsalva maneuver to evaluate for pelvic organ prolapse, specifically a rectocele.3
    Constipation is usually diagnosed clinically using problem specific history and physical exam and the Rome III criteria. Routine use of blood tests, x-rays, or endoscopy is not recommended in patients without alarm symptoms or other significant comorbidities.3 An extensive workup would be performed after 3-6 months of failed medical management. Additional studies for refractory constipation or those without an unknown underlying cause include lower GI endoscopy, colonic transit study, defecography, anorectal manometry, surface anal electromyography, and balloon expulsion.2
    The first line approach to symptomatic chronic constipation is lifestyle modifications. Recommend patients intake 1.5 L-2.0 L of water a day and 20-35 grams of fiber a day. Types of dietary fiber include oat or wheat bran. Patients can also try eating prunes for mild to moderate constipation. Educate patients on reducing fried and fatty foods, red meat, and large amounts of dairy products. First line medications can include bulk-forming agents such as methylcellulose powder, polycarbophil, or psyllium powder. Osmotic laxatives such as polyethylene glycol and lactulose are also appropriate for treating chronic constipation. Second line therapy can include stimulant laxatives such as bisacodyl (Dulcolax), but only for short-term use. Consider prosecretory agents, lubiprostone and linaclotide, for constipation refractory to dietary modifications, osmotic laxatives, and stimulant laxatives. There is insufficient evidence to support the use of enemas and probiotics in chronic constipation, but suppositories may be effective for short-term use.3

    References
    1. Agabegi ED, Agabegi SS, undefined undefined undefined. Diseases of the Gastrointestinal System. In: Step-up to medicine. Philadelphia: Wolters Kluwer; 2016:114-120.
    2. Basson M. Constipation. Medscape. https://emedicine.medscape.com/article/184704-overview. Published March 28, 2017. Accessed October 21, 2017.
    3. Ostrovsky DA. Constipation in Adults. dynamed.com. http://www.dynamed.com/topics/dmp~AN~T116186/Constipation-in-adults. Accessed October 21, 2017.
    4. Williams DA. PANCE prep pearls: a practical study guide to preparing for physician assistant national certifying exam (PANCE), recertification (PANRE) & clinical rotations. Lexington, KY: CreateSpace; 2014.

    Incorrect

    A. This medication would be used to induce remission in a patient with Ulcerative Colitis. Ulcerative colitis is a chronic inflammatory disease of the colon or rectal mucosa that usually begins in adolescence or young adulthood. Abdominal pain and left lower quadrant tenderness are present in this patient, but she denies hematochezia and tenesmus, two common symptoms of UC. Therefore, UC is unlikely the cause of symptoms in this patient’s case.1

    B. This treatment plan would be appropriate for a patient experiencing paralytic ileus, which is decreased or absent peristalsis. Common causes of paralytic ileus include medications such as narcotics and anticholinergics. Other common causes include hospitalization, immobilization, shock, and spinal cord injury. This patient is an otherwise healthy female who denies any medication use, significant past medical history, or past surgeries so paralytic ileus is highly unlikely.1

    C. This patient admits to 2-3 bowel movements a week with small hard stools and episodes of diffuse cramping abdominal pain. These symptoms are consistent with constipation. Modification of lifestyle factors is the first line approach to symptomatic constipation. Patients should increase fiber in their diet, and reduce consumption of red meat, fried and fatty foods, and dairy. It is also recommended that patients increase their water intake to 1.5-2 liters a day. Patient should also work to increase their daily physical activity.3

    D. This medication would be used to treat a patient with Ogilvie’s Syndrome, specifically with a cecal diameter > 12 cm. Ogilvie’s Syndrome is a colonic pseudo-obstruction, meaning dilation of the colon in the absence of mechanical obstruction. Symptoms include constipation and abdominal pain. However, the hallmark of Ogilvie’s Syndrome is abdominal distention. This patient was found to have a non-distended abdomen upon physical exam so Ogilvie’s Syndrome is a doubtful diagnosis.4

    Constipation is defined as unsatisfactory defecation with infrequent or difficult to pass stools, or both.3 It is a symptom, rather than a disease. It is the most common digestive complaint in the US, affecting 2-27% of the North American population.2,3 Risk factors for constipation include female sex, age older than 65 years, a low fiber diet, lack of physical activity, or having an endocrine or neuromuscular disorder.
    Constipation can be divided into two main groups, primary and secondary constipation. There are three types of primary constipation: functional constipation, slow transit constipation, and outlet dysfunction constipation. Functional constipation is either idiopathic or caused by irritable bowel syndrome. Slow transit constipation is caused by idiopathic colonic inertia or delayed movements. Outlet dysfunction constipation is most commonly due to the inability of pelvic floor and anal muscles to relax during straining. Other causes of outlet dysfunction constipation include Hirschsprung disease or impaired rectal sensation.3 Secondary constipation can be due to dietary issues and reduced levels of exercise.2 Many medications can contribute to constipation such as narcotic analgesics, antacids, calcium channel blockers, and anticholinergics. Structural causes of secondary constipation include anal fissures, thrombosed hemorrhoids, and obstructing tumors. Systemic diseases such as scleroderma, hypothyroidism, hypercalcemia, and diabetic autonomic neuropathy can also cause secondary constipation.1
    The Rome criteria were initially introduced in 1988 to become the research-standard definition of constipation. It has been modified twice since then, and is now the Rome III criteria. In order to meet the Rome III criteria for constipation, a patient must have experienced at least two of the follow symptoms over the preceding three months: fewer than 3 bowel movements per week, straining during bowel movements, lumpy or hard stools, sensation of anorectal obstruction or incomplete defecation, or manually facilitating defecations. Additionally, the patient should not meet criteria for irritable bowel syndrome and loose stools are rarely present without laxatives. 2
    Review the patient’s current medication list, ask about past abdominal and pelvic surgeries, and medical conditions with increased risk for constipation. Always review potentially modifiable risk factors for constipation such as daily fiber, fluid intake, and level of physical activity. It is important to ask the patient about “red flag” signs and symptoms that may be suggestive of a colorectal malignancy. These signs and symptoms include rectal bleeding, change in the caliber of stool, bloody stools, weight loss, anemia, or family history of colorectal cancer.3
    Physical exam findings may include a distended abdomen with tenderness upon palpation. Palpation of the abdomen may also reveal a palpable colon with hard stool, nonfecal masses, and scars from previous abdominal or pelvis surgeries. External anorectal exam may reveal fistulas, fissures, hemorrhoids, or rectal mucosal prolapse. Other physical exams may include assessment of the anal wink, perineal descent, and digital rectal exam. Evaluate the patient’s anal wink by stroking the surrounding anal skin and looking for a reflexive external sphincter contraction. Assess patient’s perineal descent by observing the patient’s perineum both at rest and while bearing down. Normal perineal descent is 1.0-3.5 cm. A descent < 1 cm may indicate the inability to relax the pelvic floor muscles. A descent >3.5 cm may indicate laxity of perineum. A digital rectal exam may reveal the presence of a mass of stool impaction. Also feel for possible strictures, assess sphincter tone, and note any tenderness. Ask women to perform a Valsalva maneuver to evaluate for pelvic organ prolapse, specifically a rectocele.3
    Constipation is usually diagnosed clinically using problem specific history and physical exam and the Rome III criteria. Routine use of blood tests, x-rays, or endoscopy is not recommended in patients without alarm symptoms or other significant comorbidities.3 An extensive workup would be performed after 3-6 months of failed medical management. Additional studies for refractory constipation or those without an unknown underlying cause include lower GI endoscopy, colonic transit study, defecography, anorectal manometry, surface anal electromyography, and balloon expulsion.2
    The first line approach to symptomatic chronic constipation is lifestyle modifications. Recommend patients intake 1.5 L-2.0 L of water a day and 20-35 grams of fiber a day. Types of dietary fiber include oat or wheat bran. Patients can also try eating prunes for mild to moderate constipation. Educate patients on reducing fried and fatty foods, red meat, and large amounts of dairy products. First line medications can include bulk-forming agents such as methylcellulose powder, polycarbophil, or psyllium powder. Osmotic laxatives such as polyethylene glycol and lactulose are also appropriate for treating chronic constipation. Second line therapy can include stimulant laxatives such as bisacodyl (Dulcolax), but only for short-term use. Consider prosecretory agents, lubiprostone and linaclotide, for constipation refractory to dietary modifications, osmotic laxatives, and stimulant laxatives. There is insufficient evidence to support the use of enemas and probiotics in chronic constipation, but suppositories may be effective for short-term use.3

    References
    1. Agabegi ED, Agabegi SS, undefined undefined undefined. Diseases of the Gastrointestinal System. In: Step-up to medicine. Philadelphia: Wolters Kluwer; 2016:114-120.
    2. Basson M. Constipation. Medscape. https://emedicine.medscape.com/article/184704-overview. Published March 28, 2017. Accessed October 21, 2017.
    3. Ostrovsky DA. Constipation in Adults. dynamed.com. http://www.dynamed.com/topics/dmp~AN~T116186/Constipation-in-adults. Accessed October 21, 2017.
    4. Williams DA. PANCE prep pearls: a practical study guide to preparing for physician assistant national certifying exam (PANCE), recertification (PANRE) & clinical rotations. Lexington, KY: CreateSpace; 2014.

  202. Question 202 of 220
    202. Question

    A retired landscaper presented as a new patient to a dermatologist. He is concerned about a lesion on his hand that has been present for over a year that doesn’t seem to go away. He’s concerned because it now appears as if there is an ulcer in the center of the lesion. The lesion has an erythematous base and central ulceration but no signs of infection or drainage. He denies any pain, itching, or bleeding. Which of the following conditions will the patient most likely be diagnosed with?

    Correct

    Explanation: The initial presentation of squamous cell carcinoma is typically a nonhealing ulcer or growth on a sun exposed area. The patient in this case question presents with this common presentation specifically.
    Choice A: Melanoma is typically diagnosed by a pigmented lesion that varies in color and/or diameter, height, or asymmetry of borders.
    Choice C: Basal Cell Carcinoma is typically diagnosed by a slowly enlarging lesion that does not heal and may bleed when traumatized. These lesions often have a central dimple as well.
    Choice D: Seborrheic Keratosis is a benign growth that often appears as a waxy stuck –on-the-skin look. They often start small and slowly develop a warty appearance. They can range in color from white to black.
    Squamous Cell Carcinoma

    Squamous Cell Carcinoma is the second most common type of skin cancer following behind basal cell carcinoma. Approximately 700,00 new cases are diagnosed in the U.S. each year (3). Unfortunately, up to 8,000 people die of this disease each year and the incidence has increased two hundred percent in the last three decades (2). This type of cancer typically appears on sun-exposed areas of the body such as the dorsal aspect of the hand, face, neck, lips, and ears. Women may commonly get them on the lower legs as well (1). Unlike Basal Cell Carcinoma, this type of skin cancer is more likely to spread to other parts of the body although it is still unlikely (1). Squamous Cell Carcinoma often appears as red scaly patches, open sores, appear to have a central ulceration, or may even take on a warty appearance (2). Although they are most common on sun exposed areas Squamous Cell Carcinoma may also appear on mucous membranes and genitals (2).

    Risk factors for Squamous Cell Carcinoma include pale or light colored skin, light eyes, blonde hair, inability to tan, significant sun exposure, use of tanning beds, exposure to cancer-causing chemicals, and smoking. Previously, diagnosed actinic keratosis also known as solar keratosis may evolve into SSC, although there is only about a 5-7% chance (3). SSC occurs in men twice as much as woman and typically in individuals over the age of 50. Incidence of women in their 30-40s being diagnosed with SCC has increased due to increased use of indoor tanning bed (2). Skin inflammation, chronic infections, and a weakened immune system from HIV, chemotherapy, or anti-rejection drugs all increase the risk of developing squamous cell carcinoma (2). A tumor suppressor gene called TP53 is the gene most often altered in SCC. When TP53 is altered it allows abnormal cells to go on to live longer and potentially become cancerous (1).

    Symptoms of Squamous Cell Carcinoma include a small, pink, dry, scaly patch on the skin. This spot may feel rough or irritated to the touch. The lesion may also cause the patient a burning or itching sensation on the spot. When SCC affects the lips it may present as consistently feeling dry and may have a whitish color and feel scaly (3). SCC often appears as sores on the skin that over time refuse to heal and may even slowly progress to a larger lesion (1). Unfortunately, the appearance of SCC can vary greatly from patient to patient so it is most important to pay close attention to new or changing skin growths, spots that don’t heal, or areas that itch, bleed, or burn (1). From time to time SSC may arise unprovoked on otherwise normal or healthy looking skin and researchers tend to believe these lesions are inherited and that there is a familial factor involved (1).

    In the office, a dermatologist along with obtaining a history and physical exam, including the palpitation of lymph nodes, may also use a dermoscopy. This is also called a epiluminescence microscopy which allows the provider to see the skin more closely and clearly (1). The only way to obtain a definitive diagnosis of Squamous Cell Carcinoma or any type of skin cancer is with a skin biopsy. Typically, this is done by a simple shave biopsy, but a punch biopsy may also be done (3). Another option is an incisional versus excisional biopsy. An incisional biopsy involves just removing only a portion of the tumor while an excisional biopsy removed the entire tumor (1). Although it is rare sometimes both Squamous Cell Carcinoma and Basal Cell Carcinoma may spread beyond the skin. When it does spread the first place it typically travels to is nearby lymph nodes. If a patient presents with lymphadenopathy and a positive biopsy a fine needle aspiration biopsy may be performed to analyze for any cancer cells. At times fine needle aspiration biopsies come back negative but a provider may still have a strong clinical suspicion that the cancer has spread to the lymph system. In this case a surgical excisional lymph node biopsy may be performed (1).

    For Squamous Cell Carcinoma it is often important to use a staging system in order determine a patient’s prognosis and chose the best treatment options. For Basal Cell Carcinoma and Squamous Cell Carcinoma often the American Joint Commission on Cancer TNM system. T represents the main tumor and its site, location, and how far it has extended within the skin. N represents the spread to nearby lymph nodes. M represents metastasis to other parts of the body (1).

    Squamous Cell Carcinoma when diagnosed and treated in the early stages is usually always curable. If not treated over time they will invade underlying tissue and become disfiguring. A very small percentage can even spread to the lymph system, other tissues, and even organs becoming fatal (2). One treatment option included MOHs surgery which offers the highest cure rate for difficult Squamous Cell Cancers. In this procedure the provider cuts out the lesion in addition to a very small amount of normal looking skin. The provider then uses a microscope to analyze the frozen cross section of the specimen to see if any cancer cells can be identified. If the margins are not clear the provider will continue to go back and remove more tissue and repeat the process until clear margins are obtained. MOHs is often used in difficult areas or areas where tissue conservation is important such as the face, scalp, neck, hands, and shins. Excision is also a method of treatment use for SCC in which the lesion is excised along with 4-8mm of surrounding healthy tissue. The tissue is then sent out to a pathologist to be checked to make sure there are clear margins. Radiation is a treatment option that is only used for lesions that can’t be cut out. For early SCCs curettage and electrodessication or laser treatments may be used. Chemotherapy cream such as 5-fluorouracil (5-FU) can also be used to treat SCC in its early stages (3).

    References:
    Basal and Squamous Cell Skin Cancer. American Cancer Society. https://www.cancer.org/cancer/basal-and-squamous-cell-skin-cancer.html. Accessed October 21, 2017.
    Skin Cancer Foundation. Squamous Cell Carcinoma (SCC) – SkinCancer.org.
    http://www.skincancer.org/skin-cancer-infortmation/squamous-cell-carcinoma. Accessed October 21, 2017.
    Squamous cell carcinoma. Squamous cell carcinoma American Academy of Dermatology. https://www.aad.org/public/diseases/skin-cancer/squamous-cell-carcinoma. Accessed October 21, 2017.

    Incorrect

    Explanation: The initial presentation of squamous cell carcinoma is typically a nonhealing ulcer or growth on a sun exposed area. The patient in this case question presents with this common presentation specifically.
    Choice A: Melanoma is typically diagnosed by a pigmented lesion that varies in color and/or diameter, height, or asymmetry of borders.
    Choice C: Basal Cell Carcinoma is typically diagnosed by a slowly enlarging lesion that does not heal and may bleed when traumatized. These lesions often have a central dimple as well.
    Choice D: Seborrheic Keratosis is a benign growth that often appears as a waxy stuck –on-the-skin look. They often start small and slowly develop a warty appearance. They can range in color from white to black.
    Squamous Cell Carcinoma

    Squamous Cell Carcinoma is the second most common type of skin cancer following behind basal cell carcinoma. Approximately 700,00 new cases are diagnosed in the U.S. each year (3). Unfortunately, up to 8,000 people die of this disease each year and the incidence has increased two hundred percent in the last three decades (2). This type of cancer typically appears on sun-exposed areas of the body such as the dorsal aspect of the hand, face, neck, lips, and ears. Women may commonly get them on the lower legs as well (1). Unlike Basal Cell Carcinoma, this type of skin cancer is more likely to spread to other parts of the body although it is still unlikely (1). Squamous Cell Carcinoma often appears as red scaly patches, open sores, appear to have a central ulceration, or may even take on a warty appearance (2). Although they are most common on sun exposed areas Squamous Cell Carcinoma may also appear on mucous membranes and genitals (2).

    Risk factors for Squamous Cell Carcinoma include pale or light colored skin, light eyes, blonde hair, inability to tan, significant sun exposure, use of tanning beds, exposure to cancer-causing chemicals, and smoking. Previously, diagnosed actinic keratosis also known as solar keratosis may evolve into SSC, although there is only about a 5-7% chance (3). SSC occurs in men twice as much as woman and typically in individuals over the age of 50. Incidence of women in their 30-40s being diagnosed with SCC has increased due to increased use of indoor tanning bed (2). Skin inflammation, chronic infections, and a weakened immune system from HIV, chemotherapy, or anti-rejection drugs all increase the risk of developing squamous cell carcinoma (2). A tumor suppressor gene called TP53 is the gene most often altered in SCC. When TP53 is altered it allows abnormal cells to go on to live longer and potentially become cancerous (1).

    Symptoms of Squamous Cell Carcinoma include a small, pink, dry, scaly patch on the skin. This spot may feel rough or irritated to the touch. The lesion may also cause the patient a burning or itching sensation on the spot. When SCC affects the lips it may present as consistently feeling dry and may have a whitish color and feel scaly (3). SCC often appears as sores on the skin that over time refuse to heal and may even slowly progress to a larger lesion (1). Unfortunately, the appearance of SCC can vary greatly from patient to patient so it is most important to pay close attention to new or changing skin growths, spots that don’t heal, or areas that itch, bleed, or burn (1). From time to time SSC may arise unprovoked on otherwise normal or healthy looking skin and researchers tend to believe these lesions are inherited and that there is a familial factor involved (1).

    In the office, a dermatologist along with obtaining a history and physical exam, including the palpitation of lymph nodes, may also use a dermoscopy. This is also called a epiluminescence microscopy which allows the provider to see the skin more closely and clearly (1). The only way to obtain a definitive diagnosis of Squamous Cell Carcinoma or any type of skin cancer is with a skin biopsy. Typically, this is done by a simple shave biopsy, but a punch biopsy may also be done (3). Another option is an incisional versus excisional biopsy. An incisional biopsy involves just removing only a portion of the tumor while an excisional biopsy removed the entire tumor (1). Although it is rare sometimes both Squamous Cell Carcinoma and Basal Cell Carcinoma may spread beyond the skin. When it does spread the first place it typically travels to is nearby lymph nodes. If a patient presents with lymphadenopathy and a positive biopsy a fine needle aspiration biopsy may be performed to analyze for any cancer cells. At times fine needle aspiration biopsies come back negative but a provider may still have a strong clinical suspicion that the cancer has spread to the lymph system. In this case a surgical excisional lymph node biopsy may be performed (1).

    For Squamous Cell Carcinoma it is often important to use a staging system in order determine a patient’s prognosis and chose the best treatment options. For Basal Cell Carcinoma and Squamous Cell Carcinoma often the American Joint Commission on Cancer TNM system. T represents the main tumor and its site, location, and how far it has extended within the skin. N represents the spread to nearby lymph nodes. M represents metastasis to other parts of the body (1).

    Squamous Cell Carcinoma when diagnosed and treated in the early stages is usually always curable. If not treated over time they will invade underlying tissue and become disfiguring. A very small percentage can even spread to the lymph system, other tissues, and even organs becoming fatal (2). One treatment option included MOHs surgery which offers the highest cure rate for difficult Squamous Cell Cancers. In this procedure the provider cuts out the lesion in addition to a very small amount of normal looking skin. The provider then uses a microscope to analyze the frozen cross section of the specimen to see if any cancer cells can be identified. If the margins are not clear the provider will continue to go back and remove more tissue and repeat the process until clear margins are obtained. MOHs is often used in difficult areas or areas where tissue conservation is important such as the face, scalp, neck, hands, and shins. Excision is also a method of treatment use for SCC in which the lesion is excised along with 4-8mm of surrounding healthy tissue. The tissue is then sent out to a pathologist to be checked to make sure there are clear margins. Radiation is a treatment option that is only used for lesions that can’t be cut out. For early SCCs curettage and electrodessication or laser treatments may be used. Chemotherapy cream such as 5-fluorouracil (5-FU) can also be used to treat SCC in its early stages (3).

    References:
    Basal and Squamous Cell Skin Cancer. American Cancer Society. https://www.cancer.org/cancer/basal-and-squamous-cell-skin-cancer.html. Accessed October 21, 2017.
    Skin Cancer Foundation. Squamous Cell Carcinoma (SCC) – SkinCancer.org.
    http://www.skincancer.org/skin-cancer-infortmation/squamous-cell-carcinoma. Accessed October 21, 2017.
    Squamous cell carcinoma. Squamous cell carcinoma American Academy of Dermatology. https://www.aad.org/public/diseases/skin-cancer/squamous-cell-carcinoma. Accessed October 21, 2017.

  203. Question 203 of 220
    203. Question

    A 55 year-old man presents to the ER with acute onset right sided flank pain associated with nausea and vomiting. He has no fever. WBC count is wnl. UA is positive for microscopic hematuria. Non contrast CT shows a right sided 8mm mid ureteral stone. The right renal pelvis appears distended. What is the appropriate management of this patient?

    Correct

    Ureteral stenting is the correct answer. In this scenario the patient is noted to have a dilated right renal pelvis proximal to the visualized stone. This means the patient is obstructed. This requires an urgent referral to urology and the immediate insertion of a ureteral stent past the obstruction with a cystoscope. If the pressure on the kidney is not relieved expediently kidney damage can result.
    SWL is not the correct answer because it is not guaranteed to work to relieve the obstruction and cannot be coordinated quickly enough to be of use. SWL can however be used once the stent has been placed to fragment the stone.
    PCNL does not have an application here because it is only useful in large renal stones. Also it cannot be coordinated in time to be of use.
    Medical expulsive therapy is inappropriate because there is a currently great risk to the kidney. It may be true that expulsive therapy with alpha blockers and CCB will relax the ureter enough for the stone to pass but this is not a given and cannot be relied upon. Stenting is the quickest and most effective treatment.

    Etiology:
    Renal calculi develop when normally soluble components of urine such as calcium, uric acid, oxalate, and sodium become supersaturated and fall out of suspension 1. When this happens they precipitate within the urinary tract and over time collect more solute. If they grow large enough they can irritate tissues and even block the flow of urine.
    Epidemiology: Renal stones are more prevalent in men than women 3:1. More prevalent in whites than others. More prevalent in hot and dry geographic locations. Most common in the 4th to 6th decades of life but rates are rising in younger populations. There is a positive correlation between renal calculi and BMI 1.
    Pathophysiology: It appears that the pathophysiology of renal calculi is still not well understood. Certainly the precipitation of supersaturated solutes in the urine plays a role but it is not undisputed which part of the kidney this takes place in. Some studies have shown that plaques can form within the loop of Henle 3. These plaques may provide a collecting point for solutes to form on before they travel distally and collect in the calyces, renal pelvis, ureters, and bladder.

    History & physical exam: The common presentation of acute renal colic is one of a patient waking from sleep with excruciating flank pain. Depending on the location of the stone the pain has different radiculopathy patterns. If the stone is in the kidney, most of the time it is asymptomatic unless the flow of urine has been obstructed. If obstruction is present the pain will be in the deep flank without radiation to the groin. There may be some associated nausea and vomiting because the kidney shares nerve innervation with the stomach and intestines. If the stone is in the upper ureter it will cause severe colicky flank pain that often radiates anterior and caudally 2. This pain may be accompanied by nausea. If the stone is impacted at the UO a patient may experience radiation to the groin or testicle/labia majora. This may be accompanied by urinary frequency and dysuria. Once the stone has passed into the bladder it is most times passed without incident.
    On physical exam a patient will be in severe pain and writhing in contrast to a patient with peritonitis who will stay very still. Most often there will be the presence of microhematuria on UA. They may have migrating flank pain with radiation to groin. Hypoactive bowel sounds may be present. Fever is absent unless they have developed an infection secondary to obstruction.
    Diagnostics:Non contrast CT is most widely used and sensitive diagnostic imaging test. Some will order a KUB if the patient is a known stone factory and their stones can be seen on KUB. Ultrasound can be used for children or pregnant women when radiation exposure is of greater concern. IVP is declining in use but is a useful exam that demonstrates the urinary tract anatomy and function. It requires IV contrast which carries a risk of allergy. A urinalysis will show microhematuria 85% of the time 2. It can show the offending crystals which is a big clue. Leukocytes, nitrites, and pH can show infection. CMP will show renal function and electrolyte abnormalities which may explain the propensity to form stones. A 24 hour urine collection will show if there are elevated crystals or other abnormalities in the urine. CBC may show elevated WBC count if infection present.

    Management and treatment: The first line managements for renal or ureteral colic are supportive measures. These may include fluids, pain relief with morphine or ketorilac, and anti emetics. Medical expulsive therapy which consists of alpha blockers and CCBs is used for non obstructing stones smaller than 1cm 1. Most stones pass within 4-6 weeks and if they don’t will need to be operated on if symptomatic. Obstruction is an indication for cystoscopy and stent insertion. Infection will need to be managed with IV antibiotics and relief of obstruction. If the stone is larger than 1cm it is unlikely that it will pass on its own. In such scenarios extracorporeal shockwave lithotripsy, ureteroscopy, or percutaneous nephrolithotomy can be performed. The benefits of ESWL are that general anesthesia is not requited, it is the least invasive option, and complication rates are low (but not unheard of). Unfortunately ESWL success rates are lower than other interventions. Ureteroscopy requires general anesthesia and entering the urinary system. It’s success rates are higher for most locations of stones except for proximal ureter and renal stones larger than 2cm. For large renal stones PCNL is most effective. It involves general anesthesia and entering the kidney percutaneously through the flank. Risks of complication are higher with this procedure but is the last resort for many large renal stones.
    Depending on the chemistry of the patient’s urine there are certain alkalization agents and binders that can be prescribed to keep the offending salt from forming stones. Uric acid stones can be dissolved by potassium citrate and sodium bicarbonate which lowers the urine pH. Goal pH is 6.5-7 1. Allopurinol can be used for recurrent calcium oxalate stones in patients with a normal urine calcium. Recurrent calcium stones with hypercalciuria can be treated with HCTZ or potassium supplement. Intractable struvite stones can be treated with a urease inhibitor. In addition to these medications 2L of urine output should be maintained along with a low sodium, low protein, low oxalate diet.

    Sources
    1. Antonelli J. Nephrolithiasis. Point of Care Medical Applications | Epocrates. http://www.epocrates.com/. Published April 6, 2017. Accessed October 30, 2017.
    2. Chirag D. Nephrolithiasis. Medscape. http://www.medscape.com. Published December 3, 2016. Accessed October 30, 2017.
    3. McCance KL, Huether SE. Pathophysiology: the biologic basis for disease in adults and children. St. Louis, MO: Elsevier; 2014.

    Incorrect

    Ureteral stenting is the correct answer. In this scenario the patient is noted to have a dilated right renal pelvis proximal to the visualized stone. This means the patient is obstructed. This requires an urgent referral to urology and the immediate insertion of a ureteral stent past the obstruction with a cystoscope. If the pressure on the kidney is not relieved expediently kidney damage can result.
    SWL is not the correct answer because it is not guaranteed to work to relieve the obstruction and cannot be coordinated quickly enough to be of use. SWL can however be used once the stent has been placed to fragment the stone.
    PCNL does not have an application here because it is only useful in large renal stones. Also it cannot be coordinated in time to be of use.
    Medical expulsive therapy is inappropriate because there is a currently great risk to the kidney. It may be true that expulsive therapy with alpha blockers and CCB will relax the ureter enough for the stone to pass but this is not a given and cannot be relied upon. Stenting is the quickest and most effective treatment.

    Etiology:
    Renal calculi develop when normally soluble components of urine such as calcium, uric acid, oxalate, and sodium become supersaturated and fall out of suspension 1. When this happens they precipitate within the urinary tract and over time collect more solute. If they grow large enough they can irritate tissues and even block the flow of urine.
    Epidemiology: Renal stones are more prevalent in men than women 3:1. More prevalent in whites than others. More prevalent in hot and dry geographic locations. Most common in the 4th to 6th decades of life but rates are rising in younger populations. There is a positive correlation between renal calculi and BMI 1.
    Pathophysiology: It appears that the pathophysiology of renal calculi is still not well understood. Certainly the precipitation of supersaturated solutes in the urine plays a role but it is not undisputed which part of the kidney this takes place in. Some studies have shown that plaques can form within the loop of Henle 3. These plaques may provide a collecting point for solutes to form on before they travel distally and collect in the calyces, renal pelvis, ureters, and bladder.

    History & physical exam: The common presentation of acute renal colic is one of a patient waking from sleep with excruciating flank pain. Depending on the location of the stone the pain has different radiculopathy patterns. If the stone is in the kidney, most of the time it is asymptomatic unless the flow of urine has been obstructed. If obstruction is present the pain will be in the deep flank without radiation to the groin. There may be some associated nausea and vomiting because the kidney shares nerve innervation with the stomach and intestines. If the stone is in the upper ureter it will cause severe colicky flank pain that often radiates anterior and caudally 2. This pain may be accompanied by nausea. If the stone is impacted at the UO a patient may experience radiation to the groin or testicle/labia majora. This may be accompanied by urinary frequency and dysuria. Once the stone has passed into the bladder it is most times passed without incident.
    On physical exam a patient will be in severe pain and writhing in contrast to a patient with peritonitis who will stay very still. Most often there will be the presence of microhematuria on UA. They may have migrating flank pain with radiation to groin. Hypoactive bowel sounds may be present. Fever is absent unless they have developed an infection secondary to obstruction.
    Diagnostics:Non contrast CT is most widely used and sensitive diagnostic imaging test. Some will order a KUB if the patient is a known stone factory and their stones can be seen on KUB. Ultrasound can be used for children or pregnant women when radiation exposure is of greater concern. IVP is declining in use but is a useful exam that demonstrates the urinary tract anatomy and function. It requires IV contrast which carries a risk of allergy. A urinalysis will show microhematuria 85% of the time 2. It can show the offending crystals which is a big clue. Leukocytes, nitrites, and pH can show infection. CMP will show renal function and electrolyte abnormalities which may explain the propensity to form stones. A 24 hour urine collection will show if there are elevated crystals or other abnormalities in the urine. CBC may show elevated WBC count if infection present.

    Management and treatment: The first line managements for renal or ureteral colic are supportive measures. These may include fluids, pain relief with morphine or ketorilac, and anti emetics. Medical expulsive therapy which consists of alpha blockers and CCBs is used for non obstructing stones smaller than 1cm 1. Most stones pass within 4-6 weeks and if they don’t will need to be operated on if symptomatic. Obstruction is an indication for cystoscopy and stent insertion. Infection will need to be managed with IV antibiotics and relief of obstruction. If the stone is larger than 1cm it is unlikely that it will pass on its own. In such scenarios extracorporeal shockwave lithotripsy, ureteroscopy, or percutaneous nephrolithotomy can be performed. The benefits of ESWL are that general anesthesia is not requited, it is the least invasive option, and complication rates are low (but not unheard of). Unfortunately ESWL success rates are lower than other interventions. Ureteroscopy requires general anesthesia and entering the urinary system. It’s success rates are higher for most locations of stones except for proximal ureter and renal stones larger than 2cm. For large renal stones PCNL is most effective. It involves general anesthesia and entering the kidney percutaneously through the flank. Risks of complication are higher with this procedure but is the last resort for many large renal stones.
    Depending on the chemistry of the patient’s urine there are certain alkalization agents and binders that can be prescribed to keep the offending salt from forming stones. Uric acid stones can be dissolved by potassium citrate and sodium bicarbonate which lowers the urine pH. Goal pH is 6.5-7 1. Allopurinol can be used for recurrent calcium oxalate stones in patients with a normal urine calcium. Recurrent calcium stones with hypercalciuria can be treated with HCTZ or potassium supplement. Intractable struvite stones can be treated with a urease inhibitor. In addition to these medications 2L of urine output should be maintained along with a low sodium, low protein, low oxalate diet.

    Sources
    1. Antonelli J. Nephrolithiasis. Point of Care Medical Applications | Epocrates. http://www.epocrates.com/. Published April 6, 2017. Accessed October 30, 2017.
    2. Chirag D. Nephrolithiasis. Medscape. http://www.medscape.com. Published December 3, 2016. Accessed October 30, 2017.
    3. McCance KL, Huether SE. Pathophysiology: the biologic basis for disease in adults and children. St. Louis, MO: Elsevier; 2014.

  204. Question 204 of 220
    204. Question

    A 45-year-old male with long standing history working as a painter presents to clinic complaining of right shoulder pain that increases with overhead activity. Patient denies any recent injury or trauma to his shoulder. On exam he has tenderness to palpation directly on the AC joint and lateral shoulder. Full passive ROM is noted. Active flexion, abduction, and internal rotation are limited due to pain. Hawkin’s, Neer’s, and crossbody test are positive. Patient denies fever/chills, erythema and/or edema of the shoulder, or muscle weakness. Which of the following is the most likely diagnosis?

    Correct

    Answer is C. Shoulder impingement syndrome is a common diagnosis in the orthopedic and primary care setting. The patient population that is most at risk for developing shoulder impingement syndrome is those who reach at the level or above the level of their shoulder for either work or sports on a constant basis (1). Increasing age can also cause osteoarthritic changes and therefore can be another cause of shoulder impingement syndrome. Shoulder impingement syndrome, otherwise referred to as SIS, is not due to a direct injury itself but a compilation of shoulder symptoms (1). SIS is the compression of the anatomical structures within the shoulder joint (1). The shoulder joint is comprised of the rotator cuff muscles (supraspinatus, infraspinatus, teres minor, subscapularis), the subacromial bursa, labrum, and long head of the biceps (1). These structures are located between the humeral head and under the acromion. SIS can occur from a high riding humeral head due to a rotator cuff tear and/or humeral instability, the make up of the acromion, for example numerous osteophytes and/or AC joint arthritis (1). Lastly, SIS can be caused by reduced space between the acromion and humeral head (1). The acromion plays a large role in the development of impingement syndrome. When reviewing a patient’s x-rays it is important to look at the morphology of the acromion. The acromion can be separated into 3 types; Type 1 (Flat), Type 2 (Curved), and Type 3 (Hooked) (1). The increase in type is associated with a decrease in space between the acromion and humeral head (1). This correlates an increase in severity of impingement and increased potential for a rotator cuff tear.
    A patient with shoulder impingement will present in clinic typically with a chief complaint of shoulder pain that is worse with overhead activity. The patient may point directly to the AC joint and/or subacromial space as the point of greatest discomfort and they may report radiation to their deltoid or lateral arm (1). During physical exam, patients typically have pain with range of motion. A patient may express discomfort at 30-40 degrees of abduction and at approximately 90 degrees of flexion (1,2). The patient will have minimal difficulty and pain with external rotation however they are characteristically unable to internally rotate due to pain (2). The patient will not have a decrease in strength. Special tests should be performed during exam. A patient with shoulder impingement syndrome will typically have a positive Hawkins test and Neer test which indicates supraspinatus impingement (2). A positive crossbody test indicates AC joint impingement.
    Once the diagnosis of shoulder impingement is made either clinically and/ or with x-rays, the management is typically conservative. Further testing, such as a MRI is not clinically indicated unless there is a question of a rotator cuff tear (3). Initial treatment includes rest, ice, NSAIDs, pendulum exercises, and strengthening of the rotator cuff with physical therapy (1,2). If initial management fails it is recommended to try a cortisone injection in the AC joint and/or subacromial space (1, 2). Lastly, if a patient fails conservative management it is recommended to undergo surgical treatment, which is subacromial decompression with or without distal clavicle resection.
    Patient’s presenting with a rotator cuff tear will have a decrease in strength in the rotator cuff muscles up on exam and severe tears will have a positive drop arm test (2). The Hawkins, Neer, and Crossbody test may be positive or negative, however the key difference is muscle strength when forming a differential diagnosis between the two. Adhesive capsulitis, also known as frozen shoulder, can be ruled out due this patient’s ability to actively and passively move their shoulder. Patients with adhesive capsulitis will deny pain at rest however they have significant pain with range of motion (2). Due to inflammation and scar tissue adhesions the patient’s range of motion will be limited in all planes (2). A patient with biceps tendinois will also complain of pain with overhead motion however when asked to point to location of pain they will point anterior to the humerus in the intertubercular groove (2). The patient has a short head and long head of the biceps, which when inflamed will cause pain (2). Upon exam patients will have tenderness to palpation of that area and will have pain with resisted supination (2). Biceps tendinosis exam results in a positive Speed’s test and Yergason’s test.

    References:

    1. Simons SM, Kruse D, Dixon JB. Shoulder Impingement Syndrome. UpToDate. March 2016. https://www.uptodate.com. Accessed July 5, 2017.
    2. Rempel DM, Amirtharajah M, Descatha A. Rempel D.M., Amirtharajah M, Descatha A Rempel, David M., et al.Shoulder, Elbow, & Hand Injuries. In: LaDou J, Harrison RJ. LaDou J, Harrison R.J. Eds. Joseph LaDou, and Robert J. Harrison.eds. CURRENT Diagnosis & Treatment: Occupational & Environmental Medicine, 5e New York, NY: McGraw-Hill; 2013.http://accessmedicine.mhmedical.com/content.aspx?bookid=1186&sectionid=6647855. Accessed July 06, 2017.
    3. Watts AR, Williams B, Kim SW, Bramwell DC, Krishnan J. Shoulder impingement syndrome: a systematic review of clinical trial participant selection criteria. Shoulder & Elbow. 2016;9(1):31-41. doi:10.1177/1758573216663201.

    Incorrect

    Answer is C. Shoulder impingement syndrome is a common diagnosis in the orthopedic and primary care setting. The patient population that is most at risk for developing shoulder impingement syndrome is those who reach at the level or above the level of their shoulder for either work or sports on a constant basis (1). Increasing age can also cause osteoarthritic changes and therefore can be another cause of shoulder impingement syndrome. Shoulder impingement syndrome, otherwise referred to as SIS, is not due to a direct injury itself but a compilation of shoulder symptoms (1). SIS is the compression of the anatomical structures within the shoulder joint (1). The shoulder joint is comprised of the rotator cuff muscles (supraspinatus, infraspinatus, teres minor, subscapularis), the subacromial bursa, labrum, and long head of the biceps (1). These structures are located between the humeral head and under the acromion. SIS can occur from a high riding humeral head due to a rotator cuff tear and/or humeral instability, the make up of the acromion, for example numerous osteophytes and/or AC joint arthritis (1). Lastly, SIS can be caused by reduced space between the acromion and humeral head (1). The acromion plays a large role in the development of impingement syndrome. When reviewing a patient’s x-rays it is important to look at the morphology of the acromion. The acromion can be separated into 3 types; Type 1 (Flat), Type 2 (Curved), and Type 3 (Hooked) (1). The increase in type is associated with a decrease in space between the acromion and humeral head (1). This correlates an increase in severity of impingement and increased potential for a rotator cuff tear.
    A patient with shoulder impingement will present in clinic typically with a chief complaint of shoulder pain that is worse with overhead activity. The patient may point directly to the AC joint and/or subacromial space as the point of greatest discomfort and they may report radiation to their deltoid or lateral arm (1). During physical exam, patients typically have pain with range of motion. A patient may express discomfort at 30-40 degrees of abduction and at approximately 90 degrees of flexion (1,2). The patient will have minimal difficulty and pain with external rotation however they are characteristically unable to internally rotate due to pain (2). The patient will not have a decrease in strength. Special tests should be performed during exam. A patient with shoulder impingement syndrome will typically have a positive Hawkins test and Neer test which indicates supraspinatus impingement (2). A positive crossbody test indicates AC joint impingement.
    Once the diagnosis of shoulder impingement is made either clinically and/ or with x-rays, the management is typically conservative. Further testing, such as a MRI is not clinically indicated unless there is a question of a rotator cuff tear (3). Initial treatment includes rest, ice, NSAIDs, pendulum exercises, and strengthening of the rotator cuff with physical therapy (1,2). If initial management fails it is recommended to try a cortisone injection in the AC joint and/or subacromial space (1, 2). Lastly, if a patient fails conservative management it is recommended to undergo surgical treatment, which is subacromial decompression with or without distal clavicle resection.
    Patient’s presenting with a rotator cuff tear will have a decrease in strength in the rotator cuff muscles up on exam and severe tears will have a positive drop arm test (2). The Hawkins, Neer, and Crossbody test may be positive or negative, however the key difference is muscle strength when forming a differential diagnosis between the two. Adhesive capsulitis, also known as frozen shoulder, can be ruled out due this patient’s ability to actively and passively move their shoulder. Patients with adhesive capsulitis will deny pain at rest however they have significant pain with range of motion (2). Due to inflammation and scar tissue adhesions the patient’s range of motion will be limited in all planes (2). A patient with biceps tendinois will also complain of pain with overhead motion however when asked to point to location of pain they will point anterior to the humerus in the intertubercular groove (2). The patient has a short head and long head of the biceps, which when inflamed will cause pain (2). Upon exam patients will have tenderness to palpation of that area and will have pain with resisted supination (2). Biceps tendinosis exam results in a positive Speed’s test and Yergason’s test.

    References:

    1. Simons SM, Kruse D, Dixon JB. Shoulder Impingement Syndrome. UpToDate. March 2016. https://www.uptodate.com. Accessed July 5, 2017.
    2. Rempel DM, Amirtharajah M, Descatha A. Rempel D.M., Amirtharajah M, Descatha A Rempel, David M., et al.Shoulder, Elbow, & Hand Injuries. In: LaDou J, Harrison RJ. LaDou J, Harrison R.J. Eds. Joseph LaDou, and Robert J. Harrison.eds. CURRENT Diagnosis & Treatment: Occupational & Environmental Medicine, 5e New York, NY: McGraw-Hill; 2013.http://accessmedicine.mhmedical.com/content.aspx?bookid=1186&sectionid=6647855. Accessed July 06, 2017.
    3. Watts AR, Williams B, Kim SW, Bramwell DC, Krishnan J. Shoulder impingement syndrome: a systematic review of clinical trial participant selection criteria. Shoulder & Elbow. 2016;9(1):31-41. doi:10.1177/1758573216663201.

  205. Question 205 of 220
    205. Question

    An 82-year-old Caucasian male presents to his primary care for his annual exam. He has no concerns today; he lives with his wife of 50 years, states he is very healthy, eats well, exercises daily and loves to play golf at least 4 days a week. His vitals and labs are all within normal limits. On physical exam, you notice a 1 cm nodular, indurated, scaly lesion with central ulceration on his scalp with mild erythema surrounding it. When asked, the patient says, “Oh that’s been there for almost a year, it doesn’t bother me.” What is your most likely diagnosis based on his history and exam findings?

    Correct

    Answer: D: Squamous Cell Carcinoma

    Discussion:

    The diagnosis is invasive squamous cell carcinoma. The prevalence of skin cancers, most commonly non-melanoma skin cancers, is continuously increasing. Skin cancer is the most common form of cancer, with more people diagnosed in the U.S. each year than all other types of cancer combined.1 Following basal cell carcinoma, squamous cell carcinoma (SCC) is the second most common type of skin cancer, with over 1 million cases diagnosed each year in the U.S1. Males are two times more likely as females to develop SCC and it is far more common in patients over 60 years old.2 SCC is most commonly caused by prior long term sun damage from UV radiation or damage to the skin from chemical exposure, ulcers, or burns.2 Chronic exposure to the sun’s UVA or UVB radiation leads to skin damage slowly over time. Epidermal keratinocytes in the skin begin to transform into neoplastic cells due to this damage, eventually becoming malignant, entering the dermal layer of the skin and becoming a SCC.2 This damage can also occur from previous chemical or burn damage to the skin. Often the beginning stage of this skin transformation presents as an actinic keratosis (sun damaged skin) before becoming SCC. Patients want to be looking for precancerous AK lesions; small, rough, or scaly spots on their skin. A mutation in the tumor suppressor gene P53, caused by UV exposure, can be found in close to 90% of patients with SCC.1
    The most common presentation of squamous cell carcinoma’s as well as many other skin cancers, which differentiates them from benign lesions, is a wound or lesion that will not heal. SCC’s can occur anywhere on the body but are most commonly found in areas of frequent sun-exposure such as the head and neck in males, and arms, head and neck in females.2 The lesion will most often be indurated, erythematous and scaly, often bleeding easily.3 If the patient presents late in the development of SCC, the lesion may have central ulceration or clearing. Risk factors include fair skin that easily burns/doesn’t tan, light colored hair/red hair, light colored eyes, and family history of skin cancer. Also, their history will often include exposure to UV radiation including outdoor sun exposure, indoor tanning beds, radiation therapy, phototherapy booth use or history of skin damage due to chemical exposure, chronic ulcerations, or burns. It is important to ask about medications such as immunosuppressive therapy and those which increase photosensitivity, also other health conditions, and symptoms that could indicate metastasis.
    Based on the physical exam findings, location of the lesion, and history of the patient you can easily include squamous cell carcinoma in your ddx but there are many other lesions both benign and malignant that resemble SCC. The only way to confirm the diagnosis is by histopathology of the excised lesion via skin biopsy. If cancerous cells are found, further treatment is needed. If the lesion is small it can often be fully excised via shave biopsy, excisional surgery, or ED&C. If the lesion is larger, a small sample of the lesion with either a shave or punch biopsy for initial histopathology assessment is preferred. Once the diagnosis is confirmed, the patient is often referred to a Mohs surgeon to ensure full excision of all the cancer cells surrounding the lesion by viewing the tissue under a microscope.
    If found early and removed, squamous cell carcinoma’s have a good prognostic outcome. The faster they are treated, the less likely they are to spread. If they go untreated there is a risk of the SCC metastasizing to other tissues, lymph nodes, and even organs which the result can be fatal.1 If sun induced SCC, the rate of metastasis is around 3-7%.4 It is always important to educate patients on the importance of regularly checking their skin for new or changing lesions as well as the importance of sunscreen and protection from the sun. Using SPF 15 or higher sunscreen regularly decreases your risk of developing SCC by 40%.1

    The other differential diagnoses were fully ruled out based on the pathology report from the initial shave biopsy. Based on appearance, basal cell carcinomas typically are more erythematous, shiny, scar like, or an open sore. This patient’s lesion was crusty, indurated, and keratinized; slightly different appearing. Also, common for squamous cell, are patients who were chronically out in the sun versus for basal cell it’s those who received small extreme bursts of sun. Seborrheic keratosis present as waxy, stuck-on appearing lesions, tan-brown in color. They are more plaque like than nodular like this patient’s lesion. This lesion was not likely to be benign because it had been present for over a year with no sign of healing, there is no specific reason for it to still be there, and it bleeds easily. All signs of atypical or cancerous cell presentation.

    References:

    1.Skin Cancer Foundation. Skin Cancer Facts & Statistics – SkinCancer.org https://www.skincancer.org/skin-cancer-information/skin-cancer-facts. Updated January 31, 2018. Accessed March 10, 2018.
    2.DynaMed Plus. Cutaneous squamous cell carcinoma. http://www.dynamed.com/ topics/dmp~AN~T116909/Cutaneous-squamous-cell-carcinoma#References. Published 1995. Updated October 23, 2017. Accessed March 10, 2018.
    3.Squamous Cell Carcinoma. Dermatology Glossary: define your skin. http://missinglink.ucsf.edu/lm/DermatologyGlossary/squamous_cell_carcinoma.html
    Accessed March 10, 2018.
    4.McPhee Sj, Papadakis MA, Rabow MW. Current medical diagnosis & treatment 2017. New York: McGraw Hill Education; 2017.

    Incorrect

    Answer: D: Squamous Cell Carcinoma

    Discussion:

    The diagnosis is invasive squamous cell carcinoma. The prevalence of skin cancers, most commonly non-melanoma skin cancers, is continuously increasing. Skin cancer is the most common form of cancer, with more people diagnosed in the U.S. each year than all other types of cancer combined.1 Following basal cell carcinoma, squamous cell carcinoma (SCC) is the second most common type of skin cancer, with over 1 million cases diagnosed each year in the U.S1. Males are two times more likely as females to develop SCC and it is far more common in patients over 60 years old.2 SCC is most commonly caused by prior long term sun damage from UV radiation or damage to the skin from chemical exposure, ulcers, or burns.2 Chronic exposure to the sun’s UVA or UVB radiation leads to skin damage slowly over time. Epidermal keratinocytes in the skin begin to transform into neoplastic cells due to this damage, eventually becoming malignant, entering the dermal layer of the skin and becoming a SCC.2 This damage can also occur from previous chemical or burn damage to the skin. Often the beginning stage of this skin transformation presents as an actinic keratosis (sun damaged skin) before becoming SCC. Patients want to be looking for precancerous AK lesions; small, rough, or scaly spots on their skin. A mutation in the tumor suppressor gene P53, caused by UV exposure, can be found in close to 90% of patients with SCC.1
    The most common presentation of squamous cell carcinoma’s as well as many other skin cancers, which differentiates them from benign lesions, is a wound or lesion that will not heal. SCC’s can occur anywhere on the body but are most commonly found in areas of frequent sun-exposure such as the head and neck in males, and arms, head and neck in females.2 The lesion will most often be indurated, erythematous and scaly, often bleeding easily.3 If the patient presents late in the development of SCC, the lesion may have central ulceration or clearing. Risk factors include fair skin that easily burns/doesn’t tan, light colored hair/red hair, light colored eyes, and family history of skin cancer. Also, their history will often include exposure to UV radiation including outdoor sun exposure, indoor tanning beds, radiation therapy, phototherapy booth use or history of skin damage due to chemical exposure, chronic ulcerations, or burns. It is important to ask about medications such as immunosuppressive therapy and those which increase photosensitivity, also other health conditions, and symptoms that could indicate metastasis.
    Based on the physical exam findings, location of the lesion, and history of the patient you can easily include squamous cell carcinoma in your ddx but there are many other lesions both benign and malignant that resemble SCC. The only way to confirm the diagnosis is by histopathology of the excised lesion via skin biopsy. If cancerous cells are found, further treatment is needed. If the lesion is small it can often be fully excised via shave biopsy, excisional surgery, or ED&C. If the lesion is larger, a small sample of the lesion with either a shave or punch biopsy for initial histopathology assessment is preferred. Once the diagnosis is confirmed, the patient is often referred to a Mohs surgeon to ensure full excision of all the cancer cells surrounding the lesion by viewing the tissue under a microscope.
    If found early and removed, squamous cell carcinoma’s have a good prognostic outcome. The faster they are treated, the less likely they are to spread. If they go untreated there is a risk of the SCC metastasizing to other tissues, lymph nodes, and even organs which the result can be fatal.1 If sun induced SCC, the rate of metastasis is around 3-7%.4 It is always important to educate patients on the importance of regularly checking their skin for new or changing lesions as well as the importance of sunscreen and protection from the sun. Using SPF 15 or higher sunscreen regularly decreases your risk of developing SCC by 40%.1

    The other differential diagnoses were fully ruled out based on the pathology report from the initial shave biopsy. Based on appearance, basal cell carcinomas typically are more erythematous, shiny, scar like, or an open sore. This patient’s lesion was crusty, indurated, and keratinized; slightly different appearing. Also, common for squamous cell, are patients who were chronically out in the sun versus for basal cell it’s those who received small extreme bursts of sun. Seborrheic keratosis present as waxy, stuck-on appearing lesions, tan-brown in color. They are more plaque like than nodular like this patient’s lesion. This lesion was not likely to be benign because it had been present for over a year with no sign of healing, there is no specific reason for it to still be there, and it bleeds easily. All signs of atypical or cancerous cell presentation.

    References:

    1.Skin Cancer Foundation. Skin Cancer Facts & Statistics – SkinCancer.org https://www.skincancer.org/skin-cancer-information/skin-cancer-facts. Updated January 31, 2018. Accessed March 10, 2018.
    2.DynaMed Plus. Cutaneous squamous cell carcinoma. http://www.dynamed.com/ topics/dmp~AN~T116909/Cutaneous-squamous-cell-carcinoma#References. Published 1995. Updated October 23, 2017. Accessed March 10, 2018.
    3.Squamous Cell Carcinoma. Dermatology Glossary: define your skin. http://missinglink.ucsf.edu/lm/DermatologyGlossary/squamous_cell_carcinoma.html
    Accessed March 10, 2018.
    4.McPhee Sj, Papadakis MA, Rabow MW. Current medical diagnosis & treatment 2017. New York: McGraw Hill Education; 2017.

  206. Question 206 of 220
    206. Question

    A 74-year-old Caucasian male with a history of hypertension, hyperlipidemia, atrial fibrillation and severe chronic anemia presents with multiple complaints of fatigue, dyspnea and weakness. Patient states that his symptoms have worsened over the past two weeks. Patient denies any signs of hematochezia or hematuria. CBC reveals a Hgb of 7.6 g/dL, HCT of 24.1% and a MCV of 94 fL. COOMBS test is positive. A blood smear shows spherocytes along with IgG antibodies. Guaiac was positive. The patient is currently taking Brilinta, Methyldopa, Atorvastatin and Aspirin. What is the most likely diagnosis?

    Correct

    Correct Answer: (C)
    The patient’s complaints of weakness, fatigue and dyspnea are common symptoms of chronic anemia. The patient’s blood work revealed spherocytes, IgG antibodies and a low H&H. Most importantly, the COOMBS test came back positive, which indicates that it is an acquired/autoimmune type of anemia. The antibodies (IgG) tells you that it is a warm autoimmune hemolytic anemia because the IgG antibodies attack the red blood cells at normal body temperature. In fact, Methyldopa may be cause of the patient’s autoimmune hemolytic anemia. Although it is rare, Methyldopa has been shown to cause warm autoimmune hemolytic anemia, which would cause these same symptoms and lab results.

    Incorrect Answers:
    (A) Patient’s with autoimmune hemolytic anemia – cold antibodies would present with similar symptoms but would also complain of painful fingers and toes with purplish discoloration, particularly with cold exposures. Lastly, it is not the correct answer because autoimmune hemolytic anemia – cold antibodies is associated with IgM antibodies, not IgG. (B) Hemolytic anemia – hereditary spherocytosis also presents with similar symptoms and also shows spherocytes on a peripheral smear. Although, since the COOMBS test came back positive, Hemolytic anemia – hereditary spherocytosis is then ruled out since it would be negative in someone with hereditary spherocytosis. (D) Glucose-6-phosphate dehydrogenase (G6PD) also presents with similar symptoms but is ruled out since G6PD would present with a negative COOMBS test and would show bite cells and heinz bodies on a peripheral smear.

    Discussion:
    Epidemiology:
    Demand ischemia is known to peak in incidence at 60-years-old in men and 70-years-old in women1. Men are more commonly affected than women up until the ninth decade of life1. Cardiovascular disease is a significant health issue around the world, primarily in the westernized countries and is the major cause of death each year; accounting for one of every two deaths5. Several of these patients will also be anemic, which may be due to several different reasons5. According to the World Health Organization, anemia is defined as a hemoglobin <13 g/dL in men and <12 g/dL in women3.

    Etiology:
    There are three main causes of demand ischemia, which include increased demand in the presence of hypertension or tachycardia, diminished oxygen-carrying capacity (anemia or carbon dioxide poisoning) and a reduction in coronary blood flow caused by atherosclerotic disease2. In the presence of diminished oxygen-carrying capacity, as in the patient case above, chronic anemia shows a drop in circulating erythrocyte mass along with hemoglobin and hematocrit levels. This causes a decrease the amount of oxygen that is bound, transported and delivered to tissues3. Such changes can cause coronary artery disease, which may lead to myocardial damage3.

    Pathophysiology:
    Myocardial oxygen requirements increase heart rate, contractility, and/or LV wall stress. The oxygen supply to the myocardium, is determined by coronary artery flow and oxygen extraction6. The pathophysiology behind the presence of anemia, which may lead to adverse cardiovascular consequences include increased preload, increased cardiac output, reduced afterload and in turn, may lead to increased cardiac output4. As hemoglobin concentration and oxygen content decreases, an increase in heart rate helps to preserve total oxygen delivery5. However, this increases the amount of workload on the heart, which requires an increase in oxygen demand. But in someone with anemia, the oxygen demand cannot be met due to the diminished oxygen-carrying capacity5. This later causes a change in oxygen demand. When local oxygen requirements increase, coronary artery blood flow can increase to either hold more oxygen delivery consistent as arterial content decreases or when increased oxygen requirements are met5. In those who suffer chronically, the left ventricle may hypertrophy, which is a known risk factor for cardiovascular disease outcomes and mortality4. Lastly, in the presence of anemia, cardiac ischemia may be exacerbated as a result of either increased oxygen demand or decreased supply3.

    History and exam findings:
    Patients that have demand ischemia with anemia will complain of shortness of breath, chest, jaw, shoulder and/or back pain, fatigue, palpitations, headache, vision changes and chills7. Upon exam, a provider will notice abnormal vitals such as tachycardia, tachypnea, hypothermia and hypotension along with hepato/splenomegaly, cold extremities, pallor skin, fissures at the corners of the mouth, spooning of fingernails, accessory muscle use and scleral icterus7.

    Labs and imaging:
    Labs to help diagnose demand ischemia triggered by anemia would include CBC with differential, BNP, LFT’s, chemistry profile, LDH, troponin, lipid profile, coagulation, guaiac test and possibly COOMBS test. These labs would help determine the if troponin levels are increased, which would indicate myocardial damage. CBC with differential is used to determine the number of blood cells such as RBC’s, reticulocyte count, hemoglobin, hematocrit, MCV, MCH, platelet count and WBC’s. This information helps determine the size and shape of the RBC’s, presence of the type of anemia, possible clotting disorders and infections. Elevated BNP helps diagnose heart failure and an elevated troponin and other cardiac biomarkers help indicate a myocardial infarction. Chemistry profiles are also very important to follow in the setting of ischemia/MI, especially potassium, magnesium and creatinine levels prior to initiating specific medications such as ACEi9.

    Imaging/diagnostic modalities for demand myocardial ischemia include electrocardiogram (EKG), which records the electrical activity of the heart. And EKG can display signs of ischemia, heart blocks, irregular rhythms, atrial and ventricle enlargement7. An echocardiogram uses sound waves to help identify the functionality of the myocardium, wall thickness and wall motion7. A nuclear scan uses radioactive material that is injected into the patient’s bloodstream, which evaluates blood flow to the myocardium during exercise and post-exercise7. A nuclear scan helps identify areas of ischemia. A coronary angiography uses dye that is injected into the vessels of the heart. X-ray machine is used to take a series of images, which provides an inside image of the coronary blood vessels7. A cardiac CT scan can be sued to identify the presence of significant coronary artery calcification, which would indicate atherosclerosis7. Lastly, a stress test can be used to monitor the patient’s blood pressure, oxygen saturation/breathing, heart rhythm and rate while walking on a treadmill7. Throughout the stress test, the speed and incline of the treadmill gradually increases, causing the patient’s heart to work harder and faster. During this time, heart problems may become noticeable. There are currently not any imaging tests for anemia.

    Treatment/management:
    Management of demand ischemia triggered by anemia can be treated by correcting the balance between the myocardial supply and demand by increasing coronary blood flow, reducing myocardial oxygen requirements or both6. The choice of therapy and its effectiveness depend on the underlying cause of anemia. For those with anemia, correcting the underlying type of anemia many be curative by administering iron, vitamin B12 or folic acid8. In patients with chronic anemia, a blood transfusion(s) or injections of synthetic erythropoietin can help stimulate RBC production. A bone marrow stem cell transplant may also be an opinion for patients diagnosed with aplastic anemia or bone marrow disease8. Lastly there are medications that help induce and treat anemia, so it is important to review patient’s medications.
    As mentioned above, the physiologic role of oxygen delivery to the myocardium, anemia may cause severe cardiovascular diseases10. Medications such as aspirin, nitrates, beta blockers, calcium channel blockers, ACE inhibitors and cholesterol-lowering medications are used to help reduce heart rate, blood pressure, cardiac contractility, relax arterial and coronary vessels (vasodilation), increase blood flow to the heart and thin the blood7. In some cases, more aggressive treatment may be required. Procedures such as angioplasty with stenting and/or coronary artery bypass may be needed to improve blood flow10. For these types of patient’s, it is important to encourage and educate these patient’s on following a healthy life-style by quit smoking, managing underlying health conditions such as anemia, eating healthy, exercise regularly, maintaining a healthy weight and to manage stress by deep breathing or practicing yoga7.

    References:

    1. Sanchis-Gomar, F., Perez-Quilis, C., Leischik., R and Lucia, A. “Epidemiology of coronary heart disease and acute coronary syndrome”. Ann Transl Med. 2016 July; 4(13): 256. Accessed March 9, 2018 from https://www.ncbi.nlm.nih.gov/pmc/articles/PMC4958723/
    2. “Anemia”. Emedicine, 2017, November 2009. Accessed March 9, 2018 from https://emedicine.medscape.com/article/198475-overview
    3. Vlagopoulos, P., Tighiouart, H., Weiner, D., Griffith, J., Pettitt, D., Salem, D., Levey, A and Sarnak, M. “Anemia as a Risk Factor and All-Cause Mortality in Diabetes: The Impact of Chronic Kidney Disease”. Journal of the American Society of Nephrology. September 14, 2005. Web. Accessed March 9, 2018 from http://jasn.asnjournals.org/content/16/11/3403.full
    4. Bellotto, F. and Cati, A. “Anermia and myocardial ischemia: relationship and interferences”. Recenti Prog Med. 2006 March; 97 (3): 153-64. Accessed March 9, 2018 from https://www.ncbi.nlm.nih.gov/pubmed/16700423
    5. “Anemia and Cardiovascular Disease”. Network for the Advancement of Patient Blood Management, Hemostasis and Thrombosis. 2018. Web. Accessed March 9, 2018 from http://www.nataonline.com/np/440/anemia-and-cardiovascular-disease
    6. Cohn, P. F. “Treatment of chronic myocardial ischemia: rationale and treatment options”. Cardiovasc Drugs Ther. 1998 Oct; 12 Supple 3:217-23. Accessed March 10, 2018 from https://www.ncbi.nlm.nih.gov/pubmed/9800050
    7. “Myocardial Ischemia”. Mayo Clinic, Mayo Foundation for Medical Education and Research, 9 March 2018. Accessed March 9, 2018 from https://www.mayoclinic.org/diseases-conditions/myocardial-ischemia/symptoms-causes/syc-20375417
    8. “Anemia”. Mayo Clinic, Mayo Foundation for Medical Education and Research, 14 March 2018. Accessed March 9, 2018 from https://www.mayoclinic.org/diseases-conditions/anemia/symptoms-causes/syc-20351360
    9. “Myocardial Infarction Workup: Approach Considerations, Electrocardiography, Cardiac Biomarkers”. Emedicine, 2018. Accessed March 8, 2018 from https://emedicine.medscape.com/article/155919-workup#c1
    10. Zeidman, A., Fradin, Z., Blecher, A., Oster, H. S., Avrahami, Y., and Mittelman, M. “Anemia as a risk factor for ischemia heart disease”. Isr Med Assoc J. 2004 Jan.; 6(1):16-8. Accessed March 9, 2018 from https://www.ncbi.nlm.nih.gov/pubmed/14740503

    Incorrect

    Correct Answer: (C)
    The patient’s complaints of weakness, fatigue and dyspnea are common symptoms of chronic anemia. The patient’s blood work revealed spherocytes, IgG antibodies and a low H&H. Most importantly, the COOMBS test came back positive, which indicates that it is an acquired/autoimmune type of anemia. The antibodies (IgG) tells you that it is a warm autoimmune hemolytic anemia because the IgG antibodies attack the red blood cells at normal body temperature. In fact, Methyldopa may be cause of the patient’s autoimmune hemolytic anemia. Although it is rare, Methyldopa has been shown to cause warm autoimmune hemolytic anemia, which would cause these same symptoms and lab results.

    Incorrect Answers:
    (A) Patient’s with autoimmune hemolytic anemia – cold antibodies would present with similar symptoms but would also complain of painful fingers and toes with purplish discoloration, particularly with cold exposures. Lastly, it is not the correct answer because autoimmune hemolytic anemia – cold antibodies is associated with IgM antibodies, not IgG. (B) Hemolytic anemia – hereditary spherocytosis also presents with similar symptoms and also shows spherocytes on a peripheral smear. Although, since the COOMBS test came back positive, Hemolytic anemia – hereditary spherocytosis is then ruled out since it would be negative in someone with hereditary spherocytosis. (D) Glucose-6-phosphate dehydrogenase (G6PD) also presents with similar symptoms but is ruled out since G6PD would present with a negative COOMBS test and would show bite cells and heinz bodies on a peripheral smear.

    Discussion:
    Epidemiology:
    Demand ischemia is known to peak in incidence at 60-years-old in men and 70-years-old in women1. Men are more commonly affected than women up until the ninth decade of life1. Cardiovascular disease is a significant health issue around the world, primarily in the westernized countries and is the major cause of death each year; accounting for one of every two deaths5. Several of these patients will also be anemic, which may be due to several different reasons5. According to the World Health Organization, anemia is defined as a hemoglobin <13 g/dL in men and <12 g/dL in women3.

    Etiology:
    There are three main causes of demand ischemia, which include increased demand in the presence of hypertension or tachycardia, diminished oxygen-carrying capacity (anemia or carbon dioxide poisoning) and a reduction in coronary blood flow caused by atherosclerotic disease2. In the presence of diminished oxygen-carrying capacity, as in the patient case above, chronic anemia shows a drop in circulating erythrocyte mass along with hemoglobin and hematocrit levels. This causes a decrease the amount of oxygen that is bound, transported and delivered to tissues3. Such changes can cause coronary artery disease, which may lead to myocardial damage3.

    Pathophysiology:
    Myocardial oxygen requirements increase heart rate, contractility, and/or LV wall stress. The oxygen supply to the myocardium, is determined by coronary artery flow and oxygen extraction6. The pathophysiology behind the presence of anemia, which may lead to adverse cardiovascular consequences include increased preload, increased cardiac output, reduced afterload and in turn, may lead to increased cardiac output4. As hemoglobin concentration and oxygen content decreases, an increase in heart rate helps to preserve total oxygen delivery5. However, this increases the amount of workload on the heart, which requires an increase in oxygen demand. But in someone with anemia, the oxygen demand cannot be met due to the diminished oxygen-carrying capacity5. This later causes a change in oxygen demand. When local oxygen requirements increase, coronary artery blood flow can increase to either hold more oxygen delivery consistent as arterial content decreases or when increased oxygen requirements are met5. In those who suffer chronically, the left ventricle may hypertrophy, which is a known risk factor for cardiovascular disease outcomes and mortality4. Lastly, in the presence of anemia, cardiac ischemia may be exacerbated as a result of either increased oxygen demand or decreased supply3.

    History and exam findings:
    Patients that have demand ischemia with anemia will complain of shortness of breath, chest, jaw, shoulder and/or back pain, fatigue, palpitations, headache, vision changes and chills7. Upon exam, a provider will notice abnormal vitals such as tachycardia, tachypnea, hypothermia and hypotension along with hepato/splenomegaly, cold extremities, pallor skin, fissures at the corners of the mouth, spooning of fingernails, accessory muscle use and scleral icterus7.

    Labs and imaging:
    Labs to help diagnose demand ischemia triggered by anemia would include CBC with differential, BNP, LFT’s, chemistry profile, LDH, troponin, lipid profile, coagulation, guaiac test and possibly COOMBS test. These labs would help determine the if troponin levels are increased, which would indicate myocardial damage. CBC with differential is used to determine the number of blood cells such as RBC’s, reticulocyte count, hemoglobin, hematocrit, MCV, MCH, platelet count and WBC’s. This information helps determine the size and shape of the RBC’s, presence of the type of anemia, possible clotting disorders and infections. Elevated BNP helps diagnose heart failure and an elevated troponin and other cardiac biomarkers help indicate a myocardial infarction. Chemistry profiles are also very important to follow in the setting of ischemia/MI, especially potassium, magnesium and creatinine levels prior to initiating specific medications such as ACEi9.

    Imaging/diagnostic modalities for demand myocardial ischemia include electrocardiogram (EKG), which records the electrical activity of the heart. And EKG can display signs of ischemia, heart blocks, irregular rhythms, atrial and ventricle enlargement7. An echocardiogram uses sound waves to help identify the functionality of the myocardium, wall thickness and wall motion7. A nuclear scan uses radioactive material that is injected into the patient’s bloodstream, which evaluates blood flow to the myocardium during exercise and post-exercise7. A nuclear scan helps identify areas of ischemia. A coronary angiography uses dye that is injected into the vessels of the heart. X-ray machine is used to take a series of images, which provides an inside image of the coronary blood vessels7. A cardiac CT scan can be sued to identify the presence of significant coronary artery calcification, which would indicate atherosclerosis7. Lastly, a stress test can be used to monitor the patient’s blood pressure, oxygen saturation/breathing, heart rhythm and rate while walking on a treadmill7. Throughout the stress test, the speed and incline of the treadmill gradually increases, causing the patient’s heart to work harder and faster. During this time, heart problems may become noticeable. There are currently not any imaging tests for anemia.

    Treatment/management:
    Management of demand ischemia triggered by anemia can be treated by correcting the balance between the myocardial supply and demand by increasing coronary blood flow, reducing myocardial oxygen requirements or both6. The choice of therapy and its effectiveness depend on the underlying cause of anemia. For those with anemia, correcting the underlying type of anemia many be curative by administering iron, vitamin B12 or folic acid8. In patients with chronic anemia, a blood transfusion(s) or injections of synthetic erythropoietin can help stimulate RBC production. A bone marrow stem cell transplant may also be an opinion for patients diagnosed with aplastic anemia or bone marrow disease8. Lastly there are medications that help induce and treat anemia, so it is important to review patient’s medications.
    As mentioned above, the physiologic role of oxygen delivery to the myocardium, anemia may cause severe cardiovascular diseases10. Medications such as aspirin, nitrates, beta blockers, calcium channel blockers, ACE inhibitors and cholesterol-lowering medications are used to help reduce heart rate, blood pressure, cardiac contractility, relax arterial and coronary vessels (vasodilation), increase blood flow to the heart and thin the blood7. In some cases, more aggressive treatment may be required. Procedures such as angioplasty with stenting and/or coronary artery bypass may be needed to improve blood flow10. For these types of patient’s, it is important to encourage and educate these patient’s on following a healthy life-style by quit smoking, managing underlying health conditions such as anemia, eating healthy, exercise regularly, maintaining a healthy weight and to manage stress by deep breathing or practicing yoga7.

    References:

    1. Sanchis-Gomar, F., Perez-Quilis, C., Leischik., R and Lucia, A. “Epidemiology of coronary heart disease and acute coronary syndrome”. Ann Transl Med. 2016 July; 4(13): 256. Accessed March 9, 2018 from https://www.ncbi.nlm.nih.gov/pmc/articles/PMC4958723/
    2. “Anemia”. Emedicine, 2017, November 2009. Accessed March 9, 2018 from https://emedicine.medscape.com/article/198475-overview
    3. Vlagopoulos, P., Tighiouart, H., Weiner, D., Griffith, J., Pettitt, D., Salem, D., Levey, A and Sarnak, M. “Anemia as a Risk Factor and All-Cause Mortality in Diabetes: The Impact of Chronic Kidney Disease”. Journal of the American Society of Nephrology. September 14, 2005. Web. Accessed March 9, 2018 from http://jasn.asnjournals.org/content/16/11/3403.full
    4. Bellotto, F. and Cati, A. “Anermia and myocardial ischemia: relationship and interferences”. Recenti Prog Med. 2006 March; 97 (3): 153-64. Accessed March 9, 2018 from https://www.ncbi.nlm.nih.gov/pubmed/16700423
    5. “Anemia and Cardiovascular Disease”. Network for the Advancement of Patient Blood Management, Hemostasis and Thrombosis. 2018. Web. Accessed March 9, 2018 from http://www.nataonline.com/np/440/anemia-and-cardiovascular-disease
    6. Cohn, P. F. “Treatment of chronic myocardial ischemia: rationale and treatment options”. Cardiovasc Drugs Ther. 1998 Oct; 12 Supple 3:217-23. Accessed March 10, 2018 from https://www.ncbi.nlm.nih.gov/pubmed/9800050
    7. “Myocardial Ischemia”. Mayo Clinic, Mayo Foundation for Medical Education and Research, 9 March 2018. Accessed March 9, 2018 from https://www.mayoclinic.org/diseases-conditions/myocardial-ischemia/symptoms-causes/syc-20375417
    8. “Anemia”. Mayo Clinic, Mayo Foundation for Medical Education and Research, 14 March 2018. Accessed March 9, 2018 from https://www.mayoclinic.org/diseases-conditions/anemia/symptoms-causes/syc-20351360
    9. “Myocardial Infarction Workup: Approach Considerations, Electrocardiography, Cardiac Biomarkers”. Emedicine, 2018. Accessed March 8, 2018 from https://emedicine.medscape.com/article/155919-workup#c1
    10. Zeidman, A., Fradin, Z., Blecher, A., Oster, H. S., Avrahami, Y., and Mittelman, M. “Anemia as a risk factor for ischemia heart disease”. Isr Med Assoc J. 2004 Jan.; 6(1):16-8. Accessed March 9, 2018 from https://www.ncbi.nlm.nih.gov/pubmed/14740503

  207. Question 207 of 220
    207. Question

    An obese 47 year old female presents to the ED with her two children and has diffuse upper abdominal pain. She mentions that her pain is radiating to her back. Since the pain started she has been unable to eat without getting nauseous and vomiting for the past two days. On physical examination she has epigastric tenderness and mild distention. What diagnostic study is initially indicated?

    Correct

    The correct answer is B due to the fact that this is the best initial study for a patient presenting with acute pancreatitis, as it allows the provider to assess for biliary stone obstruction as a potential source of the patient’s symptoms. This patient is especially susceptible to biliary tract disease since she is and obese, female, who is fertile and in her forties. Answer A, is very helpful in determining the extent of injury, but is not very useful in assessing for biliary stone obstruction which is likely the cause of this patient’s pancreatitis. Answer C endoscopic ultrasound is often used in sever pancreatitis with pseudocyst and abscess formation to rule out malignancy via biopsy. Answer D has little utility of assessing pancreatitis in the acute setting.

    Epidemiology
    Acute pancreatitis is a relatively uncommon with an incidence rate of around 40 cases per 100,000 adults in the United States. The age of onset varies amongst etiologies. The top four etiologies and their average age of onset are alcoholism 39 years, biliary tract disease 69 years, trauma 66 years and drug-induced 42 years. Acute pancreatitis affects males more often than woman. However, the etiology contributing to the increased incidence in men is alcoholism. While acute pancreatitis as a whole is less common in women they have a higher predilection for biliary tract pathophysiology then men. When it comes to race, African American have a 3 times higher risk of developing acute pancreatitis in comparison to their Caucasian counterparts.
    Pathophysiology
    Pancreatitis is classified as mild or severe. These classifications are based on the extent of injury to the pancreas. In mild pancreatitis there is inflammation and edema confined solely to the pancreas. In severe pancreatitis the extent of injury leads to necrosis of pancreatic tissue and subsequent damage to adjacent organs. The specific trigger to the inflammatory cascade in acute pancreatitis is largely unknown. However, there is a known association between a combination of extracellular neural and vascular responses and intracellular events like digestive enzyme activations, increased calcium signaling and heat shock proteins. The initial inflammatory response is primarily mediated TNF-alpha and IL-1 which are produced by pancreatic cells. These inflammatory markers draw in neutrophils to the pancreas. This primary inflammatory response is what leads to all the secondary systemic sequela. Once the inflammatory cascade is triggered the disease state begins with peripancreatic fat necrosis. This necrosis eventually begins to encompass parenchymal tissue. Once parenchymal tissue is involved there is hemorrhage and true damage to the gland itself. Often hemorrhagic necrotizing pancreatitis leads to walling off of digestive enzymes which present as pseudocyst and bacterial seeding progressing to abscess formation.

    History
    Acute pancreatitis usually present as a boring, gnawing type epigastric pain that radiates to the back. The location of the pain can lateralize left or right, but is generally reported as an upper quadrant pain. The pain usually starts are brief episodes of dull aching which over time become more frequent and intense. The pain is occasionally relieved when sitting up or bending forward. The evolution of the pain is what most often leads to patients presenting in clinic. Often nausea and vomiting are present along with anorexia. It is important to ask these patients if they have had any recent ERCP or non-surgical trauma, are they drinkers, do they have any previous biliary tract disease and what medications are they taking.

    Physical
    On physical exam tenderness to palpation and guarding will be noted in the epigastrium, right upper quadrant or left upper quadrant. The abdomen may be mildly distended with decreased bowel sounds. Patients with severe cases of pancreatitis may develop acute respiratory distress syndrome which presents with dyspnea or tachypnea. These patients may have rales present on auscultation primarily in the left lung fields. Hemorrhagic pancreatitis can lead to hematemesis or melena. These hemorrhagic patients will often present febrile, pale and diaphoretic. Occasionally seen on exam in hemorrhagic pancreatitis are Cullen sign (ecchymosis around the umbilicus) and Turner sign (ecchymosis in the left flank).

    Labs/Imaging
    When working up someone who presents with acute pancreatitis there are several labs that are crucial in both the diagnosis and prognosis. The first lab values of interest is these patients are amylase and lipase. When looking at the amylase and lipase it is crucial that these lab value are three times the upper limit of normal. Without an amylase or lipase of three times the upper limit of normal acute pancreatitis becomes significantly less likely as a diagnosis. Next labs of great importance after assessing the amylase and lipase is the patients liver function tests. The AST/ALT, bilirubin and alkaline phosphatase will help in guiding the provider in determining the pathophysiology behind this episode of acute pancreatitis. A patient with an increased liver function tests will likely have acute pancreatitis stemming from obstructive gallstones. When tissue necrotizes is begins to increase its uptake of calcium. This increase in calcium uptake can often lead to hypocalcemia, so it is important that calcium levels are checked. In addition to calcium, there may also be an increase in glucose due to decrease insulin production. Get a lipid panel as increased triglycerides will suggest the patient’s pancreatitis is stemming from alcohol abuse. Finally, a CBC w/ diff will often show and elevated white count and a left shift.
    The Imaging of choice in a patient with acute pancreatitis is going to be an abdominal CT. While abdominal is the diagnostic gold standard ultrasonography is often a very viable initial diagnostic study. Ultrasound is better at assessing the presence of biliary stone obstruction than CT. However, CT imaging allows radiologist to grade the extent of disease based on amount of gland enlargement, gland consistency (haziness), Inflammatory collections and gas in or near the pancreas.

    Management and Treatment
    Initial management is based around supportive care. First and most for most the patient is to be made NPO in order to allow their pancreas to relax from its digestive role. While NPO the patient will be placed on IV fluids. Initially fluid delivery will be aggressively administered and then tailored back to around 250-500ml per hour. During this time careful attention should be paid to ins and outs and fluid overload can put the patient at risk of pulmonary edema. For the patient’s pain Demerol has shown to be effective. Once stabilized if the patient’s pancreatitis was stemming from biliary tract obstruction it is recommended that they have a cholecystectomy during that same hospital stay. For patients who present due to alcohol abuse it is vital that they abstain from drinking in the future, as inevitably with continued alcohol abuse they will have another episode. Once their NPO status is revoked patients should be started on a low-fat diet. This low-fat diet should be maintained as an outpatient.

    References
    1.Tenner, S., Baillie, J., Dewitt, J., & Vege, S. S. (2013). American College of Gastroenterology Guideline: Management of Acute Pancreatitis. The American Journal of Gastroenterology,108(9), 1400-1415. doi:10.1038/ajg.2013.218

    2.Papadakis, M. A. (2016). Current Medical Diagnosis and Treatment. McGraw-Hill Education / Medical.

    3.Ilhan, M., & Als, H. (2012). Acute Biliary Pancreatitis. Acute Pancreatitis. doi:10.5772/26272

    Incorrect

    The correct answer is B due to the fact that this is the best initial study for a patient presenting with acute pancreatitis, as it allows the provider to assess for biliary stone obstruction as a potential source of the patient’s symptoms. This patient is especially susceptible to biliary tract disease since she is and obese, female, who is fertile and in her forties. Answer A, is very helpful in determining the extent of injury, but is not very useful in assessing for biliary stone obstruction which is likely the cause of this patient’s pancreatitis. Answer C endoscopic ultrasound is often used in sever pancreatitis with pseudocyst and abscess formation to rule out malignancy via biopsy. Answer D has little utility of assessing pancreatitis in the acute setting.

    Epidemiology
    Acute pancreatitis is a relatively uncommon with an incidence rate of around 40 cases per 100,000 adults in the United States. The age of onset varies amongst etiologies. The top four etiologies and their average age of onset are alcoholism 39 years, biliary tract disease 69 years, trauma 66 years and drug-induced 42 years. Acute pancreatitis affects males more often than woman. However, the etiology contributing to the increased incidence in men is alcoholism. While acute pancreatitis as a whole is less common in women they have a higher predilection for biliary tract pathophysiology then men. When it comes to race, African American have a 3 times higher risk of developing acute pancreatitis in comparison to their Caucasian counterparts.
    Pathophysiology
    Pancreatitis is classified as mild or severe. These classifications are based on the extent of injury to the pancreas. In mild pancreatitis there is inflammation and edema confined solely to the pancreas. In severe pancreatitis the extent of injury leads to necrosis of pancreatic tissue and subsequent damage to adjacent organs. The specific trigger to the inflammatory cascade in acute pancreatitis is largely unknown. However, there is a known association between a combination of extracellular neural and vascular responses and intracellular events like digestive enzyme activations, increased calcium signaling and heat shock proteins. The initial inflammatory response is primarily mediated TNF-alpha and IL-1 which are produced by pancreatic cells. These inflammatory markers draw in neutrophils to the pancreas. This primary inflammatory response is what leads to all the secondary systemic sequela. Once the inflammatory cascade is triggered the disease state begins with peripancreatic fat necrosis. This necrosis eventually begins to encompass parenchymal tissue. Once parenchymal tissue is involved there is hemorrhage and true damage to the gland itself. Often hemorrhagic necrotizing pancreatitis leads to walling off of digestive enzymes which present as pseudocyst and bacterial seeding progressing to abscess formation.

    History
    Acute pancreatitis usually present as a boring, gnawing type epigastric pain that radiates to the back. The location of the pain can lateralize left or right, but is generally reported as an upper quadrant pain. The pain usually starts are brief episodes of dull aching which over time become more frequent and intense. The pain is occasionally relieved when sitting up or bending forward. The evolution of the pain is what most often leads to patients presenting in clinic. Often nausea and vomiting are present along with anorexia. It is important to ask these patients if they have had any recent ERCP or non-surgical trauma, are they drinkers, do they have any previous biliary tract disease and what medications are they taking.

    Physical
    On physical exam tenderness to palpation and guarding will be noted in the epigastrium, right upper quadrant or left upper quadrant. The abdomen may be mildly distended with decreased bowel sounds. Patients with severe cases of pancreatitis may develop acute respiratory distress syndrome which presents with dyspnea or tachypnea. These patients may have rales present on auscultation primarily in the left lung fields. Hemorrhagic pancreatitis can lead to hematemesis or melena. These hemorrhagic patients will often present febrile, pale and diaphoretic. Occasionally seen on exam in hemorrhagic pancreatitis are Cullen sign (ecchymosis around the umbilicus) and Turner sign (ecchymosis in the left flank).

    Labs/Imaging
    When working up someone who presents with acute pancreatitis there are several labs that are crucial in both the diagnosis and prognosis. The first lab values of interest is these patients are amylase and lipase. When looking at the amylase and lipase it is crucial that these lab value are three times the upper limit of normal. Without an amylase or lipase of three times the upper limit of normal acute pancreatitis becomes significantly less likely as a diagnosis. Next labs of great importance after assessing the amylase and lipase is the patients liver function tests. The AST/ALT, bilirubin and alkaline phosphatase will help in guiding the provider in determining the pathophysiology behind this episode of acute pancreatitis. A patient with an increased liver function tests will likely have acute pancreatitis stemming from obstructive gallstones. When tissue necrotizes is begins to increase its uptake of calcium. This increase in calcium uptake can often lead to hypocalcemia, so it is important that calcium levels are checked. In addition to calcium, there may also be an increase in glucose due to decrease insulin production. Get a lipid panel as increased triglycerides will suggest the patient’s pancreatitis is stemming from alcohol abuse. Finally, a CBC w/ diff will often show and elevated white count and a left shift.
    The Imaging of choice in a patient with acute pancreatitis is going to be an abdominal CT. While abdominal is the diagnostic gold standard ultrasonography is often a very viable initial diagnostic study. Ultrasound is better at assessing the presence of biliary stone obstruction than CT. However, CT imaging allows radiologist to grade the extent of disease based on amount of gland enlargement, gland consistency (haziness), Inflammatory collections and gas in or near the pancreas.

    Management and Treatment
    Initial management is based around supportive care. First and most for most the patient is to be made NPO in order to allow their pancreas to relax from its digestive role. While NPO the patient will be placed on IV fluids. Initially fluid delivery will be aggressively administered and then tailored back to around 250-500ml per hour. During this time careful attention should be paid to ins and outs and fluid overload can put the patient at risk of pulmonary edema. For the patient’s pain Demerol has shown to be effective. Once stabilized if the patient’s pancreatitis was stemming from biliary tract obstruction it is recommended that they have a cholecystectomy during that same hospital stay. For patients who present due to alcohol abuse it is vital that they abstain from drinking in the future, as inevitably with continued alcohol abuse they will have another episode. Once their NPO status is revoked patients should be started on a low-fat diet. This low-fat diet should be maintained as an outpatient.

    References
    1.Tenner, S., Baillie, J., Dewitt, J., & Vege, S. S. (2013). American College of Gastroenterology Guideline: Management of Acute Pancreatitis. The American Journal of Gastroenterology,108(9), 1400-1415. doi:10.1038/ajg.2013.218

    2.Papadakis, M. A. (2016). Current Medical Diagnosis and Treatment. McGraw-Hill Education / Medical.

    3.Ilhan, M., & Als, H. (2012). Acute Biliary Pancreatitis. Acute Pancreatitis. doi:10.5772/26272

  208. Question 208 of 220
    208. Question

    A 65 year old male with a PMH of uncontrolled HTN, HLD, and DM presents to the emergency department with the chief complaint of tearing chest pain that radiates to his back which began two hours ago. The patient states that he was lifting weights at the gym when his symptoms began. He denies ever feeling any pain like this before. He tried taking Ibuprofen 400 mg PO for the pain which didn’t help. He admits to feeling SOB and nauseous. He states his father died of a MI at age 60, and his mother has a PMH significant for HTN and HLD. He admits to smoking 1 PPD for the past 40 years, and to drinking one glass of wine at dinner each night. Vitals include a BP in the RUE is 165/92 mmHg, and LUE is 64/44 mmHg, HR is 122 bpm in NSR, O2 sat is 94% on RA, Temperature is 98.6 F oral. An EKG was performed which showed no ST elevation, depression, or T wave inversions. The PA decides to get a CXR. What would you expect to see on the the CXR with the suspected diagnosis?

    Correct

    Discussion: a) Fleischner sign is an enlarged pulmonary artery seen on CXR in patients with a pulmonary embolism 3 b) Hampton’s hump is a wedge shaped opacity seen in the peripheral region of the chest seen on CXR in patients with a pulmonary embolism3 c) is the correct answer as a widened mediastinum typically greater than 8cm is associated with an aortic dissection3 d) A watterbottle heart is an enlarged cardiac silhouette which is seen in patients with pericardial effusion.

    An aortic dissection occurs when there is an intimal tear in the medial layer of the aorta, resulting in a separation of the layers of the aorta.6 Typically, there is fibrosis and degradation of the medial layer of the aorta. Thus the vessel is less elastic and becomes hard and weak. Eventually a tear will occur leading to blood entering the medial layer of the aorta.6 Thus resulting in a true and false lumen. The false lumen is typically bigger with slow turbulent flow, while the true lumen is small with fast laminar flow.5 There are two types of aortic dissections. Type A occur in the ascending aorta or aortic arch. Type B involve the descending aorta.6 Aortic dissections are more common in men, and in those aged 60 or greater.6 Risk factors include uncontrolled hypertension, bicuspid aortic valve, atherosclerosis, coarctation of the aorta, and aortic aneurysms. Having turners syndrome, marfans syndrome, or Ehlers Danlos syndrome also puts one at greater risk for dissection.1 Pregnant women, especially those with marfans in the third trimester are at risk for a dissection. Smoking, diabetes, cocaine use, inflammatory vessel diseases, COPD, and chronic steroid use also put one at risk.6 Lastly patients may have a family history of dissection. Complications may occur in patients with aortic dissections, including death due to ruptured aorta, end organ damage due to decreased flow, aortic valve damage, cardiac tapenade, and paraplegia if the artery of adamkiewcz is involved .1,3 Symptoms of an aortic dissection include, sudden onset of severe ripping tearing chest pain that may radiate to the neck or back. Patients may feel sort of breath, weak, have trouble speaking, vision loss, paresthesia in the distal extremities, or may experience syncope .1,7 On physical exam patients may have a difference in systolic blood pressure in the upper extremities, may have a weak pulse in one arm compared to the other ,a new aortic regurgitation murmur, the patient may be hyper or hypotensive. Some patients may present in cardiogenic shock, renal failure, having a myocardial infarction, paraplegia, in a coma or stroke, or with a pleural effusion.6 Patients who come in with these symptoms should initially get an EKG. Next, patients may receive a chest x-ray, however if there is high suspicion for dissection immediately get a CT with contrast, TEE, or MRI. If the patient is unstable, perform a TEE or CT; while stable patients should get a CT or MRI.6 An x-ray may show a widened mediastinum, measuring greater than 8cm. CT may show a double lumen, an intimal flap, a mercedez benz sign (when all three layers are affected), or a windsock sign.3 Patients with type A dissection need urgent surgical repair. Those with type B dissections may require thoracic endovascular aortic repair, or may be managed medically. If patients are unstable give IVF, followed by vasopressor with a goal SBP of greater than 70 mmHg. In stable patients initial medical management includes blood pressure control using IV beta blockers, or CCB, if still not at goal add a vasodilator. Followed by initiation of an ACE.6 All patients with a dissection should be sent home on medications to control their blood pressure, and lipid levels. Patients should be encouraged to quit smoking, avoid illicit drugs and high energy exercises.6

    Works Cited Page

    1. “Aortic Dissection: When Your Aorta Tears.” Mayo Clinic, Mayo Foundation for Medical Education and Research, 28 Oct. 2014, http://www.mayoclinic.org/diseases-conditions/aortic-dissection/basics/definition/con-20032930.
    2. D’Souza, Donna. “Pulmonary Embolism | Radiology Reference Article.” Radiopaedia.org, radiopaedia.org/articles/pulmonary-embolism.
    3. Gaillard, Frank. “Aortic Dissection | Radiology Reference Article.” Radiopaedia.org, radiopaedia.org/articles/aortic-dissection.

    4. Gaillard, Frank. “Pericardial Effusion: Water Bottle Sign | Radiology Case.” Radiopaedia.org, radiopaedia.org/cases/pericardial-effusion-water-bottle-sign-1.
    5. Macura , Katarzyna J, et al. “Pathogenesis in Acute Aortic Syndromes: Aortic Dissection, Intramural Hematoma, and Penetrating Atherosclerotic Aortic Ulcer : American Journal of Roentgenology : Vol. 181, No. 2 (AJR).” American Journal of Roentgenology, http://www.ajronline.org/doi/full/10.2214/ajr.181.2.1810309.
    6. “Thoracic Aortic Dissection.” Dynamed.com, 22 Nov. 2016, http://www.dynamed.com/topics/dmp~AN~T113709/Thoracic-aortic-dissection.
    7. “What Is Aortic Dissection?” WebMD, WebMD, http://www.webmd.com/heart-disease/what-is-aortic-dissection#2-5.

    Incorrect

    Discussion: a) Fleischner sign is an enlarged pulmonary artery seen on CXR in patients with a pulmonary embolism 3 b) Hampton’s hump is a wedge shaped opacity seen in the peripheral region of the chest seen on CXR in patients with a pulmonary embolism3 c) is the correct answer as a widened mediastinum typically greater than 8cm is associated with an aortic dissection3 d) A watterbottle heart is an enlarged cardiac silhouette which is seen in patients with pericardial effusion.

    An aortic dissection occurs when there is an intimal tear in the medial layer of the aorta, resulting in a separation of the layers of the aorta.6 Typically, there is fibrosis and degradation of the medial layer of the aorta. Thus the vessel is less elastic and becomes hard and weak. Eventually a tear will occur leading to blood entering the medial layer of the aorta.6 Thus resulting in a true and false lumen. The false lumen is typically bigger with slow turbulent flow, while the true lumen is small with fast laminar flow.5 There are two types of aortic dissections. Type A occur in the ascending aorta or aortic arch. Type B involve the descending aorta.6 Aortic dissections are more common in men, and in those aged 60 or greater.6 Risk factors include uncontrolled hypertension, bicuspid aortic valve, atherosclerosis, coarctation of the aorta, and aortic aneurysms. Having turners syndrome, marfans syndrome, or Ehlers Danlos syndrome also puts one at greater risk for dissection.1 Pregnant women, especially those with marfans in the third trimester are at risk for a dissection. Smoking, diabetes, cocaine use, inflammatory vessel diseases, COPD, and chronic steroid use also put one at risk.6 Lastly patients may have a family history of dissection. Complications may occur in patients with aortic dissections, including death due to ruptured aorta, end organ damage due to decreased flow, aortic valve damage, cardiac tapenade, and paraplegia if the artery of adamkiewcz is involved .1,3 Symptoms of an aortic dissection include, sudden onset of severe ripping tearing chest pain that may radiate to the neck or back. Patients may feel sort of breath, weak, have trouble speaking, vision loss, paresthesia in the distal extremities, or may experience syncope .1,7 On physical exam patients may have a difference in systolic blood pressure in the upper extremities, may have a weak pulse in one arm compared to the other ,a new aortic regurgitation murmur, the patient may be hyper or hypotensive. Some patients may present in cardiogenic shock, renal failure, having a myocardial infarction, paraplegia, in a coma or stroke, or with a pleural effusion.6 Patients who come in with these symptoms should initially get an EKG. Next, patients may receive a chest x-ray, however if there is high suspicion for dissection immediately get a CT with contrast, TEE, or MRI. If the patient is unstable, perform a TEE or CT; while stable patients should get a CT or MRI.6 An x-ray may show a widened mediastinum, measuring greater than 8cm. CT may show a double lumen, an intimal flap, a mercedez benz sign (when all three layers are affected), or a windsock sign.3 Patients with type A dissection need urgent surgical repair. Those with type B dissections may require thoracic endovascular aortic repair, or may be managed medically. If patients are unstable give IVF, followed by vasopressor with a goal SBP of greater than 70 mmHg. In stable patients initial medical management includes blood pressure control using IV beta blockers, or CCB, if still not at goal add a vasodilator. Followed by initiation of an ACE.6 All patients with a dissection should be sent home on medications to control their blood pressure, and lipid levels. Patients should be encouraged to quit smoking, avoid illicit drugs and high energy exercises.6

    Works Cited Page

    1. “Aortic Dissection: When Your Aorta Tears.” Mayo Clinic, Mayo Foundation for Medical Education and Research, 28 Oct. 2014, http://www.mayoclinic.org/diseases-conditions/aortic-dissection/basics/definition/con-20032930.
    2. D’Souza, Donna. “Pulmonary Embolism | Radiology Reference Article.” Radiopaedia.org, radiopaedia.org/articles/pulmonary-embolism.
    3. Gaillard, Frank. “Aortic Dissection | Radiology Reference Article.” Radiopaedia.org, radiopaedia.org/articles/aortic-dissection.

    4. Gaillard, Frank. “Pericardial Effusion: Water Bottle Sign | Radiology Case.” Radiopaedia.org, radiopaedia.org/cases/pericardial-effusion-water-bottle-sign-1.
    5. Macura , Katarzyna J, et al. “Pathogenesis in Acute Aortic Syndromes: Aortic Dissection, Intramural Hematoma, and Penetrating Atherosclerotic Aortic Ulcer : American Journal of Roentgenology : Vol. 181, No. 2 (AJR).” American Journal of Roentgenology, http://www.ajronline.org/doi/full/10.2214/ajr.181.2.1810309.
    6. “Thoracic Aortic Dissection.” Dynamed.com, 22 Nov. 2016, http://www.dynamed.com/topics/dmp~AN~T113709/Thoracic-aortic-dissection.
    7. “What Is Aortic Dissection?” WebMD, WebMD, http://www.webmd.com/heart-disease/what-is-aortic-dissection#2-5.

  209. Question 209 of 220
    209. Question

    A 45-year-old man arrives to the ED complaining of RLQ abdominal pain that began last night. He notes having nausea early in the morning which has since subsided and denies hematemesis. He also presents with a fever and elevated WBC. Which physical exam finding is most expected to be positive?

    Correct

    Answer is A. Rosving’s sign, when positive is a clinical indicator of appendicitis. The maneuver involves palpation of the left lower quadrant that elicits pain in the right lower quadrant which is the site of the appendix itself. This type of referred pain occurs because the neurons sensing pain in the inflamed appendix are stimulated merely by the stretching of the peritoneal lining.
    Cullen’s sign refers to visible ecchymosis in the fatty tissue of the abdomen surrounding the belly button. Cullen’s sign can appear in multiple instances such as acute pancreatitis, abdominal rupture and ectopic pregnancy. This finding would not be present in acute appendicitis. Also, it is typically accompanied by Turner’s sign, which is ecchymosis of the flank. Since no visible ecchymosis is associated with appendicitis, Cullen’s sign does not relate to this diagnosis.
    Costovertebral angle tenderness, or CVA tenderness refers to pain that is brought on by the percussion of the posterior area overlying the kidney. This sign has multiple causes, most commonly pyelonephritis. This sign can also be positive in renal syndrome or kidney stones. However, due to the location of the appendix, percussion of the costovertebral angle will not elicit pain in appendicitis.
    Murphy’s sign is most closely associated with cholecystitis. It is a sign of low specificity but high sensitivity with sensitivity lowest in elderly populations. Murphy’s sign is demonstrated during the abdominal exam as the physician presses his hand just below the costal margin and asks the patient to breathe inward. The arrest of inspiration during palpation of this area constitutes a positive Murphy’s sign. This test is then performed on the left side with the expectation of a negative result. This finding is not present in acute appendicitis.
    Resource: Shogilev DJ, Duus N, Odom SR, Shapiro NI. Diagnosing appendicitis: evidence-based review of the diagnostic approach in 2014. West J Emerg Med 2014;15:859-71

    Discussion:
    Appendicitis is one of the most common causes of abdominal surgery throughout the world. The largest population of appendicitis patients consists of 15-29 year olds with men predominating over women.1 Recently, it has been shown that seasons may also in fact play a role. One study demonstrated that cases of appendicitis were 11.76% higher in the summer months when compared to winter.2
    The exact cause of appendicitis remains a mystery. However, some key factors have been recognized. The most common etiology of appendicitis appears to be an obstruction. Obstructions are often caused by foreign bodies i.e. mucous plugs or edema. However, they can also arise through strictures or deformities. In developed countries, fecaliths are among the most common cause for luminal obstruction resulting in appendicitis. Eventually, the obstruction will lead to an impairment of blood flow and bacterial overgrowth. This ultimately leads to inflammation of the appendix which then causes appendicitis’ symptomatic presentation.
    The most typical appendicitis patient will arrive in the emergency department complaining of significant pain and discomfort in the right-lower abdominal quadrant. Frequently, the patient will complain of pain that started In the periumbilical area and travel to lower and to the right.3 Most often, the patient will have a low-grade fever along with an elevated white blood cell count. Anorexia, along with nausea and vomiting are other common features of appendicitis. On physical exam, the point of maximal tenderness will typically be noted in “McBurney’s point”. This is the area one-third the distance from the umbilicus to the anterior-superior iliac spine. The patient may also demonstrate rebound tenderness and exhibit positive clinical signs i.e. Rosving’s, Psoas, and Obturator signs.
    Once appendicitis is suspected, imaging may then be performed in addition to obtaining a CBC to further confirm the diagnosis. In the majority of cases, the patient’s white blood cell count in an appendicitis will be elevated. CT scans remain most accurate imaging modality to assess for appendicitis. However, due to the large amount of radiation associated with computed tomography, ultrasound is being utilized exponentially more. The accuracy of ultrasound in the diagnosis of acute appendicitis is more user-dependent however it’s been demonstrated to have a sensitivity of 83.7% and a specificity of 95.9%.4
    Urgent appendectomy remains the “gold standard” treatment of appendicitis. Immediately following the diagnosis, an antibiotic regimen is started. Typical antibiotics used in appendicitis are Zosyn, Flagyl and Levaquin. In some cases, of mild infection or inflammation, the appendicitis is able to be treated conservatively with antibiotics alone. While this prevents the patient from having to undergo surgery, it remains likely that they will later experience a recurrence necessitating an appendectomy.5

    References:
    1. Lin KB, Lai KR, Yang NP, Chan CL, Liu YH, Pan RH, Huang CH. Epidemiology and socioeconomic features of appendicitis in Taiwan: a 12-year population-based study. World J Emerg Surg. 2015;10:42

    2. Oguntola A S, Adeoti M L, Oyemolade T A. Appendicitis: Trends in incidence, age, sex, and seasonal variations in South-Western Nigeria. Ann Afr Med 2010;9:213-7

    3. Shogilev DJ, Duus N, Odom SR, Shapiro NI. Diagnosing appendicitis: evidence-based review of the diagnostic approach in 2014. West J Emerg Med 2014;15:859-71.

    4. Humes DJ, Simpson J. Acute appendicitis. BMJ 2006;333:530-4

    5. Salminen P, Paajanen H , Rautio T , et al . Antibiotic therapy vs appendectomy for treatment of uncomplicated acute appendicitis. JAMA 2015;313:2340

    Incorrect

    Answer is A. Rosving’s sign, when positive is a clinical indicator of appendicitis. The maneuver involves palpation of the left lower quadrant that elicits pain in the right lower quadrant which is the site of the appendix itself. This type of referred pain occurs because the neurons sensing pain in the inflamed appendix are stimulated merely by the stretching of the peritoneal lining.
    Cullen’s sign refers to visible ecchymosis in the fatty tissue of the abdomen surrounding the belly button. Cullen’s sign can appear in multiple instances such as acute pancreatitis, abdominal rupture and ectopic pregnancy. This finding would not be present in acute appendicitis. Also, it is typically accompanied by Turner’s sign, which is ecchymosis of the flank. Since no visible ecchymosis is associated with appendicitis, Cullen’s sign does not relate to this diagnosis.
    Costovertebral angle tenderness, or CVA tenderness refers to pain that is brought on by the percussion of the posterior area overlying the kidney. This sign has multiple causes, most commonly pyelonephritis. This sign can also be positive in renal syndrome or kidney stones. However, due to the location of the appendix, percussion of the costovertebral angle will not elicit pain in appendicitis.
    Murphy’s sign is most closely associated with cholecystitis. It is a sign of low specificity but high sensitivity with sensitivity lowest in elderly populations. Murphy’s sign is demonstrated during the abdominal exam as the physician presses his hand just below the costal margin and asks the patient to breathe inward. The arrest of inspiration during palpation of this area constitutes a positive Murphy’s sign. This test is then performed on the left side with the expectation of a negative result. This finding is not present in acute appendicitis.
    Resource: Shogilev DJ, Duus N, Odom SR, Shapiro NI. Diagnosing appendicitis: evidence-based review of the diagnostic approach in 2014. West J Emerg Med 2014;15:859-71

    Discussion:
    Appendicitis is one of the most common causes of abdominal surgery throughout the world. The largest population of appendicitis patients consists of 15-29 year olds with men predominating over women.1 Recently, it has been shown that seasons may also in fact play a role. One study demonstrated that cases of appendicitis were 11.76% higher in the summer months when compared to winter.2
    The exact cause of appendicitis remains a mystery. However, some key factors have been recognized. The most common etiology of appendicitis appears to be an obstruction. Obstructions are often caused by foreign bodies i.e. mucous plugs or edema. However, they can also arise through strictures or deformities. In developed countries, fecaliths are among the most common cause for luminal obstruction resulting in appendicitis. Eventually, the obstruction will lead to an impairment of blood flow and bacterial overgrowth. This ultimately leads to inflammation of the appendix which then causes appendicitis’ symptomatic presentation.
    The most typical appendicitis patient will arrive in the emergency department complaining of significant pain and discomfort in the right-lower abdominal quadrant. Frequently, the patient will complain of pain that started In the periumbilical area and travel to lower and to the right.3 Most often, the patient will have a low-grade fever along with an elevated white blood cell count. Anorexia, along with nausea and vomiting are other common features of appendicitis. On physical exam, the point of maximal tenderness will typically be noted in “McBurney’s point”. This is the area one-third the distance from the umbilicus to the anterior-superior iliac spine. The patient may also demonstrate rebound tenderness and exhibit positive clinical signs i.e. Rosving’s, Psoas, and Obturator signs.
    Once appendicitis is suspected, imaging may then be performed in addition to obtaining a CBC to further confirm the diagnosis. In the majority of cases, the patient’s white blood cell count in an appendicitis will be elevated. CT scans remain most accurate imaging modality to assess for appendicitis. However, due to the large amount of radiation associated with computed tomography, ultrasound is being utilized exponentially more. The accuracy of ultrasound in the diagnosis of acute appendicitis is more user-dependent however it’s been demonstrated to have a sensitivity of 83.7% and a specificity of 95.9%.4
    Urgent appendectomy remains the “gold standard” treatment of appendicitis. Immediately following the diagnosis, an antibiotic regimen is started. Typical antibiotics used in appendicitis are Zosyn, Flagyl and Levaquin. In some cases, of mild infection or inflammation, the appendicitis is able to be treated conservatively with antibiotics alone. While this prevents the patient from having to undergo surgery, it remains likely that they will later experience a recurrence necessitating an appendectomy.5

    References:
    1. Lin KB, Lai KR, Yang NP, Chan CL, Liu YH, Pan RH, Huang CH. Epidemiology and socioeconomic features of appendicitis in Taiwan: a 12-year population-based study. World J Emerg Surg. 2015;10:42

    2. Oguntola A S, Adeoti M L, Oyemolade T A. Appendicitis: Trends in incidence, age, sex, and seasonal variations in South-Western Nigeria. Ann Afr Med 2010;9:213-7

    3. Shogilev DJ, Duus N, Odom SR, Shapiro NI. Diagnosing appendicitis: evidence-based review of the diagnostic approach in 2014. West J Emerg Med 2014;15:859-71.

    4. Humes DJ, Simpson J. Acute appendicitis. BMJ 2006;333:530-4

    5. Salminen P, Paajanen H , Rautio T , et al . Antibiotic therapy vs appendectomy for treatment of uncomplicated acute appendicitis. JAMA 2015;313:2340

  210. Question 210 of 220
    210. Question

    A 52-year-old Caucasian female presents to the GI office with new onset of crampy abdominal pain, persistent diarrhea with bright red blood, and weight loss. She also reports an urgency to have a bowel movement, even shortly after evacuating her bowels. Much of her diet consists of cereals, sandwiches, pizza, and hamburgers. Which of the following disorders is her most likely diagnosis?

    Correct

    Ulcerative Colitis is an inflammatory bowel disease that begins in the rectum and can progress to involve the entire colon. It is due to an abnormal immune response in the large intestine which causes inflammation of the mucosal layer of the colon. The exact cause of the disease is unknown, although there are many genetic and environmental factors involved, such as NSAID and antibiotic use, which can increase risk of the disease.1 Peak onset for ulcerative colitis is between the ages of 10 and 40, but can occur at any age and it affects both men and women equally.2 Approximately 900,000 Americans currently are affected by ulcerative colitis.2 Symptoms consist of persistent bloody diarrhea, abdominal pain, tenesmus, an increased number of daily bowel movements, and weight loss.3 Diagnosis is made with colonoscopy and tissue biopsy. Treatment consists of oral and rectal 5-ASA drugs such as mesalamine. Oral steroids may be used temporarily for severe flares. If unable to retain remission with 5-ASA medications, then anti-TNF therapies may be used.4 Surgical removal of the colon is curative, but saved as a last resort option.

    Peptic ulcers may cause abdominal pain, but typically present with upper gastrointestinal symptoms such as acid reflux or nausea. If the ulcer were to bleed, it would appear as black tarry stool (melena) as opposed to bright red blood.
    Celiac Disease may present with abdominal pain, diarrhea, and weight loss. Also, the patient does consume a large amount of foods which contain gluten. But it does not lead to blood in stools.
    Crohn’s Disease is another form of inflammatory bowel disease which also causes crampy abdominal pain, bloody diarrhea, and weight loss. Typically, the diarrhea is less severe and blood may not always be present in the stool. Also, the tenesmus reported by the patient is more indicative of ulcerative colitis.

    References
    1. UpToDate. (2018). Clinical manifestations, diagnosis, and prognosis of ulcerative colitis in adults. [online] Available at: https://www-uptodate-com.une.idm.oclc.org/contents/clinical-manifestations-diagnosis-and-prognosis-of-ulcerative-colitis-in-adults [Accessed 11 Mar. 2018]

    2. Dynamed.com. (2018). Inflammatory Bowel Diseases. [online] Available at: http://www.dynamed.com/topics/dmp~AN~T114507/Ulcerative-colitis [Accessed 11 Mar. 2018].

    3. Crohnscolitisfoundation.org. (2018). What is Crohn’s | What is Colitis | What is IBD | Crohn’s & Colitis Foundation. [online] Available at: http://www.crohnscolitisfoundation.org/what-are-crohns-and-colitis/ [Accessed 11 Mar. 2018].

    4. Mayo Clinic. (2018). Ulcerative colitis. [online] Available at: https://www.mayoclinic.org/diseases-conditions/ulcerative-colitis/symptoms-causes/syc-20353326 [Accessed 11 Mar. 2018].

    5. National Institute of Diabetes and Digestive and Kidney Diseases. (2018). Digestive Diseases. [online] Available at: https://www.niddk.nih.gov/health-information/digestive-diseases/ [Accessed 11 Mar. 2018].

    Incorrect

    Ulcerative Colitis is an inflammatory bowel disease that begins in the rectum and can progress to involve the entire colon. It is due to an abnormal immune response in the large intestine which causes inflammation of the mucosal layer of the colon. The exact cause of the disease is unknown, although there are many genetic and environmental factors involved, such as NSAID and antibiotic use, which can increase risk of the disease.1 Peak onset for ulcerative colitis is between the ages of 10 and 40, but can occur at any age and it affects both men and women equally.2 Approximately 900,000 Americans currently are affected by ulcerative colitis.2 Symptoms consist of persistent bloody diarrhea, abdominal pain, tenesmus, an increased number of daily bowel movements, and weight loss.3 Diagnosis is made with colonoscopy and tissue biopsy. Treatment consists of oral and rectal 5-ASA drugs such as mesalamine. Oral steroids may be used temporarily for severe flares. If unable to retain remission with 5-ASA medications, then anti-TNF therapies may be used.4 Surgical removal of the colon is curative, but saved as a last resort option.

    Peptic ulcers may cause abdominal pain, but typically present with upper gastrointestinal symptoms such as acid reflux or nausea. If the ulcer were to bleed, it would appear as black tarry stool (melena) as opposed to bright red blood.
    Celiac Disease may present with abdominal pain, diarrhea, and weight loss. Also, the patient does consume a large amount of foods which contain gluten. But it does not lead to blood in stools.
    Crohn’s Disease is another form of inflammatory bowel disease which also causes crampy abdominal pain, bloody diarrhea, and weight loss. Typically, the diarrhea is less severe and blood may not always be present in the stool. Also, the tenesmus reported by the patient is more indicative of ulcerative colitis.

    References
    1. UpToDate. (2018). Clinical manifestations, diagnosis, and prognosis of ulcerative colitis in adults. [online] Available at: https://www-uptodate-com.une.idm.oclc.org/contents/clinical-manifestations-diagnosis-and-prognosis-of-ulcerative-colitis-in-adults [Accessed 11 Mar. 2018]

    2. Dynamed.com. (2018). Inflammatory Bowel Diseases. [online] Available at: http://www.dynamed.com/topics/dmp~AN~T114507/Ulcerative-colitis [Accessed 11 Mar. 2018].

    3. Crohnscolitisfoundation.org. (2018). What is Crohn’s | What is Colitis | What is IBD | Crohn’s & Colitis Foundation. [online] Available at: http://www.crohnscolitisfoundation.org/what-are-crohns-and-colitis/ [Accessed 11 Mar. 2018].

    4. Mayo Clinic. (2018). Ulcerative colitis. [online] Available at: https://www.mayoclinic.org/diseases-conditions/ulcerative-colitis/symptoms-causes/syc-20353326 [Accessed 11 Mar. 2018].

    5. National Institute of Diabetes and Digestive and Kidney Diseases. (2018). Digestive Diseases. [online] Available at: https://www.niddk.nih.gov/health-information/digestive-diseases/ [Accessed 11 Mar. 2018].

  211. Question 211 of 220
    211. Question

    A 48-year-old male presents to his PCP with complaints of abdominal pain and fatigue that has been progressively worsening over the last few months. He denies any unintentional weight loss or dizziness. His vital signs are stable. Physical exam reveals hepatomegaly and a bronze discoloration of his skin, particularly around his ankles. He denies any alcohol use, illicit drug use, or prior history of liver disease. He has a history of asthma and hyperlipidemia and only takes medications for these conditions, which are both controlled. What would you expect the results of your diagnostic studies to show?

    Correct

    Hereditary hemochromatosis (HH) is an autosomal recessive disease most commonly caused by a mutation in the HFE gene on chromosome 6, resulting in a cysteine to tyrosine substitution at amino acid 282 (C282Y).1 The mutation causes increased intestinal absorption of iron. Other genetic mutations that affect hemojuvelin, hepcidin, ferroportin 1, and other proteins can also cause HH, but about 85% of people with HH are homozygous for the C282Y mutation.1,3 Another mutation, an amino acid substitution of histidine to aspartic acid at position 63 (H63D), is also commonly associated with increased iron stores, especially if people have one copy of the C282Y mutation and another of the H63D mutation.2 However, heterozygotes have no, or minimal, increases in iron stores.
    HH is most commonly found in peoples of northern European descent, in which 1 in 10 are heterozygous carriers, and 0.3-0.5% are homozygotes.2 The disease is much less commonly seen in blacks and Asian-American populations.1 The expression of the disease is about 5-10 times more common in men, as women can mask the disease as they lose blood monthly through menstruation.2 The penetrance of the genetic mutation is variable, as some studies have shown that about 30% of homozygous individuals did not have evidence of iron overload.2 About 70% of patients will begin to develop symptoms between ages 40-60 years old, and it is rare to manifest before age 20.2
    Iron is typically lost in sweat, skin cells, and the GI tract at 1 mg/day and women lose an additional 0.5-1 mg/day during menstruation.3 The losses are balanced out by the fact that 10% of the average 10-20 mg of iron consumed in a Western diet is absorbed daily by the duodenum.3 The amount absorbed is typically about 1 mg/day in men and 1.5 mg/day in menstruating women, but in hemochromatosis, the intestinal absorption of iron exceeds the body’s needs and equates to about 4 mg/day or more.2 This accumulation of iron leads to elevations in plasma iron, an increased transferrin saturation, and elevation of plasma ferritin levels. The retention of an extra 3 mg of iron daily accumulates to about 1 g extra yearly, which explains why it would take about 20 years for a person to have an excess of 20 g of iron in the body creating symptoms, which would typically present around or after age 40.3
    Hemochromatosis is characterized by the accumulation of iron as the storage form of hemosiderin in the parenchymal cells of major organs including the liver, pancreas, heart, adrenals, testes, pituitary, and kidneys.1 Iron may be increased 50- to 100-fold in the liver and pancreas and 5- to 25-fold in the heart.2 The most common initial symptoms of HH include weakness, weight loss, hyperpigmentation of skin color as hemosiderin deposits in the skin, symptoms of diabetes, and abdominal pain.2 Excessive skin pigmentation is present at the time of diagnosis in about 90% of patients.2 Sometimes referred to as “bronze diabetes,” the characteristic metallic grayish hue of the skin results from increased iron and melanin in the dermis.2 The liver is usually the first organ to be affected from HH and hepatomegaly is found in over 95% of patients.2 Iron deposition in the liver can be associated with elevated liver enzymes and can lead to fibrotic changes and eventually cirrhosis.2 Patients are at risk for the development of cirrhosis with ferritin levels >1000 mcg/L. Iron potentiates liver disease because it acts as an electron donor, which can accelerate the generation of unstable free radicals, leading to membrane injury and inflammatory responses.3 Splenomegaly, portal hypertension, and varices can also be associated with hepatic changes.2 Reversibility of these changes can be achieved with early therapeutic iron removal if diagnosed early.3 About 50% of all deaths associated with HH are due to hepatocellular carcinoma, which is one of the most serious complications of excess hepatic iron deposition.3 Diabetes mellitus develops in approximately half of patients with HH and is more common in those with a positive family history, but is also associated with direct damage to the pancreatic islets by iron deposition.2 Arthropathies are also commonly associated with HH, and typically the second and third metacarpophalangeal joints are first involved, which can then lead to a progressive polyarthropathy afflicting the hips, ankles, wrists and knees.2 HH can also lead to dilated cardiomyopathy resulting from congestive heart failure, from the excess of iron deposited in the myocardium.2 Similarly, iron can deposit in the pituitary gland and can lead to hypogonadotropic hypogonadism, which can manifest as loss of libido, impotence, amenorrhea, testicular atrophy, gynecomastia, and sparse body hair.2
    Diagnosis of HH should be suspected in patients with a positive family history and with the association of hepatomegaly, skin hyperpigmentation, diabetes, heart disease, arthropathies, and hypogonadism. Lab studies should assess iron levels including the measurement of serum iron, transferrin saturation, and serum ferritin. An elevated plasma iron level with a >45-50% transferrin saturation and elevated ferritin levels point toward the diagnosis of HH.1-2 An increase of 1 microgram/L in serum ferritin levels reflect an increase in about 65 mg in body stores of iron.2 Elevated liver enzymes (AST, alkaline phosphatase) can indicate liver involvement and should prompt imaging of the liver through MRI or CT scan to assess for fibrotic changes, and ultimately liver biopsies or fibroscans to assess for cirrhosis.1-3 Genetic testing of the HFE mutations should also be performed in patients with iron overload to confirm the diagnosis.1 The arthropathies associated with iron overload are associated with squared-off bone ends and hook-like osteophytes in the MCP joints, which are characteristic radiologic findings.3 Cardiac involvement from HH can be diagnosed from echocardiograms or endomyocardial biopsies.3
    Treatment for HH should be initiated to avoid complications of commonly affected organs as outlined above. For patients without organ damage or iron overload (those identified through familial genetic testing) and who have ferritin levels <500 mcg/L and transferrin <60%, clinical status can be monitored yearly without the need for phlebotomy or iron chelation treatments. Most patients with iron overload and symptoms benefit from phlebotomy. Weekly or bi-weekly phlebotomy treatments are indicated for symptomatic HH or those with ferritin levels >500 mcg/L or transferrin >60%.4 Each 500mL of whole blood removed contains 200-250 mg of iron.4 Phlebotomies are generally performed weekly for about 1-2 years or until iron stores are normalized. Guidelines vary for exactly when weekly phlebotomies can cease, but hepatologists define it as serum ferritin concentrations <50 mcg/L and transferrin saturation <50%.4 Maintenance phlebotomy schedules every 2-4 months should then be initiated in patients to prevent re-accumulation of iron and maintain ferritin concentrations between 50 and 100 mcg/L.4 Chelating agents, such as deferoxamine, are indicated for patients with HH who cannot tolerate phlebotomies. The drug is administered subcutaneously or intravenously in a dose of 20-40 mg/kg/day and can mobilize about 30 mg of iron/day, which is significantly less than phlebotomy.1 Chelating agents are also more expensive, treatment can be painful and time-consuming, and the agents have a number of side-effects and drug interactions.1-2 Excess alcohol intake is a major risk factor for the development of liver disease or hepatocellular carcinoma in patients with HH, so it should be avoided.3 People with HH should also avoid foods rich in iron, like red meat, vitamin C, which increases the absorption of iron, shellfish, and supplemental iron.1 Iron studies and HFE genetic testing are indicated for all first-degree relatives of someone with HH.1
    The course of HH is vastly changed by the initiation of phlebotomy treatments. The deaths associated with HH patients are associated with hepatocellular carcinoma, cardiac failure, and portal hypertension. Life expectancy is greatly improved by removing excess iron stores through phlebotomy. The 5-year survival rate increases from 33% without treatment to 89% once initiated.2 Cardiac, diabetic, and pituitary complications should also be managed medically.
    Answer A is incorrect as it describes laboratory findings in Wilson’s Disease, which refers to the excessive deposition of copper in the liver and the brain. It is a rare autosomal recessive disorder that usually occurs in people under the age of 40. It typically presents as liver disease and should be considered in patients with hepatomegaly, but neurological symptoms are also common. The pathognomonic finding is a brownish Kayser-Fleischer ring along the cornea, but other skin pigmentation changes are not present.1 Answer C is incorrect as it describes the laboratory findings associated with Hepatitis C. HCV is usually spread through IV drug abuse, of which this patient denies, making this option less likely. It can also be spread through blood transfusions or sexual intercourse, but there was not enough information in the stem to make this a likely option. In some cases, hepatitis can cause a patient to appear jaundiced, but it does not cause a brownish tint at the ankles. Answer D is incorrect as it describes the laboratory findings associated with Addison’s disease. Adrenal insufficiency can present with fatigue and abdominal symptoms, but it typically presents with associated low blood pressure. This patient’s vital signs are stable. Adrenal insufficiency can be stimulated by cessation of exogenous steroid use, but that information was not provided and should not be inferred. It is also usually associated with diarrhea, whereas this patient was suffering from constipation.1 Answer E is incorrect as it describes the associated tumor marker elevation suggesting hepatocellular carcinoma (HCC). The development of HCC is usually associated with alcohol abuse, for which this patient denies, and is associated with abnormal weight loss and jaundice, both of which are absent in this presentation.

    References:
    1. Papadakis MA, McPhee SJ, Rabow MW, eds. CURRENT Medical Diagnosis & Treatment 2015. 54th ed. New York: McGraw-Hill Education; 2015.
    2. Powell LW, Isselbacher KJ. Hemochromatosis. In: Braunwald E, Hauser SL, Fauci AS, Longo DL, Kasper DL, Jameson JL, eds. Harrison’s Principles of Internal Medicine. 15th ed. New York: The McGraw-Hill Companies, Inc; 2001:2257-2261.
    3. Schrier SL, Bacon, BR. Clinical manifestations and diagnosis of hereditary hemochromatosis. UpToDate. https://www.uptodate.com/contents/clinical-manifestations-and-diagnosis-of-hereditary-hemochromatosis?source=search_result&search=hemochromatosis&selectedTitle=1~81. Updated December 9, 2016. Accessed November 18, 2017.
    4. Schrier SL, Bacon, BR. Management of patients with hereditary hemochromatosis. UpToDate. https://www.uptodate.com/contents/management-of-patients-with-hereditary-hemochromatosis?source=history_widget. Updated February 8, 2017. Accessed November 18, 2017.

    Incorrect

    Hereditary hemochromatosis (HH) is an autosomal recessive disease most commonly caused by a mutation in the HFE gene on chromosome 6, resulting in a cysteine to tyrosine substitution at amino acid 282 (C282Y).1 The mutation causes increased intestinal absorption of iron. Other genetic mutations that affect hemojuvelin, hepcidin, ferroportin 1, and other proteins can also cause HH, but about 85% of people with HH are homozygous for the C282Y mutation.1,3 Another mutation, an amino acid substitution of histidine to aspartic acid at position 63 (H63D), is also commonly associated with increased iron stores, especially if people have one copy of the C282Y mutation and another of the H63D mutation.2 However, heterozygotes have no, or minimal, increases in iron stores.
    HH is most commonly found in peoples of northern European descent, in which 1 in 10 are heterozygous carriers, and 0.3-0.5% are homozygotes.2 The disease is much less commonly seen in blacks and Asian-American populations.1 The expression of the disease is about 5-10 times more common in men, as women can mask the disease as they lose blood monthly through menstruation.2 The penetrance of the genetic mutation is variable, as some studies have shown that about 30% of homozygous individuals did not have evidence of iron overload.2 About 70% of patients will begin to develop symptoms between ages 40-60 years old, and it is rare to manifest before age 20.2
    Iron is typically lost in sweat, skin cells, and the GI tract at 1 mg/day and women lose an additional 0.5-1 mg/day during menstruation.3 The losses are balanced out by the fact that 10% of the average 10-20 mg of iron consumed in a Western diet is absorbed daily by the duodenum.3 The amount absorbed is typically about 1 mg/day in men and 1.5 mg/day in menstruating women, but in hemochromatosis, the intestinal absorption of iron exceeds the body’s needs and equates to about 4 mg/day or more.2 This accumulation of iron leads to elevations in plasma iron, an increased transferrin saturation, and elevation of plasma ferritin levels. The retention of an extra 3 mg of iron daily accumulates to about 1 g extra yearly, which explains why it would take about 20 years for a person to have an excess of 20 g of iron in the body creating symptoms, which would typically present around or after age 40.3
    Hemochromatosis is characterized by the accumulation of iron as the storage form of hemosiderin in the parenchymal cells of major organs including the liver, pancreas, heart, adrenals, testes, pituitary, and kidneys.1 Iron may be increased 50- to 100-fold in the liver and pancreas and 5- to 25-fold in the heart.2 The most common initial symptoms of HH include weakness, weight loss, hyperpigmentation of skin color as hemosiderin deposits in the skin, symptoms of diabetes, and abdominal pain.2 Excessive skin pigmentation is present at the time of diagnosis in about 90% of patients.2 Sometimes referred to as “bronze diabetes,” the characteristic metallic grayish hue of the skin results from increased iron and melanin in the dermis.2 The liver is usually the first organ to be affected from HH and hepatomegaly is found in over 95% of patients.2 Iron deposition in the liver can be associated with elevated liver enzymes and can lead to fibrotic changes and eventually cirrhosis.2 Patients are at risk for the development of cirrhosis with ferritin levels >1000 mcg/L. Iron potentiates liver disease because it acts as an electron donor, which can accelerate the generation of unstable free radicals, leading to membrane injury and inflammatory responses.3 Splenomegaly, portal hypertension, and varices can also be associated with hepatic changes.2 Reversibility of these changes can be achieved with early therapeutic iron removal if diagnosed early.3 About 50% of all deaths associated with HH are due to hepatocellular carcinoma, which is one of the most serious complications of excess hepatic iron deposition.3 Diabetes mellitus develops in approximately half of patients with HH and is more common in those with a positive family history, but is also associated with direct damage to the pancreatic islets by iron deposition.2 Arthropathies are also commonly associated with HH, and typically the second and third metacarpophalangeal joints are first involved, which can then lead to a progressive polyarthropathy afflicting the hips, ankles, wrists and knees.2 HH can also lead to dilated cardiomyopathy resulting from congestive heart failure, from the excess of iron deposited in the myocardium.2 Similarly, iron can deposit in the pituitary gland and can lead to hypogonadotropic hypogonadism, which can manifest as loss of libido, impotence, amenorrhea, testicular atrophy, gynecomastia, and sparse body hair.2
    Diagnosis of HH should be suspected in patients with a positive family history and with the association of hepatomegaly, skin hyperpigmentation, diabetes, heart disease, arthropathies, and hypogonadism. Lab studies should assess iron levels including the measurement of serum iron, transferrin saturation, and serum ferritin. An elevated plasma iron level with a >45-50% transferrin saturation and elevated ferritin levels point toward the diagnosis of HH.1-2 An increase of 1 microgram/L in serum ferritin levels reflect an increase in about 65 mg in body stores of iron.2 Elevated liver enzymes (AST, alkaline phosphatase) can indicate liver involvement and should prompt imaging of the liver through MRI or CT scan to assess for fibrotic changes, and ultimately liver biopsies or fibroscans to assess for cirrhosis.1-3 Genetic testing of the HFE mutations should also be performed in patients with iron overload to confirm the diagnosis.1 The arthropathies associated with iron overload are associated with squared-off bone ends and hook-like osteophytes in the MCP joints, which are characteristic radiologic findings.3 Cardiac involvement from HH can be diagnosed from echocardiograms or endomyocardial biopsies.3
    Treatment for HH should be initiated to avoid complications of commonly affected organs as outlined above. For patients without organ damage or iron overload (those identified through familial genetic testing) and who have ferritin levels <500 mcg/L and transferrin <60%, clinical status can be monitored yearly without the need for phlebotomy or iron chelation treatments. Most patients with iron overload and symptoms benefit from phlebotomy. Weekly or bi-weekly phlebotomy treatments are indicated for symptomatic HH or those with ferritin levels >500 mcg/L or transferrin >60%.4 Each 500mL of whole blood removed contains 200-250 mg of iron.4 Phlebotomies are generally performed weekly for about 1-2 years or until iron stores are normalized. Guidelines vary for exactly when weekly phlebotomies can cease, but hepatologists define it as serum ferritin concentrations <50 mcg/L and transferrin saturation <50%.4 Maintenance phlebotomy schedules every 2-4 months should then be initiated in patients to prevent re-accumulation of iron and maintain ferritin concentrations between 50 and 100 mcg/L.4 Chelating agents, such as deferoxamine, are indicated for patients with HH who cannot tolerate phlebotomies. The drug is administered subcutaneously or intravenously in a dose of 20-40 mg/kg/day and can mobilize about 30 mg of iron/day, which is significantly less than phlebotomy.1 Chelating agents are also more expensive, treatment can be painful and time-consuming, and the agents have a number of side-effects and drug interactions.1-2 Excess alcohol intake is a major risk factor for the development of liver disease or hepatocellular carcinoma in patients with HH, so it should be avoided.3 People with HH should also avoid foods rich in iron, like red meat, vitamin C, which increases the absorption of iron, shellfish, and supplemental iron.1 Iron studies and HFE genetic testing are indicated for all first-degree relatives of someone with HH.1
    The course of HH is vastly changed by the initiation of phlebotomy treatments. The deaths associated with HH patients are associated with hepatocellular carcinoma, cardiac failure, and portal hypertension. Life expectancy is greatly improved by removing excess iron stores through phlebotomy. The 5-year survival rate increases from 33% without treatment to 89% once initiated.2 Cardiac, diabetic, and pituitary complications should also be managed medically.
    Answer A is incorrect as it describes laboratory findings in Wilson’s Disease, which refers to the excessive deposition of copper in the liver and the brain. It is a rare autosomal recessive disorder that usually occurs in people under the age of 40. It typically presents as liver disease and should be considered in patients with hepatomegaly, but neurological symptoms are also common. The pathognomonic finding is a brownish Kayser-Fleischer ring along the cornea, but other skin pigmentation changes are not present.1 Answer C is incorrect as it describes the laboratory findings associated with Hepatitis C. HCV is usually spread through IV drug abuse, of which this patient denies, making this option less likely. It can also be spread through blood transfusions or sexual intercourse, but there was not enough information in the stem to make this a likely option. In some cases, hepatitis can cause a patient to appear jaundiced, but it does not cause a brownish tint at the ankles. Answer D is incorrect as it describes the laboratory findings associated with Addison’s disease. Adrenal insufficiency can present with fatigue and abdominal symptoms, but it typically presents with associated low blood pressure. This patient’s vital signs are stable. Adrenal insufficiency can be stimulated by cessation of exogenous steroid use, but that information was not provided and should not be inferred. It is also usually associated with diarrhea, whereas this patient was suffering from constipation.1 Answer E is incorrect as it describes the associated tumor marker elevation suggesting hepatocellular carcinoma (HCC). The development of HCC is usually associated with alcohol abuse, for which this patient denies, and is associated with abnormal weight loss and jaundice, both of which are absent in this presentation.

    References:
    1. Papadakis MA, McPhee SJ, Rabow MW, eds. CURRENT Medical Diagnosis & Treatment 2015. 54th ed. New York: McGraw-Hill Education; 2015.
    2. Powell LW, Isselbacher KJ. Hemochromatosis. In: Braunwald E, Hauser SL, Fauci AS, Longo DL, Kasper DL, Jameson JL, eds. Harrison’s Principles of Internal Medicine. 15th ed. New York: The McGraw-Hill Companies, Inc; 2001:2257-2261.
    3. Schrier SL, Bacon, BR. Clinical manifestations and diagnosis of hereditary hemochromatosis. UpToDate. https://www.uptodate.com/contents/clinical-manifestations-and-diagnosis-of-hereditary-hemochromatosis?source=search_result&search=hemochromatosis&selectedTitle=1~81. Updated December 9, 2016. Accessed November 18, 2017.
    4. Schrier SL, Bacon, BR. Management of patients with hereditary hemochromatosis. UpToDate. https://www.uptodate.com/contents/management-of-patients-with-hereditary-hemochromatosis?source=history_widget. Updated February 8, 2017. Accessed November 18, 2017.

  212. Question 212 of 220
    212. Question

    A 52 year-old female presents to her dermatologist for an acute rash. The rash has been present for about a week. It began on her anus and has since spread to her labia and her sacrum. Patient reports 6/10 burning pain in the areas of the rash and denies any associated symptoms, alleviating or aggravating factors. Patient denies any medical or surgical history and is up-to-date on immunizations. She recalls having chickenpox as a child. Patient reports a monogamous relationship with her husband of 12 years and denies sick contacts with similar symptoms, recent travel or use of any new medications, foods or products. Upon examination patient is afebrile without any palpable lymphadenopathy. Skin exam positive for multiple grouped vesicles on an erythematous base visualized on the left side of the anus, left posterior labium majus and the left side of the sacrum. These lesions are in various stages of healing and do not cross the midline. What is the most appropriate treatment for this patient?

    Correct

    This patient’s suspected diagnosis is Herpes Zoster Virus (HZV). This diagnosis is based primarily on clinical presentation. This patient’s description of a burning pain in the areas of the lesions is characteristic of HZV. Additionally, the description of grouped vesicles on an erythematous border that do not cross the midline is pathognomonic for HZV as the lesions present along the dermatomes.1 Laboratory studies may be helpful in supporting the diagnosis but are not necessary unless the diagnosis is uncertain due to an atypical presentation. A Tzanck smear is able to confirm that a lesion is herpetic but does not distinguish between the different types of herpes viruses. PCR assay of vesicular fluid is more sensitive and is able to differentiate between Herpes Simplex Viruses (HSV) and Herpes Zoster Virus. Skin biopsy of HZV characteristically shows histologic findings of multinucleated giant cells which is the same as HSV. If underlying leukocytoclastic vasculitis is present HZV may be differentiated from HSV.1
    A. Reassurance. These lesions are self limited and will resolve in 4 weeks- Untreated HZV may take 2 months or longer to resolve. HZV that is treated early will generally resolve by 3-4 weeks. It is important to treat HZV in order to reduce the incidence of complications like post-herpetic neuralgia.2
    B. Application of Mupirocin (Bactroban) topical ointment to affected areas BID until resolution of lesions- Mupirocin ointment is indicated for treatment of impetigo.3 Impetigo typically presents at the site of minor trauma (i.e. eczema, insect bite, HSV) and initially appear as vesicles or pustules that erupt and dry resulting in the characteristic honey-crusted lesions that readily spread to other areas. Impetigo is common on the face especially the nose and the mouth but may occur anywhere on the skin.3
    C. Application of Desoximetasone (Topicort) 0.25% topical steroid to affected areas BID for 4 weeks and weekly after resolution of lesions in order to prevent recurrence- The short term application of a topical corticosteroid may be indicated in some cases of HZV but should be used in combination with antiviral agents. High potency steroids are for short term use and should not be used longer than 2 weeks. It is important to counsel patients about the adverse effects of prolonged use of topical steroids including: telangiectasis, hypopigmentation, atrophy.1
    D. Valacyclovir (Valtrex) 1g PO QID for 7 days- this is the correct answer. Cases of suspected HZV should be treated with Valtrex in order to reduce healing time and prevent complications such as post herpetic neuralgia. Although it may be difficult to differentiate HZV from HSV the treatment is the same so it is important to begin therapy and not wait for laboratory results.1 Other antiviral treatments include Acyclovir (Zovirax) and Famciclovir (Famvir).1
    E. Administer a single dose of Zostavax 0.65mL IM once Zoster Virus is confirmed by biopsy of the lesion- Zostavax is a live attenuated virus vaccine that is administered for prevention of HZV in individuals older than 50. Zostavax is not used to treat acute HZV. Further, you would not want to delay treatment with an antiviral until laboratory results confirm the diagnosis.1

    Case resources:
    1. Cohen K, Salbu L, Israel I. Presentation and Management of Herpes Zoster (Shingles) in the Geriatric Population. BioMed Central Infectious Diseases. 2013. 38(4): 217-224, 227. PMCID: PMC3684190
    2. Pica F, Gatti A, Divizia M, et al. One-year Follow-up of Patients With Long-lasting Post-herpetic Neuralgia. BioMed Central Infectious Diseases. 2014. 14: 556. DOI: 10.1186/s12879-014-0556-6
    3. Cole C, Gazewood J. 2007. Diagnosis and Treatment of Impetigo. American Family Physician. 15;75(6)859-864. http://www.aafp.org/afp/2007/0315/p859.html

    Incorrect

    This patient’s suspected diagnosis is Herpes Zoster Virus (HZV). This diagnosis is based primarily on clinical presentation. This patient’s description of a burning pain in the areas of the lesions is characteristic of HZV. Additionally, the description of grouped vesicles on an erythematous border that do not cross the midline is pathognomonic for HZV as the lesions present along the dermatomes.1 Laboratory studies may be helpful in supporting the diagnosis but are not necessary unless the diagnosis is uncertain due to an atypical presentation. A Tzanck smear is able to confirm that a lesion is herpetic but does not distinguish between the different types of herpes viruses. PCR assay of vesicular fluid is more sensitive and is able to differentiate between Herpes Simplex Viruses (HSV) and Herpes Zoster Virus. Skin biopsy of HZV characteristically shows histologic findings of multinucleated giant cells which is the same as HSV. If underlying leukocytoclastic vasculitis is present HZV may be differentiated from HSV.1
    A. Reassurance. These lesions are self limited and will resolve in 4 weeks- Untreated HZV may take 2 months or longer to resolve. HZV that is treated early will generally resolve by 3-4 weeks. It is important to treat HZV in order to reduce the incidence of complications like post-herpetic neuralgia.2
    B. Application of Mupirocin (Bactroban) topical ointment to affected areas BID until resolution of lesions- Mupirocin ointment is indicated for treatment of impetigo.3 Impetigo typically presents at the site of minor trauma (i.e. eczema, insect bite, HSV) and initially appear as vesicles or pustules that erupt and dry resulting in the characteristic honey-crusted lesions that readily spread to other areas. Impetigo is common on the face especially the nose and the mouth but may occur anywhere on the skin.3
    C. Application of Desoximetasone (Topicort) 0.25% topical steroid to affected areas BID for 4 weeks and weekly after resolution of lesions in order to prevent recurrence- The short term application of a topical corticosteroid may be indicated in some cases of HZV but should be used in combination with antiviral agents. High potency steroids are for short term use and should not be used longer than 2 weeks. It is important to counsel patients about the adverse effects of prolonged use of topical steroids including: telangiectasis, hypopigmentation, atrophy.1
    D. Valacyclovir (Valtrex) 1g PO QID for 7 days- this is the correct answer. Cases of suspected HZV should be treated with Valtrex in order to reduce healing time and prevent complications such as post herpetic neuralgia. Although it may be difficult to differentiate HZV from HSV the treatment is the same so it is important to begin therapy and not wait for laboratory results.1 Other antiviral treatments include Acyclovir (Zovirax) and Famciclovir (Famvir).1
    E. Administer a single dose of Zostavax 0.65mL IM once Zoster Virus is confirmed by biopsy of the lesion- Zostavax is a live attenuated virus vaccine that is administered for prevention of HZV in individuals older than 50. Zostavax is not used to treat acute HZV. Further, you would not want to delay treatment with an antiviral until laboratory results confirm the diagnosis.1

    Case resources:
    1. Cohen K, Salbu L, Israel I. Presentation and Management of Herpes Zoster (Shingles) in the Geriatric Population. BioMed Central Infectious Diseases. 2013. 38(4): 217-224, 227. PMCID: PMC3684190
    2. Pica F, Gatti A, Divizia M, et al. One-year Follow-up of Patients With Long-lasting Post-herpetic Neuralgia. BioMed Central Infectious Diseases. 2014. 14: 556. DOI: 10.1186/s12879-014-0556-6
    3. Cole C, Gazewood J. 2007. Diagnosis and Treatment of Impetigo. American Family Physician. 15;75(6)859-864. http://www.aafp.org/afp/2007/0315/p859.html

  213. Question 213 of 220
    213. Question

    A 43 year old obese American male develops acute left lower quadrant pain that they first noticed while eating a double-cheeseburger. They describe it at the time as being a, 8/10 “sharp, stabbing” sensation for which they can identify no aggravating or alleviating factors. They report that 24 hours later it progressed to a 10/10 pain that includes associated non-bloody, non-mucosy diarrhea. PMH is positive for prostate cancer in remission, diverticulosis, and paroxysmal atrial fibrillation for which they have refused any anticoagulant. He endorses a 45 pack year smoking history. They are allergic only to bee stings, which cause anaphylaxis. On exam, they are diaphoretic with a fever of 101.8 degree F, have a BP of 108/74 RUE sitting, HR of 102 that is regular, have a non-distended but acutely tender LLQ of their abdomen with normactive bowel sounds, non-sunken eyes, and good capillary refill. Labs are remarkable for leukocytosis, a lipase under 350, a Cr of 0.77, and a glucose of 188. You order an abdominal ultrasound, but due to patient’s body habitus, it is not helpful. Abdominal CT confirms your suspected diagnosis and reveals localized bowel wall thickening of 6mm, an increase in soft tissue density within the pericolonic fat with associated fat stranding, and adjacent diverticula. There is no visible air-fluid levels, dilated loops of bowel, fistulas, or discernible necrosis on CT. They report that they can tolerate an oral diet. What is the most appropriate treatment for this patient?

    Correct

    The correct answer is E. This patient has a classic case of acute uncomplicated diverticulitis as evidenced by the CT findings. Because they can tolerate oral intake and have no complications of their diverticulitis, they should be treated on an outpatient basis with a one time IV loading dose of Cipro and metro and then sent home on PO meds to follow up with their primary care provider. There is no need for them to be placed on a special diet.
    Answer A is incorrect because the patient would only need to be admitted to the hospital if he had complications such as obstruction, perforation, or ileus. There is no evidence of dehydration as they have non-sunken eyes, good capillary refill, and an appropriate BUN and serum creatinine. Only consider hospitalization if complications or significant comorbities, which this patient does not have.
    Answer B is incorrect because it neglects to give the patient an IV loading dose of the antibiotics. Remember, outpatient treatment success was only measured in those who received an IV loading dose of the Cipro and metro in the emergency department (Laméris et al.). Answer B is also wrong because there is no need for them to be fasting as the patient has uncomplicated diverticulitis.
    Answer C is incorrect because the patient has no complications of their diverticulitis. They have no ileus as evidenced by intact bowel sounds and no air fluid levels on abdominal CT indicative of perforation. This patient should be managed medically and only referred to surgical services if they develop complications or fail to respond to medical treatment (Regenbogen et al.).
    Answer D is extremely tempting, but this is not a specialist level problem.

    Epidemiology:
    The epidemiology of diverticular disease is 1-2% in patients under 30 years of age (Parks TG) and as we age climbs to 66% of patients over the age of 80 year old (Hughes). Of the patients who developed diverticular disease, approximately 10-25% or patients globally will progress to acute episodes of diverticulitis (Hobson) and 4% of patients in the United States will have an acute episode of diverticulitis (Parks). Diverticular disease is more common in developed nations and is most prevalent in nations which have low dietary fiber intake, high red meat consumption, and an aging population.
    Etiology:
    Low dietary fiber has been associated with higher rates of diverticular disease. Additionally, patients who eat a diet that is high in red meat show an increased rate of diverticular disease when compared with patients who consume a vegetarian diet (Aldoori et al). Contrary to popular belief, nut, corn, and popcorn consumption has not been shown to correlate with increased rates of diverticular disease (Strate et al.). The single most common cause of diverticulitis is prior diverticular disease, such as chronic diverticulosis. Physical activity and regular exercise has also been shown to decrease the risk of diverticulitis, as well as GI bleeding in those who do develop diverticulitis. Diverticulitis has been shown to be more common in older patients, with the mean age of admission for acute diverticulitis being 63 years old. Lastly, obesity and a smoking history have both been shown to be risk factors for the development of diverticular disease.
    Pathophysiology:
    Diverticular disease typically emerges from areas of weakness within the gastrointestinal tract. Colonic diverticulum occurs when both the mucosa and submucosa herniate through the musculature and are covered only by the serosa. This typically occurs secondary to abnormal colonic motility from increased intraluminal pressure leading to herniation. The chronic diverticular disease that forms as a result of these herniations progresses to an acute episode of diverticulitis when a perforation of the diverticulum occurs, often due to fecalith obstruction or erosion of the diverticular wall by sustained increases in intraluminal pressure. These perforations lead to the acute abdominal pain in the region of the perforated segment and typically a fever as well. Diverticulitis can also cause the formation of fistulas between the colon and nearby viscera as a result of the significant inflammation.
    Pertinent historical and physical findings:
    The clinical manifestation of diverticulitis varies depending upon the severity of the inflammation and perforation that the patient is experiencing. The most common presenting complaint for a patient with diverticulitis is abdominal pain. In Western nations, this abdominal pain is most commonly located in the left lower quadrant. However, in sigmoid colitis and in patients of Asian ancestry, right lower quadrant pain is common. Nausea and emesis are commonly presenting symptoms as well and are more likely to be seen in patients who have bowel obstruction or ileus. A near universal symptom is fever. However, as not everyone’s body reads Tintinalli’s before presenting to the emergency department, providers cannot rule out diverticulitis simply due to the absence of a fever. Approximately half of patients with diverticulitis will also present with constipation and approximately one third will present with diarrhea. Despite having recent changes in bowel movements, other bowel abnormalities such as hematochezia are rare. Up to 15% of patients with sigmoid diverticulitis will also present with urinary symptoms such as dysuria, increased urinary frequency, and suprapubic discomfort due to the inflamed sigmoid colon pressing against the patient’s bladder. Approximately 1-2% of patients will develop a perforation secondary to diverticulitis and present with a distended abdomen, fecal peritonitis, ileus, and absent bowel sounds. Although this complication is rare, the mortality rate is approximately 20% so it should be considered a life threatening emergency.
    Diagnostics:
    Patients with diverticulitis may present with leukocytosis on their CBC. However, 45% of patients will have a normal white count (Ambrosetti et al.). Sterile pyuria is possible in patients with urinary symptoms from nearby inflammation of the sigmoid colon. Colonic flora in a urinalysis would be associated with a colovesicular fistula.
    Abdominal ultrasound can be utilized to assess for complications of diverticulitis. Most notably, ultrasound would show hypoechoic peridiverticular inflammation, abscess formation, bowel wall thickening, and would be able to identify non-inflamed diverticula in adjacent segments. High resolution compression ultrasonography has similar sensitivity to abdominal CT for diagnosing diverticulitis (Laméris et al.). Its advantages are that is it widely available in emergency departments and exposes the patient to no radiation. However, its sensitivity has been shown to be highly user-dependent. Due to this wide variance in sensitivity, abdominal CT remains the most commonly used imaging modality for the assessment of diverticulitis. The sensitivity of abdominal CT for diagnosing diverticulitis is 94% and has a specificity of 99% (Laméris et al.). Abdominal CT is also accurate for assessing for diverticulitis complications such as perforation, air-fluid levels with peritonitis, or necrosis. One additional imaging modality would be MRI. While MRI has the advantage of not exposing the patient to any radiation, it is not widely available to all emergency departments and there are no studies assessing the sensitivity, specificity, or cost-effectiveness of MRI against the current standard, abdominal CT. As such, more cost effective and proven imaging modalities would be preferred over MRI.
    Treatment:
    Diverticulitis can be treated non-operatively on both an inpatient and outpatient basis. In patients who received an intravenous loading dose of antibiotics in the emergency department for acute uncomplicated diverticulitis, there is no difference in outcomes between those who are admitted inpatient to the hospital versus those who are discharged home on oral antibiotics with instructions to follow up with their primary care providers (Biondo et al.). Hospital admission should be strongly considered in patients with complications such as perforation, abscess, obstruction, fistulas, sepsis, immunosuppression, a fever over 102.5 degrees F, diffuse peritonitis, advanced age, needing intravenous hydration and nutrition, or a previous failure of outpatient treatment. Absent any of the previous inpatient criteria, patients should be discharged home on oral antibiotics.
    The mainstay of treatment for acute diverticulitis is ciprofloxacin 500mg BID combined with metronidazole 500mg TID for ten days. In areas where the local biogram shows ten percent of greater resistance of E. Coli to fluoroquinolones, ciprofloxacin should not be used as it will be an ineffective treatment. Medical providers should review the black box warning of tendon strain and rupture on all fluoroquinolones with their patients prior to prescribing them. If patients cannot tolerate this first line treatment, they should be prescribed Bactrim DS BID combined with metronidazole 500mg TID. In the event that a patient cannot tolerate either of the above regiments, then Augmentin 875mg TID can be considered as a tertiary treatment. If a patient is admitted inpatient due to complications or comorbidities, they are typically placed on bowel rest, given intravenous fluids, a clear liquid diet, and given intravenous pain medications such as meperidine or hydromorphone. Admitted patients would be assessed daily to ensure adequate progression of treatment. Despite being commonly prescribed, there is no evidence that a clear liquid diet has any benefit in the treatment of acute uncomplicated diverticulitis (Biondo et al.). Surgical consultation should be completed in any patient who presents with complications of diverticulitis, such as perforation, abscess, or obstruction or in any patient who fails to respond to medical management.  

    References
    Aldoori, W H, et al. “A prospective study of diet and the risk of symptomatic diverticular disease in men.” The American journal of clinical nutrition., U.S. National Library of Medicine, Nov. 1994, http://www.ncbi.nlm.nih.gov/pubmed?term=7942584.
    Ambrosetti, P, et al. “Acute left colonic diverticulitis: a prospective analysis of 226 consecutive cases.” Surgery., U.S. National Library of Medicine, May 1994, http://www.ncbi.nlm.nih.gov/pubmed?term=8178252.
    Biondo, S, et al. “Outpatient versus hospitalization management for uncomplicated diverticulitis: a prospective, multicenter randomized clinical trial (DIVER Trial).” Annals of surgery., U.S. National Library of Medicine, Jan. 2014, http://www.ncbi.nlm.nih.gov/pubmed?term=23732265.
    Hobson, K G, and P L Roberts. “Etiology and pathophysiology of diverticular disease.” Clinics in colon and rectal surgery., U.S. National Library of Medicine, Aug. 2004, http://www.ncbi.nlm.nih.gov/pubmed/20011269/.
    Hughes, L E. “Postmortem survey of diverticular disease of the colon. I. Diverticulosis and diverticulitis.” Gut., U.S. National Library of Medicine, May 1969, http://www.ncbi.nlm.nih.gov/pubmed/5771665/.
    Laméris, W, et al. “Graded compression ultrasonography and computed tomography in acute colonic diverticulitis: meta-Analysis of test accuracy.” European radiology., U.S. National Library of Medicine, Nov. 2008, http://www.ncbi.nlm.nih.gov/pubmed?term=18523784.
    Matrana, Marc R., and David A. Margolin. “Epidemiology and Pathophysiology of Diverticular Disease.” Clinics in Colon and Rectal Surgery, Thieme Medical Publishers, Aug. 2009, http://www.ncbi.nlm.nih.gov/pmc/articles/PMC2780269/.
    Parks, T G. “Natural history of diverticular disease of the colon. A review of 521 cases.” British medical journal., U.S. National Library of Medicine, 13 Dec. 1969, http://www.ncbi.nlm.nih.gov/pubmed/5359917.
    Regenbogen, S E, et al. “Surgery for diverticulitis in the 21st century: a systematic review.” JAMA surgery., U.S. National Library of Medicine, Mar. 2014, http://www.ncbi.nlm.nih.gov/pubmed?term=24430164.
    Strate, L L, et al. “Nut, corn, and popcorn consumption and the incidence of diverticular disease.” JAMA., U.S. National Library of Medicine, 27 Aug. 2008, http://www.ncbi.nlm.nih.gov/pubmed?term=18728264.
    Vennix, S, et al. “Systematic review of evidence and consensus on diverticulitis: an analysis of national and international guidelines.” Colorectal disease : the official journal of the Association of Coloproctology of Great Britain and Ireland., U.S. National Library of Medicine, Nov. 2014, http://www.ncbi.nlm.nih.gov/pubmed?term=24801825.

     
    References

    Laméris, W, et al. “Graded compression ultrasonography and computed tomography in acute colonic diverticulitis: meta-Analysis of test accuracy.” European radiology., U.S. National Library of Medicine, Nov. 2008, http://www.ncbi.nlm.nih.gov/pubmed?term=18523784.
    Regenbogen, S E, et al. “Surgery for diverticulitis in the 21st century: a systematic review.” JAMA surgery., U.S. National Library of Medicine, Mar. 2014, http://www.ncbi.nlm.nih.gov/pubmed?term=24430164.

    Incorrect

    The correct answer is E. This patient has a classic case of acute uncomplicated diverticulitis as evidenced by the CT findings. Because they can tolerate oral intake and have no complications of their diverticulitis, they should be treated on an outpatient basis with a one time IV loading dose of Cipro and metro and then sent home on PO meds to follow up with their primary care provider. There is no need for them to be placed on a special diet.
    Answer A is incorrect because the patient would only need to be admitted to the hospital if he had complications such as obstruction, perforation, or ileus. There is no evidence of dehydration as they have non-sunken eyes, good capillary refill, and an appropriate BUN and serum creatinine. Only consider hospitalization if complications or significant comorbities, which this patient does not have.
    Answer B is incorrect because it neglects to give the patient an IV loading dose of the antibiotics. Remember, outpatient treatment success was only measured in those who received an IV loading dose of the Cipro and metro in the emergency department (Laméris et al.). Answer B is also wrong because there is no need for them to be fasting as the patient has uncomplicated diverticulitis.
    Answer C is incorrect because the patient has no complications of their diverticulitis. They have no ileus as evidenced by intact bowel sounds and no air fluid levels on abdominal CT indicative of perforation. This patient should be managed medically and only referred to surgical services if they develop complications or fail to respond to medical treatment (Regenbogen et al.).
    Answer D is extremely tempting, but this is not a specialist level problem.

    Epidemiology:
    The epidemiology of diverticular disease is 1-2% in patients under 30 years of age (Parks TG) and as we age climbs to 66% of patients over the age of 80 year old (Hughes). Of the patients who developed diverticular disease, approximately 10-25% or patients globally will progress to acute episodes of diverticulitis (Hobson) and 4% of patients in the United States will have an acute episode of diverticulitis (Parks). Diverticular disease is more common in developed nations and is most prevalent in nations which have low dietary fiber intake, high red meat consumption, and an aging population.
    Etiology:
    Low dietary fiber has been associated with higher rates of diverticular disease. Additionally, patients who eat a diet that is high in red meat show an increased rate of diverticular disease when compared with patients who consume a vegetarian diet (Aldoori et al). Contrary to popular belief, nut, corn, and popcorn consumption has not been shown to correlate with increased rates of diverticular disease (Strate et al.). The single most common cause of diverticulitis is prior diverticular disease, such as chronic diverticulosis. Physical activity and regular exercise has also been shown to decrease the risk of diverticulitis, as well as GI bleeding in those who do develop diverticulitis. Diverticulitis has been shown to be more common in older patients, with the mean age of admission for acute diverticulitis being 63 years old. Lastly, obesity and a smoking history have both been shown to be risk factors for the development of diverticular disease.
    Pathophysiology:
    Diverticular disease typically emerges from areas of weakness within the gastrointestinal tract. Colonic diverticulum occurs when both the mucosa and submucosa herniate through the musculature and are covered only by the serosa. This typically occurs secondary to abnormal colonic motility from increased intraluminal pressure leading to herniation. The chronic diverticular disease that forms as a result of these herniations progresses to an acute episode of diverticulitis when a perforation of the diverticulum occurs, often due to fecalith obstruction or erosion of the diverticular wall by sustained increases in intraluminal pressure. These perforations lead to the acute abdominal pain in the region of the perforated segment and typically a fever as well. Diverticulitis can also cause the formation of fistulas between the colon and nearby viscera as a result of the significant inflammation.
    Pertinent historical and physical findings:
    The clinical manifestation of diverticulitis varies depending upon the severity of the inflammation and perforation that the patient is experiencing. The most common presenting complaint for a patient with diverticulitis is abdominal pain. In Western nations, this abdominal pain is most commonly located in the left lower quadrant. However, in sigmoid colitis and in patients of Asian ancestry, right lower quadrant pain is common. Nausea and emesis are commonly presenting symptoms as well and are more likely to be seen in patients who have bowel obstruction or ileus. A near universal symptom is fever. However, as not everyone’s body reads Tintinalli’s before presenting to the emergency department, providers cannot rule out diverticulitis simply due to the absence of a fever. Approximately half of patients with diverticulitis will also present with constipation and approximately one third will present with diarrhea. Despite having recent changes in bowel movements, other bowel abnormalities such as hematochezia are rare. Up to 15% of patients with sigmoid diverticulitis will also present with urinary symptoms such as dysuria, increased urinary frequency, and suprapubic discomfort due to the inflamed sigmoid colon pressing against the patient’s bladder. Approximately 1-2% of patients will develop a perforation secondary to diverticulitis and present with a distended abdomen, fecal peritonitis, ileus, and absent bowel sounds. Although this complication is rare, the mortality rate is approximately 20% so it should be considered a life threatening emergency.
    Diagnostics:
    Patients with diverticulitis may present with leukocytosis on their CBC. However, 45% of patients will have a normal white count (Ambrosetti et al.). Sterile pyuria is possible in patients with urinary symptoms from nearby inflammation of the sigmoid colon. Colonic flora in a urinalysis would be associated with a colovesicular fistula.
    Abdominal ultrasound can be utilized to assess for complications of diverticulitis. Most notably, ultrasound would show hypoechoic peridiverticular inflammation, abscess formation, bowel wall thickening, and would be able to identify non-inflamed diverticula in adjacent segments. High resolution compression ultrasonography has similar sensitivity to abdominal CT for diagnosing diverticulitis (Laméris et al.). Its advantages are that is it widely available in emergency departments and exposes the patient to no radiation. However, its sensitivity has been shown to be highly user-dependent. Due to this wide variance in sensitivity, abdominal CT remains the most commonly used imaging modality for the assessment of diverticulitis. The sensitivity of abdominal CT for diagnosing diverticulitis is 94% and has a specificity of 99% (Laméris et al.). Abdominal CT is also accurate for assessing for diverticulitis complications such as perforation, air-fluid levels with peritonitis, or necrosis. One additional imaging modality would be MRI. While MRI has the advantage of not exposing the patient to any radiation, it is not widely available to all emergency departments and there are no studies assessing the sensitivity, specificity, or cost-effectiveness of MRI against the current standard, abdominal CT. As such, more cost effective and proven imaging modalities would be preferred over MRI.
    Treatment:
    Diverticulitis can be treated non-operatively on both an inpatient and outpatient basis. In patients who received an intravenous loading dose of antibiotics in the emergency department for acute uncomplicated diverticulitis, there is no difference in outcomes between those who are admitted inpatient to the hospital versus those who are discharged home on oral antibiotics with instructions to follow up with their primary care providers (Biondo et al.). Hospital admission should be strongly considered in patients with complications such as perforation, abscess, obstruction, fistulas, sepsis, immunosuppression, a fever over 102.5 degrees F, diffuse peritonitis, advanced age, needing intravenous hydration and nutrition, or a previous failure of outpatient treatment. Absent any of the previous inpatient criteria, patients should be discharged home on oral antibiotics.
    The mainstay of treatment for acute diverticulitis is ciprofloxacin 500mg BID combined with metronidazole 500mg TID for ten days. In areas where the local biogram shows ten percent of greater resistance of E. Coli to fluoroquinolones, ciprofloxacin should not be used as it will be an ineffective treatment. Medical providers should review the black box warning of tendon strain and rupture on all fluoroquinolones with their patients prior to prescribing them. If patients cannot tolerate this first line treatment, they should be prescribed Bactrim DS BID combined with metronidazole 500mg TID. In the event that a patient cannot tolerate either of the above regiments, then Augmentin 875mg TID can be considered as a tertiary treatment. If a patient is admitted inpatient due to complications or comorbidities, they are typically placed on bowel rest, given intravenous fluids, a clear liquid diet, and given intravenous pain medications such as meperidine or hydromorphone. Admitted patients would be assessed daily to ensure adequate progression of treatment. Despite being commonly prescribed, there is no evidence that a clear liquid diet has any benefit in the treatment of acute uncomplicated diverticulitis (Biondo et al.). Surgical consultation should be completed in any patient who presents with complications of diverticulitis, such as perforation, abscess, or obstruction or in any patient who fails to respond to medical management.  

    References
    Aldoori, W H, et al. “A prospective study of diet and the risk of symptomatic diverticular disease in men.” The American journal of clinical nutrition., U.S. National Library of Medicine, Nov. 1994, http://www.ncbi.nlm.nih.gov/pubmed?term=7942584.
    Ambrosetti, P, et al. “Acute left colonic diverticulitis: a prospective analysis of 226 consecutive cases.” Surgery., U.S. National Library of Medicine, May 1994, http://www.ncbi.nlm.nih.gov/pubmed?term=8178252.
    Biondo, S, et al. “Outpatient versus hospitalization management for uncomplicated diverticulitis: a prospective, multicenter randomized clinical trial (DIVER Trial).” Annals of surgery., U.S. National Library of Medicine, Jan. 2014, http://www.ncbi.nlm.nih.gov/pubmed?term=23732265.
    Hobson, K G, and P L Roberts. “Etiology and pathophysiology of diverticular disease.” Clinics in colon and rectal surgery., U.S. National Library of Medicine, Aug. 2004, http://www.ncbi.nlm.nih.gov/pubmed/20011269/.
    Hughes, L E. “Postmortem survey of diverticular disease of the colon. I. Diverticulosis and diverticulitis.” Gut., U.S. National Library of Medicine, May 1969, http://www.ncbi.nlm.nih.gov/pubmed/5771665/.
    Laméris, W, et al. “Graded compression ultrasonography and computed tomography in acute colonic diverticulitis: meta-Analysis of test accuracy.” European radiology., U.S. National Library of Medicine, Nov. 2008, http://www.ncbi.nlm.nih.gov/pubmed?term=18523784.
    Matrana, Marc R., and David A. Margolin. “Epidemiology and Pathophysiology of Diverticular Disease.” Clinics in Colon and Rectal Surgery, Thieme Medical Publishers, Aug. 2009, http://www.ncbi.nlm.nih.gov/pmc/articles/PMC2780269/.
    Parks, T G. “Natural history of diverticular disease of the colon. A review of 521 cases.” British medical journal., U.S. National Library of Medicine, 13 Dec. 1969, http://www.ncbi.nlm.nih.gov/pubmed/5359917.
    Regenbogen, S E, et al. “Surgery for diverticulitis in the 21st century: a systematic review.” JAMA surgery., U.S. National Library of Medicine, Mar. 2014, http://www.ncbi.nlm.nih.gov/pubmed?term=24430164.
    Strate, L L, et al. “Nut, corn, and popcorn consumption and the incidence of diverticular disease.” JAMA., U.S. National Library of Medicine, 27 Aug. 2008, http://www.ncbi.nlm.nih.gov/pubmed?term=18728264.
    Vennix, S, et al. “Systematic review of evidence and consensus on diverticulitis: an analysis of national and international guidelines.” Colorectal disease : the official journal of the Association of Coloproctology of Great Britain and Ireland., U.S. National Library of Medicine, Nov. 2014, http://www.ncbi.nlm.nih.gov/pubmed?term=24801825.

     
    References

    Laméris, W, et al. “Graded compression ultrasonography and computed tomography in acute colonic diverticulitis: meta-Analysis of test accuracy.” European radiology., U.S. National Library of Medicine, Nov. 2008, http://www.ncbi.nlm.nih.gov/pubmed?term=18523784.
    Regenbogen, S E, et al. “Surgery for diverticulitis in the 21st century: a systematic review.” JAMA surgery., U.S. National Library of Medicine, Mar. 2014, http://www.ncbi.nlm.nih.gov/pubmed?term=24430164.

  214. Question 214 of 220
    214. Question

    A 62-year-old female with a history of migraines and depression presents with a chief complaint of headaches for the last month. She describes the headaches as throbbing and intermittent in nature and surround her entire head. She states that there have been multiple times where the headaches have woken her up at night. She states she had recently gotten over a cold but does not feel ill currently. She denies fever, chills, weakness, paresthesias, cough, new medications. Which historical feature of this patient is concerning for possible organic underlying cause of the headaches?

    Correct

    A – History of migraines and depression would not point in the direction of an organic cause of the headaches and may support an alternative diagnosis
    B – Most headaches related to an underlying tumor are constant, dull, or achy
    C – Recent illness is not an important historical factor in this vignette. Underlying deep sinus infection may be on a differential diagnosis if the patient was still sick, febrile, or complaining of sinus symptoms
    D – Headaches that may cause a patient to wake up at night is a red flag and should prompt further evaluation
    E – A tension-type headache may be described in a similar way making this a less important historical feature.

    Discussion:
    Glioblastomas are a primary brain tumor of the brain that arise from the glial cells. Specifically, glioblastomas arise from the astrocytes in the brain which surround and line nerve cells.5 Unfortunately, these tumors are very aggressive and can manifest without early detection or histopathological evidence of a lesser grade tumor. Glioblastomas usually involve the cerebral hemispheres with rare case reports involving the brainstem or the spinal cord.
    Unfortunately, these invasive tumors are the most common among the adult population. 60 % of primary brain tumors are high grade glioblastomas. These tumors have a male to female ratio of about 3:2 and are slightly more common in the white population. The median age at diagnosis is 64 years old and about ¾ of these tumors will be diagnosed between the ages of 45-70.1 There are no real predisposing risk factors for primary malignant brain tumors. The only exception would be for people with rare genetic syndromes similar to a neurofibromatosis.
    Glioblastomas differentiate themselves from other gliomas due to the very aggressive and rapid manor in which they manifest. They develop de novo and have no precursor lesions. Typically, they will expand and manifest clinically as early as 3 months. This may be in part due to glioblastomas containing the most amount of genetic mutations in comparison to other primary brain tumors.1 More research is being performed to better understand the development of these tumors.
    Although patients presenting with a headache are unlikely to have a tumor as a cause, headaches are a common presentation in patients with Glioblastomas.2 These headaches have been described as constant, dull and achy which at times may be more severe than usual migraine headaches. Classically, headaches related to brain tumors may wake the patient up at night. This is thought to be in relation to intracranial CO2 and pressure at night.1 Another consideration is headaches that are different in pattern or in quality in a patient who has a history of headaches.
    More progressive symptoms may include headaches that are associated with early morning nausea and vomiting as well as change in sleeping patterns. Many people will present with the first clinical manifestation as a seizure and may affect a total of 50-80 % of people with primary brain tumors.1 Less likely, but equally concerning symptoms for a brain tumor may include focal neurological symptoms such as weakness, loss of sensation, aphasia, or visual spatial dysfunction.
    Neuroimaging is performed to help differentiate brain tumors. MRI with and without contrast and enhancement can be used to help identify certain features of tumors that likely indicate a specific diagnosis. Glioblastomas typically appear as hypo intense on a T2 weighted image and may have a ring of surrounding enhancement.2 The best way to differentiate the type of tumor is directly through biopsy. Since it is very likely that a tumor is required to be removed, biopsies are usually performed directly through excision of the tumor rather than a needle biopsy. Histopathologic evaluation of glioblastoma multiforme may be variable but are typically composed of poorly differentiated astrocytes with significant nuclear atypia.4
    The mortality rate of Glioblastomas remains high unfortunately regardless of treatment strategies. The typical course of treatment includes maximal surgical resection with radiation and or chemotherapy. Most patient presenting are good surgical candidates. In some cases with older patients or extensive disease, surgery is deferred. Temozolomide is an oral chemotherapy drug in which studies have shown to prolong the survival in patients with Glioblastoma making it a common selection for treatment.2
    Other aspects of treatment include lowering the intracranial pressure if indicated which is usually with oral glucocorticoids such as dexamethasone. This is the reduce cerebral edema and the risk of herniation.3 Treatment will also typically consist of anticonvulsant medication such as Keppra.
    There are different molecular alterations and mutations that may be tested for that may suggest more favorable or poorer prognosis that may be tested, although they are not likely to change the course of treatment.3 Even through surgical resection and extensive radiation and chemotherapy, prognosis remains unfavorable for these tumors with an average survival after diagnosis of less than 1 year.

     
    References

    Batchelor, MD. Clinical manifestations and initial surgical approach to patients with high-grade gliomas. Post TW, (ed) UpToDate. Waltham, MA: UpToDate Inc. http://www.uptodate.com (Accessed on December 3, 2017.)1
    Bruce J. Glioblastoma Multiforme. Practice Essentials, Background, Pathophysiology. https://emedicine.medscape.com/article/283252-overview. Published October 31, 2017. Accessed December 3, 2017.2
    Glioma. Mayo Clinic. https://www.mayoclinic.org/diseases-conditions/glioma/symptoms-causes/syc-20350251. Published August 10, 2017. Accessed December 3, 2017.3
    Louis, MD. Classification and pathologic diagnosis of gliomas. Post TW, (ed) UpToDate. Waltham, MA: UpToDate Inc. http://www.uptodate.com (Accessed on December 3, 2017.)4
    Papadakis MA. Current Medical Diagnosis and Treatment. McGraw-Hill Education / Medical; 2016.5

    Incorrect

    A – History of migraines and depression would not point in the direction of an organic cause of the headaches and may support an alternative diagnosis
    B – Most headaches related to an underlying tumor are constant, dull, or achy
    C – Recent illness is not an important historical factor in this vignette. Underlying deep sinus infection may be on a differential diagnosis if the patient was still sick, febrile, or complaining of sinus symptoms
    D – Headaches that may cause a patient to wake up at night is a red flag and should prompt further evaluation
    E – A tension-type headache may be described in a similar way making this a less important historical feature.

    Discussion:
    Glioblastomas are a primary brain tumor of the brain that arise from the glial cells. Specifically, glioblastomas arise from the astrocytes in the brain which surround and line nerve cells.5 Unfortunately, these tumors are very aggressive and can manifest without early detection or histopathological evidence of a lesser grade tumor. Glioblastomas usually involve the cerebral hemispheres with rare case reports involving the brainstem or the spinal cord.
    Unfortunately, these invasive tumors are the most common among the adult population. 60 % of primary brain tumors are high grade glioblastomas. These tumors have a male to female ratio of about 3:2 and are slightly more common in the white population. The median age at diagnosis is 64 years old and about ¾ of these tumors will be diagnosed between the ages of 45-70.1 There are no real predisposing risk factors for primary malignant brain tumors. The only exception would be for people with rare genetic syndromes similar to a neurofibromatosis.
    Glioblastomas differentiate themselves from other gliomas due to the very aggressive and rapid manor in which they manifest. They develop de novo and have no precursor lesions. Typically, they will expand and manifest clinically as early as 3 months. This may be in part due to glioblastomas containing the most amount of genetic mutations in comparison to other primary brain tumors.1 More research is being performed to better understand the development of these tumors.
    Although patients presenting with a headache are unlikely to have a tumor as a cause, headaches are a common presentation in patients with Glioblastomas.2 These headaches have been described as constant, dull and achy which at times may be more severe than usual migraine headaches. Classically, headaches related to brain tumors may wake the patient up at night. This is thought to be in relation to intracranial CO2 and pressure at night.1 Another consideration is headaches that are different in pattern or in quality in a patient who has a history of headaches.
    More progressive symptoms may include headaches that are associated with early morning nausea and vomiting as well as change in sleeping patterns. Many people will present with the first clinical manifestation as a seizure and may affect a total of 50-80 % of people with primary brain tumors.1 Less likely, but equally concerning symptoms for a brain tumor may include focal neurological symptoms such as weakness, loss of sensation, aphasia, or visual spatial dysfunction.
    Neuroimaging is performed to help differentiate brain tumors. MRI with and without contrast and enhancement can be used to help identify certain features of tumors that likely indicate a specific diagnosis. Glioblastomas typically appear as hypo intense on a T2 weighted image and may have a ring of surrounding enhancement.2 The best way to differentiate the type of tumor is directly through biopsy. Since it is very likely that a tumor is required to be removed, biopsies are usually performed directly through excision of the tumor rather than a needle biopsy. Histopathologic evaluation of glioblastoma multiforme may be variable but are typically composed of poorly differentiated astrocytes with significant nuclear atypia.4
    The mortality rate of Glioblastomas remains high unfortunately regardless of treatment strategies. The typical course of treatment includes maximal surgical resection with radiation and or chemotherapy. Most patient presenting are good surgical candidates. In some cases with older patients or extensive disease, surgery is deferred. Temozolomide is an oral chemotherapy drug in which studies have shown to prolong the survival in patients with Glioblastoma making it a common selection for treatment.2
    Other aspects of treatment include lowering the intracranial pressure if indicated which is usually with oral glucocorticoids such as dexamethasone. This is the reduce cerebral edema and the risk of herniation.3 Treatment will also typically consist of anticonvulsant medication such as Keppra.
    There are different molecular alterations and mutations that may be tested for that may suggest more favorable or poorer prognosis that may be tested, although they are not likely to change the course of treatment.3 Even through surgical resection and extensive radiation and chemotherapy, prognosis remains unfavorable for these tumors with an average survival after diagnosis of less than 1 year.

     
    References

    Batchelor, MD. Clinical manifestations and initial surgical approach to patients with high-grade gliomas. Post TW, (ed) UpToDate. Waltham, MA: UpToDate Inc. http://www.uptodate.com (Accessed on December 3, 2017.)1
    Bruce J. Glioblastoma Multiforme. Practice Essentials, Background, Pathophysiology. https://emedicine.medscape.com/article/283252-overview. Published October 31, 2017. Accessed December 3, 2017.2
    Glioma. Mayo Clinic. https://www.mayoclinic.org/diseases-conditions/glioma/symptoms-causes/syc-20350251. Published August 10, 2017. Accessed December 3, 2017.3
    Louis, MD. Classification and pathologic diagnosis of gliomas. Post TW, (ed) UpToDate. Waltham, MA: UpToDate Inc. http://www.uptodate.com (Accessed on December 3, 2017.)4
    Papadakis MA. Current Medical Diagnosis and Treatment. McGraw-Hill Education / Medical; 2016.5

  215. Question 215 of 220
    215. Question

    A 58 year old male presents to the emergency department with 2 hours of substernal chest pain that occurred after walking home from work. He states that he has had similar pain in the past, but it typically goes away with rest and this time it lasted longer than it ever has. He additionally notes shortness of breath, which prompted him to come in for evaluation. He denies a history of MI in the past, but states that he thinks he has “blood pressure and high cholesterol” but doesn’t like taking medication. Initial EKG shows normal sinus rhythm with non-specific T wave inversions, and cardiac enzymes are negative. He is currently chest pain free after being administered nitroglycerin and aspirin in the ED and wants to know when he can go home. What is the next step in management of this patient?

    Correct

    The correct answer is C. This patient is presenting with unstable angina, and should be closely monitored on telemetry to make sure he does not progress into worsening ACS. The patient cannot be sent home, as he needs to undergo serial troponins and is not currently in a stable position to go home even though he is without chest pain currently. While a cardiac stress test is a useful diagnostic in evaluating angina, it should not be done when acute MI has definitively been ruled out as there is a risk of sudden death. Stress tests do not need to be done immediately, and are of more use once patient is stabilized to determine underlying ischemic changes in the heart. Lastly, this patient is not currently showing signs of ischemic occlusion as seen in STEMI or significant NSTEMI presentations, and intervention with catheterization is not indicated. As the patient is monitored in the hospital, if cardiac enzymes increase or ischemic changes develop on the EKG, then a cardiac catheterization can be considered.

    Chest pain is a nonspecific symptom that can have many cardiac or non cardiac causes. Acute coronary syndrome is a common clinical presentation of chest pain and is sub-divided intro three categories: unstable angina, non-ST elevation myocardial infarction and ST-elevation myocardial infarction. Unstable angina is diagnosed when a patient presents with ischemic symptoms suggestive of ACS with no elevated cardiac enzymes, and with or without EKG changes indicating ischemia. It can take up to 12 hours for troponin to elevate and be detectable, so initially many patients are admitted to the hospital in order to appropriately determine if there is an NSTEMI present. There can be ST segment and/or T wave changes in unstable angina, but unlike NSTEMI presentations, they are typically transient and not long-lasting. The incidence of unstable angina is increasing every year, and there are 1 million patients hospitalized annually with the primary diagnosis of unstable angina. Many cases of unstable angina also occur outside of the hospital, with patients not seeking care, so this number is likely much higher.
    There are different classifications for unstable angina. One is any new onset angina that occurs with minimal exercise or at rest, and this comes with a worse prognosis than chronic stable angina. Any form of rest angina is classified as unstable. Early post-MI angina is also classified as unstable and correlates with more complex coronary artery disease which may lead to further episodes of AMI. Additionally, patients are classified as having unstable angina in the absence of significant coronary disease on cardiac catheterization (similar to the presentation of my patient above). Most of these patients will undergo catheterization for suspected NSTEMI and at that time there will not be significant disease noted, which carries a better prognosis. In the case presented above, although NSTEMI was unlikely given EKG findings, she was still brought to the cath lab as it was decided another non-invasive testing would not give suitable information as she had recently undergone evaluation, and it was desired by her attending physician to be certain of presence of CAD/rule out acute MI.
    The underlying etiology and pathophysiology of unstable angina is rather broad. Risk factors that predispose someone to develop this disease include hypertension, diabetes, hyperlipidemia, coronary artery disease, peripheral artery disease, family history of coronary artery disease, and obesity. There factors involved in the pathophysiology of this cardiac presentation include supply-demand mismatch, plaque disruption, thrombosis, and/or vasoconstriction. Supply-demand mismatch occurs when the body is requiring more oxygen than what the heart can provide. This typically occurs when there is an increased demand in cases like fever, exercise, tachycardia, etc. Plaque disruption can occur when there is an accumulation of LDL foam cells in the arteries which are unstable and prone to rupture, resulting in either transient or long-lasting ischemia.
    The signs and symptoms of unstable angina are also exhibited in patient’s with a myocardial infarction. They can include chest pain or pressure, sweating, dyspnea, nausea/vomiting, dizziness and fatigue. Of note, these symptoms are usually occurring at rest when angina becomes unstable or become more severe/frequent than before. Additionally, patients note that their symptoms do not follow their usual pattern and do not respond to nitroglycerin or rest as they previously did. Typically the physical examination is unremarkable but tachycardia, diaphoresis, signs of PAD and/or transient signs of myocardial dysfunction can be seen. At presentation, patients should get a 12-lead EKG and the following blood work completed: cardiac biomarkers, CBC, CMP and lipid panel. Additional diagnostics that are commonly used for assessment of patients with suspected unstable angina include chest x-ray, echocardiogram, CT or MRI angiography, and cardiac stress tests.
    Patients who present with unstable angina are admitted to the hospital as the course is highly variable, and more pronounced acute coronary syndrome (NSTEMI or STEMI) or life threatening arrhythmias could potentially occur. Patients are monitored on telemetry, given supplemental oxygen, and started on either medical management or undergo invasive treatment. Patient’s with a significant history of CAD and other risk factors may undergo invasive intervention with PCI if attending physician believes further evaluation is needed. Medical management can include dual antiplatelet treatment with aspirin and clopidigrel, statins, beta blockers, anticoagulation during hospitalization, angina nitrates, and angiotensin-converting enzyme inhibitors.
    Prognosis for patient’s presenting with unstable angina varies widely depending on concomitant risk factors and underlying medical conditions. When there are EKG changes notes, the risk for MI with or without complications in the near future increases significantly. Poorer outcome is also noted in patients with underlying CHF, presence of decreased left ventricular ejection fraction, hemodynamic instability, recurrent angina despite medical therapy, mitral regurgitation, and sustained ventricular tachycardia. As coronary artery disease is the underlying problem in unstable angina, the most important factor for a better prognosis is preventing further disease progression with aggressive medical management listed above. This includes smoking cessation, good control or hypertension, managing diabetes, controlling weight and establishing a healthy diet, lowering lipids, and living an overall healthy lifestyle. Patients with a history of unstable angina should be followed by a cardiologist for close monitoring throughout their lives.

    Incorrect

    The correct answer is C. This patient is presenting with unstable angina, and should be closely monitored on telemetry to make sure he does not progress into worsening ACS. The patient cannot be sent home, as he needs to undergo serial troponins and is not currently in a stable position to go home even though he is without chest pain currently. While a cardiac stress test is a useful diagnostic in evaluating angina, it should not be done when acute MI has definitively been ruled out as there is a risk of sudden death. Stress tests do not need to be done immediately, and are of more use once patient is stabilized to determine underlying ischemic changes in the heart. Lastly, this patient is not currently showing signs of ischemic occlusion as seen in STEMI or significant NSTEMI presentations, and intervention with catheterization is not indicated. As the patient is monitored in the hospital, if cardiac enzymes increase or ischemic changes develop on the EKG, then a cardiac catheterization can be considered.

    Chest pain is a nonspecific symptom that can have many cardiac or non cardiac causes. Acute coronary syndrome is a common clinical presentation of chest pain and is sub-divided intro three categories: unstable angina, non-ST elevation myocardial infarction and ST-elevation myocardial infarction. Unstable angina is diagnosed when a patient presents with ischemic symptoms suggestive of ACS with no elevated cardiac enzymes, and with or without EKG changes indicating ischemia. It can take up to 12 hours for troponin to elevate and be detectable, so initially many patients are admitted to the hospital in order to appropriately determine if there is an NSTEMI present. There can be ST segment and/or T wave changes in unstable angina, but unlike NSTEMI presentations, they are typically transient and not long-lasting. The incidence of unstable angina is increasing every year, and there are 1 million patients hospitalized annually with the primary diagnosis of unstable angina. Many cases of unstable angina also occur outside of the hospital, with patients not seeking care, so this number is likely much higher.
    There are different classifications for unstable angina. One is any new onset angina that occurs with minimal exercise or at rest, and this comes with a worse prognosis than chronic stable angina. Any form of rest angina is classified as unstable. Early post-MI angina is also classified as unstable and correlates with more complex coronary artery disease which may lead to further episodes of AMI. Additionally, patients are classified as having unstable angina in the absence of significant coronary disease on cardiac catheterization (similar to the presentation of my patient above). Most of these patients will undergo catheterization for suspected NSTEMI and at that time there will not be significant disease noted, which carries a better prognosis. In the case presented above, although NSTEMI was unlikely given EKG findings, she was still brought to the cath lab as it was decided another non-invasive testing would not give suitable information as she had recently undergone evaluation, and it was desired by her attending physician to be certain of presence of CAD/rule out acute MI.
    The underlying etiology and pathophysiology of unstable angina is rather broad. Risk factors that predispose someone to develop this disease include hypertension, diabetes, hyperlipidemia, coronary artery disease, peripheral artery disease, family history of coronary artery disease, and obesity. There factors involved in the pathophysiology of this cardiac presentation include supply-demand mismatch, plaque disruption, thrombosis, and/or vasoconstriction. Supply-demand mismatch occurs when the body is requiring more oxygen than what the heart can provide. This typically occurs when there is an increased demand in cases like fever, exercise, tachycardia, etc. Plaque disruption can occur when there is an accumulation of LDL foam cells in the arteries which are unstable and prone to rupture, resulting in either transient or long-lasting ischemia.
    The signs and symptoms of unstable angina are also exhibited in patient’s with a myocardial infarction. They can include chest pain or pressure, sweating, dyspnea, nausea/vomiting, dizziness and fatigue. Of note, these symptoms are usually occurring at rest when angina becomes unstable or become more severe/frequent than before. Additionally, patients note that their symptoms do not follow their usual pattern and do not respond to nitroglycerin or rest as they previously did. Typically the physical examination is unremarkable but tachycardia, diaphoresis, signs of PAD and/or transient signs of myocardial dysfunction can be seen. At presentation, patients should get a 12-lead EKG and the following blood work completed: cardiac biomarkers, CBC, CMP and lipid panel. Additional diagnostics that are commonly used for assessment of patients with suspected unstable angina include chest x-ray, echocardiogram, CT or MRI angiography, and cardiac stress tests.
    Patients who present with unstable angina are admitted to the hospital as the course is highly variable, and more pronounced acute coronary syndrome (NSTEMI or STEMI) or life threatening arrhythmias could potentially occur. Patients are monitored on telemetry, given supplemental oxygen, and started on either medical management or undergo invasive treatment. Patient’s with a significant history of CAD and other risk factors may undergo invasive intervention with PCI if attending physician believes further evaluation is needed. Medical management can include dual antiplatelet treatment with aspirin and clopidigrel, statins, beta blockers, anticoagulation during hospitalization, angina nitrates, and angiotensin-converting enzyme inhibitors.
    Prognosis for patient’s presenting with unstable angina varies widely depending on concomitant risk factors and underlying medical conditions. When there are EKG changes notes, the risk for MI with or without complications in the near future increases significantly. Poorer outcome is also noted in patients with underlying CHF, presence of decreased left ventricular ejection fraction, hemodynamic instability, recurrent angina despite medical therapy, mitral regurgitation, and sustained ventricular tachycardia. As coronary artery disease is the underlying problem in unstable angina, the most important factor for a better prognosis is preventing further disease progression with aggressive medical management listed above. This includes smoking cessation, good control or hypertension, managing diabetes, controlling weight and establishing a healthy diet, lowering lipids, and living an overall healthy lifestyle. Patients with a history of unstable angina should be followed by a cardiologist for close monitoring throughout their lives.

  216. Question 216 of 220
    216. Question

    A 40-year-old woman presents to the ER with complaints of nausea, vomiting and diarrhea after eating at a seafood buffet several hours earlier. During your history and physical, she reveals she has a congenital condition called Romano Ward syndrome which is a form of long QT syndrome. Which of the following treatments should be avoided with this patient?

    Correct

    The correct answer is Ondansetron (Zofran) 4mg IVP for treatment of nausea and vomiting. This patient has a congenital condition which causes QT prolongation on EKG. Medications that can further prolong QT such as Zofran should be avoided to prevent possibility of development of torsade de pointes. Sending labs would be appropriate because electrolyte abnormalities can predispose patients to torsade. IV fluid hydration with either LR or NS is appropriate to volume resuscitate.
    Discussion:
    Torsade de Pointes, also known simply as torsade, is an uncommon and specific type of polymorphic ventricular tachycardia (VT). It is characterized by rapid, irregular QRS complexes which give the appearance of being twisted around the EKG isoelectric baseline. This appears as an alternating pattern of increasing and decreasing amplitude of the QRS complexes. Torsade is associated with a prolonged QT interval which could be congenital or acquired. It usually terminates spontaneously but frequently recurs and may lead to ventricular fibrillation (VF).
    Etiology
    Prolongation of the QT interval may be congenital such as in Jervell and Lange-Nielson syndrome and the Romano Ward syndrome. These are associated with sudden death due to VF or from torsade that leads to VF. Brugada syndrome and Takotsubo cardiomyopathy can lead to torsade also. Acquired conditions include electrolyte disturbances, most commonly hypokalemia and hypomagnesemia and many antiarrhythmics such as procainamide, quinidine, flecainide, sotalol, amiodarone. Amiodarone routinely prolongs QT but is less commonly associated with the clinical consequences of long QT. There are also many other drugs associated with torsade including antibiotics such as macrolides, abuse of loperamide, cocaine, and citrate accumulation from massive blood transfusions. Other conditions associated with torsade include endocrine disorders, cardiac conditions, intracranial disorders, and nutritional disorders.
    Epidemiology
    The prevalence of torsade is unknown. It is a life-threatening arrhythmia and may present as sudden cardiac death. It is 2-3 times more common in women than men. It can present in any age range with the highest frequency in 35-50-year olds.
    Pathophysiology
    There has been a known association between torsade and prolonged QT interval. Over the last several years, the mechanisms involved at the cellular and ionic levels have become clearer. “The abnormality underlying both acquired and congenital long QT syndromes is in the ionic current flow during repolarization, which affects and prolongs the QT interval.”1
    History and physical exam
    Patients may present with recurrent episodes of palpitations, dizziness and syncope. However, torsade could be fatal with the first episode. Nausea, diaphoresis, shortness of breath, or chest pain may be a presenting symptom, as with any tachyarrhythmia. Physical exam may find bradycardia or PVCs. Those findings can progress to rapid pulse, low blood pressure or loss of consciousness. Ask the patient about a family history of sudden cardiac death or sudden infant death syndrome. Family history of congenital deafness may be suggestive of Jervell and Lange-Nielson syndrome.
    Diagnostics
    Diagnosis is made by obtaining an EKG. If the patient is in sinus rhythm, examine the QT interval for prolongation and presence of pathological U waves, due to abnormal ventricular repolarization. QT of 0.60 seconds or a QTc of 0.45 seconds or longer is significant. Frequently there will be brief paroxysms of 5-20 beats at a rate faster than 200 bpm and twisting of QRS complexes around the isoelectric line. Often torsade is preceded by pauses. If a patient with long QT interval develops ventricular bigeminy, it may be a sign of impending torsade. Check for electrolyte disturbances, especially hypoglycemia, hypokalemia, hypomagnesemia and hypocalcemia.

    Management and treatment
    The acute presentation of torsade can be managed pharmacologically with magnesium sulfate 2 grams IV over 1-2 minutes. Other options include mexiletine or isoproterenol. Temporary transvenous pacing is a possible treatment because QT interval shortens with a faster heart rate so pace the HR at 90-110 bpm if the patient presents with bradycardia. Immediate treatment of electrical cardioversion is indicated if the patient is unresponsive to pharmacological intervention. It is a last resort for treating torsade because it frequently recurs following cardioversion. If the patient develops VF, defibrillation is indicated.
    The long-term treatment for torsade of congenital origin is with beta blockers such as propranolol unless the presentation was bradycardia, permanent pacing, an ICD or a combination. If congenital long QT syndrome, competitive sports are prohibited. Family members should be evaluated with an EKG.
    For acquired origin, correct the inciting factor or predisposing condition that caused the QT prolongation if possible.

    Incorrect

    The correct answer is Ondansetron (Zofran) 4mg IVP for treatment of nausea and vomiting. This patient has a congenital condition which causes QT prolongation on EKG. Medications that can further prolong QT such as Zofran should be avoided to prevent possibility of development of torsade de pointes. Sending labs would be appropriate because electrolyte abnormalities can predispose patients to torsade. IV fluid hydration with either LR or NS is appropriate to volume resuscitate.
    Discussion:
    Torsade de Pointes, also known simply as torsade, is an uncommon and specific type of polymorphic ventricular tachycardia (VT). It is characterized by rapid, irregular QRS complexes which give the appearance of being twisted around the EKG isoelectric baseline. This appears as an alternating pattern of increasing and decreasing amplitude of the QRS complexes. Torsade is associated with a prolonged QT interval which could be congenital or acquired. It usually terminates spontaneously but frequently recurs and may lead to ventricular fibrillation (VF).
    Etiology
    Prolongation of the QT interval may be congenital such as in Jervell and Lange-Nielson syndrome and the Romano Ward syndrome. These are associated with sudden death due to VF or from torsade that leads to VF. Brugada syndrome and Takotsubo cardiomyopathy can lead to torsade also. Acquired conditions include electrolyte disturbances, most commonly hypokalemia and hypomagnesemia and many antiarrhythmics such as procainamide, quinidine, flecainide, sotalol, amiodarone. Amiodarone routinely prolongs QT but is less commonly associated with the clinical consequences of long QT. There are also many other drugs associated with torsade including antibiotics such as macrolides, abuse of loperamide, cocaine, and citrate accumulation from massive blood transfusions. Other conditions associated with torsade include endocrine disorders, cardiac conditions, intracranial disorders, and nutritional disorders.
    Epidemiology
    The prevalence of torsade is unknown. It is a life-threatening arrhythmia and may present as sudden cardiac death. It is 2-3 times more common in women than men. It can present in any age range with the highest frequency in 35-50-year olds.
    Pathophysiology
    There has been a known association between torsade and prolonged QT interval. Over the last several years, the mechanisms involved at the cellular and ionic levels have become clearer. “The abnormality underlying both acquired and congenital long QT syndromes is in the ionic current flow during repolarization, which affects and prolongs the QT interval.”1
    History and physical exam
    Patients may present with recurrent episodes of palpitations, dizziness and syncope. However, torsade could be fatal with the first episode. Nausea, diaphoresis, shortness of breath, or chest pain may be a presenting symptom, as with any tachyarrhythmia. Physical exam may find bradycardia or PVCs. Those findings can progress to rapid pulse, low blood pressure or loss of consciousness. Ask the patient about a family history of sudden cardiac death or sudden infant death syndrome. Family history of congenital deafness may be suggestive of Jervell and Lange-Nielson syndrome.
    Diagnostics
    Diagnosis is made by obtaining an EKG. If the patient is in sinus rhythm, examine the QT interval for prolongation and presence of pathological U waves, due to abnormal ventricular repolarization. QT of 0.60 seconds or a QTc of 0.45 seconds or longer is significant. Frequently there will be brief paroxysms of 5-20 beats at a rate faster than 200 bpm and twisting of QRS complexes around the isoelectric line. Often torsade is preceded by pauses. If a patient with long QT interval develops ventricular bigeminy, it may be a sign of impending torsade. Check for electrolyte disturbances, especially hypoglycemia, hypokalemia, hypomagnesemia and hypocalcemia.

    Management and treatment
    The acute presentation of torsade can be managed pharmacologically with magnesium sulfate 2 grams IV over 1-2 minutes. Other options include mexiletine or isoproterenol. Temporary transvenous pacing is a possible treatment because QT interval shortens with a faster heart rate so pace the HR at 90-110 bpm if the patient presents with bradycardia. Immediate treatment of electrical cardioversion is indicated if the patient is unresponsive to pharmacological intervention. It is a last resort for treating torsade because it frequently recurs following cardioversion. If the patient develops VF, defibrillation is indicated.
    The long-term treatment for torsade of congenital origin is with beta blockers such as propranolol unless the presentation was bradycardia, permanent pacing, an ICD or a combination. If congenital long QT syndrome, competitive sports are prohibited. Family members should be evaluated with an EKG.
    For acquired origin, correct the inciting factor or predisposing condition that caused the QT prolongation if possible.

  217. Question 217 of 220
    217. Question

    A 30 year old, pregnant female presents with complaint of right hand numbness, tingling, and pain that is worse after typing for long periods of time. She states the hand pain is worse at night. The only time she gets relief is resting the hand. The pain and numbness is limited to her first 3 digits of the right hand. Which of the following physical exam findings are likely in this patient?

    Correct

    This clinical case suggests the diagnosis of CTS. The history of the patient includes hand pain, numbness and tingling that is limited to the first 3 digits, which is the median nerve distribution. She states her pain is also worse at night, which is common in CTS.1 She has risk factors for CTS such as pregnancy and repeated flexion and extension of her hand (typing). She is also a female and the female to male ratio is 3:1 for CTS.1 All of the above findings point to the diagnosis of CTS. The question is asking about the possible physical exam findings in CTS. Choice A, finklestein’s test would be positive in De quervain tendinosis, not carpal tunnel. In de quervain’s there would be tendon pain of the thumb side of the wrist when one makes a fist with fingers wrapped around the thumb and flexing the wrist.2 Choice B is incorrect, in CTS, usually advanced cases, thenar atrophy is seen not hypothenar atrophy.1 Choice D, is incorrect because the median nerve distribution if the first 3 digits and radial ½ of the 4th digit.1 Ulnar nerve involvement can be considered if the other ½ of the 4th digit and the 5th digit are involved.
    Choice C is the correct option, positive Tinel’s sign is when percussion over the carpal tunnel causes pain and or numbness in the median-innervated fingers.3
    References for Vignette:
    1. Kothari M. Carpal tunnel syndrome: Etiology and epidemiology. Uptodatecom. 2017. Available at: https://www.uptodate.com/contents/carpal-tunnel-syndrome-etiology-and-epidemiology. Accessed November 18, 2017.

    2. De Quervain’s Tendinosis – Symptoms and Treatment – OrthoInfo – AAOS. Orthoinfoaaosorg. 2017. Available at: https://orthoinfo.aaos.org/en/diseases–conditions/de-quervains-tendinosis. Accessed November 18, 2017.

    3. A Practical Guide to Clinical Medicine. Mededucsdedu. 2017. Available at: https://meded.ucsd.edu/clinicalmed/joints3.htm. Accessed November 18, 2017.

    Incorrect

    This clinical case suggests the diagnosis of CTS. The history of the patient includes hand pain, numbness and tingling that is limited to the first 3 digits, which is the median nerve distribution. She states her pain is also worse at night, which is common in CTS.1 She has risk factors for CTS such as pregnancy and repeated flexion and extension of her hand (typing). She is also a female and the female to male ratio is 3:1 for CTS.1 All of the above findings point to the diagnosis of CTS. The question is asking about the possible physical exam findings in CTS. Choice A, finklestein’s test would be positive in De quervain tendinosis, not carpal tunnel. In de quervain’s there would be tendon pain of the thumb side of the wrist when one makes a fist with fingers wrapped around the thumb and flexing the wrist.2 Choice B is incorrect, in CTS, usually advanced cases, thenar atrophy is seen not hypothenar atrophy.1 Choice D, is incorrect because the median nerve distribution if the first 3 digits and radial ½ of the 4th digit.1 Ulnar nerve involvement can be considered if the other ½ of the 4th digit and the 5th digit are involved.
    Choice C is the correct option, positive Tinel’s sign is when percussion over the carpal tunnel causes pain and or numbness in the median-innervated fingers.3
    References for Vignette:
    1. Kothari M. Carpal tunnel syndrome: Etiology and epidemiology. Uptodatecom. 2017. Available at: https://www.uptodate.com/contents/carpal-tunnel-syndrome-etiology-and-epidemiology. Accessed November 18, 2017.

    2. De Quervain’s Tendinosis – Symptoms and Treatment – OrthoInfo – AAOS. Orthoinfoaaosorg. 2017. Available at: https://orthoinfo.aaos.org/en/diseases–conditions/de-quervains-tendinosis. Accessed November 18, 2017.

    3. A Practical Guide to Clinical Medicine. Mededucsdedu. 2017. Available at: https://meded.ucsd.edu/clinicalmed/joints3.htm. Accessed November 18, 2017.

  218. Question 218 of 220
    218. Question

    An 84-yo male presents to the emergency department for chest pain that woke him from sleep. On arrival you order a stat EKG and troponin. EKG shows sinus brady HR 57 and 1mm ST depression in leads II, III and aVF. The patient is diaphoretic and appears nervous. After completing a history and physical exam, repeat EKG shows 2mm ST elevation in leads II, III and aVF. Which of the following is the most appropriate course of treatment?

    Correct

    The correct answer is C. This patient is having an acute ST elevation myocardial infarction of the right coronary artery, as demonstrated on EKG by ST elevations in leads II, III and aVF showing an inferior MI. The correct course of action is to start this patient on aspirin, Plavix and start a heparin drip. Metoprolol should also be given as tolerated by heart rate, since this patient’s heart rate is already 57 metoprolol would most likely be held. Patients should be sent straight to cath lab for urgent percutaneous intervention (PCI) with balloon angioplasty and stent placement, in this case of the RCA. Fibrinolytics are only indicated for STEMI if time to PCI center is >120 minutes.

    Myocardial infarction risk factors can be grouped into modifiable and non-modifiable risk factors. Modifiable risk factors include tobacco exposure, sedentary lifestyle, obesity, hypertension, dyslipidemia, diabetes mellitus, and metabolic syndrome. Non-modifiable risk factors include family history of premature coronary artery disease (<55yo male or <65yo female) and familial hypercholesterolemia.

    EKG criteria for a STEMI include ST elevation >1mm of in two or more contiguous leads or a new left bundle branch block. Hyperacute T waves, biphasic T waves, Wellen’s T waves that invert and revert, Q waves, or reciprocal ST depressions may also be seen. Locations of the heart represented on an EKG include:
    V1-V2 – Anteroseptal
    V3-V4 – Anteroapical
    V5-V6 – Anterolateral
    I, aVL – Lateral
    II, III, aVF – Inferior
    V1-V3 ST depressions – Posterior

    References
    1. ST-elevation myocardial infarction (STEMI). Dynamed Plus. http://www.dynamed.com/topics/dmp~AN~T115392/ST-elevation-myocardial-infarction-STEMI. Updated 3/7/2018. Accessed 4/23/18.
    2. Papadakis MA, McPhee SJ. Current Medical Diagnosis and Treatment. McGraw Hill Education; 2016.
    3. Montecucco F, Carbone F, Schindler TH. Pathophysiology of ST-segment elevation myocardial infarction: novel mechanisms and treatments. European heart journal. 2016; 27 (1268-1283). https://academic.oup.com/eurheartj/article/37/16/1268/1748706. Accessed 4/23/18.
    4. UpToDate

    Incorrect

    The correct answer is C. This patient is having an acute ST elevation myocardial infarction of the right coronary artery, as demonstrated on EKG by ST elevations in leads II, III and aVF showing an inferior MI. The correct course of action is to start this patient on aspirin, Plavix and start a heparin drip. Metoprolol should also be given as tolerated by heart rate, since this patient’s heart rate is already 57 metoprolol would most likely be held. Patients should be sent straight to cath lab for urgent percutaneous intervention (PCI) with balloon angioplasty and stent placement, in this case of the RCA. Fibrinolytics are only indicated for STEMI if time to PCI center is >120 minutes.

    Myocardial infarction risk factors can be grouped into modifiable and non-modifiable risk factors. Modifiable risk factors include tobacco exposure, sedentary lifestyle, obesity, hypertension, dyslipidemia, diabetes mellitus, and metabolic syndrome. Non-modifiable risk factors include family history of premature coronary artery disease (<55yo male or <65yo female) and familial hypercholesterolemia.

    EKG criteria for a STEMI include ST elevation >1mm of in two or more contiguous leads or a new left bundle branch block. Hyperacute T waves, biphasic T waves, Wellen’s T waves that invert and revert, Q waves, or reciprocal ST depressions may also be seen. Locations of the heart represented on an EKG include:
    V1-V2 – Anteroseptal
    V3-V4 – Anteroapical
    V5-V6 – Anterolateral
    I, aVL – Lateral
    II, III, aVF – Inferior
    V1-V3 ST depressions – Posterior

    References
    1. ST-elevation myocardial infarction (STEMI). Dynamed Plus. http://www.dynamed.com/topics/dmp~AN~T115392/ST-elevation-myocardial-infarction-STEMI. Updated 3/7/2018. Accessed 4/23/18.
    2. Papadakis MA, McPhee SJ. Current Medical Diagnosis and Treatment. McGraw Hill Education; 2016.
    3. Montecucco F, Carbone F, Schindler TH. Pathophysiology of ST-segment elevation myocardial infarction: novel mechanisms and treatments. European heart journal. 2016; 27 (1268-1283). https://academic.oup.com/eurheartj/article/37/16/1268/1748706. Accessed 4/23/18.
    4. UpToDate

  219. Question 219 of 220
    219. Question

    You are a PA working in the ED when a patient comes in with weakness and paresthesia in his left arm that lasted 20 minutes and went away. Mild slurred speech was also noticed but has also resolved. PMH is notable for HTN and hyperlipidemia that are not controlled because the patient is non-compliant on his medications. You conclude that the patient likely had a TIA based on the history. Which test is most likely to be positive?

    Correct

    Correct Answer is B. A transient ischemic attack (TIA) occurs when blood flow to part of the brain stops for a short period of time but there is no evidence of infarction on brain imaging as there is with a stroke. Etiologies of TIA include: atherosclerosis, embolic sources, arterial dissection, arteritis, sympathomimetic drugs (cocaine), masses/lesions, or a hypercoagulopathy.

    There are multiple diagnostics that can be done to rule out a stroke. A CT scan should be the initial test of choice to rule out hemorrhage. An MRI with DWI is more sensitive for ischemic stroke than CT but may not show anything of significance with TIA’s, depending on the duration of symptoms. Since the majority of TIA’s and strokes are caused by atherosclerotic plaque rupture, a carotid duplex is most likely to be positive in this case. A 12 leaf EKG and echocardiogram are also helpful to evaluate for possible cardiac etiology.

    Antiplatelet therapy is the preferred treatment over anticoagulation in patients unless the patient has underlying atrial fibrillation as well as risk factors reduction like controlling blood pressure, cholesterol, diabetes, and smoking cessation. If a patient has more than 80% stenosis on carotid doppler then carotid endarterectomy should be considered.

    References:
    1. What is TIA, Stroke.org. http://www.stroke.org/understand-stroke/what-stroke/what-tia. Published October 27, 2015.
    2. The Internet Stroke Center. The Internet Stroke Center. An independent web resource for information about stroke care and research. http://www.strokecenter.org/professionals/stroke-management/for-pharmacists-counseling/clinical-presentation/.
    3. Transient Ischemic Attack Treatment & Management: Approach Considerations, Patient Disposition, Management of Hypertension. Background, Pathophysiology, Etiology. https://emedicine.medscape.com/article/1910519-treatment#d10. Published May 16, 2018.
    4. Ministroke (TIA): Symptoms, Causes, Treatments, and More. Healthline. https://www.healthline.com/health/stroke/signs-symptoms-tia-mini-stroke#risk-factors.
    5. Antiplatelet Therapy after Ischemic Stroke or TIA | NEJM. New England Journal of Medicine. https://www.nejm.org/doi/full/10.1056/NEJMe1806043.

    Incorrect

    Correct Answer is B. A transient ischemic attack (TIA) occurs when blood flow to part of the brain stops for a short period of time but there is no evidence of infarction on brain imaging as there is with a stroke. Etiologies of TIA include: atherosclerosis, embolic sources, arterial dissection, arteritis, sympathomimetic drugs (cocaine), masses/lesions, or a hypercoagulopathy.

    There are multiple diagnostics that can be done to rule out a stroke. A CT scan should be the initial test of choice to rule out hemorrhage. An MRI with DWI is more sensitive for ischemic stroke than CT but may not show anything of significance with TIA’s, depending on the duration of symptoms. Since the majority of TIA’s and strokes are caused by atherosclerotic plaque rupture, a carotid duplex is most likely to be positive in this case. A 12 leaf EKG and echocardiogram are also helpful to evaluate for possible cardiac etiology.

    Antiplatelet therapy is the preferred treatment over anticoagulation in patients unless the patient has underlying atrial fibrillation as well as risk factors reduction like controlling blood pressure, cholesterol, diabetes, and smoking cessation. If a patient has more than 80% stenosis on carotid doppler then carotid endarterectomy should be considered.

    References:
    1. What is TIA, Stroke.org. http://www.stroke.org/understand-stroke/what-stroke/what-tia. Published October 27, 2015.
    2. The Internet Stroke Center. The Internet Stroke Center. An independent web resource for information about stroke care and research. http://www.strokecenter.org/professionals/stroke-management/for-pharmacists-counseling/clinical-presentation/.
    3. Transient Ischemic Attack Treatment & Management: Approach Considerations, Patient Disposition, Management of Hypertension. Background, Pathophysiology, Etiology. https://emedicine.medscape.com/article/1910519-treatment#d10. Published May 16, 2018.
    4. Ministroke (TIA): Symptoms, Causes, Treatments, and More. Healthline. https://www.healthline.com/health/stroke/signs-symptoms-tia-mini-stroke#risk-factors.
    5. Antiplatelet Therapy after Ischemic Stroke or TIA | NEJM. New England Journal of Medicine. https://www.nejm.org/doi/full/10.1056/NEJMe1806043.

  220. Question 220 of 220
    220. Question

    A 23-year-old female presents to the office because of an uncomfortable smelly, frothy vaginal discharge. She is sexually active and is worried that she got a STD from her partner. On speculum exam there is white discharge and you smell an odor. You do not notice any red petechiae on the cervix. You take a sample of the discharge and send for culture. Instincts tell you the patient has bacterial vaginosis and you prescribe Metronidazole 500mg orally 2x day for 7 days. The following day results come back negative for bacterial vaginosis, chlamydia and gonorrhea, but positive for trichomonas. You call your patient to tell her the results and find she has already started taking her antibiotic. What do you advise your patient?

    Correct

    Correct answer is D. The gold standard treatment for trichomoniasis is metronidazole 2g orally single dose however, since the patient was already given a 7-day course of metronidazole 500mg BID there is no reason to switch the treatment since it will still cover for trichomonas. Butoconazole is for candidiasis. Azithromycin is for chlamydia. Ceftriaxone is for gonorrhea.

    Risk factors for trichomonas include having multiple sex partners, having a new sex partner, unprotected sex, lack of the vaginal lactobacilli, antibiotic use, IUD, frequent use of high dose spermicide, and low socioeconomic status. Most patients are asymptomatic but when symptomatic, complaints include vaginal burning, pruritus, dysuria, frequency, lower abdominal pain, or dyspareunia. Exam findings may include purulent or frothy, malodorous, thin vaginal discharge with possible erythema of the vulva and vaginal mucosa as well as punctate hemorrhages of the cervix (strawberry cervix).

    Bacterial vaginosis (BV) is also frequently asymptomatic but complaints can include vaginal discharge and odor but generally does not cause dysuria, dyspareunia, pruritis, or burning. Exam findings include off-white, thin, and homogeneous discharge the odor with a “fishy smell” but usually no evidence of vaginal inflammation. Diagnosis of BV is made using the Amsel criteria which requires 3 of the following to be present: 1) Homogenous thin white or gray discharge on the vaginal wall, 2) clue cells on wet mount microscopy, 3) vaginal pH  4.5, or 4) a positive whiff test alone or after addition of 10% potassium hydroxide. Having a pH  4.5 and amine odor has a high specificity of 95% for diagnosis of BV. Treatment options for BV are Metronidazole 500mg orally two times a day for seven days, or 2%, 5g clindamycin vaginal cream once daily for seven days or 0.75%, 5g Metronidazole gel for five days. Other options include clindamycin orally or ovules, or tindazole orally.
    References
    1. Papadakis MA, McPhee SJ. Current medical diagnosis & treatment 2017. (Rabow MW, ed.). New York: McGraw Hill Education; 2017. pg 766-768, 1476,1486

    2. Siegal B, Fedorowicz Z. Trichomoniasis; DynaMed Plus. Ipswich (MA): EBSCO Information Services. http://www.dynamed.com/login.aspx?direct=true&site=DynaMed&id=116226. updated Jul 19, 2017. Accessed January 28, 2018.

    3. Siegal B, Shaughnessy A. Bacterial vaginosis (BV); DynaMed Plus Ipswich (MA): EBSCO Information Services. http://www.dynamed.com/login.aspx?direct=true&site=DynaMed&id=115494. updated Sep 26, 2017. Accessed January 28, 2018

    4.Sobel Jack D, Barbieri Robert L, Eckler Kristen. Bacterial vaginosis: Clinical manifestations and diagnosis. UpToDate. June 2017. Available at: http://www.uptodate.com. Accessed January 28, 2018.

    Incorrect

    Correct answer is D. The gold standard treatment for trichomoniasis is metronidazole 2g orally single dose however, since the patient was already given a 7-day course of metronidazole 500mg BID there is no reason to switch the treatment since it will still cover for trichomonas. Butoconazole is for candidiasis. Azithromycin is for chlamydia. Ceftriaxone is for gonorrhea.

    Risk factors for trichomonas include having multiple sex partners, having a new sex partner, unprotected sex, lack of the vaginal lactobacilli, antibiotic use, IUD, frequent use of high dose spermicide, and low socioeconomic status. Most patients are asymptomatic but when symptomatic, complaints include vaginal burning, pruritus, dysuria, frequency, lower abdominal pain, or dyspareunia. Exam findings may include purulent or frothy, malodorous, thin vaginal discharge with possible erythema of the vulva and vaginal mucosa as well as punctate hemorrhages of the cervix (strawberry cervix).

    Bacterial vaginosis (BV) is also frequently asymptomatic but complaints can include vaginal discharge and odor but generally does not cause dysuria, dyspareunia, pruritis, or burning. Exam findings include off-white, thin, and homogeneous discharge the odor with a “fishy smell” but usually no evidence of vaginal inflammation. Diagnosis of BV is made using the Amsel criteria which requires 3 of the following to be present: 1) Homogenous thin white or gray discharge on the vaginal wall, 2) clue cells on wet mount microscopy, 3) vaginal pH  4.5, or 4) a positive whiff test alone or after addition of 10% potassium hydroxide. Having a pH  4.5 and amine odor has a high specificity of 95% for diagnosis of BV. Treatment options for BV are Metronidazole 500mg orally two times a day for seven days, or 2%, 5g clindamycin vaginal cream once daily for seven days or 0.75%, 5g Metronidazole gel for five days. Other options include clindamycin orally or ovules, or tindazole orally.
    References
    1. Papadakis MA, McPhee SJ. Current medical diagnosis & treatment 2017. (Rabow MW, ed.). New York: McGraw Hill Education; 2017. pg 766-768, 1476,1486

    2. Siegal B, Fedorowicz Z. Trichomoniasis; DynaMed Plus. Ipswich (MA): EBSCO Information Services. http://www.dynamed.com/login.aspx?direct=true&site=DynaMed&id=116226. updated Jul 19, 2017. Accessed January 28, 2018.

    3. Siegal B, Shaughnessy A. Bacterial vaginosis (BV); DynaMed Plus Ipswich (MA): EBSCO Information Services. http://www.dynamed.com/login.aspx?direct=true&site=DynaMed&id=115494. updated Sep 26, 2017. Accessed January 28, 2018

    4.Sobel Jack D, Barbieri Robert L, Eckler Kristen. Bacterial vaginosis: Clinical manifestations and diagnosis. UpToDate. June 2017. Available at: http://www.uptodate.com. Accessed January 28, 2018.

Proudly powered by WordPress | Theme: Motif by WordPress.com.